Download as pdf or txt
Download as pdf or txt
You are on page 1of 324

Air Force Airmen Group X & Y Exam

Practice Set-1

bedroom. This was the second stage. The Directions (8–9): Choose the word that
ENGLISH third was yet to come. is opposite in meaning to the following
Directions (1-4): Read the passage 1. The author could reduce his given word.
given below and answer the questions expenses by half by 8. ABANDON
that follow by selecting the correct/ (a) having only one meal a day (a) regain (b) retain
most appropriate options.
(b) renting rooms close to his office (c) remain (d) revive
As I kept strict watch over my way of
living, I could seethat it was necessary (c) travelling in a bus 9. HUMBLE
to economise. I, therefore, decided to (d) cooking his own meals (a) rich (b) powerful
reduce my expenses by half. My accounts 2. ........... constituted a major part of (c) haughty (d) strong
showed numerous items spent on fares. the author’s expenses. Directions (10–12): In the following
Again, my living with a family meant the (a) Meals questions, sentences are given
payment of a regular weekly bill. It also (b) Fares with blanks to be filled in with an
included the courtesy of occasionally (c) Dining out appropriate word(s). Four alternatives
taking members of the family out to (d) Entertainment are suggested for each question.
dinner and likewise attending parties with 3. The author’s choice of his new Choose the correct alternative out of
them. All this involved heavy items for residence enabled him to the four.
conveyances, especially as, if the friend 10. The department head insisted that
(a) be close to his friends
was a lady, custom required that the man he ______ absolute authority to
should pay all the expenses. Also, dining (b) work longer and harder
regulate office work.
out meant extra cost, as no deduction (c) save money on fares
(a) gives (b) is given
could be made from the regular weekly (d) keep things under control
(c) would give (d) would be given
bill for meals not taken. It seemed to 4. What gave the author a fairly
11. Vinod, two of ______ brothers
me that all these items could be saved, strong body?
attend primary school, wishes
as likewise the drain on my purse. So, (a) Yogic exercises to be a school teacher after his
I decided to take rooms on my own (b) Self-discipline graduation from the University.
account, instead of living any longer in a (c) Nourishing food (a) whom (b) them
family and also to remove from place to
(d) Long walks (c) whose (d) which
place according to the work I had to do,
thus gaining experience at the same time. Directions (5–6): In the following 12. The deadline was nearby, so Mrs.
The rooms were so selected, as to enable questions, groups of four words are Sweth had her students ______
me to reach the place of business on foot given. In each group, one word is their essays.
in half an hour and so save fares. Before correctly spelt. Find the correctly (a) complete (b) to complete
this I had always taken some kind of spelt word. (c) completing (d) completed
conveyance whenever I went anywhere, 5. (a) dominent (b) domenant Directions (13–14): In the following
and had to find extra time for walks. The (c) domenent (d) dominant questions, a part of the sentence
new arrangement combined walks and 6. (a) obsarve (b) obsarving is underlined. Below are given
economy, as it meant a saving of fares alternatives to the underlined part
(c) observer (d) observent
and gave me walks of eight or ten miles a (a), (b) and (c) which may improve
Direction (7): Choose the one option the sentence. Choose the correct
day. It was mainly this habit of long walks
which best expresses the meaning of alternative. In case no improvement is
that kept me practically free from illness
the word given below. needed, your answer is (d).
throughout my stay in England and gave
7. CONCEALED 13. His acting is too good.
me a fairly strong body.
Thus I rented a suite of rooms; (a) covered (b) closed (a) best (b) much better
one for a sitting room and another for a (c) sealed (d) hidden (c) very good (d) No improvement
2 Practice Set-1

14. The novel consists a thousand Direction (20): In the following their usual meanings, the correct
pages. question, a sentence has been given in conditions are:
(a) comprises of (b) consists of Active/Passive Voice. Out of the four V
Q2 2 C2
(c) composes (d) No improvement alternatives suggested, select the one Q1 V1 C1
Directions (15–16): In the following which best expresses the same sentence Q3 C3
questions some of the sentences have in Passive/Active Voice.
errors and some are correct. Find out 20. People call him a fool. V3
which part of a sentence has an error. (a) He has been called a fool.
The number of that part is the answer. (b) He is called a fool by the people.
If a sentence is free from errors, then (c) The people have been calling him a
fool. V
your answer is (d) i.e. No error.
(d) We all people have called him a (a) Q1 = Q2 = Q3 and V1 = V2 = V
15. I had (a)/ a test in Mathematics (b) /
fool. (b) V1 = V2 = V3 = V
today morning. (c)/ No error. (d)
16. A girl’must be gracious (a) / if she PHYSICS (c) Q1 = Q2 + Q3 and V = V1 + V2
wishes (b)/ to be a ballerina. (c) / No (d) Q2 = Q3 and V2 = V3
21. An electron and a photon have
error. (d) 26. An alternating e.m.f. given by
same de-Broglie wavelength of
Direction (17): In the following question V = V0sin (ωt) has peak value
10–10 m. Which of the two has
out of the four alternatives choose the 10 V and frequency 50 Hz. The
greater kinetic energy?
one which can be substituted for the (a) Electron instantaneous e.m.f. at t = 1 s
given words/sentence. is: 600
(b) Photon
17. A small room in a big house, hotel, (c) Both have equal K.E. (a) 10 V (b) 5 3V
ship etc. where glasses, dishes, (d) Data insufficient to decide (c) 5 V (d) 1 V
spoons, food etc. are kept. 22. An LC resonant circuit contains 27. Two concentric coils carry
(a) Portico (b) Pantry a 400 pF capacitor and a 100 µH the same current in opposite
(c) Mezzanine (d) Kitchen inductor. It is sent into oscillations
directions. The diameter of the
Direction (18): Choose the appropriate coupled to an antenna. The
inner coils is half that of the
prepositions for the given sentence: wavelength of the radiated
outer coil. If the magnetic field
18. Fire fighters were trying to lift the electromagnetic wave is:
(a) 377 mm (b) 377 cm produced by the outer coil at the
wreackage ........ the bus that had
(c) 377 m (d) 3.77 cm common centre is 1 tesla, the net
overturned .... the main highway.
23. An observer moves towards a field at the centre is:
(a) of, on (b) at, in
plane mirror with a speed of (a) 1 T (b) 2 T
(c) in, across (d) under, in
2 ms–1. The speed of the image (c) 3 T (d) 4 T
Direction (19): In the following
with respect to the observer is: 28. In the network shown below, the
question, sentence has been given in
(a) 1 ms–1 (b) 2 ms–1 equivalent resistance between A
Direct/Indirect Speech. Out of the four
(c) 4 ms –1
(d) 8 ms–1 and B is:
alternatives suggested select the one
24. At what temperature the average R R
which best expresses the same sentence
kinetic energy of a gas molecule
in Indirect/Direct Speech. will be equal to the kinetic energy
19. The boy said, “Bravo ! You have of electron accelerated through R R
done well.” 1 volt: A B
(a) The boy said that he had done (Boltzmann’s constant K = 1.38 × R
well 10–23 J/K,
(a) (b) R
(b) The boy exclaimed that he had Charge on electron = 1.6×10–19 C)
done well (a) 7619°C (b) 7619 K (c) 2R (d) 4R
(c) The boy applauded him, saying (c) 3809.5°C (d) 3809.5 K
29. 2R K2
that he had done well 25. Three capacitors C1, C2 and C3
R K1
(d) The boy said bravo he had done are connected to a battery as
well shown. With symbols having
Practice Set-1 3

Figure shows a perfectly lagged


W2 − W1 (c) electrons move from collector
coaxial cylindrical rod of two (a) to base
5Rnt
different materials. The effective (d) holes move from base to emitter
thermal conductivity of the n(W2 − W1 ) 36. Sound wave in air is expressed by:
(b) –
system is: 5Rt y = 5 ×10–4cos (100t – x/5)m
K1 + 3K 2 Where, t is in seconds, and
(a) K1 + K2 (b) (W2 − W1 )
4 (c) − x is in metres and v denotes
Rnt displacement. The phase velocity
K1 K 2 3K1 + K 2
(c) (d) n(W2 − W1 ) of wave is:
K1 + K 2 4 (d) −
Rt (a) 20 m/s (b) 500 m/s
30. At N.T.P. in a gas, the speed of → → → (c) 2500 m/s (d) 4 m/s
sound is 206 m/s. The increase 33. A vector C = A – B has a 37. What will be the imaginary
in speed for every °C rise in magnitude equal to A + B, the angular speed of the earth which
→ →
temperature is: angle between A and B is: will make the weight of an object
(a) 0.61 m/s (b) 0.38 m/s zero at the equator? The radius
π of the earth is 6400 km (g = 10
(c) 0.82 m/s (d) 1.52 m/s (a) Zero (b)
31. Charges +q, –q, +q and –q are 2 m/s2)
placed at the corners A, B, C and (c)  (d) 2 (a) 1.25 × 10–3 rad/s
D respectively of a square of side 34. An iron bar is heated at one end. (b) 1.25 × 103 rad/s
It is not lagged on its sides. Which (c) 1.25 × 103 m/s
a. The potential energy of the
one of the following graphs shows (d) 1.25 × 10–3 cm/s
1
system is times: the change in rate of heat flow 38. A charged particle of mass
4π ε 0 through any cross-section, with
m and charges q travels on a
A(+q) a B(–q) the distance from the heated end
circular path of radius r that is
of the bar?
perpendicular to a magnetic field
(a) B. The time taken by the particle
Q
(W) to complete one revolution is:
a a t
2πmq 2πq 2 B
distance (m) (a) (b)
B m
a (b) 2πBq 2πm
D(–q) C(+q) Q (c) (d)
(W) m Bq
t
q2
(a) (−4 + 2) 39. The e.m.f. and the internal
a distance (m)
resistance of a source which is
q2 (c) equivalent to two batteries which
(b) (−4 + 2) are connected in parallel having
2a Q
(W)
t e.m.f.’s E1 and E2 and internal
4q 2 resistance r1 and r2 respectively.
(c) distance (m)
a What are E and r?
2 E1 r1 + E 2 r2
(d) − 4 2q (d) (a) E = ; r = r1 + r2
a
Q
(W)
(r1 + r2 )
t
32. A coil having n turns and E1 r2 + E 2 r1
resistance R Ω is connected with distance (m) (b) E = ; r = r1 + r2
(r1 + r2 )
a galvanometer of resistance 4R 35. When n-p-n transistor is used as
Ω. This combination is moved in an amplifier: E1 r2 + E 2 r1 r .r
(c) E = ; r= 1 2
time t seconds from a magnetic (a) electrons move from base to (r1 + r2 ) r1 + r2
field W 1 weber to W 2 weber. collector
The induced current in the E1 r1 + E 2 r2 r r
(b) holes move from callector to (d) E = ; r= 1 2
circuit is: (r1 + r2 ) r1 + r2
base
4 Practice Set-1

40. Assuming the the junction  t  2x − 1


diode is ideal, the current in the (a)   (b) t µc 51. lim is equal to:
arrangement shown in figure is:  µc  x→ 0 ( 1 + x ) 1/ 2 − 1
µt tc (a) log 2 (b) 2log 2
(c) (d)
c µ 1
(c) 2 log 2 (d) 0
(a) 0 mA (b) 2 mA
MATHEMATICS
(c) 10 mA (d) 30 mA dy 3 at
41. The velocity of transverse waves 46. L e t A a n d B b e t w o s e t s 52. Find dx , when x = and y
such that A × B consists of 6 1 + t3
F
in a string is given by v = , elements. If three elements 3 at 2
M =
of A × B are (1, 4), (2, 6) and 1 + t3
where F is tension and M is mass
per unit length. In a measurement
(3, 6), then:
(a) (A × B) = (B × A) (a)
t3 − 4
(b)
(
t 2 − t2 )
the weight applied is 3.0 g. 3 3
(b) (A × B) ≠ (B × A) 1 − 2t 1 − 2t
Length of string L = 1.00 m
(c) A × B = {(1, 4), (1, 6), (2, 4)}
Mass of string = 2.5 g
What is the percentage error in (d) None of the above
(c)
(
t 2 + t3 ) (d) None of these
47. If x satisfies the equation x 2 – 3
the measurement of velocity? 1 + 2t
(a) 3.17% (b) 7.13% 2xcosθ + 1 = 0, then the value of
53. W h a t is the value of
(c) 3.71% (d) 7.31% 1
xn + sin x
42. Five identical resistors, each of x n is equal to: ∫ dx ?
sin 2 x − sin 2 α
value 1100 Ω, are connected to (a) 2 cos n θ
n
(b) 2 cos θ
n n

a 220 V battery as shown. The (c) 2 cos n θ (d) 2 cosn θ (a) sin–1(sec α cos x) + C
reading of the ideal ammeter A is: 48. If a and b are rational and b (b) cos–1(sec α cos x) + C
is not perfect square, then the (c) sin h–1(sec α cos x) + C
quadratic equation with rational
220 V (d) cos h–1(sec α cos x) + C
coefficient whose one root is 3a +
54. The su m of p terms of an
A
b is:
arithmetic progression is
(a) x3 – 6ax + 9a2 – b = 0
(3p2 + 2p), them lth term will be:
1 2 (b) 3ax2 + x – b = 0
(a) A (b) A
5 5 (a) 2l – 1 (b) 6l + 1
(c) x2 + 3ax + b =0
3 4 (c) 6l – 1 (d) 4l – 1
(c) A (d) A (d) bx 2 + x – 3a = 0
5 5 1 1+ 2 1+ 2+ 3
49. An AP consists of n (odd terms) 55. T h e v a l u e o f + + + ....
1 1+ 2 1+ 2+ 3 2! 3! 4!
43. A ball falling from height h and its middle term is m. Then, + + + ....∞ is:
the sum of the AP is: 2! 3! 4!
m rebounces to 9 m height. If
coefficient of restitution e = 0.5, (a) e (b) 2e
(a) 2mn (b) mn
then value of h will be: (c) e/2 (d) None of these
(a) 6 m (b) 9 m (c) mn (d) mn2
x2 − 3 x + 2
(c) 18 m (d) 36 m 50. What is the coefficient of x17 in 56. The function f(x) = 2 is
9 x + 2x − 3
44. If force (F), length (L) and  x3  equal to:
time (T) are presumed to be the expansion of  3 x −  ?
 6 (a) minimum at x = –3, maximum
the fundamental units, then the
x=1
dimensions of mass will be: 189
(a) (b) maximum at x = –3
(a) [FL–1 T2] (b) [FL–1 T–2] 8
(c) increasing in its domain
(c) [FL T ]
–1 –1
(d) [FL2 T2] 567
45. IF c is the velocity of light in
(b) (d) decreasing in its domain
2
vacuum, then the time taken by 57. Equation of circle which passes
the light to travel through a glass 21 through the point (1, –2) and
(c) 16
plate of thickness t and having (3, –4) and both touch the x-axis
refractive index µ is: (d) None of these is:
Practice Set-1 5

(a) x2 + y2 + 6x + 2y + 9 = 0 and these words are written out


y2 y2 y2 y2
(b) x2 + y2 + 10x + 20y + 25 = 0 as in a dictionary, then the rank (a) + =1 (b) + =1
9 25 25 9
(c) x2 + y2 + 6x + 4y + 9 = 0 of the word MODESTY is:
(d) None of the above (a) 5040 (b) 720 x 2 y2
(c) + =1 (d) None of these
58. The centre of a circle is (2, –3) (c) 1681 (d) 2520 3 5
and the circumference is 10π. 70. What is the value of sin18º cos
1 d  −1 1 
Then, the equation of the circle 64. The value of ∫−1 dx  tan x 
dx 36º?
is: is: (a) 4 (b) 2
(a) x2 + y2 + 4x + 6y + 12 = 0 (c) 1 (d) 1/4
(b) x2 + y2 – 4x + 6y + 12 = 0 (a) π (b) π
(c) x2 + y2 – 4x + 6y – 12 = 0 2 4 REASONING AND GENERAL
(d) x2 + y2 – 4x – 6y – 12 = 0 AWARENESS (RAGA)
(c) − π (d) π
59. If n N, then 11n + 2 + 122n + 1 is 2 71. The maximum strength of
divisible by: 2 members of the Rajya Sabha is:
π cos x
(a) 113 65. The value of ∫− π 1 + a x dx , where (a) 250 (b) 350
(b) 123 a > 0, is: (c) 150 (d) 200
(c) 133 (a) π (b) aπ 72. An employment situation where
(d) None of these (c) π (d) 2π the marginal productivity of an
60. A radioactive element disintegrates 2 agricultural labour is zero is
at a rate proportional to the b2 + c 2 − a 2 known as:
quantity of substance Q present at 66. In any ∆ABC, equals (a) Chronic unemployment
4 cot A
any time t. What is the differential to: (b) Seasonal unemployment
equation of the disintegration? (a) ∆ (c) Frictional unemployment
dQ (b) 1 (d) Disguised unemployment
(a) = –Q
dt ∆ 73. Whose famous work was
dQ (c) 2∆ ‘Satyarth Prakash’?

(b) = –kQ, k < 0 (a) Swami Vivekananda
dt (d) None of these
(b) Swami Dayanand Saraswati
dQ 4
(c) = –kQ, k > 0 67. If sin–11 + sin–1 = sin–1x, then (c) Raja Ram Mohan Roy
dt 5 (d) Ishwar Chandra Vidyasagar
dQ what is the value of x? 74. The Andaman Sea is a part of

(d) =Q (a) 3/5 (b) 4/5
dt the:
n n (c) 1 (d) 0 (a) Arabian Sea
61. If ∑ ( xi − 2 ) = 110, ∑ ( xi − 5 ) = 20 , 68. ABC is a triangle in which BC (b) Yellow Sea
i=1 i=1
then what is the mean? = 10 cm, CA = 6 cm and AB = 8 (c) Sea of Japan
11 2 cm. Which one of the following (d) Bay of Bengal
(a) (b) is correct? 75. The universal photosynthetic
2 11
(a) ABC is an acute angled triangle pigment is:
17 17
(c) (d) (a) Chlorophyll-a
3 9 (b) ABC is an obtuse angled triangle
62. A single letter is selected at random (c) ABC is a right angled triangle (b) Chlorophyll-b
from the word PROBABILITY. (c) Chlorophyll-c
(d) None of the above
The probability that it is a vowel (d) Chlorophyll-d
69. A man running a race course
is: 76. The first computers were
notes that sum of its distances
(a) 3/11 programmed using:
from two flag posts from him is
(b) 4/11 (a) Machine language
(c) 2/11 always 10 m and the distance
between the flag posts is 8 m. Find (b) Source code
(d) 0
the equation of the posts traced by (c) Assembly language
63. The letters of the word MODESTY
are written in all possible orders the man. (d) Object code
6 Practice Set-1

77. Which ministry has partnered 83. In a class of 60 students there are (c) √786 m in south-west
with UN’s (United Nations) 33 girls. The average weight of (d) √790 m in north-west
World Food Programme to these girls is 62 kg and average 89. A is taller than B but smaller
release the report on Food & weight of the full class is 66.5 kg. than E. C is the smallest of all. D
Nutrition Security (FNS) stating What is the average weight of the is taller than A but smaller than
that 31.4% will still be stunted boys of the class? E. Who is the tallest of the five ?
by 2022? (a) 72 kg (b) 71 kg (a) B (b) C
(a) Ministry of Health and Family (c) 70 kg (d) 73 kg (c) D (d) E
Welfare(MoHFW) 84. Rohit walks at 17 km/hr and Direction (90) : A series is given with
(b) Ministry of Home Affairs Ruchira cycles at 22 km/hr one term missing. Choose the correct
(MHA) towards each other. What was alternative from the given ones that
(c) Ministry of Statistics and the distance between them when will complete the series.
Programme Implementation they started if they meet after 44 40. flight lieutenant, squadron
(MoSPI) minutes? leader, wing commander
(d) Ministry of Women and Child (a) 42.9 km (b) 35.8 km (a) Flying officer
Development(MoWCD) (c) 21.5 km (d) 28.6 km (b) Air marshal
78. A student multiplied a number 85. The difference between simple (c) Air vice marshal
by 11/13 instead of 13/11. What and compound interests (d) Group captain
is the percentage error in the compounded annually on a Direction (91): Study the following
calculation ? certain sum of money for 2 years information carefully and answer the
(a) 39.67 percent at 18% per annum is ` 81. The questions given below:
(b) 28.4 percent sum is______ A, B, C, D, E, F, G and H are sitting
(c) 14.2 percent (a) ` 2500 (b) ` 5000 around a circle facing the centre. B is
(d) 19.83 percent (c) ` 10000 (d) ` 7500 third to the right of F and third to the
79. The H.C.F. of two numbers is 16. 86. Shobha is the niece of Ashish. left of H. C is fourth to the left of A,
Which one of the following can Ashish’s mother is Priya. Kamla who is not an immediate neighbour
never be their L.C.M.? is Priya’s mother. Kamla’s of F or B. E is not an immediate
(a) 32 (b) 80 husband is Hari. Krishna is the neighbour of B. G is second to the
(c) 46 (d) 60 mother-in-law of Hari. How is right of D.
80. If (x + y) : (x – y) = 5 : 2, find Shobha related to Hari? 91. Who is to the immediate left of
value of (4x + 5y)/ (x – 4y) (a) Daughter B?
(a) 43/5 (b) – 5/43 (b) Great grand daughter (a) D
(c) Grand niece (b) G
(c) – 43/5 (d) 5/43
(d) Great grandson’s daughter (c) D or G
81. P and Q can do a project in 60
87. If SPARK is coded as TQBSL, (d) Data inadequate
and 30 days respectively. In how Direction (92): In the questions
many days can they complete what will be the code for FLAME ?
(a) GMBNF (b) GNBNF below, some relationship have been
90% of the project if they work expressed through symbols as shown
together P? (c) GMCND (d) GMBMF
below.
(a) 9 days (b) 27 days 88. Starting from a point, Suraj
f means less than
(c) 36 days (d) 18 days walked 20 m south, then he
D means not greater than
82. A wholesaler sells a watch to a turned left and walked 18 m,
– means equal to
retailer at a gain of 36% and the he again turned left and walked
+ means not equal to
retailer sells it to a customer at 18 m, then he turned right and
× means not less than
a loss of 25%. If the customer walked 10 m. How far and in
= means greater than
pays ` 2499, what had it cost the which direction is he now from
Study the meaning of these
wholesaler? the starting point?
symbols and pick correct answer
(a) ` 2548 (b) ` 2450 (a) √784 m in north from the answer choices for each
(c) ` 3123 (d) ` 2352 (b) √788 m in south-east questions below:
Practice Set-1 7

92. x φ y + z implies Conclusions: make positive changes in


(a) x – y = z (b) x × y – z (I) Retailers play most important health of an individual.
(c) x D y f z (d) x – y × z role because they interact with (a) Only Conclusion II follows
Direction (93): Find the odd word/ customers on first hand. (b) Conclusion I and II both
letters/number from the given (II) If an organisation has strong follow
alternatives. logistics, then product can (c) Neither I nor II follow
93. (a) 22 (b) 18 easily be available to the cus- (d) Only Conclusion I follows
tomers in such an era of com-
(c) 12 (d) 15 99. Statements:
petition.
94. In the following question, select (I) 
Glaciers are receding and
(a) Only Conclusion II follows the temperature rise of the
the missing number from the (b) Conclusion I and II both world is more than last year
given series. follow this time.
(c) Neither I nor II follow (II)  Pollution level of the Earth
(d) Only Conclusion I follows is rising a lot resulting in
97. Statements: avalanches.
(I) Pre-school culture is prevailing Conclusions:
(a) 1 (b) 16 in all parts of the country now.
(I)  Pollution is resulting into
(c) 14 (d) 20 (II) Parents have less time to spend
hike in temperature of the
with their children so toddlers
95. Which one of the following world.
are admitted to school at a
diagrams best depicts the (II) Pollution is the only reason
very early age.
relationship among Kanpur, for depletion of the nature.
Conclusions:
Varanasi and Meerut? (I) N umber of pre-schools is (a) Only Conclusion II follows
increasing in all parts of (b) Conclusion I and II both
(a) (b) country. follows
(II) C hildren at a very early (c) Neither I nor II follows
age are exposed to the outer (d) Only Conclusion I follows
world resulting in changes in 100. Statements:
(c) (d)
behaviour patterns. (I)  Life is a journey with few
(a) Only Conclusion II follows known people, few partially
Direction (96-100): In the following known and mostly strangers.
(b) Conclusion I and II both
question, two statements are given
follow (II) In a journey, the importance
each followed by two Conclusions of destination is as equal as
(c) Neither I nor II follow
I and II. You have to consider the the pathway of reaching that
(d) Only Conclusion I follows
statements to be true even if they seem 98. Statements: destination.
to be at variance from commonly (I) Many rituals, if incorporated Conclusions:
known facts. You have to decide in daily routine can make (I) Importance of people in life
which of the given Conclusions, if any, positive changes in one's life. is considerable, as they are
follows from the given statements. (II) Bending forward in mosque our journey mates.
96. Statements: or prostrating before God
(II) Life teaches us many lessons
are great exercises as these

(I) 
Major companies are to face various known and
involve backstretch, squat,
nowadays surviving be- unknown difficulties.
bend and rise again.
cause of the strong supply Conclusions: (a) Only Conclusion II follows
chain system. (I) Religion offers great scope for (b) Conclusion I and II both

(II) 
Supply chain comprises exercise. follow
of logistics, retailers and (II) If only these religious rituals (c) Neither I nor II follow
distributers. are practiced daily, they can (d) Only Conclusion I follows
8 Practice Set-1

Answers with Explanation


ENGLISH 1 2E 2eV
E= ⇒v= ⇒ T=
1. (b) 2. (b) 3. (c) 4. (d) 2mv 2 m 3k
5. (d) 6. (c) 2 × 1.6 ×10−19 × 1
h h m h =
7. (d) The word Concealed λ= = = 3 × 1.4 × 10−23
mv m 2E 2mE
(Adjective) means: hidden. 2 × 1.6
h = × 104
8. (b) The word Abandon (Verb) ⇒ 2 mE = ⇒ E 3 × 1.4
λ
means: give up; forsake; to leave. = 7619 K
Look at the sentences : h2
= 25. (c)
C2 and C3 are in parallel
They had to abandon their lands to 2mλ 2 ⇒ V 2 = V3
the invading forces. hc Also, V = V1 + V2 = V1 + V3
For photon E ' =
The word Retain (Verb) means: λ Q 2 + Q3 = Q 1

to keep something; to continue to have E ' hc 2mλ 2 26. (c) V = V0 sin (2πft )
Dividing, = ×
something; preserve. E λ h2  100 π 
⇒ V = 10sin  
Look at the sentence: 2mλc  600 
=
The house retains ‘much of its h 10
⇒ V= = 5V
original charm. E' 2mλc 2
∴ =
9. (c) The word Humble (Adjective) E h 27. (a)

means: modest; showing you do not think a
2 × 9.1 × 10 × 10−10 × 3 × 103
−31

that you are as important as other people. =


6.6 × 10−34
2a
The word Haughty (Adjective)
∴ E' > E
means ; behaving in an unfriendly way
i.e., the photon possesses more
towards other people because you think K.E. than the electron. Bouter = 1T ⊗
you are better than them; arrogant. 1
22. (c) Frequency, v = Binner = 2T 
Look at the sentences: 2π LC Bnet = 2 – 1 = 1T

Be humble enough to learn from your 1 28. (a) Net resistance of the balanced
⇒v= Wheatstone Bridge is R which is
mistakes. −6
2 × 3.14 × 100 ×10 × 400 ×10 −12
connected in parallel with another
He replied with haughty disdain. resistance of same value.
1
10. (d) 11. (c) 12. (a) ⇒v=
2 × 3.14 × 2 ×10−7 R
13. (c) 14. (b) Hence, R AB =
2
15. (b) Here, a preposition related error c 29. (b) Let l be the length of the rod
⇒λ=
occurs. Hence, a test on Mathematics v having temperature difference θ. If K is
.......... should be used. the effective conductivity then:
= 3 × 10 × 2 × 3.14 × 2 × 10
8 –7

16. (d) 17. (b) 18. (a) = 377 m Q = Q1 + Q2


19. (c) The boy applauded him, saying
23. (c) If the mirror approaches K. π (2R) 2 . θ K . πR 2 . θ K 2 .π {2R) 2 − R 2 }
that he had done well. = 1 +
the object or the object approaches the l l l
20. (b) He is called a fool by the
people. K. π v(2R)
stationary mirror with speed then2 .image
θ K1 . πR 2 . θ K 2 .π {2R) 2 − R 2 }θ
approaches object with speedl 2v. = +
l l
PHYSICS 3
24. (b) kT = eV K + 3K 2
21. (b) For electron 2 ∴ K= 1
4
Practice Set-1 9

30. (b) 273 − t 35. (d) When forward bias is applied The time taken by the particle to
vt = v0 on n-p-n transistor, then it works as an complete one revolution:
273
amplifier. In biased n-p-n transistor,
 1 1  electron move from emitter to base and 2πr 2πmv 2πm
= v0 1 + . t T = = =
 2 273  holes move from base to emitter. v VqB Bq
206 36. (b) Given 39. (c) I = I1 + I2 ...(i)
= v0 + t
2 × 273  x
y = 5 × 10−4 cos 100t −  rr
∴ Per degree increase  5 
r= 12  ...(ii)
r1 + r2
206 π x
= −4
⇒ y = 5 × 10 sin  + 100t − 
546 2 5 E1 E 2
 
Further, I= +
= 0.38 m/s r1 r2

 ωx 
31. (a) y = a sin  ωt − 
 v  E E1 E 2

⇒ = +
 −q2   q2  r r1 r2
U = 4 + 2   we get ω = 100

 4π ε 0 a   4π ε 0 2a 
ω 1 E1r2 + E 2 r1
and =
⇒ E=
 1  q2 v 5 r1 + r2
U= (−4 + 2)
 4π ε 0  a ⇒ v=5× ω 40. (a) Junction diode is reverse
32. (b) The rate of change of flux or = 5 × 100 biased, so current is zero.
e.m.f. induced in the coil is: = 500 m/s 41. (c)
2
dφ 37. (a) At the equator g ' = g − ω R F FL  M
e=−n V= = ∵ m = 
dt g ' = 0, ω = g/R M M L
for
∴ Induced current ⇒ V = F1/2 M–1/2 L1/2
e n dφ 10
= ∆V 1 ∆F 1 ∆M 1 ∆L
i= =− ... (i) 6400 × 103 ⇒ = + +
R' R ' dt V 2 F 2 M 2 L
Here, R' = R + 4R = 5R, dφ = W2 – 1
=  ∆M
⇒∆V∆V 1= 1∆M  0 1+∆1M∆M1 +∆L
1 ∆L
W1, dt = 1. 800 =  M + 2 M
0
+
V V 2 2M 0 
0 2 M 2 L2 L
(Here, W1 and W2 are flux = 1.25 ×10−3 rad/s
= FM=0 g)M 0 g)
(∵ F(∵
associated with one turn) 38. (d) Magnetic force F = qvB ... (i)
Putting the given values in eqn. (i) ∆V 1  0.1  1  0.1  1  0.001 
⇒ =  +   +  
we get V 2  3  2  2.5  2  1 
v
n (W2 − W1 )
∴ i=− ∆V 1  0.1  1  0.1  1  0.001 
5R t =  +   +  
V 2  3  2  2.5  2  1 
33. (c)
→ → → ∆V
C =A −B ⇒ = 0.0371

V
C =A+B
∆V
⇒ A 2 + B2 + 2AB = A 2 + B2 − 2ABcos θ ⇒ × 100 = 3.71%
V
⇒ cos θ = − 1 2
Centripetal force F = mv ...(ii)
⇒ θ = 180° r 42. (c) Req = 220 Ω
⇒ θ =π
From equations (i) and (ii), ⇒ Itotal = 1A
34. (d) Since Q/t is proportional to Reading of ammeter is the current
∆θ / ∆x (temperature gradient which mv 2
= qvB which flows through 3 branches.
decreases with length for unlagged bar, r
mv 3
therefore, Q/t will also decrease with r= Hence, reading = A
qB 5
increase in distance).
10 Practice Set-1

43. (d) From the relation, So, A = {1, 2, 3} and B = {4, 6}


2x − 1
Hence, A × B = {1, 2, 3} × {4, 6} 51. (b) lim
e=
v
=
h2 x →0 (1 + x )1/2 − 1
= {(1, 4), (1, 6), (2, 4), (2, 6),
u h1
9
(3, 4), (3, 6)}
and B × A = {4, 6}×{1, 2, 3} = lim
(2 − 1){(1 + x)
x 1/2
+1 }
⇒ (0.5)2 =
h
(putting h1 = h)
= {(4, 1), (4, 2), (4, 3), (6, 1),
x →0
{(1 + x) 1/2
−1 }{(1 + x) 1/2
+1 }
∴ h = 9 (6, 2), (6, 3)} 2x − 1
0.25 = lim {(1 + x )1/2 + 1}
47. (c) Given, x2 – 2x cosθ + 1 = 0 x →0 1 + x − 1
= 36 m
Alternately, velocity of striking the ∴ x=
2 cos θ ± 4 cos2 θ − 4
2 (1)
= lim
2x + 1
x →0 x x →0
1/2
lim (1 + x ) + 1 { }
ground = (log2) 2 = 2log2
= cosθ + i sinθ
= 2gh
⇒ x = eiθ, e–iθ ⇒ xn = einθ 2x − 1 0 2 x log 2
[because lim   = lim ]
recoil velocity = e 2 gh
⇒ x–n = e–inθ x →0 x  0  x →0 1
= log 2]
∴ xn + x– n = einθ + e–inθ = 2cos n θ
t
( )
2
Now O = e 2 gh −2× g ×9 48. (a) If one root of any quadratic 52. (b) x = 3a ×
1 − t3
equation is in the form 3a + b then
e2 × 2gh = 2g × 9

other root of this equation should be 3a dx (1 + t ) dtd t − t dtd (1 + t )
3 3

9 dt = 3a.
– b.
(1 + t )
2
⇒ h= 2
3
e ∴ Required equation is
9
x2 – (Sum of roots) . x +
(product of roots) = 0 = 3a.
(1 + t ).1 − t . 3t = 3a 1 − 2t
3 2 3
=
(1 + t ) (1 + t )
2 2
(0 − 5) 2 3 3
   ⇒ x2 – (3a + b ) + (3a – b ). x
= 36 m + {(3a + b ) (3a – b )} = 0 t2
Again, y = 3a.
Force 1 + t3
44. (a) Mass = x2 – 6ax + 9a2 – b= 0
Acceleration
49. (c) Middle term =
Tn +1 dy
∴ dt = 3a.
1 + t3
dt (
d 2 2 d
t −t
dt
)
1+ t 3 ( )
[F]
( )
2 2
= 1 + t3
[LT -2 ]  n +1 

= [FL−1T 2 ]
∴ a+ 
 2
− 1 d = m (Given)

= 3a.
(1 + t ) 2t − t .3t
3 2 2
= 3a.
(
t 2 − t3 )
(1 + t ) (1 + t )
2 2
45. (c) 2a + (n – 1)d = 2m  ...(i) 3 3

Velocity of light in vaccum c h


Now Sn = [2a + (n – 1)d] dy dy / dt
µ= = 2 =
Velocity of light in glass plate c ' = nm [from eq. (i)] dx dx / dt

( ) × (1 + t ) = t (2 − t )
r 2
c  x3  3at 2 − t 3 3 3
⇒ c' = 50. (a) Tr + 1 = Cr (3x)
9 9–r − 
µ =
(1 + t ) 3a (1 − 2t ) 1 − 2t
 6 2 3 3
3
Time taken r
 1 sin x
=
Distance
=
t
=
µt
= 9Cr x9 + 2r(3)2 – r  − 6  53. (a) Let I = ∫ dx
Velocity c/µ c sin x − sin 2 α
2

For coefficient of x17, 9 + 2r sin x


MATHEMATICS = 17 ⇒ r = 4
= ∫ dx
1 − cos2 x − sin 2 α
46. (b) Since (1, 4), (2, 6) and (3, 6) 4 sin x
 1
are elements of A×B, it following that ∴ T5 = 9C4 (3)9 – 4  −  = ∫ dx
6 cos α − cos2 x
2
1, 2, 3 are elements of A and 4, 6 are
1 189 Put cos x = t
elements of B. It is given that A×B has 6 = 126 × 35 × 4 =
6 8 ⇒ –sinx dx = dt
elements.
Practice Set-1 11

1 we get the required equation The number of words start with


I = −∫ dt x2 + y2 – 6x + 4y + 9 = 0 MD = 5! = 120
cos2 α − t 2
58. (c) Circumference = 2πr ⇒ 10π The number of words start with ME
 t  = 2πr = 5! = 120
= cos–1  +C
 cos α  ⇒ r = 5 and Now, the first word start with MO
= cos–1 (secα cosx) + C centre = (2, –3) is MODESTY.
54. (c) Sum of p terms = 3p2 + 2p By standard equation of circle, Hence, rank of MODESTRY= 720
First term a = 3.12 + 2.1 = 5 (x – 2)2 + (y + 3)2 = 52
+ 720 + 120 + 120 = 1681
Sum upto second term ⇒ x2 + 4 – 4x + y2 + 9 + 6y = 25
= 3.22 + 2.2 = 16 ⇒ x2 + y2 – 4x + 6y – 12 = 0 d  −1 1 
∴ Second term = 16 – 5 = 11 64. (c)  tan 
59. (c) Let P(n) = 11n + 2 + 122n + 1 dx  x
∴ d = 11 – 5 = 6 On putting n = 1
lth term = a + (l – 1).d then P(1) = 111+2 + 122×1+1 = 113 + 123 
1 1 −1
=  1− 2  =
= 5 + (l – 1) × 6
= 1331 + 1728 = 3059 1  x  1 + x2
1+ 2
= 6l – 1 which is divisible by 133 x
55. (c) Let S 60. (c) Since, a radioactive element 1 d  −1 1 
= +
1 1+ 2 1+ 2 + 3
+ + ....∞ disintegrates at a rate proportional to the ∴ Ι= ∫−1 dx 
tan
x 
dx
2! 3! 4! quantity of substance Q present at any
1 + 2 + ... + n time t 1 dx
Here, Tn = ( n + 1)! = −∫
dQ dQ −1 1 + x2
∴ ∝ −Q ⇒ – kQ, k > 0
n dt dt
2
( n + 1) 1 Which is required differential =  − tan −1 x 
1
=−
π
= =   −1 2
( n + 1)! 2 ( − 1)!
n equation.
∞ ∞ n 65. (c) By using property,
1 1

1
S = ∑ Tn = ∑ = e 61. (c) ∑ ( xi − 2) = 110
( n − 1) 2 b b
∫a (a + b − x ) dx
n =1 2 n =1

i =1
x1 + x2 +...xn – 2n = 110
∫ f ( x ) dx =

a
( x − 2)( x − 1) ( x − 2)
56. (c) y= = ⇒ x1 + x2 + ...xn = 2n + 110 ....(i)
( x + 3)( x − 1) ( x + 3) , I =
π cos2 x
∫−π 1 + a x dx
n
x ≠ 1, x ≠ –3 and ∑ ( xi − 5) = 20
x+3−5 5 i =1 [ a + b = 0] ...(i)
⇒ y= =1−
x+3 x+3 x1 + x2 + ... + xn – 5n = 20 2
Ι =
π cos (0 − x ) dx

dy
=
5
= positive ⇒ x1 + x2 + ... + xn ∫−π 1 + ax
dx ( x + 3)2 = 5n + 20 ....(ii)
Always for all values of x in its From Eqs. (i) and (ii), we get π a x cos2 x
domain. 5n + 20 = 2n + 110 ∴ I= ∫ −π x
dx
1+ a
...(ii)

So, y = f(x) is an increasing function ⇒ 3n = 90


On adding Eqs. (i) and (ii), we get
in its domain. ⇒ n = 30
57. (d) Since, the circle touches x1 + x2 + .... + xn π 1 + ax 2
Now, mean = 2I = ∫−π 1 + a x cos x dx
X-axis. n
∴ (x – h)2 + (y – k)2 = k2....(i) 5 × 30 + 20 170 17
= = = π
Also, it passes through given points 30 30 3 = 2∫ cos2 x dx
0
(1 – h)2 + (–2 –k)2 = k2....(ii) 62. (b) Total ways are 11.
There are three vowels and one is π /2 1 π
and (3 – h)2 + (–4 –k)2= k2 ....(iii) ∴ I = 2∫ cos2 x dx = 2. .
repeated, so favourable cases = 4
π 2 2
On subtracting Eq. (iii) from Eq.
(ii), we get 4 π
∴ Required probability = =
h=k+5 11 2
On solving these equations, we get 63. (c) The number of words start wit 66. (a) Taking LHS
k = –10, –2 and h = –5, 3 D = 6! = 720
The number of words start with E b2 + c 2 − a 2
By putting the value of (h, k) i.e., =
= 6! = 720 4 cot A
= (–5, –10) or (3, –2) in Eq. (i)
12 Practice Set-1

are (c, 0) and (–c, 0) in partnership with Ministry of Statistics


b2 + c 2 − a 2
= sin A Therefore, distance between S and and Programme Implementation (MoSPI)
4 cos A
S' is 2c = 8 ⇒ c = 4 stated that 31.4% of Indian children
b2 + c2 − a 2  c2 = a2 – b2 under five years will still be stunted by
= × 2bc × sin A
(
4 b2 + c2 − a 2 ) ∴ 16 = 25 – b2
⇒ b2 = 25 – 16
2022.
1 ⇒ b2 = 9 ⇒ b = 3 78. (b) Let the number be x.
= × bc × sinA = ∆
2 Hence equation of path (ellipse) is 13 11
x– x
4 11 13 × 100%
67. (a) Given, sin–11+ sin–1 = sin–1x % error =
5 x 2 y2 x 2 y2 13
x

2
+ 2 =1 ⇒ + =1
11
a b 25 9
 −1 −1 −1  2 2
∵ sin x + sin y = sin x 1 − y + y 1 − x 48
   
5 −1 x
70. (d)  sin18º = and cos36º 11 × 13
  4    = × 100%
∵ sin −1 x + sin −1 y = sin −1  x 1 − y 2 + y 1 − x 2  
   =
1
4
( 5 −1) 13
11
x
 5 − 1 1
⇒ sin –1
 2 
1. 1 −  4  + 4 1 − (1)2  sin18º cos36º =  4  4
( 5 +1)      =
48 x × 11
× 100%
  5 5  11 × 13 × 13
 
(5 − 1) = 4 1 4800
= sin–1x =    = % = 28.4%
= 16 16 4 169
 16 4  79. (d) H.C.F. of two numbers
⇒ sin–1 1. 1 − + 1 − 1  REASONING AND GENERAL
 25 5  divides their L.C.M. exactly.
AWARENESS (RAGA)
5
= sin–1x 71. (a) Article 80 of the Constitution 80. (c) =
2
 9  lays down the maximum strength of
⇒ sin–1  + 0  = sin–1x ⇒ 2x + 2y = 5x — 5y
 25  Rajya Sabha as 250, out of which 12
3x = 7y
members are nominated by the President
 3 and 238 are representatives of the states 7
⇒ sin–1   = sin–1x =
 5 3
and of the two Union Territories.
x
3 72. (d) T h e t e r m " d i s g u i s e d 4. + 5
∴ x= 4x + 5 y y
5 unemployment" commonly refers to a =
68. (c) a = 10, b = 6, c = 8 situation of employment with surplus x − 4y x
−4
manpower, in which some workers have y
b2 + c2 − a 2 36 + 64 − 100
cos A = = 2×6×8 zero marginal productivity so that their 7
2bc 4× +5
removal will not affect the volume of 3
⇒ cosA = 0 =
total output. 7
∴ A = 90º −4
73. (b) Satyarth Prakash is a 1875 3
So, ∆ABC is a right angled triangle.
69. (b) Clearly, path traced by the book written originally in Hindi by 28 + 15
man will be ellipse. Maharishi Dayanand Sarawati, a 3
renowned religious and social reformer = 7 − 12 = =
Given, SP + S'P = 10
i.e. 2a = 10 ⇒ a = 5 and the founder of Arya Samaj. 3
Since, the coordinates of S and S' 74. (d) The Andamans are made up
81. (d) P and Q's one day work
of 204 islands of varying size and are
located in the eastern Indian ocean as
=
part of the Bay of Bengal.
75. (a)  76. (a)
77. (c) Ministry of Statistics and = = =
Programme Implementation Explanation:
P and Q's whole work will be done
The report on Food & Nutrition Security
(FNS) Analysis of India developed by UN = in 20 days
(United Nations) World Food Programme
Practice Set-1 13

81 = 90. (a) 'Flying officer' completes the


P and Q's 90% of work = series.
= 18days 91. (a) By analysing the circular
82. (b) Let the cost price of P= arrangement, it is clear that immediate
wholesaler be x left of B  D.
= ` 2500
92. (c) According to the meaning of
S.P. of wholesaler = = 86. (b) Shobha  niece of Ashish the given symbols,
So, Ashish  uncle of Shobha. xfy+zx<y≠z
C.P. of retailer =
Priya  mother of Ashish. x l y f z  x >| y < z.
So, Priya  grandmother of Shobha.
S.P. of retailer = × Therefore, x f y + z  x D y f z.
Hari  father of Priya.
93. (a) Except 22, all other numbers
Therefore, ‘Shobha’ is the great are multiples of 3.
= grand daughter of ‘Hari’.
3 × 16 = 18
87. (a) As,
3 × 4 = 12
2499 = S P A R K T Q B S L
+1 3 × 5 = 15
+1
+1 But, 3 × 7 + 1 = 22
x = +1
+1 94. (c) (17 – 8) = 9 → (9)2 → 81
Similarly,
= = 2450 (16 – 11) = 5 → (5)2 → 25
F L A M E G M B N F
+1
24 – ? = 100
Hence, C.P. of wholesaler is ` 2450. +1 24 – ? = (10)2
83. (a) Total weight of 33 girls +1
+1 24 – ? = 10
= 33 × 62kg = 2046 kg +1
88. (b) ? = 24 – 10 = 14
Total weight of full class
start
A Hence, the missing term is 14.
= 60 × 66.5 kg
= 3990 kg 95. (a) Kanpur, Varanasi and
20 m Meerut, all are cities of Uttar Pradesh
weight of 27 boys but, they are entirely different from
= 3990 — 2046 = 1944 one another. This can be expressed by
18 m
option (a).
Average weight = = 72 kg 18 m

84. (d) Total distance covered by B


10 m end
them N

= 96. (a) According to the statements,


W E
only Conclusion II follows.
= 97. (d) According to the statements
S
of the question, only Conclusion I
= = = ∴ AB =
follows.
98. (b) According to the statements,
= Conclusion I and Conclusion II follows.
= = 28.6 km
= = m 99. (c) According to the statements,
85. (a) Difference of simple & and the direction is south-east. neither Conclusion I nor Conclusion II
compound interest 89. (d) According to sitting follows.
arrangement,
100. (d) According to the statements,
ID = E, A, B, D, C
E>A>B>D>C only Conclusion I follows.
Finally, E is the tallest of the five. qqq
14 Practice Set-2

Air Force Airmen Group X & Y Exam


Practice Set-2

(c) begining Directions (12–13): In the following


ENGLISH
(d) chamaleon questions, some of the sentences have
Directions (1-3): Read the following error and some have none. Find out
5. (a) irrepairable
passage and answer the questions that which part of a sentence has an error.
follow. (b) irreparable
The number of that part is your
The Kingfisher is a bird that gets its name (c) irreparrable
answer. If there is No error, the answer
because it truly is a king among the fish (d) irepairable is (d).
catching birds. In catching fish it sits on 6. (a) Separate (b) Ocur 12. I Promise (a)/ I call you (b)/ as soon
a limb of a tree that hangs over a stream, (c) Recieve (d) Accomodate as I get home. (c)/ No error (d).
looking into the water beneath. Then, as Directions (7–8): In following 13. I am afraid (a)/ I did a mistake (b)/
it sights a fish, it drops like a stone into questions, choose the word opposite in the calculation. (c)/ No error (d).
the water. Seconds later it pops up with in meaning to the given word. Directions (14–16): In the following
the fish in its bill. In addition to fish, 7. VAGUE questions, sentences are given
it eats worms, crabs and shellfish. It is
(a) accurate (b) sharp with blanks to be filled in with an
found mostly near the freshwater streams
(c) precise (d) actual appropriate word(s). Four alternatives
of Europe and North America. The
8. HARMONY are suggested for each question.
Kingfisher’s cry is shrill and piercing.
(a) disagreement Choose the correct alternative out of
The North American Kingfisher, called
(b) melody the four.
the belted Kingfisher, because of a belt
(c) confusion 14. He works hard by day and sleeps
of blue feathers across its white breast, is soundly at night with ______ clear
larger than that of the European variety. (d) discord
conscience.
The European Kingfisher is a beautiful 9. In the following question, out
of four alternatives, choose one (a) a (b) an
bird. Its feathers are bright blue, orange,
which can be substituted for the (c) the (d) None of these
deep red, pink and green.
1. Find out the word from the text given words/sentence. 15. The lights went out while we
which means ‘branch’. One who makes an official ______ the examination.
examination of accounts. (a) were taken (b) are taken
(a) bill (b) limb
(a) auditor (b) accountant (c) were taking (d) are taking
(c) shrill (d) pops up
(c) clerk (d) official 16. The exact reasons for their
2. The opposite of ‘polluted’ is–
10. Choose the appropriate preposition disagreement ______ yet to be made
(a) fresh (b) belted public.
(c) piercing (d) bright for the given sentence:
The dog may bite you if you try to (a) are (b) is
3. Which of the word is correctly (c) had (d) was
take ..... the bone it is chewing.
spelt? 17. Choose the one option which best
(a) on (b) away
(a) Enthusiastic(b) Enthusastjc (c) off (d) over expresses the meaning of the given
(c) Enthuesistic(d) Enthesistic 11. Choose the option that best fills in word.
Directions (4–6): In the following the sentence: PAUCITY
questions, groups of four words are Mumbai is ..... but Kolkata is ....... (a) sincerity (b) shortfall
given. In each group one word is in area. (c) publicity (d) downfall
correctly spelt. Find the correctly (a) big, bigger 18. In the following questions, the first
spelt word. (b) big, biggest and the last sentences/parts of the
4. (a) misdemenour (c) bigger, biggest passage/sentence are numbered
(b) manoeuvre (d) bigger, more bigger 1 and 6. The rest of the passage/
Practice Set-2 15

sentence is split into four parts 26. Two lamps P and Q are connected
PHYSICS
and named P, Q, R and S. These in parallel in an electric circuit.
four parts are not given in their 21. A machine gun fires 10 bullets/s, Lamp P glows brighter than
proper order. Read the sentence/ each of mass 10 g, the speed to lamp Q. If RP and RQ are their
passage and find out which of the each bullet is 20 cms–1, then force respective resistance then:
four combinations is correct. Then of recoil is: (a) RP > RQ (b) RP < RQ
find the correct answer. (a) 200 dyne (b) 2000 dyne (c) RP = RQ (d) None of these
1. The decade of the seventies. (c) 20 dyne (d) None of these 27. The displacement of a particle
22. An engine pumps water through is represented by the equation
P. and is a proper field of study
a hose pipe. Water passes through π 
Q. of the idea that y = 3cos  – 2ω t  . The motion
4 
the pipe and leaves it with a
R. women’s literature exists
velocity of 2 ms–1. The mass per of the particle is:
S. saw a growing acceptance. (a) Simple harmonic with period
unit length of water in the pipe is
6. inmost parts of the world. 2π
100 kg m–1. What is the power of
(a) RSPQ (b) QPSR the engine? ω
(c) PQRS (d) SQRP (a) 400 W (b) 2000 W π
19. In the following question a sentence (b) Simple harmonic with period
(c) 100 W (d) 800 W ω
has been given in Active Voice/ (c) Periodic but not simple harmonic
23. A body of mass m was slowly
Passive Voice. Out of the four (d) Non-periodic
pulled up the hill by a force
alternatives suggested, select the
F which at each point was 28. A loop made of straight edges
one which best expresses the same directed along the tangent of has six corners at A(0, 0, 0), B(L,
sentence in Passive/Active Voice. the trajectory. All surfaces 0, C), C(L,L,0), D(0, L, 0), E(0,
Their dog bit our neighbour are smooth. Find the work L, L) and F(0, 0, L). A magnetic
yesterday. →
performed by this force?
field B = Bσ (iˆ + kˆ ) T is present
(a) Our neighbour bit their dog in the region. The flux passing
yesterday, through the loop ABCDEFA (in
(b) Our neighbour bitten by their dog that order) is:
yesterday (a) B0L2Wb (b) 2B0L2Wb
(c) Our neighbour was bitten by their (c) 29 Β0 L2 Wb (d) 4B0L2Wb
dog yesterday h
F 29. The wave of wavelength 5900 Å
(d) Our neighbour were bit by their emitted by any atom or molecule
m
dog yesterday must have some finite total
20. In the following question, a length which is known as the
l
sentence has been given in Direct/ coherence length. For sodium
Indirect Speech. Out of the four (a) mgl (b) –mgl light, this length is 2.4 cm. The
alternatives suggested select the (c) mgh (d) zero number of oscillation in this
one which best expresses the same 24. The limits of Poisson’s ratio (σ) length will be:
sentence in Indirect/Direct Speech. are: (a) 4.068×108 (b) 4.068×104
(a) from 1 to 1 (c) 4.068×10 6
(d) 4.068×105
“What a stupid fellow you are !”
(b) from 0 to –1 30. W h i c h o f t h e f o l l o w i n g i s
she remarked.
1 most suitable as the core of
(a) She exclaimed that what a stupid (c) from –1 to transformers?
2
fellow he was (a) Steel (b) Alinco
(b) She exclaimed that he was a very 1 1
(d) from – to (c) Soft iron (d) None of these
stupid fellow 2 2 31. When the volume of a gas is
(c) She exclaimed in disgust how he 25. How much work can be done by reduced at constant temperature,
could be so stupid 250 cal of heat? the pressure exerted by the gas
(d) She asked h im what a stupid (a) 1050 J (b) 1045 erg on the walls of the container
fellow he was (c) 1045 W (d) Zero increases because:
16 Practice Set-2

(a) each molecule hits the walls 37. The binding energy of a H-atom, (a) TV
with greater speed considering an electron moving (b) Radio
(b) each molecule loses more around a fixed nuclei (proton), (c) Telephone
energy when it strikes the wall me 4 (d) Walky-talky (wireless used in
is B = (m = electron
(c) e a c h m o l e c u l e l o s e s 8 n 2 ε 02 h2 the Army)
momentum when it strikes the mass). If one decides to work in 42. In a circuit L, C and R are
wall a frame of reference where the connected in series with an
(d) the number of molecules electron is at rest, the proton alternating voltage source of
striking the wall per unit time would be moving around it. By frequency f. The current leads
increases similar arguments, the binding the voltage by 45°. The value of
32. If the distance between the earth Me 4 C is:
and sun were half its present energy would be B = − 2 2 2 1
8n 0 h (a)
value, the number of days in a 2π (2π f L + R)
(M = proton mass).
year would have been:
This last expression is not 1
(a) 64.5 (b) 129 (b)
correct because: π f (2π f L + R)
(c) 182.5 (d) 730
(a) n would not be integral
33. Under the influence of a uniform
(b) Bohr-quantisation applies only 1
magnetic field a charged particle to electron (c) 2π f (2π f L − R)
is moving in a circle of radius R
(c) the frame in which the electron
with constant speed V: is at rest is not inertial 1
The time period of the motion: (d)
(d) the motion of the proton would π f (2π f L − R)
(a) depends on V are not on R not be in circular orbits, even
43. In the following circuit, the
(b) depends on both R and V approximately
output Y for all possible inputs A
(c) is independent of both R and V 38. If the magnetic field is parallel to and B is expressed by the truth
(d) depends on R and not on V a surface, then the magnetic flux table:
through the surface is:
34. The resistivity of an n-type A
(a) Zero
extrinsic semiconductor is 0.25 B
Ω-m. If the electron mobility (b) Small but not zero
i s 8 . 2 5 m 2 / V- s , t h e n t h e (c) Infinite (a) A B Y (b) A B Y
concentration of donor atoms (d) Large but not infinite 0 0 0 0 0 1
will be (in m–3): 39. A positively charged particle 0 1 0 0 1 1
(a) 3.0×1016 (b) 3.0×1017 falls vertically downwards.
The horizontal component of 1 0 0 1 0 1
(c) 8.0×1018 (d) 3.0×1019 1 1 1 1 1 0
the Earth’s magnetic field will
35. Ratio of amplitude of two deflect is towards: (c) A B Y (d) A B Y
interfering waves is 2 : 1, then (a) West (b) East 0 0 1 0 0 0
ratio of amplitude of maxima to
(c) South (d) North 0 1 0 0 1 1
minima is:
40. For Fraunhofer single slit 1 0 0 1 0 1
(a) 4 : 1 (b) 9 : 1 diffraction:
(c) 3 : 1 (d) 9 : 4 1 1 0 1 1 1
(a) Width of central maxima is
36. The current density at every 44. A ball rolls off the top of stair
proportional to λ
point in a uniform conductor is way with a horizontal velocity of
(b) On increasing the slit width, magnitude 1.8 m/s. The steps aer
4 × 108 A m–2. The drift velocity the width of central maxima
of electrons in the conductor is 0.20 m high and 0.20 M wide.
decreases
2 × 10–4 ms–1. The carrier charge Which step the ball hit first?
(c) On making the slit width a = λ,
density is: (a) First (b) Second
central maxima spreads in the
(a) 2×1012 cm–3 range ± 90° (c) Third (d) Fourth
(b) 2 × 1018 cm–3 (d) All of the above are correct 45. Which of the following pairs of
(c) 0.5 × 108 cm–3 41. Which one of the following is a physical quantities does not have
(d) None of the above full duplex transmission system? same dimensional formulae?
Practice Set-2 17

(a) Work and torque 53. If A be n × n non-singular (a) l1 and l2 both are tangents at P
(b) Angular momentum matrix, then which of the (b) l2 and l3 both are tangents at P
(c) Tension and surface tension following is true? (c) l3 and l1 both are tangents at P
(d) Impulse and linear momentum (a) | Adj (Adj A) | = | A |2(n – 1) (d) None of the two lines are
(b) | Adj (Adj A) | = | A |(n – 1)2 tangents at P
MATHEMATICS (c) Adj (Adj A) = |A|n – 1 61. The eccentricity of the rectangular
46. If X = {4n – 3n – 1; n ∈ N}, Y = hyperbola is:
(d) None of these
{9(n – 1); n ∈ N}, then: (a) (b) 2
54. T h e c o e f f i c i e n t s o f t h r e e 2
(a) X ⊂ Y (b) X ⊆ Y
successive terms in expansion 1 1
(c) X = Y (d) X ⊇ Y of (1 + x)n are 165, 330 and 462 (c) (d)
47. If x = a + b, y = aω + bω 2 2 2
respectively, then the value of n
and z = aω2 + bω, then will be: 62. The angle between the lines a1x +
x3 + y3 + z3 is equal to: b1y + c1 = 0 and a2x + b2y + c2 = 0
(a) 11 (b) 10
(a) a3 + b3 (b) 27(a3 + b3) is:
(c) 12 (d) 8
(c) 3(a3 + b3) (d) None of these −1 a b + a2 b1
13 π (a) tan 1 2
48. If A = {1, 2, 3, 4} and R = {(1, 3), 55. tan is equal to: a1a2 − b1b2
(3, 4), (1, 4)}, then R is: 12
(a) 1 (b) 3 a1b1 − a2 b2
(a) Reflexive (b) Symmetric (b) tan −1
(c) Transitive (d) None of these (c) 2 + 3 (d) 2 − 3 a1a2 + b1b2
49. T h e r a t i o b e t w e e n s u m s 56. If the mean of five observations −1 a1b1 − a2 b2
of n terms of two series is (c) cot
x, x + 2, x + 4, x + 6, x + 8 a1a2 + b1b2
(2n + 1) : (2n + 5). The ratio is 11 then the mean of the first
between their 4th term will be: a1 a2 + b1b2
three observations: (d) cot −1
(a) 19 : 23 (b) 17 : 23 a1b2 – a2 b1
(a) 9 (b) 11
(c) 16 : 19 (d) 15 : 19 63. Solution of the differential
(c) 13 (d) None of these
50. Sum of n terms of series 1.4 + equation:
3.04 + 5.004 + 7.004 +.......will 57. The maximum value of sin θ +
cos θ is: (y – xy2)dx – (x + x2y)dy = 0 is:
be:
x
(a) n2 (a) 1 (b) 2 2 (a) log xy – =C
y
41 (c) 2 (d) 2
(b) n2 +   x
9n 58. If 3x – 4y + 4 = 0 and 6x – 8y – 7 (b) log – xy = C
y
4 = 0 are the tangents to the same
(c) n2 + (1 – 10n) circle, the radius of circle is:
9 x
(c) log + xy = C
3 y
4 1  (a) 3 (b)
1− 2
(d) n2 + 9  10n 
x2
3 3 (d) log – x2y2 = C
51. How many numbers between (c) (d) y2
4 8
2000 and 3000 may be formed 59. If ∠A = 90º, then the value of 1
from the digits 2, 3, 5, 7, 0, ? rr1 64. If cos–1 = θ , then tan θ will
will be: x
(a) 120 (b) 60 r2 r3
be equal to:
(c) 48 (d) 24 (a) 2 (b) 3
52. If one root of x2 – 4x + k = 0 is 2 (c) 4 (d) 1 1
(a) (b) x2 + 1
+ i, then the value of k is: 60. If l1, l2 and l3 are three lines x – y x2 − 1
(a) 4 (b) 5 + a = 0, 2x – 4y + a = 0 and 4x –
2y + a = 0 and P is a parabola (c) (d)
(c) 6 (d) 7 1 − x2 x2 − 1
y2= 4ax, then:
18 Practice Set-2

70. An ellipse having eccentricity (c) Ozone and carbon dioxide


x−2 2
65. lim x→∞ is equal to: (d) Nitrous oxide and sulphur
x−8  4
 7  and foci are ( 7, 0) and dioxide
0 1 77. Who will chair the Central
(a) (b)
0 2 ( − 7, 0) its equation will be:
Government formed committee
1 1 for coordination among the
(c) (d) (a) 4x2 + 4y2 = 25 Ministry of Petroleum and
4 2 2
Natural Gas(MoPNG), the
66. An integer is chosen at random 4 x2 4 y 2 Ministry of Environment and
from first 200 digits. What is the (b) + =1
49 21 Forest and Climate Change
probability that an integer chosen (MoEFCC) ?
(c) 2x2 + 3y2 = 17 (a) P K Sinha
is divisable by 6 or 8?
(a) 33/200 (b) 25/200 (b) Ajit Seth
(d) 2x2 + 2y2 = 18
(c) Nripendra Misra
(c) 58/200 (d) 50/200
REASONING AND GENERAL (d) Pramod Kumar Misra
π /2 sin x AWARENESS (RAGA) 78. When a number is increased by
67. The value of ∫0 1 + cos 2 x
dx is:
26, it becomes 113% of itself.
71. The Concurrent List includes: What is the number?
π π
(a) (b) (a) Preventive detention (a) 200 (b) 312
2 4 (b) Excise on alcoholic liquor (c) 390 (d) 234
π π (c) Census
(c) (d) 79. The L.C.M. of two numbers is 495
3 6 (d) Ancient and historical and their H.C.F is 5. If the sum
dx monuments of the numbers is 10, then their
68. ∫ is equal to: 72. Megasthenes visited India difference is:
(1 − x ) 1 + x 2
2
during the reign of: (a) 10 (b) 46
(a) Chandragupta II (c) 70 (d) 90
1 1 + x2
(a) cosh −1 (b) Chandragupta Maurya 80. By increasing the price of entry
2 1 − x2 (c) Ashoka ticket to a fair in the ratio
1 − x2 (d) Harsha 11 : 16, the number of visitors
(b) 2 cosh −1 73. Tirupati Hills are found in: to the fair has decreased in the
1 + x2
(a) Satpura Ranges ratio 19 : 9. In what ratio has
(b) Western Ghats the total collection increased or
(c) 1 1 − x2
cosh −1 (c) Aravalli Ranges decreased?
2 1 + x2 (d) Eastern Ghats (a) decreased in the ratio 209:144
(d) None of these 74. Lichen is an association of: (b) increased in the ratio 144:209
(a) Algae and Fungi (c) increased in the ratio 99:304
69. The nth differential coefficient of
(b) Fungi and Moss (d) decreased in the ratio 304:99
xn–1 log x is: (c) Moss and Fern 81. In a class of 56 students there are
 n n(n − 1) (d) Fungi and Fern 21 girls. The average weight of
(a) x2n  log x + −
 x 2 x2 75. Malicious software is known as: these girls is 56 kg and average
(a) Malware weight of the full class is 62.875
n −1
n ! kgs. What is the average weight
+ n(n − 1)(3n − 2) + ..... + (−1) n  (b) Maliciousware
3x nx  of the boys of the class ?
(c) Freeware
(a) 69.75 kg (b) 72.5 kg
(n − 1)! (d) Shareware
(b) (c) 67 kg (d) 65 kg
x 76. Acid rain is caused by the
pollution of environment by: 82. Find the roots of the quadratic
(c) xn log x + (log x)2 xn – 1 +.......+ equation 6x2 – 11x – 35 = 0 :
(a) Carbon dioxide and nitrogen
(log x)n x (a) 5/3, – 7/2 (b) – 5/3, 7/2
(b) Carbon monoxide and carbon
(d) None of these dioxide (c) – 3/5, 2/7 (d) 3/5, – 2/7
Practice Set-2 19

83. A missile travels at 1296 km/h. Direction (90): A series is given with
How many metres does it travel one term missing. Choose the correct
in one second ? alternative from the given ones that
(a) 320 metres (b) 360 metres will complete the series.
(c) 341 metres (d) 334 metres 90. 12, 24, 72, 288, ?
84. On a certain principal if the (a) 17 (b) 32
(a) 1440 (b) 3250 (c) 15 (d) 27
simple interest for two years is
` 2400 and Compound interest (c) 3025 (d) 2025 95. In the given diagram, square
for the two years is ` 2544, what Direction (91): Study the following represents doctors, triangle
is the rate of interest ? information carefully and answer the represents lady and circle
(a) 12 percent (b) 6 percent questions given below: represents surgeon. Which of
(c) 24 percent (d) 18 percent A, B, C, D, E, F, G and H are sitting the following letters represents
85. When one litre of water is added around a circle facing the centre. B is those ladies who are doctors and
to a mixture of acid and water, third to the right of F and third to the surgeon both?
the new mixture contains 20% left of H. C is fourth to the left of A,
acid. When one litre of acid is who is not an immediate neighbour
added to the new mixture, then of F or B. E is not an immediate
the resulting mixture contains neighbour of B. G is second to the
1 right of D.
33 % acid. The percentage of
3 91. Who is to the immediate right of
acid in the original mixture was. H? (a) M (b) G
(a) 20% (b) 22% (c) Q (d) P
(a) A
(c) 24% (d) 25% (b) E Directions (96–100): In the following
86. B is brother of D. D is sister of E. question, two statements are given
(c) F
E is brother of F. How is F related each followed by two Conclusions
(d) Data inadequate
to B? I and II. You have to consider the
Direction (92): In the questions
(a) Sister statements to be true even if they seem
below, some relationship have been
(b) Data Inadequate to be at variance from commonly
expressed through symbols as shown
(c) Brother known facts. You have to decide
below.
(d) Brother or sister which of the given Conclusions, if any,
f means less than follows from the given statements.
87. If TORTISE is coded as
VQTVKUG, ELEPHANT is D means not greater than 96. Statements:
coded as – means equal to (I) C o r r u p t i o n h a s c o m p l e t e
(a) GRJPVNOR + means not equal to establishment in each sector of
× means not less than our country.
(b) RNRQGCOV
= means greater than (II) Corruption has gone into their
(c) GNGRJCPV
Study the meaning of these deep roots of every process and
(d) GRJCPVGN symbols and pick correct answer system.
88. Starting from his house, Manish from the answer choices for each Conclusions:
moves 25 km towards south- questions below:
east then turns to the west and (I) People think about corruption in
92. x f y – z implies a routine way and not especially.
covers another 7 km. How far (in
(a) x × y – z (b) x – y × z (II) The eradication of corruption
kilometres) is he now from his
house? (c) x + y × z (d) x × y = z depends upon every individual.
Direction (93): Find the odd word/
(a) 21 (b) 23 No government can enforce rules
letters/number from the given
(c) 24 (d) 26 or regulation on people.
alternatives.
89. B is older than C but not as old (a) Only Conclusion II follows
as D. E is not as old as B. Who is 93. (a) UQ (b) JG (b) Conclusion I and II both
the oldest of all? (c) RN (d) NJ follow
(a) B (b) E 94. In the given figure, how many (c) Neither I nor II follow
(c) C (d) D people like cricket and tennis both ? (d) Only Conclusion I follow
20 Practice Set-2

97. Statements: Conclusions: (a) Only Conclusion II follows


(I) Wages are determined by (I) Children can connect well (b) Conclusion I and II both
minimum wage legislation in through specific cartoon follow
each country. characters. (c) Neither I nor II follow
(II) The mass campaign of
(II) Increase in wage rate is (d) Only Conclusion I follows
learning to school kids can
the sign of growth in an 100. Statements:
be effective only through
economy. making it interesting. (I) Demonetization has evolved
Conclusions: (a) Only Conclusion II follows need for learning electronic
(I) Average wage rate signifies (b) Conclusion I and II both payment system (EPS).
the wealth of a country. follow (II) EPS will enable people to
(II) Minimum wage legislation (c) Neither I nor II follow go cashless and use plastic
does not account wage (d) Only Conclusion I follows money for most of their
imbalances. 99. Statements: transactions.
(a) Only Conclusion II follows (I) Management includes
managing people, process, Conclusions:
(b) Conclusion I and II both (I) EPS is quick, safe, and easy
and plans.
follow way of transaction of money
(II) Handling people is most
(c) Neither I nor II follow difficult in terms of dealing from one account to another.
(d) Only Conclusion I follows up with various diversities (II) Cashless transactions will
98. Statements: and opinions. further curb the black
(I) Disney has launched 'Hour Conclusions: marketing of money in
of Code' campaign in India (I) Diversity brings in difference upcoming time.
for teaching computer of opinion because of
differences in cognitive (a) Only Conclusion II follows
programming to school
abilities. (b) Conclusion I and II both
children.
(II) Management is a multi­ follow
(II) Children watch cartoon for tasking procedure keeping
their entertainment and (c) Neither I nor II follow
in view 360 degrees of a
learning. situation or problem. (d) Only Conclusion I follows

Answers with Explanation


ENGLISH Hence, the words vague and sharp Look at these sentences :
are antonymous. You will pass if you work hard. I will
1. (b) 2. (a) 3. (a) 8. (d) The word Harmony (Noun) wait here until he comes back.
4. (b) Manoeuvre is the correctly means: a state of peaceful existence and I will wait here until he comes back
spelt word. The correct spellings of other agreement; the way in which different ↓ ↓
words are: misdemeanour, beginning; notes combine to make a pleasant Simple Future Simple Present
chameleon. sound. 13. (b) Make a mistake is a correct
5. (b) The correct spelling is: Look at the sentence: idiomatic expression. Hence, I made a
A family should live together in mistake – should be used.
irreparable.
perfect harmony. 14. (a) 15. (c) 16. (a)
6. (a) Separate
The word Discord (Noun) means: 17. (b) The word Paucity. Noun
7. (b) The word Vague (Adjective) disagreement; arguing; dissonance’. means: a small amount of something ;
means : not clear; indistinct; not giving Look at the sentence: less than enough of something: deficit
enough Information. The word Precise A note of discord surfaced during the short fall.
proceedings. Hence, the words harmony 18. (d)
(Adjective) means: clear and accurate;
and discord are antonymous. 19. (c) Our neighbour was bitten by
exact; meticulous. their dog yesterday.
9. (a) 10. (b) 11. (a)
The word Sharp (Adjective) means: 12. (c) Here, simple Future i.e. I will 20. (b) She exclaimed that he was a
clear and definite; having a fine edge. call you–should be used very stupid fellow.
Practice Set-2 21

PHYSICS Area vector of ABCD = L2 k̂ 1


34. (c) ρ =
1 neµ0
21. (b) Time taken for 1 bullet = Area vector of DEFA = L2 iˆ
n 1 1
Force = the rate of change of →
⇒ n= =
Total area vector A = L (iˆ + kˆ)
2
ρeµ0 0.25 × 1.6 × 10−19 × 8.25
momentum = mvn
= 10× 20× 10 = 2000 dyne = 3.0 × 1018 m −3
22. (b) Power 35. (c) We have
D (0,L,0) C(LL, O)
d m )E a2 2
= (kinetic energy) = × v 3 (0,L,L =
dt l k a1 1
i
2
m I max  a2 + a1 
Given, =100, v = 2 m / s X =
 a2 − a1 
A B(L,0,0)
l F(0,0,L) (0,0,0,) I nin
P = 100 ×(2)3 Total magnetic flux,  a2 
2

∴ = 800 W +1
→ →  a1   3
2

φ = B . A = B0 (iˆ + kˆ) . L2 (iˆ + kˆ) = B0 L2 (1 + 1) =  =   = 9 :1


23. (c) W = change in potential a  1
= 2B0 L2 Wb  2 − 1
energy = mgh.  a 
1
24. (c) The limits of σ are: (iron) –1 29. (b) Number of oscillation in
1 coherence length amax 9
to . ∴ =
2 l 0.024 amin 1
25. (a) W = JQ ⇒ W = 42 × 250 = = = 4.068 × 104
λ 5900 × 10−10
⇒ W = 1050 J 3
= = 3 :1
26. (b) 30. (c) The core of the transformer 1
V2 Q 1 P R should have high permeability and low 36. (a)
Q= t or P ∝ ⇒ P = O >1
R t R PQ RP coercivity. Also the area of the B-H curve I nevd A
∴ RP < RQ should be small, so that loss of energy due J= = = (ne) vd
A A
27. (b) Given to hysteresis is low.
∴ Carrier charge density i.e.,
31. (d) It is because the number of
π  molecules striking the wall per unit time J 4 × 108 × 104
y = 3cos  − 2ωt  ...(i) ne = = = 2 × 1012 cm −3
4  increases. vd 2
dy π 
Velocity, V = = 3 × 2 ω sin  − 2ωt  32. (b) From Kepler’s law: T 2 ∝ r 3 37. (c) In a hydrogen atom, electron
dt 4 
2 3 revolving around a fixed proton nucleus
Acceleration:  T1  r 
  = 1 has some centripetal acceleration.
dV π   T2   r2  Therefore, its frame of reference is non-
A= = − 4ω2 × 3cos  − 2ωt 
dt 4   365 days  2
2 3
inertial. In the frame of reference, where
⇒   =  
 T  1 the electron is at rest, the givn expression
As, A ∝ Y and negative sign shown cannot be true for binding energy as the
that it is directed towards equilibrium ⇒ T = 129 days
frame in which electron is at rest would
(or mean position), hence particle will 33. (c) When magnetic field is
execute SHM. not be intertial.
perpendicular to motion of charged
Comparing (i) with equation particle, then: 38. (a) If the magnetic field is
y = r cos (φ − ω ' t ) parallel to a surface, then the magnetic
mv 2 mv
⇒ = Bqv ⇒ R = flux through the surface is zero.
We have, R Bq → → →

2π π 39. (b) Apply Fm = q0 v × B . Here,


ω ' = 2ω ⇒ = 2ω ⇒ T ' = Further, time period of the motion,

T' ω v is downward and B is along the

 mv 
→ 2π   →
28. (b) Here, B = B (iˆ + kˆ) 2πR  Bq  2πm north. Applying right hand first rule, Fm
0 T= = ⇒T=
v v Bq comes out to be along the east.
22 Practice Set-2

40. (d) For Fraunhofer diffraction Horizontal distance travelled = nx According to questions,
of a single slit we have all the condition = ut n
true i.e., central maxima has width [2a + (n − 1)d ] 2n + 1
nx 2
∝ λ , on increasing the slit width, the t= = 2n + 5
u
n
width of central maxima decreases. Also [2a1 + (n − 1)d1 ]
2
1 n2 x2
2
if a  λ the central maxima has angular 1 2 1  nx 
ny = gt = g .   = g
separation of ± 90° . 2 2  u  2 u2 2a + (n − 1)d 2n + 1
⇒ =
41. (c) Telephone is a full duplex 2u 2 2
y 2 × (1.8) × 0.2 2a1 + (n − 1)d1 2n + 5
transmission system: n= = =4
9 x 9.8 × (0.2) 2
1 n −1
ωL − =3
ωC 45. (c) Tension = Force = [M L T ]1 1 –2
2
42. (d) tan φ =
R Force a + 3d 15
Surface tension =
Length ⇒ putting n = 7, =
φ being the angle by which the current a1 + 3d1 19
leads the voltage Given, φ = 45°
[M1L1T − 2 ]
1 = = [M1L0 T − 2 ] Therefore, ratio of 4th term = 15: 19
ωL − L
∴ tan 45° = ω C 50. (d) Sn = 1.4 +3.04 + 5.004 +
R So, tension and surface tension does
not have same dimensional formula. 7.0004 +..........upto n terms
1 = (1 + 0.4) + (3 + 0.04) + (5 +
ωL −
⇒ 1= ωC MATHEMATICS
0.004) + (7 + 0.0004) +.............n terms
R 46. (a) X = {0, 9, 54,..........} and
= (1 + 3 + 5 + 7 +........upto n
1 Y = {0, 9, 18, 27,36, 45, 54,...............}
⇒ R = ωL − terms) + (0.4 + 0.04 + 0.004 + 0.0004
ωC Hence, X ⊂ Y.
47. (c) x + y + z +.............upto n terms)
1
⇒ ωC = = (a + b) + (aω + bω2) + (aω2 + bω) n
(ωL – R) = [2.1 + (n – 1).2]
= a(1 + ω + ω2) + b(1 + ω + ω2) 2
1 1
⇒ C4 = = =a×0+b×0 4 4 4 
ω (ωL − R) 2πf (2πf L − R) +  + 2 + 3 + ......upto n terms 
43. (d) We can simplify the gate [ 1 + ω + ω2 = 0] = 0 10 10 10 
circuit as:  x3 + y3 + z3 – 3xyz
A A+B = (x + y + z) (x2 + y2 + z2   1 n 
B 1 −   
– xy – yz – zx) 4  10  
= n + 
2
Here, gates I and II are NOR gates. ∴ x3 + y3 + z3 – 3xyz = 0 10  1
The output (A+B) of gate I will appear ⇒ x3 + y3 + z3 = 3xyz  1−   
  10  
as input of gate II. The final output is: = 3(a + b)(aω + bω2) (aω2 + bω)
= 3(a + b) [a2ω3 + abω2 + abω4 + b2ω3] 2 4 1 
Y=A+B=A+B = n + 1 − n 
= 3(a + b) [a2 + ab(ω2 + ω) + b2] 9  10 
This is the Boolean expression of OR = 3(a + b) (a2 – ab + b2) 51. (d) All those numbers which are
gate whose truth table is given below: (because ω2 + ω = – 1) in between 2000 and 3000 will be of 4
= 3(a3 + b3) digits and of their first digit will be 2,
A B Y
48. (c) Since, 1R3 and 3R4 ⇒ 1R4 therefore assuming 2 as fixed and taking
0 0 0
∴ R is transitive 3 digits out of 4, it can be put in 4P3 i.e.,
0 1 1
Thus, R is only transitive 24 ways.
1 0 1
R is clearly neither reflexive nor ∴ Number of total numbers in
1 1 1 symmetric. between 2000 and 3000 by these given
44. (d) Let ball strike the nth step of 49. (d) Let first term of first series is digits
stairs, Vertical distance travelled = ny = a and common difference is d and first
1 = 4P3
term of second series is a1 and common
n × 0.20 = gt 2
2 difference is d1.
= 4 × 3 × 2 = 24
Practice Set-2 23

52. (b) If one root is 2 + i other root 56. (a)


rr1 ∆2 ( s − b)( s − c)
is 2 – i. Then sum of the roots is 4 and ∴ = .
x + x + 2+ x + 4+ x +6+ x +8 r2 r3 s ( s − a ) ∆2
product of roots = 5 = k. l1 =
5
( s − b)( s − c) 2 A
53. (b) We know that, ⇒ 5x + 20 = 55 = = tan
s( s − a) 2
⇒ x=7
| Adj A | = | A |n – 1
∴ Mean of first three observations 90°
= tan 2 =1
If | A | ≠ 0 ....(i) 2
7 + 9 + 11
= =9 60. (c) x–y+a=0⇒y=x+a
Replacing A by Adj A in (i), we get 3
On putting y = x + a in y2 = 4ax, we get
| Adj (Adj A) | = | Adj A |n–1 57. (d) sin θ + cos θ
(x + a)2 = 4ax
 1 1  ⇒ x2 – 2ax + a2 = 0

= {| A |n – 1}n – 1 = 2 sin θ + cos θ 
2
 2 2  ∴ ∆ = b2 – 4ac

= | A |(n – 1)
= 2(cos 45° sin θ + sin 45° cos θ) = 4a2 – 4a2 = 0
54. (a) Let coefficients of three
= 2 sin(θ + 45°) ∴ l1 touches the parabola.
consecutive terms, i.e. (r + 1)th, (r + 2)th
and (r + 3)th in expansion of (1 + x)n are 2x + a
= 2 ×1 2x – 4y + a = 0 ⇒ y =
165, 330 and 462 respectively then 4
[Because max. value of sin (θ + 45°) = 1]
2x + a
coefficient of (r + 1)th term = nCr = 165 = 2 Putting y = in y2 = 4ax, we get
4
coefficient of (r + 2)th term = nCr + 1 = 330 58. (c) Since, the given tangents are
parallel to each other, therefore radius 2
coefficient of (r + 3)th term = nCr + 2 = 462  2x + a 

of the circle will be half of the distance   − 4ax = 0
n
Cr +1 n−r  4 
∴ = =2 between these tangents.
n
Cr r +1 ⇒ 4x2 + 4ax + a2 – 64ax = 0
 Perpendicular distance from
(0, 0) to the line 3x – 4y + 4 = 0 is ⇒ 4x2 – 60ax + a2 = 0
⇒ n – r = 2(r + 1) ∴ ∆ = b2 – 4ac
4 4
1 = and perpendicular = 3600 a2 – 4 × 4 × a2
⇒ r = (n − 2) 32 + 42 5
3 = 3584 a2
distance from (0, 0) to the line 6x – 8y – 7
∴ l2 does not touch the parabola
n
Cr + 2 n − r −1 −7 −7
is = 4x + a
and n = 2
6 +8 2 10 Similarly, 4x – 2y + a = 0 ⇒ y =
Cr +1 r+2 2
∴ Distance between these two lines
231 4x + a
= 4  −7  15 3 Putting y = in y2 = 4ax, we get
165 = −   = = 2
5  10  10 2 2
⇒ 165(n – r – 1) = 231 (r + 2)  4 x + a  – 4ax = 0

 
⇒ 165n – 627 = 396 r 3  2 
∴ The radius of the circle = 4
1
⇒ 165n – 627 = 396 × (n − 2) a2
3 ∆ ∆ ⇒ 4x2 – 2ax + =0
59. (d) Numerator rr1 = . 4
⇒ 165n – 627 = 132 (n – 2) s s−a
∴ ∆ = b2 – 4ac
2
⇒ 33n = 363 ∆
= a2
∴ n = 11 s( s − a) = 4a2 – 4 × 4 × =0
4
13π tan13 × 180° ∆ ∆ ∴ l3 touches the parabola
55. (d) tan =
12 12 and denominator r2r3 = s − b . s − c 61. (a) In rectangular hyperbola a = b
= tan 195°  b2 = a2(e2 – 1)
∆2
= tan (180° + 15°) = ⇒ b2 = b2 (e2 – 1)
( s − b)( s − c)
= tan 15° = 2 − 3 ⇒ 1 = e2 – 1
24 Practice Set-2

⇒ e2 = 2 ⇒ log x – xy – log y = C 67. (b) Let cos x = t


⇒ e = 2 . ∴ – sin x dx = dt
x
⇒ log − xy = C and when x = 0, t = 1, and when x
62. (d) The slope m1 of the line a1x + y
a π
b1y + c1 = 0 is − 1 . 1 = , then t = 0
b1 64. (d)  cos–1 =θ 2
x π/ 2 sin x
and the slope m2 of the line a2x + b2y ∴ I = ∫0 dx
1 + cos 2 x
a
+ c2 = 0 is − 2 . 0 dt 1 dt
b2
x
=– ∫1 1 + t 2 = ∫0 1 + t 2

x –1
If θ be the angle between the lines,

2
–1 1
then = (tan t )0
m2 − m1 θ
= tan–1 (1) – tan–1 (0)
    tan θ = 1
1 + m1m2 π π
1 = −0 =
∴ cos θ = 4 4
 a2   a1  x dx
− −− 
 b2   b1 
68. (a) Let I = ∫ (1 − x ) 1 + x 2
2
= ∴ tan θ = x2 − 1
 a  a 
1 +  − 2  − 1  1 + x2 t 2 −1
 b2  b1  x −2 2  0 Let t2 = ⇒ x2
65. (c) lim x →8 form 0  1 − x2 t2 +1
x −8  
a1 a2 4t dt
− ⇒ 2x dx =
b1 b2 a b − a2 b1 1 (1 + t 2 ) 2
= = 1 2
a1 a2 b1b2 + a1a2
1+ = lim x →8 2 x 1
b1 b2 1 ∴I= ∫ .
  t − 1  
2  t 2 −1 
b1b2 + a1a2 [By L’ Hospital rule] 1 −  2   1 +  2 
∴ cot θ =   t + 1    t +1
a1b2 − a2 b1
1 1
= = 2t dt
a1a2 + b1b2 2 8 4 2 2 2
= x(1 + t )
a1b2 − a2 b1
a1a2 + b1b2 66. (d) The number (< 200) which
−1 1 1 1
∴ θ = cot are divisible by 6 cosh −1 t
a1b2 − a2 b1 =
2
∫ 2
t −1
dt =
2
 200 
63. (b) The given equation can be =   = 33
written as: (1 – xy)ydx – 1(1 + xy)xdy = 0  6 
1 1 + x2
The number (< 200) which are    = cosh −1
Here, M= y – xy and N = – x – x y
2 2
2 1 − x2
divisible by 8
Mx – Ny = (xy – x2y2) – (–xy – x2y2) 69. (b) y = xn – 1 log x
 200 
= 2xy ≠ 0 =   = 25
 8  1
1 1 y1 = xn – 1. + log x. (n – 1). xn – 2
∴ I.F. = = The number which are divisible by x
Mx − Ny xy
6 and 8 both
∴ Multiplying the given equation = xn – 2 [1 + (n – 1) log x]
 200 
by I.F., we get =  =8
 24   n −1 
1  1  y2 = xn – 2.   + [1 + (n – 1) logx].
 − y  dx −  + x  dy = 0, ∴ Total number which are  x 
x  y  divisible by 6 or 8 (n – 2) xn – 3
which is exact equation. Its solution = 33 + 25 – 8 = 50
is: ∴ Required probability = xn – 3 [(2n – 3) + (n – 1) (n – 2) log x]
1  1 (n − 1)(n − 2)
∫  x − y  dx + ∫ − y dy =C =
50 1
= y3 = xn – 3 + [(2n –3) +
200 4 x (n –1)
Practice Set-2 25

(n – 2) log x](n –3)xn – 4 75. (a) Malicious software, 80. (a) Let required ratio x/y, then
= xn – 4 [(3n2 – 12n + 11) + (n – 1) commonly known as malware, is any
(n – 2) (n – 3) log x] software that brings harm to a computer = =
yn –1 = xn – n [(some terms in n) + system. Malware can be in the form
(n – 1)! log x] x : y = 209 : 144
of worms, viruses, trojans, spyware,
= d ecreased in the ratio 209:144
= (some terms in n) + (n – 1)! log x adware and rootkits, etc., which steal
protected data, delete documents or add 81. (c) Total weight of 21 girls = 56
(n − 1)! (n − 1)! × 21
∴ yn = 0 + = software not approved by a user.
x x = 1176 kg
76. (d) Acid rain is caused by
70. (b) 2ae = 7+ 7 Total weight of full class
chemical reaction that begins when
= 62.875 × 56
⇒ 2ae = 2 7 compounds like sulphur dioxide and
= 3521 kg
nitrogen oxides are released into the air.
4 Total weight of (56 – 21) = 35 boys
a = 7 77. (a) P K Sinha Explanation: On June
7 23, 2019, Central Government has = 3521 – 1176 = 2345 kg
∴ a = 7/2 formed a committee for coordination Average weight of 35 boys
b2 = a2 (1 – e2) among the Ministry of Petroleum and = = 67 kg
49  4  Natural Gas(MoPNG), the Ministry of
= 1 −  Environment and Forest and Climate 82. (b) 6x2 – 11x – 35 = 0
4  7
49  3  Change(MoEFCC) and the state 6x2 – 21x + 10x – 35 = 0
21
=   = governments after a tug of war between 3x (2x – 7) + 5 (2x – 7) = 0
4 7 4
the two ministries. The panel formation (2x – 7) (3x + 5) = 0
∴ Equation of ellipse will help in speeding up hydrocarbon 2x – 7 = 0 or 3x + 5 = 0
exploration-related clearances.
x2 y 2
+ =1 Members: The newly formed committee x= or x =
49 21
will have P K Sinha, Cabinet Secretary
4 4 Distance
as chairman and secretaries of home, 83. (b) Speed = = 1296
4x 4y 2 2
km/h Time
+ =1 petroleum and environment ministry,
49 21 a representative from the NITI Aayog
& and chief secretaries of petroleum- 5
REASONING AND GENERAL = 1296 × m/sec
/ sec
producing states as members. 18
AWARENESS (RAGA) 78. (a) Let the number be x 216
71. (a) = × 5 = 72 × 5
According to question 3
72. (b) Megasthenes was a Greek x + 26 = x × = 360 m/sec
historian who came to India in the
Therefore, the missible will cover
fouth century B.C. as a representative 100x + 2600 = 113x
360 metres in one second.
or ambassador of Seleucus Nicator.   113x – 100x = 2600
He lived in the court of Chandragupta 13x = 2600 84. (a) ID = IC – IS = (2544 – 2400)
Maurya for about five years (302-298        = `144
x= = 200
B.C.)  RI 
ID =  S 
73. (d) The Tirumala Hills are 79. (a) Let the numbers be x and  200 
located along the Seshachalam- (10 – x)
Then, x (10 – x) = 5 × 495 2400 × R
Velikonda Range of the Eastern Ghats. 144 =
⇒ x2 – 10x – 2475 = 0 200
The Palar River cuts through the ranges.
⇒ (x – 55) (x + 45) = 0 144 × 200
74. (a) A lichen is a composite ⇒ x = – 55 or x = 45 ⇒ R= = 12%
organism that arises from algae or 2400
∴ The numbers are 45 and 55
cyanobacteria living among filaments of Required difference = (55 – 45) 85. (d) Let the original mixture of
multiple fungi. = 10 acid/water be x litre.
26 Practice Set-2

Then, in (x + 1) litre of mixture, acid is The Blood relationship tree/chart The missing term is 1440.
20%. Quantity of acid can be constructed as given below: 91. (b) By analysing the circular
B(+) → D(–) → E(+) → F(?) arrangement, it is clear that immediate
20  x + 1
= ( x + 1) × =  Therefore, ‘F’ is either brother or Right of ‘H’  E.
100  5 
sister of ‘B’. 92. (c) According to the meaning of
x +1 [Note: ‘+’  Male; ‘–’  Female].
+1 the given symbols,
100
Again, 5 × 100 = 87. (c) As,
x+2 3 xfy–zx<y=z
T O R T I S E V Q TVK U G
x +1+ 5 100 +2 Using the proper symbols in option
or × 100 =
5( x + 2) 3 +2 (c), we get x + y × z  x ≠ y >| z  x <
+2
5( x + 2) +2 y = z.
or x+6= +2
3 Therefore, x f y – z  x + y × z.
+2
or 3 x + 18 = 5 x + 10 +2 93. (b)
–4
Q
UU  → Q
18 − 10 = 5 x = 3 x –3
or Similarly,   JJ   G
→ G
or 8 = 2x ELEPHANT G NG RJCPV –4
RR  → N
N
+2 –4
or x = 4L +2 NN  → JJ
4 +1 +2
And quantity of acid = = 1L +2 94. (b) The number of people who
5 +2 like cricket and tennis both can be
+2
Percentage of acid in original +2 represented by the numbers common to
mixture +2 the triangle and the circle. Such numbers
1 88. (c) are 17 and 15.
= × 100 = 25%
4 O
start N ∴ Required answer = (17 + 15) = 32
25 km W E 95. (a) It is clear from the diagram
Alternate Method
that M is common in all the three figures
Here, we assume that total quantity end S
B
A i.e., square, triangle and circle. It means
of the mixture be 4L as after adding 1 L of 7 km
that the required letter is M.
water, the quantity of acid in the mixture
∴ OB = 96. (d) According to the statements,
1
becomes 20%, i.e. And adding 1L of only Conclusion I follows.
5 = = = 24 km
97. (d) According to the Statements,
acid in the mixture, the quantity of acid Finally, Manish is 24 km far from only Conclusion I follows.
1+1 1 his house.
becomes 6 = 3 i.e. 33.33%. 98. (b) According to statements,
89. (d) According to question, the Conclusion I and II follow.
 In original mixture, quantity of sequence is,
99. (a) According to statements,
acid D>B>C>E only Conclusion II follows.
1
= × 100 = 25% Finally, D is the oldest of all. 100. (c) According to statements,
      4
90. (a) neither Conclusion I nor Conclusion II
86. (d) B  brother of D.
12 24 72 288 1440 follow.
D  sister of E.
E  brother of F. ×2 ×3 ×4 ×5 qqq
Practice Set-3 27

Air Force Airmen Group X & Y Exam


Practice Set-3

(c) easy-more easy-most easy 12. He was sorry ______ late last
ENGLISH night.
(d) special-specialates-specialist
Directions (1-4): Read the following Directions (5–6): In the following (a) about coming
passage and answer the questions that questions, groups of four words are (b) to come
follow. given. In each group, one word is (c) to coming
Animals may become extinct in many correctly spelt. Find the correctly (d) from coming
ways. First of all they may evolve spelt word.
5. (a) aknoledge (b) acknowledge Directions (13–14): In the following
into another species and not really die questions, a part of the sentence
out of all. For example, through time, (c) acknoledge(d) acknolege is underlined. Below are given
many early form of horses and human 6. (a) posess (b) possess alternatives to the underlined part
beings have progressively changed by (c) posses (d) possus (a), (b) and (c) which may improve
evolution into new species. The old the sentence. Choose the correct
species has changed and not really die 7. Choose the one which best
alternative. In case no improvement is
out, so this is called pseudo extinction. expresses the meaning of the given
needed, your answer is (d).
The second common way, in which word. 13. If I studied well, I will pass.
animals have died out, is for a single RELISH (a) I passed
species to disappear because of some (a) realise (b) taste (b) I pass
local disturbances. Many animals that (c) enjoy (d) reveal (c) I would pass
have very special diets, for example, Directions (8–9): In the following (d) No improvement
could die out very easily if their source questions, choose the word that is 14. She reminded where to leave the
of food goes down. It is thought that opposite in meaning to the given word. papers.
certain species of dinosaurs were adapted 8. EXEMPTIONS
to eating particular kinds of reedy plants. (a) me where I had to leave
(a) generalisation (b) where I had to leave
When these disappeared, those particular (b) liberalisation
species of dinosaurs starved and died out. (c) me where to leave
(c) exclusions (d) No improvement
1. T h e w o r d s ‘ e x t i n c t ’ a n d
(d) inclusions Directions (15–16): In the following
‘particular’ are–
9. PROMINENT questions some of the sentences have
(a) adjectives (b) adverbs
(c) nouns (d) verbs (a) unknown (b) eminent errors and some are correct. Find out
2. Which of the following phrases has (c) renowned (d) important which part of a sentence has an error.
a determiner in it? Directions (10–12): In the following The number of that part is the answer.
(a) For example questions, sentences are given If a sentence is free from errors, then
with blanks to be filled in with an your answer is (d) i.e. No error.
(b) The old species appropriate word(s). Four alternatives 15. The Committee is thinking (a) / not
(c) Local disturbances are suggested for each question. to bring around any change (b) /in
(d) Starved and died out Choose the correct alternative out of the basic structure. (c)/No error. (d)
3. The verb in the sentence “The old the four.
10. Our new digital photo-print 16. The Chief Minister had no time (a)/
species has changed and not really
machine, the manual ______ is lost to pay attention (b) on such simple
died out” is in:
awaits to be set up properly, matters.(c) / No error. (d)
(a) simple present
(b) simple past (a) of what (b) where 17. In the following questions, out of
the four alternatives choose the
(c) present perfect (c) by whom (d) of which
one which can be substituted for
(d) past perfect 11. Ramu is in the habit of revising
the given words/sentence.
4. Which of the following has the just once, but Suresh never
understands a subject ______ he A foreigner who settles in a
three degrees of the adjective in
revises it for atleast three times. country.
their correct form?
(a) in case (b) thus (a) Immigrant (b) Emigrant
(a) early-earlier-earliest
(c) although (d) unless (c) Alien (d) Visitor
(b) hot-more hot-hottest
28 Practice Set-3

18. Choose the correct from of verb diameter half of the first one, the
that is in agreement with the
subject:
increase in length will be: (b)
1
3
(iˆ + ˆj + kˆ )
(a) 0.5 cm (b) 2 cm
My friend and I..............invited to
the party.
(c) 4 cm (d) 8 cm
23. The wings of an aeroplane
(c)
1
3
(iˆ + ˆj + kˆ )
(a) is being (b) has been
(c) have been (d) was are 10 m apart. The plane is
19. In the following questions, moving horizontally towards the (d)
1
2
( ˆj + kˆ )
sentence has been given in Direct/ north with a velocity of 200 ms–1
Indirect Speech. Out of the four at a place where the vertical 27. In an adiabatic change, the
alternatives suggested select the component of earth’s magnetic internal energy changes by –100
one which best expresses the same field is 0.5 × 10–4 T. The induced J. The work done by the gas will
sentence in Indirect/Direct Speech. e.m.f. set up between the tips of be:
The captain said to the army, the wings is: (a) 0 (b) –100 J
“March forward now.” (a) 0.1 V (b) 0.15 V (c) 100 J (d) 200 J
(a) The captain said to the army that (c) 1 V (d) 1.5 V 28. The minimum initial velocity a
march forward now 24. A projectile has a horizontal range rocket must be given so that if
(b) The captain ordered the army to
march forward then R for two different angles. If h1 reaches the target 100 km away
(c) The captain ordered the army to and h2 are the maximum height in the same horizontal plane is:
march on that day reached then: (a) 10 ms–1
(d) The captain ordered the army to (a) R = h1 h2 (b) 100 ms–1 at 45° with the horizon
attack the enemy (b) R = h1 h2 (c) 1000 ms–1
20. In the following question, a (c) R = 4 h1 h2 (d) 1000 ms –1 at 45° with the
sentence has been given in Active/ horizon
Passive Voice. Out of the four (d) R = 2 h1 h2 29. In the given circuit the current is:
alternatives suggested, select the 25. The minium surface density of B
A C
one which best expresses the same charge on the plate, so that a 2Ω 10 V 5 V 2Ω
sentence in Passive/Active Voice. body of mass 2 kg/m2 may just
It is being read by us. be lifted is:
(a) We are reading it. (a) 2.84 × 10–5 c/m2
(b) It will be read by us. (b) 2.25 × 10–5 c/m2 E D
(c) We can read it. 1Ω
(c) 1.86 × 10–5 c/m2 (a) 1.0 A from A to C via B
(d) We have to read it.
(d) None of these (b) 1.0 A from C to A via B
PHYSICS 26. A non-planar loop of conducting (c) 3.0 A from A to C via B
→ wire carrying a current I is
21. An electric field E and magnetic (d) 3.0 A from C to A via B
placed as shown in the figure.

Each of the straight sections 30. A c a r i s a p p ro a c h i n g t h e
field B exist in a region. If these road-crossing with speed
fields are not perpendicular to each of the loop is of length 2a. The
magnetic field due to this loop at 72 km/hr. To the sepoy standing
other, then the electromagnetic
the point (a, 0, a) points in the on the crossing, the frequency of
wave:
direction: horn of the car appears 260 Hz.
(a) will not pass through the Y The speed of sound is 332 m/s. The
region Z
actual frequency of the horn is:
(b) will pass through region (a) 260 Hz (b) 280 Hz
(c) may pass through the region (c) 244.3 Hz (d) 275.7 Hz
(d) nothing is definite 31. Two positively charged particles
22. W h e n a c e r t a i n w e i g h t i s each having charge Q are d
suspended from a long uniform X distance apart. A third charge is
wire, its length increases by 1 cm. introduced in midway on the line
If the same weight is suspended joining the two. Find nature and
from another wire of the same
material and length but having a
(a)
1
2
( − ˆj + kˆ ) magnitude of third charge, so that
the system is in equilibrium:
Practice Set-3 29

Q Q (a) Proportional to R A
(a) q = − (b) q =
4 4 3Q (b) Proportional to 1/R

(c) q = 3Q (d) q = − (c) Proportional to 1/R2 q
4 4
32. In a sample of helium gas, its (d) Independent of R
molecules have root-mean mean- 36. The surface tension of a liquid is
B
square velocity 5/7 times the 70 dynes/cm. It may be expressed E
rms velocity of hydrogen gas in MKS system is: C D
molecules. If the temperature (a) 7×10–2 newton/m
(a) is least along the path AB
of the hydrogen as is 0°C then (b) 70 N/m
(b) is least along the path AD
temperature of helium gas will (c) 7×102 N/m (c) is zero along any one of the
nearly be: (d) None of these path AB, AC, AD and AE
(a) 5°C (b) 4K 37. A circuit shown in the figure, is (d) is least along AE
(c) 273°C (d) 100°C in the form of letter Y. The three 41. When p-n junction diode is
33. A charge q moves with a velocity terminals of the circuit A, B, C forward biased, then:
2 m/s along x-axis in a uniform have potentials 6 V, 3 V and 2V (a) the depletion region is reduced
→ respectively. The potential of node and barrier height its increased
magnetic fieldB = ( iˆ + 2 ˆj + 3kˆ )T O is:
then charge will experience a (b) the depletion region is widened
+6 V +3 V and barrier height is reduced
force. A B
(a) In z-y plane (c) both the depletion region and
(b) Along-y axis 6Ω 3Ω barrier height are reduced
(c) Along +z axis (d) both the depletion region and
O barrier height are increased
(d) Along-z axis
34. Heat is passing from reservoir A to 2Ω 42. A small object of mass m is
B through two rods of brass and attached to a light string, which
iron of same length and area of C passes through a hollow tube.
+2 V The tube is held by one hand
cross-section. If K1 and K2 are the
thermal conductivities of brass (a) V0 = 5 V (b) V0 = 6 V and the free end of the string by
and iron, then equivalent thermal (c) V0 = 3 V (d) V0 = 2 V the other. The object is set into
conductivity is given by 38. The frequency of the horn of the revolution in a horizontal circle
car appears to be 2.5% different of radius r1 with a speed v1. The
K1 K 2 K + K2
(a) (b) 1 from its actual frequency to the string is pulled down, shortening
K1 + K 2 K1 K 2
observer towards whom the car the radius of the circular path to
(c) K1 + K2 (d) K1 . K2 is approaching. If the speed of r2. The relation of the new angular
35. In the fig. given below, a weight sound in air is 320 m/s, then the velocity ω2 to the original angular
W is attached to a string wrapped speed of car will be: speed ω1 is:
round a solid cylinder of mass (a) 7.8 m/s (b) 800 m/s
M mounted on a frictionless (c) 7.5 m/s (d) 6 m/s (a) (b)
horizontal axle at 0. If the weight 39. If I is the moment of inertia. B is
starts from rest and falls a the magnetic induction, M is the (c) (d)
distance h, then its speed at that magnetic moment, the dimensions
instant is: of I are: 43. A signal of 20 mV is applied
R MB to common emitter transistor
amplifier circuit. Due to this, the
O (a) [M0L0T0] (b) [M0L0T1]
change in the base current and the
(c) [ML2 T–2] (d) [ML1T–1] change in collector current are 20
40. In the electric field of a point µA and 2 mA. The load resistance
charge q, a certain charge is is 10 kΩ. The voltage gain is:
carried from point A to B, C, D (a) 20 V (b) 10 V
W
and E. Then the work done: (c) 50 V (d) None of these
30 Practice Set-3

44. Mark correct option or options: 50. Find the coefficient of x2 in the (a) Only I
(a) The image formed by a convex expansion of (b) Only II
lens may coincide with object  1 (c) Both I and II
(b) The image formed by a plane  3x − x  is: (d) Neither I or II
mirror is always virtual 57. If log10 2, log10 (2n – 1), log10 (2x
(c) If one surface of convex lens (a) 405 (b) 7290 + 3) are three consecutive terms
is silvered, then the image may (c) 2430 (d) 1215 of an AP, then which one of the
coincide with the object 51. Which one of the following following correct?
(d) Both (a) and (b) are correct function is differentiable for all (a) x = 0 (b) x = 1
45. If red light is replaced by blue real values of x? (c) x = log35 (d) x = log52
light illuminating the object in a (a) x / | x | (b) x | x | 58. The greatest positive integer,
microscope, the resolving power (c) 1 / | x | (d) 1/ x which divides (n + 2) (n + 3)
of the microscope: 52. If f(x) = loga (loga x), then f '(x) is (n + 4) (n + 5) (n + 6) for all n ≠
(a) will decrease equal to. N, is
(b) will increase (a) 4 (b) 120
log a e log e a (c) 240 (d) 24
(c) will get halved (a) (b)
(d) will remain unchanged x loge x x loga x 59. Which one of the following
log e a x differential equations represents
(c) (d)
MATHEMATICS x log e a the system of circles touching
log x y-axis at the origin?
46. The equation of the ellipse, whose 53. What is ∫ dx equal to?
centre is at origin (0, 0), foci (+ 1, (1 + log x )2 (a)
dy
= x2 – y 2
1 1 dx
0) and eccentricity , is (a) +C
dy
2 (1 + log x )3 (b) 2 xy = y 2 – x 2
y2 y2 x2 y 2 1 dx
(a) + = 1 (b) + =1 +C
3 4 4 2 (b) dy
(1 + log x )2 (c) 2 xy = x 2 – y 2
2 2 dx
(c) x + y = 1 (d) x2 + y2 = 12 (c)
1
+C dy
(d) = y 2 – x2
3 4 (1 + log x) dx
 π  π dy
1 60. If xm + ym = 1, such that =
47. If zr= cos   + i sin  r  , r = 1, (d) +C dx
2r  2 (1 + log x )2 x
,
– y then what should be the
2… ∞, z1, z2, z3 … ∞ is equal to 54. If the lines 3y + 4x = 1, y = x + 5
value of m?
(a) 1 (b) – i and 5y + bx = 3 are concurrent,
(a) 0 (b) 1
(c) i (d) – 1 then what is the value of b?
(c) 2 (d) –1
48. If a, b and c are positive and are (a) 1 (b) 3
61. M e d i a n , f o r t h e f o l l o w i n g
is AP, The roots at the quadratic (c) 6 (d) 0
distribution is
equation ax2 + bx + c = 0 are real 55. The coefficient of x4 will be in the
for: 5x x Wages (`) Number of labour
expansion of e + e . 0-10 22
c e3 x 10-20 38
(a) –7 ≥4 3 (a) 4/3 (b) 2/3
a 20-30 46
(c) 3/4 (d) 1/2
a 56. Consider the following state- 30-40 35
(b) –7 <4 3
c ments in respect of circles 40-50 20
x2 + y2 – 2x – 2y = 0 and x2 + y2 = 1 (a) 24.46 (b) 14.80
(c) All ard B
(c) 34.21 (d) None of these
(d) No and C. I. The radius of the first circle
62. The total number of selections of
49. If the nth term ot an arithmetic is twice that of the second circle.
atmost n things with 10 points as
progression in 3n + 7, then what II. Both the circles pass through
vertices, n of them being collinear,
is the sum, of its first 50 terms? the origin.
is 110. Then, n is:
(a) 3925 (b) 4100 Which of the above statements is/ (a) 13 (b) 4
(c) 4175 (d) 8200 are correct? (c) 5 (d) 6
Practice Set-3 31

63. T h e a n g l e o f i n t e r s e c t i o n 2 2 (a) Stem


of the curves y = x2 and (b) x + y = 1 (b) Leaf
x = y2 at (1, 1) is: 9 3
(c) leaflets
–1  4  2 2 (d) Petiole
(a) tan
tan–1  3  (b) tan–1(1) (c) x + y = 1
  81 9 76. Which of the following is
–1  3  being touted as the ‘Whatsapp
(c) 90º (d) tan
tan–1  4  2 2
  (d) x + y = 1 moment’?
64. A a n d B a r e t w o e v e n t s 9 81 (a) Paytm
such that P(A) = 0.3 and (b) M-Pesa
70. If sin A = and cos B = ,
P (A ∪ B) = 0.8. If A and B are
where A and B are acute angles,
(c) United Payment Interface
independent, then P(B) is:
(a) 2/3 then what is the value of A + B? (UPI)
(b) 3/8 (a) 135º (b) 90º (d) New Featured Whatsapp
(c) 2/7 (c) 75º (d) 60º (NFW)
(d) None of these 77. Which country has signed
–1 REASONING AND GENERAL an agreement with Bhagwan
65. What is the value of ∫ tan 2x dx ?
1
0 1+ x
AWARENESS (RAGA) Mahaveer Viklang Sahayata
71. The proceeding regarding Samiti (BMVSS), Jaipur to
π2 π2 impeachment against the provide financial and technical
(a) (b)
8 32 President can be started in: assistance for research and
(a) The Lok Sabha only development in the area of
π π
(c) (d) (b) The Rajya Sabha only prosthetic limbs?
4 8
(c) Either of the Houses (a) Japan (b) China
66. If c2 + a2 – b2 = ac, then ∠B is equal
to: (d) The Vidhan Sabha only (c) Russia (d) South Korea
π π 72. With an increase in production, 78. Reduce 2530/1430 to lowest
(a)
2 (b)
3 the average fixed cost: terms.
π π (a) rises (a) 47/17 (b) 23/13
(c) (d) (b) falls (c) 47/19 (d) 29/17
6 4
67. If a = 16, b = 24, c = 20, then (c) remains constant 79. The H.C.F. of two numbers is 16.
 B (d) fluctuates Which one of the following can
cos   is equal to:
 2 73. Under which of the following never be their L.C.M?
3 1 Delhi Sultans did Malik Kafur (a) 32 (b) 80
(a) (b)
4 4 serve as a military general? (c) 46 (d) 60
1 1 (a) Alauddin Khilji 80. If 2x + y = 17; y + 2z = 15 and x +
(c) (d)
2 3 (b) Jalaluddin Khilji y = 9, then what is the value of 2x
68. If sin–1 x + cot–1 (1/2) = π/2, then (c) Muhammad bin Tughlaq + 3y + 4z?
what is the value of x? (d) Aurangzeb (a) 61 (b) 47
74. ‘Green Revolution’ in Indian (c) 45 (d) 55
(a) 0 (b) 1/ 5 81. The mean of marks secured by
agriculture refers to:
(c) 2/ 5 (d) 3/2 45 students in division A of class
(a) scientific methods for
X is 69, 65 students of division B
69. A rod of length 12 cm moves improved irrigation
is 65 and that of 50 students of
with its ends always touching (b) intensive afforestation division C is 63. Find the mean
the positive coordinate axes. (c) increase in agricultural of marks of the students of three
Determine the equation of the production due to scientific divisions of Class X.
locus of a point P on the rod,
methods (a) 65.5 (b) 64.8
which is 3 cm from the end in
contact with the x-axis (d) improved techniques in (c) 64.1 (d) 66.9
cultivation of fodder crops 82. The average revenues of 9
2 2
(a) x – y = 1 75. In pea, the tendril is a consecutive years of a company
81 9 modification of: is ` 65 lakhs. If the average of
32 Practice Set-3

first 5 years is ` 60 lakhs and that (a) North (b) South 92. x – y + z implies
of last 5 years is ` 72 lakhs, find (c) North-west (d) North-east (a) x + y + z (b) x f y – z
the revenue for the 5th year. 89. Rahul, Prem, Shyam, Geet and (c) x – y f z (d) x D y = z
(a) ` 77 lakhs (b) ` 75 lakhs Rinku are five friends among
Direction (93): Find the odd word/
(c) ` 73 lakhs (d) ` 71 lakhs which Shyam is the tallest. Rahul letters/number from the given
83. If 2 pairs of trouser is offered is shorter than Prem but taller alternatives.
free on purchase of 3 pairs of than Rinku. Geet is little shorter 93. (a) SMS (b) Speed post
trousers priced ` 900 each what than Prem but little taller than (c) Letter (d) Money order
is the effective discount on each Rahul. Who is taller than Rinku
pair of trouser? Direction (94): A series is given with
but shorter than Geet? one term missing. Choose the correct
(a) 40 percent
(a) Rahul (b) Rinku alternative from the given ones that
(b) 33.33 percent
(c) 30 percent (c) Prem (d) Geet will complete the series.
(d) 20 percent Direction (90): A series is given with 94. 6, 12, 20, 30, ?
84. If (x + y) : (x – y) = 11 : 1, find one term missing. Choose the correct (a) 40 (b) 41
value of (5x + 3y)/(x – 2y). alternative from the given ones that
(c) 42 (d) 48
will complete the series.
(a) 45/4 (b) 4/45 Direction (95): Study the following
(c) – 45/4 (d) – 4/45 90. FU, GT, HS, IR, ?
information carefully and answer the
85. A man travels 800 km in partly (a) JQ (b) KO
questions given below :
by rail and partly by steamer. (c) MN (d) EF
There are five persons A, B, C, D and
He spends 7 hours more time Direction (91): Study the following E, each having different height. E is
on steamer. If the velocity of information carefully and answer the
shorter than only two persons. B is
the steamer is 20 km/hr and the questions given below:
not the tallest. C is taller than A. C is
velocity of rail is 55 km/hr, how A, B, C, D, E, F, G and H shorter than B.
much distance does he cover by are sitting around a circle facing 95. Who among the following is the
steamer? the centre. B is third to the right tallest?
(a) 316 km (b) 379 km of F and third to the left of H. C is (a) C
(c) 474 km (d) 569 km fourth to the left of A, who is not an (b) D
86. Rajendra is the brother of immediate neighbour of F or B. E is (c) E
Jitendra. Geeta is the sister of not an immediate neighbour of B. G (d) Cannot be determined
Harendra. Jitendra is the son of is second to the right of D. Directions (96-100): In the following
Geeta. How is Rajendra related 91. Which of the following pairs question, two statements are given
to Geeta? represent the immediate neighbours each followed by two Conclusions
(a) Brother (b) Nephew of F? I and II. You have to consider the
(c) Father (d) Son (a) CH (b) ED statements to be true even if they
87. Karan travels 20 km towards the (c) HD (d) CE seem to be at variance from commonly
south, and then takes a left, and Direction (92): In the questions known facts. You have to decide which
travels 35 km further. Again, he below, some relationship have been of the given Conclusions, if any, follows
takes a left and travels 20 km expressed through symbols as shown from the given statements.
further. How far is he from his below.
96. Statements:
original position? f means less than (I) Air purifier is the latest
(a) 35 km (b) 25 km D means not greater than innovation of this decade.
(c) 30 km (d) 40 km – means equal to (II) Marketers identified the need
88. I am facing north. I turn left and + means not equal to and grabbed the opportunity
× means not less than of polluted atmosphere and
walk 30 m. Then I turn right
created a new product for
and walk 20 m, then I turn left = means greater than people.
and walk 10 m and then turning Study the meaning of these Conclusions:
right and walk 50 m. Then I turn symbols and pick correct answer (I) The reason for invention of
left and walk 50 m. In which from the answer choices for each air purifier is rising levels of
direction am I from the starting pollution.
questions below:
point?
Practice Set-3 33

(II) The need is created in market (II) Happiness is a state of mind. If we (a) Only Conclusion II follows
about the product and then it can't find it, we should create it. (b) Conclusion I and II both follow
is established. Conclusions:
(a) Only Conclusion II follows (I) Creation of happiness itself (c) Neither I nor II follow
(b) Conclusion I and II both follow makes an individual satisfied (d) Only Conclusion I follows
(c) Neither I nor II follow and happy because creativity (e) Data insufficient
(d) Only Conclusion I follows is constructive.
100. Statements:
(II) It's all about analysing the
97. Statements: (I) In the GST Model in India,
situational factors to understand
(I) Flowers give everything to
the world before shedding off happiness. it would be dual tax (GST)
from the plant. (a) Only Conclusion II follows with the center and states
(II) Human beings should inherit (b) Conclusion I and II both follow simultaneously levying tax on
these good qualities from (c) Neither I nor II follow a common base tax.
flowers. (d) Only Conclusion I follows
(II) The central and state GST
Conclusions: 99. Statements:
(I) Petrol prices are increasing in would be levied on every
(I) Imparting education and good
learnings to the world is a the economy. transaction of supply of goods
noble deed. (II) The price of crude oil in and services simultaneously.
(II) Flowers teach us to live a international market has not Conclusions:
meaningful and significant life. shown reasonable increase.
Conclusions: (I) This new tax regime would
(a) Only Conclusion II follows benefit common man of country.
(b) Conclusion I and II both follow (I) The inflation has hit various
commodities like petrol in the (II) There would be more clarity on
(c) Neither I nor II follow state and central component.
country.
(d) Only Conclusion I follows
(II) Demand for petrol is increasing (a) Only Conclusion II follows
98. Statements: but supply has not picked up (b) Conclusion I and II both follow
(I) Happiness is around everyone, with the same rate, hence (c) Neither I nor II follow
we have to find it.
resulting into hike in prices. (d) Only Conclusion I follows

Answers with Explanation


ENGLISH Look at the sentence: PHYSICS
He played a prominent part in the
1. (a) 2. (b) 3. (c) 4. (a) 21. (c) The electromagnetic wave
campaign.
5. (b) 6. (b) Its antonym should be unknown. being packets of energy moving with
7. (c) The word Relish (Verb) 10. (d) 11. (d) 12. (d) speed of light may pass through the
means: enjoy; to get great pleasure from 13. (c) 14. (a) region.
something. 15. (b) Here, bring about any change
Look at the sentence: TL
............ should be used. Phrase bring 22. (c) Y=
Nobody relishes cleaning the oven. πr 2 × l
about means: to make something happen;
8. (d) The word Exemption (Noun) cause.
TL
means: official premission not to do Look at the sentence: = 2 (T → tension applied)
something or pay something that you What brought about the change in     πr
have to do or pay his attitude? (l = 1 cm)
16. (c) Here, to such simple matters   
The word Inclusion (Noun)
means; the fact of including somebody/ .... should be used In second case
something. 17. (a) 18. (c) TL
19. (b) The captain ordered the army Y=
9 . ( a ) T h e w o r d P ro m i n e n t  r
2

(Adjective) means: distinguished; to march forward then. π  × l '


important or well known; noticeable. 20. (a) We are reading it.  2 
34 Practice Set-3

4TL gR →
 → →
l' = 2 u2 = F = q  v × B
πr Y sin(2θ) 33. (a)  

4TL × πr 2 u2 is minimum, when sin (2θ)


= q [(2iˆ) × (iˆ + 2 ˆj + 3kˆ)]
= = 4 cm = max = 1
πr 2 × TL
θ = 45° = (4q ) kˆ − (6q ) ˆj
23. (a) 4
ξ = (0.5 × 10 ) (10) (200) and then u2 = gR 34. (c) In this case the two rods are in
ξ = 0.1 V u = 1000 ms–1 {Take g = 10 parallel, therefore, the equivalent thermal

ms–2} resistance is found by:
24. (c) R φ = R 90 − φ
1 1 1
10 – 5 = +
= 4 hφ h90 − φ
29. (a) I= = 1A (∆x / KA) (∆x / K1A) (∆x / K 2 A)
2 + 2 +1
R = 4 hh 30. (c) vs = 72 km/hr = 20 m/s KA KA K A
1 2 ⇒ = 1 + 2
v = 332 m/s ∆x ∆x ∆x
25. (c) Force per unit area
v0 = 0 ⇒ K = K1 + K 2
σ2
= = mg 35. (d) Torque:
2ε 0 n' = 260

From the relation τ = T× R = Iα
σ = 2ε 0 mg = 1.88 × 10−5 C / m 2
v 1  ω
26. (d) The magnetic field at P(a, 0, n' = n T× R =  MR 2  ×
v − vs 2  t
a) due to the loop is equal to the vector
sum of the magnetic fields produced by 332 v
260 = × n Now, ω =
loops ABCDA and AFEBA as shown in 332 − 20 R
the figure:    n = 244.3 Hz
R
C 31. (a) Condition for equilibrium:
P(a,0, a)
kQ 2 kqQ
+ =0 T
D B k
d2 (d / 2) 2 T
E
j kQ
(Q + 4q ) = 0
d2
i
A F Q
q= −
4 1 v
Magnetic field due to loop ABCDA will ∴ T× R = MR ×
2 Rt
be along and due to loop AFEBA. (vrms ) He (T) He (M) He
32. (a) = 2Tt 0
Along magnitude of magnetic field (vrms ) H (T) H (M) He ⇒ v= R
M
due to both the loops will be equal.
5 (T) He × 2 Thus,
Therefore, direction of resultant =
7 (273 + 0) × 4  2Tt 
1 ˆ Thus, v ∝ R 0 ∵ is constant 
magnetic field at P will be (i + kˆ) .  M 
2  5 (T) He
2

27. (c) In adiabatic process Q = 0   = Hence, speed is independent of R.


7 2 × 273

   dW = − dU = − (− 100) 1 −2
1 g  1
= 100 J (T) He =
5 × 5 × 2 × 273 36. (a) n1 = 70    
 
28. (d) R = 100 km = 105 7×7  kg  1 

m = 278.57 K [surface tension = MT –2 ]
2
u sin (2θ) = 278.57 – 273
Since R = 70 = 7 × 10−2 N/m
g ∴ n1 =
= 5.57°C 100
Practice Set-3 35

ω2
37. (c) 6 − V0 + 3 − V0 + 2 − V0 = 0 ω1 a+c
48. (a)  b =
6 3 2 2
1 1 1 6 3 3 r2 r
V0  + +  = + + 1 For read roats b2 – 4ac > 0
6 3 2 3 5 2 2
a+c
V0 = 3 V ∴      – 4ac > 0
 2 
38. (a) For the car approaching the ⇒    a2 + c2 – 14 ac > 0
observer, apparent frequency is higher 2
by 2.5%, Therefore, ⇒  c  – 2 × 7 × c + 49 – 49 + 1 > 0
 a a
102.5
n' = n 2
100
⇒  c – 7  > 48
since, v = 320 m/s 2 a 
From relation ω1 r 
⇒ = 1 c
ω2  r2  ⇒ –7 >4 3
v a
n' = n 43. (a) Voltage gain = ∆ICRL
v − vs 49. (c) Given that Tn = 3n + 7
= 2 × 10–3 × 10 × 103

102.5 320 S, = ΣTn = Σ(3n + 7) = 3Σn + 7Σ1
n= n

= 20 V
100 320 − vs 44. (c) Only for real image, the plane 3n ( n + 1)
= + 7n
mirror forms virtual image. When one 2
102.5 × 320 –102.5 vs = 320 × 100 surface of lens is silvered then silvered  3n + 3 + 14   3n + 17 
surface behaves as mirror. So, the final = n  = n
320 × 2.5  2  2 
∴ vs = = 7.8 m/s image may coincide with the object. Now, sum of 50 terms.
102.5
45. (b) It will increase.
 3 × 50 + 17 
I S50= 50  
39. (b) T = 2π MATHEMATICS  2
MB
46. (b) Here, foci = (± 1, 0) and
167 
⇒ [T]=[ML2 ]1/ 2 [L2 A]−1/ 2 [MA −1 T −2 ]−1/ 2 1 = 50  = 25 × 167 = 4175
eccentricity, e =  2 
2
40. (c) Since, all A,B,C,D and E lie 1 50. (d) 1215
∴ ae = 1 end e = ⇒ a = 2
on an equipotential surface so, W = 0 2 51. (b) Let us take the function
 1
∴ b2 = a2(1 – e2), ⇒ b2 = 4 1 –  f(x) = x| x |
41. (c) When p-end of p-n junction  4
3 Redefine this function f(x) =
is connected to positive terminal of = 4× = 3
4
battery and n-end to negative terminal So, the equation of required ellipse in  x 2 , if x ≥ 0

of battery, then p-n junction is said to 
x2 y 2  − x 2 , if x < 0
be in forward bias. In forward bias, + =1
4 3 .
the more numbers of electrons go from f(x) is a polynomial function, so it is
n-region to p region and more number 47. (d) Z1Z2Z3 ...... ∞ = [cos (π/2) + i
differentiable for every value of x except
of holes go from p-region to n-region. sin (π/2)]
at x = 0.
Therefore, major current due to both [cos (π/22) + i sin (π/22)] [cos (π/23) + i
Now, we have to check the differentiability
types of carriers takes place through the sin (π/23)]....
at x = 0.
junction causing a reduction in height of = cos {(x/2 + π/22 + π/23+.......)} + i sin
f ( 0 − h ) − f ( 0)
depletion region and barrier potential is Lf ' (0) = lim
 π π   h →0 h
also reduced.  + 2  + ........
 2 2  
− ( − h) − 0
2
42. (c) Angular momentum: −h2
   = lim = lim
L = I1 ω1 = I 2 ω2 π/ 2  π/ 2  h →0 −h h →0 − h
= cos + i sin  
1 – 1/ 2  1 – 1/ 2  ( + h) = 0
= hlim
2 2
mr1 ω1 = mr2 ω2    →0
= cos π + i sin π = 1
36 Practice Set-3

On solving Eqs. (i) and (ii), we get ∴ Put n = 1


f ( 0 + h ) − f ( 0)
Rf ' (0) = lim x = – 2 and y = 3 P(1) = (1 + 2) (1 + 3) (1 + 4) (1 + 5) (1 + 6)
h →0 h
It these lines are concurrent, then there = 3 × 4 × 5 × 6 × 7 = 120 × 21
2
h −0 values must satisfy the third equation Put n = 2,
= lim = lim h = 0
h →0 h h →0 15 – 2b = 3 ⇒ 2b = 12 ⇒ b = 6 P(2)= (2 + 2) (2 + 3) (2 + 4) (2 + 5) ( 2 + 6)
y 5x x = 4 × 5 × 6 × 7 × 8 = 120 × 56
55. (a) e + e = e2x + e–2x
e3 x Hence,P(n) is always divisible by 120
59. (b) The equation of circle is
 ( 2 x )2 ( 2 x )4 
= 2 1 + + + ...... (x – a)2 + y2 = a2
 2! 4!  ⇒ x2 + a2 – 2ax + y2 = a2
x' x
⇒ x2 + y2 – 2ax = 0 ...(i)
f(x) = x|x| 24 4
Coefficient of x4 = 2. = x2 + y 2
4! 3 ∴ a=
56. (d) The equation of first circle is 2x
On differentiating Eq. (i) with respect
x2 + y2 – 2x – 2y = 0
y' to x, we get
∴ Lf ' (0) = Rf '(0)
Radius of this circle = (1)2 + (1)2 dy
2x + 2 y
– 2a = 0
( )
So, f(x) is differentiable for all real
dx
= 2 by g 2 + f 2 – c
values of x. dy x 2 + y 2
⇒ 2 x + 2 y − =0
52. (a) fx = loga(logax) and equation of second circle is x2 + y2 = 1 dx x
Radius of this circle = 1 dy
 l og e x  ⇒ 2 x 2 + 2 xy − x2 − y 2 = 0
= log a  From above it is clear that the radius of dx
 log e a  first circle is not twice that of second dy
= loga (logex) – loga (logea) circle. ⇒ 2 xy = y2 – x2
dx
Also, first circle passes through the origin
log e (log e x ) 60. (c) Given. xm + ym = 1
⇒ f(x) = − log a (log e a ) while second circle does not pass through
log e a the origin. On differentiating both sides w.r.t. x,
Hence,neither l nor ll statement is correct. we get
1  1 1 dy
⇒ f '(x) =  .  57. (c) Given that    mxm–1 + mym–1 =0
log e a  log e x x  dx
log10 2, log10 (2x – 1) and log10 (2x + 3)
⇒ f '(x) = logae / xlogex are in AP dy mx m –1 − x m –1  xm   y 
⇒ = = m –1 =  m   
log x Then, dx my m –1 y  y  x 
53. (c) We have, ∫ (1 + log x )2 dx 2 log10 (2x – 1)= log10 2 + log10 (2x + 3) m
−x  x  y 
⇒ log10 (2x – 1)2 = log10 (2x + 1 6) Given, = –  m  
1 y  y  x 
Put log x = t ⇒ dx = dt  
x ⇒ ( 2x – 1)2 = (2x + 1 + 6) m –2
tet dt ⇒ 22x + 1 – 2·2x = 2·2x + 6
x
=1
∴ I = ∫ ∴ 
(1 + t )2 ⇒ 22x – 4·2x – 5 = 0  y
et et ⇒ (2 ) – 4(2x) – 5 = 0
x 2 which is true when m = 2
= ∫ dt – ∫ dt
61. (a)
1+ t (1 + t )2 ⇒ y2 – 4y = 0 (Let y = 2x)...(i)
et 1 et ⇒ y – 5y + y – 5 = 0
2
Class f cf
= – ∫ – et dt – ∫ (1 + t )2 dt ⇒ (y – 5) (y + 1) = 0 ⇒ y = – 1.5 0-10 22 22
1+ t (1 + t )2 10-20 38 60
⇒ 2x = – 1.5 [from eq. (i)] 20-30 46 106
x
= +C ∴ x = log2 (– 1), log2 (5) 30-40 35 141
1 + log x
So, x = log2 (5) 40-50 20 161
54. (c) The equation of given lines are
but x ≠ log2 (– 1) N 161
3y + 4x = 1 ....(i) Here, = = 80.5
y = x + 5 ....(ii) 58. (b) Let P(n) = (n + 2) (n + 3) (n + 2 2
and 5y + bx = 3 ....(iii) 4) (n + 5) (n + 6) Median class is 20–30.
Practice Set-3 37

N  dx π
−C Put tan–1 x = t, then = dt =
Median = I +  2  1+ x 2 2
 f h
  When x = 0 then t = 0 2
π

 80.5 − 60 

When x = 1 then t =
π ⇒ x1−  2  + 2
 
5 5
(1 − x ) = sin 2
2

= 20 +  × 10 = 24.46 4
 46 
π /4 2 2
62. (c) Out of 10 points, let n points
be collinear. Then, number of triangles ∴ ∫0
π /4 t2 
t dt =  
1π
=   =
2 4 
π2
32
⇒ x1−
 2 
  +
5
2
5
(1 − x ) = 1
2

 2 ,
are 10C3 – nC3 = 110 On squaring both sides, we get
66. (b) By cosine rule,
10 × 9 × 8 n ( n − 1) ( n − 2)

3×2

3×2
= 110
c2 + a 2 – b2
con B = ⇒ cos B =
ac x2
1 4
5 5
(
+ 1 − x2 +
2⋅ x 2
5

5
)
1 − x2 = 1
⇒ 720 – 660 2ac 2ac
= n(n – 1) (n – 2) ( c + a2 – b2 = ac) x2 4 4 x2 4 x
⇒ + − + 1 − x2 = 1
⇒ n(n – 1) (n – 2) = 60 1 π 5 5 5 5
⇒ n(n – 1) (n – 2) = 5 × 4 × 3 ⇒ cos B = ⇒ cos B = cos
2 3 2
∴ n=5 ⇒ −3 x + 4 x 1 − x 2 = 1 – =
π 5 5
63. (d) The slope of the curve y = x2 ∴ B=
3
dy ⇒ − 3x 2 + 4 x 1 − x 2 = 1
⇒ = 2x 67. (a) Here, a = 16, b = 24, c = 20
dx By inspection method, we put the value
a + b + c 16 + 24 + 20 1
 dy  s= = = 30 of x from option, x = ,
  = 2 = m1 and
⇒ 2 2 5
 dx (1,1)
x = y2
s ( s − b)  1  1
2
 1 
2
∴ cos B/2 = ⇒ − 3  + 4 ⋅ 1−  
dy dy 1 ac  5 5  5
⇒ 1 = 2 y ⇒ =
dx dx 2y
30 (30 − 24) 30 × 6
 dy  = = 3 4 2 −3 8
1 16 × 20 16 × 20 = + ⋅ = + =1
⇒   = = m2 5 5 5 5
 dx (1,1) 2
∴ Angle of intersection at the point of 9 3 69. (c) Let AB be the rod of length 12
= = cm.
(1, 1) 16 4
1 Let OA = a and OB = b
m1 − m2 2− 1 π Using by Pythagoras theorem in ∆OAB.
2 3 68. (b) sin–1 x + cot –1   =
tan θ = = = 2
  2 (Base)2 + (Perpendicular)2 = (Hypotenuse)2
1 + m1m2 1 4
1+ 2 ×
2   y
−1  3   
⇒ θ = tan    1 1 B
 4 cot −1   = sin −1  
 2  2
64. (c) P(A ∪ B) = 0.8  1 +  1 
⇒ P(A) + P(B) – P(A ∩ B) = 0.8
  2   12
P
⇒ P(A) + 1 – x – P(A) (1 – x) = 0.8 b
Where x = P(B) −1 1
= sin = sin–1
⇒ P(A) (1 – 1 + x) + 1 – x = 0.8 5 A
a x
⇒ xP(A) + 1 – x = 0.8 4 O A
⇒ (0.3) x – x = 0.8 – 1
⇒ 0.7x = 0.2 Now, sin– 1 + sin–1 = a2 + b2 = (12)2
⇒ a2 + b2 = 144
x=
⇒ ⇒ P(B) =
 2  ⇒ AP = 3 cm
1 tan
−1
x 
 2 
⇒ sin −1  x 1 −   +
 5
2
1 − x2 

( ) ⇒PB = AB – AP = 12 – 3 = 9 cm
65. (b) Let I = ∫0 1 − x 2 dx 
5
 ⇒AP.BP = 3 : 9 = 1: 3
38 Practice Set-3

Let P = (h, k) divides the the points A 80. (b) 2x + y = 17 ...(i)
REASONING AND GENERAL y + 2z = 15 ...(ii)
and B internally in the ratio m: n then
AWARENESS (RAGA) x + y = 9 ...(iii)
1 3
71. (c) As per Article 61, President Solving equations (i), (ii) and
A P B
of India can be impeached on ground of (iii) we get : x = 8 y = 1 and z = 7.
(a, 0) (n, k) (0, b)
violation of the Constitution. However, \ 2x + 3y + 4z = 2 × 8 + 3 × 1 + 4
mx2 + nx1 my2 + ny1 what amounts to violation of the
h= ,k= × 7 = 47
m+n m+n Constitution has not been defined. The 81. (a) Mean marks of 3 divisions
process of impeachment can begin in of class X
1× 0 + 3 × a 3a 4h
h= ⇒h= ⇒a= any of Lok Sabha or Rajya Sabha. The
1+ 3 4 3 45 × 69 + 65 × 65 + 50 × 63
charges for impeachement should be =
signed by 1/4 members of the house in 160
1+ b + 3× 0 b which the process begins and a notice of 3105 + 4225 + 3150
k= = = b = 4k =
1+ 3 4 14 days should be given to the President. 160
2 72. (b) 10480
 4h  = = 65.5
From Eq. (i),   + (4k)2 = 144 73. (a) Malik Kafur was an eunuch 160
 3  slave who became a general in the army
82. (b) Total revenues of 9 years
of Alauddin Khilji, ruler of the Delhi
⇒ = 144 = 65 × 9 = 585 lakhs
Sultanate from 1296 to 1316 A.D.
74. (c) The green revolution Total revenues of first 5 years
h2 h2 k 2 consists of technological improvements = 60 × 5 = 300 lakhs
+ k 2 − 9 = 81 + 9 = 1
9 which were mainly adopted to increase Total revenues of last 5 years
agriculture productivity. The green = 72 × 5 = 360 lakhs
x2 y 2
Hence, locus of point (h, k) is + revolution occurs as a result of adoption
81 9 Total revenues of 10 years
=1 of new agriculture strategy during mid
= 300 + 360 = 660 lakhs
70. (a) Given that, A and B acute 60's by Government of India to achieve
self-sufficiency in the foodgrains Revenues of 5th year
angles.
2 production. = 660 – 585
i.e., A < 90º and B < 90º and sin A = , 75. (b) The leaves of a pea plant are
5 = ` 75 lakhs
2 modified into tendrils that help the plant
cos B = , 83. (a) 3 pairs of trousers + 2 pair +
in climbing.
10 trousers = ` 900
We know that sin2 θ + cos2 θ = 1 76. (c)
5 pairs of trousers = 900
77. (d) South Korea Explanation:
∴ cos A = 1 - sin 2 A 900
On June 24, 2019, South Korean 1 pair of trousers = = `
5
4 Ambassador to India Bong-kil Shin and 180
= 1- =
5 Bhagwan Mahaveer Viklang Sahayata 5 pair of trousers = 300 × 5
3 Samiti (BMVSS) signed agreement to = ` 1500
1
sin B = 1 - cos2 B = 1- = provide assistance in prosthetic limbs Effective discount %
10 10
sin (A + B) sector. Korea will grant financial and  1500 – 900 
=  × 100%
= sin A · cos B + cos A · sin B technical assistance for research and  1500 
2 1 1 3 development collaboration in the med- 600
= ⋅ + ⋅ = × 100 = 40 percent
5 10 5 10 tech sector in the area of prosthetic 1500
limbs, bionic arms, 3-D printing-based
x + y 11
= = Flat foot solution and mobility solutions. 84. (c) =
x– y 1
2530 253 ÷ 11 23 By using componendo and
78. (b) = =
= = 1430 143 ÷ 11 13 dividendo
79. (d) H.C.F. of two numbers 2x 12 6
= =
= sin 135º ∴ A + B = 135º divides their L.C.M. exactly. 2 y 10 5
Practice Set-3 39

x The Blood relationship tree/chart 91. (d) By analysing the circular


= 6/5 can be constructed as given below: arrangement, it is clear that immediate
y
neighbours of ‘F’  C & E
 x 
5x + 3 y  5 y + 3  92. (c) According to the meaning of
= the given symbols,
x – 2y x 
 y – 2  x–y+zx=y≠z

 6  Therefore, ‘Rajendra’ is the son of Using the proper symbols in option


 5 × + 3  ‘Geeta’.
5 (c), we get x – y f z  x = y < z  x
=
6 87. (a) = y ≠ z.
–2
5
start end N Therefore, x – y + z  x – y f z.
360 + 15 45
= = = – 45/4 W E 93. (a) SMS (Short Messaging
6 –10 –4 20 km 20 km
Service) is sent through a mobile phone,
85. (a) Let the man travel by rail be 35 km S while all others are sent through post
t hours Finally, Karan is 35 km far from his office.
and by steamer (t + 7) hours originzal position.
94. (c)
According to question 88. (c)
t × 55 + (t + 7) × 20
6 12 20 30 42
= 800 km
55t + 20t = 800 – 140 +6 +8 +10 +12
75t = 660
660 +2 +2 +2
t=
75 Finally, I am in North-west 95. (b) Clearly, D is the tallest.
Distance covered by steamer direction from the starting point. 96. (d) According to statements,
= ( t + 7 ) × 20 89. (a) According to question the only Conclusion I follows.
 660  sequence is, 97. (b) According to statements,
=  + 7  × 20
 75  Shyam→Prem→Geet→Rahul→ Conclusion I and II both follow.
 660 + 525  Rinku. 98. (c) According to statements,
=   × 20 neither Conclusion I nor Conclusion II
 75  Finally, Rahul is taller than Rinku
1185 follow.
= × 20 but Shorter than Geet.
99. (d) According to statements,
75 90. (a)
only Conclusion I follows.
1185 × 4
= 100. (c) According to statements,
15
neither Conclusion I nor Conclusion II
= 79 × 4 = 316 km follow.
86. (d) Rajendra  brother of qqq
Jitendra. Finally, the missing term is JQ.
40 Practice Set-4

Air Force Airmen Group X & Y Exam


Practice Set-4

We enjoy every right in theory, but Directions (4–5): In the following


ENGLISH questions, groups of four words are
not in practice. Real democracy will come
Directions (1-3): Read the following into being only when the masses are given. In each group one word is
passage and answer the questions that awakened and take part in the economic correctly spelt. Find the correctly
follow. spelt word.
and political life of the country. There
Democracy is a form of government 4. (a) superfluous (b) superflous
is inequality in every sphere social,
in which people are governed by their (c) superfluos (d) supperflous
economic and political. Illiteracy is the
own elected representatives. It is a main cause of inequality. The illiterate 5. (a) capracious (b) auspicious
government of the people, for the people. masses get easily lured by money during (c) fallicious (d) dalicious
In this system of government, it is the such an event. Also some of our legislators Direction (6): In the following question,
people who are supreme and sovereign. have criminal records against them. The four alternatives are given for the
They are free to elect a government of people who make the laws themselves Idiom/Phrase printed in underline in
their own choice. Freedom of choice is break them. For the democracy to be fully the sentence. Choose the alternative
the core of democracy. successful, the electorate should be literate which best expresses the meaning of
Democracy existed in ancient Greek and politically conscious. They should be the Idiom/Phrase.
and Roman republics but with little fully aware of their rights and privileges. 6. Once his crime was discovered, he
success. It had very little scope in ancient 1. What is the core of democracy? had no option but to come clean.
India. Democracy entered its golden
(a) Freedom of choice (a) to deny the crime
stage in the 20th century. Many countries
in the world today follow the democratic (b) Supremacy of people (b) to confess to the crime
form of government. Democracy depends (c) Government by elected (c) to accuse someone else of the
on the following conditions. (i) co- representatives crime
existence of ideas and of parties, (ii) the (d) to apologize for the crime
(d) Government of the people
right to free discussions, and (iii) periodic Directions (7–8): In following
elections. 2. What is wrong with the politics questions, choose the word opposite
India is the largest democracy in today? in meaning to the given word.
the world. The Constitution of India was (a) We have multi-party system 7. He did it PURPOSELY.
enforced on 26th January, 1950. India (b) Political parties are the vehicles (a) half-heartedly
became a democratic republic infused (b) timidly
of ideas
with the spirit of justice, liberty, equality (c) unintentionally
and fraternity. The Indian political system (c) I t h a s b e c o m e a g a m e o f
(d) hesitatingly
is a multi-party system. Political parties opportunism and corruption 8. GUILTY
are the vehicles of ideas. Parties act as (d) All of the above (a) innocent (b) ignorant
the bridge between social thought and (c) irreverent (d) immature
3. “The people who make the laws
political decision in democracy. However, Direction (9): In the following question,
themselves break them.” Who are
gradually politics has become a game out of four alternatives, choose one
of opportunism and corruption. Most ‘the people’ here?
which can he substituted for the given
political parties are only interested in (a) Masses words/sentence.
coming to power. Some try to influence (b) Illiterate masses 9. One who looks at the bright side
the people through caste politics. Some of things
(c) Literate masses
try to raise the religious sentiments of the (a) pessimist (b) optimist
people. (d) Legislators (c) introvert (d) extrovert
Practice Set-4 41

10. Choose the option that is opposite Direction (17): In the following (a) The police ordered the thief not
in meaning to the word italicized questions, out of the four alternatives, to move
in the following sentence: choose the one which best expresses the (b) The police ordered the thief that
The little girl looked absolutely meaning of the given word. he should not move
beautiful in her new dress and 17. AVERT (c) The police told the thief that he
shoes. (a) hide (b) obey did not move
(a) dirty (b) lovely (c) excuse (d) avoid (d) The police ordered to the thief to
Direction (18): In the following not move
(c) filthy (d) ugly
question, the first and the last
11. Choose the correct form of tense sentences/parts of the passage/sentence
for the given sentence:
PHYSICS
are numbered 1 and 6. The rest of the
Maya ........ of giving up her job. passage/sentence is split into four parts 21. A ship of mass 3×107 kg initially
and named P, Q, R and S. These four at rest is pulled by a force of 5 ×
(a) is thinking
parts are not given in their proper 105 N through a distance of 3 m.
(b) thinks order. Assume that the resistance due
(c) will have been thinking 18. Read the sentence/passage and find to water is negligible, the speed
(d) has thought out which of the four combinations of the:
Directions (12–14): In the following is correct. Then find the correct
answer. (a) 1.5 ms–1 (b) 60 ms–1
questions, some of the sentences have
1. With a chuckle she handed him (c) 0.1 ms–1 (d) 5 ms–1
error and some have none. Find out
her machine 22. The potential energy of system
which part of a sentence has an error.
P. she opened the lid increase, if work is done:
The number of that part is your
Q. and the receivers over her ears
answer. If there is No error, the answer (a) b y t h e s y s t e m a g a i n s t a
R. which he held, while
is (d). conservative force
S. and attached the wires
12. The number of applications has risen
6. Looking very similar to a (b) by the system against a non-
(a)/ this year by (b)/ as many as 50%.
wireless operator conservative force
(c)/ No error (d).
(a) RQPS (b) RPSQ (c) u p o n t h e s y s t e m b y a
13. I was driving under the bridge, (a)/
as a football (b)/ hit my window. (c)/ (c) RSQP (d) RQSP conservative force
No error (d). Direction (19): In the following
(d) upon the system by a non-
question, a sentence has been given
14. If I would know (a)/ what you in Active Voice/Passive Voice. Out of conservative force
wanted (b)/I would help you. (c)/ the four alternatives suggested, select 23. A man of mass m is standing on
No error (d). the one which best expresses the same a plank of equal mass m resting
Directions (15–16): In the following sentence in Passive/Active Voice. on a horizontal surface. The man
questions, sentences are given 19. The MD told us about the new starts moving on the plank with
with blanks to be filled in with an project.
appropriate word(s). Four alternatives speed u relative to the plank. The
(a) We were told about the new
are suggested for each question. speed of the man relative to the
project by the MD
Choose the correct alternative out of ground is:
(b) We are told about the new project
the four. by the MD
15. The firm has been dealing ______ (c) We told by MD about the new m
luxury goods for more than two project
decades.
(d) We had been told by MD about
(a) in (b) with project
(c) out (d) on 20. In the following question, a
16. She ______ in Delhi for two years sentence has been given in Direct/
but has not seen the India Gate. Indirect Speech. Out of the four
alternatives suggested select the u
(a) is lived (a) 2u (b)
one which best expresses the same 2
(b) had lived sentence in Indirect/Direct Speech.
(c) has been living The police said to the thief, “Don’t u
(c) zero (d)
(d) has lived move.” 4
42 Practice Set-4

24. Two water pipes of diameters 2 it into a voltmeter of range   x 


cm and 4 cm connected with the nV volt, will be: (a) y = 0.2 sin  2π  (6t + 
  60  
main supply line. The velocity of (a) (n – 1)G (b) G/n
flow of water in the pipe of 2 cm  x 
(c) nG (d) G/(n – 1) (b) y = 0.2 sin  π  (6t + 
diameter is:   60  
29. Two pure inductors each of
(a) 4 times that in the other pipe
inductance L are connected   x 
1 (c) y = 0.2 sin  2π  (6t – 
(b) times that in the other pipe in parallel but well separated   60 
4 from each other. The equivalent
(c) 2 times that in the other pipe   x 
inductance of the combination is: (d) y = 0.2 sin  π  (6t – 60  
1   
(d) times that in the other pipe (a) L/4 (b) L/2
2 35. Sensitivity of potentiometer can
(c) 2 L (d) 4 L
25. An ideal gas undergoes four be increased by:
30. An electric charge oscillating with
different processes from the same (a) increasing the emf of the cell
a frequency of 1 kilo cycle/s can
initial (figure). Four processes are (b) increasing the length of the
radiate electromagnetic wave of
adiabatic, isothermal, isobaric potentiometer wire
and isochoric. Out of 1, 2, 3 and wavelength:
(c) decreasing the length of the
4 which one is adiabatic: (a) 100 km (b) 200 km
potentiometer wire
(c) 300 km (d) 400 km
(d) None of the above
31. The volume of a given mass
4 36. The radius of the earth is 6.4 × 106
of a gas at 27°C, 1 atm is
p m. The specific charge of proton is
3 100 cc. What will be its volume
at 327°C? 9.6 × 107 Ckg–1. What should be
2
the value of minimum magnetic
1 (a) 50 cc (b) 100 cc
field at the equator of earth such
V (c) 150 cc (d) 200 cc
that a proton moving with velocity
(a) 4 (b) 3 32. Nickel shows ferromagnetic 1.92 × 107 ms–1 may revolve round
(c) 2 (d) 1 property at room temperature.
the earth?
26. Sound waves of wavelength λ If the temperature is increased
(a) 1.57 × 10–8 T (b) 3.12 × 10–8 T
travelling in a medium with a beyond Curie temperature, then
(c) 5.36 ×10–6 T (d) 7.83 × 10–6 T
speed of v m/s enter into another it will show:
37. What is necessary for easy
medium where its speed is 2 v m/s. (a) Paramagnetism
Wavelength of sound waves in the o c c u r re n c e o f F re s n e l ’s
(b) Anti-ferromagnetism
second medium is: diffraction?
(c) No magnetic property
λ (a) Obstacle should of the order of
(a) λ (b) (d) Diamagnetism wavelength
2
33. Escape velocity on Earth is 11.2 (b) Narrow opening should be of
(c) 2 λ (d) 4 λ
kms–1. What would be the escape the order of wavelength
27. In figure values of Ix and Iy are
velocity on a planet whose mass is
respectively: (c) Source and screen should be at
1000 times and radius is 10 times
0.3 A finite distance from the obstacle
0.7 A A B 0.4 A 0.4 A that of earth?
lx 2.2 A C ly (d) All of the above
1.0 A (a) 112 kms–1 (b) 11.2 kms–1
38. Taking the Bohr radius as a0 =
(a) 1 A, 1 A (b) 1.2 A, 1.2 A (c) 1.2 kms–1 (d) 3.7 kms–1
53 pm, the radius of Li++ ion in
(c) 0.8 A, 0.8 A (d) 1 A, 1.2 A 34. A wave travelling in positive its ground state, on the basis of
28. The range of a voltmeter of x-direction with A = 0.2 m, Bohr’s model, will be about:
resistance G Ω is V volt. The velocity = 360 ms–1 and λ = 60 m,
resistance required to be connected (a) 53 pm (b) 27 pm
then correct expression for the
in series with it in order to convert wave is: (c) 18 pm (d) 13 pm
Practice Set-4 43

39. In a transistor, the base is: (c) hole concentration in p-region 50. The sum of m terms is equatl to
(a) an insulator resistance is more as compared to n-region sum of r terms of any arithmetic
(b) a conductor of low resistance (d) All of the above progression, then sum of (m + r)
(c) a conductor of high resistance terms will be:
45. The physical quantity which does
(d) an extrinsic semiconductor not have the same dimensions as (a) 1 (b) 0
40. Advantages of optical fibre the other three is: (c) –1 (d) m r
communication over two wire (a) Spring constant 51. How many numbers of 9 digit may
transmission line or co-axial cable be formed from different digits?
(b) Surface tension
transmission are: (a) 8 × 8 ! (b) 8 × 9 !
(a) low bandwidth, low transmission (c) Surface energy
(c) 8 ! (d) 9 × 9 !
loss (d) Acceleration due to gravity
52. O n e o f the roots of
(b) h i g h b a n d w i d t h , h i g h x + 4 x − 4 80
MATHEMATICS − =
transmission loss x − 4 x + 4 9 is:
(c) h i g h b a n d w i d t h , l o w 46. Let A = {1, 2, 3,...9}, B = {2, 4, 6, 7,
(a) 5 (b) 4
transmission loss 8} and C = {3, 4, 5, 8, 9, 10}, then
(c) – 4 (d) 1
(d) l o w b a n d w i d t h , h i g h (A – B) ∪ C is:
53. If ω is the cube root of –1, then the
transmission loss (a) {1, 3, 4, 5, 8, 9, 10}
41. If α, β and γ rays carry same 1 −ω ω2
(b) {1, 2, 3, 4, 5, 6, 7, 8, 9} value of −ω ω2 1 is:
momentum which has the longest
(c) {2, 4, 6, 7, 8} ω2 1 −ω
wavelength?
(a) α-rays (d) {1, 3, 4, 5, 8, 9} (a) 1 (b) 0
2
(b) β-rays a + ib (c) 1 + ω (d) None of these
2 2 2
(c) γ-rays 47. If x + iy = c + id , then (x + y ) 54. The middle term in expansion of
(d) All have same wavelength is equal to: 10
 1
x− will be:
42. When a ball is thrown up a 2 − b2 a 2 + b2  x 
vertically with velocity v 0 , it (a) (b)
c2 + d 2 c2 + d 2 (a) – 252 (b) – 232
reaches a maximum height of h. If
2
a +b 2 2
a −b 2 (c) – 208 (d) – 224
one wishes to triple the maximum
(c) 2 2 (d) 55. The ratio of the radius of two
height, then the ball should be c −d c2 − d 2
thrown with velocity: circles at the centres of which two
48. For A = {x, y} which of the
arcs of the same length subtend
(a) 3 v0 (b) 3v0 following relation is not true? π π
(a) The void relation is φ angles 3 and is:
(c) 9v0 (d) 3/2 v0 4
43. An inductor of reactance 1 Ω and (b) The universal relation is (a) 4: 3 (b) 3: 4
a resistor of 2 Ω are connected {(x, x), (x, y) (y, y), (y, x)} (c) 3: 2 (d) 2: 3
in series to the terminals of a 6 {(x, x), (x, y) (y, x), (y, y)} 56. When simplified the trignometric
V (rms) AC source. The power 1 + sin 2 θ sec 2 θ
(c) The identity relation is {(x, x)
dissipated in the circuit is: expression
(y, y)} 1 + cos 2 θ cosec 2θ
(a) 8 W (b) 12 W
(c) 14.4 W (d) 18 W (d) The largest relation is {(x, x), reduces to:
44. In an unbiased p-n junction, (x, y) (y, x) (y, y)} (a) sin θ (b) cot θ
2
holes diffuse from the 49. Which term of series 18 – 12 + 8 (c) tan θ (d) tan θ
p-region to n-region because: –..............is 512 ? 57. Equation of the circle which
(a) free electrons in the n-region 729 touches the line x = 0,
attract them (a) 8 th
(b) 9th y = 0 and 3x + 4y = 4 is:
(b) they move across the junction (c) 7th (d) 10th (a) x2 – 4x + y2 + 4y + 4 = 0
by the potential difference (b) x2 – 4x + y2 – 4y + 4 = 0
44 Practice Set-4

(c) x2 + 4x + y2 + 4y + 4 = 0 (b) 2y – 2x + (b – a) log (x2 + y2 + 69. The mean of the following


(d) x2 + 4x + y2 – 4y + 4 = 0 2xy – ab) = C distribution is 50, but the
58. The perimeter of a right angled (c) 2y + 2x – (b – a) log (x2 + y2 + frequencies of 20–40 and 60–80 are
triangle is 70 cm and its in radius 2xy – ab) = 0 unknown. They are represented
is 6 cm. The sides of triangle in cm (c) 2y + 2x + (b + a) log (x2 + y2 – by f2 and f4 respectively.
will be: 2xy – ab) = 0
(a) 25, 27, 29 (b) 23, 25, 27 64. The value of sin (cot–1 x) will be Group Frequency
0 − 20 17
(c) 21, 29, 20 (d) 22, 23, 27 equal to:
20 − 40 f2
59. If the average daily wages for the (a) 1 + x2 (b) x
workers of two factories are as 40 − 60 32
1 60 − 80 f4
follows: (c) (1 + x2)–3/2 (d)
1 + x2 80 − 100 19
N u m b e r o f Factory A Factory B
Total 120
wage Earners 250 200 e n  1/ n
65. The value of lim n→∞   is: f2 and f4 are equal to:

Average daily ` 2.00 ` 2.50  π
wage (a) 18, 14 (b) 28, 24
1
then the average of daily wages (a) 1 (b) (c) 20, 24 (d) 22, 28
π
(in `) of the workers of the two
(c) e (d) ∞ 70. If the latus rectum of an ellipse is
factories combined is:
66. There are 4 one rupee coin, 3 fifty equal to half of its minor axis, its
(a) 2.25 (b) 2.24
paise coins placed at random in a ecentricity will be:
(c) 2.22 (d) 2.20
line, then the chance that extreme 1 1
60. The coordinates of the focus of the (a) (b)
places must be occupied by 50 2 4
parabola x2 = –8y are:
paise only is:
(a) (2, 0) (b) (0, 2) 3 3
(a) 5/42 (b) 1/35 (c) (d)
(c) (–2, 0) (d) (0, –2) 2 4
(c) 1/7 (d) None of these
2x
61. The derivative of tan –1
b log x REASONING AND GENERAL
1 − x2
1− x 2
67. ∫a x
dx is equal to:
AWARENESS (RAGA)
w.r.t. cos–1 2
is:
1+ x b 71. Who amongst the following was
(a) log   log (ab) the founder of Fascist Ideology?
1 a
(a) (b) 1
2 (a) Hilter
b
(c) 0 (d) 2 (b) 2 log   log (ab) (b) Mussolini
a
62. A line inclined at an angle of 135° (c) Napoleon Bonaparte
to the x-axis passes through A(3, 1 b (d) Bismarck
4), P and Q are two points on this (c) log   log (ab)
2 a 72. Which among the following
line such that AP = AQ = 2 2 .
(d) None of these States is created as the 29th State
Coordinates of P and Q are:
a+ x of India?
(a) (–1, –6) (5, 2) 68. ∫ a− x
dx is equal to:
(a) Jharkhand (b) Telangana
(b) (1, –6) (–5, 2)
x 1 2 (c) Chhattisgrah (d) Sikkim
(c) (1, 6) (5, –2) (a) sin–1 – a − x2
a a
(d) (1, 6) (5, 2) 73. The coldest place in the northern
–1 x a2 − x2 hemisphere is:
63. Solution of the different equation (b) a sin +
a
 x + y − a  dy x+ y+ a (a) Reykjavik
= is x
  x + y − b  dx x+ y+ b (c) a sin–1 – a2 − x2 (b) Alaska
2 2 a (c) Greenland
(a) 2y – 2x + (a – b) log (x + y +
2xy – ab) = C (d) None of these (d) Verkhoyansk
Practice Set-4 45

74. The rhinoceros and hippotamus 82. Rajesh sells a machine for ` 57 takes a turn towards south-east
are found in the forests of: lakhs at a loss. Had he sold it for and covers 5 km. Taking starting
(a) Assam ` 67 lakh, his gain would have point as the origin, in which
(b) Uttrakhand been 7 times the former loss. Find direction is he now?
the cost price of the machine. (a) North-east (b) South-east
(c) Karnataka
(a) ` 58.25 lakhs
(d) Madhya Pradesh (c) South (d) East
(b) ` 65.75 lakhs
75. Dodders is a/an: 89. Vineet is taller than Raman but
(c) ` 75.14 lakhs
(a) Stem parasite (d) ` 50.75 lakhs shorter than Jyoti. Sumit is the
(b) Root parasite 83. Marked price of an item is ` 900. shortest. Deepak is taller than
(c) Leaf parasite On purchase of 2 items discount Sumit but shorter than Raman.
is 6%, on purchase of 3 items Who is the tallest?
(d) Endophyte
76. Which Indian organisation first discount is 12%. Rajani buys (a) Jyoti (b) Raman
developed the Supercomputer? 5 items, what is the effective (c) Vineet (d) Deepak
(a) C-DAC (b) TCS discount?
Direction (90): A series is given with
(c) Infosys (d) Wipro (a) 17.5 percent (b) 9.6 percent one term missing. Choose the correct
77. Kalu police station was ranked (c) 35 percent (d) 37 percent alternative from the given ones that
as the India’s best police station, 84. The third proportional of two will complete the series.
it is located in which state? numbers 4 and 28 is....................
90. OP, RS, VW, ? , GH
(a) Dharwad district, Karnataka (a) 52 (b) 56
(a) ZA (b) AB
(b) Murshidabad district, West (c) 84 (d) 196
Bengal 85. To travel 672 km, an Express (c) CD (d) GE
(c) Bikaner district, Rajasthan train takes 14 hours more than Direction (91): Study the following
(d) Nicobar district, Andaman & Rajdhani. If however, the speed information carefully and answer the
Nicobar Islands of the Express train is doubled, it questions given below:
78. What should be the missing digit takes 8 hours less than Rajdhani. A, B, C, D, E, F, G and H are sitting
so that the number 347_547 The speed of Rajdhani is : around a circle facing the centre. B is
becomes exactly divisible by 11? (a) 30.5 km/hr third to the right of F and third to the
(a) 5 (b) 3 (b) 14.3 km/hr left of H. C is fourth to the left of A,
(c) 9 (d) 2 (c) 38.7 km/hr who is not an immediate neighbour
79. The L.C.M. of two numbers is of F or B. E is not an immediate
(d) 22.4 km/hr
495 and their H.C.F is 5. If the neighbour of B. G is second to the
sum of the numbers is 10, then 86. My mother's sister's husband's right of D.
their difference is: only son's sister is related to me 91. In which of the following pairs is
(a) 10 (b) 46 as the first person to the immediate
(c) 70 (d) 90 (a) Niece right of the second person?
y 15 (b) Sister
80. If 3x – 2y = 5 and = , (a) BG (b) GA
x + y 17 (c) Uncle (c) CF (d) HE
then what is the value of x – y? (d) None of the above Direction (92): In the questions
39 49 87. In a certain language, BUTTER below, some relationship have been
(a) − (b)
2 2 is coded as CVUUFS, BREAD expressed through symbols as shown
(c) 9 (d) 2 is coded as CSFBE, then how below.
81. The average revenues of 7 COFFEE is coded? f means less than
consecutive years of a company (a) DPGGFF (b) GGDPFF D means not greater than
is ` 83 lakhs. If the average of (c) GDPGFF (d) FFDPGG
first 4 years is ` 78 lakhs and that – means equal to
88. A man travels 10 km towards
of last 4 years is ` 90 lakhs, find + means not equal to
west, then he takes a turn
the revenue for the 4th year. × means not less than
towards north-east and travels
(a) ` 91 lakhs (b) ` 93 lakhs
20 km in the direction. He again = means greater than
(c) ` 89 lakhs (d) ` 87 lakhs
46 Practice Set-4

Study the meaning of these (a) E (b) B Conclusions:


symbols and pick correct answer (c) D (d) C (I) If children don't play with toys
from the answer choices for each Directions (96–100): ln the following some part of the personality
questions below: question, two statements are given is underdeveloped.
each followed by two Conclusions (II) Toys are mandatory and
92. x – y = z implies undetachable part of their
l and ll. You have to consider the
(a) x + y – z (b) x D y × z personality.
statements to be true even if they seem
(c) x + y f z (d) x + y × z to be at variance from commonly (a) Only Conclusion II follows
Direction (93): In the following known facts. You have to decide (b) Conclusion I and II both
which of the given Conclusions, if any, follow
questions, find out the odd word/
follows from the given statements. (c) Neither I nor II follows
letter/number from the given
alternatives. 96. Statements: (d) Only Conclusion I follows
(I) Empty set is a subset of any 99. Statements:
93. (a) Volt (b) Watt
set. (I) All shawls are carpets.
(c) Ampere (d) Kilogram (II) A set is a subset of power set. (II) No carpet is a pullover.
Direction (94): A series is given with Conclusions: Conclusions:
one term missing. Choose the correct (I) Empty set is a power set. (I) No shawls are pullovers.
(II) A set is a subset of power set. (II) All carpets are shawls.
alternative from the given ones that
(a) Conclusion l follows (a) Conclusion I follows
will complete the series.
(b) Conclusion ll follows (b) Conclusion II follows
94. Question Figures: (c) Neither l nor ll follows (c) Neither I nor II follows
(d) Both l and ll follows (d) Both I and II follows
? 97. Statements:
100. Statements:
(I) Some books are toys.
(I) Women generally prefer
Answer Figures : (II) No toy is red. window shopping.
Conclusions:
(II) Males generally are not fond
(I) Some books are red.
of window shopping and they
(II) Some toys are books.
feel it is sheer waste of time.
(a) Conclusion I follows
(a) (b) (c) (d)
(b) Conclusion II follows Conclusions:
Direction (95): Study the following (I) Shopping behavior is different
(c) Neither I nor II follows
information carefully and answer the (d) Both I and II follows for males and females.
questions given below: 98. Statements: (II) If males go for window
There are five persons A, B, C, D and (I) Kids rejoice with toys shopping, they don't utilise
E, each having different height. E is and toys give them new much time in selecting a
shorter than only two persons. B is opportunities to think in product.
various ways. (a) Only Conclusion II follows
not the tallest. C is taller than A. C is
shorter than B. (II) Toys are non-living things (b) Conclusion I and II follow
that create a virtual world
around kids to think and (c) Neither I nor II follow
95. Who among them is taller than
only A? perceive in various ways. (d) Only Conclusion I follows

Answers with Explanation


5. (b) The correct spellings of 7. (c) The word purposely (Adverb)
ENGLISH other words are: capricious; fallacious, means : on purpose; deliberately;
1. (a) 2. (c) 3. (d) delicious. intentionally.
4. (a) The correct spelling is : 6. (b) Idiom come clean means: to Hence, the words purposely and
admit and explain something that you unintentionally are antonymous.
superfluous. have kept as a secret.
Practice Set-4 47

8. (a) The word Guilty (Adjective) velocity of man relative to ground


V1 V T 
means : having done something illegal; u ⇒ = 2 ⇒ V2 = V1  2 
being responsible for something bad. =u−v = T1 T2  T1 
2
Look at the sentence : T1 = 273 + 27 = 300 K
24. (a) dA = 2 cm and dB = 4 cm
The jury found the defendant guilty T2 = 273 + 327 = 600 K
of the offence. \ rA = 1 cm and rB = 2 cm
The word Innocent (Adjective) From equation of continuity , av =  600 
But, V1 = 100 cc ⇒ V2 = V1  
means : not guilty of a crime, not having constant  300 
done something wrong. V a π(r ) 2
  2
2
V2 = 2V1
\ V = a = π(r ) 2 =  1  ⇒ VA = 4VB
A B B
Hence, the words guilty and innocent B A A V2 = 2 × 100 = 200 cc
are antonymous.
9. (b) 10. (d) 11. (a) 12. (d) 25. (c) 1 is isobaric process, 4 is 32. (a) Nickel exhibits ferromagnetism
isochoric of 2 and 3, 2 has the smaller because of a quantum physical
13. (c) Here, hit the window of my
slope (magnitude), hence, it is isothermal effect called exchange coupling in
car etc./head — should be used. Remaining process is adiabatic. which the electron spins of one atom
14. (a) Replace If I would know by 26. (a) In the first medium frequency: interact with those of neighbouring
If I knew. atoms. The result is alignment
V
Look at these sentences: V = of the magnetic dipole moments of
λ
If a tiger came, all would run away. the atoms, in spite of the candomising
as V' = V
If I had wings, I would fly to you. tendency of atomic collisions.
V' 2V V This persistent alignment is what
15. (a) 16. (c) ⇒ = =
λ' λ' λ gives ferromagnetic materials their
17. (d) The word Avert (Verb) means
∴ λ ' = 2λ permanent magnetism.
: to prevent something bad or dangerous
from happening; to turn your eyes; avoid. 27. (b) At the node A, In + 0.7 + 0.3 33. (a)
18. (b) = 2.2 GM M
Ve = 2 gR = 2 R ⇒ Ve ∝
19. (a) We were told about the new i.e., In = 1.2A. At the node B, 2.2 R2 R
project by the MD. A enters the node while the other three Mass is 1000 times and radius is 10
20. (a) The police ordered the thief currents leave the node. The unknown times. Therefore, escape velocity will
not to move. become 10 times.
current at B is:
⇒ Ve = 11.2 × 10 =112 kms –1
PHYSICS 2.2 – (1.0 + 0.4) = 0.8 A
2 2 2
21. (c) v = u + 2as ⇒ v = 2as Then at the node C, 0.8 + 0.4 = ly 2π π
34. (c) =
 ...(i) i.e. Iy = 1.2 A λ 20
Force on the ship F = ma ...(ii) By Kirchhoff’s 1st law current in ∴ λ = 40 cm
From Eqs. (i) and (ii), we get branch CD is 3A. Separation between two consecutive
2Fs 28. (a) V = IgG nodes:
v= λ 40
m nV = Ig(G + R) = = = 20 cm
2 2
4
2 × 5 × 10 × 3 On dividing R = (n – 1)G
= = 0.1 ms −1 2
35. (b) The sensitivity of potentiometer
3 × 107 29. (b) L p = L1L 2 /(L1 + L 2 ) = L = L can be increased by decreasing the
2L 2
22. (a) The potential energy of a potential gradient i.e., by increasing the
system increases if work is done by the 30. (c) length of potentiometer wire.
system against a conservative force: c 3 × 108 36. (b)
λ = = = 3 × 105 m = 300 km
− ΔU = Wconsarvative force v 1000 mv 2
qvB =
R
u 31. (d) If pressure of a given mass of
23. (b) m(u − v) = mv ⇒ v = v 1.92 × 107
2 the gas is kept constant, then ⇒ B= =
 q 9.6 × 107 × 6.4 × 106
→u–v   R
V m
V V∝T ⇒ = constant
m(u – v) = mv T = 3.12 × 10–8 T
48 Practice Set-4

37. (d) If either source or screen both 43. (c) Here, 512
are at finite distance from the diffracting X L = 1Ω, R = 2Ω, E v = 6V, P = ? ∴ = ar p –1
729
device the diffraction is called Fresnel p−1
Z = X 2L + R 2 = 12 + 22 = 5Ω  2
diffraction. = 18  − 
Ev 6  3
38. (c) On the basis of Bohr’s model Iv = = A
Z 5 p−1
n2 h2 6 2  2 512
r= = a0 n 2 / Z P = E v Iv cos θ = 6 × × = 14.4 W ⇒ −  =
4π2 mKZe 2 5 5  3 729 × 18
8
for Li++ ion, Z = 3, n = 1 for ground 44. (c) In an unbiased p-n junction,  2
=  − 
state the diffusion of charge carriers  3
Given a0 = 53 pm across the junction takes place
So, p – 1 = 8
from higher concentration to lower
53 ×12 concentration. ⇒ p=9
\ r=  18 pm
3 45. (d) Spring constant, surface 50. (b) According to question,
39. (b) In a transistor the base is a tension and surface energy have the m r
conductor of low resistance. same dimensions namely [ML 0 T –2 ] [2a + (m − 1)d ] = [2a + (r − 1)d ]
2 2
40. (c) In optical communication, light acceleration due to gravity has dimensions
2am + m(m – 1)d= 2ar + r (r – 1)d
is transmitted in the form of light pulses [LT–2]
2am – 2ar = r(r – 1)d – m(m – 1)d
along an optical fibre as transmission
medium. Since, wavelength of the MATHEMATICS 2a(m – r) = (r2 – r – m2 + m)d
light is very small, it provides a very 46. (a) A – B = {1, 3, 5, 9}, C = {3, 2a(m – r) = {r2 – m2 + (m – r)}d
large bandwidth and can carry of two 4, 5, 8, 9, 10} 2a(m – r) = [(r – m) (r + m) + (m – r)
wire transmission line or co-axial cable ∴ (A – B) ∪ C= {1, 3, 4, 5, 8, 9, 10} d]
transmission are that they are quite cheap, 2a(m – r) = (m – r) [– (m + r) + 1]d
a + ib
very eassy to install and less electrical 47. (b) ·.· x + iy = ∴ 2a = – md – rd + d
c + id
disturbances (noise).
Sum of (m + r) term
So, the required answer is high a − ib
∴ x – iy = m+r
bandwidth and low transmission loss. c − id = [2a + (m + r − 1)d ]
41. (d) de-Broglie suggested that the 2
a + ib
dual nature is not only of light, but each ∴ (x + iy) (x – iy)= × m+r
c + id = [−md − rd + d + md + rd − d ]
moving material particle has the dual 2
nature. The wavelength of wave, where a − ib m+r
c − id = × 0 = 0
p is momentum is given by: 2
h a 2 + b2 51. (d) All the numbers are formed
λ= ⇒ x2 + y2 =
p c2 + d 2 by following ten digits:
It is given that α, β, γ-rays carry same 0,1, 2, 3, 4, 5, 6,7, 8, 9
a 2 + b2
momentum, so they will have same ⇒ 2 2 2
(x + y ) = Out of them, taking 9 digits, number
wavelength. Hence option (d) is true. c2 + d 2 of 9 digits number obtained will be =
48. (d) We know that universal 10
42. (a) v 2 = u 2 − 2 gh P9
relation is the largest relation and
Here v = 0, u = v0 Out of above nine digit numbers those
universal relation is {(x, x) (x, y) (y, x)
numbers are also exist which start from
0 = v02 − 2 gh (y, y)} which is largest relation.
\ 0, those number = 9P8
v0 = 2 gh 512
⇒ 49. (b) Let pth term of series is Leaving this, required 9 digit numbers
729
where h' = 3h, then = 10P9 – 9P8
a = 18,
v '0 = 2 g × 3h = (10) ! – 9 !
12 2
r =– = − = 9 ! × 10 – 9 !
= 3 2 gh = 3 v0 18 3

= (10 – 1) × 9 ! = 9 × 9 !
Practice Set-4 49

x+4 x−4 80 π 3 l r1 ⇒ ∆ = 6s = 6 × 35
52. (a) − = ....(i) ∴ × × ⇒ Area of ∆ = 210 cm2
x−4 x+4 9 4 π = r2 l
⇒ 1
Since x = 5 satisfies (i), hence one root r1 3 ac = 210,
is ‘5’ or, = 2
r2 4 ·.·∆ABC is right angled triangle.
53. (b) Since,ω is the cube root of –1
1 + sin 2 θ sec 2 θ ac = 420  ....(iv)
∴ ω3 = –1, i.e., ω3 + 1 = 0 56. (d)
1 + cos 2 θcosec 2 θ From (i) and (iii),
⇒ (ω + 1) (ω2 – ω + 1) = 0 [70 – (a + c)2] = a2 + c2
1
Hence either ω = –1 or ω2 – ω + 1 = 0 1 + sin 2 θ 4900 – 140[ (a + c) + (a + c)2]
cos 2 θ
= = a2 + c2
1 −ω ω2 1
1 + cos 2 θ 2 or, 4900 – 140[(a + c) + (a2 + c2 +
Let A = −ω ω
2
1 sin θ
2ac) = a2 + c2
ω2 1 −ω 1 + tan 2 θ sec 2 θ or, 4900 – 140 (a + c) + 2(420)
= = cosec 2 θ = 0 [from (iv)]
Applying C1 + C2 + C3 1 + cot 2 θ
sec θ or, 140 (a + c) = 4900 + 840
1 − ω + ω2 −ω ω2
= 5740
= −ω + ω2 + 1 ω2 1 = cosec θ
or, a + c = 41 ....(v)
1 sin θ
ω2 + 1 − ω 1 −ω = × = tan θ On solving (iv) and (v) a = 21, c = 20
cos θ 1
0 −ω ω2 57. (b) If the coordinates of the ∴ From (iii) and (v), b = 70 – 41 = 29
2 centre of circle are (h, k) then h = k and ∴ a = 21 cm, b = 29 cm, c = 20
= 0 ω 1 =0
3h + 4k − 4 cm
0 1 −ω =h
5 59. (c) Total wages of the earners in
(·.· given 1 – ω + ω = 0)2
⇒ 3h + 4h – 4 = 5h A
54. (a) ·.· n = 10 ⇒ h =2=k (because h= k) = 250 × 2.00 = ` 500.00
and total wages of the earners in B
∴ Number of terms in expansion
= 200 × 2.50 = ` 500.00
= (10 + 1) = 11
∴ Total wages of both A and B
Therefore, middle term of an
= ` 500 + ` 500
3x

expansion
+

= ` 1000
4y

1
=
x=0

th
= (11 + 1) term
4

2 ∴ Average of wages of both A and B


= 6th term 1000
= = ` 2.22
th
= (5 + 1) term y=0 250 + 200
5 60. (d) Coordinates of the focus of x2
= C5 x  − 1 
10 5
∴ The equation of the circle will be: = –8y are (0, –2)
 x (x – 2)2 + (k – 2)2 = (2)2 Y
1 ⇒ x2 + y2 – 4x – 4y + 4 = 0
= –252 x5 −
x5 58. (c) Let angle B of triangle ABC
= –252 = 90°, then
55. (b) Let the radius be r1 and r2 b2 = a2 + c2 ....(i)
(0, 0)
According to the question, X' X
π l
∴ 3 = 2s = a + b + c
r1 S–
= 70 or s = 35....(ii)
where l is the length of arc ⇒ b = 70 – (a + c)....(iii)
π l ∆ (0, –2)
and = and r = 6 ⇒ =6 Y'
4 r2 s
50 Practice Set-4

2x ∴ x1 + y1 = x2 + y2 1
61. (b) Let u = tan–1 = lim n →∞ [n log e − log π]
1− x 2 But the relation x1 + y1 = x2 + y2 is n
possible only from the answer (d).
 2 tan θ   log π 
Put x = tan θ u= tan–1   63. (b) Put x+y =v = lim n →∞ 1 −
 1 − tan 2 θ   n 
dy dv = 1 – 0 = 1
= tan–1 (tan 2 θ) ⇒ 1+ =
dx dx ∴A=e
u = 2θ = 2 tan–1 x
 v − a  dy  v +a 66. (c) Total number of coins = 4 + 3
du 1 ∴   − 1 = v + b
= 2.  v − b  dx  =7
dx 1 + x2 Total of ways of placing 7 coins
dv (v + a )(v − b)
1 − x2 = (v − b)(v + b) + 1 7!
Also let v = cos –1
dx = = 35
1 + x2 4!3!
Put, x = tan θ dv 2v 2 − 2ab If we will place two 50 paisa coins at
⇒ = 2
dx v + (b − a )v − ab extreme place there will be 4 one rupee
1 − tan 2 θ
∴ v = cos–1 coins and 1 fifty paisa coin they can be
1 + tan 2 θ  v 2 + (b − a )v − ab  placed between these two fifty paisa coins
⇒   dv = 2dx
= cos–1 (cos 2θ)  v 2 − ab  5!
in i.e., 5 ways.
v = 2θ = 2 tan–1 x Integrating, we find 4!
dv 1 5 1
∫ 1 + v 2 − ab  dv = 2 ∫
∴ = 2.  (b − a )v  dx + C ∴Required probability = =
2
dx 1+ x 35 7
2x 67. (c)Let log x = t
∴ Diff. Coefficient of tan–1 b−a
1 − x2 ⇒ v+ log(v 2 − ab) = 2x + C 1
w.r.t. 2 ∴ dx = dt
x
1 − x2 b−a
du / dx ⇒x +y+ log{( x + y ) 2 − ab} when x = a, then t = loge a and when
cos–1 2 = 2
1+ x dv / dx x = b, then t = log b
= 2x + C
2 / (1 + x 2 ) b log x log b
= =1 ⇒ 2y – 2x + (b – a) log {x2 + y2 + ∴ ∫a x dx = ∫log a t dt
2 / (1 + x 2 ) 2xy – ab} = C
log b
62. (d) ·.· AP = AQ = 2 2 64. (d) Let cot–1 x be θ 1 2 
=  t 
 2  log a
∴ PQ = 2 × 2 2 = 4 2
2
x 1
1+ 1 = [(log b)2 – (log a)2]
2
Q 1
θ = [(log b + log a) (log b – log a)
A 2
x
P ∴ cot θ = x 1  b
log   log (ab)
1 2  a
135º ∴ sin θ =
1 + x2 a+x
68. (c) ∫ a−x
dx
1
∴ sin cot–1 x =
and the slope of the line QAP = tan (a + x)(a + x)
1 + x2
135 = –1
1/ n
= ∫ (a − x)(a + x)
dx
If P = (x1, y1) and Q = (x2, y2) then  en 
65. (c) Let A = lim n →∞  
The equation of PQ will be:
 π (a + x) 2
  = ∫ a2 − x2
dx
y – y1 = –1(x – x1) 1 e n 
∴ log A = lim n →∞ log   a+x
or, x + y = x1 + y1 n  π  = ∫ a2 − x2
dx
Similarly, x + y = x2 + y2
Practice Set-4 51

dx x or, 2b = a 45
= a ∫ +∫ dx y=
2 2 2 2 but b2 = a2 (1 – e2) 2
a −x a −x
∴ b2 = 4b2 (1 – e2) 45 39
Which gives x + y = 3 – =–
x x 3 2 2
= a sin–1
a
+ ∫ a −x2 2
dx ⇒ e=
2 81. (a) Total revenues of 7 years
= 83 × 7 = 58 lakhs
x REASONING AND GENERAL
Let I = ∫ a2 − x2
dx
AWARENESS (RAGA)
Total revenues of first 4 years
= 78 × 4 = 312 lakhs
71. (b) 72. (b) 73. (d) 74. (a) Total revenues of last 4 years
Put a2 – x2= t2 ⇒ –2x dx = 2t dt
75. (a) 76. (a) = 90 × 4 = 360 lakhs
−t dt 77. (c) Bikaner district, Rajasthan Total revenues of 8 years
∴ I= ∫ = ∫ −dt Explanation: On June 25, 2019, The
t2 Ministry of Home Affairs (MHA) = 672 lakhs
released the rankings of police stations Revenues of 4th year
2 2
= – t = − a − x for the year 2018. The Kalu police = 672 – 581 = ` 91 lakhs
station in Bikaner district of Rajasthan 82. (a) Let cost price be x of machine
a+x x
∴∫ dx = a sin–1 − a 2 − x 2 was ranked the best police station in the
a−x a S.P. = 57 lakhs
country. It was awarded the first rank
69. (b) Loss = CP – SP = x – 57
for its performance in crime prevention,
S.P. – C.P. = gain = 7 (x – 57)
Mid value Frequency investigation and disposal of cases,
Group
(x) (f)
f .x 67 – x = 7 (x – 57)
crime detection, community policing and
0 − 20 10 17 170 67 – x = 7x – 399
maintenance of law and order, women
20 − 40 30 f2 30 f 2 8x = 67 + 399 = 466
40 − 60 50 32 1600 help desk facility, drinking water facility
60 − 80 70 f4 70 f 4 and WiFi.Campbell Bay police station in 466
x= = ` 58.25 lakhs
80 − 100 90 19 1710 Nicobar district of Andaman & Nicobar 8
Total 120 3480 + 30f 2 + 70f 4
Islands was ranked second. 83. (b) C.P. of 2 items
·.· 68 + f2 + f4 = 120 78. (b) Let missing digit be x
Sum of odd place – sum of even (100 − 6 )
∴ f2 + f4 = 120 – 68 = × 900 × 2
place = 0, 11, 22, ..... 100
= 52
(3 + 7 + 5 + 7) – (4 + x + 4) = 94 × 9 × 2
and 3480 + 30 f2 + 70 f4 = 120 × 50 = 0, 11, 22, .... 22 – (8 + x) (100 − 12 )
= 6000 = 0, 11, 22 = ` 846 × 2
100
∴ 30f2 + 70f4 = 6000 – 3480 ⇒ 22 – 8 – x = 11 = `1692
= 2520 79. (a) Let the numbers be x and
(10 – x) C.P. of 3 items = × 900 × 3
⇒ 3f2 + 7f4 = 252
Then, x (10 – x) = 5 × 495 = 88 × 9 × 3 = ` 79 2 × 3
and 7 f2 + 7f4 = 364 ⇒ x2 – 10x – 2475 = 0 = ` 2376
– – =– ⇒ (x – 55) (x + 45) = 0 Total C.P. of Rajani = 1692 + 2376
–4f2 = –112 ⇒ x = – 55 or x = 45 = ` 4068
−112 ⇒ The numbers are 45 and 55 Total price with out discount
f2 = = 28 = 900 × 5 = ` 4500
−4 Required difference
Discount = (4500 – 4068) = ` 432
and f4 = 52 – 28 = 24 = (55 – 45) = 10
Effective Distance %
2b 2 80. (a) 3x – 2y = 5 ...(i)
70. (c) ·.· Latus rectum = 432 × 100
a y 15 = = 9.6%
and = 4500
and minor axis = 2b x – y 17
84. (d) Let numbers be 4 : 28 :: 28 : x
2b 2 ⇒
y = 17y
4 28
a ⇒ 15x – 2y = 0
...(ii) =
∴ =b 28 x
Solving these equations we get x = 3,
52 Practice Set-4

28 × 28 And, Finally the missing term is AB.


x= = 7 × 28 91. (c) By analysing the circular
4
arrangement, it is clear that required
= 196
pair  CF i.e., ‘C’ is to the immediate
85. (d) Let speed of Rajdhani train
right of ‘F’.
be u at time t, speed of Express train
= 2u Similarly, 92. (b) According to the meaning of
Speed of Rajdhani u the given symbols,

D 672 x–y=zx=y>z
u= = ...(i)
T (t + 14) Using the proper symbols in option
Speed of Express train (b), we get x D y × z  x >| y <| z  x =
672
2u= ...(ii) y > z.
t –8 88. (b)
Dividing equation (i) by (ii), we get Therefore, x – y = z  x D y × z.
5 km
1 t –8 km
20 93. (d) Except kilogram, all units are
=
2 t + 14 end related to current.
10 km
t + 14 = 2t – 16 start 94. (b) In each subsequent figure, one
2t – t = 14 + 16 = 30 N circle from the lower side is deleted and
N-W N-E
t = 30 hours the first circle is increasing in size.
672 W E
Speed of Rajdhani train = 95. (d) Clearly, C is taller than only A.
30
S-W S-E 96. (a) According to statements,
= 22·4 km/hr. S
Conclusion I follows.
86. (d) My mother's sister's husband's Finally the man is in south-east
only son  My aunt's son i.e., cousin. direction from the starting point. 97. (b) According to statements,
Therefore, my cousin's sister  my 89. (a) According to question, the Conclusion II follows.
sequence is:
cousin. 98. (c) According to statements,
87. (a) As, Jyoti > Vineet > Raman > Deepak
> Sumit. neither Conclusion I nor II follows.
So, Jyoti is the tallest. 99. (a) According to statements,
90. (b)
Conclusion I follows.
100. (d) According to statement, only
Conclusion I follows.
qqq
Practice Set-5 53

Air Force Airmen Group X & Y Exam


Practice Set-5

But the lioness was getting old and 35 (a) The bag together with the books
ENGLISH years of fighting on foreign-land had are lost.
Directions (1-3): Read the passage taken its toll. She decided to return (b) The bags together with the book
given below and answer the questions to her motherland. Her health was is lost.
that follow by selecting the most worsening. After reaching Bombay, she (c) The bag together with the books
appropriate option. was hospitalised and died on 13th of is lost.
Madam Cama’s Paris home became a August, 1936. (d) The bags together with the books
shelter for world revolutionaries. Even 1. The author’s attitude to Madam is lost.
Lenin, the father of Russian revolution 9. The boy stood on the table to catch
Cama can be described as–
the attention of the audience. The
visited her house and exchanged views. (a) laudatory (b) critical underlined phrase is :
Savarkar got all encouragement in (c) contradictory (d) hostile (a) A Noun phrase
writing the history of the “First Indian 2. The word ‘ingeniously’ most (b) A Verb phrase
War of Independence” from Cama. (c) An Adjective phrase
nearly means
She helped its printing in Holland as (d) An Adverbial phrase
no English publisher came forward (a) cleverly (b) wilfully
10. What does the idiom “Be-all and
to publish it. It was a banned book (c) devilishly (d) secretly end-all” mean?
but found its way to India. Smuggled 3. The word which is opposite in (a) From beginning to end
ingeniously under “Don Quixote” covers! meaning to ‘famous’ is– (b) That which begins well
She became the publisher of “Vande (a) undesirable (b) mysterious (c) That which ends well
Mataram”, a revolutionary magazine (c) unknown (d) unpopular (d) The whole and sole
and its distributor, an extremely difficult 11. ‘To deal in something’ means :
Directions (4-6): Fill in the blanks.
task in the days of British espionage. (a) to distribute something
4. Will you be here_______the
Another magazine “Madan’s Talwar” (b) to trade in something
weekend? (c) to be about something
was also started in memory of Madan (a) in (b) on (d) to handle something
Lal Dhingra who laid down his life for (c) about (d) at 12. The most appropriate one word
the country. Both the magazines were 5. A good judge never jumps_______ substitute for “A person living at
outlawed in India and England. Madam the conclusion. the same time as another” is :
Cama somehow found the ways to send (a) at (b) for (a) Archaic (b) Nostrum
them to Indian revolutionaries. (c) Contemporary (d) None of these
(c) to (d) on
Madam Cama also fought for the 13. The antonym of Familiar–
6. Such remarks are certainly
cause of women. Speaking at National derogatory______ your reputation. (a) Known (b) Infamiliar
Conference at Cairo, Egypt in 1910, she (c) Unfamiliar (d) Unknown
(a) for (b) to 14. Complete the sentence.
asked, “Where is the other half of the
(c) with (d) of ‘Aesthetic’ means_________
Egypt? I see only men who represent
half of the country!” She stressed the 7. You must look into this matter. (a) sorrow (b) despair
(Change the voice) (c) beauty (d) pleasure
role of women in building a nation.
When the First World War broke (a) This matter has been looked into 15. What is the synonym of ‘wisdom’?
out in 1914, Madam Cama took an anti- by you. (a) swell (b) sagacity
British stand and tried her best to make the (b) This matter may be looked into (c) foolishness (d) surrender
by you.
Indian people aware of the exploitative 16. Which one of the following is the
(c) This matter should be looked into correct spelling?
nature of British imperialism.
by you.
The British had banned her entry into (a) Ceasarean (b) Caesaraen
(d) This matter into looked by you.
India, being afraid of her revolutionary (c) Caesarean (d) Ceasaraen
8. The correct sentence is:
past and staunch nationalistic outlook.
54 Practice Set-5

Directions (17-18): In the following kΩ. The heat produced in the same then acceleration due to
questions, some of the sentences gravity on the earth surface:
have errors and some have none. resistance is:
find out which part of a sentence has (a) 0.16 J (b) 0.32 J (a) Increases by 1%
an error. The number of that part is (c) 0.64 J (d) 1.28 J (b) Decreases by 2%
your answer. If there is no error, your 25. A c o i l o f a re a 1 0 c m 2 , 1 0 (c) Decreases by 1%
answer is (d).
17. I found (a)/the two first chapters of turns and resistance 20 Ω (d) Increases by 2%
the book (b)/particularly interesting. is placed in a magnetic field directed 30. µ0 and ε0 denote the permeability
(c)/No error (d). perpendicular to the plane of the and permittivity of free space
18. Bacon, the father of the English essay coil and changing at the rate of the dimensions of µ0ε0 are:
(a)/had a thirst (b)/of knowledge. (c)/
No error (d). 108 gauss/second. The induced (a) [LT–1] (b) [L2T–2]
19. Choose the word opposite in current in the coil will be: (c) [L T ]
–2 2
(d) [L–1T]
meaning to the given word. (a) 5 A (b) 0.5 A 31. Identical charges of magnitude
Despair (c) 0.5 A (d) 50 A Q are placed at (n – 1) corners
(a) Belief (b) Trust 26. The magnetic field due to a
(c) Hope (d) Faith of a regular polygon of n sides
20. Choose the most appropriate current carrying circular coil each corner of the polygon is at
preposition out of four options: on the axis at a large distance x a distance r from the centre. The
She has three children .......... her from the centre of the coil varies field at the centre is:
first husband.
(a) of (b) by approximately as:
(c) in (d) from (a) x–1 (b) x–3/2 (a) kQ
r2
(c) x –3
(d) x–2
PHYSICS kQ
27. Three ammeters A, B and C (b) (n − 1)
21. A proton and an electron have of resistances R A , R B and R C r2
same de-Broglie wavelength respectively are joined as shown. n kQ
which possesses more kinetic (c)
When some potential difference (n −1) r 2
energy? is applied across the terminals T1
(a) Proton (n − 1) Q
and T2, their reading are IA,IB and (d) k 2
(b) Electron n r
IC respectively: 32. The core of any transformer is
(c) Both possess equal K.E.
(d) Data insufficient to decide laminated so as to:
A B
22. A large parallel plate capacitor, (a) reduce the energy loss due to
whose plates have an area of eddy currents
T1 T2
1 m 2 and are separated from (b) make it light weight
each other by 1 mm, is being (c) make it robust and strong
C
charged at a rate of 25 V/s. If the (d) increase the secondary voltage
dielectric between the plates has (a) IA = IB 33. A ball having kinetic energy E
the dielectric constant 10, then (b) IA RA+ IB RB = ICRC is projectred at an angle of 45°
the displacement current at this with the horizontal. Its kinetic
IA R
instant is: (c) = C energy at the highest point will
IC RA be:
(a) 25 µA (b) 11 µA
(c) 2.2 µA (d) 1.1 µA IB RC
(d) = 1
23. For Van der Waal’s gas the value IC RA + RB (a) E (b) E
2
RTc 28. A particle exectues S.H.M. with a 1
of is: E
Pc Vc period of 6 sec and an amplitude (c) (d) 0
2
(a) 3/8 (b) 8/3 of 3 cm. Its maximum speed in
34. Two copper rods of area of
(c) 5/3 (d) None of these cm/s is:
(a) π/2 (b) π cross-section 5 cm2 and
24. A 4µF capacitor is charged to 10 cm2 have lengths 10 cm and
400 V and then its plates are (c) 2π (d) 3π
29. If the radius of the earth reduces 20 cm respectively. Rods have
joined through aresistance of 1 one end in steam and other in
by 1% and its mass remains
Practice Set-5 55

ice. The ratio of ice melted per 39. In a common-emitter transistor conductivity of A is twice that of
second because of two rods will amplifier circuit β = 100, input B. The temperature difference
be in ratio: resistance R 1 = 1 kΩ, output across the wall is 36°C. The
(a) 1 : 1 (b) 1 : 2 resistance R 2 = 10 kΩ. The temperature difference across
(c) 2 : 1 (d) 2 : 3 voltage gain of circuit is: the layer A is:
35. A train of mass 103 kg is coming (a) 100 (b) 1000 (a) 6°C (b) 12°C
down an inclined plane at 30° to (c) 10 (d) 5000 (c) 18°C (d) 24°C
the horizontal. The retardation 44. A diver in a lake wants to signal
40. The average e.m.f. during the
is produced in the train by his distress to a person sitting
positive half cycle of an A.C.
on the edge of the lake flashing
applying brakes when the speed supply of peak value E0 is: his water proof torch. He should
of train is 100 m/s. How much
E0 direct the beam
force is needed to be applied by E0
(a) (b) (a) vertically upwards
the brakes to stop the train in π 2π (b) horizontally
500 m distance? E0 2E 0 (c) at an angle to the vertical which is
(a) 104 N (b) 1.5 ×104 N (c) (d) slightly less than the critical angle
2π π
(c) 3 × 10 N4
(d) 2.5 × 104 N 41. The equation of simple harmonic (d) at an angle to the vertical which
36. The velocity of water waves motion of amplitude 5 m and time is slightly more than the critical
may depend on their wavelength angle
period 0.5 s is:
λ, the density of water ρ and 45. The mirrors are perpendicular
(a) y = 5 sin 4πt
the acceleration due to gravity to each other as shown in figure.
(b) y = 5 sin 6πt
g. The method of dimensions A light ray AB is incident on the
2π mirror M 1. Then the reflected
gives the relation between the (c) y = 0.5 sin t
5 ray will also suffer a reflection
quantities as :
(d) y = 0.5 sin 10πt from the mirror M2. Then the
(a) v2 = kλ–1g–1 ρ–1 42. A rectangular loop carrying a
(b) v2 = kgλ final ray after reflection from M2
curent i is situated near a long will be parallel to the incident
(c) v2 = kgλρ
straight wire such that the wire ray, if:
(d) v2 = kλ3g–1 ρ–1
is parallel to one of the sides of
37. T h e e q u i v a l e n t re s i s t a n c e
the loop and is in the plane of
between points A and B is:
A
the loop. If a steady current I is
B
2Ω 15 Ω established in the wire as shown
in the figure, the loop will:
8Ω 10 Ω

20 Ω 10 Ω
30 Ω 40 Ω (a) i = 45°
(b) i = 60°
(a) 32.5 Ω (b) 22.5 Ω (c) i < 30°
(c) 2.5 Ω (d) 42.5 Ω (d) for any i between 0° and 90°
38. When two tuning forks A and B
MATHEMATICS
are sounded together, 4 beats per (a) rotate about an axis parallel to the
second are heard. Now if arm of wire 46. For real numbers x and y we write
A is loaded with a little wax and (b) move away from the wire xRy ⇔ x – y + is an irrational
then they are sounded together, (c) move towards the wire number, Then, the relation R is:
2 beats per second are heard. If (d) remain stationary
(a) reflexive
the frequency of A is 256 then 43. A wall has two layers A and
(b) symmetric
frequency of B will be: B, each made of a different
(c) transitive
(a) 250 (b) 252 material. Both the layers have
(d) None of these
(c) 260 (d) 262 same thickness. The thermal
56 Practice Set-5

47. If z1 = 8 + 4i, z2 = 6 + 4i and arg


1 1 1
58. + + is
= , then z satisfies. tan–1 at x = 0, is: log xy xyz log yz xyz log zx xyz
equal to:
(a) 1/8 (b) 1/4 (a) 0 (b) 1
(a) | z – 7 – 4i | = 1
(c) 1/2 (d) 1 (c) 2 (d) None of these
(b) | z – 7 – 5i | = 59. What is the only solution of
(c) | z – 4i | = 8 53. W h a t i s t h e i n i t i a l v a l u e p ro b l e m
equal to? y' = t (1 + y), y (0) = 0?
(d) | z – 7i | =
(a) x ex + C (a) y=–1+ e
t2 / 2

48. If the root of the equation ax + 2


2
(b) ex +C (b) y = 1 + et / 2
bx + c = 0 are of the form (c) y=–t
(c) 2 ex +C (d) y=t
(d) 2xex + C
and , than (a + b + c)2 is 60. If f (x) = xex(1 – x), then f (x) is:
54. If (–5, 4) divides the line segment
 1 
equal to: between the co-ordinate axes in (a) increasing on  − ,1
(a) b2 – 4ac  2 
the ratio 1: 2, then what is the
(b) decreasing on R
(b) b2 – 2ac equation?
(c) increasing on R
(a) 8x + 5y + 20 = 0
(c) 2b2 – ac 1
(b) 5x + 8y – 7 = 0 (d) decreasing on  − ,1
(d) Σa2 (c) 8x – 5y + 60 = 0  2 
49. If the number n – 3, 4n – 2, 5n + (d) 5x – 8y + 57 = 0
61. If y = tan –1 [{ 1 + x 2 ) − 1} / x] ,
1 are in AP, What is the value of 55. If i = , then the value of then
n? (a) y' (0) = 1
(a) 1 (b) 2 (b) y' (0) = 1/2
is:
(c) 3 (d) 4 (c) y (0) = 0
50. Find the equation of lines (d) None of these
(a)
parallel to 3x – 4y – 5 = 0 at a 62. Mean and mode of the following
unit distance from it. data are respectively.
(a) 3x – 4y = 0, 3x – 4y – 10 = 0 (b)
Class t1
(b) 3x + 5y = 0, 4x + 5y + 1 = 0 0 – 10 22
(c) 3x – 4y = 0, 3x + 4y + 1 = 0 (c) 10 – 20 38
(d) None of these 20 – 30 46
(d) None of these
51. What is the value of k for which 30 – 40 35
56. What is the equation to circle 40 – 50 20
the following function f (x) is which touches both the axes and (a) 24.56, 24.26
continuous for all x? has centre on the line x + y = 4? (b) 24.26, 24.56
(a) x2 + y2 – 4x + 4y + 4 = 0 (c) 39.55, 16.1
(b) x2 + y2 – 4x – 4y + 4 = 0 (d) None of these
f (x)=

(c) x2 + y2 + 4x – 4y – 4 = 0 63. The number of ways in which 9
(d) x2 + y2 + 4x + 4y – 4 = 0 identical balls can be placed in
(a) 3 (b) 2 57. What is the value of three identical boxes is:
(c) 1 (d) –1 9!
2log8 2 – ? (a) 55 (b)
52. T h e d e r i v a t i v e o f t a n – 1
(3!) 4
(a) 0 (b) 1
with respect to 9!
(c) 2 (d) 1/3 (c) (d) 12
(3!)3
Practice Set-5 57

64. The chance that an event E (a) tan–1 (b) 0 (a) United National Educational,
2
‘occurs’ or ‘does not occur’ is: (c) π (d) π/2 Scientific ad Cultural
(a) 0 (b) 1 Organizational(UNESCO)
(c) 2 (d) None of these cot 54° tan 20° (b) World Vision International
70. What is + equal
tan 36° cot 70° (c) Save the Children
b log x to?
65. What is ∫a x
dx equal to? (d) United Nations Children’s
(a) 0 (b) 1
Fund (UNICEF)
 1  b (c) 2 (d) 3
(a)   log(ab).log   78. If 9/4th of 7/2 of a number is
2 a
REASONING AND GENERAL 126, then 7/2th of that number is
log b ...........
(b) AWARENESS (RAGA)
log a (a) 56 (b) 284
71. Fundamental Rights are
 b guaranteed in which Part of (c) 72 (d) 26
(c) log   79. The H.C.F. of two numbers is 11
a Indian Constitution?
(a) Part III (b) Part IV and their L.C.M. is 7700. If one
 1  ( a + b) 
(d)   log  of the numbers is 275, then the
2  ab  (c) Part V (d) Part VI
other is:
72. An example of vestigial organ is:
66. What is equal to? (a) 279 (b) 283
(a) Lung
(a) 1 (b) –1 (c) 308 (d) 318
(b) Pancreas
(c) 0 (d) e 80. If 2x + y = 5 and 3x – 4y = 2, then
(c) Nictitating membrane
67. If the angles of a triangle are 30° the value of (x + y)2 is:
(d) Thyroid gland
and 45° and the included side (a) 9 (b) 5
73. Which one of the following is
is ( 3 + 1) cm, then what is the (c) 3 (d) 4
luminous?
area of the triangle? 81. The mean of marks secured by
(a) Pole Star (b) Earth
60 students in division A of class
(c) Moon (d) Uranus
3 +1 2 X is 66, 45 students of division B
(a) cm 74. Which among the following is an
2 is 62 and that of 75 students of
item of import in India?
division C is 60. Find the mean
(b) 2( 3 + 1) cm2 (a) Handicrafts
of marks of the students of three
(b) Textiles
(c) Pearls divisions of class X.
3 +1
(c) cm2 (d) Engineering goods (a) 61.8 (b) 62.5
3
75. The largest dome in India is: (c) 61.1 (d) 63.9
3 −1 2 (a) Jama Masjid 82. Marked price of an item is ` 800.
(d) cm On purchase of 1 item discount
2 (b) Taj Mahal
(c) Gol Gumbaz is 15%, on purchase of 4 items
 1  1
68. What is tan –1  2  + tan –1  3  (d) Qutub Minar discount is 38%. Rajshri buys
equal to? 76. Website is a: 5 items, what is the effective
(a) Home page discount.
π π (a) 33.4 percent
(a) (b) (b) Space in the web
2 3 (b) 16 percent
(c) Web page
π π (d) Place where you can find (c) 9 percent
(c) (d) related web pages (d) 17.5 percent
4 4
77. Which organisation was released 83. When a number is increased by
69. If a chord which is normal to
the report titled “State of the 21, it becomes 114% of itself.
the parabola at one end subtend
Education Report for India What is the number?
a right angle at the vertex, then
2019: Children with Disabilities” (a) 150 (b) 252
angle to the axis is:
at an event in New Delhi? (c) 315 (d) 189
58 Practice Set-5

84. The sum of the ages of husband 90. Which set of letters when Direction (94): Arrange the given
and wife at present is 100. Ten sequentially placed at the gaps words in the sequence in which they
years ago the ratio of their ages in the given letter series shall occur in the dictionary.
was 9:7. What is the age of the complete it? 94. i. Hunger ii. Horse
husband? j_l_jk_m_kl_ iii. Hungry iv. Handmade
(a) 45 years (b) 55 years (a) jkljm (b) kmjjj (a) iv, ii, i, iii
(c) 65 years (d) 40 years (c) kmljm (d) mkjlm (b) iii, iv, ii, i
85. To travel 732 km, an Express Direction (91): Study the following (c) iv, iii, i, ii
train takes 6 hours more than information carefully and answer the (d) i, iv, ii, iii
Rajdhani. If the speed of the questions given below: (e) None of these
Express train is doubled, it takes A, B, C, D, E, F, G and H are sitting Direction (95): The following ques-
3 hours less than Rajdhani. around a circle facing the centre. B is tions are based on the five three digit
The speed of Rajdhani third to the right of F and third to the numbers given below:
is : left of H. C is fourth to the left of A, 394  632  783  576
(a) 81.3 km/hr (b) 61 km/hr who is not an immediate neighbour of
95. If the positions of the first and
(c) 40.7 km/hr (d) 101.7 km/hr F or B. E is not an immediate neigh-
the second digits within each
86. How is Golu's sibling's bour of B. G is second to the right of
grandfather's only daughter's number are interchanged, which
D.
only child related to Golu? of the following will be the
91. Who is third to the left of E? second highest number?
(a) Nephew (a) A
(b) Cousin (a) 632 (b) 783
(b) C
(c) Cannot be determined (c) 576 (d) 394
(c) G
(d) Niece Directions (96–100): In the following
(d) Data inadequate
87. If CLOUD can be coded as 59432 question, two statements are given
Direction (92): In the questions be-
and RAIN as 1678, how can each followed by two Conclusions I
low, some relationship have been ex-
AROUND be coded? and II. You have to consider the state-
pressed through symbols as shown
(a) 614832 (b) 614382 ments to be true even if they seem to
below.
(c) 641382 (d) 461382 be at variance from commonly known
f means less than
88. Riya travels 5 km in the east facts. You have to decide which of the
D means not greater than
direction, takes a right turn and – means equal to given Conclusion, if any, follows from
travels another 8 km. Next, she the given statements.
+ means not equal to
turns left to travel 4 km and then 96. Statements:
× means not less than
takes another left turn to travel (I) The DRDO has tested the
= means greater than
8 km more. Finally, she turns Smart Anti Airfield Weapon
towards the east and travels Study the meaning of these
from an Indian Air Force
5 km. How far is she now from symbols and pick correct answer
aircraft.
her original position? from the answer choices for each
(II) The lightweight high-precision
(a) 20 km (b) 14 km questions below:
guided bomb is one of the
(c) 15 km (d) 16 km 92. x – y = z does not imply
world class weapons systems.
89. Sanjeev scored the highest marks (a) y f x = z (b) z f y f x
Conclusions:
in the class. Rahul scored more (c) z f x = y (d) y f x f z
(I) DRDO Chairman Dr. S.
than Nirbhay but lesser than Direction (93): In the following ques-
Christopher congratulated
Sameer. Arpit scored more than tions, find out the odd word/letter/
the DRDO and the IAF teams
Rahul. Who got the 4th Rank out number from the given alternatives.
for the successful mission.
of the five? 93. (a) ZSLE (b) YRKD (II) The test was carried out by
(a) Sameer (b) Nirbhay (c) XQJC (d) WIPB IAF's Aircraft and System
(c) Rahul (d) Arpit (e) None of these Testing Establishment.
Practice Set-5 59

(a) Only Conclusion II follows (II) People generally go for outing (b) Conclusion I and II both
(b) Conclusion I and II both in holidays and get tired in follows
follows those days as well. (c) Neither I nor II follows
(c) Neither I nor II follows Conclusions: (d) Only Conclusion I follows
(d) Only Conclusion I follows (I) Holidays are meant for 100. Statements:
97. Statements: resting. (I) Children start writing with a
(I) 28 athelets are facing legal (II) Outing and different pen in sixth standard.
cases due to doping incidents. environment automatically (II) Ink pen is prescribed by most
(II) The new wave of Olympic rejuvenate. of the schools for school kids
doping cases is based on (a) Only Conclusion II follows to write with as it maintains
evidences. the handwriting and comfort
(b) Conclusion I and II both
Conclusions: of child.
follows
(I) The report of doping is based Conclusions:
(c) Neither I nor II follows
on the test that happen. (I) Till fifth standard kids are
(d) Only Conclusion I follows unable to use pen on their
(II) D o p i n g p r a c t i c e i s v e r y
99. Statements: own completely.
common these days among
athelets. (I) If we are lucky then things (II) Ink pen maintains a proper
automatically fall in place. flow of ink on paper and
(a) Only Conclusion II follows
(II) Luck follows only when we takes effort to put words on
(b) Conclusion I and II follows
work hard. paper.
(c) Neither I nor II follows
Conclusions: (a) Only Conclusion II follows
(d) Only Conclusion I follows
(I) Hard work is the key to success. (b) Conclusion I and II both
98. Statements:
(II) If you don't work hard, luck follows
(I) Holidays are meant to relax, will not follow you. (c) Neither I nor II follows
rejuvenate and recollect energy.
(a) Only Conclusion II follows (d) Only Conclusion I follows

Answers with Explanation


ENGLISH 12. (c) Contemporary is the word for use Ordinal Adjective first and then the
people living at the same time. Hence Cardinal Adjective.
1. (a) 2. (a) 3. (c)
option (c). Look at the sentences :
4. (b) ‘on’ is correct.
13. (c) The antonym of Familiar I have read the three first chapters.
5. (a) ‘to’ 6. (b) ‘to’
is–Unfamiliar. (×)
7. (c) The changed voice is (c) This
14. (c) ‘Acsthetic’ means beauty I have read the first three chapters.
matter should be looked into by you.
or having a sense fo the beautiful, ()
8. (c) ‘The bag’ and books are
characterised by love. 18. (c) Preposition ‘for’ is used with
subjects, so will be the verbs in according.
15. (b) Synonym of ‘wisdom’ is the word ‘thirst’. Hence, replace ‘of
Hence option (c) will be correct.
‘sagacity’. Wisdom means wise saying
9. (d) U n d e r l i n e d p a r t i n t h e knowledge’ by ‘for knowledge’.
or teachings; and sound judgement
sentence–on the table is–an adverbial 19. (c) The meaning of the word
discernment or insight.
phrase. Here on the table shows the 16. (c) Caesarean is correctly spelt. It despair (Noun) is: the felling of having
adverb of place. is an operation by which a fetus is taken lost all hope.
10. (d) Idiom “Be-all and end-all” from the uterus by cutting through the Look at the sentences :
means ‘the whole and sole’ or something walls of the abdomen and uterus. A deep sense of despair over-
that is very best or most important, 17. (b) Replace group of words ‘the whelmed him.
something so good that it will enc the two first chapters of the book’ by ‘the first He gave up the struggle in despair.
search for something better. two chapters of the book.’ When cardirial Hence, the words hope and despair
11. (b) ‘To deal in something’ means and ordinal Adjectives are used before are antonymous.
to trade in something. a Noun in a sentence, then we should 20. (b)
60 Practice Set-5

24. (b)
PHYSICS 1
=c 
1 30. (c)
21. (b) For proton U = CV 2 ε 0µ 0
2
1 (c = speed of light)
Ep = m p v 2p 1
2 ⇒ U = (4 × 10−6 ) (400) 2
2 1
mpvp = 2m p E p ⇒ U = 0.32 J ∴ ε 0µ 0 =
   c2
ξ 1
Similarly, me ve = 2me E e 25. (a) I= ∴ Dimensions of ε 0µ0 =
R [LT −1 ]2
h h 1  ∆B 
Now λp = = I= NA  = L−2 T 2
mp vp 2E p m p R  ∆t 
31. (a) The net electric field at the
h2 1  10  4
I= (10)  10 centre is zero, if all charges were placed
⇒ Ep =
2m p λ 2p 20  10000 
at the corners.
h2 Hence, field due to any one charge is
Similarly, Ee = equal (and opposite) to the field due to all the
2m p λ e2
µ0 NIR 2 other (n – 1) charges.
me 26. (c) Baxis =
Ep 2(R 2 × x 2 )3 / 2 32. (a) The core of transformer is
Dividing, =
Ee mp For, x >> R laminated to reduce energy loss due to
Since me < mp µ0 NIR 2 eddy currents.
Baxis =
E1 < E2 2x 3 1
i.e. electron has more K.E. than 33. (b) Initial K.E. = mv 2 = E
⇒ Baxis ∝ x −3 2
proton
ε KA 27. (d) 27(A, B, D) VA + VB = VC Velocity at the highest point
22. (c) C = 0
d IARA + IBRB = ICRC u
v = u cos θ = u cos 45° =
−12
(8.85 × 10 ) × 10 × 1 Since same current must flow through 2
=
10−3 A and B:
K.E. at the highest point
So IA = IB
= 8.85 × 10−8 F IB(RA + RB) = ICRC 1 2 1 u2 1
= mv = m = E
d dV RC
2 2 2 2
i = (CV) = C IB
dt dt =
IC RA + RB 34. (a) Amount of ice which melts
= 8.85 × 10 –8 × 25 per second will be proportional to the
28. (b) T = 65,
amount of heat passing per second.
= 2.2 × 10 –6 A a = 3,
Rate of ice melting due to
= 2.2 µ A 2π
vmax = aω = a.
T Q K.5.(100 − 0)
23. (b) For Van der waal’s gas: A ∝ =
2π t 10
8a = 3. =π
TC = 6 = 50 k
27Rb
GM Rate of ice melting due to
a 29. (d) g = 2
PC = R
27b 2 Q K.10.(100)
∆g 2∆R B∝ =
and VC = 3b and = − t 20
g R = 50 k
(G and M are constants)
RTC 8a 27b 2 1 Ratio is = 1 : 1
=R. × × ∆g
PC VC 27 Rb a 3b Thus, = − 2 (−1%)
g 35. (b) Decrease in potential energy
8
= = 2% increase in coming from A to B:
3
Practice Set-5 61

VA = 100 m/s ⇒ V2 = λg (omitting the constant k) current in opposite direction and hence
A 37. (b) The simplified circuit can be repel each other.
drawn as: But attraction is strong and repulsion is
500 m
15 Ω 10 Ω weak. So, loop moves towards the wire.
VB = 0 2Ω 8Ω
43. (b) KA = 2 K and KB = K
30° B Since same heat (Q) passes through both
C A 40 Ω 10 Ω
B the layers of the wall.
30 Ω 20 Ω
l l
D


25 Ω
= mg (AC)
2Ω 8Ω
= mg AB sin θ A B
50 Ω 36ºC θ 0ºC
= mg AB sin 30° 50 Ω
1
=1000 ×10 × 500 ×


2 2Ω 25 Ω 8Ω

= 2.5 ×106 J
A B A B

Decrease in kinetic energy The equivalent resistance K A . A1 . ∆θ1 K . A . ∆θ2


Q= = B 2
25 l1 l2
1 2 R AB = 2 + +8
mv − 0 = 2
2 2KA(36 − θ) KA(θ − 0)
= 2 + 12.5 + 8 ⇒ =
1 l l
= × 1000 × (100) 2 = 22.5
2 72 – 2q = q
= 5 × 106 J 38. (b) Frequency of B = 256 ± 4 \ q = 24

i.e. 260 ⇒ 252 Difference of temperature across layer
Total change in energy:
Since on loading A with wave, its A is:
= (2.5 + 5) × 106 frequency decreases and beat frequency also = 36 – 24 = 12°C
decreases, therefore, frequency of B is 252.
6
44. (c) Since light has to travel from
= 7.5 × 10 J
R2 denser to rarer medium so, it must be
Work = F × S 39. (b) Av = β .
R1 made incident in the denser medium at
= F × 500 = 7.5 × 106
10 × 103 = 1000 an angle less than critical angle.
F = 1.5 × 104 N = 100 ×
1 × 103 45. (d) T h e d e v i a t i o n o f r a y
36. (b) V ∝ λ a ρb . g c
by combination of plane mirrors is
or,
2E 0
V = k λ a ρb g c 40. (d) E av = independent of angle of incidence. It
π
(k = constant) depends only upon the angle between the
writing dimensions on both sides: 41. (a) y = a sin ωt plane mirrors.
[M 0 LT −1 ] = [L]a [ML−3 ]b [LT −2 ]c Here a=5m
MATHEMATICS
⇒   M 0 LT −1 = M b . La − 3b + c T −2 c and ω = 2π 46. (a) If y = x then x – y + 2 =
T
Comparing the dimensions of both
x − x + 2 = 2 , an irrational number.
sides 2π
= = 4π i.e., xRx, ∀ x ⇒ R is reflexive.
b =0 0.5
1 = a − 3b + c  2 R1 does not imply 1R 2
Required equation is:
−1 = − 2c ⇒ R is not symmetric.
y = 5 sin 4πt
1 1 Also R1 and 1R does not
c = ,a= 42. (c) Wires placed close to each other
Hence, 2 2 carry current in the same direction and imply R
1/2. 0. 1/2
V = kλ ρ g hence attract wires placed far apart carry Hence, R is not transitive.
62 Practice Set-5

z − z1 x + iy − 8 − 4i ⇒ (a + c)2 + 2b (a + c) = – 4ac = tan–1 tan 2φ = 2φ = 2 sin–1


47. (b) = Add b2 on both sides,
z − z2 x + iy − 6 − 4i dz 2
(a + c + b)2 = b2 – 4ac ⇒ = = 2 at x = 0
dx 1 − x2
( x − 8) + i ( y − 4) 49. (a) Given that, (n – 3), (4n – 2),
   =
( x − 6) + i ( y − 4) (5n + 1) are in A.P. dy dy dz 1/ 2 1
∴ (4n – 2) – (n – 3) = (5n + 1) – (4n – 2) = = =
 z − z1  π dx dx . dx 2 4
arg
 z − z  = 4 [Given] ⇒ 3n + 1 = n + 3 ⇒ 3n – n = 3 – 1
 1 
2 x
⇒ 2n = 2 53. (b) ∫e  x +  dx
2 x
 y − 4 –1 
y − 4 ∴ n=1
⇒ tan–1   – tan  
 x−8 x − 6 50. (a) Take a point (–1, –2) on the x 1 ex
line 3x – 4y – 5 = 0, which is at a unit
= ∫e x dx +
2∫ x
dx
π distance from its parallel line 3x – 4y +
=
4 λ = 0. x 1 1 ex
= e x −∫ e x dx +
2∫ x
dx + C
 1 1  | 3(−1) − 4(−2) + λ | 2 x
( y − 4)  − ∴ =1
⇒ tan–1  x − 8 x − 6  = π 9 + 16 x
( y − 4) 2 4 = e ( x) + C
1+ ⇒ |5+λ|=1
( x − 8)( x − 6) ⇒ 5+λ=+5 54. (c) Let A (a, 0) and B(0, b) are two
points on respective co-ordinate axes
2 ⇒ λ = 0 or – 10
( y − 4) and (–5, 4) divides AB in the ratio1: 2.
( x − 8)( x − 6) Hence, required equations are:

( x − 8)( x − 6) + ( y − 4) 2 3x – 4y = 0 and 3x – 4y – 10 = 0 1× 0 + 2 × a
∴ –5=
( x − 8)( x − 6) 3
 x3 − 3 x + 2
 , for x ≠ 1 −15 1× b + 2 × 0
= tan =1 51. (a) f (x) =  ( x − 1) 2 ⇒ a=
2
and 4 =
3

 k, for x = 1 ⇒ b = 12
⇒ x2 – 14x + 64 + y2 – 8y = 2y – 8 Since, the function is continuous Hence, equation of line joining
⇒ (x – 7)2 + (y2 – 10y + 25 ) = 2
∴ lim f ( x) = f (1)  15  and (0, 12) is:
 | z – 7 – 5i | = x →1  − , 0
2
k +1 k + 2 b x3 − 3 x + 2 12 − 0  15 
+ ⇒ k = lim
48. (a) We have, = − x →1 ( x − 1) 2 (y – 0) =
15 
x+ 
k k +1 a 2
and 0+
( x − 1) 2 ( x + 2) 2
= lim
k +1 k + 2 c x →1 ( x − 1) 2 4
. = ⇒ y= (2x + 15)
k k +1 a 5
⇒ k = (1 + 2) = k = 3
k+2 c ⇒ 5y = (8x + 60)
⇒ = 52. (b) On putting x = tan θ we have
k a ⇒ 8x – 5y + 60 = 0
2 c c−a θ θ 1 −1 55. (c) We know that,
⇒ = −1 = y = tan–1 tan
= = tan x
k a a 2 2 2
e x − e− x x 2 x 4 x6
dy 1 1 1 =1+ + + + .......∞
2a = . = at x = 0 2 2! 4! 6!
∴ k= dx 2 1 + x 2 2
c−a Putting ix on x, we get
Now, eliminate k. Again, putting x = sin φ, we get
Putting the value of k in Ist relation, we eix + e −ix x2 x4
 2sin φ cos φ  = 1− + + ......∞
get z = tan–1  2 2! 4!

c+a 2c b  1 − 2sin 2 φ 
+ = − 56. (b) We know that the equation of
2a c+a a circle, which touches both the axes is:
sin 2φ
⇒ (c + a)2 + 4ac = – 2b(a + c) = tan–1
cos 2φ x2 + y2 – 2rx – 2ry + r2 = 0 ....(i)
Practice Set-5 63

The centre (r, r)of this circle lies on the ⇒ f '(x) = –2ex(1 – x) A
b log x
line Now, exponential function is always 65. (a) Let I = ∫a x
dx
x+y=4 positive and the sign of f '(x) will be
∴ r+r=4 opposite to the sign of A which is negative dx
Put log x = t ⇒ = dt
⇒ r=2 x
 1  , Hence, f(1) is positive, so that
in − ,1
On putting the value of r in Eq. (i), we  2 

t2  1
b
b 2 b
get f (x) is an increasing function in this ∴ I = ∫ t dt =   = 2 [(log x) ]a
x2 + y2 – 4x – 4y + 4 = 0 a 2
  a
interval.
Which is required equation of circle. 61. (b) Put x = tan a 1 2 2
  = [(log b) − (log a ) ]
1  sec θ − 1 2
57. (a) 2 log8 2– log3 9 ∴ y = tan–1 
3  tan θ 
1
1  1 − cos θ    = [(log b + log a) (log b – log a)]
= 2 log23 2 – log 3 (3) 2 = tan–1  2
3  sin θ 
1  b
2 1 θ 1   = log (ab) log  
= log 2 2 – .2 log 3 3 y = tan tan = θ
–1 2  a
3 3 2 2
1
1 x
∴ loga b = (loga b.loga bn =
1
tan −1 x
66. (a) Let I = ∫0 x e dx
n 2 Using integration by parts
= n loga b log a = 1)
dy 1 1 1
⇒ = . = at x = 0 1 x
2 2
= (1) − (1) = − = 0
2 2 dx 2 1+ x 2 2 I = [x.ex]01 –
∫0 1. e dx = (1.e – 0) – ex]01
3 3 3 3
62. (a) = e – (e – 1) = e – e + 1 = 1
58. (c) Given expression can be
rewritten as: Class xi fi xi fi 67. (a) Let ∠A = 30°, ∠B = 45° and AB
logxyz xy + logxyz yz + logxyz zx 0–10 5 22 110 = 3 +1
  = logxyz (xy . yz . zx) = logxyz (x2y2z2) 10– 20 15 38 570 Then, ∠C = 180° – (∠A + ∠B)
   = 2 logxyz (xyz) = 2 × 1 = 2 (since, the sum of internal angles of a
20–30 25 46 1150
59. (a) Given equation is: triangle is 180°).
30–40 35 35 1225 ⇒ ∠C = 180° – (30° + 45°)
dy 1
= t (1 +y) ⇒ ∫ dy 40–50 45 20 900 = 180° – 75° = 105°
dt 1+ y
161 3955 By sine rule,
= ∫ t dt ⇒ log(1 + y)
sin 30° sin105°
   =
=
t2
Mean =
∑ fi xi =
3955 BC 3 +1
+C
2 ∑ fi 161
 2 2  1
 y (0) = 0 ⇒ BC = ( 3 + 1) ×   × = 2
= 24.56 (approx.)  3 + 1 2
∴ log 1 = C ⇒ C = 0
Median = 24.26 B
t2  (as solved in solution 1)
∴ log (1 + y) =
2 45° ( 3 + 1)
2
Mode = 3 (24.46) – 2 (24.56)
⇒ y = – 1 + et /t
= 24.26
60. (a) f '(x) = e x(1 – x). 1 + x.e x(1– x) 63. (d) ∴ Required number of ways
(1 – 2x)
= (3C1 × 2C1 × 1C1) ×2 105°
⇒ f '(x) = ex(1 – x) [1 + x – 2x2] 30°
C A
⇒ f '(x) = – ex(1 – x)[2x2 – x – 1] = 3 × 2 × 1 × 2 = 12
⇒ f '(x) = – ex(1 – x)(x – 1) (2x + 1) 64. (b) Let E be the event, then Again, now by sine rule

 1    P(E or not E) = P(E ∪ Ec) = 1 sin 45° sin105°


⇒ f '(x) = –2ex(1 – x)  x +  ( x − 1) =
 2    = P(E) + P(Ec) = 1 AC 3 +1
64 Practice Set-5

( 3 + 1) 2 2 REASONING AND GENERAL 3960 + 2790 + 4500


⇒ AC = × =2 =
2 ( 3 + 1) AWARENESS (RAGA) 180
11250
∴ Area of ∆ABC 71. (a) Fundamental Rights is a = = 62.5
charter of rights contained in Part III of 180
1 85
= × BC × AC × sin105° Constitution of India.
2 82. (a) S.P. of 1 item = 800 × =
72. (c) 73. (d) 74. (c) 75. (c) ` 680 100
1 ( 3 + 1) 3 +1 2
= ×2 2× = cm 76. (d) 100 – 38
2 2 2 2 S.P. of 4 items = 3200 ×
77. (a) United National Educational, 100
 1 Scientific ad Cultural Organizational
68. (c) tan–1   + tan–1  1  = 3200 ×
62
= 1984
 2  3 (UNESCO) 100
Explanation: On July 3, 2019,
 1 1 S.P. of 5 items = ` 4000
+ The first 2019 report of “State of the
= tan  2 3 
–1
education Report for India 2019: Children ∴ effective discount %
 1
 1−  with Disabilities” has released by the 4000 – (1984 + 680)
 6 UNESCO(United Nations Education, = × 100
4000
Scientific and Cultural Organization) at
 −1 −1 −1  x + y   an event in New Delhi. The Report, which 4000 – 2664
∵ tan x + tan y = tan   = × 100
  1 − xy   will be a annual publication, highlights 4000
accomplishments and challenges with = 33.4%
 5 6 regards to the right to education of 83. (a) Let the number be x
= tan–1  ×  = tan–1 (1) children with disabilities (CWDs).
 6 5 114 x
78. (a) Let the number be x x + 21 =
 π π 100
= tan–1  tan  = 9 7
  4 4 of of x = 126 114 x – 100 x
4 2 21 =
69. (a) The equation of any chord 100
which is normal, is 9 7 14x = 2100
× x = 126
y = mx – 2am – am3 4 2
2100 300
⇒ mx – y = 2am + am3 63 x = = = 150
x = 126 14 2
Let the equation of parabola is y2 = 8
4ax 126 × 8 14 × 8 84. (b) According to question
The straight lines joining the origin x= = = 16 H + ⇒ = 100 ⇒ H = 100 – W
63 7
to the intersection of these two, given by
7 7 H–10 9
the equation x = × 16 = 56 =
y2 (2am + am3) – 4ax (mx – y) = 0 2 2 W–10 7
If these be at right angles, then 7H – 70 = 9W – 90
79. (c) Other number
2am +am3 – 4am = 0 7(100 –W) –70 = 9W –90 ...(i)
( coefficient of y2 + coefficient  11 × 7700  From equation (i)
of x2 = 1) =   = 308
 275  700 – 7W – 70 = 9W – 90
⇒ m=+ 2 80. (a) 2x + y = 5 ...(i) 700 + 90 – 70 = 9W + 7W
and 3x – 4y = 2 ...(ii) 720 = 16W
⇒ tan θ = 2 ⇒ θ = tan–1 2 Multiplying (i) by 4 and adding (ii) W = 45
to it, we get : 11x = 22 or x = 2. Age of wife = 45 years
cot 54° tan 20° Putting x = 2 in (i) , we get : y = 1 H = 100 – W = 100 – 45
70. (c) +
tan 36° cot 70° So, 2xy = 2 × 2× 1 = 4. = 55 years
cot 54° tan 20° (x + y)2 = x2 + y2 + 2xy Age of husband (H) = 55 years
= + 4+1+4=9
tan(90° − 54°) cot(90° − 20°) 85. (b) Let Rajdhani take time = t
81. (b) Mean Marks hours
cot 54° tan 20° 60 × 66 + 45 × 62 + 75 × 60 Express train take time = (t + 6)
= + =1+1=2 =
cot 54° tan 20° 60 + 45 + 75 hours
Practice Set-5 65

Speed of Express train = v 88. (b) yfxfzy<x<z x > y > z.


When Speed of Express train = 2v
Then time taken = (t – 3) start 4 km end [∴ x <| z i.e., x > z]
5 km 5 km
Distance Therefore, x = y = z y f x f z.
8 km 8 km
Speed = 93. (d) Except the letters group
Time
4 km WIPB, all are in certain sequence as
732 
v =   km/h ...(i) N
follow.
t + 6
732  W E
2v =   km/h ...(ii)
t – 3
S
Dividing equation (i) by (ii),
Total distance = (5 km + 4 km + 5 km) =
v 732 (t – 3) t – 3
= × = 14 km
2v (t + 6) 732 t+6
Finally, Riya is 14 km far from her
1 t –3 original position.
=
2 t+6 89. (c) According to question ,
t + 6 = 2t – 6 Sanjeev > Nirbhay > Arpit > Rahul >
t = 6 + 6 = 12 hours Sameer
94. (a) According to dictionary, the
Speed of Rajdhani train Finally, Rahul got the 4th rank.
arrangement of given words is,
732 732 90. (c) The series is:
= = = 61 km/h (iv) Handmade, (ii) Horse,
t 12 j k l m / jk l / m/ j k l m (i) Hunger, (iii) Hungry.
86. (b) Golu's sibling's grandfather Finally, iv, ii, i, iii is the correct
 Golu's paternal grandfather ∴ the set of letters kmljm complete
sequence.
the series.
 [ Golu has a sibling.) 95. (d) 394 ⇒ 934; 632 ⇒ 362;
So, Paternal grandfather's only [Explanatory Notes:] According to the
given information, the circular arran-gement is
783 ⇒ 873; 576 ⇒ 756;
daughter  Golu's aunt
Therefore, only child of Golu's aunt as follows: Second Highest Number ⇒ 934 = 394
 Golu's cousin. 96. (c) According to statements,
87. (b) As, neither Conclusion (I) nor Conclusion
C L O U D (II) follows.
↓ ↓ ↓ ↓ ↓ 97. (c) According to statements,
5 9 4 3 2 neither Conclusion I nor Conclusion II
And follows.
91. (c) By analysing the circular
R A I N 98. (d) According to statements of
arrangement, it is clear that, Third to
the question, only Conclusion I follows.
↓ ↓ ↓ ↓ the left of ‘E’ = G.
99. (b) According to statements,
1 6 7 8 92. (d) According to the meaning of
Conclusion I and II both follows.
Similarly, the given symbols,
100. (c) According to statements,
A R O U N D x = y = z  x > y > z i.e., x > z. neither Conclusion I nor Conclusion II
↓ ↓ ↓ ↓ ↓ ↓ Using the proper symbols in option follows.
6 1 4 3 8 2 (d), we get qqq
66 Practice Set-6

Air Force Airmen Group X & Y Exam


Practice Set-6

ENGLISH 1. Adversity provides us with an 9. What is the antonym of ‘Manly’?


Directions (1-4): Read the passage opportunity to– (a) Feminine (b) Effeminate
given below and answer the questions (a) introspect (c) Masculine (d) Strong
by selecting the most appropriate (b) develop our character Directions (10-11) : In the following
option. (c) test our friends questions, some of the sentences have
Adversity provides us with an opportunity (d) evaluate our own character errors and some have none. Find
to develop our character in a natural, 2. The author quotes Solomon to– out which part of a sentence has an
recurring and powerful way that only the (a) lend force to his argument error. The number of that part is
challenges of adversity offer. According (b) show his veneration for him your answer. If there is no error, your
to Solomon, adversity refines and reveals (c) emphasise that adversity is part answer is (d).
the gold and silver of our character. of life 10. I found (a)/the two first chapters of
A lot of times adversity comes our (d) embellish his prose the book (b)/particularly interesting.
way as a direct or indirect result of our 3. Most often our misfortunes are (c)/No error (d).
own actions. We make a bad choice or the result of our own–
11. Bacon, the father of the English
a bad decision or we simply fail to do (a) idleness (b) haste
something we should have done. When essay (a)/had a thirst (b)/of
(c) follies (d) actions
I made bad investment decisions, I had knowledge. (c)/No error (d).
4. The synonym for ‘cautioned’ is–
to accept responsibility for my greed Directions (12): Choose the word
(a) warned (b) threatened
and naive choices. Yes, several men had (c) suggested (d) persuaded opposite in meaning to the given word.
misrepresented the opportunities to me 5. Which of the following sentences 12. In toto
but the fact is, I am the one who made is correct? (a) Bluntly (b) Partially
the decisions. And I experienced the (a) Tell me where do you live. (c) Entirely (d) Strongly
very consequences, that Solomon had (b) Tell me where you live. Directions (13): In the following
cautioned us about. Any time you make (c) Tell me you live where. question, four alternatives are given for
a contribution to your own adversity, (d) Tell me where you live. the underlined idiom/phrase. Choose
you need to accept responsibility for the alternative which best expresses
6. Fill in the blank with appropriate
it. Do not simply blame someone or the meaning of the underlined idiom/
gerund or infinitive :
something else. phrase.
“The shopkeeper managed
Nonetheless throughout our lives we
_________ the article that I was 13. The principal has to carry out
will experience a great deal of adversity
looking for.” the orders issued by the higher
that is not a result of our own actions.
(a) Finding (b) In finding authorities.
It is critically important that we do not
assign fault to ourselves or to those who (c) To find (d) To finding (a) obey (b) communicate
had nothing to do with it. When a friend 7. “My mother would never do (c) execute (d) modify
of mine lost his daughter to leukemia, anything that______against her Directions (14-15) : In the following
he confided to me that he felt God conscience.” questions, out of the four alternatives,
was punishing him for his past sins. In (a) would have gone choose the one which can be subs-
other words, he was blaming himself. (b) will go tituted for the given words/sentence.
It is believed that adversity sometimes (c) go 14. To be biased against
has a purpose that we cannot know or (d) went (a) Partial
understand. As tempting as it may be, to 8. Choose the correctly spelt word : (b) Objective
try to figure out such a mystery is not only (a) farenheit (b) ferenheit (c) Prejudiced
an exercise in futility, it is foolish also. (c) fahrenheit (d) fahrenheit (d) Predestined
Practice Set-6 67

15. Motion of head, hands etc., as a 20. Choose the option that expresses 25. A capillary tube is attached
mode of expression indicating the meaning of the given word : horizontally to a constant heat
attitude CONJECTURE arrangement. If the radius of
(a) Gesture (b) Grin (a) Introduction the capillary tube is increased
(c) Gestation (d) Grimace (b) Hesitation by 10%, then the rate of flow of
Directions (16-17) : In the following (c) To prevent liquid will change nearly by:
questions, four words are given in (d) Guess (a) +10% (b) +46%
each question, out of which only (c) –10% (d) –40%
one word is wrongly spelt. Find the PHYSICS 26. Figure shows the circular motion
wrongly spelt word. 21. With what minimum acceleration of a particle. The radius of
16. (a) Accomplice can a fireman slide down a rope the circle, the period, sense of
(b) Accompaniment revolution and the initial position
while breaking strength of the
(c) Accomplishment are indicated on the figure. The
(d) Accomodation rope is 2 of the weight?
3 simple harmonic motion of the
17. (a) Replaceable
2 x-projection of the radius vector
(b) Replaceing (a) g (b) g of the rotating particle p is:
(c) Replacement 3 y
(d) Replaced 1
Directions (18): In the following (c) g (d) Zero P(t = 0)
3
question, the first and the last part of 22. The block of mass M moving T = 30 s
the sentence are numbered 1 and 6.
The rest of the sentence is split into on the frictionless horizontal B
four parts and named P, Q, R and S. surface collides with the spring of
x
These four parts are not given in their spring constant k and compresses O
proper order. it by length L. The maximum
18. Read the parts and find out momentum of the block after
which of the four combinations collision is:
is correct. Then find the correct M
 2π t 
answer. (a) x(t ) = Bsin 
 30 
1. Early to bed, early to rise,
makes a man healthy, wealthy  πt 
(b) x(t ) = Bcos  
and wise. 2
kL  15 
P. But for the morning tea, I had (a) L
MK (b)
to wait for someone to get up 2M  πt π 
(c) x(t ) = Bsin  +
before me. 2  15 2 
Q. This saying inspired me to rise (c) Zero (d) ML
k  πt π 
early. 23. A particle performing uniform (d) x(t ) = Bcos  +
 15 2 
R. That day I was first to get up. circular motion has angular
S. One day I got up early in the momentum L. If its angular 27. A, B and C are three points in
morning. frequency is doubled and its a uniform electric field. The
6. One day I realised that it was kinetic energy halved, then the electric potential is:
a waste of time to get up early
new angular momentum is: B A
and wait for the morning tea. E
(a) QSRP (a) (b) 2L C
(b) QPRS
(c) PQRS (c) 4L (d) (a) maximum at A
(d) SPQR (b) maximum at B
19. Choose the option that is opposite 24. If the coefficient of performance
(c) maximum at C
in meaning to the given word : of a refrigerator is 5 and operates
(d) same at all the three points A, B
CONSENSUS at the room temperature (27°C),
and C
(a) Disagrement find the temperature inside the
28. Band spectrum is produced by:
(b) Agreement refrigerator:
(a) H (b) He
(c) Bold (a) –23°C (b) –20°C
(c) H2 (d) Na
(d) Quality (c) –15°C (d) –31°C
68 Practice Set-6

29. When a wire of uniform cross- 35. Assuming the earth to be a sphere (a) Repulsion between like poles
section a, length l and resistance of uniform mass density, how (b) Attraction between unlike poles
R is bent into a complete circle, much would body weight half way (c) Null points in the magnetic field
re s i s t a n c e b e t w e e n t w o o f down to the centre of earth if it of a bar magnet
diametrically opposite points weighed 250 N on the surface? (d) No existence of isolated
will be: (a) 150 N (b) 175 N magnetic poles
R (c) 125 N (d) 200 N 42. The conductivity of a semi-
(a) R (b) 36. In Fraunhofer diffraction the conductor increases with increase
4 8
centre of diffraction image is: in temperature because:
(c) 4R (d) R
(a) always bright (a) number density of free current
2
30. What determines the charge that (b) always dark carriers increases
flows through a circuit due to the (c) sometimes bright and sometimes (b) relaxation time increases
induced emf? dark (c) Both (a) and (b)
(a) The rate of change of magnetic (d) bright for large wavelength and (d) number density of current
flux dark for low wavelength carrirs increases, relaxation time
(b) The total change in magnetic 37. Avalanche breakdown is due to: decreases but effect of decrease in
flux (a) collision of minority charge relaxation time is much less than
(c) The initial magnetic flux carrier increase in number density
(d) The final magnetic flux (b) deflection layer thickness 43. The magentic moment has
31. Calculate the number of atoms in increase dimensions of:
39.4 g gold. Molar mass of gold is (c) DL thickness decreases (a) [LA] (b) [L2 A]
197 g/mol: (d) None of the above (c) [LT A] –1
(d) L2T–1A]
(a) 1.5×1023 atoms
38. Which of the following does not 44. A convex lens and a concave lens,
(b) 1.20×1023 atoms
(c) 2.50 × 1023 atoms depict the correct link between each having same focal length of
(d) None of these technology and Physics? 25 cm, are put in contact to form a
32. A cyclotron is used to obtain (a) Opticals fibres – total ternal combination of lenses. The power
a 2 meV beam of protons. The reflection of light (in dioptres) of the combination
alternating potential difference (b) Nuclear reactor –nuclear fusion is:
applied between the dees has a peak (c) Electron microsocope – wave (a) 25 (b) 50
value of 20 kV and its frequency is nature of electrons (c) infinite (d) zero
5 MHz. The intensity of magnetic (d) Electric generator – l a w s o f 45. The ratio of the mean value over
field to be applied for resonance electromagnetic induction half cycle to the r.m.s. value of an
will be: 39. In the Davisson – Germer AC of:
(a) 0.0327 T (b) 0.327 T
experiment if the incident beam 2 2 :π
(c) 3.27 T (d) None of these (a) 2 :π (b)
consists of electrons, then the
33. Electric potential is: 2 :π 2 :1
(a) scalar and dimensionless diffracted beam consists of: (c) (d)
quantity (a) protons (b) neutrons
(b) vector and dimensionless (c) α-particles (d) electrons MATHEMATICS
quantity 40. A body initially at rest is moving 2
(c) scalar and dimensional quantity with uniform acceleration a m/ 46.  2 i  is equal to:
1+ i 
(d) vector and dimensional quantity s2. Its velocity after n s is V. The
34. A s m a l l m a g n e t o f d i p o l e displacement of the body in last 2 (a) 1 (b) 2i
moment M is kept on the arm s is: (c) 1 – i (d) 1 – 2i
of a deflection magnetometer set 2v(n − 1) v(n − 1) 47. A relation R in the set of integers
in tan A position at a distance of (a) (b) defined as ‘divides’, then which of
0.2 m. If the deflection is 60°, the n n
the following is true?
value of M is (BH = 0.4 × 10–4 T) v(n + 1) 2v(2n + 1) (a) Reflexive and symmetric
(c) (d)
(a) 2. 77 Am2 n n (b) Reflexive and transitive
(b) zero (c) Symmetric and transitive
41. Which of the following shows that
(c) 10.82 Am2 (d) Equivalence Relation
the Earth behaves as a magnet?
(d) None of the above
Practice Set-6 69

48. The sum of n arithmetic mean 2n ! (a) 9 (b) 6


(c) (c) 4 (d) 36
terms between y and 3y will be: (n + 3) ! (n − 1) !
(a) ny (b) 2ny 62. If a point A is on the line 3x + 5y =
2n ! 15 and it is equidistant from both
(c) 3ny (d) 4ny (d)
(n − 2) ! (n + 2) ! the coordinate axes, then it will be
49. What will be the fourth term
55. What is the value of sin 60° cos 30° situated in:
from the last of that geometric
+ cos 150° sin 120°? (a) Second quadrant
progression whose last term is 512
3 (b) Third quadrant
and common-ratio is 2? (a) 0 (b)
2 (c) First quadrant
(a) 16 (b) 32
(d) Nothing can be said
(c) 64 (d) 144 (c) 1 (d) − 3 63. Solution of the differential
50. How many words may be formed 2 equation
from the word VOWELS which 1 3 1
56. If x + = 2 cos θ , then x + 3 (x2y – 2xy2) dx – (x3 – 3x2y)dy = 0 is:
start from letter O and end with x x
equal to: x y2
letter L? (a) + log 3 = C
(a) cos 3θ (b) 2 cos 3θ y x
(a) 24 (b) 12
1 1 x y
(c) 48 (d) 144 cos 3θ cos 3θ (b) + log = C
51. If α, β are the roots of x2 – px + q (c) 2 (d) 3 y x
57. The equation of the circle, which
= 0, then α2 + β2 is: x y3
passes through the points (2, 3) (c) + log 2 = C
(a) p2 + 2q (b) p2 – 2q y x
and (4, 5) and of which the centre
(c) p(p – 3q)
2
(d) p2 – 4q 3
52. The system
lies on the straight line y – 4x + 3 (d) xy + log y = C
= 0 is: x 2
x + 2y 6  =  3  .[− 1, 2] (a) x2 + y2 + 4x – 10y + 25 = 0
2 x − y  0  has the 64. If tan x + tan–1 y + tan–1 z = π,
–1
 0 (b) x2 + y2 – 4x – 10y + 25 = 0 1 1 1
solution: (c) x2 + y2 – 4x – 10y + 16 = 0 then + + is equal to:
xy yz zx
(d) x2 + y2 – 14y + 8 = 0
(a) (a) xyz (b) 0
58. I f a r e a o f a ∆ A B C i s 8
square unit, then the value of (c) 1 (d)
(b) r r1 r2 r3 will be:
(a) 36 (b) 49
(c) 64 (d) 144 65. lim x→ 0 1 + x − 1 − x is given
(c) by: x
59. The rainfall recorded in a city for
(a) 0 (b) – 1
the ten year period 1981-1990 is
(d) as follows (in cm): (c) 1 (d) 1
89, 95, 70, 102, 29, 79, 63, 85, 72, 2
53. The conjugate of is: 50. 66. If xa yb = (x – y)a + b, then dy is
The mean rainfall is: given by: dx
12 − 5i − 12 + 5i

(a) (b) (a) More than 100 a b
169 169 (a) (b)
(b) Less than 100 b a
12 + 5i
(c) − 12 − 5i (d) (c) Between 95 and 100
x
169 169 (d) Less than 40 (c) (d) y
y x
54. The value of C0C2 + C1C3 + C2C4 60. The straight line lx + my + n = 0
is a tangent to the parabola y2 = 4 π /2
+ ... Cn–2 Cn will be: 67. The value of ∫− π / 2 cos x dx is:
bx if:
2n ! (a) mn = bl2 (b) lm = bx2 (a) 1 (b) 2
(a)
(n + 1) ! (n − 1) ! (c) ln = bm 2
(d) bm = ln2 (c) 2 (d) 0
61. The difference of the focal
2n ! 68. ∫ ( x + 1) 2 x 2 + x − 3 dx is equal
(b) distances of any point on the
(n + 2) ! (n + 1) ! to:
hyperbola 4x2 – 9y2 = 36, is:
70 Practice Set-6

(c) Rig Vedic Age (a) 6 (b) 7


(a) 1 (2 x 2 + x − 3)3/2 +
4 (d) Age of Mauryas (c) 10 (d) 20
1 75 4x + 1 74. The common settlement type of 81. In the first 43 overs of a cricket
(4 x + 1) 2 x 2 + x − 3 − cosh −1 match, the run rate was 5 runs/
16 64 2 5 mountains are:
(a) Linear-clustered over. Calculate the required run
(b) 1 (2 x 2 + x − 3)3/ 2 + 3 (4 x + 1) rate in the remaining 7 overs to
6 32 (b) Dispersed
reach the target of 285 runs.
(c) Semi-clustered (a) 10.6 (b) 11.2
75 4x + 1
2 x2 + x − 3 − cosh −1 (d) Agglomerated (c) 10 (d) 9.4
64 2 5
75. Which one of the following is an 82. If the sum of the measures of all
1 1 antidiuretic hormone? the interior angles of polygon is
(c) (2 x 2 + x − 3)3/ 2 + (4 x + 1)
6 16 (a) Cortisone 1800º, find the number of sides of

75 4x + 1 (b) Vasopressin the polygon.
2 x2 + x − 3 − cosh −1 (c) Oxytocin (a) 12 (b) 14
32 2 5
(d) ACTH (c) 16 (d) 8
(d) None of these 76. A computer used by people who 83. The total cost of a microwave
69. If mean of 7, 3, a and 8 be 6, what travel from one place to another is: oven with mixer was ` 36750.
is the value of a? (a) Laptop The mixer was sold at a profit
(a) 5 (b) Desktop of 32% and the microwave oven
(b) 8 (c) Supercomputer at a loss of 22%. If the sale price
(c) 7 (d) Work Station was the same in both the items,
(d) 6
77. Which organisation has released, then the cost price of the cheaper
70. What will be the standard 2nd edition of “Healthy States, item was ________
deviation of the set of numbers 1, Progressive India” report which (a) ` 13100 (b) ` 18375
4, 7, ... 16, 19? ranks states and union territories (c) ` 13650 (d) ` 16850
(a) 10 based on health outcomes, as well 84. The ratio of present ages of
(b) + 6 as, their overall performance? Ramaya and Saurabh is 8:7.
(c) 6 (a) National Development After 10 years the ratio of their
(d) 36 Council ages will be 12:11. What is
(b) National Integration Council Ramaya's present age?
REASONING AND GENERAL (a) 20 (b) 14
(c) National Institution for
AWARENESS (RAGA) Transforming India (NITI) (c) 63 (d) 45
71. If both demand for a commodity Aayog 85. To cover a distance of 315 km
and the supply of the commodity (d) Planning Commission in 2.8 hours what should be
increase, equilibrium price will: 78. The ten's digit of a 2 digit the average speed of the car in
(a) Decrease number is greater than the units metres/second ?
digit by 4. If we subtract 36 from (a) 112.5 m/s (b) 56.25 m/s
(b) Remain the same
the number, the new number (c) 62.5 m/s (d) 31.25 m/s
(c) Increase
obtained is a number formed by 86. Pointing towards a girl, a
(d) Any of the above
interchange of the digits. Find professor said, "She is the only
72. Preamble is:
the number. daughter of the only son of the
(a) Depends upon the circumsta- wife of the father-in-law of my
nces (a) 37 (b) 18
wife. "How is the girl related
(b) Justiciable (c) 81 (d) 73
with the professor?
(c) Non-Justiciable 79. Three numbers are in the ratio
(a) Daughter-in-law
(d) Enforceable by Executive of 5 : 6 : 7 and their L.C.M. is
(b) Daughter
73. The oldest inscriptions 360. Their H.C.F. is :
(c) Niece
deciphered so far are those which (a) 20 (b) 80
(d) Aunt
belong to the: (c) 120 (d) 200 87. In a code language the following
(a) Age of Kushans 80. If 2p + 3q = 18 and 2p – q = 2, alphabet are coded in a particular
(b) Harappan Civilization then 2p + q = ? way:
Practice Set-6 71

A B C D EM N O S R U 92. Arrange the given words in the Conclusions:


λ P / sequence in which they occur in (I) If I determines to achieve
Which word can be decoded the dictionary. three goals in near future,
from the following? i. Misleader ii. Monarchy my energy will get divided
/ into three parts and I can
iii. Mobil iv. Manage
(a) BOUND (b) BONUS never succeed.
v. Movies (II) Making a prior plan for
(c) BUNCH (d) BOARD (a) i, iv, iii, ii, v (b) iv, i, iii, v, ii achieving those three things
88. Amar moved 20 km towards east
(c) i, iii, ii, v, iv (d) iv, i, iii, ii, v I can get near to all my goals.
in car from his home to reach
Direction (93): In the following ques- (a) Only Conclusion II follows
hospital. From there he turned tions, find out the odd word/letter/ (b) Conclusion I and II both
left and moved 30 km and again number from the given alternatives. follows
moved 20 km left to reach his 93. (a) 25 (b) 36 (c) Neither I nor II follows
school. How far is his school (c) 78 (d) 144 (d) Only Conclusion I follows
from hospital? Direction (94): Arrange the given 97. Statements:
(a) 25 km (b) 30 km words in the sequence in which they (I) Children play in the
(c) 27 km (d) 35 km occur in the dictionary. playground.
89. There are five boys Prem, Raju, 94. i. Carrier ii. Capability (II) It gives them a different
Sundar, Hari and Ompal. Raju iii. Cause iv. Course approach, solving and
has more share of land than decision making.
(a) ii, i, iii, iv (b) i, ii, iii, iv
Sundar but lesser than Prem. Conclusions:
(c) ii, iii, i, iv (d) ii, iv, i, iii
Ompal has the least share and (I) Playing outdoor is very
Direction (95): The follow­ ing ques-
Hari has lesser share of land than tions are based on the five three digit necessary for overall
Sundar. Who owns the highest numbers given below: development of child.
share of land? 394  632  783  895 (II) There should be balance
(a) Sundar (b) Raju 95. If the first and second digits of between playing and
(c) Hari (d) Prem each of the numbers are added, studying for a child.
Direction (90): A series is given with the resulting sum of which of the (a) Only Conclusion II follow
one term missing. Choose the correct following numbers will not be (b) Conclusion I and II both
alternative from the given ones that exactly divisible by 3? follows
will complete the series. (a) 895 (b) 394 (c) Neither I nor II follows
90. CD, GH, KL, ? (c) 783 (d) 632 (d) Only Conclusion I follows
(a) PN (b) NO Directions (96-100): In the following 98. Statements:
(c) NP (d) OP question, two statements are given (I) India's GDP continues to
each followed by two Conclusions I grow at one of the reasonable
Direction (91): Study the following
and II. You have to consider the state- rates but job creation is not
information carefully and answer the
ments to be true even if they seem to picking up the similar pace.
questions given below: be at variance from commonly known (II) Every year over 20 million
A, B, C, D, E, F, G and H are sitting facts. You have to decide which of the young Indians join the
around a circle facing the centre. H is given conclusions, if any, follows from workforce.
fourth to the left of B and second to the given statements. Conclusions:
the right of F. A is third to the left of 96. Statements: (I) Population increase is more
C, who is not an immediate neighbour (I) Determination is the key than the job creation rate.
of F. G is second to the left of A. D is to success. if a person (II) Supply basket is less and the
second to the right of E. determines to achieve a demand basket for jobs is
91. Who is to the immediate right of certain goal in his or her very high.
F? life, no one can stop him/her
(a) Only Conclusion II follows
from achieving it.
(a) H (b) Conclusion I and II both
(II) Prioritising various
(b) A follows
determined goals is another
(c) G (c) Neither I nor II follows
important aspect of decision
(d) Data inadequate making. (d) Only Conclusion I follows
72 Practice Set-6

99. Statements: (a) Only Conclusion II follows Statement I: All scissors are
(I) Retail shops increasingly (b) Conclusion I and II both knives.
want to certain potential follows Statement II: Some blades are
buyers. (c) Neither I nor II follows scissors.
(II) 10% of the customers are lost (d) Only Conclusion I follows Conclusion I: Some knives are
due to out of stock products. 100. In the question two statements blades.
Conclusions: are given, followed by two Conclusion II: All blades are
(I) People don't find their sizes conclusions, I and II. You have to knives.
or specific colour in stock consider the statements to be true (a) Only conclusion I follows
and then they leave the store even if it seems to be at variance (b) Only conclusion II follows
without any purchase. from commonly known facts. (c) Both conclusions I and II
(II) Retailers want maximum You have to decide which of the follows
footfall to get converted into given conclusions, if any, follows (d) Neither conclusion I nor
purchases. from the given statements. conclusion II follows

Answers with Explanation


ENGLISH I have read the three first chapters. (×) 2 2g 1
= m( g − a ) = mg ⇒ a = g − = g
1. (b) 2. (a) 3. (d) 4. (a) I have read the first three chapters. () 3 3 3

5. (d)‘Tell me where you live’ is the 11. (c)Preposition ‘for’ is used with 22. (a) According to conservation of
correct sentence. the word ‘thirst’. Hence, replace ‘of energy,
knowledge’ by ‘for knowledge’. 1 2 1
6. (c) ‘To explain’ is appropriate kL = Mv 2
infinitive to fill in the blank, In traditional 12. (b)The meaning of the word In toto 2 2

description of English, the infinitive is (Adverb) is : completely; including all
(Mv) 2
the basic dictionary from of a verb used parts. ⇒ kL2 = ⇒ MkL2 = ρ2 (ρ = Mv)
M
non-finitely, with or whithout ‘To’. So The meaning of the word Partially ∴ ρ = L Mk
correct sentence is—“The shop keeper (Adverb) is : Partly; not completely.
So, just after collision momentum of
managed to find the article that I was Look at the sentences :
block (maximum) would be L Mk .
looking for”. Hence option (c) is correct. The road was partially blocked by a
7. (d)Appropriate from of verb-went fallen tree. 23. (a) Angular momentum, L = lω
is suitable. When Principal Clause is (i) Kinetic energy,
Hence, the words in toto and partially
in Past Tense Sub. Ordinate Clause are antonymous. 1 2 1
KE = lω = Lω [From Eq. (i )]
also would be in past tense. So correct 13. (c) 14. (c) 15. (a) 2 2
sentence—“My mother would never 2KE
16. (d)The correct spelling is— ∴ L =
do anything that went against her ω
accommodation.
conscience.” Hence option (d) is correct.  KE 
17. (b)The correct spelling is— 2
8. (c)Correct spelling is fahrenheit.
replacing.  2  L
Now, L ' = ⇒ L' =
9. (b)Antonym of ‘Manly’ is 2ω 4
18. (a) 19. (a) 20. (d)
Effeminate, weak or cowardly. 24. (a)
10. (b)Replace group of words ‘the PHYSICS
Q2
two first chapters of the book’ by ‘the 21. (c) If man slides down with same = 5 ⇒ Q 2 = 5W and Q1 = 6W
W
first two chapters of the book.’ When acceleration. Then, its apparent weight
T2 5 T
cardirial and ordinal Adjectives are used decreases. For critical condition rope = = , T2 = 250 K = − 23°C
2 T1 6 300
before a Noun in a sentence, then we can bear only of his weight. If a
should use Ordinal Adjective first and 3 25. (b) Volume of liquid flouring
then the Cardinal Adjective. is minimum acceleration. Breaking through capillary per second is given by
Look at the sentences : strength: poiseuille’s formulae as:
Practice Set-6 73

4 30. (b) If the circuit of conductor is Acceleration due to gravity at depth


πρr 4 V r 
V = ⇒ 2 =  2 complete h from Earth’s surface:
8 ηl V1  r1 
dφ  h
4 then e = g' = g 1 −
 110  dt 
∴ V2 = V1  = V1 (1.1) 4 = 1.4641V  R
 100  e

dq 1 d φ Re
∆V V2 − V1 1.4641V – V and i = =
∴ = = dt R dt Here, h =
V V V 2
= 0.46 or 46%
Hence dq = φ2 − φ1  Re 
26. (a) Given T = 30 s, OQ = B. R    1
g ' = g 1 − 2  = g 1 − 
The projection of the radius vector Therefore total charge that flows R  2
 e 
on the diameter of the circle when a through AB depend upon total change of  
particle is moving with uniform angular flux and resistance of the circuit. g
g' =
velocity (ω) on a circle of reference is 31. (b) M o l a r m a s s = M a s s o f 2
SHM. Let the particle go from P to Q Avogadro’s number of atoms (molecules) Weight of the body at depth h
in time t. = 6.023 × 1023 atoms mg
Given, molar mass of gold w ' = mg ' =
Then ∠ POQ = ωt = ∠ OQR. The 2
projection of radius OQ on. x-axis will = 197 g/mol
now 250
be OR = x(t) say. Using Eq. (i), we get w ' = = 125 N
∵ 197 g of gold contains 2
x(t )
In ∆ OQR, sinωt = = 6.023×1023 atoms Weight of the body will be 125 N.
B
27. (b) The electric field is maximum at ∴ 1 g of gold contain 36. (a) In Fraunhofer diffraction the
B, because electric field is directed 6.023 × 1023 centre of diffraction image is always
= atoms
along decreasing potential. Hence 197 bright.
VB > VC > VA ∴ 39.4 g of gold contains 37. (a) Avalanche breakdown is due
28. (c) Molecules give band spectrum, to collision of minority carriers with
6.023 × 1023 × 39.4
since, H2 is a molecule it gives band = semiconductor atoms, so that they get
197 ionised and more free electrons are
spectrum. = 1.20 × 1023 atoms available for carrying current
29. (a) When wire is bent to form a Bq 2πn
n= ⇒B= 38. (d) (i) Optical fibre work on the
complete circle, then: 32. (b) 2πm q principle of total internal reflection.
R  
2πr = R ⇒ r = m (ii) A nuclear reactor is a device
2π in which nuclear fission can be carried
2 × 3.14 × 5 × 106
Resistance of each semi-circle = = 0.327 Wbm –2 out through a sustained and a controlled
96 × 106
πR R chain reaction.
= πr = = = 0.327 T (iii) An electron microscope is a
2π 2
33. (c) Electric potential is a scalar and form of microscope that uses a beam of
R/2 electrons instead of beam of light to form
dimensional quantity.
34. (a) B = BH tanθ a large image of a very small object. It is
A B based on wave nature of light.
µo 2M (iv) An electric generator is a
= 0.4 × 10−4 tan 60°
4π r 3 device used to convert mechanical
R/2 µ0 M energy to electrical energy. It is based
= 0.4 × 10−4 tan 60°
Thus, net resistance in parallel 2π(0.2)3 on the principle of electromagnetic
combination of two semi-circular induction. When a coil is rotated about
0.4 × 2π × (0.2)3 × (10−4 ) × 3 an axis perpendicular to the direction of
resistances: ⇒M =
µ0 uniform magnetic field, an induced emf
R R R2 = 2.77 Am2 is produced across it.
× R 35. (c) Weight of the body at Earth’s 39. (d) I n D a v i s s o n – G e r m e r
R' = 2 2 = 4 =
R R R 4 surface experiment working on the scattering
+
2 2 w = mg = 250 N ..(i) basis, if the incident beam consists
74 Practice Set-6

of electrons, then the diffracted beam 51. (b) α + β= p, αβ = q


4i 2
consists of electrons. = [  i2 = – 1] = 2i ∴ α2 + β2= (α + β)2 – 2αβ
2i
40. (a) Displacement in the last = p2 – 2q
1 1 47. (b) Let Z be the set of integers
2 s = an 2 − a (n − 2) 2 52. (b) Given that
2 2 (i) Since, every integer divides
itself. x + 2y 6  3
= 2a (n − 1)  0 ⋅ [− 1 2]
Hence, R is reflexive i.e., xRx, ∀ x ∈ Z.  2 x − y  0 
=
Acceleration (ii) If integer x divides to integer x + 2y 6  3.( −1) 3.2 
v y, then y does not divide to x, hence R is  0  0.( −1) 0.2
a= (t = ns ) 2x − y
n not symmetric i.e., ⇒  =  

Displacement in the last xRy and yRx, ∀ x, y ∈ Z. x + 2y 6   − 3 6


(iii) If x divides to y and y divides to  0 2x − y  0 0
2v (n − 1) ⇒  =  
2s = z, then x divides to z, hence R is transitive
n ⇒ x + 2y = – 3
i.e.,
41. (c) Null points in the magnetic xRy but yRx ⇒ xRz, ∀ x, y, z ∈ Z. and 2x – y = 0
field of bar magnet shows that the earth Thus, R is Reflexive and Transitive. Solving these equations, we get
behaves as a magnet. 48. (b) According to questions, x = – 3/5 and y = – 6/5.
42. (d) As temperature increases, (n + 2)th term = 3y 1 1
increase in number density of current ∴ 3y = y + (n + 1)d 53. (d) Conjugate of =
12 + 5i 12 − 5i
carriers is dominant, hence conductivity 2y
increases. ∴ d = (12 + 5i )
n +1 =
43. (b) Magnetic moment M = IA So, first arithmetic mean term (12 − 5i )(12 + 5i )
Thus, dimensions of M 2y 12 + 5i 12 + 5i
= [A][L2] = [L2A] = y+ = =
n +1 144 + 125 169
44. (d) Focal length of combination of Sum of n arithmetic mean term between
lenses placed in contact is: y and 3y 54. (d) (1 + x)n = C0 + C1x + C2x2 + ...
+ Cr xr + ... + Cnxn
1 1 1 n  2y  2y 
= +  2 ⋅ 2  y + n + 1 + (n − 1) ⋅ n + 1  1
F f1 f2 =  Putting in this   in place of x.
 x
For convex lens, f1=25cm  2 (n − 1) 
ny 1 + + n
For concave lens, f2= –25cm =  n + 1 n + 1   1
1 + 
Hence, 1 = 1 + 1 = 1 − 1 = 0 x
 n + 1 + 2 + n − 1
F 25 −25 25 25 ny  
=  n +1 C1 C2 C C
C0 + + 2 + ... + rr + ... + nn
1
 2(n + 1)  = x x x x
∴ F= 0 =∞ ny  = 2ny
 (n + 1)  On multiplying both the expansion
=
1  C1 C2 C C 
Hence, power of combination, p = =0 49. (c) Common ratio of series from  C0 + + + ... + rr + ... + nn 
F x x2 x x 
the last will be
1
2IQ I0
lm = and I ms = r × (C0 + C1x + C2x2 + ... + Crxr + ... + Cnxn)
π 2 Fourth term from the last n
45. (b)  1 n
Im 2 1 +  × (1 + x)
Hence, =2 = = x
I ms π
1
(1 + x) 2 n
MATHEMATICS = = 64 xn
= ...(i)
2
4i 2 On equating coefficient of x in both the2
46. (b)  2i  = 50. (a) The positions of letter O and
1+ i 1 + 2i + i 2 sides of equation (i)
L are fixed therefore, the method of
4 × ( −1) −4 arrangement of only remaining four C0C2 + C1C3 + C2C4 + ... + Cn – 2.Cn
= =
1 + 2i − 1 2i letters is 4P4 = 4 ! = 24. = Coefficient of x 2 in expansion of
Practice Set-6 75

1 ∆ ∆ ∆ ∆ ∂M
n
(1 + x) 2 n ∴ rr1r2r3 = ⋅ ⋅ ⋅ = x2 – 4xy
x s s−a s−b s−c ∂y
= Coefficient of xn + 2 in expansion of ∆4 ∂N
(1 + x)2n = = ∆ 2 = (8)2 = 64 = – 3x2 + 6xy
∆ 2 ∂x
(2n) !
= 2nCn + 2 = 59. (b) The mean rainfall
(n − 2) !.(n + 2) ! ∂M ∂N
89 + 95 + 70 + 102 + 29 + 79 Since, ≠
∂y ∂x
55. (a) sin 60° cos 30° + cos 150° sin = + 63 + 85 + 72 + 50
⇒ The equation is not exact.
120° 10
The given equation is homogeneous and
= sin 60° cos 30° + cos (90° + 60°) sin = 73·4 = less than 100
Mx + Ny = (x3y – 2x2y2) + (– x3y + 3x2y2)
(90° + 30°) 60. (c)  lx + my + n = 0
= x2y2 ≠ 0
= sin 60° cos 30° – sin 60° · cos 30° = 0 ⇒ y = (– l/m)x – (n/m)
1 1
1 y = mx + c is the tangent of y2 = 4 ax ∴ I.F. = = 2 2
56. (b)  x + = 2 cos θ Mx + Ny x y
x a
if c = Multiplying the given equation by
3 m
 1 ∴ lx + my + n will be tangent to y2
∴  x +  = 8 cos θ
3
1
x , we get
= 4bx, if 2 2
x y
3 1  1
⇒ x + 3 + 3  x +  = 8 cos3 θ −n
b
x x
= l  x 2 y 2 xy 2   x3 3x 2 y 
m −  2 2 − 2 2  dx −  2 2 − 2 2  dy
1 m x y x y  x y x y 
⇒ x3 + + 3 × 2 cos θ = 8 cos3 θ
x3 −n −bm
⇒ = =0
m l
3 1  1 2  x 3
⇒ x + = 8 cos3 θ – 6 cos θ ⇒ ln = bm2.
x3 ⇒  y − x  dx −  2 − y  dy = 0
61. (b) Equation of the hyperbola is y 
= 2(4 cos3 θ – 3 cos θ) 4x2 – 9y2 = 36 which is an exact equation. Its solution is
= 2 cos 3θ  1 2 3
57. (b) Let the coordinates of the ⇒
x2 y 2
− =1 ∫  y − x  dx + ∫ y dy = C
9 4
centre O be (h, k).
x
∴ a2 = 9 ⇒ 2a = 6. ⇒ − 2 log x + 3log y = C
y
Since, the difference of the focal distances
B(4, 5)
A of any point on the hyperbola = 2a x y3
⇒ + log 2 = C
∴ Reqd. distance = 6. y x
O
(3, 3)

62. (c) Let the coordinates of the reqd. 64. (c) tan–1 x + tan–1 y + tan–1 z = π
y–

point be (h, k)  x + y + z − xyz 


4x

∴ tan −1   =π
+

Since, the point is on the line 3x + 5y = 15 1 − xy − yz − zx 


3=

∴ 3h + 5k = 15 ...(i)
0

∴ k – 4h + 3 = 0 ...(i) x + y + z − xyz
 The point (h, k) is equidistant from ⇒ = tan π = 0
and (h – 2)2 + (k – 3)2= (h – 4)2 + (k – 5)2 1 − xy − yz − zx
the axes.
⇒– 4h + 4 – 6k + 9= – 8h + 16 – 10k + 25 ∴ x + y + z = xyz
h = k or, h – k = 0 ...(ii)
⇒k + h – 7 = 0 ...(ii) 1 1 1
On solving the eqns. (i) and (ii), we get ⇒ + + =1
On solving equations (i) and (ii), we get yz zx xy
h = 2 and k = 5 and radius = 2 15 15
h= and k =
∴ Equation of the circle will be: 8 8 65. (c) lim x→0 1 + x − 1 − x
(x – 2)2 + (y – 5)2 = 4 ∴ The point is situated in first quadrant. x
⇒ x + y2 – 4x – 10y + 25 = 0
2 63. (c) Here, M = x2y – 2xy2 1+ x − 1− x
= lim x→0
58. (c) Area of triangle ABC, s = 8 and N = – x3 + 3x2y x
76 Practice Set-6

 1+ x + 1− x  1 3 2  1 3 1
2 = 3× 4 = ±6
×
 1 + x + 1 − x  4 ∫ t dt +
4 ∫  x +  − − dx
4 2 16
REASONING AND GENERAL
(1 + x) − (1 − x) In first part, put t = 2x2 + x – 3 AWARENESS (RAGA)
= lim x→0
x( 1 + x + 1 − x ) 1 t 3/ 2 3 2 1
2 2
71. (b)
  5
4 ∫ 
= +  x +  −   dx
2x 4 3 4 4 y
= lim x→0 D' S
x( 1 + x + 1 − x ) 1 D S'
= (2 x 2 + x − 3)3/ 2 E
2 6 P E'
=
1+ 0 + 1− 0 3 2 1  1 
2
1  5
2 S
+ ⋅ x+  x+  −  D'
2 4 2  4  4  4 S' D
= =1
1+1
2 x
66. (d) Take ‘log’ of both sides  5 4 x + 1 O q q1
−   cosh −1 
 4 5 
a log x + b log y = (a + b) log (x – y) As a result or increase in de-mand
1 3 2  4x + 1 2 1 3
and supply the demand and supply
= (2 x 2 + x − 3)3/ 2 +  x + x− curves will shift to righward resulting
6 8  4 2 2
into the increase in demand. However,
dy  b a + b  a+b a 25 4 x + 1 equilibrium price OP is the same.
+ = − − cosh −1
dx  y x − y  x− y x 16 5  72. (c)
dy  bx − by + ay + by  1 3 73. (d) The earliest inscriptions
(2 x 2 + x − 3)3/ 2 + (4 x + 1) 2 x 2 + x − 3 are found on the seals of Harappa
dx  y( x − y) 
 6 32
belonging to about 2500 BC and written
ax + bx − ax + ay 75 4x + 1
− cosh −1 in pictographic script but they have not
= 64 2 5
( x − y) x been deciphered. The oldest inscription
7 + 3+ a +8 deciphered so far was issued by Ashoka
dy y 69. (d) =6
= 4 in 300 BC. These were first deciphered
dx x by James Prince in 1837.
π/ 2 π/ 2
∴ 18 + a = 24
74. (b) 75. (b) 76. (a)
67. (b) ∫−π/ 2 cos x dx = 2∫0 cos x dx ∴ a = 24 – 18 = 6
77. (c) National Institution for
70. (b) d = 10 and i = 3
because cos x is an even function Transforming India (NITI) Aayog
2
π/ 2 x−d  x−d
= 2 (sin x)0 x x−d   Explanation: On June 25, 2019, The
= 2 (sin π/2 – sin 0) i i  National Institution for Transforming
= 2 (1 – 0) = 2 1 −9 −3 9 India (NITI) Aayog released the Second
4 −6 −2 4 Edition of “Healthy States, Progressive
68. (b) Here, d (2 x 2 + x − 3) India” Report. It ranked States and
dx 7 −3 −1 1
Union territories (UTs) innovatively on
= 4x + 1 10 0 0 0 their year-on-year incremental change
1 3 13 3 1 1 in health outcomes and their overall
∴ x + 1 = (4 x + 1) +
4 4 16 6 2 4 performance.
19 9 3 9 78. (d) Let two digit number be
Now, ∫ ( x + 1) 2 x 2 + x − 3 dx
Total 0 28
10 x + y
According to question
= ∫ { 1
4
(4 x + 1) +
3
4 } 2 x 2 + x − 3 dx ∴ S.D.
2
In case I,
x=y+4
 x−d   x −d
1 Σ − Σ 
= ∫ (4 x + 1) 2 x 2 + x − 3 dx =  i    i 
 x–y=4
4 i In case II
n
3 2 1 3 10x + y – 36 = 10y + x
28 − 0
4 ∫
+ x 2 + x − dx = 3 a(x – y) = 36
2 2 7
Practice Set-6 77

x–y=4 84. (a) Let ages of Ramaya and 90. (d)


By solving, x = 7, and y = 3 Saurabh
+4 +4 +4
We get the number = 10 × 7 + 3    = 8x and 7x C  → G  → K  →O
= 73 According to question, +4
D  +4
→ H  +4
→ L  →P
79. (a) Let the numbers be 5x, and 8 x + 10 12 Finally, the missing set of letters
6x and 7x. Then, their L.C.M. = 18x, So. =
7 x + 10 11 is OP.
18x = 3600 or x = 20
88x + 110 = 84x + 120 91. (b) By analysing the circular
∴ The numbers are (5 × 20), 4x = 10 ⇒ x = 2.5 arrangement, it is clear that Immediate
(6 × 20) and (7 × 20) Hence, present age of Ramaya right of 'F'  A.
Hence, required H.C.F. = 20 = 8 × 2.5 = 20 years.
92. (d) According to dictionary, the
80. (c) (2p + 3q) + (2p – q) = 18 + 2 Distance
⇒ 4p + 2q = 20 sequence of words is: iv, i, iii, ii, v
85. (d) Speed of car =
Time (iv) Manage (i) Misleader
⇒ 2(2p + q) = 20
315 km (iii) Mobil (ii) Monarchy
⇒ 2p + q = 10 =
2.8 hr. (v) Movies
81. (c) Total runs in the first 43 overs
= 43 × 5 = 215 runs. 3150 3150 × 1000 m Finally (iv), (i) (iii), (ii) (v) is a
= =
Required runs in 7 overs 28 28 × 3600 sec correct sequence.
= 285 – 215 = 70 450 × 5 93. (c) Except 78, all are square
70 = = 31.25 m/s numbers.
Run rate in 7 overs = 4 × 18
7 94. (a) According to dictionary, the
= 10 runs / over 86. (b) Wife of the father-in-law of arrangement of words is,
82. (a) Let the number of sides be n person's wife  person's mother. So, (ii) Capability, (i) Carrier,
in a polygon only son of person's mother  person
himself (iii) Cause, (iv) Course
(n – 2) × 180º = 1800º
Finally, ii, i, iii, iv is the correct
1800º Therefore, the 'girl' is the only
n–2 = = 10 sequence.
daughter 'professor'.
180º 95. (a) 3 + 9 = 12; 6 + 3 = 9;
n = 10 + 2 = 12 87. (d)
7 + 8 = 15; 8 + 9 = 17
Hence, the number of sides is 12. 17 is not exactly divisible by 3.
83. (c) Let the cost price of mixer B O A R D
96. (a) According to statements,
be x 88. (b) only Conclusion II follows.
and microwave be (36750 – x) N
According to question.
start
D 20 km
97. (d) According to statement only
C
Conclusion I follows.
(100 + 32) (100 − 22) W E
x = (36750 – x) 30 km 98. (b) According to statements, (I)
100 100
end A
S and (II), both the Conclusions follows.
132x = 78 (36750 – x) 20 km B
99. (b) According to statements,
132x + 78x = 78 × 36750 The distance AD = (20 + 30) — 20 Conclusion I and II both follows.
210x = 78× 36750 = (50 — 20) km = 30 km
100. (a)
78 × 36750 Finally, his school is 30 km far Knives
x= from hospital.
210 Scissors Blades
= 78× 175 = 13650 89. (d) According to question,
The cost price of mixer is ` 13650 Prem > Raju > Sundar > Hari >
and the cost price of microwave = 36750 Ompal
– 13650 = ` 23100 According to statement only
Finally, Prem owns the highest conclusion I follows.
The cost price of the cheaper item
mixer is ` 13650 share of land. qqq
78 Practice Set-7

Air Force Airmen Group X & Y Exam


Practice Set-7

ENGLISH “We do not have a place for the children (c) He will succeed; you never.
Directions (1-4): Read the passage to play;” said de Sousa, a shop owner, (d) He will succeed : you never.
given below and answer the questions “They are stuck in the houses, bored”. 8. AEOLIAN means:
that follow by selecting the most The most adventurous, though, will find (a) flavored with garlic
appropriate option. a game somewhere. (b) producted by the wind
In this floating village in Brazil, there 1. The floating village is famous for (c) bewitched
is only one way to travel. Students go its– (d) light headed through lack of
to school by boat. Locals go to worship (a) black river sleep
by boat. Taxis arrive by boat. Even the (b) craze for sports 9. Give the antonym of ‘pragmatic’.
soccer field is often a boat. There are three (c) annual flooding (a) Vague (b) Pessimistic
homemade fields on land, but they are (d) suffering of the locals (c) Confused (d) Idealistic
submerged now in the annual flooding 2. The wooden goal posts had nets 10. Choose the incorrect sentence.
of the Black River. If the wooden goal that are ______ during floods. (a) His T-shirt are loose
posts had nets, they would be useful this (a) floating around (b) My books are polished
time to year only for catching fish. So, (b) used as nets (c) The cattle is grazing in the field
young players and adults improvise. (c) useless (d) Her hari is thick
They play soccer at a community centre (d) misused 11. Whose car has been stolen still
that has a roof but no walls. They play 3. “The passing boats sometimes remains a mystery
on the dock of a restaurant. And they knocked down power lines” (a) Adjective/Relative clause
suggests that the (b) Adverb clause
play on a parked ferry, a few wearing
(a) rescue boats are careless (c) Principal clause
life jackets to cushion their fall. The
(b) water was charged (d) Noun clause
high-water mark in the Rio Negro this
(c) the flood levels were high Directions (12-13): In each of the
year was the fifth highest in more than a
(d) electric poles had fallen following questions choose the best
century of measurements.
4. A word that can best replace the option to complete the sentence.
As scientists study the impact of
word ‘improvise’ is 12. Despite his best efforts to conceal
deforestation on the Amazon basin and
(a) contrive (b) share his anger,
the cooling and warming of the Pacific
(c) plan (d) devise (a) he failed to give us an impression
Ocean, extreme patterns observed over
5. Find the correctly spelt word of his agony.
the last 25 or 30 yr raise an important
(a) Gramar (b) Havene (b) people came to know that he
unanswered question. “Are these trends
(c) Haven (d) Macaber was annoyed.
human-induced climate change or can
6. The misspelt word is (c) he could succeed in going it
we explain this with natural variability?”
(a) Comittee easily.
Villagers said that passing boats
(b) Nursery (d) we could detect that he was
sometimes knocked down power lines
(c) Miscellaneous quite happy.
during periods of exceptionally high
(d) Occurrence
water. And while the soccer fields are 13. The strike was _________ after
7. Choose the correctly punctuated
usually available for about half the year, negotiations.
version.
the land has recently been dry enough (a) called off (b) cut off
(a) He will succeed you, never.
for only four or five months of play; (c) taken off (d) call up
(b) He will succeed, you never.
Practice Set-7 79

Directions (14-15): In the following ELUDE (c) The electric potential is highest
questions, some of the sentences have (a) allure (b) leave at the centre
errors and some have none. Find (c) deceive (d) escape (d) The electric potential is lowest
out which part of a sentence has an at the centre and increases
error. The number of that part is PHYSICS towards its ends
your answer. If there is no error, the 26. The magnitude of vector product
21. An electron and a photon have
answer is (d).
the same de-Broglie wavelength. → →
14. Two hours have passed (a)/since (b)/ of two non-zero vectors A and B
Which of the two has greater
he had fallen asleep. (c)/No error →
‘total energy’? is zero. The scalar product of A
(d).
(a) Electron → →
15. Having broken down (a)/the driver and A + B is equal to:
(b) Photon
sent the car (b)/to the grage. (c)/No (a) Zero (b) AB
(c) Both have equal total energy
error (d). (c) A2 (d) A2 + AB
(d) Data insufficient to decide
Directions (16-17): In the following 27. With what minimum acceleration
questions, choose the word opposite 22. A radiowave has a maximum
electric field intensity of 10–4 Vm–1 can a person slide by holding the
in meaning to the given word. rope which can bear tension equal to
on arrival at a receiving antenna.
16. SURVIVAL two-third of the weight of the
The maximum magnetic flux
(a) evolution (b) development density of such a wave is: person?
(c) creation (d) extinction (a) Zero (b) 3 × 104T 2
(a) g (b) g
17. INSERT (c) 5.8×10–9T (d) 3.3 × 10–13 T 3
(a) depict (b) extract 23. The length of a wire is increased
(c) 1 g (d) Zero
(c) defame (d) enhance by 1 mm on the application 3
Directions (18-19): In the following of given load. In a wire of the
28. A particle of charge –16×10 –
questions, the sentences have been same material but of length and 18
C moving with velocity
given in Active/Passive voice. From radius twice that of the first on
10ms–1 along the x-axis enters a
the given alternatives, choose the application of the same force,
one which best expresses the given region where a magnetic field of
extension, produced is:
sentence in Passive/Active voice. induction B is along the y-axis and
(a) 2 mm (b) 0.5 mm
an electric field of magnitude 104
18. It is time to take tea. (c) 4 mm (d) 0.25 mm
V/m is along the negative z-axis.
(a) It was time that tea was taken 24. A charge q = 2 µC is moved by
If the charged particle continues
(b) It is time for tea to be taken some external force from infinity
moving along the x-axis, the
(c) It is time that tea should be taken to a point were electric potential is
magnitude of B is:
(d) It is time that tea had been taken 104 V. The work done by external
force is: (a) 103 Wb/m2 (b) 105 Wb/m2
19. The members should adhere to all (c) 1016 Wb/m2 (d) 10–3 Wb/m2
the decisions. (a) 1×10–2J (b) 2×10–2J
(c) 0.2 ×10 J (d) 12×10–2J
–2 29. When a resistance of 9.5 Ω is
(a) All the decisions should adhere connected across a battery, the
to the members 25. A m e t a l r o d m o v e s a t a
constant velocity in a direction voltage across the resistance is
(b) All the decisions adhered to the 11.4 V. If the resistance connected
perpendicular to its length. A
members across the same battery is 11.5 Ω,
constant uniform magnetic field
(c) All the decisions should be the voltage across the resistance
exists in space in a direction
adhered to by the members is 11.5 V:
perpendicular to the rod as well
(d) All should adhere to the (a) the emf of the battery is 12.0 V
its velocity. Select the correct
decisions of the members statement(s) from the following: (b) the internal resistance of the
20. In the following question, out (a) The entire rod is at the same battery is 0.5 Ω
of the four alternatives, choose potential (c) the internal resistance of the
the one which best expresses the (b) There is an electric field in the battery is 11.45 Ω
meaning of the given word. rod (d) the e.m.f. of the battery is 11.5 V
80 Practice Set-7

30. In a region the electric potential is 35. A slab of 40 kg mass is placed


given by V = 2x + 2y – 3z. Obtain on a frictionless floor. A block of
the expression for electric field: 10 kg mass is placed on it. The
coefficient of static and dynamic
(a) –2iˆ − 2 ˆj + 3kˆ
friction between slab and block
ˆ ˆ ˆ
(b) 3i + 4 j − 2k are 0.6, 0.4 respectively. A force
ˆ ˆ ˆ of 100 N is applied horizontally
(c) 2i − 2 j – 2k on 10 kg mass:
(d) None of the above Block B
A
31. An inductance, a capacitance and
10 kg 100 N (a) 1 Ω (b) 2 Ω
a resistance are connected in series
(c) 3 Ω (d) 4 Ω
across a source of alternating 39. In a thermodynamic process,
voltage. At resonance, the applied 40 kg when the pressure of a fixed
voltage and the current flowing Slab mass of gas is changed 30 J heat
through the circuit will have a is evolved and 10 J work is done
Frictionless floor on it. If internal energy is 100 J,
phase difference of:
If g = 9.8 m/s2, then acceleration then final internal energy is:
π
(a) (b) Zero of slab is: (a) zero (b) 80 J
4
(a) 0.98 m/s2 (b) 1.47 m/s2 (c) 20 J (d) 60 J
π
(c) (d) 40. The circuit has two oppositely
4 (c) 1.52 m/s2 (d) 6.1 m/s2
connected ideal diodes in parallel.
→ 36. A certain physical quantity is What is the current flowing in the
32. I f a = 2iˆ − 3 ˆj + kˆ and
calculated from the formul circuit?

b = 3iˆ + ˆj – 2kˆ the cosine of π 2 4Ω
(a − b 2 )h where h, a, b are all D1
3 D2
angle θ between them is equal to:
lengths. The quantity being
5 1 12 V 3Ω 2Ω
calcualted is:
(a) 14 (b) 7
(a) velocity (b) length
3 1 (c) area (d) volume (a) 1.71 A (b) 2.00 A
(c) (d)
14 14 37. Two particles X and Y having (c) 2.31 A (d) 1.33 A
33. In the absolute version of the equal charge, after being 41. A convex lens forms real image
Fahrenheit scale, the freezing accelerated through the same at a point P, A plane mirror is
point of weater would be: placed at 45° to the line joining
potential difference enter a region
pole O of mirror and before point
(a) –32°F (b) +180°F of uniform magnetic field and
P at distance 10 cm, then the final
(c) 491.4F (d) –459.67ºF describe circular path of radii R1 image is:
34. In the spectrum of light received and R2 respectively. The ratio of (a) virtual
from a heavenly body, the the mass of X to that of Y is: (b) on line the PO produced
2
wavelength of a line appears 3737 R1 R  (c) above the PO produced at
(a) (b)  2  distance 10 cm
Å while the actual wavelength is R2  R1 
(d) below the PO produced at
3700 Å. The velocity of heavenly 2
R2
 R1  distance 10 cm
body is: (c)   (d)
 R2  R1 42. A source of light gives radiation
(a) 3×106 m/s away from the earth of wavelength 6000 Å. When the
38. Sixteen resistors each of resistance same light is received from a star
(b) 3× 106 m/s towards the earth
16 Ω are connected in the circuit its wavelength is found to be 6010
(c) 3× 107 m/s away from the earth
as shown. The net resistance Å. The speed of star with respect
(d) 3× 108 m/s towards the earth between AB is: to earth is:
Practice Set-7 81

(a) 5× 105 m/s away from the earth π π 2


(a) (b)  1 1 
(b) 5× 105 m/s towards the earth 3 55. The value of  1 + 2 ! + 4 ! + ... ∞ 
2  
(c) 6× 105 m/s away from the earth π 2
 1 1 
(d) 6× 105 m/s towards the earth (c) (d) π − 1 + + + ... ∞  is:
4  3 ! 5 ! 
43. If the electric flux entering and 50. The coefficient of the term (a) 0 (b) 1
leaving an enclosed surface independent of x in the expansion
(c) – 1 (d) 2
respectively is φ 1 and φ 2 , the  x 3 
10
56. The lines 2x – 3y = 5 and 3x – 4y =
electric charge inside the surface of  3 + 2  is:
2x 7 are diameters of a circle of area
will be: 154 sq units. Then, the equation
(a) 9/4 (b) 3/4
(a) (φ2 – φ1) ε0 (b) (φ2 + φ1)/ ε0 of this circle is:
(c) 5/4 (d) 7/4
(c) (φ2 – φ1)/ ε0 (d) (φ2 + φ1) ε0 51. A function f is defined as follows (a) x2 + y2 + 2x – 2y = 62
44. Which of the following cubic (b) x2 + y2 + 2x – 2y = 47
 1
system possess closest packing? f (x) = x p cos   , x ≠ 0,
(c) x2 + y2 – 2x + 2y = 47
x
(a) sc (b) bcc (d) x2 + y2 – 2x + 2y = 62
f (0) = 0
57. If a = log24 12, b = log36 24, c = log48
(c) fcc (d) hcp What conditions should be
36. Then, 1 + abc is equal to:
45. If the refractive index of water is imposed on p so that f may be
(a) 2ac (b) 2bc
4 5 continuous at x = 0?
and that of glass is , then the (c) 2ab (d) None of these
3 3 (a) p = 0 (b) p > 0
critical angle of incidence for light (c) p < 0 (d) No value of p 58. If A =  −cos θ sin θ  ,
then for n
 sin θ cos θ 
tending to go from glass to water 52. If y = sin x + sin x + sin x + ..., ∞ ,
is: ∈ N, An is equal to:
dy  cos n θ sin n θ 
3 3 then is equal to: (a) 
(a) sin −1   (b) sin −1   dx 
4 5 n
 − sin θ cos n θ 
cos x cos x
4 2 (a) (b)  cos nθ sin nθ 
−1
(c) sin   −1
(d) sin   2x − 1 2 y −1 (b) 
5 3  − sin nθ cos nθ 
cos x
(c) (d) None of these
2x + 1  n cos θ n sin θ 
MATHEMATICS (c) 
53. Evaluate ∫ x log x dx :  − n sin θ n cos θ 
46. If A = {a, b, c}, then what is the (d) None of these
number of proper subsets of A? x2 x4
(a) log x + +C 59. The solution of the differential
(a) 5 (b) 6 2 4
equation
(c) 7 (d) 8 x2 y2 x2y dy – (x dy – y dx) = 0 is:
(b) 4 log x − +C
6 i −3 i 1 4 (a) x2y – 2x = 2Cy
47. If 4 3 i −1 = x + iy, then
20 3 i 2 2 (b) xy2 – 2y = 2Cx
(c) x log x − x + C
(a) x = 3, y =1 (b) x = 1, y =3 2 4 (c) xy2 + 2x = 2Cy
(c) x = 0, y =3 (d) x = 0, y = 0 (d) None of these (d) xy2 + 2y = 2Cx
54. I f p b e t h e l e n g t h o f t h e 1 2 3 4
48. If α and β are the roots of the 60. The value of + + + +
2 ! 3! 4! 5!
equation x2 + 6x + 1 = 0, then what perpendicular from the origin on
is | α – β | equal to? the straight line x + 2by = 2p, then ..... ∞ is:
what is the value of b? 2
(a) 6 (b) 3 2
1 (a) e – 2 (b) 3 e − 1
(c) 4 2 (d) 12 (a) (b) P
p (c) 1 (d) 3/2
49. The angles of a triangle are in AP
3 61. The mean life of a sample of 60
and the least angle is 30°. What is (c) 1 (d) bulbs was 650 h and the standard
2 2
the greatest angle (in radian)? deviation was 8 h. A second sample
82 Practice Set-7

of 80 bulbs has a mean life of 660 h 5  (a) Tomato


1 
and standard deviation 7 h. Find (a)  , 1 (b)  , 1 (b) Sweet potato
4 4
the over all standard deviation. (c) Onion
 5 5 
(a) 8.97 (b) 8.98 (c) 1,  (d)  , 0 (d) Turmeric
4 4
(c) 8.94 (d) None of these 76. The term PC means:
x
62. T h e f u n c t i o n f ( x ) = l o g e 1 − tan 2 (a) Professional Calculator
69. What is 2 equal to?
( x3 + x6 + 1) is one of the x (b) Personal Computer
1 + tan 2
2
following types: (c) Private Computer
(a) Even (b) Odd (a) sin x.cos x (b) tan x
(d) Personal Calculator
(c) Decreasing (d) None of these (c) sin x (d) cos x
77. Which state has topped among
cos α − sin α  ,
63. In how many ways can the letters 70. If A =  sin α cos α  then find
the larger states for overall
 performance on health indicators
of the word PERMUTATIONS be
arranged, if the words start with the value of ATA. as per the NITI Aayog report
P and end with S? titled “Healthy States, Progressive
(a) O (b) I
India”?
(a) 1814400 (b) 1814405 (c) A (d) AT
(c) 1824050 (d) None of these (a) Maharashtra
64. The solution of the differential REASONING AND GENERAL (b) Kerala
dy 3 AWARENESS (RAGA) (c) Andhra Pradesh
equation x4 ++xx3yy++cosec
cosec( xy(xy)
) = 0,
dx (d) Odisha
71. Raj Committee was associated
= 0, is equal to: 78. (91 + 92 + 93 + ________ + 110) is
with:
(a) 2 cos (xy) + x–2 = C equal to
(a) Banking (a) 4020 (b) 2010
(b) 2 cos (xy) + y–2 = C
(b) Agriculture (c) 6030 (d) 8040
(c) 2 sin (xy) + x–2 = C
(d) 2 sin (xy) + y–2 = C (c) Large-scale industry 79. The L.C.M. of two numbers is
(d) Small-scale industry 48. The numbers are in the ratio
1 n2
65. What is the value of ∫0 xe dx ?
72. The final cause of the failure of 3 : 4. The sum of the numbers is:
the Mutiny (1857) was: (a) 28 (b) 80
(e − 1)
(a) (b) e2 – 1 (c) 120 (d) 200
2 (a) Lack of unity
80. If a + b = 5 and 3a + 2b = 20, then
(c) 2(e – 1) (d) e – 1 (b) Death of Rani of Jhansi (3a – b) will be:
66. ABC is a triangle in which AB = (c) Selfishness of the leaders (a) 5 (b) 25
6 cm, BC = 8 cm and CA = 10 cm. (d) Treachery (c) 15 (d) 35
What is the value of cot (A/4)? 73. Under which Article of the Indian 81. Among three numbers, the first
(a) 5−2 (b) 5+2 Constitution has the Right to is twice the second and thrice
the Freedom of Speech and the third. If the average of three
(c) 3 −1 (d) 3 +1 numbers is 517, then what is the
Expression been guaranteed? difference between the first and
67. The equation sin–1 (3x – 4x3) = 3 (a) Article 22 (b) Article 19 the third number?
sin–1 (x) is true for all values of x
(c) Article 20 (d) Article 21 (a) 564 (b) 759
lying in which one of the following
74. Mariana Trench is the deepest (c) 485 (d) 799
intervals?
trench located in the ________ 82. In an isosceles trapezium
 1 1 1  ________ .
(a)  − 2 , 2  (b)  ,1 Ocean.
2  (a) Opposite sides are parallel
(a) Arctic (b) Pacific (b) Opposite angles are
 1
(c)  −1, −  (d) [– 1, 1] (c) Atlantic (d) Indian supplementary
 2
75. In which plant does the stem (c) Opposite anlges are not equal
68. The focus of the parabola y2 – x –
perform the functions of storage (d) Diagonals bisect opposite
2y + 2 = 0 is:
and perennation? angles
Practice Set-7 83

83. A rice trader buys 16 quintals 89. There are five boys P, Q, R, S and Direction (94): Arrange the given
of rice for ` 5632. 20% rice is T standing in a line. Q is taller words in the sequence in which they
lost in transportation. At what than T but shorter than P. S is occur in the dictionary.
rate should he sell to earn 25% shorter than T but taller than R. 94. i. Savage ii. Sausage
profit? Who is the shortest?
iii. Savour iv. Saviour
(a) ` 225.2 per quintal (a) S (b) R
(a) iii, iv, i, ii (b) ii, i, iv, iii
(b) ` 550 per quintal (c) T (d) P
(c) ii, iii, i, iv (d) iii, iv, ii, i
(c) ` 440 per quintal Direction (90): A series is given with Direction (95): The follow­ ing
(d) ` 563.2 per quintal one term missing. Choose the correct questions are based on the five three
84. Find two numbers such that alternative from the given ones that
digit numbers given below:
their mean proportional is 12 will complete the series.
394  783  576  895
and third proportional is 768. 90. EDCB, IHGF, MLKJ, ?
95. If 2 is added to the last digit of
(a) 3 and 24 (b) 3 and 48 (a) QPON (b) QLNO each number and then the posi­
(c) 6 and 48 (d) 6 and 24 (c) KNOL (d) LMON tions of the first and the third
85. Rizvan walks at 15 km/hr and Direction (91): Study the following digits are interchanged, which of
Ruchitha cycles at 20 km/hr information carefully and answer the the following will be the highest
towards each other. What was question given below: number?
the distance between them when (a) 576 (b) 895
they started if they meet after 42 A, B, C, D, E, F, G and H are sitting
around a circle facing the centre. H is (c) 783 (d) 394
minutes ?
fourth to the left of B and second to Direction (96–100): Read the
(a) 36.8 kms (b) 30.6 kms
the right of F. A is third to the left of following information carefully and
(c) 24.5 kms (d) 18.4 kms C, who is not an immediate neighbour answer the questions that follow:
Direction (86): Read the following of F. G is second to the left of A. D is Six lectures A, B, C, E and F are to
information carefully and then answer second to the right of E. be orgainsed in a span of 7 days–from
the question given below it: 91. Who is third to the right of A? Sunday to Saturday, only one lecture
A + B means A is the father of (a) C on each day in accordance with the
B, A – B means A is the wife of B; (b) F following.
A × B means A is the brother of B; A ÷
(c) B (i) A should not be organised on
B means A is the daughter of B.
(d) Data inadequate Thrusday.
86. If P + R ÷ Q, which of the following
is true ? Direction (92): In following question (ii) C should be orgainsed immediately
(a) P is the husband of Q you have to identify the responses which after F.
would be a correct inference from the (iii) There should be a gap of 2 days
(b) P is the brother of Q
given premises stated according to the
(c) P is the son of Q between E and D.
following systems:
(d) P is the father of Q 'A' stands for Not greater than. (iv) One day there will be no lecture
87. If PALE is coded as 2134, (Friday is not that day), just before
'B' stands for Greater than
EARTH is coded as 41590, how that day D will be organised.
'C' stands for Not equal to
is PEARL coded in that code? (v) B should be organised on Tuesday
'D' stands for Equal to
(a) 29530 (b) 24153 and should not be followed by D.
'E' stands for not less than
(c) 25413 (d) 25430 96. On which day there is no lecture?
'F' stands for less than.
88. Neeraj is facing north, then he (a) Monday
92. Premises: (a E b) and (b E c)
turns 45 degree right and goes (b) Friday
25m, then turns in south-east (a) c B a (b) a F c
direction to move 25 m and from (c) c B b (d) a E c (c) Sunday
there 25 m to east. In which Direction (93): Select the one which (d) Cannot be determined
direction/place is he from his is different from the other three 97. How many lecutres are orgainsed
original place? responses. between C and D?
(a) North (b) East 93. (a) 21-37 (b) 42-26 (a) None (b) One
(c) West (d) South (c) 35-51    (d) 56-38 (c) Two (d) Three
84 Practice Set-7

98. Which day will the lecture F be (a) A finding the complete sequence of
orgainsed? (b) B organisation of lectures?
(a) Thursday (b) Friday (c) C (a) (i) only
(c) Saturday (d) Sunday
(d) Cannot be determined (b) (ii) only
99. Which of the following is the last 100. Which of the following (c) (i) and (v) only
lecture in the series? information is not required in (d) All are required

Answers with Explanation


strike was called off after negotiations. E
ENGLISH Hence option (a) is correct. 22. (d) B =
c
1. (c) 2. (c) 3. (d) 4. (d) 14. (c) ‘He fell asleep’ will be the 10−4
5. (c) The correctly spelt word is correct usage. =
3 × 108
Haven which means harbor, port of 15. (b) The driver sent the car.
any place of shelter and safety, refuge, = 3.3 × 10−13 T
16. (d) The word ‘Survival’ (Noun)
asylum. means the state of continuing to live 23. (b) Young’s modulus
6. (a) The mis-spelt word is ‘comittee’, or exist in spite of difficulty of danger. F× L
Y=
the correct word is ‘committee’ which Hence, extinction and survival are πr × 1 × 10−3
2
  
means a group of persons elected or antonymous. F × 2L
appointed to perform some service of 17. (b) The word ‘Insert’ (verb) In second case Y =
π × (2r ) 2 × l
function, as to investigate, report on, or means put, fit or place something
act upon a particular matter. between two, things. Hence, insert and F×L F × 2L
∴ 2 −3
=
7. (b) Correctly punctuated version extract are antonymous. πr × 1 × 10 4π × r 2 × l
is—He will succeed you never. 18. (b) 19. (c) ∴ l = 0.5 mm
8. (b) Aeolian means (b) producted 20. (d) The word ‘elude’ (verb) 24. (b)
by the wind. means escape, especially by a clever (W∞ − a )ext force = qV
9. (d) Pragmatic means practical. tick. For example,
Vague means uncertain. = (2 × 10 –6 ) × (10 –4 )
He eluded capture for weeks by hid-
Pessimistic means Depressed. ing underground. = 2 × 10 –2 J
Idealistic means Impractical. 25. (b) An induced e.m.f. is set up
Hence, the antonym of pragmatic is PHYSICS across the ends of the rod and hence an
idealistic 21. (a) For electron, E = mc2 electric field is set up in the rod.
10. (c) Plural verb should be used h h 26. (d)
Also, λ= or m =
with cattle. Hence,‘‘the cattle are graz- mv λv →
|A × B | = 0

ing in the field’’ is the correct option. hc 2 ⇒ A Bsi n θ = 0


11. (c) Whose car has been stolen \ E = ⇒ θ = 0°
λv
still remains a mystery, clause found in → → → → → → →

' hc Further A . (A + B) = A . A + A . B
the underlined word is noun clause. A For photon, E = hv = → → →
λ ⇒ A . (A + B) = A 2 + ABcos θ
noun clause is a group of words which
contains a subject and a predicate of its E hc 2 λ c
→ →
⇒ A . (A + B) = A 2 + AB

Dividing, = × =
own, and does the work of a noun. E′ λv hc v
27. (c) mg – T = ma
12. (b) Despite his best efforts to E
As c > v, >1 2
conceal his anger (b) people came to E' or, mg − mg = ma
3
know that he was annoyed. ⇒ E > E'
13. (a) The phrasal verb ‘called off’ i.e. the ‘total energy, of an electron is g
∴ a=
would be used in the sentence only. The greater than the ‘total energy’ of a photon. 3
Practice Set-7 85

28. (a) The force on a particle is: ∂V ∂ Hence, quantity is volume.


= (2 x + 2 y − 3 z ) = 3 1 2mV
∂y ∂y 37. (c) r =
B my
∂V ∂
B = (2 x + 2 y − 3 z ) = − 2
∂z ∂z R1 mx
V
=

R2 my
X E = − (2 ˆj + 2 ˆj − 3k )
2
E
31. (b) At resonance no phase mx R 
⇒ =  1
difference exists between applied my  R2 
Z voltage. 38. (c)
32. (d) R

 → → → 4
So, F = q  E + v × B →
a . b

= ab cos θ
 
⇒ 6 − 3 − 2 = (4 + 9 + 1) (9 + 1 + 4) (cos θ)
→ → → R R
or F = Fe + Fm or , 1 = 14 cos θ 4 4
→ →
1
Fm = q E ⇒ cos θ = A B
14
( )
→ → R
∴ Fe = q E = − 16 × 10 –18
× 10 × – kˆ
4
33. (d) For the absolute version 4

= 16 × 10 –14
kˆ of Fahrenheit scale, let us find the 3R 3(16)
⇒ R AB = =
equivalent F temperature to absolute zero 16 16
( )

and Fm = − 16 × 10 –18 10iˆ × Bˆj from the relation: = 3Ω
⇒ = − 16 × 10 × B(+ kˆ )−17 K − 273.15 F − 32
=
5 9 39. (B) Given Q = –30 J,
= − 16 × 10−17 B kˆ for K =0
∆W = – 10J
since, particle will continue to move 9
F = – × 273.15 + 32 Since, ∆U = Q – ∆W
along + x-axis so, resulant force is 5
= − 459.67 °F or, Uf – Ui = Q –W
equal zero.
→ → 34. (a) Uf – 100 = – 30 –(–10) = –20
Fe + Fm = 0 v \ Uf = 100 – 20 = 80 J
∆λ = ×λ
∴ 16 × 10−14 = 16 × 10−17 B c 40. (b) In the given circuit, diode D1
–14 ∆λ
16 × 10 or, v =c is reverse baised while D2 is forward
⇒ B = = 103 λ
16 × 10 –17 biased, so the circuit can be redrawn as:
 3737 − 3700 
∴ v = 3 × 108  m/ s
∴ B = 103 Wb/m 2  3700  4Ω
D1 D2
= 3 × 106 m / s
29. (a) (A, B) 11.4 = I1 (9.5)
The increase in wavelength decrease 3Ω 2Ω
12 V
I1
⇒ = 1.2 A the frequency therefore, source of
light must be moving away from the
since, E – 11.4 = I1 r earth.
⇒ E – 11.4 = 1.2 r 35. (a) The speed will be provided by [ For ideal diodes, reverse biased
dynamic friction. means open and forward biased means
similarly, I2 = IA
If α is the acceleration of slab, then short)
⇒ E – 11.5 = 1 r
µmg = mα Apply KVL to get current flowing
0.1 = 0.2 r or, 0.4 × 10 × 9.8 = 40 α through the circuit.
⇒ r = 0.5 Ω and E = 12 V − 12 + 4i + 2i = 0
\ α=
30. (a) 12
= 0.98 m/s2 ⇒ i = = 2A
 ∂V ˆ ∂V ˆ ∂V ˆ 
→ 6
E = i + j+ k 
 ∂x ∂y ∂z  36. (d) π (a 2 − b 2 ) h = (L2 – L2 ) L 41. (d) If mirror is rotated through 45°,
∂V ∂ 3 image is rotated through 90° in the same
Here, = (2 x + 2 y − 3 z ) = 2
∂x ∂x = L2 . L = [L3 ] sense:
86 Practice Set-7

⇒ 0 = 0 + 0i 52. (b) We have,


∴ x = 0, y = 0
y= sin x + sin x + sin x + ...∞
48. (C) Since, α and β are the roots of
S 45º O P the equation y = sin x + y ⇒ y2 = sin x + y
10 cm
x2 + 6x + 1 = 0 On differentiating both sides w.r.t x,
10 cm ∴ α + β = – 6 and αβ = 1 we get
Now (α – β)2 = (α + β)2 – 4αβ dy dy
P' 2y = cos x +
= (– 6)2 – 4 dx dx
Hence, P behaves as virtual object for = 36 – 4 = 32
dy
the plane mirror. The image formed by ∴ | α – β | = | 32 | = 4 2 ⇒ (2y – 1) = cos x
dx
plane mirror is real and situated 10 cm 49. (a) Let the angles of triangle are dy cos x
below the optical axis. a, a + d and a + 2d ∴ =
dx 2 y −1
42. (a) v Given: a = 30°
∆λ = λ a + a + d + a + 2d = 180° 53. (c) Taking log x as the first
c function and x as the second function.
∆λ ∴ 3a + 3d = 180°
or, v = c Since,
⇒ 3 × 30° + 3d = 180°
λ
⇒ 90° + 3d = 180° ∴ I = ∫ x log x dx
6010 − 6000
= 3 × 108 ⇒ 3d = 90° ⇒ d = 30° x2 x2  1 
6000 ∴ Angles of triangle are 30°, 60° and ⇒ I= log x − ∫   dx
8 2 2  x
3 × 10 90°.
=
600 Hence greatest angle = 90° = π/2 x2 1
= log x − ∫ x dx
= 5 × 105 m/s 10 − r r 2 2
10  x  3 
50. (c) Tr+1 = Cr  x2 x2
Since, wavelength increases i.e.  3   2 
2x = log x − + C
frequency decreases therefore, star is 10 − r
2 4
moving away from the earth. x 2 3r 54. (d) Length of perpendicular from
10
43. (a) From Gauss’s law, = Cr 10 − r · r 2r origin to the given equation
Charge enclosed
2 .x
= flux leaving the surface 3 2 0 + 0 − 2 p = p
ε0  2 
q 33r − 10  1 + 4b 
⇒ = φ 2 – φ1 10 − 5 r
ε0 = 10 Cr 2 x 2 2p
⇒ q = ( φ 2 – φ1 ) ε 0 2 ⇒ =p
− .x 1 + 4b 2
44. (c) The closest packing is for fcc 2
system. 10 − 5r ⇒ 4 = 1 + 4b2
For term independent of x, = ⇒ 4b2 = 3
µrarer µ 2
45. (c) sin C = = w 3
µdenser µg 0 ⇒ r=2 ⇒ b =
Since, term T3 is independent of x. 2
4/3 4
⇒ sin C = = 10 1 5  e + e −1   e − e −1 
2
5/3 5 ∴ Its coefficient = Cr 2 3 = 55. (b)  −
2 3 4  2   2 
MATHEMATICS
 p  1 1
46. (c) Given, A = {a, b, c}, Number of  x cos   x≠0   = (4 . e.e −1 ) = 1
51. (b)  f (x) =  x 4
subset of A = 2n = 23 = 8
 0 x=0
∴ Proper subset of A = 2n – 1 = 8 – 1  56. (c) If r is radius of the circle, then
=7 p  1 πr2 = 154
47. (d) We have lim f ( x) = lim x cos   7
x→0 x → 0 x ∴ r2 = 154 ×
6i −3i 1 6i 0 1 22
Since, f (x) is continuous at x = 0
4 3i −1 = 4 0 −1  22 
 1    = 49  taking π = 
20 3 i 20 0 i lim x p cos   = 0 7
x→0
 x
Applying C2 → C2 + 3iC3 Which is possible only, if p > 0. ⇒ r=7
Practice Set-7 87

Also, solving the equation of two  y ∴ f (– x) = – f (x)


given diameters, we get the coordinates ⇒ − d   + ydx = 0 Hence, f (x) is odd
 x
of the centre as (1, – 1). Again,
Hence, the equation of the circle is On integrating both sides, we get
dy 1  2 6 x5 
(x – 1)2 + (y + 1)2 = 72 = 49 y y2 =  3x + 

+ =C dx x 6 + 1 + x3  2 x 6 + 1 
⇒ x2 + y2 – 2x + 2y = 47 x 2
log12 log 24 log 36 ∴ xy2 – 2y = 2xC 3x 2
57. (b) abc = · · = = positive, ∀ x ∈ R
log 24 log 36 log 48 n n +1−1
60. (c) Tn= = x6 + 1
log12 (n + 1) ! (n + 1) !
Hence f (x) is an increasing function.
     = log 48
n +1 1 63. (a) The word PERMUTATIONS
= −
log 48 + log12 (n + 1) ! (n + 1) ! has
∴ 1 + abc = P — 1 time
log 48 1 1
= − E — 1 time
log (48 × 12) (n) ! (n + 1) !
= R — 1 time
log 48 ∴ T1 + T2 + T3 + .... M — 1 time
log 242 1 1  1 1  1 1 U — 1 time
= =  −  +  −  +  −  + ...
log 48  1 ! 2 !  2 ! 3 !  3 ! 4 ! T — 2 times
A — 1 time
log 24 1 1 1   1 1 1 
= 2· =  1 ! + 2 ! + 3 ! + ... −  2 ! + 3 ! + 4 ! + ... I — 1 time
log 48 O — 1 time
log 24 log 36  1 1 1  N — 1 time
= × 1 + + + + ... − 1
=  1 ! 2 ! 3 !  S — 1 time
log 36 log 48
 1 1 1  (i) Word start with P and end with S
= 2 log36 24 × log48 36 = 2bc + 1 + + + + ... − 1 − 1 i.e.,
 1! 2 ! 3 ! 
 cos θ sin θ  P S
58. (b) Given, A =  − sin θ = (e – 1) – (e – 2) = 1
 cos θ  First and last position are already
61. (c) Given, n1 = 60, x1 = 650, σ1 fieled by
 cos θ sin θ 
A2 =  − sin θ = 8,
Now,
 cos θ  P and S, respectively
n = 80, x2 = 660, σ = 7
2 2
Remaining 10 positions can be filled
 cos θ sin θ  ∴ Combined SD 10 !
=  = ways
 − sin θ cos θ  2!
n1σ12 + n2 σ 22 n1n2 ( x1 − x2 ) 2
= +
cos 2 θ − sin 2 θ 2sin θ cos θ  n1 + n2 (n1 + n2 ) 2 10 × 9 × 8 × 7 × 6 × 5 × 4 × 3 × 2 !
=  =
 − 2sin θ cos θ cos 2 θ − sin 2 θ  2!
60 × 64 + 80 × 49 60 × 80(650 − 660) 2
= +
 cos 2θ sin 2θ  60 + 80 (60 + 80) 2 = 720 × 7 × 6 × 5 × 4 × 3
=  − sin 2θ = 720 × 2520 = 1814400
 cos 2θ  3840 + 3920 (4800 × 100)
= + 64. (a) We have,
∴ By induction, 140 (140) 2 dy
x4 + x3 y + cosec ( xy ) = 0
 cos nθ sin nθ  7760 480000 776 4800 dx
An =  = + = +
 − sin nθ cos nθ  140 19600 14 196 3  dy 
⇒ x  x + y  + cosec ( xy ) = 0
59. (b) We have, x2ydy – (x dy – y dx) dx
= 55.42 + 24.49 = 79.91 = 894
=0 Now, on putting xy = v
6 3
or it can be rewritten as 62. (b) f (x) = log{x x + 1 + x } ⇒ y+x
dy
=
dv
y dx – x dy + x2y dy = 0 (i) dx dx
6 3
f (– x) = log{ x + 1 − x }
2
On dividing Eq. (i) by x we get ∴ The differential equation becomes,
ydx − ndy ∴ f(– x) + f (x) = log {x6 + 1 – x6} dv
+ ydy = 0 x3 + cosec v = 0
x2 = log 1 = 0 dx
88 Practice Set-7

dv dx  A  cos α sin α 
⇒ = 3 ∴ cot   = 2 + 5 ⇒ 2 – 5 AT= 
−cosec v x  4
 − sin α cos α 

⇒ − ∫ sin v dv = ∫ x −3 dx + C1 67. (a) sin–1 (3x – 4x3) = 3 sin–1 x  cos α sin α 


–1
Letsin (3x – 4x ) = θ3  − sin α cos α 
x −2 Therefore, ATA =  
⇒ cos v = + C1 3 sin–1 (x) = θ
−2 cos α − sin α 
θ θ  sin α
⇒2 cos (xy) + x–2 = 2C1 sin–1 (x) = , x = sin  cos α 
3 3
∴2 cos (xy) + x–2 = C cos 2 α + sin 2 α 0 
sin θ = 3x – 4x3
1 2
=  
∫0 xe
x
65. (a) f (x) = dx  0 2 2
sin α + cos α 
We know that,
2 dt – 1< sin–1 (3x – 4x3) < 1 1 0
Put x = t, 2x dx = dt, x dx = =  =I
2 π π 0 1 
t − < sin–1 (3x – 4x3) <
1 x2 1e dt 2 2
∴ ∫0 xe dx =
∫0 2 REASONING AND GENERAL
π
− < θ < π, −π < θ < π AWARENESS (RAGA)
1 t 1 1 2 2 6 3 6
= 2 [e ]0 = (e − 1) 71. (b) 72. (a) 73. (b) 74. (b)
2 π
66. (b) Here, a = 8, b = 10 and c = 6 − < sin θ ≤ 1 , − 1 ≤ x ≤ 1 75. (d) Turmeric is a prostate, thick,
2 3 2 2 2 fleshy, usually horizontally grown,
8 + 10 + 6
s = = 12  −1 1  modified stem. It grows beneath the soil
2 So, x lies between each = 
 2 2  surface.
A (12 − 10)(12 − 6)
68. (b) We have, y2 – 2y = x – 2 76. (b)
∴ tan =
2 12(12 − 8) 77. (b) Kerala Explanation: Among
⇒ (y – 1)2 = 1(x – 1)
⇒ y2 = x Larger States, Kerala, Andhra Pradesh
1 1
= = and Maharashtra topped the list in terms
4 2 where, Y = y – 1 and X = x – 1
of overall performance in the NITI
A Here, a = 1 Aayog’s Health Index 2019. ii. Haryana,
⇒ cot =2 4
2 Rajasthan and Jharkhand emerged as
∴ Focus is X = a, Y = 0 ⇒ x – 1 = the top three ranking States in terms of
 A
Now, cot   = cot  A + A  1 annual incremental performance.
 2   , y–1=0
4 4 4 78. (b) (91 + 92 + 93 + .... + 110)
A 5  = (1 + 2 + 3 ... + 110)
−1 cot 2 ⇒ Focus of parabola is  , 1
= 4 4 – (1 + 2 + 3 + ...90)
A where n1 = 110 and n2 = 90
2 cot  2 x
4 sin
 2 n1 (n1 + 1) n2 (n2 + 1)
A 1−   = −
cot 2 − 1 x 2 x 2 2
4 1 − tan 2  cos 
= 69. (d) 2 = 2
A 110 × 111 90 × 91
2 cot 2 x  2 x −
4 1 + tan sin 2 2
2  2
1+   = 55 × 111 – 45× 91
 A 2 x
Let cot   = x  cos 
 4 2 = 6105 – 4095 = 2010
79. (a) Let the numbers be 3x and 4x
x2 − 1 x x
∴ 2 = − sin 2 cos 2 Then, their L.C.M. = 12x
2x = 2 2 So, 12x = 48 or x = 4
2 x x ∴ The numbers are 12 and 16
⇒ x – 4x – 1 = 0 sin 2 + cos 2
2 2 80. (d) a + b = 5 ...(i)
4 ± 16 + 4
⇒ x= cos x and 3a + 2b = 20 ...(ii)
2 = = cos x Multiplying (i) by 2 and subtracting
1
4±2 5 2± 5 from (ii), we get: a = 10.
= = 70. (b) We have, Putting a = 10 in (i), we get :
2
Practice Set-7 89

b=–5 xy = 144 (ii) 91. (c) By analysing the circular


\ (3a – b) = 3 × 10 – (–5) = 30 + 5 x : y : : y : 768 arrangement, it is clear that third to the
= 35. x y left of 'A'  B.
81. (a) Let three numbers be p, q, r =
y 768 92. (d) According to the given
According to question 144 information
p = 2q = 3r y2 = 768x = 768 ×
y a E b ⇒ a <| b  ...(i)
y3 = 768 × 144
p+q+r and b E c ⇒ b <| c  ...(ii)
Average = 1 3×
1
3 y = (16 )

3 ×3 3
From (i) and (ii), we get
2 y = 16 × 3 = 48
2q + q + q a <| b <| c ...(iii)
3
517 = 144 144 From (iii), we get
3 x = = =3
y 48
6q + 3q + 2q a <| b ⇒ a E c.
= Hence, the numbers are 3 and 48.
9 85. (c) Let the distance between 93. (d) Except '56-38', all others have
11 Rizwan and Ruchitha be x. difference 16.
517 = q Relative speed = 15 + 20 = 35 km/hr. 94. (b) According to dictionary, the
9
 When they started to walk in arrangement of the words is:
517 × 9 opposite, then distance (ii) Sausage, (i) Savage,
= 47 × 9 = 423
11 35 × 42 (iv) Saviour, (iii) Savour.
= = 24.5 kms
p = 2q = 2 × 423 = 846 60 Finally, ii, i, iv, iii is the correct
2 2 86. (a) P + R  P is the father of R. sequence.
r= q = × 423
3 3 R ÷ Q  R is the daughter of Q or 95. (a) 394 + 2 ⇒ 396 ⇒ 693
= 2 × 141 = 282 Q is the mother of R. 632 + 2 ⇒ 634 ⇒ 436
Difference of the first and the third So, P father of R daughter of Q. 783 + 2 ⇒ 785 ⇒ 587
number Therefore, ‘P’ is the husband of Q. 576 + 2 ⇒ 578 ⇒ 875
= p – r = 846 – 282 = 564. 87. (b) As, 895 + 2 ⇒ 897 ⇒ 798
82. (b) Opposite angles are supple-
P A L E And E A R T H So, it is clear 576 is the highest
mentary. B B B B B B B B B number.
2 1 3 4 4 1 5 9 0
96. (a) By analysing the above table,
Similarly, it is clear that no lecture organised on
P E A R L Monday.
83. (b) C.P. of 16 quintal rice B B B B B 97. (d) By analysing the above table,
= ` 5632 2 4 1 5 3 it is clear that between C and D ⇒ three
16 × 80
Left rice = = 12.8 quintal 88. (b) According to statement, Neeraj lectures i.e. B, E and F.
100
is in East direction from his original 98. (a) By analysing the above table,
S.P. of rice organised on
place. it is clear that Lecture F
5632 × (100 + 25) 89. (b) According to question,
= Thursday.
100 R<S<T<Q<P 99. (a) By analysing the above table,
5632 × 125 5632 × 5 Finally, R is the shortest among five
= = it is clear that Last lecture i.e. 7th
100 4 boys. in the series
Lecture A.
= 1408 × 5 = ` 7040 90. (a) E +4 +4 +4
→ I  → M  → Q 100. (d) By analysing the given
7040 +4 +4 +4 information, it is clear that all the
Rate of selling rice = = ` 550 D  → H  → L  → P
12.8 +4 +4 +4 statements are required in finding the
per quintal C  → G  → K  →O
complete sequence of organisation of
84. (b) Let two numbers be x and y +4
B  +4
→ F  +4
→ J  → N lectures.
xy = 12 (i) Finally, the missing term is QPON. qqq
90 Practice Set-8

Air Force Airmen Group X & Y Exam


Practice Set-8

1. The information presented here (a) They had been laughing at the
ENGLISH about Birju Maharaj can be clown
Directions (1-4): Read the given found in a/an (b) They were laughing at the clown
passage and answer the questions (c) They laughed at the clown
(a) newspaper article
that follow by selecting the most (d) They were laughing on the
appropriate option. (b) diary
clown
He has reservations on the treatment (c) encyclopedia
7. Find out the correct sentence.
of dance in Indian films, but given (d) autobiography (a) Alm is given to the begger
a chance to work on his own terms, 2. The observation that ‘dance is (b) Alms is given to the beggars
legendary Kathak Dancer Pandit Birju adulterated’ means, that the (c) Alms are given to the beggars
Maharaj would like to work more in dance form is (d) None of these
Bollywood. The 75-year-old tells us,
(a) not practiced according to 8. Which of the following words is
“In my opinion, dance is adulterated in
tradition the “Synonym” of “Apathetic”?
Bollywood. To make it more dramatic,
(b) found in adult entertainment (a) Hostile (b) Callous
the dancers are asked to perform in
(c) Repulsive (d) Ominous
an exaggerated manner. That makes (c) performed only in films
any kind of dance impure, especially 9. Point out the Indirect speech of
(d) suitable to be performed by the given sentence.
classical dance. I’d like to work more in adults ‘O, God do not put me to the test,”
Hindi films; provided my dance is not
3. H e r e , “ t o p e r f o r m i n a n said I
tampered with.”
exaggerated manner” suggests (a) I Prayed to God not to put me
The Kathak maestro tells us
that performers. to the test.
that over the years he’s been highly
(a) are not professionally trained (b) I cried to God to not put me to
impressed with how some female actors
the test.
have showcased classical dance on (b) deliberately distort the dance
screen. On being asked on how he sees (c) I requested God to not put me
form
to the test.
the passion for dance among youngsters (c) only dance for a selected
in the country, Birju Maharaj says, “I (d) I asked God to not put me to the
audience test.
see that the young generation is divided
(d) cannot dance 10. Point out the Direct Speech of the
in their response to classical dance. But
in all my interactions with the younger 4. A ‘packed house’ during his given sentence.
performance suggests that it was The traveller enquired of the
lot, I have been impressed. These
(a) jammed in tightly farmer if he could tell him the way
children have such amazing presence to the nearest inn.
of mind, listening and learning while (b) filled into
(a) The traveller said to the farmer,
I talk and teach them.” It is often said (c) exceeding allotted time “Please tell me which way is the
that classical dance doesn’t receive due (d) well-attended nearest inn?”
credit, but the man who is an authority 5. Find out the correct sentence: (b) The traveller said to the farmer,
on the subject thinks Delhi receives (a) He is rich enough to starve “Tell me where is the way to
the art well. “I feel that classical dance nearest inn?”
(b) He is too rich to starve
might not be on a rise, in popularity, but (c) The traveller said to the farmer,
(c) He is too rich not to starve “Can you tell me where is way
I have always been overwhelmed by the
response that I have received in Delhi. (d) None of the above to the nearest inn?”
My performances have always 6. Point out the Active Voice of the (d) The traveller said to the farmer,
been applauded by packed houses in the given sentence: The clown was “Can you tell me the way to the
Capital,” he opines. being laughed at by them. nearest inn?”
Practice Set-8 91

11. Which is the correct spelling? 19. In the following question, choose 24. The graph between restoring
(a) address (b) adress the word opposite in meaning to force and time in case of SHM is:
(c) adrres (d) addres the given bold word. (a) a straight line (b) a circle
12. Find out the misspelt word. Omit (c) a parabola (d) a sine curve
(a) disparage (b) licentious (a) exclude 25. A rectangular frame is to be
(c) malleable (d) dilema (b) include suspended symmetrically by two
Direction (13): Fill in the blank with (c) undertake strings of equal length on two
appropriate option out of the four (d) add supports (figure). It can be done
altenatives. 20. Choose the most appropriate in one of the following three ways:
13. There is hardly_______reason to preposition out of four options :
doubt his intentions.
Her complaints _____ headache.
(a) some (b) much
(a) of (b) off
(c) every (d) any
(c) about (d) from
14. Fill in the blank with Auxiliaries.
He_______ride a bicycle. (ability) (a) (b) (c)
PHYSICS
(a) do (b) can The tension in the strings will be:
21. Two blocks of masses m1 = 4 kg
(c) dare (d) need (a) the same in all cases
and m2 = 2 kg are connected to the
Direction (15): In the following (b) least in A
ends of a string which passes over
question, out of the four alternatives, (c) least in B
a massless, frictionless pulley. The
choose the one which can be substituted
total downward thrust on the (d) least in C
for the given words/sentence.
15. That which lasts for a short time. pulley is nearly: 1
(a) 27 N (b) 54 N 26. An engine has an efficiency of
(a) Regular (b) Transitory 6
(c) Rotatory (d) Repository (c) 2.7 N (d) None of these When the temperature of sink is
Direction (16): In the following 22. A force F acting on an object
reduced by 62°C, its efficiency is
question, the following sentences have varies with distance x as shown
doubled. Temperature of the
been given in Active/passive voice. here. The force is in Newton and x
is in metre. The work done by the source is:
From the given alternatives, choose
force in moving the object from x (a) 124° C (b) 37° C
the one which best expresses the given
sentence in Passive/Active voice. = 0 to x = 6 m is: (c) 62° C (d) 99° C
16. We all know that there is only one F(N) 27. As a result of change in the
God. magnetic flux linked to the closed
3
(a) We are all known that there is loop shown in the figure, an e.m.f.
2
only one God.
1 V. V is induced in the loop. The
(b) It is known to us all that there
0 x (m ) work done (J) in taking a charge
is only one God. 1 2 3 4 5 6 7
(c) We have all known that there is q C once along the loop is:
(a) 4.5 J (b) 13.5 J
only one God. (c) 9.0 J (d) 18.0 J
(d) Only one God is known by us 23. A small object of uniform density
all.
rolls up a curved surface with an
Directions (17-18): In the following initial velocity v. It reaches upto
questions, some of the sentences have
3v 2
errors and some have none. Find out a maximum height of with
which part of a sentence has an error. 4g (a) qV (b) Zero
If there is no error in the sentence, respect to the initial position. The qV
(c) 2qV (d)
then mark (d) as your answer. object is: 2
17. To die with honour (a)/is better (a) ring 28. Consider the following two
than (b)/live with dishonour. (c)/No (b) solid sphere statemetns regarding a linearly
Error (d). (c) hollow sphere polarised place electromagnetic
18. It is I (a)/who is to blame. (b)/for (d) disc wave:
this bad situation (c)/No Error (d).
92 Practice Set-8

(i) Electric field and the magnetic 1 β


2 2
(a) 2 ρA ω
2 2
field have equal average (b) ρA ω (c) (µ – 1)t  λ  , towards A on the
 
values 2 2
(c) 2ρA ω
2 2
(ii) Electric energy and the (d) 4ρA ω screen
β
magnetic energy have equal 33. Identify the wrong statement (d) (µ – 1)t   , towards B on the
average values in the following Coulomb’s law λ
(a) (i) is true correctly described the electric screen
force that: 37. For the ground state, the electron
(b) (ii) is true
(a) binds the electrons of an atom
(c) Both are true in the H-atom has an angular
to its nucleus
(d) Both are false momentum = h, according to
(b) binds the protons and neutrons
29. The correct value of the dip angle the simple Bohr model. Angular
in the nucleus of an atom
at a place is 45°. The dip circle momentum is a vector and hence
(c) binds atoms together to form
is rotated through 45° out of the molecules there will be infinitely many orbits
magnetic meridian. The apparent (d) binds atoms and molecules to with the vector pointing in all
angle of dip will be: form solids possible directions. In actuality,
this is not true:
(a) tan
−1
( )
2 −1  1 
(b) tan 
 2

34. An electric kettle has two heating
coils. When one of the coils is (a) because Bohr model gives
connected to an AC source, the incorrect values of angular
−1  1 
water in the kettle boils in 10 min. momentum
(d) tan  2 
−1
(c) tan (2)
When the other coil is used the (b) because only one of these would
30. The ratio of specific heats (γ) of water boils in 40 min. If both the have a minimum energy
an ideal gas is given by: coils are connected in parallel, the
(c) angular momentum must be in
1 time taken by the same quantity
(a) of water to boil will be: the direction of spin of electron
R
1− (a) 8 min (b) 4 min (d) because electrons go around
Cp
(c) 25 min (d) 15 min only in horizontal orbits
R 35. A square current carrying loop is 38. How many gates are required to
(b) 1 +
Cv suspended in a uniform magnetic design P = X + XY ?
Cp field acting in the plane of the (a) 1 (b) 2
(c) loop. If the force on one arm of (c) 3 (d) 4
Cp − R →
the loop is F , the net force on the 39. Beyond which frequency, the
(d) All of the above ionosphere bends any incident
remaining three arms of the loop
31. The density of newly discovered is: electromagnetic radiation but do
planet is twice that of Earth. The → → not reflect it back towards the
(a) 3 F (b) – F earth?
acceleration due to gravity at the

surface of the the planet is equal (c) –3 F (d) F

(a) 50 MHz (b) 40 MHz
to that at the surface of the earth. (c) 30 MHz (d) 20 MHz
36. In Young’s double slit experiment
If the radius of the earth is R, the A and B are two slits. A thin film 40. The combination of the gates
radius of the planet would be: of thickness t and refraction shown in the figure below
µ is placed in front of slit A. produces:
(a) 2 R (b) 4R
1 If the fringe width is β. Then, A
(c) 1 R (d) R displacement of maxima will
4 2
be: Y

32. A sound wave y = A sin (ωt – kx) β


(a) µt   , towards A on the screen B
is propagating through a medium λ
of density ρ. What is the sound (a) NOR gate (b) OR gate
β
energy per unit volume? (b) µt   , towards B on the screen (c) AND gate (d) XOR gate
λ
Practice Set-8 93

41. An electron is moving with an (c) (c) V



intitial velocity v = v0 iˆ and is in
→ v
ˆ
a magnetic field B = B 0 j . Then
it’s de-Broglie wavelength:
(a) remains constant u
O
T/2 t T
(b) increases with time
(c) decreases with time (d)
(d) i n c r e a s e s a n d d e c r e a s e s v (d) V
periodically
42. At a metro station, a girl walks
u
O
up a stationary escalator in time
t1. If the remains stationary on the 45. The current (I) in the inductance t
escalator, then the escalator take is varying with time according to T
T/2
her up in time t2. The time taken the plot shown in figure.
by her to walk up on a moving I
escalator will be:
MATHEMATICS
(t1 t2 ) 46. The set which is comparable with
(a) (t1 + t2 ) / 2 (b) {1, 2, 3, 4} is the set out of the
(t2 − t1 )
T/2 following:
(c) (t1 t2)/(t2+t1) (d) t1 – t2 (a) {3, 4, 5} (b) {2, 3, 5}
43. If momentum (p), area (A) an time (c) {1, 2, 3} (d) {4, 5, 6}
t T 47. If 1, ω and ω are the cube roots
2
(T) are taken to be fundamental
quantities, the energy has the Which one of the following is the of unity, then the roots of the
equation (x – 1)3 + 8 = 0, are:
dimensional formulae: correct variation of voltage with
(a) – 1, 1 + 2ω, 1 + 2ω2
(a) [PA–1T1] (b) [P2A1 T1] time in the coil? (b) – 1, 1 – 2ω, 1 – 2ω2
1 1

1 1
(c) [P A T ] 2 1 −1
(d) [P A T ]
2 (a) I (c) – 1, – 1, – 1
(d) None of these
44. The graph between the image
48. If relation R = {(a, c) (b, d), (c, c)},
distance (v) and object distance
then domain and range are:
(u) from the convex lens is: (a) {a, b, c}, {a, b, c}
(a) t (b) {a, c, a}, {c, b, d}
T/2 T (c) {a, b, c}, {c, d}
v (d) {c, d}, {a, b, c}
49. There are r arithmetic mean
O (b) I terms between 2 and 41. If ratio
u between 4th and (r – 1)th arithmetic
(b) mean terms is 2: 5, then the value
of r will be:
v (a) 12 (b) 11
(c) 10 (d) 8
50. If z = y + y + y + ... ∞ and y is
2 3

T/2 T positive and less than one, then


O u t
value of y will be:
94 Practice Set-8

1+ z 57. The equation of the circle is x2 + (c) e2x (x2 + y2) = C


z
(a) (b) y2 + 3x – 3y + 2 = 0. A point inside (d) ex (x2 – y2) = C
1+ z z
the circle is: 65. The value of cos (tan–1 x) is:
z 1− z (a) (– 1, 3) (b) (– 3, 2)
(c) (d) (a) 1 + x2 (b) 1 + x2
1− z z (c) (– 2, 1) (d) (2, 1)
51. How many words are formed 58. If in triangle ABC, angle C is right 1
(c) (d) None of these
from the word EQUATION, angled, then the value of R + r will 1 + x2
which start from consonant and be:
66. The value of lim (cosec x) x is:
finish with consonant? 1 x→ 0
(a) (a + b) (b) (a + b)
(a) 4320 (b) 2470 2 (a) 1 (b) e
(c) 3260 (d) 4260 1 1 (c) e–1 (d) 0
(c) (a + b) (d) (a + b)
52. The relationship between x and y 3 4 67. A problem in Mathematics given
as shown in table, is: 59. The expenditures of 100 families to 3 students A, B, C whose
x 0 1 2 3 4 are given below: changes of solving problem
y 50 90 70 40 0 Expenditure No. of families are 1/2, 1/3, 1/4 respectively.
0 – 10 14 Probability that the problem will
(a) y = 100 – 10x
10 – 20 x1 be solved, is:
(b) y = 100 – 5x – 5x2
20 – 30 27 (a) 1/4 (b) 1/24
(c) y = 100 – 5x2
30 – 40 x2 (c) 23/24 (d) 3/4
(d) y = 20 – x – x2
40 – 50 15 π /2
a h g 68. The value of ∫0 sin x sin 2 x dx is:
53. If A = [x y z], B =  h b f  and The mode of the distribution is 25, 4 1
 g f c 
then the missing frequencies are: (a) (b)
5 3
 x (a) 22 and 22 (b) 23 and 21
C =  y  , then [ax2 + by2 + cz2 + 3 2
 z  (c) 24 and 20 (d) None of these (c) 4 (d)
60. If each side of an equilateral 3
2hxy + 2gzx + 2fyz] is: 69. Standard deviation of 1, 4, 5, 7
triangle is 2 3 cm, then its
(a) (BA) C (b) (AB)C and 8 is 2.45. If 10 is added to each
exradius will be:
(c) (AC) B (d) None of these (a) 3 (b) 4 term, what is the new standard
54. The coefficient of xn in expansion (c) 7 (d) 11 deviation?
x + 1 − 2 x2 61. The focus of the parabola y2 = 4(y (a) 2.45 (b) 24.5
of will be: (c) 0.245 (d) 12.45
(1 − x) 3 – x) is:
(a) (1, 2) (b) (0, 2) 70. ∫ 4 x 2 − 4 x − 1 dx is equal to:
(a) n + 1 (b) 2n + 1
(c) (2, 0) (d) (2, 1)
(c) 3n + 1 (d) 3n + 2 1
62. The centre of the hyperbola (a) (2 x − 1) 4 x 2 − 4 x − 1
55. The angle, whose sine is − 1 , is: 9x2 – 16y2 + 18x + 32y – 151 = 0, 4
2
is: 1
25π + log{(2 x − 1) + 4 x 2 − 4 x − 1}
(a) radians (a) (1, 1) (b) (– 1, 1) 2
3
(c) (1, – 1) (d) (1, 2) 1
(b) – 1680° (degrees) (b) (2 x − 1) 4 x 2 − 4 x − 1
63. The nearest point on the line 3x – 4
(c) 2402 (grades) 4y = 25 from the origin is: 1
(d) 1290° (degrees) (a) (3, – 4) (b) (– 2, 1) − log{(2 x − 1) + 4 x 2 − 4 x − 1}
2
56. If sec A + tan A = t, then cos A =? (c) (3, 4) (d) (4, 3)
1
2t 1+ t2 64. Solution of the differential (c) (2 x − 1) 4 x 2 − 4 x − 1
(a) (b) equation 4
1+ t2 2t
(x2 + y2 + 2x) dx + 2y dy = 0 is: + log{(2 x + 1) + 4 x 2 − 4 x − 1}
1− t2 t (a) ex (x3 + y3) = C
(c) (d) (d) None of these
1+ t2 1+ t2 (b) ex (x2 + y2) = C
Practice Set-8 95

Technology Management in the ratio 1 : 2 : 5. How much


REASONING AND GENERAL does Chiranjeev get?
AWARENESS (RAGA) Agencies (ATMA)?
(a) 2022 (b) 2020 (a) ` 46800 (b) ` 23400
71. Primary Agricultural credit (c) 2025 (d) 2030 (c) ` 117000 (d) ` 58500
Societies (PACSs) were started 78. The ten's digit of a 2-digit 85. A car travels a certain distance
to provide: number is greater than the units at 34 km/h and comes back at 66
(a) Refinancing facilities to rural digit by 7. If we subtract 63 from km/h. What is the average speed
borrowers the number, the new number for total journey ?
(b) Short-term loans to rural obtained is a number formed by (a) 50 km/hr
borrowers interchange of the digits. Find (b) 39.76 km/hr
(c) Medium-term loans to rural the number. (c) 55.12 km/hr
borrowers (a) 81 (b) 18 (d) 44.88 km/hr
(d) Long-term loans to rural (c) 62 (d) 26 86. If P × R – Q, which of the
borrowers 79. Three numbers which are co- following is true?
prime to each other are such that (a) P is the brother of
72. Annie Besant belonged to the:
the product of the first two is 551 (b) P is the father of
(a) Gadar Party and that of the last two is 1073. (c) P is the brother-in-law of Q
(b) Arya Samaj The sum of the three numbers is: (d) P is the uncle of Q
(c) Prarthana Samaj (a) 75 (b) 81 87. The word ‘UNITED’ is coded as
(d) Theosophical Society (c) 85 (d) 89 ‘SLGRCB’. How should the word
73. Trees are leafless for a shorter or 80. Which of the following values of ‘DISOWN’ be coded ?
longer season of the year in: x and y satisfy the following (a) BGQMUL (b) CGRLTK
(a) Deciduous forests equations I and II?
(c) CGRTLK (d) BGQLUM
(I) 2x + y = 15 (II) 3x – y = 25
(b) Evergreen forests 88. A man moves 24 metres in south
(a) – 8, 1 (b) –8, –1
(c) Mangrove forests direction and turns 90 degrees
(c) 8, – 1 (d) 8, 1
(d) Scrub forests anticlockwise and moves another 7
81. Of the 3 numbers whose average
74. Which of the following devices metres and takes a right turn and
is 70, the first is 1/9 times the sum
converts electrical energy into moves 3 metres and then moves 3
of other two. The first number is
mechanical energy? metres in the north direction. Find
:
the distance between his initial
(a) Inductor (a) 32 (b) 21
and his final position.
(b) Dynamo (c) 14 (d) 42
82. ∆DEF is similar to ∆GHI. Length (a) 25 m (b) 30 m
(c) Transformer
of DE is 16 cm and length of the (c) 27 m (d) 35 m
(d) Electric motor
corresponding side GH is 25 89. P, Q, R, S and T are standing in a
75. Which of the following articles of cm. What is the ratio of areas of line. P is taller than S but shorter
IPC is related to unnatural sex? ∆DEF : ∆GHI ? than Q. T is shorter than S but
(a) 376 (b) 377 (a) 4 : 5 (b) 16 : 25 taller than R. Who is the second
(c) 370 (d) 374 (c) 256 : 625 (d) 25 : 16 tallest of them all?
76. By default, to run an application 83. A rice trader buys 20 quintals of (a) T (b) S
through mouse, which one of the rice for ` 8,580. However, 22% (c) R (d) P
following has to be done? rice is lost in transportation. At Direction (90): A series is given with
(a) Single click what rate should he sell to earn one term missing. Choose the correct
(b) Double click 20% profit? alternative from the given ones that
(c) Right click (a) ` 278.8 per quintal will complete the series.
(d) Left click (b) ` 514.8 per quintal 90. 13, 21, 48, 112, ?
77. Which year is set as a target to (c) ` 660 per quintal (a) 237 (b) 280
double farmers income using (d) ` 654.2 per quintal (c) 255 (d) 290
digital technology in rural India 84. Profit of ` 187200 has to be Direction (91): Study the following
and setting up Krishi Vigyan divided among three partners information carefully and answer the
Kendras (KVK) & Agricultural Amit, Brijmohan and Chiranjeev questions given below:
96 Practice Set-8

A, B, C, D, E, F, G and H are sitting 95. Which of the following pairs of


(d) Neither conclusion I nor
around a circle facing the centre. H is words have the same relationship conclusion II follows
fourth to the left of B and second to as Fan : Heat : :? 98. Statement:
the right of F. A is third to the left of (a) Water : Drink I: No cans are jars
C, who is not an immediate neighbour (b) Light : Night
of F. G is second to the left of A. D is II: All tins are cans
(c) Teach : Student Conclusion:
second to the right of E.
91. In which of the following pairs (d) Food : Hunger I: All jars are tins
is the first person sitting on the Direction (96–100): In the question
II: No tins are jars
immediate left of the second two statements are given, followed by
person? (a) Only conclusion I follows
two conclusions, I and II. You have
(a) EH (b) CE to consider the statements to be true (b) Only conclusion II follows
(c) AF (d) DB even if it seems to be at variance from (c) Both conclusions I and II
Direction (92): In following questions commonly known facts. You have to follow
you have to identify the responses decide which of the given conclusions, if (d) Neither conclusion I nor con-
which would be a correct inference any, follows from the given statements. clusion II follows
from the given premises stated 96. Statement I: Some roses are 99. Statement I: All pictures are
according to the following systems: flowers photographs
'A' stands for Not greater than. II: All paintings are pictures
Statement II: Some red are roses
'B' stands for Greater than Conclusion I: All photographs are
Conclusion I: Some flowers are
'C' stands for Not equal to paintings
red
'D' stands for Equal to
Conclusion II: All red are flowers II: All paintings are photographs
'E' stands for not less than
(a) Only conclusion I follows (b) Only conclusion II follows
'F' stands for less than.
(b) Only conclusion II follows (c) Both conclusions I and II
92. Premises: (a E b) and (l A m)
(c) Both conclusions I and II follow
(a) l B m (b) l D m
follows (d) Neither conclusion I nor
(c) l E m (d) l F m
conclusion II follows
Direction (93): Select the one which (d) Neither conclusion I nor
is different from the other three conclusion II follows 100. Statement I: Some clouds are fog
responses. Statement II: All clouds are mist
97. Statement I: No steam are vapours
93. (a) Wheat (b) Mustard Conclusion I: Some fog are mist
Statement II: All smoke are steam
(c) Barley (d) Corn Conclusion II: All mist are fog
Directions (94): Arrange the given Conclusion I: Some vapours are
smoke (a) Only conclusion I follows
words in the sequence in which they
Conclusion II: No smoke are (b) Only conclusion II follows
occur in the dictionary.
94. i. Serious ii. Senior vapours (c) Both conclusions I and II
iii. Senate iv. Statesman (b) Only conclusion II follows follow
(a) iii, ii, i, iv (b) i, ii, iii, iv (c) Both conclusions I and II (d) Neither conclusion I nor
(c) iv, i, ii, iii (d) i, iii, ii, iv follow conclusion II follows

Answers with Explanation


ENGLISH 7. (c) Alms are given to the beggars is correct spelling is ‘dilemma’ which
correct statement. ‘Alms’ is always used means any difficult or perplexing
1. (a) 2. (a) 3. (b) 4. (d) in the plural form and take a plural verb. situation or problem.
5. (b) If sentence having ‘too’ contains 8. (b) Synonym of Apathetic is Callous. 13. (d) The correct option is ‘any’ i.e.
no negative word the subordinate clause Its meaning is feeling or showing a lack there is hardly any reason to doubt his
becomes negative. If however there is of interest or concern. intentious.
a negative word in the sentence, the 9. (a) 10. (d) 14. (b) Ability is expressed by ‘can’
subordinate clause become positive. 11. (a) The correct word is “address.” auxiliary, so, He can ride a bicycle. Hence
6. (b) 12. (d) Mis-spelt word is dilema, option (b).
Practice Set-8 97

15. (b) 16. (b) k2 3 1 W


17. (c) The error lies in the part (c) of ⇒ = −1= ⇒ V= ⇒ W = Vq
r2 2 2 q
the sentence. The group of words. ‘live
1
with dishonour’ should be replaced with k2 = r2 (Equation of disc) 28. (b) Average energy is equally
‘to live with dishonour.’ 2
Hence, the object is disc. shared by electric x magnetic field.
18. (b) If a sentence has Subject and
Relative pronoun (who, which, that), However, their average magnitude is not
24. (d)
then verb agrees with Antecedent. A equal:
Noun or Pronoun which comes before F = Ma = Mω 2 x = Mω 2 A s in ωt. 1
As, | B0 | = | E 0 |
Relative pronoun is called Antecedent. Hence, graph between F and a is a sine C
For example. curve. V
I am the man who has helped you. 29. (a) tan δ = = tan 45° .
25. (c) If m be the mass of block, then: H
  ↓ ↓
Antecedent Verb T T Hence, V = H. When rotated through
It is I who have helped you. T sin θ T sin θ
45° out of magnetic meridian, the vertical
↓ ↓ θ θ
Antecedent Verb component remains unchanged, but the
T cos θ T cos θ
Therefore, ‘who is to blame’ should horizontal component changes to:
be replaced with ‘who am to blame’, as H V
‘I’ is the Antecedent. Hcos 45° = . Hence, tan δ ' = = 2
2 H
19. (b) The word ‘omit’ (Verb) 2
means ‘exclude’. Hence, its antonym is
mg 30. (d) Cp – Cv = R
‘include’.
Dividing the equation by Cv, we get:
20. (a) mg 1 Cp
2T sin θ = mg ⇒ T = ⇒ T∝ R R R
2sin θ sin θ −1= ⇒ γ −1= ⇒ γ =1 +
Cv Cv Cv Cv
PHYSICS
1 Cp Cp
21. (b) So, in fig (B) where sin θ = sin 90° = 1 = = =γ
R Cp − R Cv
is maximum, tension in string will be 1−
2m1m2 2 × 4 × 2 × 10 Cp
T= g=
m1 + m2 4+2 minimum. 31. (d)
26. (d) Efficiency of engine is given Gm
160 G = 2 Given, ρplanet = 2ρEarth
= = 26.6  27 N T R
by η = 1 − 2
6 T1 Also,
Total downward thrust on the pulley T2 1 5 GM p GM e
= 2T = 2 × 27 = 54 N  T =1− η =1− 6 = 6 g Planet = g Earth ⇒ =
22. (b) Work done in moving the 1 R ρ2ˆ R e2
object from x = 0 to x = 6 m is given by In other case, 4 3 4
W = Area of rectangle + Area of triangle G× πR ρˆ ρρˆ G × πR 3e ρe
T2 – 62 2 2 ⇒ 3 = 3
=1− η =1 − = R ρ2 R e2
1 T1 6 3
= 3 × 3 + × 3 × 3 = 9 + 4.5 = 13.5 J
2
2 2 6 1 ⇒ R ρˆ ρρ = R e ρe
T2 − 62 = T1 = × T2 ⇒ T2 = 62
2 gh 3 3 5 5
⇒ R ρˆ × 2ρe = R e ρe
23. (d) As, v =
k2 \ T2 = 310 K
1+ 2 Re R
r ⇒ R ρˆ = =
6 6 2 2
Given, Here T1 = T2 = × 310
5 5 32. (a) Intensity of the wave
3v 2 2 2 gh 2 g 3v 2 = Energy crossing per unit area per
h= ,v = = = 372 K = 372 − 273 = 99°CC
4g k2  k2  second
1+ 2 4 g 1 + 2  27. (a) Induced emf
r  r  1
Work done in taking a charge = ρcω2 A 2
6 gv 2 3 k2 3 2
=   ⇒1= ⇒1+ = q once along the loop
 k 
4 g  (1 + 2 
2
 k 
2
2 1 + 2 
r2 2 = This energy is contained in volume c
 r   r  charge q × 1.
98 Practice Set-8

Hence, energy density: It gives only the magnitude of angular Applying principle of homogeneity to
1 1 momentum, which is a vector. So, the get,
= ρcω2 A 2 / c = ρw2 A 2
2 2 given statement in the question is not a = 1, a + 2b = 2 and − a + c = − 2
33. (b) Coulomb’s law is applicable true. solving it to get,
for charge particles, it is not responsible 38. (a) P = X + XY = X (1+ Y) + XY
1
to bind the protons and neutrons in the = X + (X + X)Y a = 1, b = and c = − 1
2
nucleus of an atom.
V2   = X + 1 . Y = X + Y 1

34. (a) H = t \ E = k[P1A 2 T –1 ]


Hence, one gate i.e., OR gate.
R
39. (b) The ionosphere can reflect 44. (a) For convex lens as object
H t
⇒ 2 = = constant electromagnetic waves of frequency comes closer to lens image runs away
V R from it and also object distance is always
∴ t∝R less than 40 MHz but do not reflect
negative.
R1R 2 electromagnetic waves of frequency
∵ Rp = 45. (d) For inductor as we know
R1 + R 2 more than 40 MHz.
induced voltage
t2 40. (b) Y = A . B = A + B (OR gate) For
∴ tp = T
t1 + t2 → →
t = 0 to t =
41. (a) Here V = v 0 iˆ , B = B0 ˆj 2
10 × 40
= = 8 min Force on moving electron due to dl d  2I0 t 
10 + 40 V=L =L = constant,
magnetic field is: dt dt  T 
35. (b) When a current carrying loop → → →
F = − e ( v × B) = − e[v0 iˆ × B0 ˆj ] = − ev0 B0 kˆ T
is placed in a magnetic field, the coil For t = to t=T
experiences a torque given by
→ 2
As this force is perpendicular to v and LdI  2I0 t 
t = NBiA sin θ. Torque is maximum → → V=  T  = − constant
when θ = 90° i.e., The plane of the coil B , so the magnitude of v will not dt
is parallel to the field τ max = NBiA. change i.e. momentum Therefore, answer will represents by
F1 ( = mv) will remain constant in magnitude. graph (d).
h
Hence de-Broglie wavelength λ =
mv MATHEMATICS
remains constant. 46. (c)  {1, 2, 3} ⊂ {1, 2, 3, 4}, hance
F4 (1, 2, 3} and {1, 2, 3, 4} are comparable.
F3 L
42. (c) Velocity of girl, vg = 47. (b) We have, (x – 1)3 + 8 = 0
t1
⇒ (x – 1)3 = – 8
L ⇒ x – 1 = (– 8)1/3
Velocity of escalator, ve =
t2 ⇒ x – 1 = 2(– 1)1/3
F2
Effective velocity, of girl while on ⇒ x – 1 = 2(– 1)
36. (c) Displacement of fringes: ⇒ x – 1 = 2(– ω)
escalator
∆x = xn − ( xn ) D ⇒ x – 1 = 2(– ω2)
L L
D nDλ = vg + ve = + { (– 1) = – 1 or, – ω, or, – ω}
1/3

= [nλ + ( µ − 1)t ] − t1 t2
d d ⇒ x – 1 = 2(– 1)
Time taken, ⇒ x – 1 = – 2ω
D
= (µ − 1)t L L t t ⇒ x – 1 = – 2ω2
d t= = = 1 2 Hence, the solution of the equation
vg + ve L L t1 + t2
+ (x – 1)3 + 8 = 0 or – 1, 1 – 2ω, and
D β t1 t2
we have, = 1 – 2ω2
d λ 43. (d) Let energy
β 48. (c) Since, domain is the set of first
∆x = (µ −1)t E ∝ P a A b T c = kP a A b T c elements of ordered pair and range is the
λ
where k, is dimensionless constant set of second elements of ordered pairs.
37. (a) According to Bohr model, Given that R = {(a, c), (b, d), (c, c)}
  ⇒ [M1L2 T –2 ] = [MLT −1 ]a [L2 ]b T c
infact does not give correct values of ∴ Doman of R = {a, b, c} and
angular momentum of revolving electron.        = M a La + 2b T − a + c Range of R = {c, d}
Practice Set-8 99

49. (a) Number of terms = r + 2 = sin 210°


a h g
According to question, = sin (180 + 30)
53. (b) AB = [ x y z ]  h b f 
(r + 2)th term = 41  = – sin 30
= 2 + (r + 2 – 1)d  g f c 
1
⇒ 39 = (r + 1)d = [ax + hy + gz hx + by + fz gx + fy + cz] = −
2
39 (AB).C = [ax + hy + gz hx + by + fz
∴ d = 56. (a)  sec2 A – tan2 A = 1
r +1  x
and sec A + tan A = t
Fourth arithmetic mean term gx + fy + cz ]  y 
4 × 39 ∴(sec A – tan A) (sec A + tan A) = 1
 z 
= 2 + ⇒ (sec A – tan A)t = 1
r +1
= [ax2 + by2 + cz2 + 2hxy + 2fyz + 2gzx] 1
(r – 1) arithmetic mean term
th
⇒ sec A – tan A =
54. (c) Given expression = (1 + x – t
(r − 1) × 39
= 2 + 2x2) (1 – x)–3 But, sec A + tan A = t
r +1
Let (1 + x – 2x2) (1 – x)–3 1
According to question, ∴ 2 sec A = t +
= (1 + x – 2x2) (1 + a1x + a2x2 + ... + t
2(r + 1) + 156 r +1
× = 2 an xn + ...)  ...(i)
t2 +1
r +1 41r − 37 5 On multiplying, coefficient of x n
=
t
⇒ 10r + 790 = 82r – 74 = an + an – 1 – 2.an – 2
⇒ 72r = 864 Now, ar = coefficient of (r + 1)th term t2 +1
∴ sec A =
864 in expansion of (1 – x)– 3 2t
∴ r = = 12
72 ( −3)( −3 − 1)( −3 − 2)...[ −3 − (r − 1)] 2t
= ( −1) r ∴ cos A =
y r! 1+ t2
50. (a) S∞ = 57. (c) The coordinates of the centre
1− y 2r 3·4·5...(r + 2)
= ( −1) of the circle x2 + y2 + 3x – 3y + 2 = 0, are
y r!
⇒ z =  3 3  and its radius
1− y 3·4·5... r (r + 1) (r + 2)  − , 
= 2 2
⇒ z – zy = y 1·2·3·4·5 ... r
⇒ z = y + zy 9 9
[ (– 1)2r = 1] = + − 2 = 2.5.
⇒ z = y(1 + z) 4 4
(r + 1) (r + 2)
z = The distance of the point (– 2, 1) from
∴ y = 1·2
1+ z centre
(r + 1) (r + 2)
51. (a) There are total 8 letters in the =  ...(ii)  3 3 2 2
2  − ,  =  −2 + 3  + 1 − 3 
given word out of which 3 are consonants 2 2  2  2
(Q, T, N) and 5 are vowels (A, E, I, O, U) ∴ From equation (ii)
we have to form a word of 8 letters which (n + 1) (n + 2) 1 1
an= (on putting r = n) = + = 0.5
have first and last letter consonants. 2 4 4
In beginning and in the last we put 2 n(n + 1)  2.5 > 0.5
consonants out of 3 by 3P2 = 6 methods. and an – 1 =
2
Therefore, the point (– 2, 1) is inside
Now, in between first and last rest 6 (n − 1)n
an – 2 = the circle.
letters out of 6 letters may be put in 6P6
2 58. (b)  C = 90°
= 6 ! types. Therefore, number of words
(putting r = n – 1 and n – 2) ∴ C = 2R sin C
comprises consonant in first and in last
Required coefficient of xn = 2R sin 90°
will be
(n + 1)(n + 2) n(n + 1) (n − 1)n 1
= 6 ! × 6 = + −2 = 2R of R = C
2 2 2 2
= 720 × 6 = 4320
52. (b) Since (0, 50), (1, 90), 1 C
= (n + 1)[(n + 2) + n] − (n − 1)n and r = (s – c) tan
(2, 70), (3, 40) and (4, 0) satisfy 2 2
y = 100 – 5x – 5x2, hence statement (b) = 3n + 1 = (s – c) tan 45°
is correct. 55. (d) sin 1290 = sin (3 × 360 + 210) = (s – c)
100 Practice Set-8

1 C 4
∴ R+r = c + ( s − c) r3 = s tan y – 0 = − ( x − 0)
2 2 3
1 1 ⇒ 4x + 3y = 0
= (2 s + c − 2c) = 3 3 × =3
2 3 Now, on solving
1 3x – 4y = 25 and 4x + 3y = 0, we get
= (2 s − c) 61. (b) y = 4(y – x)
2

2 x = 3, and y = – 4
⇒ y2 = 4y – 4x
∴ The reqd. point is (3, – 4).
1 ⇒ y2 – 4y + 4 = – 4x + 4
= [(a + b + c) − c] 64. (b) Here, M = x2 + y2 + 2x, N = 2y
2 ⇒ (y – 2)2 = – 4(x – 1) ...(i) ∂M ∂N
= (a + b)
1 Now, putting x – 1 = X and y – 2 = Y ∴ = 2 y , ∂x = 0
∂y
2 in equation (i), we get Y2 = – 4X
59. (a) Let that maximum frequency 1  ∂M ∂N  1
 Focus X = – 1, Y = 0 ⇒ x – 1 = – 1 − (2 y − 0)
be 27. N  ∂y ∂x  = 2 y
and y – 2 = 0 or, x = 0 and y = 2
Therefore, modal group = (20 – 30) = 1
∴ Coordinates of the focus are (0, 2).
f − f1 which is a function of x.
∴ Mode = l1 + (l2 − l1 ) 62. (b) 9x2 – 16y2 + 18x + 32y – 151 ∫ 1.dx
2 f − f1 − f 2 I.F. = e
=0 = ex
Here, l1 = 20, ⇒ 9(x2 + 2x + 1) – 16(y2 – 2y + 1) Multiplying by I.F., the given equation
l2 = 30,
= 151 + 9 – 16 is
f = 27,
⇒9(x + 1)2 – 16(y – 1)2 = 144 ex (x2 + y2 + 2x) dx + ex 2y dy = 0
f1 = x1,
which is exact equation and its
f2 = x2 ( x + 1) 2 ( y − 1) 2
⇒ − =1 solution is
27 16 9
∴ 25 = 20 + ∫ e x ( x 2 + y 2 + 2 x) dx + ∫ 0.dy = C
2 × 27 ∴ Centre of the ellipse is = (– 1, 1)
− x1 63. (a) The nearest point from the ⇒ ex (x2 + y2) = C
     = (30 − 20) origin, on the line 3x – 4y = 25 65. (c) Let tan x be θ.
–1
− x1 − x2
will be the point where the ∴ tan θ = x
27 − x1 perpendicular from origin to the line. 1
25 = 20 + × 10 ∴ cos θ =
54 − x1 − x2 3x – 4y = 25, will intersect this line. 1 + x2
For example—
∴ x1 = x2
OP is the perpendicular from O to OP.
But 14 + x1 + 27 + x2 + 15 = 100
A x
2
x1 + x2 = 44  (x1 = x2)
1+ x
∴ x1 = x2 = 22
60. (a) 2s = a + b + c
= 2 3+2 3+2 3 =6 3 P θ
1
∴ s = 3 3 1
In equilateral triangle ∆ ABC, ∴ cos (tan–1) x =
1 + x2
A = B = C = 60°,
O
A B 66. (a) Let A = lim x→0 (cosec x) x
r1 = s tan
2 ∴ P is the nearest point from the origin log A= lim x→0 x log (cosec x)
60° to the given line.
= 3 3 tan log (cosec x)  ∞
2 3 = lim x→0  form 
 Slope of the line 3x – 4y = 25 is · 1  ∞
1 4
= 3 3 × =3 x
3 4
∴ ,, ,, ,, perp. to this line = − · 1
·( − cosec x cot x)
B 3
cosec x
Similarly, r2 = s tan and perpendicular is drawn from (0, 0) = lim x→0
2  1
∴ Equation of the perpendicular will  − 2 
= 3 3 tan 30° = 3, be x
Practice Set-8 101

[By L' Hospital Rule] factor. Also, the given two products have
x2  0
This is of the form ∫ x 2 − a 2 dx the middle number in common.
= lim x→0
tan x form 0  11 Let middle term be x
∴ I = · (2 x − 1) (2 x − 1) 2 − ( 2) 2
2x 22
= lim x→0  551 
−( 2) 2 log{(2 x − 1) + (2 x − 1) 2 − ( 2) 2 } So, 1st number =   = 19
sec 2 x  29 
[By L' Hospital Rule] 1 1
= (2 x − 1) 4 x 2 − 4 x − 1 − log  1073 
2 4 2 Third number =   = 37
= lim x→0 2 x cos x  29 
loge A = 0
{(2 x − 1) + 4 x2 − 4 x − 1 }
∴ Required sum = (19 + 29 + 37)
∴ A = e0 = 1 = 85
67. (d) Probability that problem is REASONING AND GENERAL 80. (c) 2x + y = 15 ...(i)
solved by student A = P(a) AWARENESS (RAGA) and 3x – y = 25 ...(ii)
∴ Probability that problem is not 71. (b) Primary Agricultural Credit Solving (i) and (ii), we get:
solved by student A = P(A). Similarly Societies provide crop loans to the                x = 8, y = –1
farmers. 81. (b) Let the first number = x1
for B and C are P(B), P(B) and P(C), and other two numbers = x2 and x3
72. (d)
P(C) 73. (a) Temperate deciduous forests According to question,
1 1 are most notable because they go through x1 + x2 + x3
Given P(A) = , P(B) = and P(C) four seasons. Leaves change colour in = 70
2 3 3
autumn, fall off in winter and grow back x1 + x2 + x3 = 70 × 3 ...(i)
1
= in the spring, this adoption allows plants
4 to survive cold winters. 1
and x1 = ( x2 + x3 ) ...(ii)
∴ Required probability 74. (d) 75. (b) 76. (b) 9
= Probability that problem is solved 77. (a) 2022 Explanation: The Minister ∴ From eqn (i)

by at least one student of Agriculture and Farmers Welfare,
= 1 – Probability the problem is not Narendra Singh Tomar announced that 1
( x2 + x3 ) + x2 + x3 = 70 × 3
solved by any student central Government has decided to 9
use digital technology in rural India
= 1–P(A)·P(B)·P(C) to double farmers’ income by the year
10
( x2 + x3 ) = 70 × 3
1 2 3 1 3 9
2022. For this, the Government has
= 1− · · = 1− 4 = 4
2 3 4 set up 713 Krishi Vigyan Kendras 70 × 3 × 9
x2 + x3 = = 189
π/ 2 (KVK) and 684 Agricultural Technology 10
68. (d) ∫0 sin x sin 2 x dx Management Agencies (ATMA) at
   From eqn (ii)
1 π/ 2 the district level in the country for
= 2 ∫0 (cos 3 x − cos x) dx dissemination of technologies among the 1
x1 =× 189 = 21
farming community. 9
π/ 2
1  − sin 3x  78. (a) Let the number be 10x + y Finally, the first number is 21.
= + sin x 
2  3 0 x=y+7 82. (c)
D G
1  1 3π   π  x–y=7
= 2  −  3 sin 2 − 0 +  sin 2 − sin 0  16 25
 
10x + y – 63 = 10y + x
11  2 E F H I
=  + 1 = 9 (x – y) = 63
23  3 2
x–y=7 Area of ∆DEF DE
69. (a) On adding the same number ∴ =
Area of ∆GHI GH 2
to each term, the standard deviation does We assume based on given options
not change. x = 8 and y = 1 (16) 2 256
= 2 =
70. (b) I = ∫ 4 x 2 − 4 x − 1 dx (25) 625
Hence, the number is 81.
79. (c) Since the numbers are co- Hence, the ratio of ∆DEF and DGHI
= ∫ (2 x − 1) 2 − ( 2) 2 dx prime, they contain only 1 as the common is 256 : 625.
102 Practice Set-8

83. (c) Cost price of 20 quintal rice 88. (a) Similarly, we eat food to satiate our
= ` 8580 start hunger.
A
78 1560
left rice = 20 × = 96. (d) Red Rose Flowers
100 100 N

       = 15.60 quintal 3m


24 m
W E Or
The trader wants 20% profit,
120 end B
C 7m
SP = × 8580 S
100 Red flowers
∴ AC = AB + BC
2 2 2
= 858 × 12 = ` 10296
Rate/quintal of rice = (24)2 + (7)2
Rose
= (576 + 49) = 625
10296 102960
= = ∴ AC = 25 m
15.60 156 According to statements neither
89. (d) According to question, the
= ` 660 per quintal conclusion I nor conclusion II follows.
84. (c) Given Ratio = 1 : 2 : 5 sequence is,
Sum of ratios = 1 + 2 + 5 = 8 Q > P > S > T > R Steam
Profit of Chiranjeev Finally, P is the second tallest of 97. (b) Smoke Vapours
5 × 187200 them all.
= = 5 × 23400
8 90. (a)
= ` 117000 According to statements only
13 21 48 conclusion II follows.
85. (d) Average speed of total journey +(2) 3
+(3) 3

112 237
2ab 2 × 34 × 66 +(4) 3
+(5)3 Cans
= =
a+b 34 + 66 Finally, the missing number is 237. 98. (b) Tins Jars
4488 91. (d) By analysing the circular
= = 44.88 km/hr
100 arrangement, it is clear that Required According to statements only
86. (c) P × R  P is the brother of R pair  DB i.e. 'D' is on the immediate conclusion II follows.
R – Q  R is the wife of Q. left of 'B' 99. (b)
So, P brother of R wife of Q. 92. (d) According to the given
Therefore, ‘P’ is the brother-in law information Photographs
of ‘Q’. Pictures
ICm⇒l≠m ...(i)
87. (a) As,
and I A m ⇒ l >| m  ...(ii) Paintings

From (i) and (ii), we get


l < m ⇒ I F m. ...(iii)
93. (b) Except 'Mustard', all others
are foodgrains. According to statement only
conclusion II follows.
94. (a) According to dictionary,
Similarly, arrangement of given words is: mist
(iii) Senate, (ii) Senior, Clouds fog
(i) Serious, (iv) Statesman. 100. (a)
Finally, iii, ii, i, iv is the correct
sequence. According to statements only
conclusion I follows.
95. (d) As, fan circulates air to keep
BGQMUC is the correct answer. us cool. qqq
Practice Set-9 103

Air Force Airmen Group X & Y Exam


Practice Set-9

being spent on social purpose. It was 4. The present constitution of our


ENGLISH Blackstone, not Lenin, who wrote, “The society is based on:
Directions (1-4): Read the passage law not only regards life and protects (a) the principle of equality
given below and answer the questions every man in enjoyment of it but also (b) the principle of human dignity
by selecting the most appropriate furnishes him with everything necessary (c) entrenched class privileges
passage option. for its support. For there is no man so (d) justice and fair play
If reason in the region of thought is indecent or wretched but that he may
the aim of science, the rule of equality 5. Spot the correct spelling:
demand a supply sufficient for all the
in the region of behaviour is the aim of (a) pussilanimous
necessities of life from the more opulent
democracy. Democracy is not a political part of the community.” (b) pusilannimous
arrangement or a form of government. It 1. What is the fundamental tenet (c) pusillanimous
is a pattern of life, an active conviction of democracy according to the (d) pusilanimous
which informs and inspires every author? 6. The correctly punctuated version
thought, word and deed. Our present (a) Right to choose representatives is:
constitution of society induces in its (b) Right to stand for public offices (a) He asked me, “whether I had
more fortunate members far too great written my exercise.”
(c) The presence of effective
readiness to accept privilege as though (b) He asked me, “whether I had
opposition
it were inherent in the social order, as written my exercise.”
(d) The operation of the principle
though it were normal and even proper (c) He asked me, “whether I had
of equality
and just. written my exercise.”
If we are sincere in our professions 2. What is democracy essentially
(d) He asked me whether I has
of democracy, we should not shut our according to the author?
written my exercise.
eyes to the most obvious defects of the (a) A form of government
7. T h e m e a n i n g o f t h e w o r d
present social order. A system which (b) A political arrangement
GORGON is:
does not offer security and decent (c) A pattern of life itself (a) an ugly woman
employment to multitudes of trained (d) Both (a) and (b) (b) a type of cheese
young men suffers from fundamental 3. “______ an active conviction
(c) a keyboard instrument
vice. Society is in danger of splitting to which informs and inspires every
(d) a stone figure
pieces if the few who have the benefits thought, word and deed.”
of civilization are not willing to share What does it mean? 8. Choose the word opposite in the
them with the rest. No state is stable (a) A democrat admires the virtues meaning to the word given below
unless it procures for all its members of democracy in whatever he underlined.
the essentials of a good life. We thinks, says and does The attack on the freedom of the
acknowledge that health is better than (b) A democrat has sound self press is a retrograde step.
disease, sufficiency better than poverty, confidence in regard to the (a) declining (b) progressive
shelter better than cold an exposure, correctness of what he thinks, (c) punitive (d) stubborn
ease of mind better than racking says and does 9. The underlined clause is:
anxiety. It is our duty to obtain these (c) A democrat always thinks Farming depends on water, which
essentials of civilized life for the mass aright, speak aright and acts will be available only during the
of the population, to work for basic aright monsoons.
economic justice for all, if necessary (d) A democrat believes in the (a) Adjective/Relative clause
by the imposition of higher taxes on concept of equality and this (b) Adverb clause
incomes, land value and inheritance. belief is reflected in his thinking, (c) Principal clause
Riches were created by the Maker for speech and action (d) Noun clause
104 Practice Set-9

Directions (10-11): Fill in the blanks (c) Cronology L


L
given below: (d) Chronalogy (a) (b) 2
10. Practically, very little work could 4
18. (a) Scintillating (b) Scintillatinge
be completed in the last week as (c) L (d) 2L
(c) Scintilating (d) Scintilliting
it was: 25. In the given figure, two points
Direction (19-20): In the following charges q1 and q2 are placed at
(a) a very busy week. question, four alternatives are given
(b) full of holidays. distances a and b from centre of
for the Idiom/Phrase mentioned below. a metallic sphere having charge
(c) a very hectic week. Choose the one which best expresses Q. The electric field due to the
(d) loaded with lost of work. the meaning of the Idiom/Phrase. metallic sphere at the point P is:
11. The chief guest will________the 19. A dark horse. q1
prizes. (a) an unforeseen competitor
(a) give over (b) give up (b) a black horse a
(c) give in (d) give away (c) a nightmare
Directions (12-13): In the following (d) an unknown person
questions, some of the sentences have 20. Bed of roses q2
errors and some have none. Find O P b
(a) Dull life
out which part of a sentence has an
(b) Very soft bed
error. The number of that part is
your answer. If there is no error, the (c) Full of joys
answer is (d). (d) Belong to
2 2
12. He is one of those writers (a)/who 1  q1   q2 
PHYSICS (a)  2  +  
has won acclaim (b)/ the world over. 4πε 0 a  b2 
(c)/No error (d). 21. C a l c u l a t e t h e d e - B r o g l i e
wavelength of an electron of (b) 1 Q
13. The mason will not (a)/do the work × 2
(b)/except give the order. (c)/No energy 400 eV: 4πε 0 R
error (d). ( h = 6.6 × 10 –34 Js , m = 9.1 × 30−31 kg) 2 2
1  q1   q2 
Direction (14): In the following (a) 0.61Å (b) 0.305 Å (c)  2  +  
question out of the four alternatives, 4π ε 0 a  b2 
(c) 1.61 Å (d) 0.92 Å
choose the one which best expresses (d) None of these
22. A T.V. tower has a height of
the meaning of the given word. 100 m. How much population 26. Water is flowing from a horizontal
14. PREMONITION is covered by the T.V. broadcast pipe fixed at a height of 2 metre
(a) insight (b) uncertainty if the average population from the ground. If it falls at:
(c) forewarning (d) scope density around the tower is
Directions (15-16): In the following 1.000 km–2 ? (Radius of the earth
questions, choose the word opposite = 6.37 × 106 m)
2m
in meaning to the given word. (a) 4 lakh (b) 4 billion
15. INCONGRUOUS (c) 40,000 (d) 40 lakh
(a) conflicting (b) contradicting 23. If the speed of sound in air be Cs 3m
(c) ill-matched (d) harmonious and r.m.s. speed be C then: A horizontal distance of 3 metre as
16. DEMENTED (a) Cs < C shown in figure. What is the speed
(a) sensible (b) sensitive (b) Cs = C of water when it leaves the pipe is
(Take g = 9.8 ms–2)?
(c) sensual (d) sensuous 1/ 2
γ (a) 47 ms–1 (b) 4.7 ms–1
Directions (17-18): In the following (c) C s = C  
 3 (c) 7.4 ms–1 (d) 14.7 ms–1
questions, a word has been written
in four different ways out of which (d) None of the above 27. A charged particle is released
only one is correctly spelt. Find the 24. Two inductors, each of inductance from rest in a region of steady and
correctly spelt word. L, are connected in parallel but uniform electric and magnetic
17. (a) Choronology are well separated from each fields which are parallel to each
(b) Chronology other. The effective inductance is: other. The particle will move in a:
Practice Set-9 105

(a) straight line (b) circle (a) 4 A (b) 2A (c) B o t h n u m e r i c a l l y a n d


(c) helix (d) cycloid (c) 6A (d) 10A dimensionally corret
28. The circuit shown here is used to 33. A passenger in a train drops a (d) N e i t h e r n u m e r i c a l l y n o r
compare the e.m.f.’s of the cells E1 ball from the window of the train dimensionally correct
and E2(E1 > E2). When the running at an acceleration a. A 38. Two concentric circular coils
pedestrian on the ground, by the of 10 turns each are situated in
side of the rails, observes the ball the same plane. Their radius
are 20 cm and 40 cm and they
falling along:
C carry currents of 0.2 A and 0.3 A
A B (a) a vertical with an acceleration
respectively in opposite directions.
E1 g 2 + a2 If µ 0 is the permeability of
vacuum then the magnetic field in
E2 G (b) a v e r t i c a l a c c e l e r a t i o n
Wbm–2 at the common centre is:
g 2 + a2 35µ0 5µ0
galvanometer is connected to E1, (a) (b)
(c) a parabola with an acceleration 4 4
the null point is at C. When the
galvanometer is connected to E2, g 2 + a2 µ0 7µ0
(c) 80 (d)
the null point will be: 80
(d) a parabola with an acceleration
(a) to the left of C 39. In the network, the current
g
(b) to the right of C flowing through the resistance 2
34. What is the maximum area upto
(c) at C itself which the transmission can be R as shown in the figure is:
(d) no where on AB received using antenna of above
29. In stationary wave: E
problem?
(a) strain is maximum at nodes (a) 6,036 m2 (b) 3,018 km2
(b) strain is maximum at antinodes (c) 1,509 km 2
(d) None of these 4R
R
(c) strain is minimum at nodes 35. One gram of ice, when melts
(d) amplitude is zero at all the required 336 Joule of heat. The 2R
points increase in internal energy will
E E
30. When a liquid is boiled to be: (a) (b)
(a) equal to 336 J R 7R
form vapours, the process
is: (b) more than 336 J 2E 2E
(c) (d)
(a) Adiabatic (b) Isothermal (c) less than 336 J 7R R
(c) Isochoric (d) Isobaric (d) equal to zero 40. In Kundt’s tube containing air
31. In a region, electric field varies as 36. Two bodies of mass M and m(M > when a rod clamped in the middle
E = 2x2 – 4, where x is distance m) respectively are dropped from is set into vibration, the distance
in SI from origin along x-axis, A the same height. If the resistive between the dust heaps is 3.46 cm
positives charge of 1µC is released force of air is same for the two and when the tube is filled with a
with minimum velocity from bodies, they reach the ground: gas this distance is 3.16 cm. The
(a) simultaneously ratio of velocities of sound in air
infinity for crossing the origin,
and in the gas:
then the kinetic energy at: (b) bigger body reaching first
(a) 1.095 : 1 (b) 2 : 1
(a) the origin must be zero (c) smaller body reaching first (c) 1 : 2.03 (d) 3 : 2
(b) the origin may be zero (d) question is misleading 41. A solid which is not transparent
(c) x = 2 m must be zero 37. With the usual notation the to visible light and whose
(d) x = 2 m may be zero following equation is: conductivity increases with
32. In a transformer, number of turns temperature is formed by:
1
in the primary are 140 and that in Sn = u + a (2n − 1) (a) ionic binding
the secondary are 280. If current 2 (b) covalent binding
in primary is 4 A, then that in the (a) Numerically correct only (c) Van der Waals binding
secondary is: (b) Dimensionally correct only (d) metallic binding
106 Practice Set-9

42. If 1 g cm2 . s–1 = x joule-s, then the 47. The number of solutions of the 54. The vertices of a ∆ ABC has
number x is equal to: 2 co-ordinates (cos θ, sin θ), (sin
equation z = z is:
(a) 1 × 10–7 (b) 1×107 (a) 2 (b) 3 θ, – cos θ), (1, 2). As θ varies the
(c) 1×10 –5
(d) 1×105 (c) 4 (d) None of these locus of centroid of the triangle is
43. In the ratio of pressure drops 48. Find the equation whose roots are the circle:
across the narrower tube to the 1 1
(a) x2 + y2 – 2x – 4y + 1 = 0
pressure drop across wider tube is: and (b) 3(x2 + y2) – 2x – 4y + 1 = 0
aα + b aβ + b
(a) 16 (b) 1 (c) x2 + y2 – 2x – 4y + 3 = 0
(a) cax2 – bx + 1 = 0
1 (d) None of these
(c) 2 (d) (b) cax2 + bx + 1 = 0
2 (c) cax2 + bx – 1 = 0 55. The value of log e {(1 + x) 1+x
44. A body in the floating in a liquid (d) None of these (1 – x)1–x} is:
contained in the beaker. The 2 4 6
system shown in figure falls freely
49. If 1, x, y, z, 16 are in geometric (a) x + x + x + ... ∞
progression, then what is the 2 4 6
with acceleration g. The force value of x + y + z?
of buoyancy on the body due to x 2 x 4 x6
(a) 8 (b) 12 (b) + + + .... ∞
liquid will be equal to: 1.2 3.4 5.6
(c) 14 (d) 16
(a) zero 50. H o w m a n y t e r m s a r e  x 2 x 4 x6 
(b) the weight of liquid displaced (c) 2  + + + ....∞ 
t h e re i n t h e e x p a n s i o n o f 1.2 3.4 5.6 
(c) the weight of body in air (1 + 2x + x2)10?
(d) the weight of body inside the (d) None of these
(a) 11 (b) 20
water (c) 21 (d) 30 56. What will be the equation
51. Find the equation of the parabola, of tangents of ellipse
whose latusrectum is 4 units, axis 4x2 + 3y2 = 6 which is parallel to
is the line 3x + 4y – 4 = 0 and the straight line y = 2x + 3?
tangent at the vertex is the line (a) y = 2 x ± 2 2
4x – 3y + 7 = 0.
(a) (3x + 4y – 4)2 = 20(4x – 3y + (b) y = x ± 2 2
45. Two convex lenses of focal lengths 7) (c) y = 2 x ± 3 2
f1 and f2 are mounted coaxially (b) (3x + 4y – 4)2 = 10(4x – 3y + 7)
separated by a distance. If the (c) (4x – 3y + 7)2 = 20(3x + 4y – (d) y = x ± 2
power of the combination is zero, 4) 57. For what value (s) of x is
the distance between the lenses is: (d) (4x – 3y + 7)2 = 10(3x + 4y – 4) log10 (999 − x 2 − 3 x + 3) = 3?
(a) |f1 – f2| (b) f1 + f2 (1 + x) + (1 − x)
f1 f 2 52. I f y = sin −1 , (a) 0 (b) 1
(c) f1 f 2 (d) 2
(c) 2 (d) (b) and (c)
|f –f | | f1 + f2 | dy
1 2 then is equal to: 58. The value of
dx
 3  5  7  2n + 1
MATHEMATICS  1 +  1+  1+  ... 1 +  is:
1 1 1  4  9   n2 
(a) (b) −
46. Consider the function f: R → {0, (1 − x 2 ) (1 − x 2 ) (a) (n + 1) (b) (n + 1)2
1} such that
f (x) = 1, {
if x is rational
0, if x is irrational
(c) −
1
2 (1 − x ) 2
(d) None of these
(c) 2(n + 1)2 (d) None of these
59. Solution of differential equation
dy
Which one of the following is dx equal to? = sin (x + y) + cos (x + y) is equal
53. What is ∫ dx
correct? x log x
to:
(a) The function is one-one into (a) log (log x) + C
 x + y
(b) The function is many-one into (b) log x + C (a) log  2 + sec  = x+C
2 
(c) The function is one-one onto (c) (log x)2 + C
(d) The function is many-one onto (d) None of these (b) log [1 + tan( x + y )] = y + C
Practice Set-9 107

 x + y 66. W h a t is the value of


(c) log 1 + tan  = y+C 4 1 REASONING AND GENERAL
 2  sin −1
+ 2 tan −1 ?
5 3 AWARENESS (RAGA)
 x + y 71. Imperial Bank was constituted
(d) log 1 + tan 2  = x + C (a)
π
(b)
π
in the year:
3 2
60. What is the least value of the (a) 1955 (b) 1921
π
standard deviation of 5 integers, (c) π (d) (c) 1930 (d) 1935
no two of which are equal? 4 6 72. Radioactive materials should be
67. The equation of the ellipse where kept in the container made of:
(a) 5
focus is (2, 0) and the directrix is (a) Pb (b) Steel
(b) 2
x + y + 1 = 0 and eccentricity is (c) Fe (d) Al
(c) 2 1 73. The Indian Judiciary is headed
equal to , is:
(d) Can’t be determined 2 by:
61. Divide 20 into two parts such that (a) 3x2 + 3y2 + 2xy + 18x + 2y + 15 (a) The Supreme Court
the product of one part and the =0 (b) The Parliament
cube of the other is maximum. (b) 3x2 + 3y2 – 2xy + 18x + 2y + 15 (c) The President
The two parts are: =0 (d) The Prime Minister
(a) (10, 10) (b) (12, 8) (c) 3x2 + 3y2 – 2xy – 18x – 2y + 15 74. Who has been called the
=0 Napoleon of India?
(c) (15, 5) (d) None of these
(d) None of the above (a) Samudragupta
62. Two cards are drawn at random
1 (b) Skandagupta
from a pack of 52 cards. The 68. If cosec θ – cot θ = , where θ ≠
3 (c) Chandragupta
probability of these two being aces
0, then what is the value of cos θ? (d) Vikramaditya
is:
75. Density of population shows:
1 1 (a) 0 (b) 3
(a) (b) 2 (a) Land labour ratio
26 221
(b) Man land ratio
1 1 1
(c) (d) None of these (c) (d) (c) Land capital ratio
2 2 2 (d) Land product ratio
63. Given 5 flags of different colours, 69. What is the value of sin (1920°)? 76. Fifth generation computers do
how many different signals can be 1 1 not have:
(a) (b)
generated, if each signal requires 2 2 (a) Speech recognition
the use of 2 flags, one below the (b) Artificial intelligence
3 1
other: (c) (d) (c) Very large scale integration
2 3
(a) 18 (b) 20 (d) Vacuum tubes
70. If A is a (3 × 3) matrix such that | A
(c) 19 (d) 23 | = 4, then what is A (adj A) equal 77. Which country gifted Imphal
π /4
tan n x dx, then what is to? Peace Museum to mark the
64. If In = ∫0
1 0 0 75th anniversary of the Battle
In + In–2 equal to? (a)   of Imphal at Maibam Lokpa
 0 1 0 Ching, Manipur?
1 1 0 0 1 
(a) (b) (n − 1) (a) Bangladesh (b) Vietnam
n
4 0 0 (c) China (d) Japan
1 (b)  0
(c)
n
(d) (n − 2)  4 0 78. What is the value of (81 + 82 + 83
(n − 1)  0 0 4 + ________ +130)?
65. If in a ∆ ABC, a + c = 2b, then the (a) 5275 (b) 10550
A C 16 0 0
value of cot is equal to:  (c) 15825 (d) 21100
0 
· cot
2 2 (c)  0 16
79. The sum of two number is 528
(a) 4.6 (b) 3  0 0 16
and their H.C.F. is 33. The
(c) 1.5 (d) 1 (d) Cannot be determined number of pairs of numbers
108 Practice Set-9

satisfying the above conditions (i) ‘A$B’ means ‘A is mother of B’ the right of F. A is third to the left of
is: (ii) ‘A#B’ means ‘A is the father of B’ C, who is not an immediate neighbour
(a) 4 (b) 6 (iii) ‘A@B’ means ‘A is the husband of of F. G is second to the left of A. D is
(c) 8 (d) 12 B’ second to the right of E.
4 x 21 (iv) ‘A%B’ means ‘A is daughter of B’ 91. Which of the following pairs
80. If 4x + 5y = 83 and = , 86. P@Q$M#T indicates what represents the immediate
3 y 33
then x + y = ? relationship of P with T? neighbours of E?
(a) 4 (b) 8 (a) Maternal Grandfather (a) DH
(c) 7 (d) 18 (b) Paternal Grandfather (b) HC
(c) Maternal Grandmother (c) CA
81. The average revenues of 11
(d) Cannot be determined (d) Data inadequate
consecutive years of a company is
87. If the word ‘EARTH’ can be Direction (92): In following questions
` 77 lakhs. If the average of first written as ‘QPMZS’ in coded
6 years is ` 72 lakhs and that of you have to identify the responses
words, how can ‘HEART’ be which would be a correct inference
last 6 years is ` 84 lakhs. What written following the same from the given premises stated
is the revenue for the sixth year. coding? according to the following systems:
(a) ` 91 lakhs (b) ` 87 lakhs (a) SQMPZ (b) SQPZM 'A' stands for Not greater than.
(c) ` 85 lakhs (d) ` 89 lakhs (c) SQPMZ (d) SPQZM 'B' stands for Greater than
82. Find the sum of the measures 88. Amit is facing North. He turns 'C' stands for Not equal to
of all the interior angles of a 90º right and walks for 50 m, 'D' stands for Equal to
polygon having 10 sides. then turns towards the South and 'E' stands for not less than
(a) 2520° (b) 2880° walks 50 m, then turns right to 'F' stands for less than.
(c) 1440° (d) 3240° walk for another 50 m and from 92. Premises: (3x A 2y) and (3x Bz)
there, he walks 50 m towards
83. 30% discount is offered on (a) P D 2r (b) P F 2r
North. In which direction/
an item. By applying a promo (c) 2y C z (d) 2y E z
position is he from his original
code the customer wins 20% position? Direction (93): Select the one which
cash back. What is the effective is different from the other three
(a) South-west
discount? responses.
(b) At the original position
(a) 44 percent 93. From the given alternatives,
(c) North-east
select the word which cannot be
(b) 52.8 percent (d) North-west formed using the letters of the
(c) 50 percent 89. C has more money than E and given word.
(d) 26 percent E has more money than B. C is
CELEBRATION
84. The third proportional of two the second richest after A. Who
(a) TAILOR (b) ACTION
numbers 7 and 21 is : has the least money out of A, B,
C and E? (c) CREATE (d) BREATH
(a) 35 (b) 63 Direction (94): Arrange the given
(a) C (b) A
(c) 42 (d) 21 words in the sequence in which they
(c) B (d) E
85. 9 hrs after a goods train passed a occur in the dictionary.
Direction (90): A series is given with
station, another train travelling 94.
one term missing. Choose the correct
at a speed of 72 km/hr following alternative from the given ones that (i) Agent (ii) Argentine
that goods train passed through will complete the series. (iii) Assiduous (iv) Accentuate
that station. If after passing the 90. Infant, Childhood, Adolescence? (a) iii, ii, i, iv (b) iv, iii, i, ii
station the train overtakes the (a) Bachelor (b) Adult (c) iv, ii, i, iii (d) iv, i, ii, iii
goods train in 3 hours, what is (c) Widow (d) Old 95. Q types faster than R but not as
the speed of the goods train? Direction (91): Study the following fast as V. T types faster than R
(a) 21.6 km/hr (b) 27 km/hr information carefully and answer the but slower than V. S types faster
(c) 18 km/hr (d) 14.4 km/hr questions given below: than V. Who amongst the five of
Direction (86): Study the following A, B, C, D, E, F, G and H are sitting them types the fastest?
information carefully to answer these around a circle facing the centre. H is (a) V
questions. fourth to the left of B and second to (b) T
Practice Set-9 109

(c) S (c) Mukta 99. Which of the following groups


(d) Data Inadequate (d) Cannot be determined has the first person sitting
Direction (96-100): Study the 97. Who is second to the left of between the other two?
following information carefully and Rani? (a) Meeta-Ashwini-Geeta
answer the given questions. (a) Ashwini (b) Rani-Sudha-Geeta
Ashwini, Priya, Sudha, Rani, Meeta, (b) Geeta (c) Mukta-Priya-Rani
Geeta and Mukta are sitting around a (c) Priya (d) Mukta-Priya-Sudha
circle facing the centre. Ashwini is third (d) Sudha 100. Which of the following is the
to the left of Mukta and to the immediate 98. Which of the following pairs correct position of Rani with
right of Rani. Priya is second to the of persons has the first person respect to Mukta?
left of Geeta, who is not an immediate sitting to the immediate left of I. Third to the right
neighbour of Meeta. the second person? II. Third to the left
96. Who is to the immediate right of (a) Rani-Meeta III. Fourth to the left
Priya? (b) Ashwini-Geeta IV. Fourth to the right.
(a) Meeta (c) Sudha-Priya (a) I only (b) II only
(b) Sudha (d) Geeta-Sudha (c) Both I & III (d) Both II & IV

Answers with Explanation


ENGLISH after ‘of’ should be considered as PHYSICS
1. (d) 2. (c) 3. (d) 4. (a) antecedent and verb is used accordingly 21. (a)
5. (c) Pusillanimous is the correct For example. h
λ =
pelling which means lacking courage She is one of those who does not 2mE
or resolution, cowardly, faint-hearted, accept this view wrong. 6.6 × 10−34
= m
m
timid. She is one of those who do not accept 2 × 9.1 × 10−31 × 400 × 1.6 × 10−19
6. (c) He asked me, “whether I had this view correct. = 0.61 × 10−10 m
written my exercise” is the correctly = 0.61 Å
Hence, ‘who have won acclaim’ is
punctuated sentence.
correct usage. 22. (d) d = 2hR
7. (a) Gorgon means (a) an ugly
woman. 13. (c) ‘Rather’ should be used in Popuplation covered = πd 2 ×
8. (b) Opposite of retrograde would stead of except. population density
be progressive. Hence option (b). = π × 2hR × population density
14. (c) The word ‘premonition’
9. (a) Underlined clause in the above (Noun) means a feeling that something 3.14 × 2 × 100 × 6.37 × 106 × 1000
=
sentence will be adjective/relative clause. 1000 × 1000
is going to happen unpleasanl. Hence,
An adjective clause is a group of words = 4 × 106
premonition and forewarning are
which contains a subject and a predicate = 40 lakh
synonymous.
of its own, and the work of an adjective.
15. (d) The word ‘incongruous’ γP
10. (b) Practically, very little work 23. (c)
(Adjective) means strange because Cs =
could be completed in the last week as it d
was (b) full of holidays. not in harmony with the surroundings; 1
out of place. Hence, incongruous and ∴ P = d × C2
11. (d) The phrasal verb ‘give away’ 3
would only be used in the sentence. The harmonious are antonymous.
γC 2
chief guest will give away the prizes. 16. (a) The word ‘demented’ Cs =
3
Hence option (d). (Adjective) means mad. Hence, sensible
γ
12. (b) When ‘one of’ is used in the and demented are antonymous. Cs = C
3
sentence, the Noun/Pronoun coming 17. (b) 18. (c) 19. (a) 20. (c)
110 Practice Set-9

L1L 2 L 29. (b) B e c a u s e a m p l i t u d e i s Mg − F F


24. (b) L p = = maximum. = =g−
L1 + L 2 2 M M
30. (b) During boiling temperature
25. (a) The net electric field at P is Acceleration a2 in smaller mass
remains constant and thus process is m
zero:
→ → → →
isothermal. mg − F
E pˆ = E1 + E 2 + E sphere 31. (c) For minimum velocity, the =
\ m
→ → → velocity of point charge will be zero at
⇒ 0 = E1 + E 2 + E sphere F
the point where electric field is zero. =g−
m
→ → → Let at x = x0, electric field is zero.
\   E sphere = – (E1 + E 2 ) Since, M > m, therefore, a1 > a2, thus
 E = 2x2 – 4
→ → → 2
bigger mass will reach first.
\ E sphere = E + E
2 2 ⇒ 0 = 2 x0 − 4 37. (b) Sn = displacement in nth sec.
1 2

2 2 x0 = 2m LT −2
q  q  [LT −1 ] = LT −1 + (2T − 1)
=  12  +  2  Hence, (c) is correct. 2
a   b2  i p ns = LT −1 + LT −1
32. (b) =
1  q1 
2
 q2 
2
is n p
= Hence, dimensionally correct.
4πε 0  a2  +  b  4 280
   2 ⇒ = 38. (b)
is 140 µ NI µ NI 2
26. (b) Since , BC = 0 1 − 0
so, is = 2A 2r1 2r2
8x2
y= 33. (c) Resultant acceleration equals  
2u 2  0.2
µ N 0.3 
(9.8) (9) g + a and the particle will follow a
2 2
⇒ BC = 0  − 
⇒ 2 = 2   20   40  
2u 2 parabolic path.  
  100   100  
⇒ u 2 = 21.99 34. (b) Area covered:
A
⇒ u = 4.7 ms −1 0.3
A = πd 2 = π × 2rh
27. (a)Since the charged particle is at = 3.14 × 2 × 6.4 × 106 × 75 2A 20
0.
rest, so initially, the magnetic field will 2
not make it move. But initially: = 3018 km C
35. (b) When ice melts 336 joule 40
B
is supplied to it. In addition to it, the
E
volume of water formed is less than that
 3
B of ice i.e., there is decrease in the volume ⇒ BC = 5µ0 1 − 
 4
of the system hence some energy is
E 5µ0
supplied due to mechanical work done ⇒ BC =
B in decreasing (compressing) the volume. 4

the charged particle will experience Thus, increase in internal energy will be 39. (c) Taking the total current as
6I, current through 4R is 2I and current
an electrostatic force in the direction more than 336 joule.
through 2R is 4I. Applying Kirchhoff’s
of electric field as a result of which it Alternately ∆U = Q – W, here W second law, we get current through 2R is
gains a velocity parallel to E or parallel is negative since work is done on the 2E/7R.
to B and hence even after the motion system, therefore, increase in internal v l 3.46
40. (a) a = a =
of the particle it will not experience a vg lg 3.16
energy ∆U = 336 – (–W) = 336 + W
magnetic force i.e., its trajectory is a 1.095
i.e., more than 336 J. =
straight line. 1
36. (b) Let F be the resistive force
28. (a)Since E ∝ I. So, for E1 > E2 we 41. (b) If lattice constant of
acting on each mass
have I1 > I2 and hence null point will be semiconductor is decreased, then Ec and
obtained at shorter length i.e., to left of C. Acceleration a1 in bigger mass m Ev decrease but Eg increases.
Practice Set-9 111

42. (a) 1 1 3 2
 4x − 3y + 7 
3x + 4 y − 4 
1 g cm 2 / s = x Js − y2 + = 0 ⇒ y = ± ⇒ = 4 2 2
4 2 2  2
3 +4  2   4 + ( −3) 
1g = 10 –3 kg ∴ Hence, there are four solutions
z1 = 0 + i0 = 0 ⇒ (3x + 4y – 4)2 = 20(4x – 3y + 7)
and 1cm 2 = 10 –4 m 2
z2 = 1 + i0 = 1 This is the equation of the required
m2 parabola.
Hence, 10 –7 kg m 2 /s = x kg s 1 3
s2 z3 = − + i =ω 52. (c) Put x = cos θ.
2 2
x = 10 –7  θ
1 3 1 + cos θ = 2 cos2  
43. (a) Let p1 and p2 be the pressure and z4 = − − i = ω2 2
2 2
drop across the two tubes. In series 2  θ
b c 1 – cos θ = 2sin  
connection, Q is same for both the tubes, 48. (a) α + β = − , αβ = 2
a a
therefore,
The required equation is −1   1   cos θ sin θ  
πp a 4 ∴ y = sin    + 
Q = 1  1 1  1 1  2  2 2  
8 ηl x2 −  +  x+ ·
 aα + b aβ + b  aα + b aβ + b  θ π θ π
πp2 (a / 2) 4 −1
= =0 = sin sin  2 + 4  = +
8 ηl 2 4
⇒ {a2αβ + ab(α + β) + b2} x2 1 π
⇒ 16 p1 = p2
– {a(α + β) + 2b} x + 1 = 0 y = cos −1 x +
p2 (narrower) 2 4
⇒ = 16 ⇒ (ca – b2 + b2) x2 – (2b –b) x + 1 = 0
p1 (wider) dy − 1
⇒ cax2 – bx + 1 = 0 =
44. (a) A freely falling system is in 49. (c)  1, x, y, z, 16 are in geometric dx 2 (1 − x 2 )
state of weightlessness. progression. dx
45. (b) Pcomb = P1 + P2 − xP1P2 Here, a = 1, I = 16, n = 5, 53. (a) Let I = ∫ x log x
I = arn–1
1 1 x Let
0= + − 16 = 1·r4
f1 f 2 f1 f 2  I = log x
⇒ r =2 1
⇒ x = f1 + f 2 
∴ x = 1·r = 2  1 ⇒ I = ∫ ·dt = log | t | +C
dt = x dx t
MATHEMATICS y = 1·r2 = 4
z = 1·r2 = 8
46. (c) Since, on taking a straight line ⇒ I = log (log x) + C
∴ x + y + z = 2 + 4 + 8 = 14
parallel to x-axis, the group of given 50. (c) Given, 54. (b) If (x, y) be the centroid,
function intersect it at one point. (1 + 2x + x2)10 = {(1 + x)2}10 = (1 + x)20 then 3x = 1 + cos θ + sin θ, 3y = 2 – cos
∴ Total terms = 20 + 1 = 21 θ + sin θ
∴ f(x) is many-one and as range of
51. (a) Let P(x, y) be any point on the We have to eliminate the variable θ
f(x) = Codomain parabola and PM, PN be perpendiculars ∴(3x – 1)2 + (3y – 2)2 = (cos θ + sin
∴ f(x) is onto from P on the axis and tangent at the θ) + (– cos θ + sin θ)2
2

Hence, f(x) is one-one onto. vertex, respectively.


⇒ 9(x2 + y2) – 6x – 12y + 5 = l + 1
47. (c) z2 = z ⇒ (x + iy)2 = x – iy Then, PM2 = (latusrectum ) (PN)
⇒ 3(x2 + y2) – 2x – 4y + 1 = 0
4x – 3y + 7 = 0

x2 – y2 – x + i(2xy + y) = 0 55. (c) log {(1 + x)1+x (1 – x)1–x}


N
⇒x – y – x = 0 and 2xy + y = 0
2 2 P(x, y) = (1 + x) log (1 + x) + (1 – x) log (1 – x)
Now, 2xy + y = 0 gives y = 0 ⇒ x =  x 2 x3 x 4 
1 = (1 + x)  x − + − + ...
−  2 3 4 
2 Vertex M 3x + 4y – 4 = 0
 x 2 x3 x 4 
When, y = 0, x2 – y2 – x = 0 + (1 − x)  − x − − − − ...
gives x2 – x = 0 ⇒ x = 0, 1  2 3 4 
1 2 2  x 2 x 4 x5   x 4 x6 
When, x = − , x − y − x = 0 gives = 2 − − − − ... + 2  x 2 + + + ...
2  2 4 6   3 5 
112 Practice Set-9

 1 1 1 1 1   2 k + 2 + 1 ∴ 4y2(15 – y) = 0
= 2  x 2 1 −  + x 4  −  + x6  −  + ... 1 + 
  2  3 4  5 6  (k + 1) 2  ∴ y = 0, 15

 2 4 6 
= 2  x + x + x + ....
2
 2k + 3  d z
1·2 3·4 5·6  = (k + 1)2 1 +  Now, = 120y – 12y2
( k + 1) 2  dy 2

56. (a) Equation of any line parallel to = 12y (10 – y)
line y = 2x + 3 will be y = 2x + c, putting [using Eq. (i)]
y = 2x + c in equation of ellips 4x2 + 3(2x  (k + 1) 2 + 2k + 3  d2z
= (k + 1)2   At y = 15,
+ c)2 = 6  (k + 1) 2  dy 2
⇒ 16x2 + 12 cx + (3c2 – 6) = 0 = k2 + 2k + 1 + 2k + 3 = (k + 22) = [(k = 12 × 15 (10 – 15) < 0
If line y = 2x + c, is tangent of ellipse + 1) + 1]2 i.e., maximum when y = 15
4x 2 + 3y 2 = 6, then roots of above [ (a + b)2 = a2 + 2ab + b2] 4
quadiratic equation will be coincident, C2
Therefore, P(k + 1) is true when P(k) 62. (b) Required probability = 52
so two values of n will be equal. C2
∴ from condition b2 = 4ac is true. Hence, from the principle of 1
4×3
(12 c)2 = 4 × 16 × (3c2 – 6) mathematical induction, the statement = =
52 × 51 221
⇒ 144c2 = 142a2 – 384 is true for all natural numbers n. 63. (b) Number of ways choose the
dv dy first flag = 5
⇒ c = ±2 2 59. (d) Put x + y = v ⇒ −1 =
dx dx Number of ways to choose the second
∴Equation of tangents are flag from the rest of four flags = 4
dv
y = 2x ± 2 2 ∴ = 1 + sin v + cos v Hence by FPC total number of ways
dx = 5 × 4 = 20
2
57. (d) log10 (999 + x − 3 x + 3) = 3 dv π/4
⇒ v v v = dx
64. (b) In = ∫0 tan n x dx
⇒ 2
999 + x − 3 x + 3 = 1000 2 cos 2 + 2sin ·cos π/4
2 2 2
⇒ x − 3x + 3 = 1
2
1 2v
= ∫0 tan n − 2 x tan 2 x dx
sec π/4
⇒ x2 – 3x + 3 = 1 ⇒
2 2 dv
v
= dx
= ∫0 tan n − 2 x (sec2 x − 1)dx
⇒ x2 – 3x + 2 = 0 1 + tan π /4 π /4

⇒ x – 2x – x + 2 = 0
2
2 = ∫0 tan n − 2 x sec 2 x dx − ∫
0
tan n –2 x dx

⇒  x + y = x + C Put t = tan x ⇒ dt = sec2 x dx


log 1 + tan 
⇒ x(x – 2) – 1(x – 2) = 0  2  1 n−2
⇒ (x – 1) (x – 2) = 0 ⇒ x = 1, 2 60. (c) Let us consider any five
∴ In = ∫0 t dt − I n − 2

58. (b) Let the given statement be integers are 3, 4, 5, 6 and 7.  t n −1 


1

P(n). 25 =   − I n −2
Its mean = 5 = 5  n − 10
 3  5  7  2n + 1
i.e., P(n) = 1 +  1 +  1 +  .... 1 + 2  1
1 4 9 n
∴ SD = n − 1 − I n –2
= (n + 1) 2
(5 − 3) 2 + (5 − 4) 2 + (5 − 5) 2 + (5 − 6) 2 + (5 − 7) 2
For n = 1, i.e., P(1) = (1 + 1)2 = 22 = = 5 1
⇒ In + In+2 =
 3 n −1
4 = 1 +  , which is ture. 4 +1+ 0 +1+ 4
1 = = 2 A C
5 65. (b) cot ·cot
Let it is true for n = k. 2 2
61. (c) Let two numbers be x and y.
 3  5  7  2k + 1 s( s − a) s ( s − c)
i.e., 1 +  1 +  1 +  .... 1 + 2  =
1 4 9  k  Then, x + y = 20 and z = xy 3 is
( s − b)( s − c) ( s − a )( s − b)
= (k + 1)2 ...(i) maximum.
a+b+c
For n = k + 1, ∴ z = y3(20 – y) = 20y3 – y4 s
2
 3   5   7   2k + 1  dz = s −b = + +
⇒ = 60y2 – 4y3 = 0 (say) a b c
1 +  1 +  1 +  .... 1 + 2   −b
 1 4 9  k  dy 2
Practice Set-9 113

a+b+c θ Manipur Revenue minister Karam


= 2sin 2 Shyam, Japanese Ambassador to India
a+c−b 2 1
⇒ θ θ = Kenji Hiramatsu, High Commissioner
(\ a + c = 2b) 2sin ·cos 3 of United Kingdom Dominic Asquith
2 2
A C 2b + b 3b and TNF Chairman Yohei Sasakawa
∴ cot ·cot = = =3 θ among others.
2 2 2b − b b ⇒ tan = tan 30°
2 78. (a) (81 + 82 + 83 + .........+ 130)
66. (b) sin −1  4  + 2 tan −1  1  θ = 30°
 5  3 ⇒ = (1 + 2 + 3 + 4 + .........+ 130)
2 – (1 + 2 + 3 + 80)
 4/5   2/3 
= tan −1  + tan −1  ⇒ θ = 60°
 1 − 16 / 25   1 − 1/ 9  n1 (n1 + 1) n2 (n2 + 1)

1 =
∴ cos 60° = 2 2
−1  4 / 5  −1  2 / 3  2
= tan   + tan       130(130 + 1) – 80(80 + 1)
3/ 5 8 / 9 69. (c) sin (1920°) = sin (360° × 5° +
2 2
4 3 120°)
= tan −1   + tan −1   = 65 × 131 – 40 × 81
 3  4 [ sin (360° + θ) = sin θ]
= 8515 – 3240
π = sin 120° = sin (90° + 30°)
−1  4  −1  4  = 5275
= tan   + cot   = [ sin (90° + θ) = cos θ]
3 3 2 79. (a) Let the required numbers be
67. (c) Let S(2, 0) be the focus. 3 33a and 33b.
= cos 30° =
Consider P(x, y) be any point on the 2 Then,
ellipse and PM be perpendicular from P 70. (b) We know that, 33a + 33b = 528 ⇒ a + b = 16. Now
on the directrix. coprimes with sum 16 are (1, 15) (3, 13),
1 0 0 (5, 11) and (7, 9)
∴ SP = ePM 
A (adj A) = | A | In = 4 0 1 0 ∴ Required numbers are (33 × 1,
(by definition)
0 0 1  33 × 15), (33 × 3, 33 × 13), (33 × 5, 33
⇒ SP = e PM2
2 2
× 11), (33 × 7, 33 × 9).
⇒ (x – 2) + (y – 0)2
2 4 0 0 The number of such pairs is 4.
2 2 =  0 4 0 4 x 21
 1   x + y + 1  80. (d) =
= 
 2   1 + 1   0 0 4 3 y 33

⇒ 4[(x – 2)2 + y2] = (x + y + 1)2 x


REASONING AND GENERAL ⇒

⇒ 4x2 – 16x + 16 + 4y2 = x2 + y2 + 1 y
AWARENESS (RAGA)
+ 2xy + 2x + 2y  21 2  7 7
= × = ⇒ x= y
⇒ 3x + 3y – 2xy – 18x – 2y + 15 = 0
2 2 71. (b) Imperial Bank was founded  22 3  11 11
on January 27, 1921. It was nationalised = 4 x + 5 y = 83
This is the required equation of an
and converted into State Bank of India
ellipse. 7
(SBI) in 1955. SBI started functioning ⇒
y + 5 y = 83

1 on July 1, 1955. 11
68. (c) Given, cosec θ – cot θ = ,
3 72. (a) 73. (a) 74. (a) 28
y + 5 y = 83
where θ ≠ 0 75. (b) Density means persons living 11
in km2 area. Hence, it is a man land ratio. ⇒ 83 y = 83 × 11 ⇒ y = 11

1
⇒ 1 − cos θ = 76. (d) 7 7 
sin θ sin θ 3 77. (d) Japan Explanation: On \
x=y =  × 11 = 7
11  11 
June 22, 2019, Japan gifted Imphal So x + y = 11 + 7 = 18
(1 − cos θ) 1
⇒ = Peace Museum to Manipur to mark
3 81. (d) Total revenues of 11 years
sin θ the 75th anniversary of the Battle of
Imphal, one of the fiercest battles of the = 77 × 11 lakhs
  Second World War, at Maibam Lokpa = 847 lakhs
2 θ  1
⇒ 1 − 1 − 2sin 2   = Ching popularly known as Red Hill Total revenues of 6 years
3 = 72 × 6 lakhs
 sin θ  (about 20 km southwest of Imphal).
The inauguration was attended by = 432 lakhs
114 Practice Set-9

Total revenues of next 6 years 86. (b) P @ Q  P is the husband of Q. From (i) and (ii), we get
= 84 × 6 years Q $ M  Q is the mother of M. 2y > z
= 504 lakhs M # T  M is the father of T. or 2y <| z ⇒ 2y E z.
   Average of 6th years 93. (d) 'BREATH' word cannot be
= (432 + 504) – 847 formed by using the given main word
= 936 – 847 'CELEBRATION' because 'H' letter is
= ` 89 lakhs not present in the main word.
82. (c) Sum of interior angles 94. (d) According to dictionary, the
= (2n – 4) × 90º So, P  father of T's father. arrangement of the words is:
= (2 × 10 – 4) × 90° Therefore, ‘P’ is the paternal (iv) Accentuate, (i) Agent,
= (20 – 4) × 90° grandfather of ‘T’. (ii) Argentine, (iii) Assiduous
= 16 × 90° 87. (c) As, Finally, (iv), (i), (ii), (iii) is the
= 1440° correct sequence.
E A R T H
83. (a) Let M.P. be ` 100
S.P. of an article
B B B B B 95.(c) According to question
Q P M Z S V > Q > R ...(i)
100 – 30 T > R and S > V ...(ii)
= × 100 = ` 70 Here code of H ⇒ S, T ⇒ Z,
100 From Equation (i) and (ii)
R ⇒ M, A ⇒ P and E ⇒ Q
Now again 20% discount is given Similarly, Now, S > V > Q, T > R
Then H E A R T
96. (c) By analysing the above circular
  S.P. =
100 – 20
× 70
B B B B B
arrangement, it is clear that
100 S Q P M Z
Mukta ⇒ to the immediate right
80 × 70 88. (b)
= = 56 Starting point of Priya.
100 50 m
N
Hence, effective discount 97. (b) By analysing the above circular
= (100 – 56)% 50 m 50 m W E arrangement, it is clear that
= 44 percent
S Geeta ⇒ second to the left of Rani.
84. (b) Let the third proportional 50 m
number be x. According to question, Amit travels in all directions and 98. (d) By analysing the above circular
7 : 21 : : 21 : x finally he reached at the original position. arrangement, it is clear that
7x = 21 × 21 89. (c) According to question, Sitting
Geeta to the immediate
21 × 21 C>A>E>B left of Sudha
x = = 63
7 Finally, B has the least money out of (First person) (Second person)
Finally, third proportional A, B, C and E.
number is 63. 99. (b) By analysing the above circular
90. (b)Infant is related to childhood
85. (c) Let speed of goods train be arrangement, it is clear that
and the next level is adolescence. Sitting
v km/h Sudha between Rani and
Distance covered in 9 hr = 9 × v = Similarly, the next level is Adult. Finally,
9v km Adult is the word which complete the Geeta.
Relative speed = (72 – v) km/h of series.
(First person)
another train
91. (b) By analysing the circular 100. (c) By analysing the above circular
Distance covered by another train =
3 (72 – v) km arrangement, we get Immediate arrangement, it is clear that
According to question neighbours of 'E' ⇒ H & C . Rani ⇒ third to the right to Mukta
9 v = 3 (72 – v) 92. (d) According to the given and fourth to the left of Mukta.
3v = 72 – v information
4v = 72 Therefore, both (I) & (III) are
3x A 2y ⇒ 3x >| 2y ...(i)
72 and 3x Bz ⇒ 3x > z  ...(ii) correct.
         v= = 18 km/hr.
4 qqq
Practice Set-10 115

Air Force Airmen Group X & Y Exam


Practice Set-10

you wish it, I will serve you as your most 6. (a) elixir (b) elesser
ENGLISH
faithful servant and will order my sons to (c) eleseir (d) elixer
Directions (1-4): Read the passage do the same. Forgive me!” 7. What is the Indirect form of the
given below and answer the questions The king was very glad to have following sentence?
that follow. made peace with his enemy so easily, My teacher often says to me that
The king turned round and saw a bearded and to have gained him for a friend. if I don’t work hard I shall fail.
man running towards them. His hands 1. The bearded man was (a) My teacher often says to me, “If
were pressed against his stomach, from (a) a hermit you don’t work hard, you will
which blood was flowing. When he (b) the king's enemy fail.”
reached the king he fainted and fell to the (c) the king's bodyguard (b) My teacher tells me, “If I don’t
ground. The king and the hermit removed (d) the king's servant work hard, I will fail.”
the man's clothing and found a large 2. The king washed and dressed the (c) My teacher often says, “If you
wound in his stomach. The king washed would because he wanted to don’t work hard, you might fail.”
and covered it with his handkerchief but (a) make peace with the bearded man (d) My teacher advises me, “work
the blood would not stop flowing. The (b) help the hermit hard otherwise I will fail.”
king re-dressed the wound until at last (c) help a wounded man 8. _______better solution can still be
the bleeding stopped. (d) have the bearded man in his worked out.
The man felt better and asked for service (a) More (b) The
something to drink. The king brought 3. The bearded man asked for the (c) A (d) Such
fresh water and gave it to him. Then, the king's forgiveness because Directions (9-10): In the following
king with the hermit's help, carried the (a) he slept on the bed while the king questions, four alternatives are given
wounded man into the hut and laid him slept on the floor for the idiom/phrase underlined in the
on the bed. The king tired by his walk and (b) the king's bodyguard wounded sentence. Choose the alternative which
the work he had done, lay down on the him best expresses the meaning of the idiom/
floor and slept through the night. When (c) he wounded the king's bodyguard phrase.
he awoke, it was several minutes before (d) while he wished to kill the king, 9. He went on sowing wild oats; he
he could remember where he was or who it was the king who had saved his reaped suffering in his later life.
the strange bearded man lying on the bed life (a) inviting troubles as a boy
was. 'Forgive me!' said the bearded man 4. The bearded man swore revenge (b) warning others as a young-man
in a weak voice. on the king because (c) irresponsible pleasure seeking in
“You do not know me but I know (a) the king put his brother to death young age
you. I am that enemy of yours who and seized his property (d) sowing grains called oats when
swore revenge on you, because you (b) the king had gone alone to see the young
put my brother to death and seized my hermit 10. I don’t know why she has become
property. I knew you had gone alone to (c) the king's bodyguard wounded stand-offish recently.
see that hermit and I made up my mind him (a) angry (b) hilarious
to kill you on your way home. But the (d) the king wanted him to be his (c) indifferent (d) unmanageable
day passed and you did not return. So, faithful servant 11. In the following question, out of
I left my hiding place and I came upon Directions (5-6): In the following the four alternatives, choose the
your bodyguard, who recognised me and questions, find out the correct spelling. one which can be substituted for
wounded me. I escaped from him but I 5. (a) Denouement the given words/sentence.
should have died if you had not dressed (b) Dinouement Ready to believe anything.
my wounds. I wished to kill you and you (c) Denument (a) Credible (b) Incredible
have saved my life. Now, if I live and if (d) Dinoument (c) Credulous (d) Incredulous
116 Practice Set-10

Directions (12-13): In the following (a) RPQS (b) QRPS in a time T. The instantaneous
questions, the following sentences have (c) SPRQ (d) PRQS power delivered to the body as a
been given in Active/Passive voice. 17. In the following question, out function of time is given by:
From the given alternatives, choose of the four alternatives, choose
the one which best expresses the given the one which best expresses the mv 2 mv 2 2
(a) t (b) t
sentence in Passive/Active Voice. meaning of the given bold word. T2 T2
12. The people elected him Mayor. Irreverence
(a) Him was elected Mayor the 1mv 2 1mv 2 2
(a) disrespect (b) cruelty (c) 2
t (d) t
people. (c) unkindness (d) invalidity 2T 2T 2
(b) He was elected Mayor by the 18. In the following question, choose 23. When the disc rotates with
people. the word opposite in meaning to uniform angular velocity, which
(c) Mayor is elected by the people. the given bold word. of the following is not true?
(d) He is elected by the people Integration
Mayor. (a) The sense of rotation remains
(a) unity same
13. Don’t laugh at me. (b) synthesis
(a) Let me be laughed at. (b) The orientation of the axis of
(c) linking rotation remains same
(b) Let me be not laughed at.
(d) fragmentation (c) The speed of rotation is non-
(c) I am laughed at.
19. In the following question, four zero and remains same
(d) Let me be not laughed.
words are given in each question, (d) The angular acceleration is
Directions (14-15): In the following
questions, some of the sentences have out of which only one word is non-zero and remains same
errors and some have none. Find out rightly spelt. Find the correctly 24. If the tension on a wire is
which part of a sentence has an error. spelt word. removed at once, then:
If there is no error, then mark (d) as (a) Entirty (b) Gratious (a) It will break
your answer. (c) Discern (d) Contestent (b) Its temperature will reduce
14. Gowri told me (a)/his nam after (b)/ 20. Fill in the blank with suitable (c) there will be no change in its
he left. (c)/No Error (d). option. temperature
15. John would have told (a)/you the He was ______ from all charges (d) Its temperature increases
truth (b)/if you had asked him. (c)/ levied against him. 25. A thermodynamic system is
No Error (d). (a) exonerated (b) judged taken through the cycle ABCD
16. In the following question, the first (c) enigmatic (d) apprehended as shown in figure. Heat rejected
and the last sentence of the passage
by the gas during the cycle is:
are numbered 1 and 6. The rest of PHYSICS
the passage is split into four parts C
and named P, Q, R and S. These 21. Four blocks of same mass 2p D
Pressure →

four parts are not given in their connected by cords are pulled by
proper order. Read the sentence a force F on a smooth horizontal
and find out which of the four surface, as shown in figure. The
combinations is correct. tensions T1, T2 and T3 will be: p
1. A bad habit is harmful, none T3
A B
T1 T2
as harmful as smoking. F←M←M← M←M
P. But habit is second nature,
smokers remain smokers for 1 3 1 V Volume 3V
(a) T1 = F, T2 = F, T3 = F
life. 4 2 4
Q. Besides being expensive, (a) 2pV (b) 4 pV
1 1 1
smoking does injury to one’s (b) T1 = F, T2 = F, T3 = F
health. 4 2 2 (c) 1 pV (d) pV
R. In the long run he may 3 1 1 2
(c) T1 = F, T2 = F, T3 = F 26. If the length of a pendulum is
get something worse-lung 4 2 4
cancer. quadrupled, its time period is:
3 1 1
S. A smoker gets nothing but (d) T1 = F, T2 = F, T3 = F (a) quadrupled
smoke for his money. 4 2 2 (b) halved
6. Then why get that bad 22. A body of mass m is accelerated
(c) doubled
habit? uniformly from rest to a speed v
(d) unchanged
Practice Set-10 117

27. A parallel plate capacitor is 31. When an ideal monoatomic gas µ0 i µ0 i


located horizontally such that is heated at constant pressure. (a) 4 2 (b) 8 2
4π l 4π l
one of the plates is submerged in fraction of heat energy supplied
a liquid while the other is above which increases the internal µ0 i µ0 i
energy of gas is: (c) 16 2 4 π l (d) 30 2
the liquid surface. When plates 4π l
are charged, the level of liquid: 2 3 36. Rutherford’s experiments on
(a) (b)
5 5 scattering of α-particles proved
that:
(c) 3 (d) 3 (a) atom is mostly empty
7 4
(b) positive charge is uniformly
Changed Liquid 32. If the earth stops moving around distribution in atom
its polar axis then what will be (c) number of positive charges
effect on body placed at south is equal to the number of
(a) rises axis? negative charges
(b) falls (a) Remain same (d) atoms contain electrons
(c) remains unchanged (b) Increases 37. A beam of electron passes undeflected
(d) may rise or fall depending on (c) Decrease but not zero
the amount of charge through mutually perpendicular
(d) Decrease and zero
28. A current loop in a magnetic electric and magnetic fields. If the
33. Hole is:
field: electric field is switched off and the
(a) an anti-particle of electron
(a) experiences a torque whether same magnetic field is maintained,
(b) a vacancy created when an
the field is uniform or non- the electrons move:
uniform in all orientations electron leaves a covalent
bond (a) in an elliptical orbit
(b) can be in equilibrium in one
orientations (c) absence of free electrons (b) in a circular orbit
(c) can be equilibrium in two (d) an artifically created particle (c) along a parabolic path
orientation, both the equilibrium 34. Two cells having the same emf, (d) along a straight line
states are unstable are connected in series through 38. The Young’s double slit experiment
(d) can be in equilibrium in two an external resistance R. Cells when violet light of wavelength
orientations, one stable while have internal resistances circuit 4358 Å is used, the 84 fringes
the other is unstable r1 and r2 (r1 > r2), respactively. are seen in the field of view, but
29. A square of side L m lies in the When the effect is closed, the when sodium light of certain
potential difference across the wavelength is used, then 62 fringes
xy-plane in a region, where the
are seen in the field of view, the
magnetic field is given by: first cell is zero. The value of R
wavelength of sodium light is:
is:

B = B 0 (2iˆ + 3 ˆj + 4kˆ )T, r1 + r2 (a) 6893 Å (b) 5904 Å
(a) r1 – r2 (b)
2 (c) 5523 Å (d) 6420 Å
where B0 is constant. The magnitude
r−r 39. Which of the following properties
of flux passing through the square (c) 1 2 (d) r1 + r2 can be different along different
is: 2
direction in a crystalline solid?
(a) 2B0L2Wb 35. A square wire of each sidel (a) Electrical conductivity
(b) 3B0L2Wb carriers a current I. What is the (b) Refractive index
(c) 4B0L2Wb magnetic field at the mid-point (c) Mechanical strength
of the square? (d) All of the above
(d) 29 B0 L2 Wb
I 40. Which of the following principles
30. The speed of electromagnetic is being used in Sonar technology?
waves in a vacuum is given by: (a) Reflection of ultrasonic waves
I O I (b) Newton’s laws of motion
1 1
(a) (b) (c) Reflection of electromagnetic
µ0 ε 0 µ0 ε 0 waves
I
(c) µ0 ε 0 (d) µ0 ε 0 (d) Laws of thermodynamics
118 Practice Set-10

41. What is E in the Einstein’s 1 0 2 0 0


photoelectric equation, E = hv – φ0, (a) 1 (b) 0
2 2  0 5 0 
where v is the frequency of incident 53. If A =  2 0 0 −1 and B =
radiations and φ is the work 3  
(c) 1 (d) 0 5 0 2
function? 2
(a) Kinetic energy of every 0 2 0 0
0 5 0 
photoelectron MATHEMATICS  0
(b) M e a n k i n e t i c e n e rg y o f 46. If a finite set S has ‘n’ elements, 2 0 0 −1 ,then in the product
photoelectron  
then the number of proper subsets 0 5 0 2
(c) Minimum kinetic energy of
excluding empty sets and itself are: C = AB, the element C23 is equal to:
photoelectrons
(a) 2n – 2 (b) 2.2n –1 (a) – 1 (b) 0
(d) Maximum kinetic energy of
(c) 2(2 – 1)
n
(d) 2n –1 – 1 (c) 5 (d)  N o n e o f
photoelectrons
47. The smallest integer value of n for these
42. A body is thrown vertically
upward in air when air resistance n 54. C1 + 2.C2 + 3.C3 + ..... + nCn, have
 1+ i 
is taken into account, the time of which  1 − i  = 1, is: value:
ascent is t1 and time of descent is (a) 9 (b) 8 (a) n2n (b) 2n
t2, then which of the following is (c) 16 (d) None of (c) n.2 n–1
(d) n2n–2
true? these 55. In a(cos 45° cos 15° – sin 45° sin
(a) t1 = t2 (b) t1 < t2 48. In the set A, the relation of 15°) is equal to 1, then the value
equality is: of ‘a’ is:
(c) t > t1 2
(d) t <
1> t 2 (a) Reflexive and transitive
2
43. Pascal-second has the dimensions (b) Reflexive and symmetric (a) 1 (b)
of: (c) Transitive and symmetric 2 3
(a) Force (d) Equivalence relation 3
49. The sum of all even numbers (c) (d) 2
(b) Energy 2
(c) Pressure between 100 and 200 will be:
(d) Coefficient of viscosity (a) 5640 (b) 7650 56. The maximum value of
44. A microscope is focussed on a (c) 6750 (d) 6340
50. If S1, S2, S3, ...... Sp are sum of
(cos θ − )
3 sin θ is–
mark on a piece of paper and (a) 0 (b) 1
those infinite geometric series
then a slab of glass of thickness (c) 2 (d)  N o n e o f
whose first terms are 1, 2, 3,
3 cm and refractive index 1.5 ..... p and common ratios are these
is placed over the mark. How 57. The abscissae of the two point A
should the microscope be moved 1 1 1 1
, , ,...... respectively, then and B are the roots of the equation
to get the mark in focus again? 2 3 4 p +1
x2 + 2x – 4 = 0 and their ordinates
(a) 1 cm upard the value of S1 + S2 + S3 + ...... + Sp are the roots of the equation x2 +
(b) 4.5 cm downward
will be: 4x – 9 = 0. Then the equation of
(c) 1 cm downward
the circle with AB as the diameter
(d) 2 cm upward (a) p(p + 1) (b) 1
p ( p + 1) is:
45. In an electrical circuit R, L, C 2
and an AC voltage source are 1 (a) x2 + y2 + 6x + 10y + 13 = 0
(c) 1 p( p + 3) (d) p ( p + 5)
all connected in series. When 2 2 (b) x2 + y2 + 4x + 4y + 13 = 0
L is removed from the circuit, (c) x2 + y2 + 2x + 4y – 13 = 0
51. If 2n + 1 Pn – 1: 2n – 1Pn = 3: 5, then value
the phase difference between of n will be: (d) x2 + y2 + 2x + 2y – 13 = 0
the voltage and the current in (a) 2 (b) 3 58. If area of any triangle is 96 square
(c) 4 (d) 6
the circuit is π . If instead, C 52. Quadratic equation whose roots
unit and radii of escribed circles
3 are 8, 12, 24 units, then sides of
are 3 and 5 is:
is removed from the circuit, the
(a) x2 – 8x + 15 = 0 triangle will be:
π
phase difference is again . The (b) x2 + 8x + 15 = 0 (a) 12, 16, 20 (b) 10, 14, 18
3 (c) x2 + 8x – 15 = 0 (c) 13, 15, 17 (d) 20, 18, 8
power factor of the circuit is: (d) x2 – 8x – 15 = 0
Practice Set-10 119

59. In a frequency distribution, π π 2 2 2


(a) (b) (b) ± a m + b
the frequencies of two classes 4 6
are missing. However, the total π
frequency and the median of the (c) π (d) (c) ± b 2 m 2 − a 2
complete data are known. The 2 3
data is as follows: 65. In order that the function f 2 2
(d) ± a m − b
2

Class interval Frequency defined by f(x) = (x + 1)cot x be


continuous at x = 0, how must f(0) 70. Find S. D. of the following:
0–10 6
be defined?
10–20 – Expenditure No. of the goats
(a) f (0) = 0 (in `)
20–30 30
(b) f (0) = 1/e 0–4 6
30–40 28
(c) f (0) = e
40–50 – 4–8 10
The total frequency is 80 and the (d) f (0) cannot be defined to made
8–12 12
median is 28. The frequency of the f continuous at x = 0
66. The differential coefficient of tan–1 12–16 10
class 40–50 is:
(a) 9 (b) 6 1+ x is: 16–20 8
(c) 7 (d) 8 1− x
60. The length of latus rectum of the (a) 5.6 (b) 6.7
parabola 9x3 – 6x + 36y + 19 = 0, 1 −1 (c) 7.2 (d) 26.60
(a) (b)
is: 1+ x2 1+ x2
REASONING AND GENERAL
(a) 36 (b) 6 1 1
(c) (d) AWARENESS (RAGA)
(c) 9 (d) 4 1+ x 1− x 2 71. Decibel is a term connected with:
61. The order and degree of
2
( x + 1) (a) Air (b) Water
2
d r  dr 
2
67. ∫ x
dx is equal to:
(c) Sound (d) Soil
= 4 r +   are:
dπ2  dθ  2 5 / 2 3 3 / 2 1/ 2 72. The Accounting year of Reserve
(a) x + x +x
(a) 2 and 2 (b) 1and 2 5 4 Bank of India runs from:
(a) April to March
(c) 2 and 1 (d) 2 and 4 (b) 2 x5 / 2 + 4 x3 / 2 + 2 x1/ 2
62. If the straight line x cos α + y 5 3 (b) July to June
sin α = p touches the hyperbola (c) January to December
(c) 5 x5/2 + x1/2 + 1 (d) August to July
x2 y2 2
− = 1, then: 73. The food stocks that are built
a 2 b2 (d) None of these up during the years of bumper
(a) p2 = a2 cos2 α + b2 sin2 α harvest are called:
π/2 2+ x (a) Capital stock
(b) p2 = a2 cos2 α – b2 sin2 α 68. The value of ∫a 2− x
dx is:
(c) p2 = a2 sin2 α + b2 cos2 α (b) Buffer stock
(d) p2 = a2 sin2 α – b2 cos2 α 3 (c) Production stock
(a) π + 1 (b) π+ (d) Grain stock
63. If the coordinates of the centroid 2 2
of a triangle are (3, 2) and the 74. Nomination of Rajya Sabha
coordinates of two vertices are (1, (c) π + 2 (d)  N o n e of Members by the President was
these taken from the Constitution of :
1) and (2, 5), then the coordinates
69. If a straight line y = mx + c, (a) U.S.A. (b) Ireland
of the third vertex are:
(c) South Africa (d) France
(a) (6, 8) (b) (6, 0) 2 2
tonches the ellipse x + y = 1, 75. What was the name of the
 8 b2 a 2 Newspaper edited by Gandhiji
(c) (0, – 4) (d)  2,  the value of c will be: till 1933?
3
1 1 (a) Sarvodaya
64. 2tan
−1
+ tan −1 is equal to: (a)
3 7 ± b2 m2 + a 2 (b) Arya
120 Practice Set-10

(c) Times of India 83. A wholesaler sells a watch to a walks 4 m towards east. How far
(d) Young India retailer at a gain of 35% and the and in which direction is he from
76. Where does a computer add, retailer sells it to a customer at a his initial position?
compare and shuffle data? loss of 25%. If the customer pays (a) 25 m East
(a) Memory chip ` 2379.375, what had it cost the
(b) 25 m North-east
(b) CPU chip wholesaler?
(c) Floppy disk (a) ` 2350 (b) ` 2409 (c) 20 m North
(d) Hard disk (c) ` 2928 (d) ` 2291 (d) 24 m North
77. Which organisation organised 84. The sum of the ages of husband 89 . Saurav, Mohit, Mukesh, Sumit
a session on ‘Economic Policy and wife at present is 60. Four and Bhim are arranged in
– The Road Ahead’, in which years ago the ratio of their ages descending order of their height
expert panel for macroeconomics was 7 : 6. What is the age of the from top to bottom. Saurav is
and employment suggested ways husband ? at third place. Bhim is between
to achieve $5 trillion economy ? (a) 28 years (b) 32 years Sumit and Saurav while Sumit
(a) Securities and Exchange is not at the top end. Who is at
Board of India(SEBI) (c) 42 years (d) 38 years
second place from the top?
(b) National Institution for 85. To travel 648 km, an Express
(a) Mohit
Transforming India(NITI) train takes 12 hours more than
(b) Mukesh
Aayog Rajdhani. If however, the speed
of the Express train is doubled, it (c) Bhim
(c) National Development
takes 6 hours less than Rajdhani. (d) Cannot be determined
Council
The speed of Rajdhani is Direction (90): A series is given with
(d) Planning Commission one term missing. Choose the correct
78. What is the value of 5/9 + 7/8 ? _______.
alternative from the given ones that
(a) 103/72 (b) 97/42 (a) 36 km/hr (b) 18 km/hr will complete the series.
(c) 12/17 (d) 15/13 (c) 45 km/hr (d) 54 km/hr 90. In the following question,
79. The sum of two numbers is Direction (86): Study the following which one set of letters when
216 and their H.C.F. is 27. The information carefully to answer these sequentially placed at the gaps
numbers are: questions. in the given letter series shall
(a) 81, 135 (b) 81, 189 complete it?
(i) ‘A$B’ means ‘A is mother of B’
(c) 108, 108 (d) 37, 179 ab_d_bb_a_bd
(ii) ‘A#B’ means ‘A is the father of B’
x x (a) badd (b) babd
80. Find x if – = 5. (iii) ‘A@B’ means ‘A is the husband of B’
3 4 (c) badb (d) abdd
(iv) ‘A%B’ means ‘A is daughter of B’
(a) – 60 (b) – 100 Direction (91): Study the following
86. If F@D%K#H, then how is F information carefully and answer the
(c) 100 (d) 60
related to H? questions given below:
81. A factory buys 8 machines.
3 Machine A, 4 Machine B (a) Brother-in-law A, B, C, D, E, F, G and H are sitting
and rest Machine C. Prices of (b) Sister around a circle facing the centre. H is
the machines are ` 45000, ` fourth to the left of B and second to
(c) Sister-in-law
25000 and ` 35000 respectively. the right of F. A is third to the left of
(d) Cannot be determined C, who is not an immediate neighbour
Calculate the average cost (in `)
87. If ‘BE Quick’ is coded as ‘ZC of F. G is second to the left of A. D is
of these machines ?
OSGAI’, then the code of last second to the right of E.
(a) 33750 (b) 35000
letter of third word in the 91. Who is on the immediate right of
(c) 36250 (d) 32250
sentence ‘I LOVE my country’ H?
82. In DABC, D and E are points
is? (a) E
on side AB and AC respectively.
DE is parallel to BC. If lengths (a) W (b) U (b) C
of AD, DB and DE are 8 cm, 6 (c) T (d) A (c) H
cm and 7.2 cm respectively, find 88. A man walks 20 m towards (d) Data inadequate
length of BC. north, then he turns right and Direction (92): In following questions
(a) 12.6 cm (b) 9.3 cm walks 3 m, then turns left and you have to identify the responses
(c) 8.1 cm (d) 5.4 cm walks 4 m and from there he which would be a correct inference
Practice Set-10 121

from the given premises stated (a) i, ii, iii, iv (b) iv, iii, i, ii (a) Second to the left
according to the following systems: (c) ii, iii, i, iv (d) iv, iii, ii, i (b) Third to the left
'A' stands for Not greater than. Direction (95): The following questions (c) Third to the right
'B' stands for Greater than are based upon the alphabetical series
given below: (d) None of these
'C' stands for Not equal to
S L U A Y J V E I O N Q 97. Which of the following pairs has
'D' stands for Equal to the first person to the immediate
G Z B D R H
'E' stands for not less than left of the second person?
95. If 'SU' is related to 'HD' and ‘UY’
'F' stands for less than. is related to 'DZ' in a certain (a) RP (b) FJ
92. Premises: (P A 3q) and (3q F 2r). way, to which of the following is (c) VT (d) JK
(a) P D 2r (b) P F 2r YV related to, following the same 98. Who is to the immediate right
(c) P B 2r (d) P E 2r pattern?
of F?
93. From the given alternative (a) ZQ (b) IN
(a) J (b) P
words, select the word which (c) BG (d) QO
(c) R (d) V
cannot be formed using the Directions (96–100): Study the
99. Which of the following is the
letters of the given word. following information carefully to
correct position of V?
ARCHITECTURE answer these questions.
Eight friends P, R, T, V, W, J, F & K (a) Second to the left of R
(a) TACT (b) CHAT
are sitting around a circle facing the (b) Between T & P
(c) TORCH (d) RICH
centre. T is third to the right of F and (c) To the immediate right of W
Direction (94): Arrange the given (d) To the immediate left of W
second to the left of W, P & R are not
words in the sequence in which they 100. Who is second to the right of J?
neighbours of W. R is third to the right
occur in the dictionary. (a) F
of K. J is between W & K.
94. 96. Which of the following is the (b) V
i. Treasure ii. Treat correct position of T with respect (c) W
iii. Tremor iv. Trestle to K's position? (d) P

Answers with Explanation


ENGLISH The association’s stand-offish attitude combination of two things such that one
towards Mr. Sen’s bid for the presidency becomes fully a part of the other. Hence,
1. (b) 2. (c) 3. (d) 4. (a)
in the last election hurt him. its antonym is ‘fragmentation’.
5. (a) C o r r e c t s p e l l i n g i s
11. (c) 12. (b) 13. (b) 19. (c) 20. (a)
Denouement which means the final
14. (c) If two actions happended in
resolution of the intricacies of a plot, as PHYSICS
the past, it may be necessary to show
of a drama or novel.
which action happened earlier than the 21. (c)
6. (a)
other. The action happened earlier is F ← m1 → T1 ← m2 → T2 ← m3 → T3 ← m4
7. (a) The correct spelling is ‘elixir’
expressed in Past Perfect and the later T1 =
(m1 + m3 + m4 )
F
which means the quintessence or absolute (m1 + m2 + m3 + m4 )
action is expressed in Simple Past. For
embodiment of anything. example. Given, m1 = m2 = m3 = m4
8. (c) The correct option is ‘A’ i.e. A Mohan had left before Sohan arrived. 3
∴ T1 = F
better solution can still be worked out. I reached after the train had already 4
9. (c) The idiom ‘sow (one’s) wild left. Similary, T2 =
(m3 + m4 )F
oats’ means to through a youthful m1 + m2 + m3 + m4
Therefore, ‘he left’ should be replaced
period of careless, or reckless, pleasure- Given, m1 = m2 = m3 = m4 = m
with ‘he had left.
seeking’. For example, He sowed all his 1
15. (d) No Error ∴ T2 = F
2
wild oats before he married. 16. (b)
17. (a) The word “irreverence” m4 F
10. (c) The phrase ‘stand-offish’ Also, T3 =
m1 + m2 + m3 + m4
means ‘discouraging’ or distant. For (Noun) means ‘no respect’.
1
example. 18. (d) The word ‘integration’ means Given m1 = m2 = m3 = m4 = m ⇒ T3 = F
4
122 Practice Set-10

22. (a) →
net emf
A = L2 kˆ and B = B0 (2iˆ + 3 ˆj + 4kˆ), φ = ? I=
mv Net resistance
F = ma = As
T
→ 2E
Instantaneous power = Fv = mav A = L2 kˆ and B = B0 (2iˆ + 3 ˆj + 4kˆ), φ = ? ⇒ I = ...(i )
2
r1 + r2 + R
mv mv v mv
= . at = . . t = 2 .tt ∴ φ = 4B0 L2 Wb It is given that, as circuit is closed,
T T T T
30. (b) Speed in vacuum For first call i.e.,
23. (d) Since, the disc rotates with
1 E − Ir 1 = 0
uniform angular velocity, So, the angular =
dω µ0 ε 0 E
acceleration ∝ mut, be zero. So, ⇒ I = ...(ii )
dt 31. (b) For monatomic gas r1
option (d) is the only incorrect option. Cp 5
γ = = Equating Eqs. (i) and (ii), we get
24. (d) Due to tension, intermolecular Cv 3
E 2E
distance between atoms is increased and = = r1 − r2
We know that, 4 r1 + r2 + R
therefore potential energy of the wire is ∆Q = µC p ∆T and ∆U = µCv ∆T
35. (b)
increased and with the removal of force ∆U C 3
= v = µ I 
interatomic distance is reduced and so is ∆Q Cp 5 B= 0 (sin φ + sin φ 2 ) 
the potential energy.
 4π a 
i.e. fraction of heat energy to increase µ0
This change in potential energy I  1 1 
3 =4×  +
appears as heat in the wire thereby the internal energy be . 4π (l / 2)  2 2 
increasing its temperature. 5
32. (a) Variation in g due to rotation
16  µ0 I µ I
25. (a) For given cyclic process, of earth =4×  4π  =8 2  0
2 l  4π l 
∆U = 0 g ' = g − ω2 Rcos 2 λ
∴ Q =W Here ϕ1= ϕ2 = 45º
At poles, λ = 90° in the above
AlsoW = – area enclosed by the curve 36. (a) Rutherford’s experiments on
expression, we get
= AB × AD = (2p – p) (2V – V) = – p scattering of α-particles proved that atom
g pole = g − ω2 R cos 2 90°
× 2V
∴ g pole = g is mostly empty.
∴ Heat rejected = 2 pV
37. (b) If both electric and magentic
26. (c) We know that i.e, there is no effect of rotational fields are present and perpendicular to
l motion of the earth on the value of g at
T = 2π each other and the particle is moving
g poles.
perpendicular to both of them with
33. (b) When an electron leaves a Fe = Fm
l 4l
T = 2π ⇒ T ' = 2π covalent a vacancy is created which is → →
g g called hole. In this situation E ≠ 0 and B ≠ 0 .
T' 34. (a) Net resistance of the circuit Fe
⇒ =2 ⇒ T' = 2T
T = r1 + r2 + R
When l is made four times, the time Net emf in series = E + E = 2E
period is doubled. E E
E q v
27. (a) When plates of the capacitor
are charged, then opposite charges are r1 r2
I r1 > r2
induced on water so due to attractive
Fm
force water level will rise.
But if electric field becomes zero, then
28. (d) For parallel magnetic field is R only force due to magnetic field exists.
stable and for anti-parallel is unstable. Therefore, from Ohm’s law current Under this force, the charge moves along
29. (c) Here, in the circuit a circle.
Practice Set-10 123

38. (b) Its field of view is same in 43. (d) Pascal is unit of pressure, hence
MATHEMATICS
both cases its dimensional formula is [ML–1T–2]
n1β1 = n2β 2 ∴ Dimensional formula of Pascal- 46. (a) No. of subsets of A is 2n.
second is [ML–1T–1] This number inludes φ and ‘A’, hence
 Dλ   Dλ 2 
n1  1  = n2  From the formula of coefficient of set of non-empty subsets of A is 2n – 2
 d   d 
viscosity, we have = 2(2n –1 – 1).
n  F 47. (d)
λ 2 =  1  λ1 η =
 n2   ∆V  1 + i (1 + i )(1 + i ) 1 + l 2 + 2i
A   = =
 84   ∆Z  1 − i (1 − i )(1 + i ) 1− i2
λ 2 =   × 4358
 62  ∆V 1 − 1 + 2i 2i
Where F is force, A is area and = = =i
λ 2 = 5904 Å ∆Z 1+1 2
is velocity gradient.
39. (d) Electrical conductivity, ∴ Dimensions of
n
refractive index and mechanical strength [MLT −2 ]  1 + i)n 
are the properties can be different along η = = [ML1T −1 ] ∴  1 − i  =in
[L ] [LT −1 / L]
2
 
different directions in a crystalline solid.
40. (a) In sonar technology, the Hence, Pascal-second has dimensions ⇒ Since given that
reflection of ultrasonic waves principle of coefficient of viscosity. n
 1+ i 
are used. 44. (a) Apparent depth of mark as seen  1 − i  =1
41. (d) In the Einstein’s photoelectric through a glass slab of thickness x and
equation (E = hν − ω0 ), E is maximum ⇒ in =1
refractive index µ is
kinetic energy of photoelectrons. ∴ Smallest positive integer for n = 4,
Apparent depth
42. (b) First of all note that viscous
Real depth 3 because i4 = 1 i4 =1 = in.
force acts in direction opposite to the = = = 2 cm
motion of body. When body is thrown Refractive index 1.5 48. (a) Let three elements x, y, z ∈ A
up then both the de-accelerating forces As image appears to be raised by (i) Since, every element of a set is
i.e. gravity and viscous force act in same 1 cm, therefore microscope must be equal to itself.
direction. So, total de-acceleration is a1 = moved upward by 1 cm. Hence, relation of equality is reflexive.
g + a0, where a0 is de-acceleration due to 45. (c) Here, phase difference
viscous force which is assumed constant. (ii) If x = y ⇒ y = x, ∀ x, y ∈ A.
X − Xc
If U be the initial velocity and t1, be tan φ = L Hence, relation of equality is symmetric.
R
the time of ascent, then (iii) If x = y and x = z ⇒ x = z, ∀ x,
u π X − Rc y, z ∈ R
t1 = ⇒ u = t1 ( g + a0 ) ⇒ tan = L
g + a0 3 R
Hence, relation of equality is transitive.
2 When L is removed
v Thus, the relation of equality is
and h = X
2( g + a0 ) 3= c reflexive, symmetric and transitive i.e.
Also, t2 is time of descent then R relation of equality is equivalence.
1 ⇒ Xc = 3R 49. (b) Let all even numbers are n,
h = ut + a2 t22
2 then nth term of series = 200, a = 100
When C is removed and d =102 – 100 = 2
⇒ u2 1
= 0 + × [ g − a0 ] × t22 XL ∴200 =100 +(n – 1) ×2
2( g + a0 ) 2 3 =
R ⇒2n =102
[∴ a2 = g − a0 during descent]
∴n = 51
u g + a0 ⇒ X L = 3R So, required sum
⇒ t2 = = t1
( g + a0 ) ( g − a0 ) g − a0
Z= R 2 + (X L = X c ) 2 = R 1
= × 51[2 × 100 + (51 − 1) × 2]
t g + a0 Z R 2
i.e. , 2 = > 1 ⇒ t 2 > tt cos φ = = =1
t1 g − a0 R R =7650
124 Practice Set-10

a = (0.2 + 0.1 + 5.1 + 0. (–2)] = 5 ⇒ x2 + 2x – 4 + y2 + 4y – 9 =0


50. (c) Form S ∞ = 54. (c) L.H.S. = C1 + 2.C2 + 3. C3 + ..... ⇒ x2 + y2 +2x + 4y –13 =0
1− r
+ nCn 58. (a) Given
1
S1 = =2 ∆ = 96, r1 = 8, r2 = 12, r3 = 24
1 n(n − 1) n(n − 1)(n − 2)
1− =n+ 2 +3 + ....... + n
2 2 3
∆ ∆ 96
 r1 = ∴ s – a = = = 12
2  (n − 1)(n − 2)  s−a r1 8
S2 = =3 = n 1 + (n − 1) + + ...... + 1
1  2 
1− ∆ ∆ 96
3 = n[n–1c0 + n–1c1 + n–1c2 + ......  r2 = ∴s–b= = =8
+ n–1cn –1] s−b r2 12
3
S3 = =4
1 = n(1 + 1) = n. 2 n–1 n–1
∆ ∆ 96
1−  r3 = ∴s–c= = =4
4 55. (d) a(cos 45° cos 15° – sin 45° s−c r3 24
sin 15° = 1
∴ S1 + S2 + S3 + ..... Sp =2 + 3 + 4 + ....... Sp
⇒ a cos(45° + 15°) = 1 ∴ (s – a) + (s – b) + (s – c) =12 +8 + 4
P
= [2 × 2 + ( P − 1).1] ⇒ a cos 60° = 1 = 24
2
⇒ 3s – (a + b + c) =24
P 1
= [4 + p − 1] ⇒ a× =1 ⇒ 3s – 2s =24
2 2
∴ s =24
P ∴ a=2
∴ a =s – 12 = 24 – 12 = 12
= ( p + 3) 56. (c) cos θ − 3 sin θ
2 b = s – 8 = 24 – 8 = 16
51. (a) According to question, c = s – 4 = 24 –4 = 20
1 3 
2n +1
Pn – 1: 2n – 1Pn =3: 5 = 2  cos θ − sin θ Therefore, the sides of ∆ are 12, 16, 20.
2 2 
N
(2n + 1)! (2n − 1)! 3 = 2 (sin 30° cos θ – cos 30° sin θ) −F
⇒ : = 59. (b ) Median = L + 2 ×i
(n + 2)! (n − 1)! 5 = 2 sin (30° – θ)
=2×1 f
[(2n − 1)!× 2n(2n + 1)] (n − 1)! 3 [Because maximum value of sin 80
⇒ × = − (6 + f1 )
[(n − 1)!× n(n + 1)(n + 2)] (2n − 1)! 5 (30° – θ = 1] ⇒ 28 = 2020++ 2 × 10
2(2n + 1) =2 30
3
⇒ = 57. (c) If the roots of the equation x2
(n + 1)(n + 2) 5 40 − 6 − f1
+ 2x – 4 = 0 are α and β then α + β = –2 = 2020++
⇒10(2n + 1) = 3(n + 1)(n + 2) and αβ = – 4 3
⇒3n2 – 11n – 4 = 0 In the same way if the roots of the ⇒ 8 × 3 =34 – f1
⇒(n – 4)(3n + 1) = 0 equation x2 + 4x – 9 = 0 are γ and then γ ⇒ f1 = 34 – 24 = 10
1 + δ = – 4 and γδ = – 9.
n =4 or − ∴ Frequency of class 40 – 50
3 = 80 – (6 + 10 + 30 + 28) = 6
∴ The value of n will be positive 60. (d) Given parabola is 9x2 – 6x +
integer, 36y + 19 = 0
∴n =4 ⇒ (3x – 1)2 = –18(2y + 1)
(α, γ)

52. (a) Quadratic equation whose (β, δ)


2
roots are 3 and 5 is x2 – 8x + 15 =0  1  1
⇒  x −  = − 4 y + 
53. (c) Given that C =AB  3  2
c23 =(2nd row of A) (3rd column of B) ⇒ X2 = – 4AY
2 1
1  ∴ Equation of the circle will be: where X = x −
= [0 050]   (x – α)(x – β) + (y – γ)(y – δ) =0 3
1 
  ⇒ x2 – (α + β)x + αβ + y2 – (γ + δ)y 1
− 2 and Y = y +
+ γδ =0 2
Practice Set-10 125

∴ A =1  1 ( x + 1) 2
∴ Length of the latus rectum = 4A = 4  2×  67. (b) ∫ dx
1 −1 3 x
64. (a) 2 tan1 = tan  
3 1
 1− 
2  x 2 + 2 x + 1
d 2r  dr   9 =∫
61. (d) = r +  d θ 
4
−1  3 
 dx
dθ 2 = tan    x 
 4
4
 d 2r  2 1 1 = ∫ ( x3/2 + 2 x1/2 + x −1/2 )dx
 dr  ∴ 2 tan −1 + tan −1
⇒  d θ 2  = r +  3 7
 dθ x5/2 x3/2 x1/2
3 1 = + 2. +x
d 2r = tan −1 + tan −1 5 3 1
The highest derivative in it is 4 7
2 2 2 2
whose order is 2 and degree is 4. dθ
 3 1  2 5/2 4 3/2
62. (b) From the equation x cos α + + = x + x + 2 x1/2
−1  4 7 
y sin α = p. = tan  3 1 5 3
1 − ×  1/2
 4 7 2 2+ x 2 2 + x
− x cos α p 68. (c) ∫ dx = ∫0  dx
y= + 0 2− x  2 − x 
sin α sin α –1− 1
π
(1)==
tan (1)
= tan
Putting the value of y in the equation 4 Letx = 2 cos 2θ
of hyperbola, we get ∴dx = – 4 sin 20 dθ
lim (x + 1)cot x
65. (c)LetA = x→ 0
2 π
x2 1  − x cos α p  when x = 0, then θ =
− 2 + =1 lim cot x log (1 + x)
log A = x→ 4
2
a b  sin α sin α  0
and when x = 2, then θ = 0
 2  log (1 + x)  0
⇒  1 − cos a  x 2 + 2 p cos α.x = lim from 0  2 2+ x
a 2 2 2
b sin α  b 2 sin 2 α
x→0 tan x ∴ ∫0 dx
2− x
 − p2  1
+  2 2 − 1
0 cos θ
(1 + x) = – 4∫ π sin 2θ d θ
 b sin α  =0 = lim =1 4
sin θ
x→0 sec 2 x
If the straight line x cos α + y sin α = π /4
[By L’Hospital rule] = 8∫ cos 2 θ d θ
p, touches the hyperbola, then 0
⇒A=e π/4
2 For continuity lim f(x) = f (0) = 4∫ (1 + cos 2θ) d θ
0
 2 p cos α  ⇒ f (0) = e
x→ 0
 2 2  = sin 2θ 
π /4
 b sin α  = 4 θ +
1+ x  2  0
66. (a) Lety = tan −1
 1 2
cos α   − p  2 1– x  π sin 90° 
4  2 − 2 2   2 2 − 1 = 4 + 
a b sin α   b sin α  4 2 
Put x = tan θ
⇒ p =a cos α – b sin α
2 2 2 2 2 =π+2
−1 1 + tan θ 69. (a)Putting y = mx + c in the
63. (b) Let the coordinates of the third ∴y = tan equation
1– tan θ
vertex be (x, y),
x2 y2
1+ 2 + x −1 π  π       + =1
∴ =3 = tan tan  + 0 = + 0 b2 a2
3 4  4
⇒ x=6 π we get       x2 (mx + c) 2
y= + tan −1 x +
4 =1 b2 a2
1+ 5 + y
and =2
3 dy 1 1
∴ = 0+ = ⇒ (a2 + b2m2)x2 +2 mcb2x + (b2c2 –
⇒ y=0 dx 1+ x 1+ x2
2
ab)=0
2 2
126 Practice Set-10

If y = mx + c touches the ellipse, then 40 + 63 103 35% –25%


83. (a) W → R  →C
(2mcb2)2 = 4(a2 + b2m2)(b2c2 – a2b2) = =
72 72 Let C.P. of a wholesaler be x
⇒ a2 + b2m2 = c2
79. (a) Let the required numbers be 100 + 35 135 x
⇒ c = ± a 2 + b2 m2 S.P. = x =
27a and 27 b. 100 100
70. (d) 27a + 27b = 216 135 x 100 – 25
S.P. for retailer = ×
Σ× f ⇒ a + b = 8. 100 100
       X ⇒
Σf Now co-prime with sum 8 are (1, 7) 135 x 3
and (3, 5) 2379.375 = ×
⇒     476 ⇒10.34 100 4
46 ∴ Required numbers are (27×1,
2379.375 × 100 × 4
27×7) and (27× 3, 27×5) i.e., (27, 189) x=
       (X = 10.34) 3 × 135
and (81, 135). Out of these the answer
237937.5 × 400
Σ ( x − x )2 f pair is (81, 135). = = 587.5 × 4
      S.D = 3 × 135
n x x 4 x – 3x
80. (d) – =5⇒ =5 = 2350.0
3 4 12
1223.86 Hence, the cost price of a watch is
⇒        ⇒ 26.60 x = 60 ` 2350.
46
Class Mid Freq xf (x-x) (x-x)2 (x-x)2 f 81. (a) Average Cost 84. (b) Let the age of husband be
Expen point uency
3 × 45000 + 4 × 25000 x years.
diture (x) (f )
The age of wife = 60 – x
0–4 2 6 12 – 8.34 69.55 417.3 + 1 × 35000
4–8 6 10 60 – 4.34 19.09 190.9     = Four years ago
8–12 10 12 120 – 0.34 1.03 12.36
8
x−4 7
12–16 14 10 140 3.66 13.39 133.9
135000 + 100000 + 35000 =
16–20 18 8 144 7.66 58.67 469.40     = 60 − x − 4 6
Total Sf = 46 S × f = 1223.86 8
x−4 7
n = 46 476 270,000 =
    = = ` 33750. 56 − x 6
8
REASONING AND GENERAL 6x – 24 = 56 × 7 – 7x
AWARENESS (RAGA) 82. (a) In DADE and DABC
both are similar 13x = 392 + 24
71. (c) 72. (a) 73. (b) A 416
x = = 32 years
74. (b) 75. (d) 76. (a) 8 13
7.2 Age of husband = 32 years.
77. (b) National Institution for D E Age of wife = 60 – 32 = 28 years.
Transforming India (NITI) Aayog 6
Explanation: In order to make India 85. (d) Let Rajdhani take t-hours.
B C
a $5 trillion economy, expert panel Express train takes = (t + 12) hours
for macroeconomics and employment By B.P.T. theorem
Let Speed of Rajdhani be v
suggested ways to achieve the target AD DE
during an interaction with Prime = and speed of express train = 2v
AB BC
Minister Narendra Modi. The Session
648 648
is organised by The National Institution AD 7.2 − =6
= v 2v
for Transforming India (NITI) Aayog, AD + DB BC
think tank of the government of India  2 − 1
648  =6
on ‘Economic Policy – The Road Ahead’       
8
=
7.2  v 
which saw participants of over 40 8 + 6 BC
1 6 1
economists and sectoral experts. = =
7.2 ×14 2v 648 108
BC =
5 7 8 108
78. (a) The value of +         v =
= 54 km/hr.
9 8 BC = 0.9 × 14 = 12.6 cm 2
Practice Set-10 127

86. (a) F @ D  F is the husband of D. and the direction is north-east. (i) Treasure (ii) Treat
D % K  D is the daughter of K. (iii) Tremor (iv) Trestle
89. (d) According to question:
Finally, (i), (ii), (iii), (iv) is the
K # H  K is the father of H. Sumit, Bhim, Saurav, Mohit, Mukesh
correct sequence.
Place is not confirmed from top or
bottom. So, we cannot determine who is 95. (a) S L U A Y J V E I From
Left
at the second place.
+1 +1 +1
90. (c) ab b d / a bb d / a b bd H R D B Z G Q N O
From
Finally the set of letters badb will +1 +1 +1 Right
So, F  husband of D who is the complete the series.
sister of H. 96. (d) By analysing the final
Therefore, ‘F’ is the brother-in 91. (a) By analysing the circular circular arrangement, it is clear that
-law of ‘H’. arrangement, we get Immediate right of T ⇒ fourth to the right of K and fourth
'H'  E to the left of K.
87. (a)
92. (b) According to the given 97. (d) By analysing the final
B E Q U I C K circular arrangement, it is clear that
information
−2 B B -2 −2
–2 B −2 B −–22 B −–22 B −2 B P A 3q ⇒ P >| 3q...(i) J (first person) ⇒ to the immediate left
Z C O S G A I of K (Second Person).
and 3q F 2r ⇒ 3q < 2z  ...(ii)
−2 98. (b) By analysing the final
Therefore, Y  → W From (i) and (ii), we get
P  3q < 2r...(iii) circular arrangement, it is clear that
88. (b) P ⇒ to the immediate right of F.
4m End
[ P >| 3q ⇒ P  3q]
N
From (iii), it is clear that 99. (d) By analysing the final
4m circular arrangement, it is clear that
3m W E P < 2r ⇒ P F 2r.
V → to the immediate left of W.
20 m S
93. (c) There is no ‘O’ letter in given
keyword. 100. (a) By analysing the final
Start circular arrangement, it is clear that
Total distance = 20 + (4 + 4 – 3) 94. (a) According to dictionary, F ⇒ second to the right of J.
= 20 + 5 = 25 m Arrangement of words is: qqq
128 Practice Set-11

Air Force Airmen Group X & Y Exam


Practice Set-11

(d) there are over 25 types of potatoes 10. The Interrogative of – "You had a
ENGLISH
around the world comfortable journey." is:
Directions (1-4): Read the given 2. The meaning of the word ‘epoch’ (a) Had you had a comfortable
passage and answer the questions is a journey?
that follow by selecting the most (a) being great and impressive (b) Have you a comfortable journey?
appropriate option. (b) lengthy, complex poem (c) Had you a comfortable journey?
Why do poets use poems to tell about (c) type of literary work in England (d) Did you have a comfortable
social injustices? The answer is simple.
(d) distinctive point of time journey?
This way a poet can catch and hold the
3. The purpose of poetry is to ____ Directions (11-12): Fill in the blanks
reader’s attention, his/her emotions.
the reader. with the antonym of the underlined
Usually poets in their works present
facts in order to capture attention of (a) confuse (b) distract word in each of the following sentences.
many people. These are not new facts (c) appeal to (d) disturb 11. They give more importance to
that are presented to an audience. Poems 4. The themes of poems are usually materials rather than _________
are always aimed at reaching feelings about advancement
of people and thus, pulling strings. (a) important life events (a) intellectual (b) spiritual
Literature of every state shows all the (b) everyday happenings (c) emotional (d) wordly
complexity of every epoch. When the (c) heroism and death 12. Honour and _______ arise from
situation is the same at several countries, (d) romance one's own actions.
it has a worldwide significance. Before 5. Which word is correctly spelt– (a) pleasure (b) reputation
talking about poetry, we should answer (c) shame (d) progress
(a) Duely (b) Duly
the question. What is poetry? Poetry is 13. The river flows _____ the bridge.
(c) Duley (d) Dully
a special way of describing situations, (a) on (b) above
6. Select the word with its corret
things, ideas, feelings. Poets present their (c) to (d) under
spelling:
ideas in short phrases. A poem can be 14. He said, "What a beautiful scene!"
compared to a photograph as it reflects (a) Vaccum (b) Vaccume
(a) He said that what a beautiful
real life, real situations and feelings. In a (c) Vaccuum (d) Vacuum scene it was
poem a poet captures the exact moment 7. Choose the word opposite in (b) He wondered that it was a
and represents it the way he/she has meaning to the word given below beautiful scene
seen it. When you read a poem you see in bold. (c) He exclaimed what a beautiful
the poet’s subjective evaluation of facts, OSTRACISE sence it was
situations and the epoch in general. Poets (a) Amuse (b) Welcome (d) He exclamined that it was a very
of the Romantic Movement wrote their (c) Entertain (d) Host beautiful scence
poems to share their feelings. They wrote 8. In this question, four words are 15. The noun form of the verb
to help people understand their time from 'experience' is
given out of which only one is
the poet’s point of view. misspelt. Find that mis-spelt word. (a) expirience (b) expereince
1. A n e x a m p l e o f ‘ s u b j e c t i v e (a) combination(b) exageration (c) experience (d) expireince
evaluation’ is Directions (16-17): In these questions,
(c) hallucination(d) admonition
(a) the potato has a large percentage read each sentence to find out whether
9. Find the correct sentence.
of starch content there is any error in it. The error, if
(a) Tom, I and you ate sweets any, will be in one part of the sentence.
(b) everybody likes dishes prepared
with potatoes (b) I, you and tom ate sweets The number of that part is the answer.
(c) the potato is a staple in some (c) I, Tom and you are sweets If there is no error, the answer is (d)
countries (d) You, Tom and I ate sweets (Ignore errors of Punctuation. if any.)
Practice Set-11 129

16. You must either (a)/ be regular with (b) wavelength is doubled and (a) A constant clockwise induced
your studies (b)/ and study for longer frequency becomes half current will flow in closed loop
period before the examination (c)/ (c) wavelength is halved and (b) An increasing anticlockwise
No error (d). frequency remains unchanged induced current will flow in the
17. The new taxation rates (a)/ an (d) wavelength and frequency both closed loop
nounced by the government (b)/ are remain unchanged (c) A decreasing anticlockwise
bound to effect the export sector. (c)/ 22. A uniform metal rod of 2 mm2 induced current will flow in the
No error (d). cross–section is heated from 0ºC closed loop
(d) A constant anticlockwise
Directions (18-19): In these questions, to 20ºC. The coefficient of linear
induced current will flow in the
the sentences have been given in expansion of the rod is 12 × 10–6 per closed loop
Active/Passtive Voice. From the given ºC. Y = 1011 N/m2. The energy stored
alternatives, choose the one which best 25. A particle of mass M is fired with
per unit volume of the rod is:
expresses the given sentence in Passive/ speed u making angle θ with the
(a) 2828 J/m3 (b) 1500 J/m3
Active Voice. (c) 5760 J/m3 (d) 1440 J/m3 horizontal. Its potential energy at
18. They first sun-dried the garbage 23. The variation of potential with the highest point is:
for one of three days to bring down distance R from fixed point is 1
the moisture level. (a) mu 2 sin 2 θ
shown in figure. The electric field 2
(a) The moisture level was brought at R = 5 cm is:
down by sun-drying the garbage (b) 1 mu 2 cos 2 θ
for one to three days. 5 2
(b) One of three days of sun-drying
brought down the moisture level 4 (c) 1 mu 2
of the garbage 2
3
(c) The moisture level of the garabge
(d) 1 mu 2 sin 2 2θ
came donw when it was sun-dried V 2 2
for one to three days. (volt)
1 26. A proton moving with a constant
(d) The garbage was first sundired for
velocity passes through region
one to three days to bring down 1 2 4 6
the moisture level. r (in cm) of space without any change in
19. Women like men to flatter them. its velocity. If E and B represent
(a) 2.5 V/cm (b) 2 V/cm
(a) Men are liked by women to flatter the electric and magnetic fields
them (c) –2.5 V/cm (d) 2 V/cm respectively, this region of space
(b) Women like to be flattered by men 5 may have:
(c) Women like that men should
24. A long conducting wire AH (a) E = 0, B = 0
flatter them
(d) Women are liked to be flatered by is moved over a conducting (b) E = 0, B ≠ 0
men triangular wire CDE with a (c) E ≠ 0, B = 0
20. The synonym for 'bona fide' is constant velocity v in a uniform (d) E ≠ 0, B ≠ 0

(a) honourable magnetic field B directed into the 27. A c h a r g e d o i l d r o p i s
(b) genuine paper. Resistance per unit length suspended in uniform field of
(c) well made of each wire is ρ. Then:
3 × 104 V/m, so that it neither
(d) remunerative D falls nor rises. The charge on the
× B drop will be: (Take the mass of
PHYSICS
F G the charge
21. An electromagnetic wave of A H
= 9.9 × 10–15 kg and g = 10 m/s2)
frequency v = 3.0 MHz passes
from vacuum into a dielectric (a) 3.3 × 10–18 C
medium with permittivity V (b) 3.2 × 10–18 C
ε = 4.0 Then:
(c) 1.6 × 10–18 C
(a) wavelength is doubled and
C E (d) 4.8 × 10–18 C
frequency unchanged
130 Practice Set-11
  0.2 then the heat generated in 55 to x1 and x2. Find dimension of D
28. If E is an electric field and B is
the magnetic induction then the will be equal to: called as diffusion constant?
energy flow per unit area per unit (a) 9.33 cal (b) 9.33 k cal (a) [LT2] (b) [L2T–4]
time in an electromagnetic field is (c) 9.33 J (d) 9.33 erg (c) [LT ] –3
(d) [L2T–1]
given by: 34. Two paper screens A and B are 38. A full wave rectifier circuit along
    separated by a distance of 100 m. A with the output is shown in figure:
(a) E × B (b) E . B 2
bullet pierces A and then B. The hole
(c) E 2 + B2 (d) E/B in B is 10 cm below the hole in A. If
V
29. Potential difference (VC – VB) in the bullet is travelling horizontally Output
the circuit is: at the time it hits A, then the velocity I
1Ω 12A 2Ω 3V 4Ω
C B of the bullet at A is: V
1
5A 2A (a) 700 ms–1 (b) 300 ms–1
A B C D
(c) 200 ms –1
(d) 600 ms–1 I
6Ω 35. A long straight wire along
the z-axis carries a current in This contribution from the diode
3A 3A
the negative z-direction. The 1 is:
(a) 15 V (b) 38 V  (a) C (b) A, C
(c) 20 V (d) 9 V magnetic vector field B at a point
(c) B, D (d) A, B, C, D
having coordinate (x, y) on the z
30. The temperature which is same on 39. The radius of proton is 10–15 m. The
= 0 plane is:
Celsius and Fahrenheit scale is: radius of the observable universe
(a) (273 – 32)ºC µ 0 i(yiˆ − xj)
ˆ is 1027 m. Identify the distance
(a)
(b) 0ºC which is half way between these
2π(x 2 +y 2 )
(c) –40ºC two on a logarithmic scale:
(d) (273 + 32)ºC ˆ ˆ
µ 0 i(xi+yj)
(b) (a) 1021 m (b) 10–6 m
31. To an observer, the pitch of 2π(x 2 +y 2 ) (c) 10 m 6
(d) 100 m
a stationary source of sound 40. In the circuit shown in the figure,
appears reduced by 10%. If speed µ 0 i(xjˆ − yi)
ˆ the potential drop V, across the
(c)
of sound is 330 m/s, then the speed 2π(x 2 +y 2 ) resistor of 10 Ω is:
and direction of the observer will
ˆ ˆ E1 = 2 V
be: (d) µ 0 i(xi − yj) A B
r1 = 1 Ω
(a) 33 m/s away from source 2π(x +y 2 )
2

(b) 33 m/s towards the source 3Ω 4Ω


36. A toy rocket ejects 0.05 kg gas
(c) 66 m/s away from observer
per second with a speed of 400 D C
(d) 66 m/s towards the source
32. 207 J heat is required to raise the m/s. The accelerating force on the
3Ω 4Ω
temperature of 1 mole of gas at rocket is: E2 = 3 V
constant pressure through 10 K. (a) 20 dyne (b) 20 N E r2 = 1 Ω F
The energy required to raise the (c) 22 kg-wt (d) 4000 N
(a) 1.89 V (b) 1.79 V
temperature of same mass of gas 37. The number of particles is given
(c) 1.69 V (d) 1.99 V
through 10 K at constant volume by: 41. If to an observer, the apparent
is: (given, gas constant R = 8.3 J/
n2 − n1 frequency appears to be
mol K): n =− D
x2 − x1 5/4 times the actual frequency
(a) 108.7 J (b) 29 J
(c) 215.3 J (d) 124 J crossing a unit area perpendicular of the moving source. Then the
33. A body of 2 kg mass is pulled on to x-axis in unit time where n1 and speed and direction of the sound
a horizontal surface with 2 m/s n2 are number of particles per unit source is: (speed of sound in air
speed. If coefficient of friction is volume for the value of x meant
= 330 m/s)
Practice Set-11 131

(a) 66 m/s away from observer (a) A ∩ B (b) A ∩ B 54. If (a, b), (c, d) and (a – c, b – d)
(b) 66 m/s towards to observer (c) A ∩ B (d) A ∩ B are collinear, then which one of
(c) 33 m/s away from observer 47. If the cube roots of unity are 1, the following is correct?
(d) 33 m/s towards the observer ω, ω2, then the roots of equation (a) bc – ad = 0
42. Select the correct alternative (s): (x – 1)3 + 8 = 0 are: (b) ab – cd = 0
(a) A convex lens may form a real (a) –1, 1 + 2ω, 1 + 2ω2 (c) bc + ad = 0
image (b) –1, 1 –2ω, 1 – 2ω2 (d) ab + cd = 0
(b) R = 2f formula is applicable to (c) –1, –1, –1 55. The value of
only paraxial ray (d) None of these
1 + 1 + 1 + .....∞
(c) A convex lens becomes less 48. The root of the equation x2 – 8x 1.2.3 3.4.5 5.6.7
convergent when it is immersed + 16 = 0:
will be:
in water (a) are imaginary
(d) All of these (b) are distinct and real (a) log e 2 (b) log e 2 − 1
2
43. 16 cm3 of water flows per second (c) are equal and real
through a capillary tube of radius (d) cannot be determined (c) loge 2 (d) loge 4
‘a’ cm and of length l cm when 49. If n arithmetic means are inserted 56. The radius of the circle x2 + y2 + x
connected to a pressure head of between 20 and 80, such that the + c = 0 passing through the origin
h cm of water. If another tube ratio of first mean to the last mean is is:
of same length and radius a 1 : 3, then find the value of n.
(a)
1
(b) 1
2 (a) 12 (b) 13 4 2
cm is connected to the same (c) 11 (d) 14
pressure head; the quantity of 50. What is the value of (1 + i)5 + (c) 1 (d) 2
water flowing through the tube (1 – i)5, where i = −1 ? 2 − (log x )2 − 10 1
per second will be: (a) –8 (b) 8 57. If x log 3 x 3 = , then
2
(a) 1 cm3 (b) 4 cm3 x
(c) 8i (d) –8i
(c) 8 cm 3
(d) 16 cm3 x is equal to:
44. A metallic block is exposed to ax − xa (a) 3 (b) 9
51. If lim = −1 , then
electomagnetic waves. The waves x→ 0 x x − a a (c) 27 (d) 81
of which of the following will 58. 10 + 3(4 ) + 5 is divisible by
n n+2
(a) a = 1 (b) a = 0
penetrate the most? (c) a = e (d) 2 (n ∈ N).
(a) 1 Å (b) 1 nm dy
52. Find , if (a) 7 (b) 5
(c) 103 µm (d) 103 mm
dx (c) 9 (d) 17
45. The heat generated in a circuit
is given by H = I2Rt Joule where 
59. What is the order of differential
1− x
y = sin  2 tan −1
. d2 y  dy 
3
I is current, R is resistance and t  1 + x  equation, + = 0?
2  dx 
is time. If the percentage errors dx
in measuring I, R and t are 2%, (a) 1 (b) –1 (a) 1 (b) 2
1% and 1% respectively. The
x −x (c) 3 (d) 0
maximum error in measuring (c) (d)
2
x −1 1 − x2 60. T h e m e a n a n d s t a n d a r d
heat will be:
(a) 2% (b) 3% deviation of a group of 100
53. If the eccentricity of a conic
observations were found to be
(c) 4% (d) 6% x2
section is equal to , x ∈ 20 and 3, respectively. Later
MATHEMATICS x2 + 1 on it was found that three
R{0}, then conic section is: observations were incorrect,
46. If A = {x : x is a multiple of (a) a circle
3} and B = {x : x is a multiple which are recorded by 21,
(b) a parabola
of 5}, then A – B is (where, A 21 an d 18. Find the mean
(c) an ellipse
means complement of A) and standard deviation, if
(d) a hyperbola
132 Practice Set-11

the incorrect observation are dx India were nationalised in the


66. What is the value of π / 4
omitted. ∫− π /6 sin x cos x year
(a) 3.05 (b) 3.03 ?
(a) 1967 (b) 1968
(c) 3.09 (d) None of these (a) 2log 3 (b) log 3 (c) 1969 (d) 1971
x+1 (c) 2 log 3 (d) 4 log 3 74. What is the maximum gap
61. Evaluate ∫ dx 67. If angles of a triangle are in the permissible between two sessions
9 + 4 x2 of Parliament?
ratio 1 : 2 : 3, then what is the
(a) One month
(a) 1 9 − 4 x 2 + 1 sin −3  2 x  + C ratio of its corresponding sides?
4 2   3  (b) Three months
(a) 3 : 2 : 1 (b) 1: 2 : 3 (c) Six months
(b) 1 9 − 4 x 2 − 1 sin −1  2 x +C (c) 1: 3 : 2 (d) 2 : 3 : 4 (d) Twelve months
4 2 3 
68. If sin x + sin y = π/2 and cos–1
–1 –1 75. During the reign of Bindusara
there was unrest at
(c) − 1 9 − 4 x 2 + 1 sin −1  2 x + C x – cos–1 y = 0, then the values of
4 2 3  (a) Ujjayani
x and y are respectively.
(b) Pushkalavati
(d) − 1 9 − 4 x 2 − 1 sin −1  4 x + C 1 1
4 2 3  (a) ,− (b) 1 , 1 (c) Taxila
2 2 2 2 (d) Rajagriha
62. If a < 0, then the function (eax
76. Which component is mainly
+ e–ax) is a monotonic decreasing
1 1 responsible for doing calculation
function for all values of x, (c) 1 , − 1 (d) ,
where 2 2 2 2 in computer?
(a) x > 0 (b) x < 0 (a) Random access memory
69. Focus of the parabola 5x2 + 30x
(c) x > 1 (d) x < 1 (b) Control unit
+ 2y + 59 = 0 is:
(c) Arithmetic logic unit
63. If α = 1 + i 3 , then what is the (a) (3, 7) (b) (–3, –7)
2 (d) Hard disk
value of 1 + α8 + α16 + α26 + α32?  71 77. Which year is set by Government
(c)  −3, −  (d) None of these
(a) 0 (b) 1  10  of India as the target year to end
(c) ω (d) – ω2 Tuberculosis (TB)?
70. What is sin 2 20° + sin 2 70° is
64. In how many of the distinct (a) 2025 (b) 2030
equal to?
permutations of the letters in the (c) 2035 (d) 2020
(a) 1 (b) 0
MISSISSIPPI do the four I’s not 78. The sum of the digits of a 2-digit
come together? 1 number is 11. If we add 45 to
(c) –1 (d)
(a) 35810 (b) 33810 2 the number, the new number
(c) 33820 (d) None of these REASONING AND GENERAL obtained is a number formed by
65. The solution of the differential AWARENESS (RAGA) interchange of the digits. What is
dy y2 the number ?
equation = is given 71. Hydrochloric acid is also known
dx 1 − 3 xy (a) 83 (b) 38
by: as
2 (a) Galic acid (c) 64 (d) 46
(a) y3x = y + C 79. The ratio of two numbers is
2 (b) Picric acid
5 : 6 and their H.C.F. is 12. Their
2
(c) Muriatic acid
L.C.M. is:
(b) y3 = xy + C (d) Chloric acid
2 (a) 60 (b) 72
72. Kanchenzunga National Park is
located at (c) 180 (d) 360
(1 + 2Cy )
(c) x = (a) Uttar Pradesh 80. If a = 6 + x and b = 2x + 15, then
y3 for what value of x, a is twice of
(b) West Bengal
C (c) Sikkim b?
(d) x = (d) Jammu and Kashmir (a) 8 (b) – 8
y3
73. The fourteen major banks in (c) ½ (d) 2
Practice Set-11 133

81. The brother is elder to his sister (b) X is the sister of Z (ii) G is second to the left of C, who is
by 6 years. Seven years ago the (c) Y is the son of A to the immediate left of F.
product of their ages was 72. (d) B has one daughter (iii) A is third to the left of E.
What is the age of the brother ? 87. If MIND becomes KGLB and (iv) B is between D and E.
(a) 13 years (b) 12 years ARGUE becomes YPESC then 91. Which of the following is false?
(c) 6 years (d) 19 years what will DIAGRAM be in that (a) A is fourth to the right of E
82. In a triangle the length of the code?
(b) G is to the immediate right of D
side opposite the angle which (a) GLPEYKB (b) BGYEPYK
(c) F is third to the right of D
measures 30 degree is 9√3 cm, (c) LKBGYPK (d) BGYPYEK
what is the length of the side (d) B is to the immediate left of D
88. Akash leaves from his school
opposite to the angle which at 5:30 a.m. He starts walking Direction (92): In following questions
measures 60 degree? towards south and covers a you have to identify the responses
(a) 27 cm (b) 9 cm distance of 20 km. He turns left which would be a correct inference
and walks 15 km to reach point from the given premises stated
15 3 A. He turns right and walks 14 according to the following systems:
(c) 12√3 cm (d) cm km. He again turns right and
2 'A' stands for Not greater than.
walks 10 km. How far (in km)
83. The price of an article is cut by and in which direction is he now 'B' stands for Greater than
24%, to restore to its original from point A? 'C' stands for Not equal to
value, the new price must he
(a) 3 43 , South-West 'D' stands for Equal to
increased by ________
(b) 3 43 , South-East 'E' stands for not less than
(a) 24 percent 'F' stands for less than.
(b) 19.35 percent (c) 2 74 , South-West
92. Premises: (2x D y) and (3q A 3z)
(c) 30 percent (d) 2 74 , South-East (a) 2x D 6z (b) 2x E 3z
(d) 31.58 percent (c) 3z D 3y (d) 2x F 3z
89. In a row of girls, Kamya is fifth from
84. The ratio of present ages of 93. Select the missing number from
the left and Preeti is sixth from the
Rangitha and Shaheen is the given responses.
right. When they exchange their
6 : 5. After 12 years the ratio of
positions, then Kamya becomes
their ages will be 7 : 6. What is 3 15 5
thirteenth from the left. What will
Rangitha's present age?
be Preeti's position from the right? 5 35 7
(a) 60 (b) 55 (a) 7th (b) 14th
(c) 72 (d) 28 (c) 11th (d) 18th 9 ? 5

85. 4 hrs after a goods train passed a Direction (90): A series is given
station, another train travelling with one term missing. Choose the (a) 54 (b) 50
at a speed of 60 km/hr following alternative from the given ones that (c) 49 (d) 45
that goods train passed through will complete the series. 94. Arrange the given words in the
that station. If after passing the 90. ?, Vice President, Prime Minister, sequence in which they occur in
station the train overtakes the Deputy Prime Minister the dictionary.
goods train in 8 hours, what is (a) Chief Minister i. Cease
the speed of the goods train? (b) Governor ii. Consecutive
(a) 40 km/hr (b) 48 km/hr (c) Defense Minister iii. Constructive
(d) President iv. Cement
(c) 60 km/hr (d) 32 km/hr
Direction (91): Read the following (a) iii, i, iv, ii (b) i, iv, ii, iii
86. A is father of X; B is mother of Y. information carefully and answer the (c) i, iv, iii, ii (d) iv, i, ii, iii
The sister of X and Z is Y. Which questions given below:
of the following statements is Direction (95): The following questions
(i) A, B, C, D, E, F and G are sitting are based upon the alphabetical series
definitely not true. around a circle and are facing the given below:
(a) B is the mother of Z centre.
134 Practice Set-11

S L U A Y J V E I O N Q neighbour of C or H. E iso immediate (a) Only I (b) Only II


G Z B D R H right of A, who is second to the right (c) Only III (d) Both II & III
95. What will come in place of of G. 98. Who is second to the left of C?
question (?) mark in the 96. Who sits between G and D? (a) A (b) B
following series? (a) H (b) D (c) E (d) D
LA UJ YI EG ? (c) F (d) E 99. Which of the following pairs of
(a) ZH (b) IB persons has first person sitting to
97. Which of the following is the the right of the second person?
(c) NR (d) QH
correct position of B with respect (a) CB (b) AE
Directions (96-100): Study the
of H? (c) FG (d) DB
following information carefully and
answer these questions. I. Second to the right 100. Who is on the immediate right of
A, B, C, D, E, F, G and H are sitting II. Fourth to the right C?
around a circle facing the centre. F III. Fourth to the left (a) E (b) B
is third to the right of C and second IV. Second to the left (c) D (d) B or D
to the of H. D is not an immediate

Answers with Explanation


ENGLISH surface of When the object is below 20. (b) The synonym for 'bonafide'
the surface is things, it is used like a is genuine; bonafide means authentic or
1. (b) 2. (d) 3. (c) 4. (b) preposition. Such as under the table, genuincy presented or in good faith.
5. (b) Duly is correctly spelt, which under the bridge, under the tree..... etc.
means properly, fittingly, punctually 14. (d) The correct option is (d). He PHYSICS
6. (d) Vacuum is the correct spellings. explaimed that it was a very beautiful 21. (c) In vacuum, ε0 = 1
It means a space entirely devoid of scene. In medium, ε = 4
matter. 15. (c) The noun form of the verb ε
7. (a) The word ostracise (Verb) 'experience' is experience which means So,refractive index, µ=
means : to refuse to let somebody be a ε0
the process or fact of personally observing
members of a social group; to refuse to = 4
encountering, or undergoing something. =2
meet or talk to somebody; shun. 16. (c) Either _____ or. Neither 1
Look at the sentence ____nor are correct forms of correlatives. Wavelength, λ' =
λ λ
=
He was ostracised by his colleagues for Hence, or study for longer period.... μ 2
refusing to support the strike. Its antonym should be used here. and wave velocity,
should be welcome. Look at the sentence:
8. (b) The correct spelling is : Either Rames could not come or he v= c = c
exaggeration. did not want to. I am going to buy either μ 2
9. (d) The correct sentence is – You, camera or a DVA with the money. Hence it is clear the wavelength and
Tom and I ate sweets. The use of first, 17. (c) Look at the following words: velocity become half but frequency
second and third person pronouns always Effect + (v) = to make something remains unchanged when the wave is
follows the order of – second, third then happen : to effect a cure/ change/recovery passing through any medium.
first i.e. 231. Affect (v) = to produce a change in 22. (a) Lt= L0 (1 + αt)
10. (d) An assertive sentence is somebody/something : How will these Lt – L0 = L0 α t.
changed into an interrogative sentence changes affect us? Your opinion will not
affect my decision. Hence, the are bound Lt − L 0
by putting the auxiliary verb (did) = strain = αt
before the subject (you). Did you have a to effect should be replaced by are bound L0
comfortable journey? Hence option (d). to affect....
18. (d) The garbage was first sundried = 12 × 10–6 × 20
11. (b) Material means consisting of = 24 × 10–5
matter, or worldly as opposed to spiritual. for one to three days to bring down the
moisture level. Elastic Potential energy per unit
12. (c) Honour and shame refers to volume
cause and effect. 19. (b)Women like to be flattered by
13. (d) "under" means below the men.
Practice Set-11 135

1 Situation B – E = 0 i.e., no electrostatic x x − 32


= × stress × strain force, B ≠ 0, but the charge particle =
2 5 9
enters parallel to the field, so that net
= 1 × Y × strain × strain force equals to zero. or, 9x = 5x – 160
2 Situation C – E ≠ 0, i.e., the charged x = – 40
particle proton must experience an 31. (a) Actual frequency = n
1
= × Y × (strain)2 electrostatic force eE and hence must
2 accelerate. 90
Apparent frequency n' = n
1 Situation D – E ≠ 0, B ≠ 0 and both 100
= × 1011 × 24 × 10–5 × 24 × 10–5 shown in figure:
2 Since, n' > n
E so, the source must be moving away
= 12 × 24 × 10–10 × 1011 ×
× × × × × from the observer.
= 288 × 10 = 2880 joule/m3 P
v − v0
23. (b)The negative slope of V-r graph × × × × × n' = n
gives electric field. × v
B
∆V 90 330 − v0
E= − Because in such a situation =
∆r 100 330
Fm= evB (upwards)
4 andFe= eE (downwards)
At R = 5 cm, tan θ = − = −2
2 and if both are equal in magnitude 330 × 90
or, = 330 – v0
∆V then even the proton will suffer no change 100
E= − = – tan θ
∆r in its velocity and will continue to move
along the dotted line as shown in figure. ∴ v0 = 330 – 297
= 2 V/cm
= 33 m/s away from source
24. (d) Magnetic flux in ⊗ direction In such a situation, the velocity is v = E .
passing through the coil is increasing. B 32. (d)At constant pressure, energy
27. (a)In steady state.
Therefore, induced current will produce required
magnetic field in  direction. Thus, F = qE = CpdT = 207 J
the current in the loop is anticlockwise. At constant volume, energy required
Magnitude of induced current at any
instant of time is: = CvdT
Since, Cp – Cv= R,
e Bv(FG) Therefore, CpdT – CvdT= RdT
I= =
R ρ(FG + GD + DF) CvdT= CpdT – RdT
When the wire AH moves downwards mg = 207 – 8.3 × 10
FG, GD and DF all increase in the same = 124 J
electric force on drop = weight of drop
ratio. Therefore, I is constant. 33. (a) F= µR = µmg
∴qE = mg
= 0.2 × 2 × 9.8
25. (a) u= mgH ⇒ q = mg = 3.92 N
E Heat generated per second
 u2 sin 2 θ 
⇒ u= mg   = work done per second
 2g  9.9 × 10−15 × 10
=
3 × 104 =F×VJ
⇒ u= 1 mu2 sin 2 θ ∴ Heat generated in 5 sec.
= 3.3 × 10–18 C
2
28. (a)Energy flow per unit area per 3.92 × 2 × 5
26. (A, B, D) No change in velocity  = cal
unit time is called Poynting vector S = 4.2
implies no acceleration i.e., no net force  
is acting on the proton, even under the E×B = 9.33 cal
joint influence of electric and magnetic 29. (c) VC – VB = 5 × 1 – 12 + 8 × 2 gx 2
+3+4×2 34. (a) Since,y =
field. This thing is possible under the 2u 2
following situations: = –12 + 32 = 20 V Here, y= 10 cm
Situation A – E = 0, B = 0, i.e., no 30. (c) Let temperature be x
field exists in the region. 10
= m, x = 100, m, u = ?
100
136 Practice Set-11

and g = 9.8 ms–2 39. (c) Herex= 10–5 m and log x = –15 ∴ vs = 330 – 264
⇒u = 700 m/s y = 1027 m and log y = 27 = 66 m/s towards observer

35. (a) Magnetic field P is B , 42. (d)(a)A convex lens can from both
log z = log x + log y
perpendicular to OP in the direction z2 type of images (real and virtual)
shown in figure. (b) R = 2f formula is applicable to
y −15 + 27 only mirror for paraxial ray.
= =6
z2 (c) When a convex lens is immersed in
P(x, y) ∴z = 106 m.
water, its focal length increases. Hence,
θ 40. (b) In loop ABCDA
r it becomes less convergent.
3I1 + 10 (I1 + I2) + 4I1 + I1= 2
i θ 4
x ⇒ 9I1 + 5I2= 1 ...(i) 43. (a) Q1= πpa
O In loop DCFED 8 ηl
10(I1 + I2) + 4I2 + I2 + 3I2= 3
4
⇒ 10I1 + 18I2= 3 ...(ii) = πhdga
Solving Eqns. (i) and (ii), we get 8 ηl
4
3 In first case 16 = πhdga
 I1 = A, I2 = 17 A 8 ηl
So, B = B sin θ iˆ − B cos θ ˆj 112 112
and current through 10 Ω resistance, πhdg (a / 2) 4
µ i In second caseQ =
Here, B= 0 5 8 ηl
2 πr I = I1 + I2 =
28 πhdga 4 1
y x = .
sin θ = and cos θ = 5 8 ηl 16
r r Potential drop = 10 × = 1.79 V
28
16
 µ 0 i 1 2V 1Ω = = 1 cm3
∴B= 2
( yiˆ − xjˆ ) A B
16
2π r I1 44. (a) Penetration power is inversely
ˆ ˆ 3Ω 4Ω proportional to wavelength.
= µ 0 i ( yi − xj ) (as r2 = x2 + y2)
2π (x + y 2 )
2 2∆I ∆R ∆t
I1 + I2 45. (d) ∆H = + +
D C H I R t
36. (b)Force= Rate of change of
momentum ⇒ ∆H × 100 = 2 (2%) + 1% + 1%
= Rate of change of mass x v 3Ω 4Ω H
= 0.05 × 200 = 20 N
∆H
E F ⇒ × 100 = 6%
Particles 1 I2 H
37. (d) n= = 2 3V 1Ω
Area × Time L T
41. (b) Since to stationary observer, MATHEMATICS
 n2 − n1  the frequency appears more than the
n = −D  46. (b) A = {3, 6, 9, 12, 15,.....},
 x2 − x1  actual frequency, therefore, the source B = {5, 10, 15, 20,.....}
must be approaching the observer. ∴A – B = {3, 6, 9, 12, 18,......}
 1 1
= D× 3 ×  5 B = {1, 2, 3, 4, 6, 7, 8, 9, 11,.......}
L L Given n' = n,
4 ∴A ∩ B = {3, 6, 9, 12,......}
or,D = [L2T–1] v = 330 m/s 47. (b) We have,
38. (c)In first half cycle junction diode (x – 1)3 + 8 = 0 ⇒ (x – 1)3 = – 8
2 is forward biased and 1 reverse biased; v ∴ x – 1= (–8)1/3 = – 2(1)1/3
∴ n' = n
while in next half cycle junction diode 1 v − vs = –2, –2ω, –2ω2
is forward biased and 2 is reverse biased. Hence, x = –1, 1, –2ω, 1 – 2ω2
So, contributions from diode 1 are B, D. 5 330 48. (c) Given, equation is
or, n = n
4 330 − vs x2 – 8x + 16 = 0
Practice Set-11 137

⇒ (x – 4)2= 0
a a log e a − a × a n −1 log e a − 1  1 1 1 1
⇒ x = 4, 4 = = = 1 –  −  –  −  –.......
a a
a + a log e a log e a + 1  2 3  4 5 
Also, discriminate ∆ is b2 – 4ac = 0
So, the roots of the equation are equal Satisfied only when a = 1.
and real. ⇒ loge 2 = 1 – 1 − 1 − ...... ....(iii)
 1− x  2.3 4.5
49. (c) Let, A1, A2, A3.......An be n 52. (d) y = sin  2 tan −1
arithmetic mean between 20 and 80 and  1 + x  On adding Eq. (i) and (ii), we get
2.loge 2
Let d be the common difference of the On putting x = cos 2θ, we get
AP, 20. A1, A2.........An 80. Then:  1 1   1 1 
  =1+  − + − + .....
y = sin  2 tan −1 1 − cos 2θ   1.2 2.3   3.4 4.5 

80 − 20 1 + cos 2θ 
d= 
n +1 2 2
 2sin 2 θ  2.loge 2 – 1 = + + ..... ....(iv)
60  b − a = sin  2 tan −1  1.2.3 3.4.5
=  Using d =   2cos 2 θ 
n +1 n + 1 1
⇒ 1 + 1 + ....... = loge 2 –
= sin [2 tan–1 (tan θ)] = sin 2θ 1.2.3 3.4.5 2
Now, A1 = 20 + d ⇒ A1 = 20 + 60n
n +1 = 1 − cos 2 2θ 56. (b) Given equation of circle is
 n + 4 ⇒ y = 1 − x 2 = (1 – x2)1/2 x2 + y2 + x + c = 0 ....(i)
= 20 
 n + 1  Since, the equation of circle passes
dy 1 through the origin.
and An = 20 + nd ∴ = (1 − x 2 ) −1/2 . d (1 − x 2 )
dx 2 dx ∴ (0)2 + (0)2 + 0 + c = 0 ⇒ c = 0
⇒ An = 20 +9 60n 1 −x
From Eq. (i),
n +1 = . (0 – 2x) = x2 + y2 + x= 0
2 1− x 2 1 − x2 1 1
4n + 1
= 20  ⇒x2 + y2 + x + =
 n + 1  x2 4 4
53. (c) Given, e =
20(n + 4) x2 + 1 1
2 2
  1
A 1 1 ⇒  x +  + (y – 0)2 =  
∴ 1
= ⇒ n +1 = Here, we see that for every real  2 2
An 3 20(4n + 1) 3 number x, e is less than 1. Thus, the conic
n +1 section is an ellipse. So, the required radius of circle is 1 .
54. (a) (a, b), (c, d) and {(a – c), (b 2
1 –d)} are collinear.
⇒ n + 4 = ⇒ 4n + 1 = 3n + 12 57. (b)Given equation can be written
4n + 1 3 ∴ a(d – b + d) + c (b – d – b) as
+ (a – c) (b – d)= 0 2 log3 x + (log3 x)2 – 10 = – 2
∴ n = 11
⇒ 2ad – ab – cd + ab – ad – bc + cd ⇒ y2 + 2y – 8 = 0, where y = log3 x
50. (d) Given (1 + i)5 + (1 – i)5
=0 ⇒ (y – 2) (y + 4) = 0 ⇒ y= 2 or – 4
= (5C0 + 5C1i + 5C2i2 + 5C3i3 + ⇒ ad – bc= 0 ⇒ log3 x = 2 ⇒ log3 x= – 4
C4i4 +5C5i5) – (5C0 – 5C1i + 5C2 i2
5 ⇒ bc – ad= 0 ⇒ x = 32 ⇒ x= 34
– 5C3i3 + 5C4i4 – 5C5i5) 55. (b) We know that
1
(By Binomial theorem) loge 2 ∴x=9 ⇒ x =
= 2(5C1i + 5C3i3 + 5C5i5) 81
= 2 {(5i – 10i + i)} = 1 − 1  +  1 − 1  +  1 − 1  +... ....(i) 58. (c) Let P (n) = 10n + 3(4n + 2) + 5
= –2(– 4i) = –8i  2 3 4 5 6 On putting n = 2
51. (a)Using L’ Hospital rule, we get then P(1) = 102 + 3 × 44 + 5
1 1 1 = 100 + 768 + 5 = 873
a x − xa ⇒ loge 2 = + + + ...... ....(ii)
lim 1.2 3.4 5.6 which is divisible by 9.
x→ a x x − aa 59. (b) Here, highest order of
Again, loge 2
x
a log e a − ax n −1 d2y
= lim derivative is .
x→ a x x + x x log e x dx 2
138 Practice Set-11

Hence order of the given differential 1 ydy 1 If all the I’s are together, then it will
4 ∫ y2
equation is 2. I1= − = − ∫ dy be considered as one letter and remaining
4
60. (b)Given, n = 100, x = 20, σ = 3 7 letters and 1 I’s letter (including 4 I’s )
will be considered as 8 letters. So, the
 x = 20 1
= − y + C1 = − 1 9 − 4 x 2 + C1 8!
4 4 number of permutations is .
⇒ ∑ x = 20 4!2!
100 dx
I2 = ∫ = 1 sin −1  2 x + C2 Hence, total number of arrangements.
⇒ Σxi= 100 × 20 9 − 4 x2 2 3 
= 11! – 8!
⇒ Σxi= 2000 1 1 2  4!4!2! 4!2!
∴I = – 9 − 4 x 2 + sin −1  x + C
Now, incorrect observations 21, 21 4 2 3 
and 18 are omitted, then correct sum is = 11 × 10 × 9 × 8 × 7 × 6 × 5 × 4!
Where C = C1 + C2 is another arbitrary 4!× 4 × 3 × 2 × 1 × 2 × 1
Σ xi = 2000 – 21 – 21 – 18 constant.
= 2000 – 60 = 1940 dy – 8 × 7 × 6 × 5 × 4!
62. (b) = a(eax – e–ax) = negative 4!× 2 × 1
Now, correct mean of remaining 97 dx
(given) = (34650 – 840) = 33810
1940
observations is x = = 20 Since, a is negative. y2
97
1 65. (a) We have dy =
∴eax – = Positive dx 1 − 3 xy
∑ xi2 − ( x)2 e ax
Again, σ = 3 ⇒ =3
dx 1 − 3xy dx 1 3
n Again e > 1, so, eax – 1 is positive, ⇒ = 2
⇒ = − x
e ax dy y dy y2 y
⇒ ∑ xi2 – (20)2 = 9
if ax is positive
100 ⇒ x is negative i.e., x < 0 as a is
negative. ∴ dx + 3 x = 1 ....(i)
dy y y2

∑ x2 = 9 + 400 63. (d) Here, α = 1 + i 3 = – ω2
100 2 The above is a linear different equation
of the form
⇒ Σx2 = 409 × 100 = 40900
(We know that, 1 + i 3 = – ω2 )
Now, connect Σx2 is Σx2 2 dx
+ P( y ) x = Q(y)
= 40900 – (21)2 – (21)2 – (18)2 dy
= 40900 – 441 – 441 – 324 ∴ 1 + α8 + α16 + α24 + α32
= (40900 – 1206) = 39694 = 1 + (– ω2)8 + (– ω2)16 + (– ω2)24 3 1
+ (– ω2)32 Here, P = and Q = 2
Now, correct SD for remaining 97 y y
observations is =1+ω +ω +ω +ω
16 32 48 64

2
= 1 + ω.(ω3)5 + ω2 + (ω3)10 . (ω3)48 3
σ=
39694  1940  + ω.(ω3)21 ∴ IF = ∫ y dy = e 3log y = y3
− e
97  97  =1+ω+ω +1+ω 2

( ω3 = 1)( ω3 = 1) The solution of Eq. (i) is given by


= 4092 − (20) 2 = – ω + ω + 1 + ω( 1 + ω = – ω2)
2 2
1 3
= 1 + ω = – ω2 xy3 = ∫ y 2 .y dy + C
= 4092 − 400 = 9.2 = 3.03 64. (b)The word MISSISIPPI has 11
letters in which 2
61. (c)Let
M → 1 time ∴xy3 = y + C
xdx dx I → 4 times 2
I= ∫ 2
+ ∫ = I1 + I2
S → 4 times
9 − 4x 9 − 4 x2 π /4 dx
66. (b)
xdx
P → 2 times
The number of permutations of the
∫π /6 sin x cos x
Now, I= ∫ word MISSISSIPPI in which 4 I’s and
9 − 4 x2 π /4 dx
4S’s are alike π /4
1 = 2∫π /6 = 2∫ cosec 2xdx
Put 9 – 4x = y ⇒ xdx = −   y dy
2 2
11!
sin 2 x π /6
 4 = ....(i)
4!4!2!
Practice Set-11 139

74. (c) The Constitution empowers


= 2 [log tan x]ππ /4
/6 ⇒ x= 1 the President to summon each House
2
at such intervals that there should not
. 1 = log tan π − log tan π  From eq. (ii) y= 1 be more than 6-month’s gap between
2  4 6  2 the two sessions. Hence the Parliament
69. (c) Given equation of parabola must meet at least twice a year. In India,
1 the parliament conducts three sessions
= 0 – log − log 3 can be rewritten as:
3 each year.
67. (c)Since, angles of a triangle are (x + 3)2 = − 2 (y + 7) ⇒ X2 = 4AY 75. (c) During the reign of
in ratio 1 : 2 : 3. 5 Bindusara, Chandragupta Maurya’s
∆ABC= 180° son and successor, there was unrest at
4A = − 2 ∴ A = – 1 Taxila in the north-western province
∴x + 2x + 3x = 180° ⇒ x = 180° 5 10 of Sindh. He sent Asoka (his son) to
= 30° 6 quell the uprising. Taxila was a highly
Focus is X = 0, Y = A
So, the angles are 30°, 60° and 90°. ⇒ x + 3 = 0, y + 7 volatile place because of the Indo-
sin A Greek presence and mismanagement of
=K 1  71 Governor Susima.
a = − ∴  −3, − 
10  10  76. (c) An arithmetic-logic unit
sin 30° (ALU) is the part of a computer
⇒ =K 70. (a) Given sin2 20° + sin2 70°
a processor (CPU) that carries out
= sin2 20° + sin2 (90° – 20°) arithmetic and logic operations on the
1 sin B sin 60° operands in computer instruction words.
∴a = =K ⇒ =K = sin2 20° + cos2 20° = 1
2K b b It performs all arithmetic computations,
( sin2 θ + cos2 θ = 1)
such as addition and multiplication, and
∴b = 3 and sin C = K all comparison operations.
c
REASONING AND GENER-
2K 77. (a) 2025 Explanation:
AL AWARENESS (RAGA)
sin 90° 1 On June 24, 2019, Ashwini Kumar
C= = 71. (c) Hydrochloric acid was Choubey, Health and Family Welfare
K K historically called acidum salis, Minister stated that the Government
muriatic acid, and spirits of salt has developed an ambitious National
1 3 1 because it was produced from rock
Ratio in sides = : : Strategic Plan to end Tuberculosis (TB)
2K 2K K salt and green vitriol and later from by 2025. As per the Global TB report,
= 1: 3 : 2 the chemically similar substances India accounts for 27.4 lakh new TB
common salt and sulfuric acid. cases annually which are 204 per lakh
68. (d) Given, Hydrochloric acid is found naturally population for the year 2017
in gastric acid. 78. (b) Let the number be 10 x + y
π
sin–1 x + sin–1 y= ....(i) 72. (c) Kanchenjunga National
2 In case I
Park is located in Sikkim. The park
and cos–1 x – cos–1 y = 0 gets its name from the mountain x + y = 11 ...(i)
Kanchenjunga which is 8,586 metres 10x + y + 45 = 10y + x
π  π −1 
⇒  − sin −1 x –  − sin y  = 0 tall, the third-highest peak in the In case II
2   2 
world. The park is known for animals 10x + y – 10y – x = – 45
⇒ sin–1 y – sin–1 x= 0 like musk deer, snow leopard and
Himalayan Tahr. 9 (x – y) = – 45
⇒ sin–1 y= sin–1 x....(ii)
73. (c) In 1969, 14 major private x – y = – 5 ...(ii)
From Eq. (i) and (ii), we get
commercial banks were nationalised. Solving equation (i) and (ii), we get
x = 3 and y = 8
2sin–1 x= π This was followed by a second
2 phase of nationalisation in 1980, Hence, the number
when Government of India acquired = 30 + 8 = 38.
π the ownership of 6 more banks, 79. (d) Let the numbers be 5x and
⇒ sin–1 x= thus bringing the total number of
4 6x. Then, their H.C.F. = x. So, x = 12,
nationalised Banks to 20.
140 Practice Set-11

So, the numbers are 60 and 72. Required increased % BGYEPYK


⇒ Number 1 × Number 2 = LCM 24 88. (c)
× HCF x
100 × 100% 2400 600
⇒ 60 × 72 = 12 × LCM = = = A O Start
76 76 19
⇒ LCM ⇒ 360 x
100 20 km
80. (b) 2(2x + 15) = 6 + x 15 km
= 31.58% B C
4x + 30 = 6 + x
84. (c) Let present ages of Rangitha
3x = 24 14 km
and Shaheen be 6x and 5x.
x=–8 According to question, E D
10 km
81. (d) Let age of her sister = x
Brother’s age = x + 6 6 x + 12 7 In ∆ CDE,
=
7 years ego, 5 x + 12 6 EC2 = DE2 + DC2 = (14)2 + (10)2
(x – 7) (x + 6 – 7) = 72 36x + 72 = 35x + 84
= 196 + 100
⇒ (x – 7) (x – 1) = 72 36x – 35x = 84 – 72 = 296
⇒ x2 – 8x + 7 = 72 x = 12 CE = 296
⇒ x2 – 8x + 7 – 72 = 0 Rangitha’s present age = 6 × 12 =
⇒ x2 – 8x – 65 = 0 72 years. = 2 74
x – 13x + 5x – 65 = 0
2
85. (a) Let speed of the goods train Finally Akash is 2 74 km in
x (x – 13) + 5 (x – 13) = 0 be v. south-west direction.
(x – 13) (x + 5) = 0 Total time taken by goods train 89. (b) Kamya's new position is
x = 13, or x + 5 = 0 = (4 + 8) = 12 hr 13th from left. But it is the same as
Distance covered by goods train Preeti's earlier position which is 6th
x = – 5 (Invalid)
= Distance covered by another train from the right.
Brother’s Age = 13 + 6 = 19 years.
12v = 8 × 60 The row consists of (12 + 1 + 5) =
AC 480 18 girls
82. (a) tan 30º = v= = 40 km/hr.
BC 12 Now, Preeti's new position is
Kamya's earlier position which is 5th
A 86. (c) A  father of X.
from the left.
Y  sister of X. ∴ Preeti's new position from the right:
60º So, ‘Y is the daughter of A’. = 18 – 5 + 1
9√3 Therefore, option (c) is definitely = 13 + 1 = 14th
not true. 90. (d) The series is:
30º 90º
B C 87. (b) As, President, Vice President, Prime
Minister, Deputy Prime Minister.
9 3 1 9 3 Finally, President is the missing
tan 30º = ⇒ =
BC 3 BC term.
91. (c) By analysing the above
BC = 9 3 × 3 = 9 × 3 circular arrangement, it is a clear that
and
BC = 27 cm F  Fourth to the right of D.
83. (d) Let the cost price of an Therefore, option (c) is false.
article be x 92. (d) According to the given
After 24% discount information
100 − 24 76 2x Dy ⇒ 2x = y ...(i)
= x× = x Similarly, and 2y A 3z ⇒ 2y >| 3z...(ii)
100 100
From (ii), we get
Increased price of an article 2y  3z  ...(iii)
76 24 From (i) and (iii), it is clear that
= x− x= x
2x < 3z ⇒ 2x F 3z.
100 100
Practice Set-11 141

93. (d) First Row ⇒ 3 × 5 = 15 96. (c) By analysing the above 99. (d) By analysing the above
Second Row ⇒ 5 × 7 = 35 circular arrangement, it is clear that circular arrangement, it is clear that
Third Row ⇒ 9 × 5 = 45 F
Sitting
between G and D.
Sitting
C to the immediate left of
94. (b) According to dictionary, the 97. (d) By analysing the above (first person)  (second person)
arrangement of words is: circular arrangement, it is clear that Sitting
A to the immediate left of E.
(i) Cease Sitting
B ⇒ Fourth to the right of H and F to the immediate left of G.
(iv) Cement Sitting
Fourth to the left of H. H to the immediate left of A.
(ii) Consecutive
Sitting
(iii) Constructive Therefore, both II & III are correct D to the immediate right of B.
95. (d) 98. (a) By analysing the above 100. (b) By analysing the above
L +1 U +2 Y +3 E +4
Q
circular arrangement, it is clear that circular arrangement, it is clear that
A ⇒ Second to the left of C. B ⇒ to the immediate right of C.
A +2 J
+3 I +4
G
+5
H
qqq
142 Practice Set-12

Air Force Airmen Group X & Y Exam


Practice Set-12

so long as war is at all probable, because (a) She said to me that I had not
ENGLISH if one side is equipped with scientists and broken my promise.
Directions (1-4): Read the passage the other not, scientific side would almost
given below and answer the questions certainly win. (b) She told me that I must not break
that follow by selecting the most 1. What determines whether science my promise
appropriate option. is good or bad? (c) She warned me that I should keep
Science, insofar as it consists of (a) Its analytical study my promise
knowledge, must be regarded as having
value but in so far as it consists of (b) Its philosophical conception (d) She said to me that I should not
technique, the question whether it is (c) The manner of its origin break promise
to be praised or blamed depends upon (d) Its application in life 7. He said, "I clean my teeth twice a
the use that is made of the technique. day."
2. Which conception of science
In itself it is neutral, neither good nor
bad. Any ultimate view that we may has the author mentioned in the (a) He said that he cheaned his teeth
have about what gives value to this or passage? twice a day
that must come from other sources than (a) Its being good and bad (b) He said that he cleans his teeth
science. Both good and evil, almost
(b) Its being as purely empirical twice a day
everything that distinguishes our age
from its predecessors, is due to science. enquiry (c) He said that he used to clean his
In daily life we have electricity, the radio (c) Science as knowledge and science teeth twice a day
and cinema. Because of the increased as technique (d) He said that he is used to cleaning
productivity of labour, we are able to
(d) Its being both a philosophical and his teeth twice a day.
devote a far greater proportion of our
energies to wars and preparation for empirical study 8. What is the meaning of pertinent?
wars than was formerly possible and 3. What distinguishes our age from (a) Narrow minded
we are able to keep the young in school (b) Relevance
the previous ages?
very much longer than we formerly (c) Curious
could. Owing to science we are able (a) The prevalence of liberal ideas (d) Upside down
to disseminate information through (b) Lofty conception of way 9. Punctuate the following sentence:
the press and the radio to practically (c) Science I went up to him and held out my
everybody. Owing to science, we can hand how are you I asked I am not
(d) Both (a) and (b)
make it more enormously difficult than in hilarious spirits he answered."
it used to be for people whom the 4. What enables us to devote greater
Find out the correct one:
government dislikes, to escape. proportion of our energies to wars (a) I went up to him and held out
The opposition to science in past was and preparation for wars today my hand how are you I asked.
by no means surprising. Men of science than in the past? I am not in hilarious spirits, He
affirmed things that were contrary to answered.
what everybody had believed, they upset (a) Increased productivity of labour
(b) I went up to him and held out
preconceived ideas and were thought to (b) Deep nationalism my hand, how are you I asked.
be destitute for reverence. (c) Man's increasing bellicosity I am not in hilarious spirits, He
It would not be surprising if in the (d) None of the above answerd.
present day, a powerful antiscientific (c) I went up to him and held out
movement were at arise as a result of the 5. Which word is mis-spelt?
my hand, How are you I asked,
dangers to human life that are resulting (a) Zealous (b) Opposite I am not in hilarious spirits, he
from atom bombs and may result from (c) Empirical (d) Rhytoric answered
bacteriological warfare. But whatever (d) I went up to him and held out my
people may feel about these horrors, they 6. The Indirect narration of – She
hand. "How are you?" I asked "I
dare not turn against the men of science said, "You must not break your am not in hilarious spirits." he
promise". is: answered.
Practice Set-12 143

10. In which of the sentences 'above' 16. Read the passage and find out 20. As the bomb exploded people ran
is used as adverb? which of the four combinations helter-skelter.
(a) The moral law is above the civil is correct. Then find the correct (a) in great fear
(b) Analyse the above sentence answers. (b) in disorderly haste
(c) Our blessing come from above 1. A foolish wolf saw a horse (c) in haste
(d) The heavents are above grazing in a field and decided (d) in great sorrow
to eat him.
11. Which of the following words
P. The wolf went the stood behind PHYSICS
is the "Antonym" of the word
‘Auspicious’? the horse wondering if the 21. An open knife edge of mass 200
horse's tail was the passport. g is dropped from height 5 m on
(a) Oracular (b) Optical
Just then the received such a a cardboard. If the knife edge
(c) Ominous (d) Austere
kick on his face that two teeth
12. Rising price will force people to penetrates distance 2 m into the
shot out of his mouth.
______ expenditure. cardboard, the average resistance
(a) cut up (b) cut down Q. If you come and stand behind
me, I will show you what a offered by the cardboard to the
(c) cut off (d) cut across
passport is. knife edge is:
13. Fill in the blank with correct
preposition. R. "What is the passport?" (a) 7 N (b) 25 N
I bought this pen _______ five asked to the wolf. (c) 35 N (d) None of these
rupees. S. When the horse saw the wolf 22. A block of mass m moving at a
(a) in (b) with it said "you cannot eat me. I velocity v collides with another
(c) for (d) from have a passport." block of mass 2m at rest. The
Directions (14-15): In the following 6. As the wolf lay dazed the horse lighter block comes to rest after
questions the sentences have been given made good his escape. collision. Find the coefficient of
in Active/Passive voice. From the given (a) PSQR (b) SRQP restitution.
alternatives, choose the one which best
(c) RQSP (d) PQRS (a) 1/2 (b) 1
expresses the given sentence in Passive/
Directions (17): Choose the word (c) 1/3 (d) 1/4
Active Voice.
opposite in meaning to the given word
14. I don't like people keeping me 23. A body rolls down an inclined
and mark it in the answer sheet.
waiting. plane. If its kinetic energy of
17. VALIDATE
(a) I don't like people are kept me rotation is 40% of its kinetic
(a) disprove (b) authenticate
waiting. energy of translation, then the
(c) ingenuine (d) legalise
(b) I don't like people were kept me body is:
waiting. Directions (18): O u t o f t h e f o u r
alternatives, choose the one which (a) solid cylinder
(c) I don't like being kept waiting.
best expresses the meaning of the given (b) solid sphere
(d) I don't like people are being kept
waiting. word. (c) disc
15. Has a dog ever bitten you? 18. BENEVOLENT (d) ring
(a) Has a dog over bites you? (a) friendly (b) charitable 24. If an average person jogs, he
(b) Have you ever been bitten by a (c) affectionate(d) helpful produces 14.5 × 103 cal/min. This
dog? 19. The opposite of proportionate is – is removed by the evaporated per
(c) You are bitten by a dog. (a) misproportionate minute (assuming 1 kg requires
(d) Have you ever being bitten by a (b) disproportionate 580 × 103 cal for evaporation) is:
dog? (c) aproportionate
Directions (16): I n t h e f o l l o w i n g (a) 0.25 kg (b) 2.25 kg
(d) unproportionate (c) 0.05 kg (d) 0.20 kg
question, the first and the last part
Directions (20): Four alternatives are
of the passage are numbered 1 and 6. 25. Which graph represent the
given for the idiom/phrase italicised
The rest of the passage is split into four variation of surface tension
in the sentence. Choose the alternative
parts and named P, Q, R and S. These with temperature over small
which best expresses the meaning of
four parts are not given in their proper temperature ranges for water?
the idiom/phrase.
order.
144 Practice Set-12
Surface tension 28. When the current changes from + (a) F1 = F2
2A to – 2A in 0.05 s, an emf of 8 V (b) F1 > F2
is induced in a coil. The coefficient (c) F1 < F2
(a)
of self-induction of the coil is: (d) None of these
(a) 0.1 H (b) 0.2 H 33. The electric field due to a charge
Temperature (c) 0.4 H (d) 0.8 H at a distance of 3 m from it is
29. On required 11 eV of energy to 500 NC–1. The magnitude of the
Surface tension

dissociate a carbon monoxide charge is:


molecule into carbon and
 1 
(b) oxygen atoms. The minmum  = 9 × 109 Nm 2 C –2 
frequency of the appropriate  4πε 0 

Temperature electromagnetic radiation to (a) 2.5 µC (b) 1.0 µC


achieve the dissociation lies in: (c) 2.0 µC (d) 0.5 µC
Surface tension

(a) visible region 34. The resistance Rt of a conductor


(b) infrared region varies with temperature t as
(c) (c) ultraviolet region shown in figure. If the variation
(d) microwave region is represented by Rt = R0 (1 + αt
30. If the magnet is suspended at + βt2). then
Temperature
an angle 30º to the magnetic
Surface tension

meridian, the dip needle makes an


angle of 60º with the horizontal. R
(d) What is the true dip?
3
(a) tan–1 (3) (b) tan –1  
Temperature 2 t
2 1
26. A tunnel is bored along the (c) tan  
–1 –1
(d) tan   (a) α and β both negative
diameter of the Earth and a stone 3 3 (b) α is positive and β is negative
is dropped into it. What happens 31. A particle starting from the origin (c) α and β both are positive
to the stone? (0, 0) moves in a straight line in (d) α is negative and β is negative
(a) It oscillates between the two the (x, y) plane. The coordinates 35. When a proton is released from
ends of the tunnel rest in a room, it starts with an
at a later time are ( 3, 3) . The
(b) It comes to rest at the centre of initial acceleration towards West.
the Earth path of the particle markes with
When it is pojected towards
(c) It will go out of the other end of the x-axis angle of. North with a speed vo. It moves
the tunnel (a) 30º (b) 45º with an initial acceleration 3a0
(d) It will come to a permanent stop (c) 60º (d) 0º towards West. The electric and
at the other end of the tunnel 32. Let F1 is the gravitational force magnetic fields in the room are:
27. What is the total electric flux experienced by a particle at ma0 2ma 0
leaving a spherical surface of (a) West, up
3 –1 e ev0
radius 1 cm and surrounding at a height R e above the
electric dipole? 2 2 ma0 2ma 0
(b) West, down
Earth’s surface and F 2 is the e ev0
q
(a) (b) Zero gravity force experienced by a
ε0 ma0 3ma 0
3 –1 (c) Eest, up
particle at a depth Re e ev0
2q 8πr 2 q 2 2
(c) (d) ma0 3ma 0
ε0 ε0 below the Earth’s surface, then (d) Eest, down
(Re is radius of Earth) e ev0
Practice Set-12 145

36. For the study of the helical 39. The correct relation for the 43. Parsec is the unit of:
structure of nucleic acids, the cohesive energy is: (a) time
property of electromagnetic (a) Cohesive energy = free energy
(b) distance
radiation, generally used is: of atom – crystal energy
(c) frequency
(a) reflection (b) interference (b) Cohesive energy = free energy
(c) diffraction (d) polarisation of atoms + crystal energy (d) angular acceleration
37. A set of atoms in an excited state (c) Cohesive energy 44. A lens when placed on a plane
decays. mirror, then object needle and its
free energy of atoms
(a) in general to any of the states = image coincide at 15 cm. The focal
crystal energy
with lower energy length of the lens is:
(d) Cohesive energy
(b) into a lower state only when free energy of atoms × crystal
excited by an external electric =
energy
field
40. A radio wave that travels in a
(c) all together simultaneously into
straight line from the transmitting
a lower state
antenna to the receiving antenna
(d) to emit photons only when they (a) 15 cm (b) 30 cm
is known as:
collide (a) sky wave (c) 20 cm (d) ∞
38. The temperature (T) dependence (b) ground wave 45. As given in the figure, a series
of resistivity (ρ) of a semiconductor (c) space wave circuit connected across a 200 V,
is represented by: (d) lonospheric wave 60 Hz line consists of a capacitor
41. A photocell employs photoelectric of capacitive reactance 30 Ω, a
ρ
effect to convert: non-inductive resistor of 44 Ω and
(a) change in the frequency of light a coil of inductive reactance 90 Ω
(a) into a change in electric voltage and resistance 36 Ω. The power
(b) change in the intensity of dissipated in the coil is:
O illumination into a change in Xi =30 Ω
T
photoelectric current
ρ (c) change in the intensity of 200 V
illumination into a change 60 Hz R1 = 44 Ω
(b) in the work function of the
photocathode
(d) change in the frequency of light XL = 90 Ω
O
T into a change in the electric
(a) 320 W (b) 176 W
ρ current
42. T h e h o r i z o n t a l r a n g e a n d (c) 144 W (d) 0 W
maximum height attained MATHEMATICS
(c) by a projectile are R and H
respectively. If a constant 46. If A = { 0,1, 2} and B = { 2, 4, 6} ,
O horizontal acceleration a = g/4 is then ( A ∩ B ) × ( A ∪ B ) is:
T imparted to the projectile due to
ρ wind, then its horizontal range (a) {( 0, 2 ) , (1, 2 ) , ( 2, 2 ) , ( 4, 2 ) , ( 6, 2 )}
and maximum height will be:
(b) {( 2,0 ) , ( 2,1) , ( 2, 2 ) , ( 4, 2 ) , ( 2,6 )}
H
(d) (a) (R + H), (b)  R + H  , 2H
2  2 (c) {( 0, 2 ) , ( 2,1) , ( 2, 2 ) , ( 2, 4 ) , ( 2,6 )}
O (c) (R + 2H), H (d) (R + H), H (d) {( 2,0 ) , ( 2,1) , ( 2, 2 ) , ( 2, 4 ) , ( 2,6 )}
T
146 Practice Set-12

1 1 53. If α, β are the roots of the 60. If the chord joining the points t1
47. The number + , when equation:
1+ i i−1 and t2 of the parabola y2 = 4ax,
i = −1 ;is: passes through the focus, then:
2x + 3 1 1 1
(a) Pure imaginary = 0, then + is (a) t1t2 = –1 (b) t1t2 = 1
x x+2 α β
(b) A positive rational (c) t1+ t2 = – 1 (d) t1 + t2 = 1
(c) An irrational equal to: 61. The coordinates of the focu of
(d) A whole number 1 1 the hyperbola
(a) (b)
48. Let p/q means ‘p divides q’ and 4 3 9x2 – y2 – 54x + 77 = 0, are:
let (p, q) denote the (g.c.d) of two (c) –1 (d) 1
integers p and q not both zero.  2 10 
54. The value of expression 47
C4 + (a)  3 ± ,0 
Decide which of the following  3
5  
statements is/are correct? 52 − j
(I) p/q and q/p ⇒ p = q
∑ C3 will be:
 2 10 
j =1
(b)  ± ,0 
(II) p/q ⇒ p ≤ q  3
(a) 52
C4 (b) C3
52
 
(c) 52
C7 (d) C3
42
(III) ( p, q ) = ( p,q)  2 10 
3 (c)  ± ,3 
The correct answer is– 55. Given that sin θ + cos θ =  3
2  
(a) Only (I)
π (d) None of these
(b) Only (II) and 0 < θ < , the value of ‘θ’ is
(c) Only (III) 2 62. What is the order of the differential
then: equation whose solution is y = A/x
(d) (I), (II) and (III)
(a) 60° (b) 30° + B?
49. If m th term of any arithmetic
(c) 45° (d) 15°
progression is n and nth term is (a) 1 (b) 3
56. If 2 cos2 θ + 5 sin2 θ = 3, the value
m, then (m + n)th term will be: (c) 2 (d) None of these
of tan2 θ is given by:
(a) 1 (b) –1 63. In order that the lines y = x + 1,
(c) 0 (d) (m + n – 1) 1 2x – 3y + 5 = 0 and 3x – y + m = 0
(a) 2 (b)
50. The sum of reciprocals of n terms 2 may meet in a point, the value of
of series a, ar, ar2.... will be: 3 3 m should be:
(c) (d)
1− r n
1− r n 8 7 (a) –1 (b) –3
(a) (b) (c) –2 (d) –4
a a (1 − r ) 2 2
57. If the equation k( x + 1) + ( y + 2) 64. If A = tan –1 x, then sin 2A is
3 4
1− rn 1− rn = 1 represents a circle, the value equal to:
(c) n (d)
r −1 a (1 − r )r n−1 of k is: 2x 2x
(a) (b)
51. How many total factors may be 4 1 − x2 1 − x2
(a) 1 (b)
possible of number 5040, when 3 2x
(c) (d) None of these
1 and 5040 is not to be taken as 3 1 + x2
(c) 12 (d)
factor? 4 65. Two functions
(a) 54 (b) 56 58. If diameter of escribed circle  1
(c) 55 (d) 58 of any triangle is equal to f (x) =  x sin x , x≠ 0
52. The value of 'k' so that x2 + 5px + perimeter of triangle, then  0, x=0
16 = 0, has no real root: triangle will be: g (x) = | x | are given, then at
(a) Isosceles (b) Equilateral
x=0
(a) is greater than 8 (c) Right angled (d) Acute angled
5 59. The differential of log10 x w.r.t. x2 (a) Both f and g are continuous
is given by: (b) f is continuous and g is
8
(b) is less than − discontinuous
5 1 log e 1 log 10
(a) − . 10 (b) . e (c) f is discontinuous and g is
2
8 2 x 2 x2
(c) lies between − 8 and continous
5 5 1 log e (d) Both f and g are discontinous
(d) None of these (c) . 10 (d) x2 loge 10
2 x2
Practice Set-12 147

66. Three groups of children contain REASONING AND GENERAL 79. Three numbers are in the ratio
3 girls and 1 boy, 2 girls and AWARENESS (RAGA) 2 : 3 : 4 and their H.C.F. is 12.
2 boys, 1 girl and 3 boys. One The numbers are :
child is selected at random 71. Other things being equal, a (a) 24, 36, 48 (b) 10, 15, 20
from each group. Probability of decrease in quantity demanded (c) 20, 25, 30 (d) 12, 24, 36
selecting 3 children consisting of a commodity can be caused by
(a) a rise in the price of the 37 1
one girl and two boys is: 80. If = 2+ ,
commodity 13 1
x+
(a) 13/32 (b) 27/32 (b) a rise in the income of the 1
y+
consumer z
(c) 21/61 (d) 9/64
(c) a fall in the price of a where x, y, z are natural umbers,
−1 commodity
67. etan is equal to: then c, y, z are :
∫ 1 + x2 dx (d) a fall in the income of the
(a) 1, 2, 5 (b) 1, 5, 2
consumer
(a) tan–1 x (c) 5, 2, 11 (d) 11, 2, 5
72. In which year were the States
81. The average marks of 56 students
(b) e tan x
−1
recognised on a linguistic basis?
is shown as 60. It includes a
(c) sec–1 x (a) 1951 (b) 1947 wrong entry of 92 marks instead
(d) None of these (c) 1950 (d) 1956 of 29 marks. The correct average
73. From which part of Opium plant is ............ .
x 3 t d
68. If f (x) = ∫a t e dt , then f ( x) we get morphine? (a) 58.875 marks
dt
(a) Leaves (b) Stem (b) 61.125 marks
is equal to:
(a) ex(x3 + 3x2) (c) Bark (d) Fruit coat (c) 63.375 marks
(b) x3ex 74. Which of the following metals (d) 56.625 marks
has least melting point? 82. is similar to ∆PQR. Length of
(c) a3ea
(a) Gold (b) Silver AB is 36 cm and length of the
(d) None of these
(c) Mercury (d) Copper corresponding side PQ is 16 cm.
x2 y2 If area of is 1296 sq cm, what is
69. If any tangent of ellipse + 75. “Life Divine” is a book written
a2 b2 the area of?
by
(a) 128 sq cm (b) 512 sq cm
= 1 cuts intercepts of length h (a) M. K. Gandhi
(c) 64 sq cm (d) 256 sq cm
and k on axes, then the value of (b) Rabindranath Tagore
83. If 2 shirts are offered free on
 a 2 b2  (c) S. Radhakrishnan
 2 + 2  will be: purchase of 3 shirts priced `
h k  (d) Shri Aurobindo 600 each, then what will be the
76. The boiling point of water effective discount on each shirt?
(a) 0 (b) 1 decreases at higher altitudes due (a) 30 percent
(c) 2 (d) 4 to (b) 66.67 percent
70. What is the arithmetic mean (a) low temperature (c) 40 percent
(b) low atmospheric pressure (d) 25 percent
of the marks obtained by 40
(c) high temperature 84. If Girish's salary is 4/9
students in an examination? (d) high atmospheric pressure times of Harishankar's and
77. What is the rank of India in terms Shirish's salary is 5/6 times of
Marks No. of Studens Harishankar's, what is the ratio
of Tuberculosis (TB) cases per
6 6 lakh population globally? of Girish's salary to Shirish's?
(a) 27:10 (b) 10:27
4 10 (a) 33 (b) 34
(c) 8:15 (d) 15:8
3 13 (c) 35 (d) 30
85. Rehman walks at 8 km/hr
78. Two fractions are such that their and Roma cycles at 13 km/hr
5 11 product is 9/10 and sum is 77/40. towards each other. What was
What are the two fractions ? the distance between them when
(a) 0.45 (b) 4.00 (a) 2/5, 9/4 (b) 1/5, 9/2 they started if they meet after 36
(c) 4.25 (d) 4.50 (c) 4/5, 9/8 (d) 2/5, 9/4 minutes?
148 Practice Set-12

(a) 12.6 kms (b) 18.9 kms 91. Which of the following is true? towards the centre of the table in a
(c) 15.8 kms (d) 9.5 kms (a) C is fourth to the left of B restaurant. They have ordered for
86. A man said to a lady, “Your (b) A is to the immediate right different items (Pizza, Strawberry,
mother’s husband’s sister is my of G Vanilla, Burger, Pastries and Patties)
aunt.” How is lady related to the (c) D is second to the left of E as their lunch. They are wearing
(d) B is second to the right of G T-shirts of different colours, i.e. white,
man?
92. Arrange the given words in the black, green, red, yellow and blue.
(a) Grand daughter
sequence in which they occur in Order of items for the lunch and
(b) Aunt colours off T-shirts are not necessarily
(c) Daughter the dictionary.
according to the order of their names.
(d) Sister i Heredity ii Hesitate
iii Heavy iv Herald 1. The persons who have ordered
87. If GLARE is coded as 67810 and for Pizza, Vanilla and Pastries
MONSOON as 2395339 then (a) iii, iv, i, ii (b) i, iv, iii, iv
are neither in white T-shirt nor
how can RANSOM be coded? (c) ii, iii, i, iv (d) iv, iii, ii, i
in black.
(a) 183952 (b) 198532 93. Find the number of triangles in
II. The persons who are in green
the given figure.
(c) 189352 (d) 189532 and yellow T-shirts have neither
Z ordered for Pizza nor for Vanilla.
88. Parul cycles 3 km East, then
turns South and cycles 1 km, III. A is neither in white T-shirt
then turns West and cycles nor on the immediate left of
2.5 km, then turns to her right the person who has ordered for
and cycles 1 km. Where is Burger.
she now with reference to her IV. The only person who is between
starting position? X Y E and F eats Strawberry. The
person who is on the left side of
(a) 5.5 km East (b) 0.5 km West (a) 17 (b) 15 the person in white T-shirt does
(c) 0.5 km East (d) 5.5 km West (c) 13 (d) 9
not eat Patties.
89. 37 boys are standing in a row Direction (94): Arrange the given
V. D has ordered for Burger and
facing the school building. words in the sequence in which they the colour of his T-shirt is green.
Ashwini is 15th from the left end. occur in the dictionary. He is facing the person who has
If he is shifted six places to the 94. i. Rainbow ii. Rancour ordered for Strawberry.
right, what is his position from iii. Rattle iv. Rainy VI. One who has ordered for Pizza
the right end? (a) i, iii, ii, iv (b) i, iv, ii, iii is seated opposite to the person
(a) 16th (b) 21st wearing blue T-shirt, while the
(c) i, iv, iii, ii (d) ii, i, iv, iii
(c) 20th (d) None of these person nose T-shirt is of green
Direction (95): The following questions colour is on the left of the person
Direction (90): A series is given
are based upon the alphabetical series who has ordered for Pastries.
with one term missing. Choose the given below:
alternative from the given ones that VII. One who has ordered for Patties
S L U A Y J V E I O N Q is on the immediate right of the
will complete the series. G Z B D R H person in white T-shirt but on
90. Jl, LK, NM, ?
95. If in a certain code ‘Noun’ is the immediate left of the person
(a) ST (b) OP coded as ‘OPVO’ and ‘VERB’ who has ordered for Vanilla.
(c) QR (d) PO is coded as ‘WFSC’ how will VIII. C has not ordered for Vanilla
Direction (91): Read the following ‘JOHN’ be coded in the same while F has not ordered for Pizza.
information carefully and answer the code language? 96. Who among the following is in
questions given below: (a) EQDG (b) KPIO white T-shirt?
(i) A, B, C, D, E, F and G are sitting (a) A (b) B
(c) YIRO (d) VNRQ
around a circle and are facing the (c) C (d) E
centre. Directions (96-100): Read the
97. The only person, who is between
(ii) G is second to the left of C, who is following information carefully and
E and D, is wearing T-shirt of
to the immediate left of F. answer the questions given below:
the colour
(iii) A is third to the left of E. Six persons A, B, C, D, E and Fare (a) red (b) blue
(iv) B is between D and E. sitting around a round table facing
(c) black (d) yellow
Practice Set-12 149

98. Who among the following has correctly matched? 100. The colour of the T-shirt of the
ordered for Pastries? (a) A-Yellow-Burger person, who has ordered for
(a) A (b) B (b) B-Red-Vanilla Patties, is
(c) C (d) E (c) E-Red-Pizza (a) red (b) yellow
99. Which of the following is (d) F-Black-Pastries (c) blue (d) black

Answers with Explanation


ENGLISH Substa-ntiate, Underwrite etc. mass is doubled).
1. (d) 2. (c) 3. (c) 4. (a) Therefore, its proper antonym would m 2m m 2m
⇒ v/2
5. (d) The correct word for Rhytoric be disprove which is given in option (a).
Rest
is – Rhetoric Hence option (d). 18. (b) The word Benevolent
6. (b) Indirect narration is a style (Adjective) is used to different senses Now,
of 3rd person narration which uses the viz, Caring, Compassionate, Considerate, Relative velocity of separation
essence of first person direct speech Generous, Goodwill, Humane, Kind, e=
Relative velocity of approach
and change of tenses into past tense Liberal etc.
accordingly. As it is used here the indirect v/2 1
The proper meaning of Benevolent, = =
from will be - v 2
therefore, should be charitable which is
She told me that I must not break my 23. (b) According to the question,
promise. Hence option (b). given in Option (b),
7. (b) The indirect speech for the For example., 1 1
     Iω 2 = 40% of mv 2
given sentence is (b) He said that he (i) Though he is a despot, he is 2 2
cleans his teeth twice a day. benevolent is his acts.
8. (b) The meaning of pertinent 1 2 40  1 2 2 
(ii) Everyday likes him for his   ⇒ Iω =  mr ω 
is pertaining or relating directly and benevolent acts. 2 100  2 
significantly to the matter at hand,
19. (b) The opposite of proportionate 2 2
relevant or pertinent details. I= mr
9. (d) The correct sentence is – is disproportionate. 5
I went up to him and hold out may 20. (b) in disorderly haste So, the body is solid sphere.
hand, "How are you?" I asked "I am not 24. (a) 580 × 103 cal is evaporated a
in hilarious spirits," he answered. Hence PHYSICS
mass of 1 kg.
option (d). 21. (a) Velocity acquired in falling
10. (d) In the heavens are above' 14.5 × 103 cal is evaporated mass of
through height h.
sentence 'above' is used as adverb. 14.5 × 103
11. (c) Auspicious means good omen u = 2 gh = 2 × 10 × 5 = 10 ms–2 = 0.25 kg
58.0 × 103
and 'Ominous' means 'bad omen'. Again, v2 = u2 + 2as ⇒ (0)2 = (10)2
12. (b) The phrasal verb 'cut down' 25. (b) Surface tension decrease with
+ 2a × 2
would be used in the above sentence- increase the temperature.
100
Rising prices will force people to cut a = – = –25 ms–2 26. (a) The acceleration due to
down (reduce) expenditure. Hence 2× 2 gravity at a depth d is:
option (b). This is total retardation due to
13. (c) The correct preposition here  d R – d 
gravity and cardboard gd = g 1 –  = g   But
will be 'for' i.e. I bought this pen for five  R   R 
∴ Retradation due to cardboard
rupees. R – d = y is the distance of the point
14. (c) I don't like being kept waiting. a' = g + a = (10 + 25) = 35 ms–2
Force due to cardboard = Ma' from the centre of the Earth. Then,
15. (b)Have you ever been bitten by
a dog? = 200 × 10–3 × 35 = 7000 × 10–3 g
gd = y. So, the acceleration of the
16. (b) = 7N R
17. (a) The word Validate is used in 22. (a) From conservation of linear body in the tunnel is proportional to
different senses, viz., Attest Authenticate, momentum, we can see that velocity of y. And the motion is SHM with time
A u t - h o r i s e , C e r t i f y, C o n f i r m , v period.
2m will become after collision (as
Corroborate, Endorse, Legalise, Ratify 2
150 Practice Set-12

–½ Y ⇒ ev0B = 3ma0 – ma0


R 
T = 2π   [from Eq. (i)]
g A ⇒ ev0B = 2ma0
Hence, it oscillates between the two 2ma0
( 3, 3)
ends of the tunnel. ∴ B=
ev0
27. (b) From Gauss’ theorem,
36. (d) For the study of the helical
Net charge enclosed by the surface structure polarisation property is used.
φ= Q
ε0
37. (a) A set of atoms in an excited
The net charge due to a dipole is O (0, 0) B X
state decays in general to any of the state
zero, hence φ = 0. tan Q = slope of line OA with lower energy.
dI 38. (b) The resistivity of a semi-
28. (a) Use, e = L AB 3
dt = = = 3 or θ = 60º conductor decreases with increases in
OB 3
Here, E = 8V, dI = 2A – (–2A) temperature exponentially.
= 4A, dt = 0.05s 32. (c) At height h above Earth’s 39. (a) Cohesive energy = free
Hence, L = 0.1 H surface, energy of atom – crystal energy.
29. (c) E = 11 eV = 11 × 1.6 × 10–19 GM  Re 
2
40. (a) Sky waves are used for very

J = hv g' = 2 = g ×  long distance radio communication at
(R e + h)  Re + h 
11 × 1.6 × 10 –19 medium and high frequencies.
or v= At depth h below Earth’s surface,
h
A radio wave that travels in a
11 × 1.6 × 10 –19 GM (R e – h) R – h straight line from the transmitting
= g" = 3 = g s 
6.62 × 10 –34 Re  Re  antenna to the receiving antenna is
       = 2.65 × 1015 Hz As, g' < g", then F1 < F2. known as sky wave.
This frequency radiation belongs to 33. (d) Electric field intensity due to 41. (b)

Photoelectric current
ultraviolet region. charge q at a distance r.
1 q A
V E= .
30. (b) tan δ = and tan δ' 4πε 0 r 2
H
V tan δ q
= or tan δ' = 500 = 9 × 109 ×
H cos θ cos θ (3) 2
500 × 10–9 = q O
  ⇒ tan δ = tan δ' cos θ ⇒ q = 0.5 × 10–6 C lntensity
= tan (60º) cos (30º) q = 0.5 µC
Using the incident radiation of
34. (c) Graph indicates that resistance
3 3 a fixed frequency, it is found that the
= 3× = increases with increase in temperature, so
2 2 photoelectric current increases linearly
α and β both are positive.
with intensity of incident light as shown
–1  3  35. (b) Initial acceleration,
⇒ δ = tan   in figure. Hence, a photocell employs
2 eE
a0 = ...(i) photoelectric effect to convert change
31. (c) Draw the situations as shown. m in the intensity of illumination into a
OA represents the path of the particle a0 m change in photoelectric current.
starting from origin O(0, 0). Draw a ⇒ E=
e 2u y u2y
perpendicular from point A to x-axis. Let ev0 B + eE 42. (d) T= ,H = and R =
path of the particle makes an angle θ with ∴ = 3a0 uxT g 2g
m
the x-axis, then When a horizontal acceleration is
⇒ ev0B + eE = 3a0m
∴ ev0B = 3ma0 – eE also given to the projectile uy. T and H
Practice Set-12 151

will remains unchanged while the range MATHEMATICS n


will become 1  1 
46. (d)
A ∩ B ={2}, 1 −   
a   r  

1 2
R' = uxT + aT A ∪ B = {0,1, 2, 4,6} = 
1
2 1−
(A ∩ B) × (A ∪ B) r
1 g  4u g 
2

= R +  2  = {( 2,0 ) , ( 2,1) , ( 2, 2 ) , ( 2, 4 ) , ( 2,6 )} 1  rn −1 


2 4 g   
a  r n 
= R + H 47. (a) = = 
43. (b) Parsec is unit of distance used  r −1 
 
in astronomy.  r 
=
1 parsec = 3.08 × 1016 m = 3.26
rn −1 r
light year = .
44. (a) When the object is placed at ar n r − 1
the focus the rays are parallel.
48. (d) If p divides q and q divides p, 1− rn
=
then p and q are same, hence (I) is correct. a (1 − r )r n−1
If p/q i.e., if q is completely 51. (d) Number of combinations of
divisible by 'p'
total objects (p +q + r + t), when p are
i.e., p ≤ q , hence (II) is correct. similar objects of one type, q are similar
Obviously ( p, q ) = ( p , q ) . objects of second type and r are similar
Hence, (III) is correct. objects of third type and t objects are
The mirror placed at normal sends
them back. Hence, image is formed at 49. (c) According to question, different type, then
the object itself. n = a + (m – 1)d...(i)
m = a + (n –1)d...(ii) = {( p + 1)( q + 1)( r + 1)( t + 1) − 2}
45. (a)
On subtracting, [1  and 5040 is to be taken as
Xi = 30 Ω
factor]
n – m = (m – n)d
200 V given number 5040 = 24 × 32 × 5 × 7
n−m has comprises 4 similar factors of one
60 Hz R1 = 44 Ω
\ d= = −1
m−n type (24), two similar factors of second
type (32) and two different factors.
and n = a + (m − 1)(−1)

Therefore, number of possible factors
XL = 90 Ω

\ a = m + n −1
= ( 4 + 1)( 2 + 1)(1 + 1)(1 + 1) − 2
Xi = 30 Ω, R2 = 36 Ω, th
( m + n)
term
XL = 90 Ω, XC = 30 Ω = 5× 3× 2 × 2 − 2
and total resistance = a + (m + n − 1)d = 60 – 2 = 58
R = R1 + R2 = 44 + 36 = 80 Ω 52. (c) D<0
= (m + n − 1) + (m + n − 1)(−1)
⇒ 25p2 – 64 < 0
Z= 2
R + (X L – X i ) 2

= m + n −1− m − n +1 = 0 64

p2 <
= 2
(80) + (60) 2 50. (d) Series formed by reciprocals 25
1
of terms of series a, ar, ar2.... is , 8 8
Z= 6400 + 3600 = 100 a ⇒
− <p<
5 5
1 1 1
V 200 , ... whose first term = and
Current, I = = = 2A ar ar 2 a 2x + 3 1
Z 100 53. (c) Given that =0
1 x x+2
Power dissipated in the coil common ratio = .
r
Pav = I2R = (2)2 × 80 = 320 W     (2x + 3) (x + 2) – x = 0
Sum of n terms of its series
⇒ 2x2 + 6x + 6 = 0
152 Practice Set-12

⇒ x2 + 3x + 3 = 0 1
Since, α, β are the roots of this ∴ = ⇒ –t = (1 − t12 )
t1 + t2
equation
∴ α + β = –3 ⇒ –t1(t1 + t2) = 1 – t12
αβ = 3 ⇒ k= ⇒ t1t2 = –1
61. (a) 9x2 – y2 – 54x2 + 77 = 0
1 1 −3 58. (c)  Diameter of any escribed
∴ + = α+β = = –1 ⇒ 9(x2 – 6x + 9) – y2 = –77 + 81
α β αβ 3 triangle ⇒ 9(x – 3)2 – y2 = 4
= perimeter of triangle
54. (a) On putting, j = 1, 2, 3, 4, 5 y2
∴ 2r1 = 2s ( x − 3) 2
Expression, ⇒ – =1
4/9 4
= 47C4 + 51C3 + 50C3 + 49C3 + 48C3 + 47C3 ⇒ r1 = s
(taking escribed circle opposite to A) Transferring the origin at (3, 0), we
= (47C4 + 47C3) + 48C3 + 49C3 + 50C3 + 51C3
get
= (48C4 + 48C3) + 49C3 + 50C3 + 51C3 1
⇒ s tan A=s x2 y2
[∴ nCr + nCr –1 = n + 1Cr] 2 − =1
4/9 4
= (49C4 + 49C3) + 50C3 + 51C3
A 4
= (50C4 + 50C3) = 51C3 ⇒ tan = 1 = tan 45° Here, a2 = and b2 = 4
2 9
= (50C4 + 51C3) = 52C4
 A  b2 = a2 (e2 – 1)
3 ∵ r1 = s tan 
55. (d) sin θ + cos θ =  2
2 4 2
∴4= (e – 1) ⇒ e = 10 .
1 9
1 1 3 ∴ A = 45° ⇒ A = 90°
⇒ sin θ + cos θ = 2
2 2 2 2 10
Therefore, ∆ is right angled. Hence, for foci X = + and
Y=0 3
⇒ sin 45° sin θ + cos 45° cos θ
d 1
3 59. (c) (log10 x) = log10 e ∴ Coordinates of foci are:
       = dx x
2
⇒ cos (45° – 1) = cos 30° d 2 2 10  2 10 
( x ) = 2x x = 3±
3
y = 0, ⇒  3 ± 3 , 0 
⇒ 45° – 1= 30° dx  
∴ θ = 15° A
∴ Diff. of log10 x, w.r.t.
62. (c) y= +B
56. (b)  2 cos2 θ + 5 sin2 θ = 3 x
d
⇒ 2 + 5 tan2 θ = 3 sec2 θ (log10 x)
dx 1 log e dy
x2 = = . 10 A
[Dividing the each term by cos2 θ] d 2 2 x2 ∴ = − 2
(x ) dx x
⇒ 2 + 5 tan2 θ = 3(1 + tan2 θ) dx
60. (a) Point t1 = (at12, 2at1) and t2 dy
⇒ 2 + 5 tan2 θ = 3 + 3 tan2 θ ∴ A = – x2
= (at22, 2at2) dx
⇒ 5 tan2 θ – 3 tan2 θ = 3 – 2
Equation of the line joining t1 and d2y 2A
⇒ 2 tan2 θ = 1 t2 will be: and =....(i)
dx 2
x3
1 Putting the value of A in eq. (i), we get
tan2 θ = 2at2 − 2at1
2 y – 2at1 = (x – at12)
at22 − at12 d2y 2  2 dy 
57. (d) The coefficient of x and y in 2 = 3  −x 
2 2
dx x  dx 
Since, this line passes through the
k ( y + 2) 2 k 1 focus (a, 0) of the parabola y2 = 4ax. d2y 2 dy
(x + 1)2 + = 1, are and ⇒ + =0
3 4 3 4 dx 2
x dx
2
(a − at12 ) 2
respectively but in circle the coefficients ∴ 0 – 2at1 =
t1 + t2 The highest derivative in it is d y
of x2 and y2 must be equal. whose order is 2. dx 2
Practice Set-12 153

63. (b) On solving the equations y = = R. lim f ( x) = f (0) = 0 x


x + 1 and 2x – 3y + 5 = 0, we get x = 2 x→0 = [t 3et ]ax − 3∫ t 2 et dt
a
and y = 3 ∴ f(x) is continuous at x = 0. x
Now, L. lim g ( x) = [t 3et ]ax − 3∫ t 2 et dt
Since, these three line are a
x→0
concurrent, therefore, x = 2 and y = 3 x
will satisfy the equation 3x – y + m = 0 = lim g (0 − h) = lim | – h | = (t 3et ) ax − 3[t 2 et ]ax + 6∫ t e dt
t
a
h→0 h→0
∴6–3+m=0⇒m=–3 (t 3et ) ax − 3(t 2 et ) ax + 6[tet − ∫ et dt ]ax
= lim h = 0 =
64. (c) A = tan–1 x h→0
tan A = x = (t 3et ) ax − 3(t 2 et ) ax + 6(tet ) ax − (6et ) ax
R. lim g ( x) = lim g (0 + h)
x x→0 h→0 = [(t3 – 3t2 + 6t – 6)et]ax
sin A =
1 + x2 = lim | h | = (x3 – 3x2 + 6x – 6)ex – (a3 – 3a2
h →0
+ 6a – 6)ea
6 = lim h = 0 d
2
∴ f ( x)
+x
h →0
dx
1 x
g (0) = |0| = 0 = (3x2 – 6x + 6)ex + (x3 – 3x2 + 6x – 6
∴ L. lim g (x) = R. lim g (x) = g(0) = 0 ex + 0
Α x→0 x→0
1 =e x
x 3
∴ g (x) is also continuous at x = 0
69. (b) Let equation of any tangent
66. (a) Selection can be made in the of ellipse is:
1
and cos A = following manner:
1 + x2 y = mx + (a 2 m 2 + b 2 ) ...(i)
(i) Boy boy girl, probability
sin 2A = 2 sin A cos A ∴ h = intercept cut on x-axis by
1 2 1 1 line (i)
2x 1 . . =
p1 =
= . 4 4 4 32
2 (a 2 m 2 + b 2 )
1+ x 1 + x2 (ii) Boy girl boy, probability = −
m
2x 1 2 3 3 and k = intercept cut on y-axis by
= p2 = . . =
2 4 4 4 32 line (i)
1+ x
65. (a) Given functions are: (iii) Girl boy boy, probability = (a 2 m 2 + b 2 )

 1 3 2 3 9 a2 b2
 x sin , x ≠ 0 p3 = . . = ∴ +
f (x) =  x 4 4 4 32 h2 k
 0, x = 0 Since, these are mutual exclusive
a 2 m2 b2 m2
g (x) = | x |
cases. = +
∴ Required probability = p1 + p2 + a 2 m2 + b2 a 2 m2 + b2
   L lim f (x) = lim f (0 – h)
x→0 h→0 p3 = 13/32 a 2 m2 + b2
−1 = =1
 1 e tan x a 2 m2 + b2
= lim − h sin  − 
h →0  h
67. (b) Let I= ∫ 1 + x2 dx
70. (c)

1 Put tan–1 x = t Marks (x) No.of Students (f ) f.x


= lim h sin =0
h →0 h 1 6 6 36
⇒ dx = dt
R. lim f ( x) 1 + x2 4 10 40
x→0 t 3 13 39
1
∴ I= ∫ e dt = et
5 11 55
= lim f (0 + h) = lim h sin = 0
h→0 h→0 h Total 40 170
I = e tan −1 x
Also f(0) = 0 x 3 t
∴ L. lim f ( x)
68. (b) f (x) = ∫a t e dt ∴ A.M. =
x→0
154 Practice Set-12

REASONING AND GENERAL per lakh population. iii. In 2018, Over 1 2


5 lakh 62 thousand patients were cured
⇒ x+ = 1+
AWARENESS (RAGA) 1 11
of TB.The Vision is of a TB free India y+
71. (a) In economics, the law states z
is zero deaths, disease, and poverty
that, all else being equal, as the price of due to tuberculosis.The Goal is to 1
a product increases, quantity demanded
⇒ x = 1, y +
achieve a rapid decline in the burden z
falls; likewise, as the price of a product of TB, mortality, and morbidity. The
11 1 1
decreases, quantity demanded increases. requirements for moving towards TB = = 5 = 5+
So basically the quantity demanded and elimination in India have been arranged 2 2 2
the price of a commodity is inversely in four strategic areas of Detect, Treat, x = 1, y = 5, z = 2
related, other things remaining constant. Prevent and Build. ⇒
72. (d) Indian states were reorganised 78. (c) Let x and y be two fractions 81. (a) Total marks of 56 students
on 1 November 1956 under the States In case-I = 56 × 60 = 3360
9
Reorganisation Act, 1956. Andhra State xy = ...(i) Correct Average
was merged with the Telugu speaking 10
In case-II 3360 + 29 − 92
area of Hyderabad state (also known as =
Telangana) to create Andhra Pradesh in 77 56
x+y = ...(ii)
1956. Similarly Kerala in the south and 40 3297
three states (Uttar Pradesh, Bihar and Solving equations (i) and (ii), we get = = 58·875.
56
Madhya Pradesh) came into being in 77 82. (d)
the Hindi speaking area. West Bengal, x+y = A P
40
Rajasthan, and Punjab were enlarged by
addition of territories.

cm
cm
9 77
+y =

16
36
73. (d) Morphine is the predominant 10 y 40
alkaloid found in the varieties of opium B C Q R
poppy plant. It is obtained in form of 40y2 – 77y + 36 = 0
ar (∆ABC) AB (36) 2 2
liquid from the fruit capsule of the (8y – 9) (5y – 4) = 0 = =
poppy. The latex which oozes from ar (∆PQR) PQ 2 (16) 2
the incisions is collected, and dried to 9, 4
y = or y = 1296  36  9
2 2
produce “raw opium” (about 8-14% 8 5 =   =  
morphine by dryweight). ar (∆PQR)  16  4
74. (c) The melting point of the 4 9
x = or x = 16 × 1296
given metals (in Celsius) are:- Gold: 5 8 ar (∆PQR) =

1063; Silver: 961;Copper: 1083; 81
79. (a) Let the required numbers be
Mercury: -38.86. Mercury is the only 2x, 3x and 4x. Then their H.C.F = x. So, = 16 × 16
elemental metal known to melt at a x = 12. = 256 sq cm
generally cold temperature. 83. (c) Cost price of each shirt
75. (d) The Life Divine’ is a 1 37
80. (b) 2 + = ` 600
metaphysical treatise authored by Shri 1 13 = = ` 200
x+ 3
Aurobindo. It is called the most profound 1
y+ Cost price of 5 shirts
book ever written on the nature of spirit, z
creation, life, and human evolution. 11 11 = ` 200 × 5 = ` 100
76. (b) At higher altitudes, the air = 2 = 2+ Profit = ` 1000 – ` 600
13 13
pressure is decreased, which forces = ` 400
1 11
water’s boiling point to lower. The air ⇒ =
1 13 400 × 100%
pressure decreases with altitude because x+ Effective percent =
of the decrease in the density of air. 1 = 40% 1000
y+
The lowered boiling point of water z
84. (c) Let Harishankar’s salary
requires an increase incooking times or 1 11 be x
⇒ =
temperature. 1 13
x+ 4
77. (c) 35 Explanation: India ranks 1 x
y+ Girish’s salary =
35th in the world in terms of TB cases z 9
Practice Set-12 155

5 Required distance 95. (b) As,


Shirish’s salary = x = (3 km – 2.5 km) NO UN O P VO
6
= 0.5 km +1
Ratio of Girish’s salary to Shirish’s +1
salary Hence, Parul will be 0.5 km east +1
4 with reference to her starting position. +1
x
89. (d) Now, position of Ashwini
= 5 = (15 + 6)
And,
5
x VE RB WF S C
6 = 21st from left
+1
∴ Ashwini's position from right +1
4 × 6 24 8 +1
= = = = (37 + 1 – 21) = 17th
5 × 5 45 15 +1
90. (d)
= 8 : 15. Similarly,
J I L K N M P O
85. (a) Total distance covered 10 9 12 11 14 13 16 15 J O HN K P I O
36 36 +1
= 8× + 13 × +2 +2 +2 +1
60 60 +2 +2 +2 +1
+1
3 3 91. (a) By analysing the above
= 8× + 13 × circular arrangement, it is a clear that 96. (c) By analysing the above
5 5
C  Third to the left of B. circular arrangement, it is clear that
24 + 39 63
= = = 12·6 kms A  Immediate right of G. wears
5 5 C White T-shirt.
86. (d) Lady’s mother’s husband ⇒ D  Second to the right of E. 97. (d) By analysing the above
Lady’s father B  Second to the left of G. circular arrangement, it is clear that
So, Lady’s father’s sister ⇒ Lady’s between E and D ⇒ A.
Therefore, option (b) is true.
aunt wears
92. (a) According to dictionary the Therefore, A Yellow
[ Lady’s aunt ⇒ man’s aunt] arrangement of words is given below: T-shirt.
Therefore, the ‘lady’is sister of the (iii) Heavy, (iv) Herald, 98. (a) By analysing the above
‘man’. (i) Heredity, (ii) Hesitate circular arrangement, it is clear that
87. (d) As, 93. (a) Z orders
G L A R E A Pastries.
↓ ↓ ↓ ↓ ↓ 99. (c) By analysing the above
6 7 8 1 0 B circular arrangement, it is clear that
A C
And,
M O N S O O N Person wears Colour
D E
↓ ↓ ↓ ↓ ↓ ↓ ↓ E Red T-Shirt
2 3 9 5 3 3 9 X Y orders Item
F 
Similarly, The triangles are Pizza
R A N S O M ∆ZAB, ∆ZBC, ∆XAD, ∆XDF, ∆FAB Therefore, option (c) is correctly
↓ ↓ ↓ ↓ ↓ ↓ ∆FBC, ∆YEC, ∆FEY, ∆ZAC, ∆XAF, matched.
1 8 9 5 3 2
∆YFC, ∆FAC, ∆ZFX, ∆ZFY, ∆AFZ, 100. (d) By analysing the above
88. (c) ∆CFZ, ∆ZXY
circular arrangement, it is clear that F
km 94. (b) According to dictionary, the orders
arrangement of words is: Patties.
km km orders
(i) Rainbow, (iv) Rainy, Therefore, F Black T-shirt.
km
(ii) Rancour, (iii) Rattle. qqq
156 Practice Set-13

Air Force Airmen Group X & Y Exam


Practice Set-13

the nurse would paint her nails. All the 8. ALIEN


ENGLISH time, her mom would smile and hum. (a) native (b) domiciled
Directions (1-4): Read the passage 1. The use of the word 'cranking' (c) natural (d) resident
given below and answer the questions conveys the meaning that– 9. Point out the synonym of the word
that follow by selecting the correct/ (a) the key did not fit the lock PENITENCE
most appropriate options. properly (a) submission
As I kept strict watch over my way of (b) the clock doesn’t work (b) retribution
living, I could see that it was necessary (c) the key was used with irritation (c) confinement
to economise. I, therefore, decided to on the mother’s part (d) repentance
reduce my expenses by half. My accounts (d) gold was not a suitable metal for 10. The expression "gift of the gab"
a clock key means
showed numerous items spent on fares. 2. The siblings were 'awful mouths' (a) power of learning
Again, my living with a family meant the is the following figure of speech– (b) power of eloquence
payment of a regular weekly bill. It also (a) comparison (c) blessing
included the courtesy of occasionally (b) oxymoron (d) treasure
taking members of the family out to (c) transferred epithet 11. Point out which part of the
dinner and likewise attending parties with (d) epigram given sentence is grammatically
them. All this involved heavy items for 3. "Truly, there was no point to these incorrect?
conveyances, especially as, if the friend silly endless family dinners". The (a) The Third World Countries must
was a lady, custom required that the man correct transformation would be– adopt
should pay all the expenses. Also, dining (a) the subject found her siblings (b) A radically different approaches
out meant extra cost, as no deduction mentally unstable and took long for
could be made from the regular weekly to finish dinner (c) the dissemination of scientific
(b) h er mother forced her to sit information
bill for meals not taken. It seemed to through a lengthy dinner ritual (d) the view of the nature and
me that all these items could be saved, (c) the food could only appeal to the magnitude of their problems
as likewise the drain on my purse. So, youngsters, not a teenager 12. Change the following sentence into
I decided to take rooms on my own (d) the subject was preoccupied Passive Form.
account, instead of living any longer in a with some personal work and How do people learn languages?
family and also to remove from place to was impatient with others at the (a) How are languages learnt?
place according to the work I had to do, dinner table (b) How languages are learned by
thus gaining experience at the same time. 4. The tone of the story is– people?
The rooms were so selected, as to enable (a) biographical(b) narrative (c) Languages are learnt by people
me to reach the place of business on foot (c) reflective (d) autobiographical how?
in half an hour and so save fares. Before Directions (5-6): In the following (d) How languages are learnt by
questions, four words are given in each people
this I had always taken some kind of question, out of which only one word is Directions (13-14): In the following
conveyance whenever I went anywhere, wrongly spelt. Find that word. questions, fill in the blanks with
and had to find extra time for walks. The 5. (a) poignant (b) relevent appropriate alternative out of the
new arrangement combined walks and (c) prevalent (d) malignant four given.
economy, as it meant a saving of fares 6. (a) seize (b) achieve 13. The two brothers ______ over
and gave me walks of eight or ten miles a (c) wierd (d) leisure property dispute.
day. It was mainly this habit of long walks Directions (7-8): In the following (a) falls out (b) fell out
that kept me practically free from illness questions, choose the word opposite in (c) fell on (d) fell off
throughout my stay in England and gave meaning to the given bold word. 14. He has ______ a new business
me a fairly strong body. 7. APPARENT establishment.
Thus I rented a suite of rooms; one (a) illegible (b) hidden (a) set out (b) set in
for a sitting room and another for a and (c) mysterious (d) remote (c) set down (d) set up
Practice Set-13 157

15. Which of the following gives the  (a) Signal (a) comes via ionospheric
time is E = (80iˆ + 32 ˆj – 64kˆ )
correct meaning of the given mode
and the magnetic field is
idiom/phrase?  (b) Signal (b) comes via satellite
A good samaritan B = (0.2iˆ + 0.08 ˆj + 0.29kˆ ) µT. The mode
(a) an honest politician poynting vector for the fields is: (c) Both (a) and (b) are correct
(b) a good priest (a) −11.52iˆ + 28.8 ˆj (d) None of (a) or (b) is correct
(c) a clever person
(b) −28.8iˆ + 11.52 ˆj 27. The time period of a satellite
(d) a genuinely helpful person revolving in a circular path of
16. Identify the underlined clause. The (c) 28.8iˆ − 11.52 ˆj
radius R is T. The time period of
teacher told the students that their (d) 11.52iˆ − 28.8 ˆj the satellite of radius 9 R will be:
result was quite good. 23. As shown in figure, flask A has (a) T (b) 9T
(a) Adjective/Relative clause double the volume of flask B. Ideal (c) 27 T (d) T/9
(b) Adverb clause gas is filled in joint flasks. Flask 28. Two long straight wires are
(c) Principal clause A and B are maintained at 200 K set parallel to each other at
(d) Noun clause and 400 K. If amount of gas in A separation r and each carries a
17. "Where are you going for your is m then flask B will contain gas
current I in the same direction.
holidays"? he said, (Change into equal to:
The strength of the magnetic field
Indirect Speech)
at any point midway between the
(a) He said me where was I going for
your holidays
A B two wires is:
200 K 400 K
(b) He asked if I was going for your µ0 I 2µ 0 I
holidays (a) π r (b) π r
(c) He asked me where I was going µ0 I
for my holidays (a) m/2 (b) m/4
(c) (d) Zero
(d) He wanted to know where was I (c) 2m (d) 2π r
going for holidays 24. A gold leaf electroscope is charged 29. A current of 5 A is passing through
18. Which of the following is the past and the leaves are observed to a metallic wire of cross-section
form of the verb 'dwell'? area 4 × 10–6 m2. If the density of
diverge by a certain amount. The
(a) Dwells (b) Dwelling the charge carriers in the wire is
beam of X-rays is allowed to fall
(c) Dwelt (d) Dwole 5 × 1026 m–3, the drift velocity of
upon the electroscope for a short
Directions (19-20): In the following the electrons is:
period. The effect is as follows:
quesions some of the sentences have 1 1
(a) The gold leaves will melt (a) ms –1 (b) ms –1
errors and some have none. Find out 16 32
which part of a sentence has an error (b) There will be no change in the
and select the appropriate number divergence 1 1
(c) ms –1 (d) ms –1
(a, b, c). If there is no error, select the (c) The leaves will diverage further 64 128
number 'd'. (d) The leaves will collapse 30. Conduction, is the process of heat
19. A moment delay/(a)/ would have 25. If the magnetic flux linked with transfer in which:
proved costly (b)/ in the situation. a coil varies at the rate of 1 (a) heated molecules move from
(c)/ No error (d) Wbmin–1, the induced e.m.f. is: hotter to colder region
20. Ram disappointed his mother (a)/ 1 (b) wave travels from heated to
as he did not (b)/ write to her very (a) 1 V (b) V colder region
60
often. (c)/ No error (d) (c) a heated fluid flows from heated
(c) 60 V (d) None of these to colder region
26. A ground receiver station is (d) heat passes from one particle
PHYSICS
receiving a signal at (a) 5 MHz to other as a result of increased
21. Find the linear momentum of a and (b) 100 MHz transmitted vibration.
photon of energy 6 × 10–20 J. from a ground transmitter at 31. Four beats per second are heard
(a) 2.5 × 10–28 kg ms–1 a height of 300 m located at a in a gas when two waves of
(b) 2 × 10–28 kg ms–1 distance of 100 km. Give radius wavelength 1.0 m and 1.01 m are
(c) 5 × 10–28 kg ms–1 of earth = 6.4 × 106 m; Nmax of passed in it. The speed of sound
(d) None of these ionosphere = 10 12 m –3. Which in the gas will be:
22. In a region of free space the of the following statement of (a) 440 m/s (b) 404 m/s
electric field at some instant of statements is/are correct? (c) 360 m/s (d) 980 m/s
158 Practice Set-13

32. A point mass m and charge q is 41. The SI unit of mobility of charge
connected with massless spring (µ) is:
of natural length L. Initially (a) coul-s/kg
spring is in its natural length. If
(b) coul-kg/s
a horizontal uniform electric field
(c) coul-kg-s
E is switched on (shown in figure),
the maximum separation between (d) coul s–1/kg
the point mass and the wall is: 42. In the network shown, the
(Assume all surfaces are frictionless). (a) 526.67 Ω (b) 790 Ω equivalent resistance between A
Wall and B is:
(c) 1580 Ω (d) Zero
36. If C and R denote capacity and 3Ω
resistance, the dimension of CR
E are:
k (a) [M0L0T–1]

8
m (b) [M1L0T] 4


A B
(c) [M0L0T2]
(d) Not expressible in M, L, T Ω

6
2

W
37. A current i ampere flows along an
2qE qE infinitely long straight thin walled
(a) L + (b) L +
k k tube, then the magnetic induction 6Ω
(c) L (d) None of these at any point inside the tube is:
4 3
33. Two coils are placed close to each (a) Infinite (b) Zero Ω
(a) 3 (b) 4 Ω
other. The mutual inductance of µ 0 2i 2i
the pair of coils depends upon: (c) ⋅ T (d) T
4π r r 24 17
(a) the rates at which current are (c) Ω (d) Ω
38. The normal temperature of the 17 24
changing in the two coils human body on the Kelvin scale 43. If the kinetic energy of a body is
(b) relative position and orientation is:
of the two coils increased by 9% then change in
(a) 241 K (b) 309.15 K
(c) the materials of the wires of the its momentum will be:
(c) 310.05 K (d) 371.4 K
coils 39. When a tuning fork of unknown (a) 2% (b) 3%
(d) the currents in the two coils frequency is sounded with a (c) 4% (d) 9%
34. The equation of the trajectory of tuning fork of frequency 100 Hz, 2 44. A convex lens of glass has power
an oblique projectile: beats per second are heard. When P in air. If it is immersed in water
1 2 the arm of tuning fork of unknown its power will be:
y = 3 x – gx frequency is loaded with a little
2 (a) more than P
Here, x and y are in metre and g wax and both are sounded 1 beat (b) less than P
in ms–2. The angle of projection is: per second is heard. The unknown
(c) P
frequency is:
(a) 0º (b) 90º (d) more than P for some colours
(a) 102 (b) 98
(c) 45º (d) tan –1 3 and less than P for others
(c) 99 (d) 101
35. A potentiometer wire of length 45. If the volume of a cubic cell is 10–30
40. When the three axes of a crystal
100 cm has a resistance of 10 lattice are mutually perpendicular metre3 then its lattice parameter
Ω. It is connected in series with and two of the lattice parameters is:
resistance (shown in figure) and are equal, what is the crystal (a) 10–30 metre
a cell of emf 2 V and negligible system? 1
resistance. A source of emf 10 mV (a) Cubic
–30
(b) × 10 metre
is balanced against a length of 40 2
(b) Tetragonal
cm of potentiometer wire. What (c) Orthorhombic (c) 10–10 metre
is the value of R1? (d) Hexagonal (d) 10–15 metre
Practice Set-13 159

MATHEMATICS 1 − 2x dy φ ( x, y)
(c) 59. If y + x =x , then φ (xy)
1+ x 2
f ( x). f ( x ) 2 (1 − x ) 2 dx φ ' ( x, y)
46. If f (x) = , then is equal to:
1− x 1 + [ f ( x)]2 1 2
is equal to: (d) (a) ke x / 2 (b) ke y 2 / 2
1 1 (1 − x 2 )
(a) (b) (c) ke xy / 2 (d) ke xy
x x
4 6 53. What is the value of ∫a e dx ?
60. From the data given, the median
1 1 a e x x
(c) (d) (a) +C of the average deposit balance
8 2 a of saving for the branch during
1 + 7i (b) axex + C March 1982 is:
47. (2 − i )2 is equal to:
a xex Average Deposit Number of
(c) log(ae) + C
(a) ( 2, π / 4) Balance (`) Deposit
(d) None of these Less than 100 26
(b) ( 2, π / 2) 54. The point of intersection of the two
lines 2x + 3y + 4 = 0 and 4x+ 3y +2 100–200 68
(c) ( 2,3π / 4) 200–300 145
= 0 is at a distance d from origin.
(d) None of these What is the value of d? 300–400 242
48. If the coefficient of x2 and x3 in (a) 2 (b) 3 400–500 188
the expansion of (3 + ax)9 are the (c) 5 (d) 7
same, then the value of a is: 500–600 65
(a) 9/7 (b) 7/9 55. The value of
600–700 16
(c) –9/7 (d) –7/9 2 1 3 1 4 1 5 1
. + . + . + . + ......∞ is: (a) 356 (b) 300
49. Let a, b and c are in GP with 1 < a 1 3 2 9 3 27 4 81
(c) 56.2 (d) 356.2
< b < c an n > 1 is an integer. loga 1 2  π
n, logb n, logc n form sequence. (a) − log e 61. The function f (x) = cos   is
2 3 x
This sequence is which one of the increasing in the interval:
following 2
(a) HP (b) AP (b) − log e 3 (a) (2n + 1, 2n) : n ∈ N
(c) GP (d) None of these 1  1 1
 2 (b)  , :n∈N
50. If n be a positive integer and (1 + (c) 2 + log e  3   2n + 1 2n 
x)n = a0 + a1x + a2x2 +...........+ anxn,  1 1
then what is a0 + a1 + a2 +.........+ (d) None of these (c)  , :n∈N
 2n + 2 2n 
an equal to? 56. The equation x2 + y2 + 4x + 6y + 13
(a) 1 (b) 2n = 0 represents: (d) None of these
(c) 2 n–1
(d) 2n + 1 (a) a circle 62. The probability that a card drawn
51. W h a t i s t h e d i f f e re n c e i n (b) a pair of two distinct straight from a pack of 52 cards will be a
t h e ro o t s o f t h e e q u a t i o n lines diamond or a king is:
x2 – 10x + 9 = 0? (c) a pair of coincident straight lines 4
(d) a point 4
(a) 2 (b) 3 (a) (b) 13
(c) 5 (d) 8 57. If 2 log8 N = p log2 2N = q and q – p 52
x = 4, then find N. 1 2
52. If y = tan–1 + sin (a) 512 (b) 536 (c) (d)
52 13
1 − (1 − x 2 ) (c) 548 (d) 560 63. The total number of 9-digit
58. By principle of mathematical
numbers which have all different
 −1  1 − x  
 induction cos θ cos 2θ cos 4θ
 2 tan  1 + x   digits is:
  , then ................cos (2n – 1 θ),∀ n ∈ N is
(a) 10! (b) 9!
equal to:
dy/dx is equal to: n n (c) 9 × 9! (d) 10 × 10!
x (a) sin 2 θ (b) cos 2 θ 64. If the sum of the coefficient in the
(a) 2n sin θ 2n sin θ expansion of (α2x2 – 2αx + 1)51
(1 − x 2 )
vanishes, then the value of α is:
sin 2n θ
1 − 2x (c) (d) None of these (a) 2 (b) – 1
(b) 2n−1 sin θ (c) 1 (d) 2
(1 − x 2 )
160 Practice Set-13

basic features of the Indian 1


π /2 sin 3 x 80. Simplify:
65. What is ∫0 3
sin x + cos x 3
dx ? Constitution? 2
(a) Keshavananda Bharati case
π 1+ 3
(a) π (b) (b) S.R. Bommai case 2 8
2 (c) Indira Sawhney case 1+ +
3 9
(c) π (d) 0 (d) Minerva Mills case
4 2
73. The first Atom bomb was 1−
3
sin A sin( A − B) dropped on Hiroshima on
66. If = , then the
sin C sin( B − C ) (a) August 6, 1945 11 13
sides of ∆ABC are in: (b) August 9, 1945 (a) (b)
13 15
(a) AP (b) GP (c) August 9, 1946
(c) HP (d) None of these (d) August 6, 1942 13 4
74. The world’s largest island is (c) (d)
67. What is the principal value of 11 7
(
cosec–1 − 2 ? ) (a) Greenland
(b) Madagascar
81. A factory buys 7 machines.
2 Machine A, 2 Machine B
π π (c) New Zealand
(a) (b) and rest Machine C. Prices of
4 2 (d) Sri Lanka
the machines are ` 95000, `
π 75. The popular search engine
75000 and ` 43000 respectively.
(c) − (d) 0 “Google” derives its name from
4 Calculate the average cost of
the word “Googol”. What does
68. The equation x2 – 2xy + y2 + 3x + 2 the word mean? these machines.
= 0 represents a/an: (a) To search (a) 71000 (b) 67000
(a) parabola (b) hyperbola (b) To index (c) 75000 (d) 65500
(c) To crawl 82. A hexagonal prism has 18 edges.
(c) ellipse (d) circle
(d) The numeral one followed by How many vertices does it have?
69. If tan A + tan B + tan C = tan A
a hundred zeros (a) 8 (b) 12
tan B tan C, then (A + B + C) will
not be equal to: 76. The vitamin which is very labile (c) 20 (d) 22
and easily destroyed during 83. A rice trader buys 12 quintals
π
(a) 0 (b) cooking as well as storage is of rice for ` 1,870. 15% rice is
2 vitamin
(c) π (d) 2π lost in transportation. At what
(a) D (b) C
70. Let A = (aij)n × n and adj (A) = [αij]. rate should he sell to earn 20%
(c) B 6 (d) K
1 2 3 profit?
If  4 5 4  , then what is the value 77. Which country has placed
7th to implement an Central (a) ` 110.3 per quintal
 2 3 −1
Command Centre for Air Traffic (b) ` 187 per quintal
of α23?
Flow Management (C-ATFM) (c) ` 220 per quintal
(a) 1 (b) – 1
system? (d) ` 224 per quintal
(c) 8 (d) – 8
(a) Japan
84. lf a:b = 3:8, find the value of
REASONING AND GENERAL (b) Brazil
(5a–3b)/ (2a + b).
AWARENESS (RAGA) (c) South Africa
(d) India (a) 9/14 (b) 14/9
71. Debenture holders of a company
78. The simplest form of 3774/2958 (c) –9/14 (d) –14/9
are its
is: 85. Two cars travel from city A to
(a) Shareholders
(a) 43/19 (b) 37/29 city B at a speed of 30 and 44 km/
(b) Creditors
(c) 31/13 (d) 31/23 hr respectively. If one car takes
(c) Debtors
79. The G.C.D. of 1.08, 0.36 and 0.9 3.5 hours lesser time than the
(d) Directors
is: other car for the journey, then
72. Which one of the following
judgements stated that ‘Secula- (a) 0.03 (b) 0.9 the distance between city A and
rism’ and ‘Federalism’ are the (c) 0.18 (d) 0.108 city B is :
Practice Set-13 161

(a) 330 km (b) 396 km (a) SK (b) Sk 94. (i) Treadmill (ii) Treason
(c) 495 km (d) 264 km (c) Rk (d) RI (iii) Treacherous (iv) Tread
86. If Twinkle says Riya’s father Direction (91): Read the following (a) ii, iii, iv, i (b) iii, iv, ii, i
Ravindra is the only son of my information carefully and answer the (c) iii, iv, i, ii (d) i, ii, iii, iv
father-in-law Mahendra, then questions given below:
how is Priyanka, who is the sister Direction (95): Four of the following
(i) A, B, C, D, E, F and G are sitting five are alike in a certain way and so
of Riya related to Mahendra?
around a circle and are facing the form a group. Which is the one that
(a) Daughter does not belong to that group?
centre.
(b) Daughter-in-law
(ii) G is second to the left of C, who is 95. (a) Red (b) Blue
(c) Wife
to the immediate left of F. (c) Yellow (d) Colour
(d) Cannot be determined
(iii) A is third to the left of E. Direction (96-99): A series is given
87. In a coding system, PEN is with one term missing. Choose the
(iv) B is between D and E.
written as NZO and BARK correct alternative from the given
as CTSL. How can we write 91. Which of the following pairs
ones that will complete the series.
PRANK in that coding system? has the first person sitting to
96. Arctic Circle, Tropic of Cancer,? ,
(a) NZTOL (b) CSTZN the immediate left of the second
Tropic of Capricorn
(c) NSTOL (d) NTSLO person?
(a) Temperate zone
88. A leopard is chasing a prey. The (a) BE (b) CA
(b) Equator
leopard runs 2 km South, then (c) GD (d) DG
turns West and runs 2.5 km, (c) Antarctic Circle
Direction (92): In following questions,
then turns North and runs 1.5 (d) Prime Meridian
D means ‘is greater than’, % means
km, then it turns to its right and 97. 730, 1001, 1332, 1729, ?
runs 2.5 km. Where is it now with ‘is lesser than’, � means ‘is qual to’,
= means ‘is not equal to’, + means ‘is (a) 1236 (b) 3256
respect to the starting position ?
a little more than’, × means ‘is a little (c) 2910 (d) 2198
(a) 3.5 km South
less than’. 98. ABC, ZYX, DEF, WVU, ?
(b) 0.5 km North
Choose the correct alternative in each (a) GHI (b) IJK
(c) 0.5 km South
of the following questions. (c) MN0 (d) EFG
(d) 3.5 km North
92. If a D b and b + c, then 99. GB, ID, LG, ?
89. In a row of boys, Jeevan is 7th
from the start and 11th from (a) a % c (a) NO (b) BA
the end. In another row of boys, (b) c % a (c) PK (d) XY
Vikash is 10th from the start and (c) c + a Direction (100): Identify the diagram
12th from the end. How many that best represents the relationship
(d) Can't say
boys are there in both the rows among the given classes.
together? Direction (93): Select the related 100. Mathematics, Arithmetic,
(a) 38 word/letters/number from the given Algebra
(b) 40 alternatives.
(c) 32 93. School : Teacher : : Hospital : ?
(d) Cannot be determined (a) Clerk (b) Doctor (a) (b)
Direction (90): A series is given (c) Student (d) Patient
with one term missing. Choose the
Direction (94): Arrange the given
alternative from the given ones that (c) (d)
words in the sequence in which they
will complete the series.
occur in the dictionary.
90. Gw, Jt, Np, ?
162 Practice Set-13

Answers with Explanation


ENGLISH 13. (b) 'Fell out' means to quarrel, 23. (b) For m g mass of the gas
which would be the correct use here.
1. (a) 2. (c) 3. (b) 4. (b) The two brothers fell out over property PV =
5. (b) The correct spellings is: relevant. dispute. Hence option (b) is correct.
6. (c) The correct spellings is: weird. 14. (d) Set up' would only be used or, m=
7. (b) The word Apparent (Adjective) here, which means to begin or put into
means: easy to see or understand; obvious. place. He has set up a new business
Look at the sentence: establishement. mA =
It was apparent from his face that has 15. (d)
was really upset. 16. (d) Underlined clause in the mB =
Of the given alternatives, the word sentence the teacher told the students that
Hidden (Adjective) means : secret, their result was quite good is noun clause.
concealed. A noun clause in a group of words which =
Hence, the words Apparent and Hidden contains a subject and a predicate of its
are antonymous. own and does the work of noun.
8. (a) The word Alien (Adjective) 17. (c) The correct indirect speech =
means: hostile; from another country or will be 'He asked me where I was going
society: foreign. for my holiday's. 24. (d) The beam of X-rays will
Look at the sentence:
18. (c) Dwelt is the past form of neutralise the charge on gold leaf and
We should not accept an alien culture.
Of the given alternatives, the word the verb 'Dwell' is used without object. hence they will collapse.
Native (Adjective) means: indigenous Hence option (c) is correct.
connected with the place where you 19. (a) In group of words 'A moment 1
25. (b) 1 Wbmin–1 = V
were born and lived for the first years delay' the word 'moment' is a Noun, while 60
of your life. it will be proper to use its Adjective form.
Hence, A momentary delay/ A moment's 26. (c) Maximum distance covered
Look at the sentence:
It is a long time since he has visited his delay ..... will be a correct sentence. by space wave communication:
native country. 20. (d) d = 2rh
Hence, the word Alien and Native are
antonym PHYSICS = 2 × 6.4 × 106 × 300
9. (d) Pentience means "a person
who repents of sin". 21. (b) mc2 = hv = 62 km
Submission means "an act of giving a hv Since receiver-transmitter distance
document proposal or piece of writing, ⇒ p = mc =
c is 100 km, this ruled for both (a) and (b).
etc. to someone so that it can be
considered or approved". 6 × 10 –20 Further, fc for ionospheric
∴ p=
Retribution means "Punishment for doing 3 × 108
propagation is:
something wrong. = 2 × 20–28 kg ms–1
Confinement means ''Punishment for fc = 9 × (Nmax)½
22. (d) The poynting vectors
doing something or animal is kept = 9 × (1012)½
somewhere usually by force''  1  
Repentance means "regret for any past S =
μ0
(
E×B )
= 9 MHz
action" Hence, is the suitable synonym So, the (a) signal of 5 Mhz (< fc)
of Repentance. iˆ ˆj kˆ omes via ionospheric mode and the (b)
10. (b) Gift of the Gab means "the 1
= 80 32 –64 signal of 100 MHz comes via satellite
ability of speak, a way with words". 4π × 10 –7
0.2 0.08 0.29 mode.
11. (b)
12. (a) The passive form of 'How  T2 
2
 r2 
3

do people learn languages's How are 27. (c)   =   = (9)3


languages learnt? = 11.52iˆ – 28.8 ˆj T
 1  r1 
Practice Set-13 163

T2 = T1(9)3/2 – 27 T1 2qE 2000 25 × 10 –3


x0 = = –
k 2.5 2.5 × 10 –3
μ0I
28. (d) B1 = ⊗
r 2qE = 800 – 10
2π   Separation, = L+
2 k = 790 Ω
33. (b) Mutual inductance of the pair 3R1
μ0I ∴ = 790
B2 =  of coils depends on distance between two 2
r coils and geometry of two coils.
2π   790 × 2
2 34. (d) We know that, ∴ R1 =
3
So, Bnet = 0
gx 2 = 526.67 Ω
y = x tan θ –
2u cos 2 θ2
36. (a) Quantity CR is termed as
1 2 Comparing it with the given basic constant which is a certain time.
equation, Hence [M0L0T–1].
1 2 37. (b) Let R be the radius of a long
y= 3x – gx
2 thin cylindrical shell.
To calculate the magnetic induction
we get tan θ = 3 ⇒ θ = tan –1 3
at a distance r (r > R) from the axis of
35. (a) Let resistance per unit length cylinder, a circular shell of radius r is
of potentiometer is: shown.
1 10 mV
29. (c) vd =
neA R=
3 R1
2
I I I
5
⇒ vd= R' 40 λ R" = 60 λ
r
(5 × 10 ) (1.6 × 10 –19 ) (4 × 10 –6 )
26

1
⇒ vd = ms–1
64
2V
30. (d) Since adiabatics are steeper R
i.e., have greater slope than isothermals. 10 1
λ= = Ω / cm Since, no current is enclosed in the circle
100 10
1 1  so, from Ampere’s circuital law, magnetic
31. (b) n1 – n2 = v  –  induction is zero at every point of circle.
 λ1 λ2  1
∴ 10 mV = 40 λI = 40 × I = 4I Hence, the magnetic induction at any
10
 1 1  point inside the infinitely long straight
= v –  ∴ I = 2.5 × 10–3 V thin walled tube (cylindrical) is zero.
 1.0 1.01 
10 × 10 –3 38. (c) Normal temperature of human
0.01 or λI = body = 98.4ºF
or, 4 = v. 40
1.01
= 0.25 × 10–3 K – 273.15 F – 32
⇒ v = 404 m/s =
According to loop rule. 5 9
32. (a) At the maximum separation, 2 – IR– 100 λI = 0
velocity of point mass is zero. 98.4 – 32 66.4
or 2 – IR – 100 × 0.25 × 10–3 = 0 = =
9 9
W = ∆K or IR = 2 – 0.025
5
1 2 1 K = 273.15 + × 66.4
∴ WE + WS = mv – mu 2 = 0 2 – 0.025 9
2 2 ∴ R=
I
1 = 273.15 + 36.9
qEx0 – kx 20 = 0 2 – 0.025
2 = = 310.05
2.5 × 10 –3
164 Practice Set-13

39. (a) Unknown frequency 1 + 7i (1 + 7i )(3 + 4i )


X = 100 ± 2 ⇒ =
= =
3 − 4i (3) 2 − (4i ) 2
i.e., 102 or 98
−25 + 25i
On loading it, the possible ⇒ RP = = = –1 + i
25
frequencies are
100 ± 1 43. (c) p= 2mE, Since, 1 + i = ( 2, π / 4)
i.e., 101 or 0.99
109E – 1 + i = ( 2, π − π / 4)
p1 = 2m.
Since on loading with wax, the frequency 100
decreases, the frequency before loading  3π 
=  2, 
with wax should be 102, which reduced 109  4 
= 2m E
to 101 on waxing. 100 48. (a) Here, T6 + 1 = 9Cr 39 – r (ax)r
40. (b) For tetragonal crystal p Coefficient of x2 = 9C237a2
109
or, = Coefficient of x3 = 9C3 36a3
a = b ≠ c and α = β = γ = 90º p1 100
v Given 9
C2 37a2 = 9C336a3
41. (a) µ= p – p1 109 –10
E ∴ × 100 = × 100 9×8 9×8× 7
p 10 ⇒ (3) = a
velocity of current carries 1× 2 1× 2 × 3
= = 0.045 × 100
Intensity of electric field 9
= 4% ⇒ a=
F s 44. (b) On immersing in water f 7
E= ,v=
q t increases and hence P decreases. 49. (a) Since,a, b, c are in GP
45. (c) Let the lattice parameter be a, ∴ b2 = ac
s q m-coul
∴ . = then Taking log on both sides, we get
t F s-newton 2 log b = log a + log c
a3 = 10–30 m3
m . coul . s 2 coul-s ⇒ a = 10–10 m log b log a log c
= = ⇒ 2 = +
s kg m kg log n log n log n
42. (a) MATHEMATICS
2 1 1
3 2 2 ⇒ = +
1+ x (1 + x ) log b n log a n log c n
8 4 46. (d) f (x2) = 2 =
1− x (1 + x)(1 − x) Hence, loga n, loga n and logc n are
A B 2 in HP.
2 6 Now, f ( x). f ( x ) 50. (b) Given that:
1 + f ( x) 2
6 (1 + x)n = a0 + a1x + a2x2 +.........+
(1 + x) (1 + x 2 ) anxn
3 ×
(1 − x) (1 + x)(1 − x) Put x = 1,
=
1·6 2·4 (1 + x) 2 (1 + 1)n = a0 + a1 + a2 +..........+ an
A B 1+
(1 − x) 2 ⇒ a0 + a1 + a2 +..........+ an = 2n
6
(1 + x) 2 51. (d) Given equation, x2 – 10x + 9
=0
3 (1 − x) 2
= Let (α, β) be the roots of the given
4
(1 + x) 2 + (1 + x) 2
A B equation
(1 − x) 2
6
Then, α + β = 10
1 + x2 (1 + x 2 ) 1 and α.β = 9
3, 4 and 6 Ω all in parallel. Hence, = = =
2 + 2 x2 2(1 + x 2 ) 2 Now, we use the identity
1
= 1 + 7i 1 + 7i (α – β)2 = (α + β)2 – 4αβ
Rp 47. (c) = = (10)2 – 4(9) = 100 – 36 = 64
2
(2 − i ) 4 + i 2 − 4i
Practice Set-13 165

⇒ α–β=+8 2 1 3 1 4 1 For n = 1, LHS = cos θ


∴ |α–β|=8 S= . + . + . + .......
1 3 2 9 3 27 sin 2θ
52. (c) Put x = cos θ, and RHS = = cos θ
n +1 1 2sin θ
 + . + ...∞ ∴ P(1) is true
cos θ  1 − cos θ   r 3n
y = tan −1 + sin  2 tan −1  
1 + sin θ   1 + cos θ   Let P(k) is true, then
n +1 1  1 1
sin φ   θ  Where, T = = 1 +  n P(k) = cos θ cos 2θ cos 4θ........
−1 −1
= tan 1 + cos φ + sin  2 tan tan  2   n 3n  h3
   sin 2k θ
1 1 cos(2k – 1 θ) = k
π = + 2 sin θ
(where, φ = − 0 ) 3 n
n.3n
2 For n = k + 1
= tan–1 tan(φ/2) + sin(2 . θ/2) 1 1 P(k + 1) = cos θ cos 2θ cos 4θ........
∴ S = ΣTn = Σ +Σ
= (φ/2) + sin θ 3n n . 3n cos(2k θ)
= π/4 – θ/2 + 1 − cos 2 θ 1 sin 2k +1 θ
  1  =
π 1 =
3
+ − log e 1 −   2k +1 sin θ
−1
= − cos x + (1 − x )
2
1   3 
4 2 1− L.H.S. = cos θ cos 2θ cos 4θ........
3 cos(2k – 1 θ) cos 2kθ
dy 1 1
+
1 1
(−2 x)
∴ =2
dx 2
(1 − x ) 2 (1 − x 2 ) = sin 2k θ
= .cos(2k θ)
1 − 2x 2k sin θ
= 56. (d) We have x2 + y2 + 4x + 6y +
2 (1 − x 2 ) 2sin(2k θ) cos(2k θ)
13 = 0 =
x x ⇒ (x2 + 4x + 4) + (y2 + 6y + 9) = 0 2k +1 sin θ
53. (c) Let I = ∫ a e dx
⇒ (x + 2)2 + (y + 3)2 = 0 sin 2k +1 θ
= = RHS
 d   ∴ r=0 2k +1 sin θ
= ax ∫ e x dx − ∫  a x  ∫ e x dx  dx
  dx   Hence, given equation represents a ∴ For n = k + 1, P(n) is true.
x x point. Hence, by principle for mathematical
= a .e –x x
∫a .log a.e dx
57. (a) We are given that: induction for all n ∈ N, P(n) is true.
I = ax.ex – log a ∫ a x e x dx 2loge N = p  ....(i) dy dv
59. (a) Put xy = v ⇒ y + x =
I = ax.ex – log a I log2 2N = q ....(ii) dx dx
⇒ I(1 + log a) = axex and q–p=4 ....(iii)
dv φ(v)
I(log e + log a) = axex From eq. (i) 8p = N2 ∴ =x
dx φ '(v)
⇒ I. log ae = axex + C ⇒ 23p = N2 ....(iv)
From Eq. (ii), 2N = 2q φ '(v)
a xex ⇒ dv = xdx
⇒ I= +C φ (v )
log(ae) 2q
⇒ N= = 2q−1 ....(v) x2
54. (c) Given equations of two lines 2 ⇒ log φ (v) = + log k (On
integrating) 2
are From Eq. (iv) and (v)
2x + 3y + 4 = 0 ...(i) 23p = (2q – 1)2 φ (v ) x2
and 4x + 3y + 2 = 0 ...(ii) ⇒ 23p = 22q – 2 ⇒ 3p = 2q – 2 ⇒ log =
k 2
On solving eq. (i) and (ii), the co- ⇒ 2q – 3p = 2 ....(vi) x 2
/2
ordinates of the intersecting points are ⇒ φ(v) = ke ⇒ φ(xy)
On solving eq. (iii) and (vi), we get 2
(1, –2) q = 10 = ke x /2

Now, (0 − 1) 2 + {0 − (−2)}2 = d From Eq. (v). N = 210 – 1 = 29 = 512 60. (d)


58. (a) Let P(n) = cos θ cos 2θ cos Average deposit f Cumulative
⇒ d= 1+ 4 = 5 4θ..........cos (2n – 1 θ) Balance (`) frequency (c f)
55. (a) Let sin 2n θ
= ....(i) Less than 100 26 26
2n sin θ
166 Practice Set-13
4
100–200 68 94 ⇒ P(E1 ∩ E2) = sin A sin( B + C )
52 ∴ =
200–300 145 239 ∴ Probability of getting a diamond sin C sin( A + B )
or a king is
300–400 242 481 sin A sin( A − B)
P(E1 ∪ E2) = P(E1) + P(E2) – P(E1 But = (Given)
400–500 188 669 ∩ E2) sin C sin( B − C )

500–600 65 734 13 4 1 sin( B + C ) sin( A − B)


= + − ∴ =
52 52 52 sin( A + B ) sin( B − C )
600–700 16 750
16 4 { sin (A + B) sin (A – B) = sin2
N 750 = =
= = 375 52 13 A – sin2 B}
2 2 ⇒ sin2 B – sin2 C = sin2 A – sin2 B
63. (c) The total number of 9-digit
The frequency just greater than 375 ⇒ b2 – c2 = a2 – b2
numbers, having all digits are different
is 481. ⇒ 2b2 = a2 + c2
= 9 × 9P8
∴ Median class is 300-400 9 × 9! So, a, b and c are in A.P.
N  = = 9 × 9! 67. (c) cosec –1 (− 2) = – cosec –1
1!
 − c
2 (− 2)
Median = l + ×h ( In First place we select anyone of
f  π π
nine numbers except zero. In rest of = – cosec–1  cosec  = −
 4  4
375 − 239 the eight places, we select any eight
= 300 + × 100
242 numbers from remaining 9 numbers). 68. (a) For x2 – 2xy + y2 + 3x + 2 = 0
= 300 + 56.2 = 356.2 64. (c) (a2x2 – 2ax + 1)51 D = ab – h2 = 1 × 1 – (–1)2
dy  π  π  = [(1 – αx)2]51 = (1 – αx)102 =1–1=0
61. (b) Now, =  − sin  − 2  = A0 + A1x + A2x2 +...(say)
dx  x  x  and abc + 2fgh – af2 – bg2 – ch2 ≠ 0
π π Put x = 1, we get
sin Hence, conic represent a parabola
= 2 (1 – α)102 = A0 + A1 + A2 +...... = 0
x x 69. (b) tan A + tan B + tan C= tan A
(given)
tan B tan C
Since, sin θ is positive when 0 < θ ⇒ 1–α=0
< π. ⇒ α=1 ⇒ tan A + tan B
dy π 65. (c) Let = tan A tan B tan C – tan C
So, is positive when sin is
dx x = –tan C (1 – tan A tan B)
positive π/ 2 sin 3 x
π
I= ∫0 sin 3 x + cos3 x
dx ....(i)
tan A + tan B
i.e., 0< <π = – tan C
x 1 − tan A tan B
π 
sin 3  − x  ⇒ tan (A + B) = tan (– C)
⇒ 2nπ < π/x < (2nπ + π) ⇒ 2n < π/ 2  2 
1/x < (2n + 1) = ∫0 3π  3π 
dx
∴ A+B=–C
Taking reciprocal and reversing the sin  − x  + cos  − x 
2  2  ⇒ A+B+C=0
1 1
inequality, we get : <x< 3
2n + 1 2n π/ 2 cos x π
⇒ I= ∫0 sin x + cos3 x
3
dx ...(ii) ⇒ π ⇒ 2π but not
 1 1 2
∴ x∈  ,  and n ∈ N
 2n + 1 2n  On adding Eq.(i) and (ii) we get
1 2 3 
π/ 2  
62. (b) n (S) = 52 2I = ∫0 1dx 70. (c) A = 4 5 4 
Let E1 = Event of getting a π  2 3 −1
π/ 2 π
13 2I= [ x]0 = ⇒I=
diamond ⇒ P(E1) = and E2 = Event 2 4
52
66. (a) We know that:
of getting a king ⇒ P(E2) = and E1 ∩ ⇒ adj A =
sin C = sin {π – (A + B)}
= sin (A + B)
E2 = Event of getting diamond and king and sin A = sin (B + C) = [αij] = α23 = 8w
Practice Set-13 167

REASONING AND GENERAL 77. (d) India became the seventh 83. (c) Left quantity of rice
AWARENESS (RAGA) nation to implement Central Command 100 – 15
Centre for Air Traffic Flow Management           = 12 ×
100
71. (b) Companies issue debentures (C-ATFM) after the US, Europe,
instead of shares to extend their business. Australia, South Africa, Japan, Brazil. 12 × 85 1020
= =
These debentures are issue to borrow 100 100
78. (b) The simplest form of
loan from general public; interest is paid          = 10·20 quintals
3774/2958
on the borrowed money to the debenture
1887 37 120
holders. So, a debenture holder is = = S.P. at 20% profit = × 1870
essentially a creditor who simply gives 1479 29 100
loan to the company. 79. (c) Given numbers are 1.08, = 12 × 187 = ` 2244
72. (b) In the S.R. Bommai Case 0.36 and 0.90 2244
v. Union of India (1994), Juslice Selling price per quintal =
HCF of 108, 36 and 90 is 18 10·20
Sawant and Kuldip Singh observed 22440

⇒ H.C.F of given numbers = =
that federalism and secularism was an 102
essential feature of our Constitution and 0.18
= ` 220 per quintal
were a part of basic structure. In this 80. (b) Given exp.
84. (c) a : b = 3 : 8
case, the Supreme Court discussed at 1 1
= = 3
length provisions of Article 356 of the 2 2/3 ∴ a/b =
Constitution of India. 1+ 8
3 5 8
73. (a) A uranium gun-type atomic 1+ + ×3 a 3
8 3 9 5 – 3 5× – 3
bomb (Little Boy) was dropped on 5 9 5a – 3b b 8
+ = =
Hiroshima on August 6, 1945. It was 3 (1/ 3) 2a + b a 3
2 +1 2× +1
followed by a plutonium implosion-type b 8
1 1 13
bomb (Fat Man) on the city of Nagasaki = = = 15 – 24 –9
2/3 2 15 = =
on August 9, 1945. The twin bombings 1+ 1+ 6+8 14
led to Japan’s surrender in the Second (13/ 3) 13
War. 81. (b) Average cost of machines 85. (a) Let time for slow car be
74. (a) Greenland is the world’s t hours and time for fast car (t – 3.5)
95000 × 2 + 75000 × 2 + 43000 × 3 hours
largest island covering 2,175,597 =
2+2+3 Distance between two places will
square kilometers. New Guinea and
Borneo come next. Greenland is located be equal
190000 + 150000 + 129000
between the Arctic Ocean and the North = 30 t = 44(t – 3.5)
7 or 44(t – 3.5) = 30t
Atlantic Ocean, northeast of Canada and
469000 44t – 30t = 44 × 3.5
northwest of Iceland. = = ` 67000
7 t(44 – 30) = 44 × 3.5
75. (d) Google derived its name
82. (b)Hexagonal prism 44 × 3.5
from the word “googol”, a term coined t=
by the nine-year-old Milton Sirotta, 14
nephew of the American mathematician = 22 × 0.5
Edward Kasner. The story goes, Kasner = 11 hours
had asked his nephew to invent a name Distance between A and B = 30t
for a very large number - ten to the = 30 × 11 = 330 km
power of one hundred (the numeral 86. (d) Only son of Twinkle’s
one followed by 100 zeros), and Milton father-in-law ⇒ Twinkle’s husband
called it a googol. So, Ravindra ⇒ Twinkle’s husband.
76. (b) Vitamin C is quite unstable Ravindra ⇒ Riya’s & Priyanka’s
when cooked or stored improperly. It is F + V = E + 2 father.
very liable and easily destroyed during 8 + V = 18 + 2
[ Priyanka & Riya are sisters]
cooking as well as storage. It is most V = 20 – 8
susceptible to heat destruction. So, Mahendra ⇒ Priyanka’s
= 12
grandfather.
168 Practice Set-13

Therefore, ‘Priyanka’ is grand 90. (b) So, the missing term is Equator.
daughter of ‘Mahendra.’ G W J t N p S k 97. (d)
87. (c) As, 7 23 10 20 14 16 19 11 730 → 1001 → 1332 → 1729 →
P E N 2198
+4 +5
↓ ↓ ↓ –3 –4 –5 (9)3 + 1 → 729 + 1 → 730
(10)3 + 1 → 1000 + 1 → 1001
N Z O Finally, the missing term is Sk. (11)3 + 1 → 1331 + 1 → 1332
And 91. (d) D (first person)  Immediate (12)3 + 1 → 1128 + 1 → 1729
B A R K left to the G (second person)
(13)3 + 1 → 2187 + 1 → 2198
↓ ↓ ↓ ↓ Therefore, required pair = DG Finally, the missing term is 2198.
C T S L 92. (b) According to the given
Therefore, 98. (a)
information and b + c  b is a little
P R A N K more than c –3
–3
–3
↓ ↓ ↓ ↓ ↓ From (i) and (ii), we get A B C Z Y X D E F W V U G H I
1 2 3 26 25 24 4 5 6 23 22 21 7 8 9
N S T O L a>c
+3
88. (c) +3

c<a +3
+3
Start +3
End N 
c % a. +3
Finally, the missing term is GHI.
1.5 km 2 km W E 93. (b) As, teacher is related to
99. (c) G B I D L G P K
school. Similarly, doctor is related to 7 2 9 4 12 7 16 11
S
2.5 km hospital.
+2 +3 +4
the required distance = 2 – 1.5 94. (c) According to dictionary, the +2 +3 +4
= 0.5 km sequence of words is: Finally, the missing term is PK.
Hence the leopard will be 0.5 km (iii) Treacherous (iv) Tread 100. (d) Arithmetic and Algebra are
south with respect to its starting position. the branches of Mathematics. So, the
(i) Treadmill (ii) Treason
89. (a) Total number of boys best relationship among them is:
95. (d) Except colour, all others are
= (Number of boys in Jeevan's row) names particular colour. Mathematics
+ (Number of boys in Vikash's row)
96. (b) The series is, Arctic Circle, Arithmetic Algebra
= (6 + 1 + 10) + (9 + 1 + 11) Tropic of Cancer, Equator, Tropic of
= (17 + 21) = 38 Capricorn.
qqq
Practice Set-14 169

Air Force Airmen Group X & Y Exam


Practice Set-14

1. The principle of weight training is Directions (7-8): In the following


ENGLISH questions, choose the word opposite
(a) training muscles to exert more
Directions (1-4): Read the passage in meaning to the given word that is
given below and answer the questions pressure
your answer.
that follow. (b) disposing extra workload 7. DILIGENT
Our body is a wondrous mechanism (c) thickening of body through extra (a) confident (b) hardworking
and when subjected to unusual stress consumption (c) lazy (d) shy
over a period of time, it adapts itself to (d) helping the body to adapt 8. STEADFAST
deal more effectively with that stress. increased stress (a) staunch (b) feeble
Therefore, when you exert your muscles 2. Weight training makes the muscles (c) faint (d) wavering
against resistance, they are forced to (a) resist workload Direction (9): In the following question,
adapt and deal with this extraordinary (b) thicker and stronger four alternatives are given for the
workload. This is the principle of weight idiom/phrase printed in bold in the
(c) become stranded
training. Strands of muscle fibres become sentence. Choose the alternative which
(d) become intense best expresses the meaning of he idiom/
thicker and stronger in response to the
3. During weight training, the heart phrase that is your answer.
demands placed on them.
pumps 9. To cast aspersions
One of the great merits of weight
(a) more blood (a) to act as the pillar of support
training is the strength of your heart.
(b) required blood (b) to raise aspirations
During weight training, your heart is
(c) an extraordinary amount of blood (c) to make unpleasant remarks
forced to beat faster and stronger in (d) to dissolve all the differences
order to pump sufficient blood to the (d) less blood
4. A stronger and more efficient heart 10. In the following question, the
muscles being worked. In time, your 1st and the last sentences of the
heart, like your body, will adapt to this (a) needs fewer beats to pump the
passage are numbered 1 and 6.
extra-workload by becoming stronger same amount of blood
The rest of the passage in split
and more efficient. Since, your body (b) can rest longer, reducing its into four parts and named P, Q,
workload R and S. These four parts are not
needs a given amount of blood to
(c) is assisted by muscles of the body given in their proper order. Read
perform its daily tasks, your heart will
(d) beats faster and more often to the sentence and find out which of
now need fewer beats to pump the same
pump blood the four combinations is correct.
quantity of blood. Sounds good? There's
Directions (5-6): In the following Then find the correct answer.
more. Your entire circulatory system
questions, groups of four words are 1. John had some relatively new
is given a thorough workout every given. In each group, one word is clothes he had outgrown.
time you exercise, which increases its correctly spelt. Find the correctly spelt P. But his mother took them out
overall efficiency. Even the neural paths word that is your answer. and kept them neatly folded in
from your brain's command centres to 5. (a) Enterpreneur the cupboard again.
each individual muscle become more (b) Entreprenure Q. He threw them into the waste
effective, enabling easier recruitment of (c) Entrepreneur basket.
muscle fibres for carrying out physical (d) Enterprenure R. So John put the clothing into
tasks. In essence, your body becomes the family's bag of items to
6. (a) Promiscuous donate to charity.
a well-oiled and finely-tuned piece of
(b) Promescuous S. His mother found them
machinery, whirring along without any
(c) Promiscuos and put them back in his
breakdown. In today's stress-filled world,
(d) Promiscous cupboard.
you need all the help you can get.
170 Practice Set-14

6. John finally put the items in the four alternatives suggested, select 22. A uniform force of (3iˆ + ˆj ) N acts
his mother's mending basket on a particle of mass 2 kg. Hence,
the one which best expresses the same
and never saw them again. the particle is displaced from
(a) RSPQ (b) QRSP sentence in Passive/ Active Voice that
(c) QSRP (d) RQSP is your answer. position (2iˆ + kˆ ) m to position
11. Socrates said, "Virtue is its own 16. Open the door.
(4iˆ + 3 ˆj – kˆ ) m. The work done
reward.'' (a) The door must be opened.
by the force on the particle is:
(a) Socrates said that virtue had its (b) The door will be opened.
(a) 9 J (b) 6 J
own rewards. (c) The door is opened.
(b) Socrates says that virtue is it own (c) 13 J (d) 15 J
(d) Let the door be opened. 23. A thin wire of length l and mass m
reward.
(c) Socrates said that virtue is its own 17. I did not trust anybody. is bent in the form of a semi-circle.
reward. (a) Nobody was trusted by me. Its moment of inertia about an
(d) Socrates said that virtue was its (b) Anybody had been trusted by me. axis joining its free ends will be:
own reward.
(c) Nobody would be trusted by me. z
12. He said to the interviewer, "Could
you please repeat the question?" (d) Nobody has been trusted by me.
(a) He requested the interviewer if he 18. Select the appropriate synonym of
could please repeat the question. word TRUNCATE.
(b) He requested the interviewer to (a) Cancel (b) Act cruelly
please repeat the question. (c) Cut off (d) End swiftly O
(c) He requested the interviewer to 19. Choose the opposite of the word
repeat the question.
MASTICATE. m
(d) He requested the interviewer if
he could repeat the question. (a) Conceal (b) Chew
(c) Review (d) Gobble
Directions (13-14): In the following z'
questions, some of the sentence have 20. In the following questions, out
errors and some have none. Find out of the four alternatives, choose (a) zero (b) ml2
which part of a sentence has an error. the one which best expresses the ml 2 ml 2
meaning of the given word that is (c) (d)
The number of the part is your answer. π2 2π 2
If there is no error, your answer is (d) your answer. SOLECISM
24. What is the nature of graph
i.e. No error. (a) wise saying
13. Many a student (a)/have passed (b)/ between displacement (y) and
(b) witty quip
the I.I.T. examination. (c)/ No error velocity (v) of SHM?
(c) clever argument
(d) (a) Straight line (b) Parabola
(d) grammatical error
14. On the time (a)/ of the opening (c) Hyperbola (d) Ellipse
ceremony of the theatre (b)/ a large PHYSICS 25. A mild steel wire of length 2L
crowd had assembled. (c)/ No error 21. There identical blocks of masses and cross-sectional area A is
(d) m = 2 kg are drawn by a force F stretched, well within elastic limit,
15. In the following question, out of = 10.2 N with an acceleration of horizontally between two pillars
the four alternatives, choose the 0.6 ms–2 on a frictionless surface, (figure). A mass m in suspended
one which can be substituted for from the mid point of the wire
then what is the tension (in N) in
the given words/ sentence that is
the string between the blocks B strain in the wire is:
your answer.
and C? 2L
One who is unable to pay debt
x
(a) Debtor (b) Indebted C B A F
(c) Borrower (d) Insolvent
Directions (16-17): In the following (a) 9.2 (b) 7.8 m
questions, a sentence has been given (c) 4 (d) 9.8
in Active Voice/Passive Voice. Out of
Practice Set-14 171

x2 (c) may pass through the region Earth surface. It strikes the Earth
x
(a) (b) (d) Nothing is definite with speed v0, if ve is the escape
2L2 L
31. Tw o d i f f e r e n t i s o t h e r m s velocity from Earth’s surface,
x2 x2 representing the relationship v0
(c) (d)
L 2L then is:
between pressure p and volume ve
26. A sample of gas expands from V at a given temperature of the
volume V1 to V2. The amount of same ideal gas are shown for (a) 2 :1 b) 1 : 2
work done by the gas is greatest masses m1 and m2, then: (c) 1: 2 (d) 2 : 1
when the expansion is: P 35. A beam of α-particles having
(a) adiabatic specific charge 2.5 × 107 Ckg–1
(b) isobaric moves with a speed of 2 × 105 ms–1
(c) isothermal
(d) equal in all three cases m2 in a magnetic field of 0.05 T. What
m1 is the radius of the circular path
27. If a battery is disconnected from
the capacitor and then a dielectric V described by the α-particle?
substance between two plates (a) 4 cm (b) 8 cm
(a) m1 > m2
of condenser is introduced the (c) 16 cm (d) 32 cm
(b) m1 = m2
capacity potential and potential 36. T h e re l a t i o n b e t w e e n t h e
(c) m1 < m2
energy respectively. phase difference ∆φ and the
(d) Nothing can be predicted
(a) increases, decreases, decreases path difference ∆x between two
32. When a glass rod rubbed with
(b) decreases, increases, increases interfering wave is :
silk is brought near the gold leaf
(c) increases, increases, increases electroscope, the leaves diverge.  2π 
(a) ∆φ =   ∆x
(d) decreases, decreases, decreases What is the charge on the leaves?  λ 
28. Electromagnets are made of soft (a) Negative  λ 
iron because soft iron has: (b) ∆φ =   ∆x
(b) Zero  2π 
(a) low retentivity and high coercive (c) Positive  λ 
force (c) ∆φ =   ∆x
(d) Either positive or negative  4π 
(b) high retentivity and high 33. A metal rod of the length 10 cm
coercive force (d) ∆φ = 2 (2π) ∆x
and rectangular cross-section of
(c) low retentivity and low coercive 37. The rays of light incident normally
1 cm × 1/2 cm is connected to a
force upon the surface placed at a
battery across opposite faces. The
(d) high retentivity and low coercive distance 5 m from a lamp and
resistance will be:
force illumination density is 5 lux, what
29. The current in a coil changes from (a) maximum when the battery is
will be the distance of surface
connected across 1 cm × 1/2 cm
+ 10A to – 2Å in 3 ms. What is the from the lamp to get illumination
faces
induced emf in the coil ? The self- density 4 times more than earlier?
inductance of the coil is 2 mH ? (b) maximum when the battery is
6 25
(a) 8 V (b) 4 V connected across 10 cm × 1 cm (a) m (b) m
faces 25 6
(c) 0.8 V (d) 0.4 V (c) 2.5 m (d) 5 m
 (c) maximum when the battery is
30. An electric field E and magnetic 38. Outside a nucleus:
 connected across 10 cm × 1/2
field B exist in a region. If these (a) neutron is stable
fields are not perpendicular to each cm faces
(b) neutron is unstable
other, then the electromagnetic (d) same irrespective of the three (c) proton and neutron both are
wave: faces stable
(a) will not pass through the region 34. A body is dropped from a height (d) proton and neutron both are
(b) will pass through the region Re (radius of Earth) above the unstable
172 Practice Set-14

39. Which one of the following 44. The phenomena involved in less than one unit which of the
statement is false? the reflection of radiowaves by following is false?
(a) Pure Si doped with trivalent ionosphere is similar to: (a) R is reflexive
impurities gives a p-type (a) reflection of light by a place (b) R is symmetric
semiconductor mirror (c) R is transitive
(b) Majority carriers in a n-type (b) total internal reflection of light (d) R is not an equivalence relation
in air during a mirage
semiconductor are holes 49. There are 12 points in any plane,
(c) dispersion of light by water
(c) Minority carriers in a p-type out of which 5 points are in
molecules during the formation
semiconductor are electrons of a rainbow straight line, rest 7 points are
(d) The resistance of intrinsic not in straight line. How many
(d) scattering of light by the
semiconductor decreases with triangles are formed from these
particles of air
increase of temperature 45. An AC source is connected with points?
40. What fraction of the surface a resistance R and an uncharged (a) 240 (b) 210
area of earth can be covered to capacitance C in series. The (c) 200 (d) 180
establish communication by one potential difference across the th
50. If p term of an arithmetic
geostationary satellite? resistor is in phase with the initial 1
progression = and qth term = ,
potential difference across the q
1 1
(a) (b) capacitor for the first time at the
2 3 1
instant (assume that at t = 0, emf then its (pq)th term will be:
1 1 p
(c) (d) is zero)
4 8 (a) 1 (b) 0
π 2π
41. Consider a beam of electrons (a) (b) (c) pq – 1 (d) – 1
(each electron with energy E0) 4ω ω
51. If second term of a geometric
incident on a metal surface kept π 3π
(c) (d) series is 2 and sum of its infinite
in an evacuated chamber. Then 2ω 2ω terms is 8, its first term will be:
(a) no electrons will be emitted as (a) 3 (b) 2
only photons can emit electrons MATHEMATICS
(c) 1 (d) 4
(b) electrons can be emitted but all
46. If A = {a, b}, then power set of A
with an energy, E0 52. x4 + 9x2 + 25 can be factorised
(c) electrons can be emitted with any is:
as:
energy, with a maximum of E0 (a) {ab, ba}
(a) (x2 + x + 5) (x2 – x + 5)
φ (φ is the work function) (b) {aa, bb}
(b) (x2 + x – 5) (x2 – x – 5)
(d) electrons can be emitted with (c) {f, (a), (b)}
(c) (x2 + x + 5) (x2 – x – 5)
any energy, with a maximum (d) None of these.
of E0 (d) None of these
47. The values of x and y satisfying
42. An aeroplane flies 400 m due the equation  cos α sin α 
North and then 300 m due South 53. If Aa =  − sin α cos α  , then
 
and then flies 1200 m upwards. (1 + i ) x − 2 i (2 − 3 i ) y + i Aa + b equals:
+ = i,
3+ i 3− i
The net displacement is: (a) Aa + Ab
are:
(a) greater than 1200 m (b) Aa – Ab
(a) x = –1, y = 3
(b) less than 1200 m (c) Aa . Ab
(b) x = 0, y = 1
(c) 1400 m (d) None of these
(c) x = 1, y = 0
(d) 1500 m 54. If coefficients of (2r + 1)th term
(d) x = 3, y = –1 and (r + 2)th term are equal in
43. D i m e n s i o n ’s o f e l e c t r i c a l
resistance are: 48. Let A be the set of all points in expansion of (1 + x)43, then the
(a) [ML2T–3A–1] (b) [ML2T–3A–2] the space, let R be a relation in value of r will be:
(c) [ML2T–3A] (d) [ML2T3A2] A such that a1Ra2 if distance (a) 14 (b) 15
between points a1 and a2 is (c) 13 (d) 16
Practice Set-14 173

55. The value of 4 sin2 30° + 2 cos 62. The distance between the lines π /2
x y 67. The value of ∫0 sin xecos x dx is:
120° + 3 tan2 60° – 7 is: + = 1 and 3x + 4y + 8 = 0 is:
4 3
(a) 0 (b) 1 (a) 0
(a) 1 (b) 4
(c) 2 (d) – 2 (b) e – 1
4 (c) 1
1 + sin A (c) (d) 20
56. The value of is equal 5 (d) –1
1 − sin A
to: 63. If tan–1 x – tan–1 y = tan–1 A, then 68. If the mean and median of all
(a) sec A + cot A A is equal to: the terms of a table be 5 and 6
(b) cosec A + tan A (a) x – y (b) x + y respectively. What is the mode of
(c) sec A + tan A x− y x− y the series?
(c) (d)
(d) sec A + cosec A 1 + xy 1 − xy
(a) 8
57. The points of intersection of the 64. Solution of the differential
(b) 30
circles x2 + y2 = 25 and x2 + y2 – equation
(3x2y4 + 2xy)dx + (2x3y3 – x2) (c) 9
8x + 7 = 0 are:
dy = 0 is: (d) 11
(a) (4, –3) and (–4, –3)
(b) (–4, 3) and (4, 3) dx
x2 69. ∫ is equal to:
2
(c) (4, 3) and (3, 4) (a) x3y2 + =C −2 x + 3 x + 4
y
(d) (4, 3) and (4, –3)
58. If 3R = 4r, then the value of 4 1 4x − 3
x2 (a) sin −1
(cos A + cos B + cos C) will be: (b) x2y3 + =C 2 41
y
(a) 7 (b) 5
1 4x − 3
(c) 9 (d) 13 x
(b) sin −1
(c) x3y2 + =C 41 41
59. If the mean of four observations y2
is 20 and when a constant C is
1 4x + 3
added to each observation the (c) sin −1
x 2 41
mean becomes 22 the value of (d) x3y2 – =C
y
C is:
1 4x + 3
(a) –2 (b) 2 65. lim x→∞ ( x + 1 − x ) x is: (d) sin −1
2 2 41
(c) 4 (d) 6 (a) ¥ (b) 1
60. The latus rectum of the parabola 1 70. If x = a(q – sin q), y = a(1 – cos q),
(c) (d) 2
y2 + 4x + 14y – 3 = 0 is: 2 d2 y
66. A room has 3 lamp sockets. then is:
(a) 3 dx 2
From a collection of 10 bulbs,
(b) 4
6 are fused, A person selects 3 1
(c) 14 (a) −
at random and puts them in a sin 2 θ / 2
(d) None of these sockets. The probability that he
61. The eccentricity of the hyperbola 1 θ
will have the light is: (b) − cosec 2
4a 2
9x2 – y2 = 1, is: (a) 1/30
2 (b) 3/10 1 θ θ
(a) (b) 3 (c) − sec 2 cosec 2
4a 2 2
(c) 5/6
1
(c) (d) 10 (d) None of these (d) None of these
3
174 Practice Set-14

79. The L.C.M. of 148 and 185 is: ‘P ÷ Q’ means ‘P is mother of Q’.
REASONING AND GENERAL (a) 740 (b) 1480 ‘P – Q’ means ‘P is sister of Q’.
AWARENESS (RAGA) (c) 940 (d) 1880
86. Which of the following
71. How many states are there in the
2 expressions represents the
Indian Union? 80. is simplified to:
2 relationship ‘T is son-in-law of
(a) 27 (b) 28 2+ × 0.39
2 H’?
(c) 30 (d) 29
2 (a) H – L ÷ T
72. The battle of Plassey was fought 3+
3
between (b) H × F + L ÷ T
1 100
(a) Sirajudduala and Robert (a) (b) (c) H ÷ R × T
Clive. 3 111
9 9 (d) H ÷ R + T
(b) None of the options (c) (d)
(c) Mir Kasim and Robert Clive. 10 10 87. If BROTHER is coded as
(d) Mir Jafar and Robert Clive. 81. The average revenues of 9 2456784. SISTER is coded as
73. Soilless agriculture refers to consecutive years of a company 919684, what is the code for
(a) Hydroponics is ` 76 lakhs. lf the average of ROBBERS?
(b) Hygroponics first 5 years is ` 71 lakhs and that (a) 18, 15, 22, 5, 18, 19
(c) Sericulture of last 5 years is ` 83 lakhs, find (b) 4562684
(d) Inter-cropping the revenue for the 5th year.
(c) 9245784
74. Phycology is the study of (a) ` 88 lakhs (b) ` 84 lakhs
(c) ` 86 lakhs (d) ` 82 lakhs (d) 4522849
(a) Bacteria (b) Algae
(c) Fungi (d) Lichens 82. D DEF is similar to D GHI. 88. A shopper in a mart loads his
75. Barter transactions means Length of DE is 9 cm and length trolley and walks 30 m through
(a) Goods are exchanged with of the corresponding side GH is an alley which is going South,
gold. 16 cm. What is the ratio of areas then he turns to his left and
(b) Coins are exchanged for of D DEF: walks 10 m, then he turns North
goods. (a) 3:4 (b) 9:16 and walks another 10 m, then
(c) Money acts as a medium of (c) 81:256 (d) 16:9 he turns West and walks 45 m
exchange. 83. If 1 shirt is offered free on and then he turns North and
(d) Goods are exchanged with purchase of 3 shirts priced ` 500 walks 20 m. Where is he now
goods. each, then what is the effective with reference to his starting
76. Which of the following memories discount on each shirt ? position?
must be refreshed many times (a) 30 percent (b) 33 percent (a) 35 m West (b) 55 m West
per second?
(c) 20 percent (d) 25 percent (c) 35 m East (d) 55 m East
(a) ROM
(b) Dyanamic RAM 84. Find two numbers such that 89. Kavi ranked 9th from the top
their mean proportional is 8 and
(c) EPROM and 38th from the bottom in a
third proportional is 64.
(d) Static RAM class. How many students are
77. Where was the Central (a) 4 and 8 (b) 8 and 16 there in the class?
Command Centre for Air Traffic (c) 8 and 8 (d) 4 and 16
(a) 45 (b) 42
Flow Management (C-ATFM) 85. A car travels a certain distance
inaugurated recently ? at 24 km/h and comes back at 36 (c) 46 (d) 44
(a) New Delhi km/h. Find the average speed for Direction (90): A series is given
(b) Maharashtra total journey. with one term missing. Choose the
(c) West Bengal (a) 28.8 km/hr (b) 30 km/hr alternative from the given ones that
(d) Assam (c) 27.6 km/hr (d) 31.2 km/hr will complete the series.
78. The simplest form of 3565/1495 90. 17, 27, 42, 62, ?
Direction (86): These questions are
is :
based on the following information: (a) 82 (b) 87
(a) 31/13 (b) 43/19
‘P × Q’ means ‘P is wife of Q’.
(c) 23/13 (d) 31/23 (c) 81 (d) 84
‘P + Q’ means ‘P is son of Q’.
Practice Set-14 175

Direction (91): Read the following Direction (94): Arrange the given of Varsha. Dhruv is towards right of
information carefully and answer the words in the sequence in which they Deepti.
questions given below: occur in the dictionary. 96. Which of the following
(i) A, B, C, D, E, F and G are sitting 94. (i) Forehead (ii) Forensic statements is definitely true?
around a circle and are facing the (iii) Forest (iv) Foremost (a) Charu is opposite Varsha
centre.
(a) i, iv, ii, iii (b) iii, ii, iv, i (b) Deepti is to the left of Garima
(ii) G is second to the left of C, who is
to the immediate left of F. (c) i, ii, iii, iv (d) i, iv, iii, ii (c) Avinash is toward the right of
(iii) A is third to the left of E. Direction (95): Four of the following Aakah
(iv) B is between D and E. five are alike in a certain way and so (d) Moksh is sitting opposite
91. Which of the following has the form a group. Which is the one that Dhruv
middle person sitting between does not belong to that group?
the remaining two ? 97. Who is sitting opposite Dhruv?
(a) FCE 95. (a) Birds (b) Sparrow (a) Charu (b) Deepti
(b) EFB (c) Dove (d) Pigeon (c) Varsha (d) Moksh
(c) DEB (e) Parrot 98. Who is sitting opposite Deepti?
(d) None of the above Directions (96-100): Study the (a) Moksh
Direction (92): In following questions, following information carefully and (b) Charu
D means ‘is greater than’, % means (c) Varsha
answer the questions given below:
‘is lesser than’, � means ‘is qual to’,
Eight family members Dhruv, Garima, (d) Charu or Varsha
= means ‘is not equal to’, + means ‘is
a little more than’, × means ‘is a little Avinash, Varsha, Aakash, Deepti, 99. Who is next to Varsha in anti-
less than’. clockwise direction?
Charu and Moksh are sitting around
Choose the correct alternative in each a square table in such a way that two (a) Garima (b) Avinash
of the following questions. persons sit on eacth of the four sides of (c) Dhruv (d) Deepti
92. If c = a and a = b, then the table facing the centre. Members 100. Which of the following paris of
(a) b D a (b) c � a sitting on opposite sides are exactly persons has both the persons
(c) b = a (d) Can't say opposite each other. sitting on the same side with
first person sitting ot the left of
Direction (93) : Select the related Aakash and Garima are
word/letters/number from the given second person?
exactly opposite each other. Deepti
alternatives. is immediately right to Garima. (a) Aakash–Charu
93. JY : 35 : : RT : ? Dhruv and Moksh are sitting on the (b) Moksh–Charu
(a) 48 (b) 38 same side. Moksh is exactly opposite (c) Dhruv–Aakash
(c) 35 (d) 24 Avinash, who is the immediate left (d) None of the above

Answers with Explanation


ENGLISH The 'word Lazy (Adjective) means: The words Wavering (Adjective)
unwilling to work or be active; idle. means : fluctuating; unsteady; weak;
1. (d) 2. (b) 3. (b) 4. (a)
Look at the sentence: hesitating. Hence, the words steadfast
5. (c) The correct spelling is:
entrepreneur. He was not stupid, just lazy. and wavering are antonymous.
6. (a) The correct spelling is: Hence, the words diligent and lazy 9. (c) Idiom to cast aspersions
promiscuous which means: having ' many are antonymous. means: to make unpleasant remarks.
sexual partners; taken from a wide range 8. (d) The word Steadfast means: not 10. (c) 11. (c) 12. (d)
of sources. changing in your attitudes or aims; firm,
7. (c) The word Diligent (Adjective) 13. (b) Many a student certainly
unwavering.
means : showing care and effort in your gives a plural sense, but is followed by a
Look at the sentence:
work or duties. Singular Verb. Hence, has passed, should
He remained steadfast in his
Look at the sentence: be used. We can also use Simple Past
Only a diligent student can succeed. determination to bring the killers to
here.
justice.
176 Practice Set-14

14. (a) Replace 'On the time' by 'At l 29. (a)


the time'. ∴ r =
π dl 10 – (–2)
15. (d) e= L = 2 × 10 –3 × = 8V
Moment of inertia dt 3 × 10 –3
16. (d) Let the door be opened. 30. (c) The electromagnetic wave
2
17. (a) Nobody was trusted by me. mr 2 m  l  ml 2
l= =   = being packets of energy moving with
18. (c) 19. (d) 2 2 π 2π 2 speed of light may pass through the
20. (d) The word Solecism (Noun) 24. (d) The relation between y and v region.
means : a mistake in the use of language is m
in speech or writing; an example of bad v2 y2 31. (d) pV = nRT = RT
2 2 + 2 = 1. it is the equation M
manners. ωA A
Hence, the words solecism and m1 RT
grammatical error are synonymous. of an ellipse. For Ist graph, p =
M V1
25. (a) Increase in length = BO +
ρ
PHYSICS OC – BC
21. (b) The system of masses is L L
B D C
shown below x
T2 T1 O
m2
C B A F
⇒ ∆L = 2BO – 2L = 2[L + x ] 2 2 ½
m1
From the figure – 2L
F – T1 = ma ...(i)  x2 
½
V1 V2 V
⇒ ∆L = 2L 1 + 2  – 2L
and T1 – T2 = ma...(ii)  L  m2 RT
Eq. (i) gives 10.2 – T1 = 2 × 0.6 For IInd graph, p =
= x /L
2 M V2
⇒ T1 = 10.2 – 1.2 = 9N
∆L x 2 / L x2 Equating the two, we get
Again from Eq. (ii), we get 9 – T2
Strain = = = 2
= 2 × 0.6 2L 2L 2L m1 V1
26. (b) The p-V diagram for isobaric, = ⇒m∝V
⇒ T2 = 9 – 12 = 7.8 N m2 V2
 isothermal and adiabatic process of an
22. (a) Given, force F = 3iˆ + ˆj As, V2 > V1 m1 < m 2
 ideal gas is shown in graph below.
r1 = (2iˆ + kˆ)m and In thermodynamics, for same change 32. (c) The glass rod acquires positive
 charge on rubbing. When the rod is
in volume, the work done is maximum for
r 2 = (4iˆ + 3 ˆj – kˆ)m brought near the disc of the electroscope,
   the curve having maximum area enclosed
∴ s = r 2 – r1 with the volume axis. the disc will acquire negative charge and
p there will be positive charge on the far
= (4iˆ + 3 ˆj – kˆ) – (2iˆ + kˆ) end, that is, on the leaves.
1 → isobaric
  = (2iˆ + 3 ˆj – 2kˆ)m 2 → isothermal ρl
  33. (a) We know that R =
W = F. S 3 → adiabatic A
= (3iˆ + ˆj ) . (2iˆ + 3 ˆj – 2kˆ) V
V1 V2
= 3 × 2 + 3 + 0 = 6 + 3 = 9J
23. (d) For semi-circle, πr = l 27. (a) If a battery is disconnected 1 cm
from the capacitor and then a dielectric 2 1 cm
substance between the two plates of 10 cm
r condenser, so capacity potential and
(a) When the battery is connected
dr potential energy is increased at saturated
across 1 cm × 1/2 cm faces, then l = 10
point and then it will decreases.
cm, A = 1 × 1/2 cm2.
28. (d) The material suitable for
a ρ × 10
making electromagnets should have high
R1 = = 20 ρΩ
retentivity and low coercivity. 1 ×1/2
Practice Set-14 177

(b) When the battery is connected adding a small amount of pentavalent V


across 10 cm × 1 cm faces, then l = 1/2 impurity to a pure sample of ∴ R =
cm, A = 10 × 1 cm2 I
semiconductor (Ge).
ρ × 1/ 2 ρ Dimensions of

R2 = = Ω Majority charge carriers—electrons
10 × 1 20 Dimensions of V
The resistance of intrinsic =
Dimensions of l
(c) When the battery is connected semiconductors decreases with increase
across 10 cm × 1/2 cm faces, then l = 1 of temperature. [ML2 T –3 A –1 ]
=
cm, A = 10 × 1/2 cm2. 40. (b) We uses three geostationary [A]
ρ ×1 ρ satellites placed at the vertices of an = [ML2T–3A–2]
R3 = = Ω equilateral triangle, then the entire Earth 44. (b) The phenomenon involved
(10 × 1/2) 5
can be covered by the communication in the reflection of radiowaves by
GM e 1 ionosphere is similar to total internal
network, each satellite covers of the reflection of light in air during a mirage
Re 3
34. (c) V0 = globe. (angle of incidence > critical angle).
2GM e 41. (d) When a beam of electrons of 45. (d) V = V0 sin ωt (as V = 0 at
While, ve = energy E0 is incident on a metal surface t = 0)
Re
kept in an evacuated chamber, electrons VR = V0 sin ωt
Vo 1 can be emitted with maximum energy
⇒ = = 1: 2  π
Ve 2 E0 (due to elastic collision) and with any VC = V0 sin  ωt – 
 2
MV v 1 energy less than E0, when part of incident
35. (c) R= = × energy of electron is used in liberating the V and VR are in same phase. While
Bq0 B q0 / M VC lays V (or VR) by 90º.
electrons from the surface of metal.
= 2 × 105 / 0.05 × 2.5 × 107 Now, VR is in same phase with initial
42. (a) As given in question,

= 0.16 m = 16 cm potential difference across the capacitor
OA = 400 m (due North) for the first time when
2π AB = 300 m (due South)
36. (a) ∆φ = × ∆x π 3π
λ North N ωt = – + 2π =
A 2 2
37. (c) Here, r1 = 5m, r2 = ? 1200 m
(upwards) W E 3π
300 m

E1 = 5 lux, E2 = 4 × 5 lux C B ⇒ t=
4000 m

S 2ω
1 E r2
E ∝ ⇒ 1 = 22
100 m

r 2
E 2 r1
O MATHEMATICS
E 46. (d) Power set of A = {{ }, {a},
⇒ r22 = r12 × 1 ∴ OB = OA – AB = 400 – 300
E2 = 100 m {b}, {a, b}}
5 25 Also BC= 1200 m (upwards) (1 + i ) x − 2i (2 − 3i ) y + i
⇒ r22 = 25 × = 47. (d) + =i
4×5 4 In right ∆OBC, OC2 = OB2 + BC2 3+i 3−i
5 ∴ Displacement, ⇒ b2
∴ r2 = = 2.5 m (3 − i )[(1 + i ) x − 2i ] + (3 + i )
2

OC = OB2 + BC 2 [(2 − 3i ) y + i ]
38. (b) Outside the nucleus, neutron Þ
is unstable. 2 2 (3 + i )(3 − i )
= (100) + (1200)

=i
39. (d) p-type semiconductor are
= 1204 m > 1200 m Þ 3x + 3xi – 6i – xi – i2x + 2i2
obtained by adding a small amount of
trivalent impurity to a pure sample of Hence, net displacement is greater + 6y – 9yi + 3i + 2yi – 3i2y
semiconductor (Ge). than 1200 m. + i2 = (9 – i2)i
Majority charge carriers—holes 43. (b) From Ohm’s law, V = IR Þ 4x + 2xi + 9y – 7yi = 9i + i +
Minority charge carriers—electrons where V is voltage, R is resistance 6i – 3i + 3
n-type semiconductor are obtained by and I is current. Þ (4x + 9y) + (2x – 7y)i = 3 +
13i
178 Practice Set-14

\ 4x + 9y = 3 and 2x – 7y = 13 1 = 43Cr + 1
\ x = 3 and y = –1 \a=
According to questions,
pq
48. (c) (i) The distance of any point = 43C2r = 43Cr +1
in a set from itself is zero i.e., less than (pq)th term = a + (pq – 1)d = 43C43 – (r + 1)

one. 1 1  hCr = hCn – r
\ R is reflexive = + ( pq − 1) × then 2r = 43 – (r + 1)
pq pq
(ii) Let a1Ra2 Þ the distance or, 3r = 42
between a1 and a2 is less than one. pq \ r = 14
= =1
\ The distance between a2 and a1 pq 2
55. (c) 4sin 30° + 2 cos 120° + 3
is less than one. 51. (d) According to question, tan2 60° – 7
2
Hence, R is symmetric. ar = 2 1 2
= 4   + 2 cos(90 + 30) + 3( 3) − 7
(iii) Let a1Ra2 and a2Ra3 Þ the 2 2
 
distance between a1 and a2 is less than
\ a=
r 1
one and the distance between a2 and a3 a = 4× + 2 × − sin 30 + 3 × 3 − 7
4
is less than one. and =8
1− r
But it is not necessary that the 1
= 1+ 2 × − + 9 − 7
distance between a1 and a3 is less than 2 2
one.
Þ r =8 =1–1+9–7=2
1− r
\ R is not transitive. 1 + sin A
49. (b) Number of obtained
Þ 8r – 8r2 = 2 56. (c)
2
1 − sin A
triangles on joining any 3 points out of
Þ 4r – 4r + 1 = 0
12 points = 12C3.
Þ (2r – 1)2 = 0 (1 + sin A)(1 + sin A)
=
Number of obtained triangles on 1 (1 − sin A)(1 + sin A)
joining 3 points out of 5 linear points \ r=
2
= 5C3. 1 (1 + sin A) 2
a× =2 =
·.· From these 5 linear points no 2 (1 − sin 2 A)
triangle will be obtained.
\ a=4
\ Required number of triangles (1 + sin A) 2
52. (a) x + 9x + 25= (x + 5)2 – x2
4 2 2 =
= 12C3 – 5C3 cos 2 A
\= (x2 + x + 5) (x2 – x + 5)
12 × 11× 10 5 × 4 × 3 53. (c) 1 + sin A
= − =
3 × 2 ×1 3 × 2 ×1 cos A
 cos(α + β) sin(α + β) 
= 220 – 10 = 210 Aa + b =  
 − sin(α + β) cos(α + β)  =
1
+
sin A
50. (a) According to questions,
cos A cos A
1  cos α cos β − sin α sin β
= a + (p – 1)d  ....(i) =  = sec A + tan A
q  −(sin α cos β + cos α sin β)
57. (d) x2 + y2 – 8x + 7 = 0
1 sin α cos β + cos α sin β  x2 + y2 – 25 = 0
= a + (q – 1)d  ....(ii) cos α cos β − sin α sin β 
p – 8x + 32 = 0
Subtracting,  cos α sin α   cos β sin β  Þ x=4
=    − sin β cos β 
1 1  − sin α cos α    \ 16 + y2 = 25
− = (p – 1 – q + 1)d
q p Þ y=+3
= Aa. Ab
\ The points of intersections of the
1 54. (a) Coefficient of (2r + 1)th term

\ d= in expansion of (1 + x)43 = 43Cr+2 circles are (4, 3) and (4, –3).
pq 58. (a) 4 (cos A + cos B + cos C)
and coefficient of (r + 2)th term =
1 1 Coefficient of
and = a + (p – 1) ×  A B C
q pq = 4 1 + 4sin sin sin 
= {(r + 1) + 1}th term  2 2 2
Practice Set-14 179

and the length of perpendicular


 r
= 4 1 +  from (0, 0) to 3x + 4y + 8 = 0. is
65. (c) lim x →∞ ( 1+ x − x ) x
 R 
3× 0 + 4 × 0 + 8 8 x
A B C or = lim x →∞
 2
3 +4 2 5
∵ r = 4R sin 2 sin 2 sin 2  1+ x + x
  \ The distance between the lines
1
 3 8  12  = lim x →∞
= 4 1 +  = −  −  = 4 1
 4 5  5 1+ +1
x
63. (c) tan–1 x – tan–1 y = tan–1 A
7 1 1
= 4   = 7  x− y  = =
4 \ tan −1 
–1
 = tan A 1+ 0 +1 2
 1 + xy 
59. (b) Total of four observations = 66. (c) Total No. of ways of
x− y
4 × 20 = 80
\ A= selecting 3 bulbs out of 10 = 10C3
1 + xy
Total of four observations after The number of ways in which the
addition C in each observation = 80 + 64. (a) Here, M = 3x2y4 + 2xy can choose a non fused bulb = 4C1 × 6C2
4C N = 2x3y3 – x2 The number of ways in which he
80 + 4C ∂M can select two non fused bulb = 4C2 ×
\ 22 =
\ = 12x2y3 +2x 6C
4 ∂y 1
and the number of ways in which he
Þ C=2 ∂N
= 6x2y3 –2x can select three non fused bulbs = 4C3
60. (b) y2 + 4x + 14y – 3 = 0 ∂x
Þ y2 + 14y + 49 = – 4x \ Number of ways of selecting at
1  ∂N ∂M  least one non fused bulb = 4C1 × 6C2 +
+ 49 + 3  − 
M  ∂x ∂y  4C × 6C + 4C
Þ (y + 7)2 = –4(x – 13) 2 1 3
\ Latus rectum = 4 −(6 x 2 y 3 − 2 x) − (12 x 2 y 3 + 2 x) \ Required probability
61. (d) 9x2 – y2 = 1 =
3 x 2 y 4 + 2 xy ( 4 C1 ×6 C2 ) + ( 4 C2 × 6 C1 ) + 4 C3
= 10
x2 y 2 −2(3 x 2 y 3 + 2 x)
−2 C3
or, − =1 = =
1/ 9 1 2 3
y (3 x y + 2 x) y 60 + 36 + 4 5
= =
= f (y), say 120 6
1
\ a2 =
and b2 = 1 I.F. =
9 67. (b) Let cos x = t
2 \ – sin x dx = dt
− ∫ dy 1
 b2 = a2(e2
– 1) y
= e = e–2 log y = when x = 0, t = 1 and when x = p/2,
1 2 y2
i.e.,1= (e − 1) t = 0.
9 1
Multiplying by 2 , the equation π/ 2 0
y \ ∫0 sin xecos x dx = − ∫ et dt
becomes 1
\ e=
10
1 t
 2 2 2x 
x y  3 x y +  dx
= ∫0 e dt
62. (b) + =1  y 
4 3 = [et ]10
 3 2
x 
or, 3x + 4y – 12 = 0 +  2 x y − 2  dy = 0 = (e1 – e0)
\ Length of perpendicular from (0,  y 
= e – 1
0) to the line Which is exact equation and its
68. (a) Mode = Mean – 3(Mean –
3 × 0 + 4 × 0 − 12 solution is:
3x + 4y – 12 = 0 is Median)
32 + 42  2 2 2x 
−12
∫  3x y + y  dx + ∫ 0.dy = C = 5 – 3(5 – 6)
Þ = 5 – 3 × (–1)
5 x2
Þ x3y2 + =C = 5 + 3 = 8
y
180 Practice Set-14

dx −1 1 contents. DRAM is dynamic in that,


69. (a) Let I = ∫ −2 x 2 + 3 x + 4
=
θ
.
θ unlike static RAM (SRAM), it needs
2sin 2 2a sin 2 to have its storage cells refreshed or
2 2
given a new electronic charge every few
2dx 1 θ milliseconds. DRAM stores each bit in
= ∫ 2
−4 x + 6 x + 8
= −
4a
cosec 4
2 a storage cell consisting of a capacitor
and a transistor. Capacitors tend to lose
REASONING AND GENERAL their charge rather quickly thus, the
2dx
= ∫ 2 2 AWARENESS (RAGA)
need for recharging.
 41   3 77. (a) New Delhi. The Central
  −  2 x −  71. (d) India is a federal union of
 2   2 Command Centre for Air Traffic Flow
states comprising twenty-nine states and Management (C-ATFM) at Vasant Kunj,
3 seven union territories. Telangana split New Delhi to ensure higher standards

2x −  from Andhra Pradesh to become the of safety. It was inaugurated in the
1 −1  2
= 2. sin newest — 29th — State of India on 2nd presence of the Chairman of Airports
2  41  June 2014 with Hyderabad as the shared Authority of India (AAI) Guruprasad
 
 2  capital. Mohapatra, Board Members and senior
72. (a) The Battle of Plassey was AAI officials. India became the seventh
1 4x − 3 fought between Nawab Sirajud-Daulah, nation to implement C-ATFM after the
= sin −1
2 41 the last independent Nawab of Bengal, US, Europe, Australia, South Africa,
and the British East India Company Japan, Brazil.
70. (d) x = a(q – sin q) under Robert Clive on 23 June 1757.
y = a(1 – cos q) 3565
It resulted in a decisive victory of the 78. (a) The fraction is
British East India Company over the 1495
dx Nawab and his French allies and the

\ = a(1 – cos q) 31
dθ establishment of the Company rule in ∴ The simplest form =
13
Bengal.
dy 73. (a) 79. (c) LCM of 148 and 185

Þ = a sin q
dθ 74. (b) Phycology is the scientific = 2 × 2 × 5 × 37 = 940
study of algae. Phycology algology 2 148 185
dy dy / d θ is a branch of life science and often is

Þ = 2 74 185
dx dx / d θ regarded a sub-discipline of botany. 37 37 185
It includes the study of prokaryotic
a sin θ forms known as blue green algae or 1 5
=
a (1 − cos θ) cyanobacterias. 80. (b) Given exp.
75. (d) Barter is a system of
2sin θ / 2 cos θ / 2 2 2
exchange where goods or services are = =

Þ= 2 2
2sin 2 θ / 2 directly exchanged for other goods or 2+
2
× 0.39 2+
6
× 0.39
services without using a medium of 3+ 3+
(11/ 3) 11
θ exchange, such as money. Barter, as a
= cot
2 replacement for money as the method of 2
=
exchange, is used in times of monetary 2
2+ × 0.39
d2y d  θ crisis, such as whether currency may be (39 /11)

\ 2
=  cot  either unstable or simply unavailable for
dx dx  2 2 2 2
conducting commerce. = = =
22 39 22 11
2+ × 2+ 2+
= – cosec2 θ . 1 d θ 76. (b) Dynamic Random Access 39 100 100 50
2 2 dx Memory (DRAM), the most common
2 100
kind of Random Access Memory = =
(111/ 50) 111
1 2 θ 1 (RAM) for Personal computers and
= − cosec .
2 2 a (1 − cos θ) work stations, has to be refreshed many 81. (c) Total values of 9 years
times per second in order to hold its data    = 76 × 9 lakhs = 684 lakhs
Practice Set-14 181

Total values of 5 years 2 xy Option (d) : H ÷ R ⇒ H is mother


   = 71 × 5 = 355 lakhs 85. (a) Average speed = of R.
x+ y
Total values of last 5 years 2 × 24 × 36 R + T ⇒ R is son of T.
   = 83 × 5 = 415 lakhs = (wife) (husband)
Total values of 10 years
( 24 + 36 )
H+ T+
   = 770 lakhs 2 × 24 × 36
= (mother) R+ (father)
The values of 5th years 60
   = (770 – 684) lakhs 48 × 6 (son)
  = ` 86 lakhs = = 28·8 km/hr The Blood relationship tree/chart
10
D G 86. (c) can be constructed as given below:
So, H mother of R son of T.
cm

Option (a): H – L
9 cm

82. (c)
Therefore, option (c) represents the
16

⇒ H is sister of L.
E F H I required relationship.
L ÷ T ⇒ L is mother of T.
We know that 87. (d) As,
So, H sister of L mother of T.
B R O T H E R
ar (ΔDEF) DE 2 The blood relationship tree/chart ↓ ↓ ↓ ↓ ↓ ↓ ↓
= can be constructed as given below.
ar (ΔGHI) GH 2 2 4 5 6 7 8 4
H+ L– And
(9) 2 81
= = (sister) (mother) S I S T E R
(16) 2 256
↓ ↓ ↓ ↓ ↓ ↓
Ratio of areas of triangles 9 1 9 6 8 4
T(?)
= 81 : 256 Therefore,
(unknown sex)
83. (d) Selling price of three shirts R O B B E R S
= 3 × 500 = ` 1500 So, ‘T’ is the nephew or niece of ‘H’. ↓ ↓ ↓ ↓ ↓ ↓ ↓
Option (b) : H × F ⇒ H is wife of F. 4 5 2 2 8 4 9
Selling price of four shirts
F + L ⇒ F is son of L. 88. (b)
= 4 × 500 = ` 2000 End
Start
Discount = 2000 – 1500 = 500 So, H wife of F son of L. N
The Blood relationship tree/chart 20 m
500 × 100 30 m W E
Effective % = = 25% can be construced as given below:
2000 L
– 45 m 10 m S
84. (d) Let two numbers be x and y 10 m

mean = xy H– F(+) T(?) Distance from start to end


(wife) (Husband) = (45 + 10) = 55 m
8 = xy (unknown sex)
So, T is the brother-in-law or sister- required direction = west
64 = xy ...(i) Hence, the shopper will now be 55 m
x : y : : y : 64 in-law of ‘H’.
west to his starting position.
Option (c): H ÷ R ⇒ H is mother 89. (c) Required number = (38 + 9
x y 64
       = But, x = of R. – 1) = 47 – 1 = 46
y 64 y
R × T ⇒ R is wife of T. 90. (b) The series is:
64 y So, H mother of R wife of T.
     = 17 + 10 = 27
y2 64 The Blood relationship tree/chart 27 + 15 = 42
y3 = 64 × 64 can be constructed as given below:
42 + 20 = 62
= 4×4×4 × 4×4×4 H–
⇒ y = 4 × 4 = 16 (mother) 62 + 25 =
⇒ xy = 64 Finally, the missing number is 87.
⇒ x × 16 = 64 91. (d) By analysing the circular
arrangement, it is a clear that
64 R– T(+)
⇒ x = =4 C  Between A and F.
16 (wife) (husband)
Hence, x = 4, y = 16 are the numbers. F  Between E and C.
So, ‘T’ is the son-in-law of ‘H’.
182 Practice Set-14

E  Between B and F. (i) Forehead, (iv) Foremost, 98. (b) By analysing the above
Therefore, none of the options is (ii) Forensic, (iii) Forest square arrangement, it is clear that
correct. 95. (a) Except bird, all other are opposite to
Charu ← → Deepti
92. (c) According to the given different types of birds.
information 99. (a) By analysing the above
96. (c) By analysing the above
c+ac≠a square arrangement, it is clear that
arrangement on a square table, it is clear next to
and a = b  a ≠ b that Garima  → Varsha (in
From (i) and (ii), it is clear that opposite to anticlockwise direction)
Charu ← → Deepti;
b ≠ a  b = a. 100. (d) By analysing the above
93. (b) As, Deepti t
o
→ the right of Garima; square arrangement, it is clear that
J   Y to
Aakash ⇒ to the right of Charu ⇒
10 25 → (10 + 25) = 35 Avinash → the right of Aakash; on same side
Similarly, opposite to Moksh & Charu →
not
/ on same
Moksh ← → Avinash.
R   T side.
18   20 → (18 + 20) = 38 97. (c) By analysing the above Dhruv & Aakash → not
/ on same
Finally, the missing term is 38. square arrangement, it is clear that side.
94. (a) According to dictionary, the opposite to
Varsha ← → Dhruv Therefore, none of the options is
sequence of the words is:
correct.
qqq
Practice Set-15 183

Air Force Airmen Group X & Y Exam


Practice Set-15

a simple life close to nature. The Mayan Directions (5-6): In the following
ENGLISH
priests studied the Sun, Moon, Stars and questions, there are four different
Directions (1-4): Read the following Planets. They made a calendar from what words, out of which only one word
passage and answer the questions that they learned. The year was divided into is correctly spelt. Find the correctly
follow. 18 months of 20 days each with five days spelt word.
The Mayan Indians lived in Mexico left over. The Mayan calendar was more 5. (a) consumerism
thousands of years before the Spanish accurate than the European calendars of (b) communlism
arrived in the 1500s. The Maya were the time. (c) passimism
intelligent, culturally rich people whose Around the year 800, the Maya (d) optemism
achievements were many. They had left their villages and beautiful cities, 6. (a) commencement
farms, beautiful places and cities with never to return. No one knows why this
many buildings. The Maya knew a lot of (b) estableshment
happened. They may have died from an (c) announcment
about nature and the world around them. infectious disease. They may have left
This knowledge helped them to live a (d) Committment
because the soil could no longer grow
better life than most people of that time Directions (7-8): In the following
crops. Archaeologists are still trying to
because they could use it to make their questions, choose the word opposite
find the lost secrets of the maya. They
lives more comfortable and rewarding. in meaning to the given word as your
are still one of the greatest mysteries.
Knowledge about tools and farming for answer.
1. The Maya were intelligent,
instance made their work easier and more 7. Foment
culturally rich people whose
productive. (a) repulse (b) cease
a c h i e v e m e n t s w e r e m a n y.
The Maya believed in many gods, (c) control (d) quell
‘Achievement’ here means:
including Rain God, Sun God and Corn 8. Tardy
(a) skills (b) successes (a) prompt (b) gradual
God. They built large temples to honour
(c) innovations (d) secrets (c) late (d) quick
their Gods. Skillful workers built cities
2. The Maya lived in Mexico: 9. ln the following question out of the
around these temples. It was difficult for
(a) only after the arrival of the four alternative, choose the one
them to construct these cities because
Spanish which best expresses the meaning
they had no horses to carry the heavy
(b) at the same time as the Spanish of the given as your answer.
stones they used to build with. Workers
arrived Enigma
had to carry all of the building materials
(c) only a few year before the (a) truth (b) fear
themselves. Today, many of these ancient
Spanish arrived
Mayan cities and temples are still (c) difficulty (d) puzzle
(d) thousands of years before the
standing. 10. In the following question, four
Spanish arrived
Although the cities that the Maya alternatives are given for the idiom/
3. Many Mayan cities and temples
built were beautiful and the people phrase printed in Underlined
can be found in good shape even
worked hard to build them very few of in the sentence. Choose the
today because they.........
the people lived in them. Usually, only alternative which best expresses
(a) are so well built
the priests lived in the cities. The other the meaning of the idiom/phrase
(b) are not very old
people lived in small villages in the as your answer.
(c) have been rebuilt
forest. Their houses were much simpler Sarala is always ready to eat
(d) are being taken care of
than the elaborate structures in the anyone's salt.
4. Most Maya lived in...........
cities. They lived in small huts with no (a) to be one's guest
windows. The walls were made of poles (a) the beautiful cities they built
(b) huts made of mud and leaves (b) to cook tasty dishes
covered with dried mud and the roof was (c) an infectious disease
made of grass or leaves. Most Maya lived (c) in caves in the jungle
(d) in the stone temples they built (d) to deceive someone
184 Practice Set-15

Directions: (11-12): In the following which best expresses the same sentence (a) 3.33 ×10–10 T
questions a sentence has been given in in Indirect/Direct Speech. (b) 3.33 × 10–12 T
Active/Passive Voice. Out of the four 17. He said, "Ravi, why are you (c) 10–3 T
alternatives suggested, select the one sounding so depressed today?" (d) 3 × 105 T
which best expresses the same sentence (a) He asked Ravi why did he sound 23. The four wires are made of the
in Passive/Active Voice. so depressed that day. same material. Which of these will
11. They are going to build; a new (b) He asked Ravi why he was have the largest extension when
airport near the old one. sounding so depressed that day. the same tension is applied:
(a) A new airport going to be built (c) He told Ravi why he sounded so (a) L = 50 cm, r = 0.25 mm
near the old one. depressed today. (b) L = 100 cm, r = 0.5 mm
(b) A new airport is being built near (d) He asked Ravi that why was he
the old one. (c) L = 200 cm, r = 1 mm
sounding so depressed that day.
(d) L = 300 cm, r = 1.5 mm
(c) A new airport will be built near 18. He asked me, "What time will the
24. A circular cavity is made in a
the old one. sun set tomorrow ?"
conductor. A positive charge q is
(d) A new airport is going to be built (a) He asked me what time does the
near the old one. sun set the next day. placed at the centre:
12. My watch can't be repaired by (b) He asked me what time the sun
anyone. would set tomorrow. B A
(a) No one will repair my watch. (c) He asked me what time the sun
(b) No one can repair my watch. would, set the next day.
(d) He asked me what time would the
(c) No one can't repair my watch. q
sun set the next day.
(d) No one will be able to repair my
19. Fill in the blank with the correct
watch.
preposition. (a) the electric field at A and B are
Directions (13-14): In the following
He killed two birds ____ one stone. equal
questions, out of the four alternatives
(a) by (b) at (b) the electric charge density at A =
choose the one which can be substituted
(c) of (d) with
for the given words/sentence. the electric charge density at B
20. Fill in the blank using the correct
13. Giving of special favour to one's (c) potential at A and B are equal
prepositional phrase.
kith and kin (d) All of the above
Shall we play indoors ____
(a) Favouritism (b) Solecism
sweating out in the sun ? 25. In an L-R circuit connected to a
(c) Pantheism (d) Nepotism
(a) instead of (b) other than battery the rate at which energy
14. One who collects and studies coins
(c) but for (d) because of is stored in the inductor is plotted
(a) Philatelist (b) Nuncio
(c) Bibliophile (d) Numismatist PHYSICS against time during the growth of
Directions (15-16): In the following current in the circuit. Which of
21. Neutrons, in thermal equilibrium
questions, some parts of the sentences the following best represents the
with matter at temperature of
have errors and some are correct. Find resulting curve?
T kelvin, are known to have
out which part of a sentence has an
an average kinetic energy of
error. The number of that part is the
3
answer. If there is no error, then your kT. Compute the de Broglie
2
Rate

answer is (d) i.e. No error. (a)


wavelength associated with a
15. Several prominent figures (a)/
involved in the scandal are required neutron at 300 K:
to (b)/appear to the investigation (a) 0.118 Å (b) 0.118 m
Time
committee. (c)/ No error (d) (c) 11.8 m (d) 0.118 nm
22. The transmitting antenna of a
16. According to the radio (a)/it can rain
radiostation is mounted vertically.
(b)/this evening.(c) No error (d)
At a point 10 km due north of the
Rate

Directions (17-18): In the following transmitter the peak electric field (b)
questions, a sentence has been given in is 10–3 Vm–1. The magnitude of the
Direct/Indirect Speech, Out of the four
radiated magnetic field is:
alternatives suggested, select the one Time
Practice Set-15 185
 34. Earth is moving towards a
30. In a region, an electric field E = E0 iˆ
is present. The potential of points stationary start with a speed of
(c) 100 km/s. If the wavelength of
Rate

A (a, 0, 0), B (0, b, 0), C (0, 0, c) and


D ( –a, 0, 0) are VA, VB, VC and VD light emitted by star is 5000 Å, the
respectively, then: observer in the earth will notice
Time (a) VA = VB = VC = VD a change in wavelength (speed of
(b) VB = VC light = 3 × 108 m/s):
(c) VD < VC < VB < VA (a) more than 1.67 Å
Rate

(d) (d) None of these (b) less than 1.67 Å


 1 (c) zero
31. An ideal inductor of   H is
 π (d) 0.6 Å
Time connected in series with a 300 Ω
26. A body of mass 50 kg is placed 35. If  is a unit vector. The value
resistor. If a 20 V, 200 Hz A.C.
on a horizontal table for which source is connected across the dA
coefficient of static friction µs is  . =?
combination, the phase difference dt
0.5. The minimum horizontal between the voltage and the
force required in start the motion (a) 0 (b) 1
current is:
will be: (c) π (d) π
(a) 245 N (b) 2450 N  5  4 2
−1
(a) tan   (b) tan −1  
(c) 23.5 N (d) None of these 4  5
36. Two concentric coils each of
27. In a region, steady and uniform −1  3 
 4 radius equal to 2π cm are placed
(c) tan   (d) tan −1  
electric and magnetic fields are 4  3 at right angles to each other, 3A
present. These two fields are and 4A are the currents flowing
32. Three forces
parallel to each other. A charged  in each coil respectively.
particle is released from rest F1 = (3iˆ + 2 ˆj − kˆ ) N
The magnetic induction in Wb/m2
in this region. The path of the 
F 2 = (3iˆ + 4 ˆj − 5kˆ )N at the centre of the coils will be (µ0
particle will be a: = 4π × 10–7 Wb/Am):
(a) Helix (b) Straight line  (a) 12 × 10–5 (b) 10–5
and F 3 = A ( iˆ + ˆj − kˆ )N
(c) Ellipse (d) Circle (c) 5 × 10 –5
(d) 7 × 10–5
28. n resistors each of resistance R act simultaneusly on a particle. 37. Two blocks of mass 20 kg and
are joined with capacitors of In order that the particle remains 5 kg are placed in contact on a
capacity C (each) and a battery in equilibrium the value of A smooth table as shown in figure. A
of emf E as shown in the figure. should be: horizontal force of 5 N is applied
In steady state condition, ratio of (a) –6 (b) 6 on 20 kg mass. This will exert on
charges stored in the first and last (c) 9 (d) –9 5 kg mass a force of:
capacitor is: 33. The equivalent resistance between
points A and B in the steady state is:
r0
5·0 N 20 kg
E R C R C C R 5 kg
r0
r0 C0 (a) 4 N towards left
(a) n : 1 (b) 1 N towards right
(b) (n – 1) : R
r0 (c) 5 N towards right
(c) (n2 + 1) : (n2 – 1)
(d) 1 : 1 (d) Zero
A B
29. The melting point of wax is 80ºC. r0 38. An object moving with constant
Its melting point on Fahrenheit speed is stopped in 0.25 s by
3 4
scale is: (a)r0 (b) r0 applying a force of 200 N. The
4 3
(a) 112º F (b) 144º F initial momentum of the body will
5 be:
(c) 176º C (d) 212º F (c) r0 (d) None of these
3
186 Practice Set-15

(a) 200 N-s (b) 50 N-s 43. A semiconductor X is made by


MATHEMATICS
(c) 100 N-s (d) 150 N-s doping a germanium crystal
39. The equation of state of some with arsenic (Z = 33). A second 46. A survey shows that 63% of the
gases can be expressed as semiconductor Y is made by Americans like cheese where
 a  doping germanium with indium 76% like apples. If x% of the
 P + 2  (V − b ) = RT , w h e r e (Z = 49). The two are joined end Americans like both cheese and
V
T is the absolute temperature P to end and connected to a battery apples, then:
is the pressure, V is the volume as shown. Which of the following (a) x = 39
and a, b, R are constants. The statements is correct? (b) x = 63
dimensions of ‘a’ are: X Y (c) 39 < x < 63
(a) [ML5T–2] (b) [ML–1T–2]
(c) [L ] 3
(d) [L6] (d) None of these
40. Four similar prisms of same 47. If (a + ib) (c + id) (e + if)
material having same angle of (g + ih) = A + iB, then
prism are arranged. Which of the (a2 + b2) (c2 + d2) (e2 + f2) (g2 + h2)
(a) X is p-type, Y is n-type and the
following arrangements give no junction is forward biased is equal to:
net angular deviation? (b) X is n-type, Y is p-type and the (a) A2 – B2
junction is forward blased (b) A2 + B2
(a) (c) X is p-type, Y is n-type and the (c) A4 + B4
junction is reverse blased (d) A4 – B4
(b) (d) X is n-type, Y is p-type and the 48. If α and β are the roots of the
junction is reverse biased equations x2 + bx + c = 0 then
44. The wavelength associated with what is the value of α–1 + β–1?
(c)
the moving particle depends
b b
upon pth power of its mass ‘m’. (a) − (b)
(d) c c
qth power of its velocity. ‘v’ and rth
41. A person listening to two trains, power of Plank’s constant h. Then c c
(c) (d) −
one approaching him while the the correct set of values if p, q and b b
other moving away from him. The r is:
49. T h e s u m o f t h e s e r i e s
speed of both the trains is 4 m/s. (a) p = 1, q = 1, r = 1
1 1 1
If both trains give off whistle of (b) p = –1, q = –1, r = –1 + + + .....of 9 terms is:
2 3 6
their natural frequency of 240 Hz, (c) p = –1, q = –1, r = 1
then observer will listen number (d) p =1, q = –1, r = 1 5 1
(a) − (b) −
45. Under a pressure head, the rate 6 2
of beats equal to (speed of sound
in air = 320 m/s): of orderly volume flow of liquid 3
(c) 1 (d) −
(a) 6 (b) 3 through a capillary tube is Q. If 2
(c) 0 (d) 12 the length of the capillary tube be 50. If (1 + ax) n = 1 + 8x + 24x 2
R doubled and the diameter of bore
42. For a gas = 0.67 the molecules +.........then find a and b.
Cv is halved, the rate of flow would (a) 2, 8 (b) 8, 24
of gas are: become: (c) 8, 4 (d) 2, 4
(a) diatomic
Q 1 − cos 2 x
(b) diatomic and polyatomic (a) (b) 16 Q 51. lim is equal to:
4 x→ 0 x
(c) monoatomic
(d) polyatomic 1 1 (a) 0 (b) 1
(c) Q (d) Q
8 32 (c) 2 (d) 4
Practice Set-15 187

(a) 49 (b) 625 63. If 5Pr = 2 6Pr –1, then the value of
52. If x2 + 2xy + y3 = 42, then dy is
dx (c) 216 (d) 890 r is:
equal to: 58. If 49n + 16n + k is divisible by 64 (a) 10 (b) 3
−2( x + y ) −(2 x + 3 y 2 ) for n ∈ N, then the least negative (c) 0 (d) None of these
(a) (b) integral value of k is:
2x + 3y2 2( x + y ) 64. The point represented by the
(a) – 1 (b) – 2 complex number 2 – i is rotated
−2( x + y ) 2( x 2 + y 2 ) (c) – 3 (d) – 4 about origin through an angle
(c) (d)
2 y + 3x 2 2x + 3y2 1 d2 y π
59. Solve . = cosec x – 2 in the clockwise direction, the
sin x dx 2 2
53. Evaluate the integral sin x.
new position of point is:
1 − sin x cos 2 x
(a) 1 + 2i
∫ 1 − cos x dx (a) y =
4
+ C1 x + C2
(b) –1 – 2i
x cos 2 x (c) 2 + i
(a) cot x – 2 log | sin | + C (b) 2y2 = + C1 x + C2 (d) –1 + 2i
2 2 4
65. The coefficient of the middle
x x − cos 2 x term in the expansion of
(b) cot + 2 log | sin | + C (c) y = + C1 x + C2
2 2 4 (2 + 3x)4 is:
(d) None of these (a) 6 (b) 5!
x
(c) – cot x + 2 log | sin | + C 60. The value of b for which the (c) 8! (d) 216
2 2
function f (x) = sin x – bx + c is 66. The relation R in the set Z of
(d) None of the above
decreasing in the interval (– ∞, ∞) integers given by R = {(a, b) :
54. What is the angle between the
is given by: a – b is divisible by 5} is:
two straight lines y = (2 − 3)x (a) b < 1 (b) b > 1
(a) reflexive
+ 5 and y = (2 + 3) x – 7? (c) b > 1 (d) b < 1
(b) reflexive but not symmetric
(a) 60° (b) 45° 61. The minimum value of | x – 3 | +
(c) symmetric and transitive
(c) 30° (d) 15° | x –2 | + | x – 5 | is equal to:
(a) 3 (b) 7 (d) an equivalence relation
55. The value of
π
1 2 1 2 (c) 15 (d) 9 67. If sin–1 x – cos–1 x = , then what
1 + 2 + + + + + ......∞is: 6
62. Find the mean deviation about is the value of x?
2! 3! 4! 5!
mean for the data in following 1
(a) e2 (b) e + e–1 (a) − (b) 1
tables: 2
e − e −1 −1
(c) (d) 3e − e Income per day Number of 1 3
2 2 persons (c) (d)
2 2
56. The line joining (5, 0) to (10 cos θ, 0–100 4
68. If the eccentricity and length of
10 sin θ) is divided internally in 100–200 8
latusrectum of a hyperbola are
the ratio 2 : 3 at P. If θ varies,then 200–300 9
13 10
the locus of P is: 300–400 10 and units respectively,
3 3
(a) a pair of straight lines 400–500 7
then what is he length of the
(b) a circle 500–600 5 transverse axis?
(c) a straight line 600–700 4
7
700–800 3 (a) units (b) 12 units
(d) None of these 2
57. The value of (a) 358, 157.92
(b) 359, 158.92 15
811/ log 5 3 + 27 log9 36 + 3 4/ log7 9 is (c) 15 units (d) units
(c) 358, 158.20 2 4
equal to:
(d) None of these
188 Practice Set-15

69. What is tan (cos–1 x) equal to? (a) the State Government of marks of the students of three
(b) the directive principles of state divisions of class X.
1 − x2 x policy (a) 61.8 (b) 62.5
(a) (b)
x 1+ x 2 (c) the Union Government (c) 61.1 (d) 63.9
(d) the fundamental rights of the 83. The total cost of a washing
1 + x2 2 Indian Citizen machine with an electric
(c) (d) 1− x
x 76. Which disease is caused due to chimney was ` 57,750. The
deficiency of Iodine? electric chimney was sold at a
a cos φ + b θ
70. If cos θ = , then tan (a) Rickets profit of 34% and the washing
a + b cos φ 2 (b) Scurvy machine at a loss of 24%. If the
is equal to: (c) Goitre
sale price was the same in both
(d) Growth retardation
 a −b  φ the items, then the cost price of
(a)   tan 77. Which railway station has
the cheaper item was ............ .
 a + b  2
became the India’s first green
(a) ` 26850 (b) ` 20900
and clean railway station?
 a+b φ (c) ` 28875 (d) ` 25850
(b)   cos (a) Varanasi Cantt railway station
 a −b  2 (b) Udhana railway station 84. The bus fare between two cities
(c) Jaynagar railway station is increased in the ratio 5 : 11.
 a −b  φ (d) Katni Junction railway station
(c)   sin Find the increase in the fare, if the
 a + b  2 78. The sum of thrice a number and original fare is ` 275.
(d) None of these twice its reciprocal is 97/4. What (a) ` 605 (b) ` 121
is the number ? (c) ` 330 (d) ` 242
REASONING AND GENERAL (a) 9 (b) 10
(c) 8 (d) 7 85. A man travels 400 km in, partly
AWARENESS (RAGA) by rail and partly by steamer.
79. Find the lowest common multiple
71. ________ allows searching for of 36, 54 and 60. He spends 9 hours more time
information and computing on steamer. If the velocity of
(a) 340 (b) 240
derived information. (c) 440 (d) 540 the steamer is 30 km/hr and the
(a) Data definition language velocity of rail is 70 km/hr, how
(b) Data manipulation language 1 2 much distance does he cover by
2 −1
(c) Query language 5 7 steamer?
80. 10 – 10 × is equal to:
(d) OQL 1 (a) 309 km (b) 371 km
2−
72. Garadi is a folk dance of _____ . 1
4 (c) 464 km (d) 556 km
(a) Mizoram 6
Direction (86): These questions are
(b) Puducherry (a) 3 (b) 5
(c) 4 (d) 10 based on the following information:
(c) Gujarat (d) Rajasthan
86. In ‘B ÷ H + M’ how is M related
73. Which of the following measures 81. The mean of marks secured by to B?
will not increase a nation’s 65 students in division A of class
(a) Wife
wealth? X is 54, 30 students of division B
(b) Husband
(a) investing in new companies is 50 and that of 55 students of
(b) increasing efficiency of factories (c) Sister
division C is 48. Find the mean
(c) converting grasslands into (d) Cannot be determined
of marks of the students of three
orchards divisions of Class X. 87. If the word ‘PORTER’ can be
(d) By redistributing taxes as (a) 50.3 (b) 49.6 coded as ‘MBNZQN’. How can
subsidies (c) 51 (d) 52.4
‘REPORT’ be written?
74. Which is called country of canals? (a) NQMBNZ (b) NQBMNZ
82. The mean of marks secured by
(a) India (b) Pakistan (c) NBQMNZ (d) NQMNBZ
60 students in division A of class
(c) Japan (d) Myanmar X is 66, 45 students of division B 88. Ankit walks 17 km towards east.
75. A r t i c l e 3 9 A o f t h e I n d i a n is 62 and that of 75 students of He turns left and walks 17 km. He
Constitution “Equal justice and turns left again and walks 24 km to
division C is 60. Find the mean reach point A. What is the straight
free legal aid” deals with :
Practice Set-15 189

line distance (in km) between the 91. Which of the following is the Directions (96–100): On the basis of the
starting point and point A? position of F? information provided below, answer
(a) (b) (a) Fourth to the right of D these questions :
(b) To the immediate left of C Six people A, B, C, D, E and Fare
(c) (d) (c) Between A and E sitting on the ground in a hexagonal
89. Rashmi is 14th from the right end (d) To the immediate right of A shape. All the sides of the hexagon so
in a row of 40 girls. What is her 92. If a × b and b � c, then formed are of same length. A is not
position from the left end? (a) c + a (b) b D c adjacent to B or C, D is not adjacent
(a) 25th (b) 27th (c) a + c (d) c � a to C or E, B and Care adjacent, F is in
(c) 21st (d) 24th 93. AD : N : : BD : ? the middle of D and C?
90. I n t h e f o l l o w i g q u e s t i o n , (a) P (b) X
96. Which of the following is not a
which one set of letters when correct neighbour pair?
(c) Y (d) Z
sequentially placed at the gaps (a) A and F (b) D and F
94. i. Permanence
in the given letter series shall (c) B and E (d) C and
ii. Permanent 97. Who is at the same distance
complete it?
iii. Permafrost from D as E is from D?
M_N_M_MR_R_N iv. Permeability
(a) RMRMN (b) RRNNM (a) B (b) C
(a) ii, iii, iv, i (b) iii, ii, i, iv
(c) RNRNM (d) MMRRN (c) D (d) F
(c) iv, ii, i, iii (d) iii, i, ii, iv
98. Which of the following is in the
Direction (91): Read the following Direction (95): Study the following right sequence?
information carefully and answer the information to answer the given (a) A, F, B (b) F, A, E
questions given below: questions : (c) B, C, F (d) D, A, B
(i) A, B, C, D, E, F and G are sitting In a certain code ‘829’ means ‘how 99. If one neighbour of A is D, who
around a circle and are facing the art thou’ ‘958’ means ‘thou art good’ and is the other one?
centre. ‘15873’ means ‘thy good and thou bad’. (a) B (b) C
(ii) G is second to the left of C, who is 95. What may be the possible code (c) E (d) F
to the immediate left of F. for ‘thy’? 100. Who is placed opposite to E?
(iii) A is third to the left of E. (a) 1, 7 or 3 (b) 7 (a) B (b) C
(iv) B is between D and E. (c) 3 (d) 5 (c) D (d) F

Answers with Explanation


8. (a) The word Tardy (Adjective) 14. (d) Numismatist
ENGLISH
means : slow to act, move or happen; late 15. (c) Here, appear before the in
1. (b) 2. (d) 3. (a) 4. (b) in happening. vestigation committee.... should be used.
5. (a) The correct spellings are: The word Prompt (Adjective)
communalism; pessimism; optimism. 16. (b) May is used to express
means : immediate; done without delay;
6. (a) T h e c o r r e c t s p e l l i n g s possibility in affirmative sentences. Can
punctual.
are: establishment; announcement; is used in the corresponding interrogative
commitment. Look at the sentences :
and negative sentences.
7. (d) The word Foment (Verb) The law is often tardy in reacting to
means: stimulate; excite; incite; to create changing attitudes. Prompt payment of Look at the sentences:
trouble or violence or make it worse. the invoice would be appreciated. It may rain tomorrow.
The word Quell (Verb) means : 9. (d) The word Enigma (Noun)
Can this be true?
to stop or reduce strong or unpleasant means : puzzle; mystery; a person,
feelings; to stop something such as thing or situation that is mysterious and It cannot be true.
violent behaviour or protests. difficult to understand. 17. (b) He asked Ravi why he was
Look at the sentences: 10. (a) to be one's guest sounding so depressed that day.
11. (d) A new airport is going to be
Extra police were called in to quell 18. (c) He asked me what time the sun
built near the old one.
the disturbances. would set the next day.
12. (b) No one can repair my watch.
They accused him of fomenting 13. (d) 19. (d) 20. (a)
political unrest.
190 Practice Set-15

At t = 0, I = 0 80
PHYSICS F= 9× + 32
∴ rate = 0 5
21. (d) λ = h , p = 2m (K.E.) At t = ∞ , I = I0 but
dI
= 0, = 144 + 32
p dt
therefore, rate = 0 = 176ºF
= 3
2m × KT 26. (a) F = µR = µmg 30. (b) In the direction of electric
2
= 0.5 × 50 × 9.8 field, potential decreases. From figure, x
∴p= 3mkT = 245 N co-ordinate of D is lesser than A.
h 27. (b) Let E and B be along X-axis. ∴ VD > VA
∴λ= When a charged particle is released
3mKT But x co-ordinate of B and C are same.
from rest, it will experience an electric
= 6.6 × 10 −34
force. ∴ VB = V C
Y y
3 × 1.67 × 10−27 × 1.38 × 10−23 × 300
E
⇒ λ = 1.18 × 10–10 m B
= 0.118 nm
22. (b) B0 = E 0 D A
c
−3
= 10
B
3 × 108 X
E C
= 3.33 × 10–12 T Z
23. (a) Y = mgL X L Lω
πr 2 l 31. (d) tan φ = =
Z R R
or, l = mgL along the direction of electric field or
πr 2 Y  1
opposite to the direction of electric field   (2 π)(200)
Elongation l is proportional to L depending on the nature of charge. Due to ⇒ tan φ = π
r2 300
L this force, it acquires some velocity along
is highest for the pair given in (a).
r2 X-axis. Due to this motion of charge, 4
⇒ tan φ =
magnetic force cannot have non-zero 3
24. (d) The electric charge and electric  
value because angle between v and B
field inside the conductor is zero. But 4
would be either 0º or 180º. ⇒ φ = tan –1  
potential is constant. 3
(a)  EA = EB = 0 So, only electric force is acting on
particle and hence, it will move along a    
(b)  qA = qB = 0 32. (a) F1 + F2 + F3 = 0
∴ σA = σB = 0 straight line.
1 ⇒3iˆ + 2ˆj − kˆ + 3iˆ + 4ˆj − 5kˆ + Aiˆ + Aˆj − Akˆ
28. (d) Uinitial = CE 2
(c)  − dV =
∫ ∫ Edr q = 0
1 ⇒ iˆ (6 + A) + ˆj (6 + A) + kˆ ( − 6 − A)
∴ − ∫ dV = ∫ 0 dr Ulast = CE 2
q 
∴ V = Constant = 0
U initial
Hence option (d) is correct. ∴ = 1:1 ⇒6+A= 0
U last
25. (a) U = 1 LT 2 ⇒A = –6
2 29. (c) From the relation, 33. (a) In steady state, current through
C capacitor branch is zero. So, capacitor
Rate =
dU  dI 
= (LI)   = F − 32 branch may be removed.
dt 5 9
 dt 
Practice Set-15r0 191

r0
r0 µ0 I 2 a
36. (c) BP = ∴ = P
r0 C0 2R V2
r0
r0 C0 BP
r0 a
r0
or, = [ML–1T–2]
r0 L6
A r0 B B
A B Thus,a = [ML5T–2]

l2 = 3A 40. (b) S1 = S2 = S3 = S4 = S

r0 S = S 1 + S2 – S 3 – S4 = 0
r0
BQ 41. (a) For the approaching train
v
r0 r0 r0 r0 n' = ×n
l2 = 4A v − vs
r0 r0
AA B B
Q = 320 × 240 = 243 Hz
the −reaching
For320 4

train
3r0 3r 4π × 10 −7 × 4
0 = v
2 × 0.02 π n' = ×n
v + vs
= 4 × 10–5 Wb/m2
A B 320
A r0 B µ 0 I1 = × 240 = 237 Hz
r0 BQ = 320 + 4
2R
r0 × 3r0 −7 ∴ No. of beats per second
Req = = 4π × 10 × 3
r0 + 3r0 = 243 – 237 = 6
2 × 0.02 π
R 3
3 42. (c) Since, CV = = R, the
= = 3 × 10–5 Wb/m2 0.67 2
r0
4 ∴B= gas is obviously mono-atomic.
B2P + B2Q
34. (b) When distance between star 43. (d) As is pentavalent impurity and
and earth decreases, the frequency = (4 × 10 −5 )2 + ( 3 × 10 −5 )2 In is traivalent impurity; so junction, X –
increases and wavelength decreases. Y is reverse biased n-p junction.
= 5 × 10–5 Wb/m2
Shift in wavelength 44. (c) λ αmp. vq. hr
Force
v 37. (b) Acceleration = h
∆λ = × λ Mass But λ =
c 5 1 mv
= = m/s 2
100 × 103 20 + 5 5 comparing p = –1, q = –1, r = 1
= × 5000 × 10 –10 m
3 × 108 The force required by 20 kg to move
πpa 4
with this acceleration 45. (d) Q =
= 1.67 × 10–10 m (slightly less) 8 ηl
= 1.67 Å 1
= 29 × = 4N πp(a / 2) 4
ˆ .A
35. (a)We know that A ˆ = 1, where 5 Now, rate of flow =
8 η2l
 is any unit vector. Rest of the force = 5 – 4 = 1N
is acting on 5 kg mass to provide it 1 π pa 4
= .
d ˆ ˆ required acceleration. 32 8ηl
⇒ (A . A) = 0
dt 38. (b) Change in momentum = F. ∆t
ˆ ˆ = 200 × 0.25 = 1 Q
ˆ . dA + A
⇒ A ˆ . dA = 0 32
dt dt = 50 N-s
 P1 – P2 = 50 N-s
 ˆ d  MATHEMATICS
⇒ 2  A +  = 0 ∴ P1 = 50 N-s
 dt  Since,P2 = 0 46. (c) Suppose A refers the set of
Americans who like cheese and B refers
d  a  those who like apples. Let the population
⇒ Â . = 0 39. (a)  P + 2  (V − b) = RT
dt  V  of America is 100
192 Practice Set-15

So, n (A) = 63, n(B) =76 50. (d)We have,


x x
Now, we know that, (1 + ax)n = nC0 (ax)0 + nC1(ax)1 1 − 2sin   cos  
2
   2  dx
n (A ∪ B) = n (A) + n (B) – n (A ∩ B) = ∫
+ nC2(ax)2 +.......... x
⇒n (A ∩ B) = 139 – n (A ∪ B) 2sin 2  
n(n − 1) 2 2 2
∴ n (A ∩ B) > 39, = 1 + n ax + a x +......
2! 1 x x
because n (A ∪ B) < 100
2∫
Comparing with the given series, = cosec 2   dx – ∫ cot   dx
Further A ∩ B ⊂ A and A ∩ B ⊂ B 2 2
we get
n (A ∩ B) < n (A) and n(A ∩ B) < n(B)
i.e., n (A ∩ B) < 63 and n(A ∩ B) > 39 1 + 8x + 24x2 +............. x   x 
= − cot   − 2 log sin    + C
So, 39 < n (A ∩ B) < 63 n(n − 1) 2 2 2   2 
= 1 + n ax + a x +...
Hence, 39 < x < 63. 2!
m2 − m1
47. (b) We have, 54. (a) tan θ =
n(n − 1) 2 1 + m1m2
(a + ib) (c + id) (e + if) (g + ih) Hence, na = 8. a = 24
2!
= A + iB
2 2+ 3−2+ 3
On taking mode on both sides, we get 8 1 8 =
⇒a= . n(n − 1)   = 24 1 + (4 − 3)
| a + ib | | c + id | | e + if | + | g n 2 n
+ ih | = | A + iB | ⇒ 32 (n – 1) = 24n ⇒ 4(n – 1) = 3n 2 3 π
⇒ (a + b ) (c + d ) (e2 + f2) (g2 + h2)
2 2 2 2
⇒n= 4 = = 3 = tan
2 3
= A2 + B2 8 8
Hence, a – = =2 π
48. (a) Given that, α and β are the n 4 = θ= = 60°
roots of the equation 3
1 − cos 2 x 2 sin 2 x
x2 + bx + c = 0 51. (a) lim = xlim 55. (d) Sum of the series
x→0 x →0 x
Then, the sum of the roots
2 1 2 1 2
sin x  =1+2+ + + + + ....
= α+β=
−b
= – b ....(i) = lim 2   x = 2.1.0 = 0 2! 3! 4! 5!
1 x→0  x 

and product of roots 52. (a) x2 + 2xy + y3 = 42  1 1 


c = 1 + + + .... 
= α.β = = c ....(ii) d 2 d  2! 4! 
1 ∴ (x + 2xy + y3) = 42
dx dx  1 1 
1 1 α ×β −b
∴ α–1 + β–1 = + = = + 2 1 + + + .... 
α β αβ c  d dy   3! 5! 
2 dy
2 x + 2  x + y  + 3y = 0
1 1 1 2−3  dx dx  dx e + e −1 e − e −1 3e − e −1
49. (d) Here, a = , d = − =
2 3 2 6 = + 2 =
dy 2 dy
2 2 2
1 2x + 2 x + 2y + 3 y = 0
= − and n = 9 dx dx
6 56. (b) If (x, y) be the point, then by
n dy ratio formula
Sn = [2a + (n – 1)d] (2 x + 3 y 2 ) = –2 (x + y)
2 dx x = 4 cos θ + 3, y = 4 sin θ
⇒ (x –3) = 4cos θ,y = 4 sin θ
9 1  1  dy −2( x + y ) ⇒ (x – 3)2 = (4 cos θ)2, y2 = (4 sin θ)2
= 2 × × (9 − 1)  −   ⇒ =
2  2  6  dx 2x + 3y2
⇒ (x – 3)2 = 16 cos2θ, y2 = 16 sin2 θ
9  1  9  4  x x On adding we get
= 1 + 8 ×  −   = 1 −  1 − 2sin   cos  
2   6  2  3  2  2  dx ∴(x – 3)2 + y2 = 16
53. (d) I = ∫
9  3− 4  9  1 3
  x 
1 − 1 − 2sin 2    57. (d) Let T1 = 3log3 3
=   = ×−  = −   2  = 3log3 3 = 54 = 625
2 3  2  3 2
Practice Set-15 193

= – cos 2 x + C1 x + C2
2 4
log3 3
3
T2 = (3 ) 3 = 3log 3 6
3 = 350 + × 100 = 350 + 8
4 50
= 63 = 216 dy x = 358
1 60. (b) = cos x – b < 0, ∀ x ∈ R
4. log3 7 dx Mean deviation about the mean
T3 = 34log9 7 = 3 2
Since, the maximum value of cos x is 1
log 72
∴ 1 – b< 0 or 1 < b =
∑ fi | xi − x |
= 3 3 = 72 = 49
But cos x < 1 and hence, b > 1
∑ fi
∴T1 + T2 + T3 = 625 + 216 + 49 = 890
7896
58. (a) Let P(n) = 49n + 16n + k 61. (a) Given, function is: = = 157.92
50
Forn = 1, we get f (x) = |x – 3| + |x – 2| + |x – 5| ....(i)
P(1) = 491 + 161 + k = 65 + k 63. (b) Given, 5Pr = 26Pr – 1
3 x − 10 x≥5 5!
As, P(1) is divisible by 64, we take  x
 3≤ x<5 (5 − r )!
k= –1 ⇒f (x) = 
∴ P(1) = 65 – 1 = 64, which is  6− x 2 ≤ x < 3 6!  n n! 
divisible by 64
10 − 3 x x<2 = 2× ∵ Pr = (n − r )!
(6 − r + 1)!  
1 d2y Given function (i) is a modulus
59. (c) Given, . 5! 6 × 5!
sin x dx 2 function, the minimum value of modulus ⇒ = 2×
is zero (5 − r )! (7 − r )!
= cosec x – 2 sin x
| x – 3 | will be zero at x = 3 1 12
2 ⇒ =
⇒ d y = 1 – 2 sin2 x = cos 2x f (3) = 0 + 1 + 2 = 3 (5 − r )! (7 − r )(6 − r )(5 − r )!
dx 2
| x – 2 | will be zero at x = 2 ⇒ (7 – r) (6 – r) = 12
On integrating both sides with respect ⇒ 42 – 7r – 6r + r2 = 12
to x, we get f (2) = 1 + 0 + 3 = 4
⇒ r2 – 13r + 30 = 0
dy sin 2 x | x – 5 | will be zero at x = 5
= + C1 ⇒ r2 – 10r – 3r + 30 = 0
dx 2 f (5) = 2 + 3 + 0 = 5
⇒ r(r – 10) – 3(r – 10) = 0
Now, again integrating both sides with So, minimum value of the function
is at x = 3. ⇒ (r – 10) (r – 3) = 0
respect to x, we gety
∴ r = 10, 3
62. (a). But, we will take only r = 3 as 0 < r < 5
64. (b) Here, z = 2 – i
Mid x −A
d = i Let zi be the required complex number
class ( fi ) value i B fi di | xi − x | fi | xi − x |
xi A = 350, h = 100 ∴ | zi | = | z | = 22 + 12
0 − 100 4 50 −3 −12 308 1232
= 4 +1 = 5
100 − 200 8 150 −2 −16 208 1664
200 − 300 9 250 −1 −9 108 972 zi
π
| zi | − i 2
300 − 400 10 350 0 0 8 80 ∴ = e
z |z|
400 − 500 7 450 1 7 92 644
500 − 600 5 550 2 10 192 960 ∴ zi = (2 − i 5)
600 − 700 4 650 3 12 292 1168 5
700 − 800 3 750 4 12 392 1176   π  π 
Total ∑ fi = 50 4 7896 cos  − 2  + i sin  − 2  
    

Mean x = A +
∑ fi di × h  π π
= (2 – i)  cos + i sin 
∑ fi  2 2
194 Practice Set-15

= (2 – i) (0 – i) = – (2i – i2) 1 1 θ a −b φ
= –2i + i2 = – 2i – 1 ⇒ 2x2 – 1 = ⇒ 2x2 = 1 + or tan2 = tan 2
2 2 2 a+b 2
4
65. (a) Middle term = + 1 = 3rd 3 3
2 ⇒ 2x2 = ⇒ x2 = ⇒x = 3 ∴ tan
θ
=
a −b
tan
φ
Coefficient of T3 = Coefficient of 2 4 2 2 a+b 2
T2 + 1 = 4C2 = 6
13
66. (d) Given, R = {(a, b): a – b is 68. (c) Here, e = and length of REASONING AND GENERAL
3
divisible by 5} latus rectum = 10 AWARENESS (RAGA)
3
Reflexive: Let (a, a) ∈ R, then (a – 71. (c) Query language allows
a) is divisible by 5 for all a ∈ z. So, R 2b 2 10 searching for information and computing
⇒ =
is reflexive a 3 derived information. Query languages
can be classified according to whether
Symmetric: Let (a, b) ∈ R 5a they are database query languages or
⇒ b2 =
⇒ (a – b) is divisible by 5 3 information retrieval query languages.
⇒ –(b – a) is divisible by 5 We know that b2 = a2 (e2 – 1) The difference is that a database query
⇒ (b – a) ∈ R language attempts to give factual answers
5a  13  to factual questions, while an information
⇒ = a2  − 1
So, R is symmetric 3 9  retrieval query language attempts to find
Transitive: Let (a, b) ∈ R documents containing information that is
5a 4a 2 15 relevant to an area of inquiry.
and (b, c) ∈ R ⇒ = ⇒a=
3 9 4 72. (b) Garadi is a folk dance
⇒ (a – b) and (b – c) are both divisible
by 5 performed in Puducherry. It is performed
∴ Length of transverse axis
during all festivals of the region. This
⇒ a – b + b – c is divisible by 5 2 × 15 15 dance performance is usually performed
⇒ (a – c) is divisible by 5 = 2a = = units.
4 2 for five to eight hours at a stretch, for
⇒ (a, c) ∈ R several nights in a row. The origin of
69. (a) tan (cos–1 x) Garadi dance is related to the Hindu epic,
So, R is transitive
As we know that, Ramayana.
Hence, R is an equivalence relation.
73. (d) Redistribution of income and
π 1 − x2
67. (d) sin–1 x – cos–1 x = cos–1 x = tan −1 redistribution of wealth are respectively
6 x the transfer of income and of wealth from
As we know that, Now, tan (cos–1 x) some individuals to others by means
cos–1 x = sin–1 1 − x 2 of taxation, charity, welfare, public
  services, land reform, etc. Redistribution
1 − x2
2 π = tan  tan −1  of taxes and subsidies are redistribution
sin x – sin
–1 –1
1− x =  x 
6   of wealth. They do not lead to wealth
creation of a nation that is in fact the
2 2
⇒ sin–1[ x 1 − (1 − x ) 1 − x2 stream of goods and services that it
= creates and capital accumulation.
x
π 74. (b) Pakistan is known as country
– 1 − x2 1 − x2 ] =  A
6 70. (a) 1 – cos A = 2 sin2   of Canals due to its extensive network
⇒ sin–1 2 of canals from Indus and its tributaries.
 x 1 − 1 + x 2 − ( 1 − x 2 )2  = π  A Most of Pakistan’s canals are in Punjab
  1 + cos A = 2 cos2  
6 2 province that are result of India’s partition
which saw most of the canals going to the
2 2 π Applying compoundo and dividendo
⇒ x x − (1 − x ) = sin former. During India’s partition, Punjab
6 rule,
was divided into West Punjab which
1 1 − cos θ a (1 − cos φ) − b(1 − cos φ) came to be part of Pakistan and East
⇒ x. x – 1 + x2 =
2 = Punjab which remained with India. East
1 + cos θ a (1 + cos φ) + b(1 + cos φ)
Punjab in India was relatively under-
Practice Set-15 195

developed in irrigation works, while 79. (d) LCM of 36, 54 and 60


West Punjab in Pakistan included most =
= 2 × 2 × 3 × 3 × 3 × 5 = 540
of the canal systems of the pre-partition
Indus basin. 76 x = 57750 × 134 – 134 x
75. (b) Article 39A is a Directive 134 x + 76x = 57750 × 134
Principle of State Policy under Part IV of
210 x = 57750 × 134
Indian Constitution. It deals with equal
justice and free legal aid. It states that
the state shall secure that the operation x=
of the legal system promotes justice, on 80. (c) Given exp.
Cost of washing machine
a basis of equal opportunity, and provide
free legal aid, by suitable legislation or 11 9 32 = ` 36850

schemes or in any other way, to ensure = 10 – 10 × 5 7 = 8 – 8 × 35 Cost of chimney
that opportunities for securing justice 1 6
2– 2– = ` 57750 – 36850
are not denied to any citizen by reason (25 / 6) 35 = ` 20900
of economic or other disabilities. 11 9 32 The chimney is cheaper

76. (c) A goitre is a swelling of the neck 5 7 35 = ` 20900
10 – 10 × = = 8 – 8 ×
or larynx resulting from enlargement 1 6 84. (c) According to the question let
2– 2–
of the thyroid gland associated with a (25 / 6) 35 new fare be x
thyroid gland that is not functioning
properly. Worldwide, over 90% cases of 32 35 5 : 11 = 275 : x
= 10 – 10 × × = 8 – 4 = 4.
goitre are caused by iodine deficiency, 35 64
usually seen in countries that do not use 81. (c) Mean marks of 3 divisions =
iodized salt. Selenium deficiency is also m1 x 1 + m 2 x 2 + m 3 x 3
=
considered a contributing factor. m1 + m 2 + m 3 x=
77. (b) Udhana railway station: A
65 × 54 + 30 × 50 + 55 × 48 = 55 × 11 = 605
young entrepreneur from surat launched = Increased bus fare
an initiative to change Udhana railway 65 + 30 + 55
= 605 – 275 = ` 330
station in Gujarat into India’s first green 3510 + 1500 + 2640 85. (a) Let time taken by train be t
and clean railway station like Mumbai’s =
150 and by steamer = (t + 9)
Chhatrapati Shivaji Terminus. 7650 Distance = Speed × Time
= = 51 400 = 70t + 30 (t + 9)
78. (c) Let the number be x 150
400 = 100 t + 30 × 9
∴ According to question, Hence, the mean marks is 51. 400 – 270 = 100 t
82. (b) Mean Marks 130 = 100 t
= 60 × 66 + 45 × 62 + 75 × 60
= by rail =
60 + 45 + 75
= 3960 + 2790 + 4500 = 1.3 hours
= time taken by steamer = 1.3 + 9
180
12x2 + 8 = 97x = 10.3 hours
11250
12x2 – 97x + 8 = 0 = = 62.5 Distance covered by steamer
180 = 30 × 10.3 = 309 km
12x2 – 96x – x + 8 = 0 83. (b) Let the cost of washing machine
86. (b) The Blood relationship tree/
be x and the cost of chimney chart can be constructed as given below:
12x (x – 8) – 1 (x – 8) = 0
= 57750 – x
(x – 8) (12x – 1) = 0 According to question, B– (wife) M+ (husband)
(mother) (father)
x – 8 = 0 and 12x – 1 = 0

H+
x = 8, x = (son)
= B ÷ H = B is mother of H.
∴ Hence, the number is 8.

196 Practice Set-15

H + M = H is son of M. AE = Smilarly,
So, B Mother of H son of M.   B D→X
= km 2 4 24
87. (a) As, 89. (b) Required position
P O R T E R Finally, the missing term is 24.
= (40 + 1 – 14) = 27th 94. (d) According to dictionary, the
↓ ↓ ↓ ↓ ↓ ↓
90. (b) The series is sequence is:
M B N Z Q N
M_N_M_MR_R_N (iii) Permafrost (i) Permanence
Here, code of P → M, O → B,
the series becomes MRN RMN MRN (ii) Permanent, (iv) Permeability.
R → N, T → Z, E → Q
RMN 95. (a) The code for ‘thy’ may be ‘1’
Therefore,
or ‘7’ or ‘3’.
R E P O R T Finally the set of letters 'RRNNM' 96. (a) By analysing the above
↓ ↓ ↓ ↓ ↓ ↓ will complete the series. hexagonal seating arrangement, we get
N Q M B N Z 91. (a) By analysing the above Neighbours
88. (a) circular arrangement, it is clear that ⇒ D & F, B & E and C & F.
E D 24 km Therefore, incorrect neighbour pair
C 
F Position Fourth to the right of D. ⇒ A and F.
97. (b) By analysing the above
 hexagonal seating arrangement, it is
17 km F Position Immediate right of C.
clear that
 E ⇒ Second to the right or left of D.
F Position Between C and E.
So, second to the left or right of D
A B ⇒ C.
17 km 
F Position Second to the right of A. Therefore, required person at the
N
Therefore, option (a) is correct. same distance from D ad E is from D
92. (a) According to the given ⇒ C.
W E
information 98. (c) By analysing the above
hexagonal seating arrangement, we get
S a × b  a is a little less than b Right Sequence: B, C, F, D, A, E.
BC = A D = 17 km and b � c  b = c 99. (c) By analysing the above
EC = 24 km From (i) and (ii), it is clear that hexagonal seating arrangement, it is
(ED + CD) = 24 km clear that
a is a little less than c
ED = (24 – 17) = 7 km Neighbours of A ⇒ E and D.
 c is little more than a Therefore, Required neighbour of
In ∆ ADE,  c + a. A ⇒ E.
AE2 = ED2 + AD2 93. (b) As, 100. (d) By analysing the above
= (7)2 + (17)2    A D→N hexagonal seating arrangement, we get
= 49 + 289 = 338    1 4 14 Opposite to E ⇒ F.
qqq
Practice Set-16 197

Air Force Airmen Group X & Y Exam


Practice Set-16

rulers was indeed the third piece of the (c) mobilise the people of a country
ENGLISH development puzzle. I see this attitude against colonial rule
Directions (1-4): Read the passage of the Indian elite everyday in how they (d) fully and properly understand
given below and answer the questions send their children to English medium the pulse of the people of a
that follow by selecting the most schools while forcing the children of the country
appropriate option. poor into vernacular schools, extol the 4. The expression ‘walking the talk’
Max Weber laid the foundation for my virtues of poverty while living in luxury, means:
and glorify the rural life while they sit (a) being diplomatic in one’s
belief that decent and hard working
comfortably in cities. behaviour and words
people with high aspirations make great Source: ‘A Better India, A Better
nations, no matter what the odds are. This (b) practising what one preaches
World’–NR Narayana Murthy
(c) addressing public gatherings in
was the first piece of the development (Adapted)
an election campaign
puzzle for me. Mahatma Gandhi opened 1. The main purpose of the author in (d) talking to the common people
the eyes to the importance of good the above passage is to by mingling with them
leadership in raising the aspirations of (a) delineate the lessons he has Directions (5-6): Choose the correct
people, making them accept sacrifices learnt for the development of a spelling of the word.
to achieve a grand vision, and most nation 5. (a) restaurant (b) resturant
importantly, in converting that vision into (b) prioritise goals for only
(c) resteurant (d) restorant
reality. He unleashed the most powerful economic development of India
6. (a) consumpshion
(c) discuss the different writers he
instrument for gaining trust-leadership by (b) conjumption
has read
example. He ate, dressed, travelled and (d) argue why India should not be (c) consumption
lived like the poor. Walking the talk was considered a developed country (d) conjumpshan
extremely important to the Mahatma who Direction (7-8) : Find out indirect speech
2. The first piece of the development
understood the pulse of our people like of given below sentences.
puzzle, according to the author is: 7. He said he did not believe me.
no other Indian leader. The biggest lesson (a) imbuing the citizens of the country
for me from Gandhi’s book and life is (a) His reply was that he would not
with decency and aspirations
the importance of leading by example. believe me.
(b) the need for making people
I realized fairly early that this was the understand the importance of (b) He replied that he could never
leading an idealistic and simple believe me.
second piece of the development puzzle.
life (c) He said to me, “I don’t believe
Frantz Fanon’s book on the colonizer you”.
(c) creating a team of industrious
mindset of elites in a post-colonial people for national cause (d) He said how was it possible to
society opened my eyes to the role (d) the importance of decent, believe me.
of the bureaucracy and the elite in inspired and industrious people 8. Rama said he was very busy then.
decelerating the progress of the poor and for a nation’s development (a) Rama said, “I am very busy now”.
the disenfranchised. The colonial mindset 3. Mahatma Gandhi proved that only (b) Rama replied that he had been
of the ‘dark elite in white masks’ in a leadership by example can: busy.
post-colonial society–the mindset that (a) gain the trust of the people so (c) Rama could not come because
the rules and the rulers have different that they are willing to make he was busy.
sets of rights and responsibilities with sacrifices for a larger cause (d) Being busy is the reason that
a huge asymmetry in favour of the (b) inspire people to eat, dress, prevents Rama’s arrival.
travel and live like the poor
198 Practice Set-16

9. Find out correct part of the given error and some have none. Find out
sentences. which part of a sentence has an error. If (a) W = F. l (b) W =
(a) It could lead to the kind of good there is No error then mark the answer
life, both spiritual and material, is (d).
16. (a) We had a lot of difficulty/ (b) to (c) W = (d) W =
that was once the Privilege of a
lucky few find/ (c) the house (d) No Error
25. One mole of an ideal gas goes
(b) It could lead to the kind of good 17. (a) Patience as well as perseverance from an initial state A to final
life-both spiritual and material- / (b) are necessary/ (c) for success / state B via two processes. If first
that was once the privilege of a (d) No Error undergoes Isothermal expansion
lucky few Direction (18-19): In the following from volume V to 3V and then its
(c) It could lead-to the kind of good questions choose the word opposite in volume is reduced from 3V to V
life; both spiritual and material; meaning to the given word. at constrant pressure. The correct
that was once the privilege of a 18. Exodus p-V diagram representing the two
(a) influx (b) home-coming processes is:
lucky few.
(d) It could lead-to the kind of good (c) return (d) restoration
life, both spiritual, and material- 19. Inquisitive
that was once the privilege of a (a) insincere (b) indifferent
lucky few. (c) insensitive (d) insulting
10. What is the synonym of ‘elegant’? 20. Find out the incorrect sentence.
(a) The apples are grown in many (a)
(a) Gentle (b) famous
different countries
(c) Graceful (d) None of the
(b) Books are essential to a student
above
(c) Chess is a game which requires
11. What is the synonym of ‘timorous’?
great patience
(a) Up-to-date (b) Bright (d) The Sharmas live at Elgin Road
(c) Timid (d) None of these
Directions (12-13): Find out the correct PHYSICS
antonym of the underlined word. 21. A boat of mass 40 kg is at rest. A
12. There is both scarcity and _____ dog of mass 4 kg moves in the boat (b)
in the modern world. with a velocity of 10 ms–1. What is
(a) deficit (b) attraction the velocity of boat?
(c) plenty (d) toleration (a) 4 ms–1 (b) 2 ms–1
13. People have tried their best to (c) 8 ms–1 (d) 1 ms–1
provoke the flood victims.
22. A body of mass 0.5 kg travels in
(a) gratify (b) convince a straight line with velocity v =
(c) pacify (d) attract ax3/2 where, a = 5 m–1/2 s–1. The
14. Find the correct sentence– work done by the net force during (c)
(a) He is one of those boys who are its displacement from x = 0 to
wicked. x = 2 m is:
(b) He is one of those boys who is (a) 1.5 J (b) 50 J
wicked. (c) 10 J (d) 100 J
(c) He is one of those boy who is
23. In an orbital motion, the angular
wicked.
momentum vector is:
(d) He is one of those boy who are
wicked. (a) along the radius vector
(b) parallel to the linear momentum (d)
15. Fill in blank given below:
(c) in the orbital plane
Please _______ the candle when
(d) perpendicular to the orbital 26. A system is subjected to two
power is restored.
plane SHMs given by y1 = 6 cos ωt and
(a) put out (b) put down 24. The length of a wire increases by l y2 = 8 cos ωt.
(c) put up (d) put forth
due to a force F applied on it. Then, The resultant amplitude of SHM
Direction (16-17): In the following
questions, some of the sentences have the work done W in stretching the is given by:
wire will be:
Practice Set-16 199

(a) 2 (b) 10
(c) 14 (d) 20
27. A parllel plate capacitor has an
electric field of 105 Vm–1 between
the plates. If the charge on the
capacitor plates is 1 µC, the force (c)
on each capacitor plate is:
(a) 0.5 N (b) 0.05 N
(c) 0.005 N (d) None of these
28. A resistor of 6 kΩ with tolerance
10% and another of 4 kΩ with
tolerance 10% are connected
in series. The tolerance of
(d) (a) Fig. (i) (b) Fig. (ii)
combination is about:
(c) Fig. (iii) (d) Fig. (iv)
(a) 5% (b) 10%
32. When a body is lifted from surface 35. According to corpuscular theory
(c) 12% (d) 15%
of Earth to height equal radius of light, the different colours of
29. Which one of the following is not
of Earth, then the change in its light are due to:
made of soft iron?
potential energy is: (a) different electromagnetic waves
(a) Electromagnet
(a) mgR (b) 2mgR (b) different force of attraction
(b) Core of transformer
(c) Core of dynamo (c) (d) 4 mgR among the corpuscles
(d) Magnet of loudspeaker (c) different size of the corpuscles
30. A copper rod is moved in magnetic 33. Biot-Savart law indicates that (d) None of the above
field. The charge developed across 36. The simple Bohr model cannot
its ends will be proportional to: the moving electrons (velocity ) be directly applied to calculate
(a) magnetic flux produce a magnetic field such the energy levels of an atom with
(b) rate of change of magnetic flux that: many electrons. This is because.
(a) (a) of the electrons not being subject
(c)
to a central force
(b) (b) of the electrons colliding with
(d) each other
(c) it obey inverse cube law (c) of screening effects
31. Liquid oxygen at 50 K is heated (d) it is along the line joining (d) the force between the nucleus
to 300 K at constant pressure the electron and point of and an electron will no longer
of 1 atm. The rate of heating observation be given by Coulomb’s law
is constant. Which one of the 37. The main difference between
34. A point positive charge is brought
following graphs represents the Newton’s corpuscular theory and
variation of temperature with near an isolated conduction Planck’s quantum theory is:
time? sphere, figure. The electric field (a) the wave nature is not associated
is best given by: with Newton’s corpuscles while
quantum nature considered
wave nature with quantas
(b) Both involve wave nature
(c) Both do not involve wave nature
(a)
(d) the consideration of wave nature
with Newton’s corpuscles while
not with Planck’s quantas
38. If a small amount of antimony is
added to germanium crystal:
(a) the antimony becomes an
acceptor atom
(b) (b) there will be more free electrons
than holes in the semiconductor
200 Practice Set-16

(c) its resistance is increase


(a) π (b) π (d)
(d) i t b e c o m e s a p - t y p e s e m i - 4 2
conductor
39. The frequency of audio analog (c) zero (d) π 51. How many words are formed from
6 the letters of the word DAUGHTER
signals lies in the range:
when vowels (A, U, E) always are
(a) 20 kHz to 20 MHz MATHEMATICS
together?
(b) 12 Hz to 20 MHz 46. Let A = {0, 1, 3, 4}, B = {5, 6, 1, 3,
(c) 20 Hz to 20 MHz (a) 4320 (b) 2420
9} and C = {1,2, 3, 4, 9, 13} then
(d) 20 Hz to 20 kHz (A ∩ B) ∪ C is: (c) 3280 (d) 1240
52. The equation (m being real) mx2 +
40. Light of wavelength λ falls on a (a) C
2x + 1 = 0 has two distinct roots, if:
hc (b) {0, 1,2, 3, 4, 9, 13}
metal having work function . (a) m ≠ 0 (b) m ≠ 0, 1
λc (c) {0, 1, 3}
(c) m ≠ 1, –1 (d) m ≠ 0, 1, –1
Photoelectric effect will take place (d) {1, 3}
only if : 47. If z 1 and z 2 are two complex
numbers, then | z1 + z2 | is: 53. If J = , then J2 is equal
(a) λ ≥ λ0 (b) λ ≥ 2λ0
(a) < | z1 | + | z2 | to:
(c) λ ≤ λ0 (d) λ = 4λ0 (b) < | z1 | – | z2 | (a) 3 J (b) 2 J
41. 1 Wb/m2 is equal to: (c) < | z1 | – | z2 |
(a) 104 G (b) 102 G (d) > | z1 | + | z2 | (c) 4 J (d) J
(c) 10–2 G (d) 10–4 G 48. If in a set A the relation R is
54. Interior angles of any straight
42. T h e h o r i z o n t a l r a n g e o f a equivalence, then R–1 is:
line figure are in arithmetic
projective fired at an angle of 15º (a) Reflexive progression. Smallest angle is 88°
is 50 m. If it is fired with the same (b) Symmetric and common difference is 10°. The
speed at an angle of 45º, its range (c) Transitive number of sides will be:
will be: (d) All of these (a) 6 (b) 7
(a) 60 m (b) 71 m 49. p t h t e r m o f a n y a r i t h m e t i c (c) 5 (d) 8
(c) 100 m (d) 141 m 55. (n + 1) th term from the last in
43. A battery of 6 V is connected progression is and qth term =
to the terminals of a 3 m long expansion of will be:
wire of uniform thickness and , then sum of (pq) terms will be:
resistance of the order of 100
Ω. The difference of potential (a) (b)
between two points separated by (a)
50 cm on the wire will be:
(c) (d) 0
(a) 1 V (b) 1.5 V (b)
(c) 2 V (d) 3 V
44. The image formed by a convex 56. If tan2 45° – cos2 = x sin cos
(c)
mirror of focal length 30 cm is a
quarter of the size of the object. tan 60°, then the value of x is:
The distance of the object from (d)
the mirror is: (a) cos 60° (b) sin 60°
(a) 30 cm (b) 90 cm 50. The sum of n terms of series 8 + (c) cos 45° (d) sin 30°
(c) 120 cm (d) 60 cm 88 + 888 +........will be: 57. The value of
45. An AC voltage is applied to a (a) cos(90° + θ )sec( −θ )tan(180° − θ )
is
resistance R and an inductor L sec θ sin(180° + θ )cot(90° − θ )
in series. If R and the inductive equal to:
reactance are both equal to 3Ω. (b)
(a) 1 (b) ± 1
the phase difference between the (c) – 1 (d) None of these
applied voltage and the current in (c) 58. The circle x2 + y2 + 4x – 4y + 4 = 0
the circuit is: touches:
Practice Set-16 201

(a) x-axis 64. What are the order and degree


(b) y-axis of the differential equation
REASONING AND GENERAL
(c) x-axis and y-axis AWARENESS (RAGA)
(d) None of these ? 71. Which Brigadier was associated
with Jallianwala Bagh tragedy?
59. sin cos + cos sin is (a) General Dyer
(a) 1 and (b) 1 and 2
(b) Arthur Wellesly
equal to:
(c) 2 and 1 (d) 1 and 1 (c) General Harris
(a) (b) (d) Colonel Wellesly
65. The equation of the straight line
72. The oath of office is administered
which passes through the point
(c) (d) to the Governor by the:
(1, –2) and cuts off equal distances
(a) Chief Justice of India
from the two axes, will be:
60. If from vertices of a triangle A, (b) Speaker of Legislative
(a) x + y = 1 (b) x – y = 1 Assembly
B, C, perpendicular drawn on
opposite sides and its values are (c) x + y + 1 = 0 (d) x – y – 2 = 0 (c) President
p1, p2 and p3, then the value of p1 (d) Chief Justice of High Court
p2 p3 will be: 66. tan is equal to: 73. Which of the following states
is having longest coastline in
(a) (b) India?
(a) (b) (a) Andhra Pradesh
(c) (d) (b) Maharashtra
(c) (d) (c) Tamil Nadu
61. If three set of data had means of (d) Gujarat
15, 22.5 and 24 based on 6, 4, and 5 74. Which type of switching is used
67. The value of is:
observations respectively, then the in Internet?
means of these three sets combined (a) 1 (b) e2 π (a) Circuit (b) Telephone
is: (c) 0 (d) ∞ (c) Packet (d) Telex
(a) 20.0 (b) 20.5 68. An urn contains 6 balls of which 75. IMF stands for :
(c) 22.5 (d) 24.0 3 are white and 3 are black. (a) International Monetary
62. The condition that the line y = mx + They are drawn successively Function
c touches the parabola y2 = 4ax is: without replacement. What is (b) International Monetary Fund
(a) c = the probability that colours are
(c) Indian Manufacturing Firm
alternate?
(b) c = (d) Interest Minimum Firm
(a) 1/20 (b) 1/10 Function
(c) 1/5 (d) None of these 76. The gas liberated in the Bhopal
(c) c =
gas tragedy was:
69. is equal to:
(d) c = (a) Phenyl isocyanate
(a) x cos α + sin α. log {sin (x – α)} (b) Acetylene
sinx ...... ∞
63. If y = sinx sinx , then is: (b) (x – α) cos α + sin α log sin x (c) Ethylene
(c) x cos α + sin α. log sin x (d) Methyl isocyanate
(d) (x – α) cos α + sin α. log {sin x 77. Name the Indian entrepreneur,
(a)
(x – α)} who launched an initiative to
change Udhana railway station
(b) in Gujarat into India’s first
70. is equal green and clean railway station.
(a) Ritesh Agarwal
(c) to:
(b) Deepak Ravindran
(a) π (b) π/3
(c) Bhavin Turakhia
(d) None of these (c) π/2 (d) π/4
(d) Viral Sudhirbhai Desai
202 Practice Set-16

78. What is the value of 99992 × 84. The ratio of present ages of Direction (90): A series is given with
100008 ? Ranjini and Shahid is 5:4. After one term missing. Select the correct
(a) 9999999936 13 years the ratio of their ages alternative from the given ones that
(b) 9999899936 will be 6:5. What is Ranjini's will complete the series.
(c) 9999999836 present age? 90. Mercury, Venus, Earth, ?
(d) 9999999926 (a) 52 (b) 65 (a) Jupiter (b) Saturn
79. The least number by which 72 (c) 60 (d) 32 (c) Mars (d) Pluto
must be multiplied in order to Direction (91): Study the Following
85. Two cars travel from city A to
produce a multiple of 112, is: information carefully and answer the
city B at a speed of 36 and 54 km/
(a) 6 (b) 12 hr respectively. If one car takes questions given below:
(c) 14 (d) 18 3.5 hours lesser time than the A, B, C, D, E, F, G and H are sitting
other car for the journey, then around a circle facing the centre. B is
1
2+ the distance between City A and second to the right of H and third to
4
3 City B is : the left of A. D is not an immediate
80. If 5 = x, then the neighbour of either B or H and is
1 (a) 454 km (b) 567 km
2+ second to the right of F. C is fourth to
1 (c) 302 km (d) 378 km the right of G.
3+
1 86. ‘D’ is the wife of ‘B’ and ‘B’ is the
1+ 91. Who is to the immediate right of
4 brother of ‘R’. ‘R’ is the son of B?
|value of x is: ‘S’. How is ‘S’ related to ‘D’? (a) C
(a) 2 (b) 3 (a) Aunt (b) E
(b) Father-in-law (c) H
8
(c) 1 (d) (d) Data insufficient
7 (c) Data Inadequate
(d) Mother-in-law Direction (92): In following questions,
81. The average weight of Shubha, D means ‘is greater than’, % means
Govinda and Reshma is 65 kg. 87. If Z = 26, NET = 39, then NUT ‘is lesser than’, � means ‘is qual to’,
If the average weight of Shubha =? = means ‘is not equal to’, + means ‘is
and Govinda be 63 kg and that of (a) 50 (b) 53 a little more than’, × means ‘is a little
Govinda and Reshma be 70 kg,
(c) 55 (d) 56 less than’.
then the weight of Govinda is
88. Two horse drawn wagons start Choose the correct alternative in each
(a) 41 (b) 71
from the same farm. Wagon X of the following questions.
(c) 88 (d) 46 travels South 4.5 km and then
82. A student multiplied a number by 92. If c % b and b × a, then
turns right and travels 7.5 km. In
11/13 instead of 13/11. What the meanwhile wagon Y travels 9 (a) a D c (b) c � a
is the percentage error in the km North, then 3.5 km West, then (c) b � c (d) c D a
calculation ? 2 km North , then it turns left and
(a) 39.67 percent Direction (93) : Select the related
travels 4 km. Where is wagon Y
with respect to wagon X? word/letters/number from the given
(b) 28.4 percent
(a) 6.5 km North alternatives.
(c) 14.2 percent
(d) 19.83 percent (b) 15.5 km North 93. 97 : 2 : : 84 : ?
83. Ticket for an adult is ` 500 and (c) 15.5 km South (a) 4 (b) 3
a child is ` 200. 1 child goes free (d) 6.5 km South (c) 6 (d) 5
with two adults. If a group has 20 89. Lakshmi is elder than Meenu. Direction (94): Arrange the given
adults and 8 children what will Leela is elder than Meenu but words in the sequence in which they
be the discount the group get ? younger than Lakshmi. Latha is occur in the dictionary.
(a) 53.43 percent younger than both Meenu and 94. i. Catadrome ii. Catagenesis
(b) 13.79 percent Hari but Hari is younger than
iii. Catabolic iv. Catabolism
(c) 32.2 percent meenu. Who is the youngest?
(a) Lakshmi (b) Meenu (a) ii, iii, iv, i (b) iii, iv, i, ii
(d) 35.06 percent
(c) Leela (d) Latha (c) iii, iv, ii, i (d) i, ii, iii, iv
Practice Set-16 203

Direction (95): Study the following Jack. All of them have all the suits 98. Queen of spades is with
information to answer the given (spades, hearts, clubs and diamonds). (a) A (b) B
questions : (i) A has Ace of spades and Queen of (c) C (d) D
In a certain code ‘829’ means ‘how diamonds. 99. C has which of the following with
art thou’ ‘958’ means ‘thou art good’ and (ii) B has Ace of clubs and King of him?
‘15873’ means ‘thy good and thou bad’. diamonds. (a) Ace of hearts
95. What is the code for ‘thou’? (iii) C has Queen of clubs and King of (b) Jack of spades
(a) 9 (b) 8 spades. (c) King of hearts
(c) 2 (d) 5 (iv) D has Jack of clubs. (d) Queen of spades
Direction (96): Study the following 96. Ace of diamonds is with
100. D has which of the following with
information carefully and answer the (a) A (b) B
him ?
given questions : (c) C (d) D
(a) Ace of hearts
Four players A, B, C and D are 97. Jack of hearts is with
(b) Queen of hearts
holding 4 cards each. Each of them (a) A (b) B
(c) King of hearts
has an Ace, a King, a Queen and a (c) C (d) D
(d) Queen of clubs

Answers with Explanation


provoke is to anger, enrage, exasperate, 19. (b) T h e w o r d i n q u i s i t i v e
ENGLISH
or vex or to give rise to, etc. (Adjective) means tending to ask a lot of
1. (a) 2. (d) 3. (a) 4. (b) 14. (a) There is boys, a plural, before questions, especially about other people's
5. (a) Restaurant is correct spelling who so the verb after it will also be plural. affairs.
which means an establishment where Hence option (a) is correct. The word indifferent (Adjective)
meals are served to customers. 15. (d) The phrasal verb ' put out' means having no interest in somebody;
6. (c) Consumption - consuming would be used in the above sentence. not caring about somebody, Hence,
purchase and use of goods act. Please put out the candle when power inquisitive and indifferent are
7. (c) The direct speech for the above is restored. antonymous.
sentence will be. He said to me, "I don't Hence option (a) is correct. 20. (d) Remove 'the'
believe you". 16. (b) The error lies in part (b) of the
8. (a) The direct speech for the above
sentence. The infinitive has been wrongly PHYSICS
used. Hence, " to find" should be replaced 21. (d) From law of conservation of
sentence will be-
with "in finding" as difficulty (noun)
Rama said, " I am very busy now" . linear momentum, momentum of boat =
takes in as preposition. For example,
9. (b) The correct part is 'It could lead momentum of dog.
I had the greatest difficulty in
to the kind of good life-both spiritual and persuading here. m1v1 = m2v2
material-that was once the privilege of a 17. (b) It is possible to connect two Given, m1 = 40 kg, v1 = ?, m2 = 4 kg, v2
lucky few'. subjects with as well as before a verb. = 10 ms–1
10. (c) The word 'elegant' means If the first subject is singular the verb is
'graceful' and stylish in appearance or also likely to be singular. For example, ∴ 40 × v1 = 4 × 10 ⇒ v1 = = 1 ms–1
manner. So its synonym will be Graceful. Alice as well as Paula was shocked
11. (c) 'Timorous' means showing or 22. (b) Here, m = 0.5 kg, v = ax3/2,
by the news. Hence, are should be
suffering from nervousness or lack of where a = 5m–½s–1, W = ?
replaced with is.
confidence, hence its synonym will be 18. (a) The word exodus (Noun) Acceleration, A =
Timid. means a departure of many people at one
12. (c) Scarcity means the condition time. For example, The play was so awful
of deficient or inadequate amount of that there was a general exodus from the =
something as opposed to plenty, adequate, theatre at the interval.
sufficient, etc. The word influx means the sudden
A =
13. (c) People have tried their best to arrival of large numbers of peple.
provoke the flood victims. The antonym Hence, exodus and influx are
F = mA =
of provoke means pacify. Meaning of antonymous.
204 Practice Set-16

= 0.6 + 0.4 = 1 of the sphere. Electric lines of force


emitting from the point positive charge
W= = = 10%
and normally on the left side of the sphere
29. (d) Te m p o r a r y m a g n e t i s And due to positive charge on the right
= made from soft iron while magnet of side of the sphere, the electric lines of
loudspeaker is made parmanent magnet. force emanate normally from the right
W= 50 J 30. (b) As, conductor moves in the side. The electric field is best given by
magnetic field, Lorentz force acts on figure (i).
23. (d) If be the linear momentum
charged particle. Since, e = depends 35. (c) According to corpuscular
of a particle and is its position vector
from the point of rotation, then angular theory, different colours of light are due
upon the charges accumulated at both to different sizes of corpuscles.
momentum = = np sin θ , ends and rate of change of magnetic flux.
where is unit vector in the direction 36. (a) The simple Bohr model
of rotation. Hence, angular momentum 31. (a) Graph (a) shows the variation cannot be directly applied to calculate
vector is perpendicular to orbital plane. of temperature with time. At first levels of an atom with many electrons.
temperature will increase, then there This is because all the electrons in the
24. (c) Work done = Elastic potential will be state change from liquid to gas.
energy stored atom are not being subjected to one single
central force.
⇒ W= Stress × strain × volume 37. (a) The main difference between
Newton’s corpuscular theory and Planck’s
⇒W= = quantum theory is that the wave nature is
not associated with Newton’s corpuscles

25. (a) According to question first while quantum nature considered wave
gas goes from volume V to 3V and nature with quantas.
after this volume is reduced from 3V 32. (c) ∆U = (Given)
38. (b) When a small amount of
to V at constant pressure. In isothermal
antimony is added to germanium crystal,
expansion p-V curve is rectangular
hyperbola. the crystal becomes n-type semiconductor
∴ ∆U =
because antimony is a pentavalent
26. (c) The phase difference between
33. (a) According to Biot–Savart’s substrate. It excess free electrons.
the SHM is φ = 0. Therefore, resultant
amplitude is given by law, the magnetic field at a point 39. (d) We know that the frequency
distance r from a charge q moving with range of audio analog signals is 20 Hz to
A = 20 kHz.
a velocity is given by,
  hc

= μ 0 q (v × r ) 40. (c) The energy of photon, E = λ
=

= A1 + A2 = 6 + 8 = 14 4π r 3
hc
27. (b) Net charge on the parallel μ 0 v sin θ Work function of metal, W = λ
⇒ B=
plate capacitor is 1 µC 4π r 2
0

⇒ Charge on each plate of the For photoelectric effect E > W ⇒


capacitor. q = 0.5 µC The direct is along i.e., hc hc
>
Now, force is given by F = qE perpendicular to the plane containing λ λ0
Substituting q = 0.5 µC = 0.5 × 10–6 C, E and at a point obeys inverse square
= 105 Vm–1 λ ≤ λ0
law and not inverse cube law.
We get, F = 0.05 N 41. (a) We know that 1 G = 10–4 T
34. (a) When a point positive charge
28. (b) In series combination, R = R1 ⇒ 1T =
is brought near an isolatd conducting
+ R2 = 6 + 4 = 10kΩ
sphere, there developes some negative = 104 G
Error in combination,
charge on left side of the sphere and an
Also, 1 T = 1 Wb/m2
∆R= ∆R1 + ∆R2 = equal positive charge on the right side
⇒ 1 Wb/m2 = 104 G
Practice Set-16 205

u 2 sin (2 × 15º ) 48. (d) If R is equivalence, then R–1 52. (c) For real and distinct roots
42. (c) 50 = will also be equivalence. D> 0 or 4 – 4m > 0
g
49. (a) According to questions, ∴ m <1
⇒ = = a + (p – 1)d....(i)
53. (a) J2 =
In second case,
= a + (q – 1)d ....(ii)
R=
Solving equations (i) and (ii)
= =
= m a=d=

43. (a) Here, same current is passing = 3J


Spq =
throughout the length of the wire, hence 54. (c) Let number of sides in
polygon = n
⇒ = Sum of n integers angles = (n – 2) × 180°
According to question,
⇒ = = (n – 2) × 180° =

⇒ V2 = 1 V ⇒ (n – 2) × 180°= n [88 + (n – 1) × 5]
50. (a) Sn = 8 + 88 + 888 + ..........upto
⇒ 180n – 360 = 88n + 5n2 – 5n
n terms
44. (b) m = ⇒ = ⇒ ⇒ 5n2 – 97n + 360 = 0
= 8 (1 + 11 + 111 +..........upto n terms)
u = – 90 cm = upto n terms) ⇒n=

45. (a) tan φ =


= [(10 – 1) + (100 – 1) + (1000 – 1) =
tan φ = 1 +.....upto n terms]
π = [(10 + 102 + 103 +........upto n terms) =
φ = tan–1 (1) = 45º = rad
4
– (1 + 1 + 1 +..........upto n terms) ∴ n = 144 or 5
MATHEMATICS So, n=5
= 55. (a) If we have to find out any
46. (a) A ∩ B = {1, 3}.
term from the last, then we have to find
Since, C = {1, 2, 3, 4, 9, 13} what term it is from the beginning. For
∴ (A ∩ B) ∪ C = {1, 2, 3, 4, 9, 13} = this we subtract required term from total
47. (a) Let z1 = r1 (cos θ1 + i sin θ1) terms and will add 1 in that.
z2 = r2 (cos θ2 + i sin θ2) = ∴ Number of terms in expansion
∴ | z1 + z2 | = | r1(cos θ1 + i sin θ1) + r2 (cos  = (3n + 1)
θ2 + i sin θ2) | 51. (a) There are total 8 letters in ∴ (n + 1)th term from the last
=| (r1 cos θ1 + r2 cos θ2) + i (r1 sin θ1 + r2 given word, out of which 3 are vowels = [(3n + 1) – (n + 1) + 1]th term from
sin θ2) | and 5 are consonants. If we assume all beginning
vowels as one letter then now (1 + 5) = = (2n + 1)th term from beginning
= 6 letters are left which can be put as 6P6
= 6! places and 3 vowels can be placed = 3nC2n xn
on 3 places by3P3 = 3!
< Therefore, required numbers of = (–1)2n.3n C2n
words
= 6 ! × 3 !
[because cos (θ1 – θ2) < 1] < (r1 + r2) = 3n C2n
= 4320
< | z1 | + | z 2 |
206 Practice Set-16

56. (b) t a n 2 4 5 ° – c o s 2 = 59. (c) 64. (b) =0

=
⇒ =
⇒ tan2 45° – cos2 60° = x sin 45° cos
45° tan 60°
=
⇒ =
⇒ (1)2 – =
= =
Since, the highest derivative is whose
⇒1 – =
60. (d) p1p2p3 = order is 1 and degree is 2.

⇒ = 65. (c) Let each intercept be of


= = 8∆ .
3 length a.
∴ Equation of the line will be
⇒ x =
= 1
=
sin 330° = sin (360 – 30°)
⇒ x + y = a
= – sin 30° = = = Since, the line passes through (1, – 2)
∴1–2=a⇒a=–1
57. (c) 61. (a) The means of three sets ∴ Reqd. equation is x + y + 1 = 0
combined
= 66. (d) Let =x

=
cos x =
=
62. (b) Putting y = mx + c in y2 = 4ax,
we get
= (mx + c)2 = 4ax
⇒ m x + 2mcx + c2 – 4ax = 0
2 2

=–1 ⇒ m2x2 + 2(mc – 4a)x + c2 = 0


But if y = mx + c touches the parabola,
58. (c) The coordinates of the centre then
of the given circle are (–2, 2) and the (2mc – 4a)2 = 4m2c2
radius of the given circle ⇒ 4m2c2 – 16amc + 16a2 = 4m2c2 ∴ tan x =
c =

∴ x = tan–1
63. (a) y =
∴ y = (sin x)y ∴ = tan–1
Taking logarithms of both sides, we get
log y = y log (sin x)
Diff. w.r.t. x, we get
But, tan–1 + =
= y.

= =
= y cot x
∴ Distance of the centre from the
axes is equal to the radius.
∴ The given circle to touches both = tan =
the axes.
Practice Set-16 207

76. (d) The Bhopal gas tragedy


67. (a) Let A = REASONING AND GENERAL involved the leakage of poisonous
AWARENESS (RAGA) methyl isocyanate (MIC) gas and other
logA= chemicals at the Union Carbide India
71. (a) The Jallianwala Bagh
massacre took place on 13 April 1919 Limited (UCIL) pesticide plant in
when a crowd of nonviolent protesters Bhopal, Madhya Pradesh, on the night
= [By L’Hospital Rule] who had gathered in Jallianwala Bagh, of 2–3 December 1984. It is considered
Amritsar, Punjab were fired upon by the world’s worst industrial disaster.
= =0 troops of the British Indian Army under 77. (d) Viral Sudhirbhai Desai, a
the command of Colonel Reginald young entrepreneur from surat launched
∴ A = e0 = 1 Dyer. Dyer was removed from duty, but an initiative to change Udhana railway
he became a celebrated hero in Britain. station in Gujarat into India’s first green
68. (b) We can choose balls as black,
72. (d) As per Article 159 of Indian and clean railway station like Mumbai’s
white or white black alternately
Constitution, the Governor of a state has Chhatrapati Shivaji Terminus.
∴ Required probability
to take oath in the presence of the Chief Objective: In order to achieve the
=2× = Justice of the High court exercising mission of ‘Clean India-Green India’,
jurisdiction in relation to the State, or, Desai installed oxygen bomber and over
in his absence, the senior most Judge of 1700 air purifying plants at the station
69. (d) I = and also painting has been made by him
that Court available. The Governor of a
State is appointed by the President. for environmental protection on the wall
Let x – α = θ ⇒ dx = dθ
of the station.
73. (d) Gujarat, in the north
∴ I = western region of India, has the longest 78. (a) The expression,
coastline, covering more than 1,600 km. 99992 × 100008 = 9999999936
It accounts for 22% of total coastline of 79. (c) The least number 14 by
=
the country. Its coast is bordered by the which 72 is multiplied gives 1008 in
Arabian Sea and the Gulfs of Khambat order to produce a multiple of 112.
=
and Kachchh. Its coastline nurtures 80. (c)
= θ cos α + sin α log (sin θ) a diversity of habitats, especially 1 5 5
= (x – α) cos α + sin α log {sin(x – α)} mangroves, salt marshes, coral reefs, 2+ 2+ 2+
(19 / 5) 29 = 19
wetlands, and sea grasses. x= =
1 1 1
70. (d)  = 2+ 2+ 2+
74. (c) Packet switching is the 3+
1
3+
4 (19 / 5)
dividing of messages into packets before (5 / 4) 5
∴ I = they are sent, transmitting each packet 5
individually, and then reassembling 2+
= 19 =1
them into the original message once 5
2+
all of them have arrived at the intended 19
destination. A packet is the fundamental 81. (b) Total weight of Shubha,
=
unit of information transport in internet Govinda and Reshma = 65 × 3
that uses the datagram packet switching S + G + R = 195kg ...(i)
method. Most modern Wide Area Total weight of Subha Govinda
Network protocols, including TCP/ = 63 × 2 = 126kg
= IP, are based on packet-switching S + G = 126kg ...(ii)
technologies. Total weight of Govinda and Reshma
75. (b) IMF stands for International
= 2 × 70 = 140
∴2I = Monetary Fund. It is an organisation
of 188 countries, working to foster = 140 kg ...(iii)
= global monetary cooperation, secure      S + 2G + R
financial stability, facilitate international = 126 + 140 = 266kg ...(iv)
= = π/2 trade, promote high employment and Subtracting equation (i) from (iv)
sustainable economic growth, and Weight of Govinda
∴I= reduce poverty around the world. It is = 266 – 195 = 71 kg
headquartered in Washington D.C., USA.
208 Practice Set-16

82. (b) Let the number be x. Then, 89. (d) According to the question,
13 11
x– x 54(t – 3.5) = 36t Lakshmi > Meenu
% error = 11 13 × 100% (54 – 36)t = 54 × 3.5 Lakshmi > Leela > Meenu
13 54 × 3.5 Meenu > Hari > Latha
x t= = 3 × 3.5
11 18 On arranging the above data, we get
48 = 10.5 Lakshmi > Leela > Meenu > Hari
x > Latha
11 × 13 Distance = 36t = 36 × 10.5
= × 100% 21 Hence, Latha is the youngest.
13
x = 36 × 90. (c) The series is:
11 2
Mercury, Venus, Earth, Mars
48 x × 11 = 18 × 21 = 378 km
× 100% Finally, the missing term is Mars.
  = 86. (b) The Blood relationship tree/
11 × 13 × 13 91. (d) By analysing the above
chart can be constructer as given below:
4800 circular arrangement, it is clear that
= % = 28.4%
169 C or G  immediate right of B.
Therefore, the given data is
83. (b) 20 cost of tickets of adults inadequate.
and 8 children with discount 92. (a) According to the given
=  16 adults + 8 children + 4 information
adults D  B's wife. c % b  c < b
= 8 (2 adults + 1 child) + 4 adults B  R's brother. and b × a  b is a little less than a
     = 8 × (2 × 500) + 4 × 500 R  S's son. From (i) and (ii), it is clear that
= 8000 + 2000 Therefore, ‘S’ is the father-in-law c<a
= ` 10000 of ‘D’.   a > c  a D c.
Without discount cost of tickets 93. (a) As, 97 → (9 – 7) → 2
87. (c) Z  26 Position number in
= 20 adults + 8 children Similarly, 84 → (8 – 4)→ 4
English alphabet.
= 20 × 500 + 8 × 200 Finally, the missing number is 4.
As,
= 10000 + 1600 = ` 11600 94. (b) According to dictionary, the
Discount = 11600 – 10000 N E T
sequence of the words is :
= ` 1600 ↓ ↓ ↓
(iii) Catabolic,
1600 ×100% 14 + 5 + 20 = 39 (iv) Catabolism,
Discount % = Similarly,
11600 (i) Catadrome,
N U T (ii) Catagenesis.
1600 ↓ ↓ ↓
= = 13.79% 95. (b) The code for ‘thou’ is ‘8’.
116 14 + 21 + 20 = 55
84. (b) Let present ages of Rajini 96. (d) By analysing the above
88. (b) table, we get
and Shahid be 5x and 4x. End
4 km With
According to question 2 km 3.5 km Ace of diamonds D.
Horse B (Wagon Y)
5 x + 13 97. (a) By analysing the above
6 9 km
= table, we get
4 x + 13 5 With
Start Jack of Hearts A.
25x + 65 = 24x + 78 98. (d) By analysing the above
4.5 km
x = 78 – 65 = 13 Horse A (Wagon X) table, we get
End
Present age of Rajini 7.5 km
Queen of spades
With
D.
N
= 5 × 13 = 65 years 99. (a) By analysing the above
85. (d) Let the time taken by first W E table, we get
car be t hours. has
S
C Ace of hearts
Time taken by other car = (t – 3.5)
hours. 100. (c) By analysing the above
The required distance
Distance = Speed × Time table, we get
= (4.5 + 9 + 2) km = 15.5 km
But distance will be same in both has
Hence the wagon Y is 15.5 km North D king of heart.
cases.
with respect to wagon X. qqq
Practice Set-17 209

Air Force Airmen Group X & Y Exam


Practice Set-17

(b) As other energy forms pollute 8. Perfect


ENGLISH the environment (a) imperfect (b) ugly
Directions (1-4): Read the passage given (c) Increasing costs of other sources
below and answer the questions that (c) defect (d) complete
(d) All of the above
follow by selecting the correct/most Directions (9): In the following question,
2. What is the major difficulty in
appropriate options. out of the four alternatives, choose the
employing solar energy?
In these times of worldwide skyrocketing one which best expresses the meaning of
energy costs, experts are turning to (a) Fluctuations in weather the given word as your answer.
perhaps the only energy source that is (b) By nature sunlight is diffuse 9. Apparent
still immune to price fluctuating: the (c) Sunlights varies from time to time (a) glaring (b) vague
Sun. Economic necessity appears to (d) All of the above (c) similar (d) dress
be largely responsible for the rapidly 3. Experts are impressed with the 10. Fill in the blanks with a suitable
growing acceptance of solar heat as an solar energy technologies because
alternative source. Electricity has been preposition or verb.
(a) of them being good for the He ______the bus when it came.
a clean and versatile form of energy
that continues to grow in importance for environment
(a) got off (b) got on
lighting, heating and cooking, and for (b) they require less investment
(c) got in (d) none of the
powering our increasingly automated and (c) they can be employed in quick time above
computerised society. The demand for (d) All of the above 11. Choose the most appropriate
electricity is expected to grow more in 4. Why are power generating preposition:
future than the average trend to the past. companies looking at mix energy
A mix of energy sources is required as Your conduct smacks_______
options?
hedge against shortages in any one area, recklessness.
(a) The sources of energy are getting
and the electricity supply companies (a) in (b) from
scarce but demand is increasing
are taking a second look at renewable (c) with (d) of
generating technologies, particularly (b) Government is controlling price
12. The passive voice for the sentence
those based in the Sun. of electricity
is:
Every 15 minutes, the sun delivers (c) Alternative sources of energy
are available free and without The people will elect him President.
to Earth radiant energy to meet all
mankind’s power needs for a full year. regulations (a) The people will not elect him
But harnessing this energy is complicated (d) All of the above President.
by two properties of sunlight: its Directions (5-6): Select the correct (b) He will be elected President by
diffuseness and it's variability with time spelling: the people.
of day, seasons and weather conditions. 5. (a) Staff Quarters (c) Will he be elected President?
These factors pose formidable technical (b) Stuff Quarters (d) The people may not elect him.
challenges for the efficient conversion 13. Find out the correct sentence:
(c) Staff Quaters
of solar radiation into bulk, utility grade
electric power. Nevertheless, solar (d) None of the above (a) My son is desirous in joining the
technologies are attractive to utilities 6. (a) bivouck (b) bivouk Army.
because they are environment-friendly (c) bivonac (d) bivouk (b) My son is desirous on joining
and offer a lower regulatory risk, limited Directions (7-8): In the following the Army.
capital risk and less lead time. questions, choose the word opposite (c) My son is desirous of joinng the
1. Why are experts, according to the in meaning to the given word as your Army.
passage, looking at the Sun as an answer. (d) My son is desirous for joining
energy source? 7. Vauge the Army.
(a) Due to depletion of other (a) unclear (b) definite 14. What does the idiom "Hue and
sources of energy (c) cloudy (d) ambiguous Cry" means?
210 Practice Set-17

(a) to keep a loof from 26. A simple pendulum is made of


(b) to create crisis a body which is a hollow sphere
(c) a general outcry of alarm containing mercury suspended
(d) to conclude by means of a wire. If a little
(a) (b) mercury is drained off, the period
15. What is the antonym of 'Insolvent'?
(a) Bankrupt (b) Wealthy of pendulum will:
(c) Intuition (d) Translate (c) (d) (a) remain unchanged
16. Find out the correct indirect (b) increase
narration of the following direct 22. A bicyclist comes to a skidding stop (c) decrease
speech: in 10 m. During this process, the (d) become erratic
He said to them all, "Good-bye, my force on the bicycle due to the road 27. If the electric flux entering and
friends!" is 200 N and is directly opposed to leaving an enclosed surface
(a) He said to his friends good-bye the motion. The work done by the respectively is φ 1 and φ 2 , the
(b) He said to his friends good-bye cycle on the road is: electric charge inside the surface
(c) He bade good-bye to all his will be:
(a) + 2000 J (b) – 200 J
friends (c) zero (d) – 20000 J (a) (φ1 + φ2) ε0 (b) (φ2 — φ1) ε0
(d) None of these 23. Two identical ball bearings in
Directions (17-19): In the following (c) zero (d)
contact with each other and resting
questions some parts of the sentences
have errors and some are correct. Find on a frictionless table are hit head-
28. Which of the following is based on
out which part of a sentence has an on by another ball bearing of the
the law of conservation of energy?
error. The number of that part is your same mass moving initially with a
(a) Lenz’s law
answer. If a sentence is free from errors, speed v. If the collision is elastic,
then your answer is (d) i.e. No error. (b) Faraday’s law
which one of the following is a
(c) Ampere’s law
17. He caught the boy (a)/steal (b)/the possible result after collision?
(d) Biot-Savart law
parcel. (c)/No error. (d)
(a) 29. A uniform electric field and
18. I am (a)/taller than you (b) / aren't I? a uniform magnetic field are
(c)/No error. (d) produced, pointed in the same
(b) direction. An electron is projected
19. A man of fifty (a) / cannot be called
(b) / as young. (c) / No error. (d) with its velocity pointing in the
(c) same direction.
20. Change the Narration.
(d) None of the above (a) The electron velocity will
Gavaskar said, "Bravo! Azhar joy
decreased in magnitude
have done well" 24. The wettability of surface by a
(b) The electron velocity will
(a) Gavaskar exclaimed with joy liquid depends primarily on:
increased in magnitude
that azhar had done well. (a) viscosity (c) Neither (a) nor (b)
(b) Gavaskar called Azhar and (b) surface tension (d) None of the above
exclaimed that he had done done (c) density 30. I f s p e c i f i c re s i s t a n c e o f a
well. (d) angle of contact between the potentiometer wire is 10 –7 Ωm
(c) Gavaskar congratulated Azhar surface and the liquid and current flow through it is 0.1
saying that he had done work. 25. A person of mass 60 kg wants to A, cross-sectional area of wire is
(d) Gavaskar praised Azhar for his lose 5 kg by going up and down a 10–6 m2, then potential gradient
having done well. will be:
10 m high stairs, Assume he burns
twice as much fat while going up (a) 10–2 Vm–1 (b) 10–4 Vm–1
PHYSICS (c) 10–6 Vm–1 (d) 10–6 Vm–1
21. A block B is pushed momentarily than coming down. If 1 kg of fat
is burnt on expending 7000 kcal, 31. Two different masses m and 3m of
along a horizontal surface with an ideal gas are heated separately
an initial velocity v. If µ is the how many times must be go up and
in a vessel of constant volume.
coefficient of sliding friction- down to reduce his weight by 5 kg?
The pressure p and absolute
between B and the surface, block (a) 10.5 × 103 (b) 24.3 × 103 temperature T graphs for these
B will come to rest after a time: (c) 16.3 × 103 (d) 9 × 103 two cases are shown in the figure
Practice Set-17 211

as A and B. The ratio of slopes of (a) 5 I and I (b) 5I and 3I 42. What are dimensions of Young’s
curves B to A is: (c) 9 I and I (d) 9I and 3I modulus of elasticity:
36. Newton based his corpuscular (a) [ML–1T–2] (b) [MLT–2]
theory on: (c) [MLT ] –1
(d) None of these
(a) Newton’s ring 43. The number of dry cells each of
(b) rectilinear propagation of light emf 1.5 V and internal resistance
(c) thin film colours 0.5 Ω that must be joined in series
(d) dispersion of white light into with a resistance of 20 Ω so as to
various colours send a current of 0.6 A through the
37. The mother and daughter elements, circuit is:
with the emission of γ-rays are: (a) 2 (b) 8
(a) 3 : 1 (b) 1 : 3 (a) isotopes (b) isobars (c) 10 (d) 12
(c) 9 : 1 (d) 1 : 9 (c) isomers (d) isodiapheres 44. A simple telescope, consisting of an
32. The ratio of mean distance of three 38. A common emitter amplifier has objective of focal length 60 cm and
planets from the sun are 0.5 : 1 : a voltage gain of 50, an input a single eye lens of focal length 5
1.5, then the square of time periods impedance of 100 Ω and an output cm is focused on a distant object
are in the ratio of: impedance of 200 Ω. The power in such a way that parallel rays
(a) 1 : 4 : 9 (b) 1 : 9 : 4 gain of the amplifier is: emerge from the eye lens. If the
(a) 500 (b) 1000 object subtends an angle of 2º at
(c) 1 : 8 : 27 (d) 2 : 1 : 3
the objective, the angular width of
33. A capacitor having capacitance (c) 1250 (d) 50
the image is:
1 µF with air, is filled, with two 39. What should be minimum length of
dielectrics as shown. How many antenna for efficient transmission (a) 10º (b) 24º
times capacitance will increases? of signals of wavelength λ?
(c) 50º (d)
(a) (b)
45. In a LCR series circuit, the
potential difference between the
(c) (d) terminals of the inductance is 60
V, between the terminals of the
(a) 12 (b) 6 40. Photons absorbed in matter capacitor is 30 V and that across
are converted to heat. A source the resistance is 40 V. Then, supply
(c) (d) 3 emitting n photon/s of frequency voltage will be equal to:
v is used to convert 1 kg of ice at (a) 50 V (b) 70 V
34. Two charged particles traverse 0ºC to water at 0ºC. Then, the time (c) 130 V (d) 10 V
identical helical paths in a T taken for the conversion:
(a) increases with increasing n, with MATHEMATICS
completely opposite sense in a
v fixed 46. If A = {1, 2}, B = {2, 3} and C = {3,
uniform magnetic field :
(b) increases with n fixed, v 4}, then what is the cardinality of
(a) They have equal z-components increasing
of moments (A × B) ∩ (A × C)?
(c) remains constant with n and v
(b) They must have equal charges (a) 8 (b) 6
changing such that nv = constant
(c) They necessarily represents a (c) 2 (d) 1
(d) increases when the product nv
particle anti-particle pair increases
(d) The charge to mass ratio satisfy 47. is equal to:
41. A ball is moving in a circular
 e  e  path of radius 5 m. If tangential
(a) 1 (b) 2
 m  +  m  = 0 acceleration at any instant is 10
1 2 (c) 3 (d) 0
m/s 2 and the net acceleration
35. Two coherent monochromatic light makes an angle 30º with the
beams of intensities I and 4I are 48. I f , then
centripetal acceleration, then the
superposed. The maximum and instantaneous speed is:
xa . yb . zc is equal to:
minimum possible intensities in
(a) m/s (b) 9.3 m/s (a) a (b) abc
the resulting beam are:
(c) 6.6 m/s (d) 5.4 m/s (c) xyz (d) None of these
212 Practice Set-17

49. If for positive real numbers x, y 56. Which one of the following points EXAMINATION are listed as in
and c the numbers x + y, 2y and lies inside a circle of radius 6 and a dictionary, how many words are
y + z are in harmonic progression, centre at (3, 5)? there in this list before the first
then which one of the following is (a) (–2, –1) (b) (0, 1) word starting with E?
correct? (c) (–1, –2) (d) (2, –1) (a) 908300 ways (b) 907200 ways
(a) x, y and z are in geometric 57. If log12 27 = a, then log6 16 is equal (c) 907300 ways (d) None of these
progression to:
(b) x, y and z are in arithmetic 64. Two persons A and B appear in an
(a) (b)
progression interview for two vacancies. If the
(c) x, y and z are in harmonic
progression (c) (d) None of these probability of their selection are
(d) None of the above
and respectively, then the
58. 23n – 7n – 1 is divisible by:
50. What is ∑ C( n, r ) equal to? (a) 64 (b) 36 probability that none of them is

(a) 2n – 1 (b) n (c) 49 (d) 25 selected:


(c) n! (d) 2n 59. What is the degree of the equation,
(a) (b)
51. If the rth, (r + 1)th and (r + 2)th
coefficient of (1 + x)n are in AP, = ?
then x is a root of the equation. (c) (d)
(a) x2 – x (4r + 1) + 4r2 – 2 = 0 (a) 1 (b) 2
(b) x2 + x (4r + 1) + 4r2 – 2 = 0 (c) 3 (d) 4 65. Sketch a rough graph of the curve
(c) x2 + x (4r + 1) + 4r2 + 2 = 0 60. Mean of 9 observations is 100 and and find the area of the region
mean of 6 observations is 80, then bounded by y2 = x, x = 1, x = 4 and the
(d) None of the above
the mean of 15 observations is: X-axis.
52. The area of circle centred at (1, 2)
and passing through (4, 6) is: (a) 92 (b) 29
(a) sq. units (b) sq. units
(a) 5π sq. units (b) 10π sq. units (c) 1380 (d) 15
(c) 25π sq. units (d) 15π sq. units 61. If
(c) sq. units (d) None of these
53. What is is equal to? cos x + cos x + cos x + cos x + ......∞
dy 66. If a cos A = b cos B, then ∆ABC is:
then is equal to:
dx (a) isosceles (b) right angle
(a) (b)
(c) equilateral (d) Either (a) and (b)
(a) (b)
(c) (d) 2x 1 − x2
(c) (d) 67. 3 sin −1 − 4 cos −1 +
1 + x2 1 + x2
54. If A (2, 3), B (1, 4),C (0, –2) and
D (x, y) are the vertices of a 62. In the mean value theorem = 2x π
2 tan −1 = , then x is equal
parallelogram, then what is the f ( b) − f ( a) 1 1 − x2 3
= f ' ( c) , If a = 0, b =
value of (x, y)? b− a 2 to:
(a) (1, –3) (b) (2, 4) and f (x) = x (x – 1) (x – 2), the value
(c) (1, 1) (d) (0, 0) (a) (b)
of c is: (c) 1 (d) — 1
55. The value of
(a) (b) 68. Find the equation of the hyperbola,
is:
the length of whose latusrectum is
(a) loge 2 – loge 3 (c) (d) 8 and eccentricity is .
(b) loge 3 – loge 2
(c) loge 6 63. If the different permutations (a) (b)
(d) None of these of all the letters of the word
Practice Set-17 213

75. FORTRAN is called: (a) 200 (b) 312


(c) (d) None of these (a) Formula Translator (c) 390 (d) 234
(b) Format Translator 83. 30% discount is offered on an
π
69. If α + β = 2 and β + γ = α, then (c) File Translator item. By applying a promo code,
(d) Floppy Translator the customer wins 5% cash back.
tan α is equal to:
76. Whose army did Alexander, What is the effective discount ?
(a) 2 (tan β + tan γ)
the Greek ruler confront on the
(b) tan β + tan γ (a) 33.5 percent
banks of the river Jhelum?
(c) tan β + 2 tan γ (b) 35.175 percent
(a) Chandragupta Maurya
(d) 2 tan β + tan γ (c) 35 percent
(b) Ambi
 2a   1 − b2  (c) Dhanananda (d) 6.5 percent
70. If sin −1   − cos −1   =
 1 + a2   1 + b2  (d) Porus 84. Find two numbers such that
 2 x  , then what is the
77. Where was the 35th Goods and their mean proportional is 16
tan −1  Service Tax (GST) Council and third proportional is 1024.
 1 − x 2 
Meeting held recently? (a) 4 and 32 (b) 4 and 64
value of x? (a) Chennai (b) Kolkata
(c) 8 and 64 (d) 8 and 32
(a) (b) ab (c) New Delhi (d) Mumbai
78. Two fractions are such that their 85. Two cars travel from city A to
product is 4 and sum is 68/15. city B at a speed of 30 and 48 km/
(c) b (d) Find the two fractions. hr respectively. If one car takes 3
a
(a) 6/15, 10/3 (b) 6/5, 10/3 hours lesser time than the other
(c) 7/2, 8/7 (d) 10/7, 14/5 car for the journey, then the
REASONING AND GENERAL distance between City A and City
79. What least number must be
AWARENESS (RAGA) B is :
subtracted from 427398 so that
71. The most suitable soil for the remaining number is divisible
(a) 288 km (b) 240 km
production of cotton is: by 15?
(a) Black lava soil (c) 360 km (d) 192 km
(a) 3 (b) 13
(b) Alluvial soil 86. X told Y, "Though I am the son
(c) 11 (d) 12
(c) Loamy soil of your father, you are not my
80. The price of rice increases by 40
(d) Well drained soil brother". How is X related to Y?
paise every year, while the price
72. Perfectly inelastic demand is (a) Sister (b) Son
of wheat increases by 15 paise
equal to:
every year. If in 2000, the price (c) Daughter (d) Brother
(a) One
(b) Infinite of rice was ` 4.20 and that of 87. If F = 6, MAT = 34, then how
(c) Zero wheat was ` 6.30, in which year much is CAR?
(d) Greater than one rice will cost 40 paise more than
(a) 21 (b) 22
73. Impeachment Proceedings the wheat?
(a) 2011 (b) 2019 (c) 25 (d) 28
against the President for
violation of the Constitution can (c) 2012 (d) 2009 88. Laxman went 15 km to the West
be initiated in: 81. The average revenues of 9 from house, then turned left and
(a) Either House of Parliament consecutive years of a company is walked 20 km. He, then turned
(b) The Lok Sabha ` 68 lakhs. If the average of first East and walked 25 km and
(c) The Rajya Sabha 5 years is ` 63 lakhs and that of finally turning left covered 20
(d) The Supreme Court last 5 years is ` 75 lakhs, find the km. How far is he now from his
74. Soldering of two metals is revenue for the 5th year. house?
possible because of the property (a) ` 80 lakhs (b) ` 76 lakhs (a) 15 km (b) 20 km
of: (c) ` 78 lakhs (d) ` 74 lakhs
(a) Viscosity (c) 25 km (d) 10 km
82. When a number is increased by 89. In a cricket team, Dhoni is
(b) Osmosis
26, it becomes 113% of itself. taller than Virat but not as
(c) Cohesion
What is the number? tall as Raina, Rohit is shorter
(d) Surface tension
214 Practice Set-17

than Dhoni but taller than Choose the correct alternative in each Directions (96-100): Study the
Shikhar. Who among them is of the following questions. following information carefully and
the shortest? 92. If ac % bd and ab D cd, then answer the given questions:
(a) Dhoni (a) b � c (b) b D a (i) B and E are good in Dramatics
(b) Virat and Computer Science.
(c) a % c (d) Can't say
(c) Shikhar (ii) A and B are good in Computer
Direction (93): Select the related
(d) Cannot be determined word/letters/number from the given Science and Physics.
Direction (90): A series is given alternatives. (iii) A, D and C are good in Physics
with one term missing. Select the 93. In the following question, select the and History.
correct alternative from the given missing number from the given (iv) C and A are good in Physics and
ones that will complete the series. series.
Mathematics.
90. AB, BC CD, DE, EF, ?
(v) D and E are good in History and
(a) G H (b) FE 5 15 10 Dramatics?
(c) FG (d) IJ
4 17 9 96. Who is good in Physics, History
Direction (91): Study the following
and Dramatics?
information carefully and answer
the questions given below: 10 ? 10 (a) A (b) B
A, B, C, D, E, F, G and H are (c) D (d) E
sitting around a circle facing the (a) 11 (b) 12 97. Who is good in Physics, History
centre. B is second to the right of (c) 15 (d) 10 and Mathematics, but not in
H and third to the left of A. D is Direction (94): Arrange the given Computer Science ?
not an immediate neighbour of words in the sequence in which they (a) A (b) B
either B or H and is second to the occur in the dictionary. (c) C (d) D
right of F. C is fourth to the right 94. i. Foresay ii. Foreseize 98. Who is good in Computer
of G. iii. Foreseeable iv. Foresail Science, History and Dramatics?
91. Who is third to the right of E? (a) i, iii, iv, ii (b) i, iii, ii, iv (a) A (b) B
(a) C (c) iii, i, ii, iv (d) iv, i, iii, ii (c) C (d) E
(b) F
Direction (95): Study the following 99. Who is good in History,
(c) D information to answer the given Physics, Computer Science and
(d) Data insufficient questions : Mathematics ?
Direction (92): In following In a certain code ‘829’ means ‘how (a) A (b) B
questions, D means ‘is greater art thou’ ‘958’ means ‘thou art good’ and (c) C (d) D
than’, % means ‘is lesser than’, ‘15873’ means ‘thy good and thou bad’.
100. Who is good Physics, Dramatics
� means ‘is qual to’, = means ‘is 95. What is the code for ‘how’?
and Computer Science?
not equal to’, + means ‘is a little
(a) 5 (b) 8 (a) A (b) B
more than’, × means ‘is a little less
than’. (c) 2 (d) 9 (c) D (d) E

Answers with Explanation


2. (d) Sun is a vital source of clean are good for the environment and requires
ENGLISH less investment.
energy but there is major difficulty in
1. (c) In the given passage, experts employing solar energy becasue of 4. (a) T h e p o w e r g e n e r a t i n g
are looking at the sun as an energy source fluctuations in weather, diffused sunlight, companies are looking at mix energy
due to increasing costs of other sources etc. It also varies from time to time. options because the traditional sources of
3. (d) Experts are impressed with the energy are getting scarce and the demand
like coal or petrol.
solar energy technologies because they of energy is increasing rapidly.
Practice Set-17 215

5. (a) Staff Quarters is correct. Hence PHYSICS 27. (b) For a closed surface outward
option (a) is correct. flux is taken as positive and inward flux
6. (c) The correctly spelt word is – 21. (a) Retardation due to friction
is taken as negative.
bivouac which means a temporary camp = –µg
for soldiers or mountaineers. Hence Initial velocity = v Total charge enclosed 1
Net flux = = ×Q
option (c) is correct. Now using,v = u + at ε0 ε0
7. (b) The word Vague (Adjective) Final velocity is zero ⇒ 0 = v – µgt ∴ Q = ε0 (φ2 – φ1)
means: not distinct; not definite; not clear. v 28. (a) Lenz’s law is based on the law
Its antonym should be definite. ⇒t= of conservation of energy.
μg
Look at the sentence:
22. (c) As the road does not move 29. (d) There will be no force on
They had only a vague idea where the
at all, the work done by cycle on road is electron due to magnetic field (because of
place was.
zero. parallel motion) but due to force applied
8. (a) The word Perfect (Adjective)
means: complete and without faults or by electric field, velocity of electron will
23. (b) Before collision, total kinetic
weaknesses; exact and accurate. decrease.
Its antonym should be imperfect. energy of system = 30. (a) Potential gradient = Potential
9. (a) The word Apparent (Adjective) fall per unit length. Therefore, resistance
means: visible, obvious, seeming, easy to After collision, kinetic energy of system of unit length,
see or understand. ρI 10 –7 × 1
in case (a) = =
The word Glaring means very easily R = = = 10 –1 Ω
seen. A 10 –6
Look at the sentence: in case (b) = ] Potential fall across R is
It was apparent from her face that she V = IR = 0.1 × 10–1
was really upset. = 0.01 Vm–1 = 10–2 Vm–1
10. (b) Preposition 'got on' is the in case (c) =
31. (a) For a gas, pV = nRT =
correct option.
11. (d) From the above equation, it is observed
12. (b) The passive voice for above that kinetic energy is conserved only
sentence will be. He will be elected in case (b), hence case (b) is the only
president by the people. All other options possible case after collision.
are wrong. Hence option (b) is correct. 24. (d) The value of angle of contact
13. (c) The correct usage is – desirous determines whether a liquid will spread
of. Hence option (c) is correct. on the surface.
14. (c) 'Hue and Cry' means a general 25. (c) 5 × 7000 × 103 × 42 J = 60 ×
outcry of alarm. Hence option (c) is
correct. 15 × 10 × N
Slope of graph =
15. (b) Insolvent is a person unable N =
to pay his debts. So, its opposite would
be a wealthy person. Hence option (b) is Similarly, slope of graph =
= 16.3 × 103 times
correct.
26. (b) Length of simple pendulum
16. (c) The correct indirect sentence
is – He bade good-bye to all his friends. = distance between point of suspension
and centre of gravity of system. ∴ = =
Hence option (c) is correct.
17. (b) Here, Gerund i.e. stealing ..... On draining off a little mercury, the
should be used. centre of gravity gets lowered, that is,
18. (c) Here, question tag i.e. amn't I the length of simple pendulum increases. 32. (c) , r1 : r 2 : r 3 =
....... should be used. The time period of simple pendulum is
19. (c) It is not proper to use 'as' there given by:
∴ T21 : T22 : T23 = = 1 : 8 : 27
called = having a particular name. l
Look at the sentences: T = 2π ⇒
g 33. (b) As shown, the two capacitors
I don't know anyone called Mohan.
The drug is commonly known as Ecstasy. As length of simple pendulum are connected in parallel. Initially, the
increases, the time period increases. capacitance of capacitor.
20. (a)
216 Practice Set-17

to a quater of wavelength of the frequency Here, f0 = 60 cm, fe = 5 cm, α = 2º


used. So for efficient transmission of Hence, B = 24º
signals of wavelengths λ the minimum 45. (a) In LCR series circuit,
λ
length of antenna should be . V =
4
40. (c) Energy spent to convert ice
=
ε0A
C= into water = mL = (1000 g) × (40 cal/g)
d = 80000 cal
= = = 50V
Where, A is area of each plate and
Energy of photons used
d is the separation between the plates. MATHEMATICS
After filling with dielectrics, we = nT × E = nT × hv ( E = hv)
have two capacitors of capacitance. 46. (c) Given that, A= {1, 2}, B = {2,
 A ∴ nThv = mL or T = 3} and C = {3, 4}
K1ε 0  
C1 =
 2  8 ε 0 A 4ε 0 A
= = = 4C
Now, (A × B)= {1, 2} × {2, 3}
d 2 d d ∴ when v is constant, = {(1, 2), (1, 3), (2, 2), (2, 3)}
 A and (A × C) = {1, 2} × {3, 4}
K 2ε 0  
and  2  4 ε 0 A 2ε 0 A
C2 = = = = 2C when n fixed; = {(1, 3), (1, 4), (2, 3), (2, 4)}
d 2 d d
Hence, their equivalent capacitance ∴ (A × B) ∩ (A × C) ={(1, 3), (2, 3)}
Ceq= C1 + C2 = 4C + 2C = 6C , Thus T is constant if nv is So the cardinality of (A × B) ∩ (A × C)
i.e., new capacitance will be six is 2.
constant. 2  1 + i 2 + 2i 
time of the original. 1+ i 
41. (b) From relation, 47. (b)   =   =i
34. (d) In a uniform magnetic field,  2  2 
the two charged particles will traverse
tan θ =
identical helical paths in a completely
∴ = i4 = 1
opposite sense. It the charge/mass ratio ar 10 × 5
of these two particles is same and charges v2= t = = 50 3 8
tan θ tan 30º 1− i
on them are of opposite character. In this ∴   = (–i) = 1
4

situation (e/m)1 + (e/m)2 = 0 holds good.


= 50 × 1.732 = 86.6 2
35. (c) I max = ( I1 + I 2 )2 = ( I + 4I)2 = 9I v= = 9.3 m/s ∴ LHS = 1 + 1 = 2
I = ( I − I ) 2 = ( I − 4I) 2 = I
min 1 2
42. (a) γ= 48. (d) Let = = =k
36. (d) Newton based his corpuscular
theory on dispersion of white light into ⇒ log x = k(b – c) ⇒ x = 10k(b – c)
various colours. ∴ [γ] = = ⇒ log y = k (c – a)
37. (c) The mother and daughter ⇒ y = 10k(c – a)
= [ML–1T–2] ⇒ log z = k(a – b)
elements with the emission γ-rays are
Hence, Nm –1 is not the unit of ⇒ z = 10k(a – b)
called isomers.
⇒x . y . z = 10ka(b – c) + kb(c – a) + kc(a – b)
a b c
38. (c) Voltage gain = β × impedance Young’s modulus.
= 100 = 1
gain 43. (c) 0.6 =
200
49. (a)  x + y, 2y and y +z are in
⇒ 50 = β × harmonic progression.
100 This givesn = 10
⇒ β = 25
Also, power gain = β2 × impedance gain 44. (b) It is a case of normal 2y =
adjustment, Hence.
⇒ 2y (x + 2y + z) = 2(xy + xz + y2 + yz)
2 200
= (25) × β ⇒2xy + 4y2 + 2yz= 2xy + 2xz + 2y2 + 2yz
100 M = Also m =
= 1250 α ⇒ 2y2= 2xz
39. (c) For efficient radiation and ⇒ y2 = xz
reception, the height of transmitting and
β Hence x, y and z are in geometric
Therefore, = progression.
receiving antennas should be comparable α
Practice Set-17 217

Let P (k) = 23k – 7k – 1 = 49I


50. (d) Given = C(n, 0) + ∴ P (k + 1) = 23k + 3 – 7k – 8
= 8(49I + 7k + 1) – 7k – 8
C (n, 1) + C (n, 2) +...+ C(n, n)
 1 = 49(8I) + 49k = 49I1
= nC0 + nC1 + nC2 +....+ nCn    = – loge 1 −  = – loge
 3 Hence, by mathematical induction
= (1 + 1)n = 2n 23n – 7n – 1 is divisible by 49.
51. (a) Given nCr – 1, nCr nCr + 1 are in 59. (d) T h e g i v e n d i ff e r e n t i a l
  
= loge = loge 3 – loge 2
A.P. equation can be rewritten as
∴ 2. nCr = nCr – 1 + nCr + 1 56. (b) The equation of the circle of
radius 6 and centre at (3, 5) is: =y+
n! n!
⇒ 2. = + (x – 3)2 + (y – 5)2 = (6)2
r !(n − r )! (r –1)!(n − r + 1)!
Let S ≡ (x – 3)2 + (y – 5)2 – 36 = 0 Here, the power of highest order
n! At point (–2, –1)
(r + 1)!(n − r − 1)! S ≡ (–2 –3)2 + (–1 – 5)2 – 36 = 25 + 36 – derivative is 4.
2 1 1 36 = 25 > 0
⇒ = + Which represents outside the circle,. So, the degree of the equation is 4.
r (n − r ) (n − r + 1)(n − r ) (r + 1)r point (0, 1),
60. (a) n1 = 9, x1 = 100 and n2 = 6,
2      S = (0 – 3)2 + (1 – 5)2 – 36
⇒ = x2 = 80
r (n − r )       = 9 + 16 – 36 = – 11 < 0
Which represents inside the circle
⇒ n2 – n (4r + 1) + 4r2 – 2 = 0 n1 x1 + n2 x2 9 × 100 + 6 × 80
At point (–1, –2), ∴ x = =
⇒ So, x is a root of the equation n1 + n2 9+6
S = (– 1 – 3)2 + (–2 – 5)2 – 36
x2 – x (4r + 1) + 4r2 – 2 = 0
= 16 + 49 – 36 = 29 > 0 = = 92
52. (c) A = πr 2 , where r is the Which represents outside the circle.
distance between (1, 2) and (4, 6) At point (2, –1)
61. (b) y =
⇒ r = S = (2 – 3)2 + (–1 – 5)2 – 36
= 1 + 36 – 36 = 1 > 0 ∴ y= ⇒ y2 = cos x + y
= = =5 Which represents outside the circle.
∴ A = πr = π.25 = 25π sq. units.
2 Hence, point (0, 1) lies inside the On differentiating w.r.t. x, we get
circle S.
2y = – sin x +
53. (d) Let I =
57. (c) log6 16 = log6 2 = 4log6 2 +
4

Put t = x ⇒ sin x = (1 – 2y)


4 4
1 = = ....(i) sin x
⇒ dt =
dx log 2 2 + log 2 3 1 + log 2 3 =
2 x 1− 2y
I= = – 2 cos t + C and a = log12 27 = log12 33 = 3 log12 3 62. (c) From mean value theorem
3 3 3 f '(c) =
= –2 cos x + C = = =
log3 12 log3 3 + log3 4 1 + 2log3 2
54. (b) Since A (2, 3), B (1, 4), C Given, a=0
(0, –2) and D (x, y) are the vertices ⇒ a + 2a log3 2 = 3 ⇒ log3 2 = 1
of a parallelogram. We know that the ⇒ f (a) = 0 and b= ⇒ f (b) =
2
mid-points of diagonals of a parallelogram ∴ log2 3 = Now, f '(x) = (x – 1) (x – 2) + x (x – 2) +
are same. x (x – 1)
From eq. (i)
∴ f '(c) = (c – 1) (c – 2) + c (c –
∴ = and = 4 4(3 − a ) 2) + c (c – 1)
log6 16 = 2a = 3 + a
1+ = c2 – 3c + 2 + c2 – 2c + c2 – c
⇒ x = 1 and y = – 3 3− a ⇒ f '(c) = 3c2 – 6c + 2
58. (c) Let P (n)= 23n – 7n – 1 By definition of mean value theorem
55. (a) +..........∞ ∴ P (1) = 0, P (2) = 49
P(1) and P(2) are divisible by 49. f '(c) =
218 Practice Set-17

Or tan α tan β = 1 ....(i)


 3
  − 0 Area = = = Again β+γ=α
8
⇒ 3c2 – 6c + 2= = ⇒ γ=α–β
 1
  − 0
2 = sq. unit ∴ tan γ = =
⇒ 3c2 – 6c + =0 66. (b) k sinA cos A = k sin B cos B  [From Eq. (i)]
Since, this is a quadratic equation  [By sine rule] ∴ tan α = tan β + 2 tan γ
in c, ⇒ sin 2A = sin 2B
70. (d) 
∴ c = 6 ± 36 − 15 2A = 2B or π – 2B
2×3
∴ A= B i.e., ∆ABC is isosceles
21 =
= 6 ± 21 = 1 +
6 6 A+B= ∴ 2 tan–1 a – 2 tan–1 b = 2 tan–1 x
⇒ tan–1 a – tan–1 b= tan–1 x
Since, c lies between So that, C=
⇒ tan–1 = tan–1 x
∴ c=1– 21 [neglecting c i.e., ∆ABC is a right angled
6
67. (b) 3 sin–1 – 4 cos–1 ⇒ x=
=1 + ]

63. (b) The letters of given word are + 2 tan–1 = REASONING AND GENERAL
A, A, E, I, I, M, N, N, O, T, X i.e., word AWARENESS (RAGA)
starting with A are formed with the letters
2I’s. 2N’s, A, E, X, M, T, O (total 10 ⇒ 3.2 tan–1 x – 4. 2 tan–1 x + 2.2 tan–1 x = 71. (a) Black soil is most suitable for
letters). the cultivation of cotton. The deep and
Hence, number of words formed by (16 – 8 + 4) tan–1 x = ⇒ tan–1 x = medium black lava soil of the Deccan
these letters. and Malwa plateaus is considered ideal,
10! though it can be grown on alluvial and
= = ⇒x= red soil as well. The black cotton soil is
2!2!
also known as regur.
= 10 × 9 × 8 × 7 × 6 × 5 × 4 × 3 × 2 × 1 68. (b) I f t h e e q u a t i o n o f t h e
4 hyperbola be 72. (c) Price Elasticity of Demand
is a measure of the relationship between
= 10 × 9 × 8 × 7 × 6 × 5 × 3 × 2 × 1 = 1 ....(i) a change in the quantity demanded of
= 907200 ways a particular good and a change in its
64. (a) Required probability = The length of the latus rectum = 8 price. It measures the responsiveness
of demand to changes in price for a
⇒ =8 ⇒ b2 = 4a particular good. If the price elasticity
65. (a) The region is plotted in the of demand is equal to 0, demand is
⇒ a2 (e2 – 1) = 4a ⇒ a(e2 – 1) = 4
following fugure. The required region is perfectly inelastic (i.e., demand does not
ADCBA. ⇒ =4 ⇒a=5 change when price changes).
The area is given by: 73. (a) According to Article 61 of
On putting a = 5 in b2 = 4a, we get Indian Constitution, when a President
b = 20
2 is to be impeached for violation of
Hence, the equation of the required the Constitution, the charge shall be
hyperbola is: preferred by either House of Parliament.
It adds that no such charge shall be
=1 preferred unless:
• the proposal to prefer such charge
69. (c) α+β= .α= –β⇒ is contained in a resolution which has
been moved after at least fourteen days’
tan α = cot β notice in writing signed by not less
Practice Set-17 219

than one-fourth of the total number of 13x = 2600


x2 + 4 68
members of the House has been given = 2600
of their intention to move the resolution, x 15 x= = 200
15x2 + 60 = 68x 13
and
• such resolution has been passed 15x2 – 68x + 60 = 0 83. (a) After 30% discount on an
by a majority of not less than two-thirds 6 10 item the price is 70.
15 x 2 –  +  x + 60 =0 Applying 5% cash back, the price is
of the total membership of the House. 5 3 
74. (c) Soldering is the process 70 × 5
2 6 10 = = 3.5
of joining two metals by the use of a 15 x – x – x + 60 =0 100
5 3
solder alloy, and it is one of the oldest ∴ The effective discount = 30 + 3.5
 6  10 
known joining techniques. It is possible  x –  x –  =0 = 33.5%
because of the property of cohesion, the  5  3
84. (b) Let numbers be x and y.
interaction between adjacent parts of the 6 10 Mean = 16
x – = 0 and x – =0
same body and as acting throughout the 5 3 xy = 16

interior of substance. Soldering leads to 6 10
alloy formation at the layer between two x = = 0, x = ∴ Squaring both sides
5 3 xy = 256
metals.
Finally the two fractions are 256
75. (a) Fortran is a general-purpose, x =
imperative programming language 6 10 y
and respectively.
that is especially suited to numeric 5 3
x : y : : y : 1024
computation and scientific computing. 79. (a) On dividing 427398 by 15,
Its name is a contraction of FORmula x y
we get remainder = 3 =
TRANslation. It aims to provide a y 1024
∴ Required number to be
way to tell computers to calculate subtracted = 3 From (i) and (ii)
complicated mathematical expressions 80. (a) Suppose Rice x will cost 256 / y y
with more ease than assembly language. 30 paise more than wheat after a years. =
y 1024
76. (d) The Battle of the Hydaspes Then,
was fought by Alexander the Great (4.20 + 0.30a) – (6.30 + 0.15a) 256 y
2 =
in 326 BC against King Porus of the = 0.40 y 1024
Paurava kingdom on the banks of the ⇔ 0.15a = 0.40 + 2.10 y3 = 1024 × 256
river Hydaspes (Jhelum) in the Punjab 2.50 250
near Bhera. The battle resulted in a ⇔ a= = = 10 y = 3 16 × 16 × 16 × 4 × 4 × 4
0.25 25
complete Macedonian victory and the y =16 × 4 = 64
\ x will cost 40 paise more than y
annexation of the Punjab.
10 years after 2001 i.e., in 2011.  xy = 256
77. (c) On June 21, 2019, the 35th
81. (c) Total revenues of 9 years x × 64 = 256
Goods and Service Tax (GST) Council = 9 × 68 = 612 lakhs
Meeting was held in New Delhi. Total revenues of first 5 years 256
x= =4
78. (b) Let the two fractions are = 5 × 63 = 315 lakhs 64
x and y respectively. According to Total revenues of last 5 years Hence, the numbers are 4 and 64.
question, = 5 × 75 = 375 lakhs 85. (b) Let slow speed car takes
x × y = 4 ...(i) The revenues of 5th year time = t hours
68 = 315 + 375 – 612 and fast speed car takes = (t – 3)
and x+y= ...(ii) = 690 – 612 = 78 lakhs hours
15 Hence, the volume of 5th year Distance will be same in both cases
From eqn (i) x × y = 4 is ` 78 lakhs. 48(t – 3) = 30t
4 82. (a) Let the number be x 48t – 48 × 3 = 30t
y= According to question (48 – 30)t = 48 × 3
x
Putting value of y in eqn (ii) 113 48 × 3
x + 26 = x × t= = 8 hours
100 18
4 68
x+ = 100x + 2600 = 113x Distance = 30 × 8 = 240 km.
x 15 113x – 100x = 2600
220 Practice Set-17

86. (d) Son of Y's father  X. PQ = 15 km ⇒ 10 + ? + 10 = 30


Sister of X  Y. QR = TS = 20 km ⇒ ? + 20 = 30
[Y is not the brother of X] RS = 25 km ∴ ? = (30 – 20) = 10
Therefore, ‘X’ is the brother of ‘Y’. ∴ Required distance, 94. (d) According to dictionary, the
PT = QT – QP = RS – QP arrangement of the given words is:
87. (b) F  6 Position number in
the English alphabet. (iv) Foresail, (i) Foresay,
= 25 – 15 = 10 km
As, (iii) Foreseeable, (ii) Foreseize.
89. (d) The data is inadequate
M A T
because it is not given that who between 95. (c) The code for ‘how’ is ‘2’.
↓ ↓ ↓
Virat and Shikhar is taller. Hence, 96. (c) By analysing the above
13 + 1 + 20 = 34 answer cannot be determined.
Similarly, table, it is clear that D is good in Physics,
90. (c) History and Dramatics.
C A R
A B B C C D D E E F F G
↓ ↓ ↓ 1 2 2 3 3 4 4 5 5 6 6 7 97. (c) By analysing the above
3 + 1 + 18 = 22 table, it is clear that only C is good in
+1 +1 +1 +1 +1
88. (d) According to the question, +1 +1 +1 +1 +1
Physics, History and Mathematics but
the direction diagram will be as follows not in computer Science.
Finally the missing term is FG.
91. (b) By analysing the above 98. (d) By analysing the above table,
circular arrangement, it is clear that it is clear that E is good in Computer
Science, History and Dramatics.
F  third to the right of E.
99. (a) By analysing the above
92. (d) According to the given
table, it is clear that A is good in
information
History, Physics, Computer Science and
ac % bd  ac < bd Mathematics.
and ab∆cd  ab > cd
100. (b) By analysing the above
From (i) @ (2), we get
table, it is clear that B is good in Physics,
 no definite conclusion Dramatics and Computer Science.
93. (d) 5 + 15 + 10 = 30

4 + 17 + 9 = 30 qqq
Practice Set-18 221

Air Force Airmen Group X & Y Exam


Practice Set-18

(a) Thirteen (b) Twelve 9. One who looks at the bright side
ENGLISH
(c) One (d) None of things
Directions (1-4): Read the following 3. The king did not invite the (a) pessimist (b) optimist
passage and answer the questions thirteenth wise woman because–
that follow by selecting the most (c) introvert (d) extrovert
appropriate option– (a) he did not know about her Directions (10): In the following
A long time ago there lived a king and (b) he had no more golden plates left question, out of the four alternatives,
queen who longed to have a child. One (c) she could not grand fairy gifts choose the one which best expresses the
day, when the queen was resting near a (d) she could grant fairy gifts meaning of the given word.
spring, a frog crept out of the water and 4. The thirteenth wise woman was in 10. PAUCITY
said to her, "You shall have your wish. rage because– (a) sincerity (b) shortfall
Within a year you shall have a little girl." (a) the other wise women teased her (c) publicity (d) downfall
What the frog said came true. The (b) the king had not invited her 11. Fill in the blank with the correct
queen had a child who was so beautiful (c) she did not like the other wise preposition.
that the king gave a party in her honour. women He killed two birds ......one stone.
He wished to invite all the wise women (d) she did not like the princess (a) by (b) at
in the land, for these wise women could
Directions (5-6): In the following (c) of (d) with
grant fairy gifts to his little child. There
questions, groups of four words are Directions (12-13): In the following
were thirteen of them, but only twelve
given. In each group one word is questions, a sentence has been given in
were invited, as the king had only twelve
correctly spelt. Find the correctly Direct/Indirect Speech. Out of the four
golden plates.
spelt word. alternatives suggested select the one
After the dinner was over, the
wise women in turn rose from the table 5. (a) misdemenour which best expresses the same sentence
and named their fairy gifts to the little (b) manoeuvre in Indirect/Direct Speech.
princess. The first gave to her goodness, (c) begining 12. "What a stupid fellow you are !"
the second, beauty; the third, riches and (d) chameloon She remarked
so on, up to the last. Before the twelfth 6. (a) irrepairable (a) She exclaimed that what a stupid
wise woman could speak, suddenly the (b) irreparable fellow he was
thirteenth came in. This woman was (c) irreparrable (b) She exclaimed that he was a very
in great rage because she had not been (d) irepairable stupid fellow
invited and without greeting or even Directions (7): In following question, (c) She exclaimed in disgust how he
looking at anyone. She cried with a loud choose the word opposite in meaning could be so stupid
voice, "The king's daughter shoul in her to the given word. (d) She asked him what a stupid
fifteenth year prick herself with a spindle 7. VAGUE fellow he was
and fall down dead." And, without saying
(a) accurate (b) sharp 13. The Police said to the thief, "Don't
a word more, she turned round and left
(c) precise (d) actual move."
the room.
1. The king and the queen wished Directions (8-9): In the following (a) The police ordered the thief not to
for– questions, out of four alternatives, move
(a) a golden plate choose one which can be substituted (b) The police ordered the thief that
for the given words/sentence. he should not move
(b) a son
8. One who makes an official (c) The police told the thief that he
(c) a daughter
examination of accounts. did not move
(d) a child
2. How many wise women were there (a) auditor (b) accountant (d) The police ordered to the thief to
in the land? (c) clerk (d) official not move
222 Practice Set-18

Directions (14-15): In the following 18. The students could not understand 25. The mutual inductance of a pair
questions, a sentence has been given (a)/why the teacher was (b)/angry of coils is 2 H. If the current in one
in Active Voice/Passive Voice. Out of upon them. (c)/No error (d). of the coils changes from 10 A to
the four alternatives suggested, select 19. This book (a)/is quite different than zero in 0.1 s, the e.m.f. induced in
the one which best expresses the same (b)/the previous one. (c)/ No error the other coil is:
sentence in Passive/Active Voice. (d). (a) 2 V (b) 20 V
14. Their dog bit our neighbour
yesterday. 20. That day when they brought her back (c) 0.2 V (d) 200 V
for the last time, (a)/there was many 26. A force = acts at
(a) Our neighbour bit their dog
old-timers (b)/who were shocked
yesterday. a point (7, 3, 1). The torque about
and fearful. (c)/No error (d).
(b) Our neighbour bitten by their dog the origin (0, 0, 0) will be:
yesterday. PHYSICS
(c) Our neighbour was bitten by their (a)
dog yesterday. 21. A proton and an electron have
same de-Broglie wavelength. (b)
(d) Our neighbour were bit by their
dog yesterday. Which of them moves faster? (c)
15. The MD told us about the new (a) Proton
(d)
project. (b) Electron
(c) Both move with same speed 27. The radius vector of a point is
(a) We were told about the new
project by the MD. (d) Data insufficient to decide = and force =
22. A p l a n e e l e c t r o m a g n e t i c
(b) We are told about the new project acts at that point. The
by the MD. wave propagating in the moment of the force in Nm is:
(c) We told by MD about the new x-direction has wavelength of
6.0 mm. The electric field is in (a)
project.
(d) We had been told by MD about the y-direction and its maximum (b)
project. magnitude is 33 Vm–1. The equation
for the electric field as function of (c)
Directions (16-17): In the following
questions, four alternatives are given x and t is: (d)
for the Idiom/Phrase printed in (a) 11 sin π (t – x/c)
28. Two thin long, parallel wires
underlined in the sentence. Choose (b) 33 sin π × 1011 (t – x/c)
(c) 33 sin π (t – x/c) separated by a distance ‘d’ carry
the alternative which best expresses
(d) 11 sin π × 1011 (t – x/c) a current of ‘i’ A in the same
the meaning of the Idiom/Phrase.
23. A charged particle of mass m and direction. They will:
16. Once his crime was discovered, he
had no option but to come clean. charge q is released from rest in (a) attract each other with a force
(a) to deny the crime a uniform electric field E. The μ i2
of 0
(b) to confess to the crime kinetic energy of the particle after (2πd )
(c) to accuse someone else of the time t is: (b) repel each other with a force of
crime (a) (b) μ 0i 2
(d) to apologize for the crime
(2πd )
17. The employees were kept in the
dark about the latest developments. (c) (d) (c) attract each other with a force
(a) were informed after office hours μ 0i 2
of
(b) were informed when it was too 24. The diameter of the molecule (2πd 2 )
late o f o x y g e n a n d n i t ro g e n i s (d) repel each other with a force of
(c) were given secret information approximately 2 × 10–10 m. At
μ 0i 2
(d) were not informed at all N.T.P., 1 m3 of air has nearly 2.7
× 10 25 molecules. The average (2πd 2 )
Direction (18-20): In the following
questions, some of the sentences have distance between the molecules 29. Ice starts forming in a lake
error and some have none. Find out is nearly: with water at 0ºC when the
which part of a sentence has an error. (a) equal to diameter atmospheric temperature is
The number of that part is your (b) 25 times of diameter –10ºC. If the time taken for
answer. If there is No error, the answer (c) 34 times of diameter 1 cm thick layer of ice to be
is (d). (d) 17 times of diameter formed is 7 hours, the time taken
Practice Set-18 223

for the thickness of the ice to 33. The velocity of wave is 360 m/s 39. The two ends of a perfectly
change from 1 cm to 2 cm is: and frequency is 500 Hz. The two lagged metal bar are maintained
(a) 7 hours particles which are at a phase at temperatures T1 and T2 (T1
(b) less than 7 hours difference of 60º will have path > T 2). Which of the following
(c) more than 7 hours but less than difference: graph shows the variation of
14 hours (a) 0.72 m (b) 12 cm temperature with distance from
(d) more than 14 hours (c) 120 m (d) 0.72 cm the hot end?
30. The plates of a parallel plate 34. A machine of 75% efficiency takes (a)
capacitor are separated by d cm. 12 J energy to raise 1 kg mass
A plate of thickness t cm with to a certain height. The mass is
dielectric constant k1 is inserted allowed to fall through the same
and the remaining space is filled height. Its speed at the end of the
with a plate of dielectric constant fall will be:
k 2 . If Q is the charge on the (a) 9.8 m/s (b) 10 m/s
capacitor area of plates is A cm2 (c) m/s (d) m/s
each, then potential difference 35. The dimensional representation
between the plates is: of specific resistance in terms of (b)
charge Q is:
Q  t d –t 
(a)  +  (a) [ML3T–1Q–2]
ε 0 A  k1 k2  (b) [ML2T–2Q–2]
  (c) [MLT–2Q–1]
(b) 4π Q l + d − 1 (d) [ML2T–2Q–1]
A  k1 k2  36. Assuming that the junction diode
4π Q  k1 k2  is ideal, the current through the
(c)  +  diode is:
A t d – t (c)
3V 1V
Q  k1 d – t 
(d)  +  (a) 0 (b) 2 mA
ε0 A  t k2  (c) 20 mA (d) 200 mA
31. In an oscillating L-C circuit the 37. At a specific instant emission of
maximum charge on the capacitor radioactive compound is deflected
is Q. The Charge on the capacitor in a magnetic field. The compound
when the energy is stored equally can emit:
between the electric and magnetic (i) electrons (ii) protons
fields is: (iii) He2+ (iv) neutrons (d)
(a) Q/2 (b) The emission at the instant can be:
(c) (d) Q (a) (i), (ii), (iii)
(b) (i), (ii), (iii), (iv)
32. Five resistance are connected as (c) (iv)
shown in the figure. The effective (d) (ii) and (iii)
resistances between the points A 38. Figure shows four batteries of
and B is: emf E and internal resistance r
connected in series. The voltage
across each battery is: 40. In the circuit shown in figure, the
current through:

(a) (b)
(a) 2 V (b) Zero
(c) (d) (c) 1.5 V (d) 0.75 V
224 Practice Set-18

(a) the 3Ω resistor is 0.50 A MATHEMATICS (a) k = 1, 2 (b) k = 2,


(b) the 3Ω resistor is 0.25 A 46. The set {x/(x – 3) (x – 4) > 0} is
(c) the 4 Ω resistor is 0.50 A equal to: (c) k = 3, (d) None of
(d) the 4 Ω resistor is 0.25 A (a) { x | 3 < x < 4} these
41. The manifestation of band (b) { x | x < 3} ∪ { x | x > 4}
structure in solids is due to: (c) { x | x < 3} ∪ { x | x > 4} 53. Inverse of matrix is:
(a) Heisenberg’s uncertainty (d) None of these
principle 47. If x = and y = , (a)
(b) Pauli’s exclusion principle
(c) B o h r ’s c o r r e s p o n d e n c e then:
principle (a) x and y are conjugate to each
other
(d) Boltzmann’s law
(b) | x | = | y |
42. The musical interval between two (c) Real part of x = imaginary part (b)
tones of frequencies 320 and 240 of y
is: (d) None of these
(a) 80 (b) 1.33 48. L e t A = N × N a n d R b e
(c) 1.78 (d) 7 a re l a t i o n o n A s u c h t h a t (c)
43. A bead is oscillating between (a, b) R (c, d), iff a + d = b + c.
Which of the following is not
B and C in a smooth bowl. true?
The vertical height of B or C (a) R is not reflexive
above the lowest point A is (b) R is symmetric
10 cm. If g = 9.8 m/s2. The velocity (c) R is transitive (d)
of bead at A will be: (d) R is equivalence
49. If sum of n, 2n and 3n terms of
any arithmetic progression are 54. Let A is the set of lines in a
S1, S2, S3, then: plane. A relation R is defined in
(a) S3 = 3(S2 – S1) A as l1R t2, iff l1 is perpendicular
to l2, then which of the following
(b) S3 = 2(S2 – S1)
(a) 1 m/s (b) 1.4 m/s is true?
(c) 1 cm/s (d) 1.04cm/s (c) S3 = (S2 – S1)
(a) R is transitive
44. T h e l e n g t h , b r e a d t h a n d (d) S3 = (S2 – S1) (b) R is reflexive
thikckness of a block are measured (c) R is symmetric
as 125.5 cm, 5.6 cm and 0.32 cm 50. S u m o f g e o m e t r i c s e r i e s (d) R is equivalence
respectively. Which measurement 55. The coefficient of x3 in expansion
upto ∞
is least accurate? of (1 + x)3 (1 – x)6 will be:
(a) of thickness terms will be: (a) – 4 (b) – 6
(b) of length (a) (c) – 8 (d) – 2
(c) of breadth (b) 56. The value of cos2 15° + cos2 45° +
(d) All have same accuracy cos2 75° is:
45. A spherical surface of radius (c) 3 + 3 2
(a) (b)
of curvature R separates air (d) 1
(refractive index 1.0) from glass 51. Seven girls have to dance in
(refractive index 1.5). The centre a circle. In how many ways (c) (d)
they are to be stand on the
of curvature is in the glass. A point circumference of circle?
P placed in air is found to have (a) 180 (b) 360 57. If x = p sec θ and y = q tan θ,
a real image Q in the glass. The (c) 720 (d) 72 then:
line PQ cuts the surface at point (a) x2 – y2 = p2q2
O and PO = OQ. The distance PO 52. If the equation x2 – 2(k + 1) x + (b) x2q2 – y2p2 = pq
is equal to? (c) x2q2 – y2p2 = p2q2
k = 0 has two identical roots,
(a) 5 R (b) 3 R
(d) x2q2 – y2q2 =
(c) 2 R (d) 1.5 R then the value of k are:
Practice Set-18 225

58. A circle touches the y-axis at the (a) 0, 1, 1 (b) 1, 1, 0


point (0, 4) and cuts the x-axis REASONING AND GENERAL
(c) 1, 0, –1 (d) None of
in a chord of length 6 units. The these
AWARENESS (RAGA)
radius of the circle is: 66. A speaks the truth in 75% cases 71. Who invented the mobile phone?
(a) 3 (b) 5 and B in 80% cases. In what (a) Tim-Berners-Lee
(c) 4 (d) 6 percent of cases are they likely to (b) Raymond Samuel Tomlinson
contradict each other having the
(c) Chuck Hull
59. The value of (r 2 + r 3) same incident?
(d) Martin Cooper
will be: (a) 30 (b) 35
72. Who was the creator of the
(a) a (b) b (c) 40 (d) 45
famous Rock Garden of
(c) c (d) Chandigarh?
67. is equal to:
(a) Khushwant Singh
60. If (–3, 0) is the focus and x +
5 = 0 is the directrix, then the (b) Charles Corbusier
equation of the parabola is: (a) loge (c) Edward Baker
(a) y2 = 4 (x + 4) (d) Nek Chand
(b) x2 = 4 (y + 4) 73. Expand NABARD.
(c) x2 = 4 (x – 4) (b) + loge
(d) y2 = 4 (x – 4) (a) National Bank for Agriculture
61. If the straight line lx + my + and Rural Development
(c) log10
n = 0 touches the hyperbola (b) National Bank for Agri
(d) None of these Related Development
, then:
(c) National Bank for Agriculture
68. The value of is: and Resource Development
(a) a2l2 + b2m2 = n2
(b) a2l2 – b2n2 = m2 (d) National Bank for Asian
(a) – loge 2 (b) 0 Reaserch Development
(c) a2l2 – b2m2 = n2
(d) a2m2 – b2x2 = l2 (c) loge 2 (d) 74. An ordinance issued by the
62. The degree of the differential Governor has to be passed by the
equation 69. The sum of the focal distance Assembly within
of any pont on the ellipse (a) 8 weeks (b) 10 weeks
= is:
x 2 y2 (c) 12 weeks (d) 6 weeks
+ = 1 will be:
(a) 1 (b) 2 16 9 75. Who was the founder of the
(a) 9 (b) 4 Indian National Army?
(c) 3 (d) 6
63. The points (4, 3) and (–2, –1) (a) Nehru
(c) 8 (d) 6
are the opposite vertices of a (b) Subhash Chandra Bose
70. What is the median of 10, 13, 22,
parallelogram. If its third vertex (c) Bal Gangadhar Tilak
is (1, 0), then the coordinates of 25, 8, 11, 19, 18, 31 and 16?
(d) Gandhiji
the 4th vertex are: (a) 16.5 (b) 17
76. Seismography is the science of
(a) (1, 2) (b) (2, 2) (c) 16 (d) 19.5
(c) (1, 1) (d) (2, 1) (a) Rivers
63. If the different permutations
(b) Earthquakes
64. If cos–1 + cos–1
= θ, then of all the letters of the word
(c) Volcanoes
EXAMINATION are listed as in
(d) Mountains
a dictionary, how many words are
77. Who chaired the 35th Goods
is equal to: there in this list before the first word
and Service Tax (GST) Council
(a) cos2 θ (b) tan2 θ starting with E? Meeting in New Delhi?
(c) sin2 θ (d) sec2 θ (a) 908300 ways (a) Nirmala Sitharaman
(b) 907200 ways (b) Anurag Singh Thakur
65. Suppose – limx→0
(c) 907300 ways (c) Girish Chandra Murmu
= 2. Then values of a, b, c are: (d) None of these (d) Vipul Bansal
226 Practice Set-18

78. When 0.363636.....is converted (a) 648 km (b) 810 km neighbour of either B or H and is
into a fraction, then the result is (c) 432 km (d) 540 km second to the right of F. C is fourth to
_______ . 86. Pointing towards a person, a the right of G.
(a) 7/11 (b) 4/11 man said to a woman, "His 91. In which of the following pairs
(c) 14/33 (d) 8/33 mother is the only daughter of is the second person sitting to
79. If tan 5√/3 = x, then the value of your father." How is the woman the immediate left of the first
x is related to that person? person?
(a) –√3 (b) 1 (a) Daughter (b) Sister (a) BC
(c) 1/2 (d) √3/2 (c) Mother (d) Wife (b) HE
80. Raju has 31.25 metres of the 87. In a certain code language, (c) FA
cloth and he has to make 16 “FIRMLY” is written as (d) None of the above
pieces out of a metre of cloth. “TKHWJK”. How is “PUBLIC”
Direction (92): In following questions
How many pieces can he make written in that code language?
a stands for equal to’; b for ‘greater
out of this cloth? (a) DOXRAG (b) DWRAGJ
than’; γ for ‘less than’ and d for ‘not
(a) 300 (b) 400 (c) DWRAGK (d) DWRAHJ
equal to.
(c) 520 (d) 500 88. Molly travelled from point A to
92. If 6x a 5y and 2y b 3z, then
81. In a class of 65 students there are point B which is 5 ft. He, then
travelled 6 ft to his right and (a) 2x b 3z (b) 4x b 3z
39 girls. The average weight of
these girls is 60 kg and average then turned to left and went 4 ft. (c) 2x γ z (d) 4x a 3z
weight of the full class is 64 kgs. Finally, he again went 6 ft to his Direction (93): Select the odd word/
What is the average weight of the left. How far is he from the point letters/number/number pair from the
boys of the class? B now? given alternatives.
(a) 69 (b) 66 (a) 10 ft (b) 6 ft 93. (a) BEH (b) DGJ
(c) 68 (d) 70 (c) 5 ft (d) 4 ft (c) HKN (d) MNO
82. An engineering student has to 89. P, Q, R and S are four males. P Direction (94): Arrange the given
secure 25% marks to pass. He is the eldest in the group but he words in the sequence in which they
gets 56 and fails by 34 marks. is not the poorest, R is the riches occur in the dictionary.
Find the maximum marks. but not the eldest, Q is elder 94. (i) Shortage (ii) Shore
(a) 385 marks (b) 360 marks than S but he is not elder than P (iii) Shamelessly (iv) Shadiness
(c) 410 marks (d) 435 marks or R, P is riches than Q but he (v) Shallow
83. A shopkeeper by selling 8 is not richer than S. How the (a) iii, iv, v ii, i (b) iv, iii, ii, v, i
iphones, earns a profit equal to four persons can be arranged in (c) iv, v, iii, ii, i (d) iii, iv, ii, i, v
the selling price of 3 iphones. His decreasing order of their age and
profit percentage is : Direction (95): Study the following
money? information to answer the given
(a) 60 percent (a) PQRS, RPSQ
(b) 37.5 percent questions:
(b) PRQS, RSPQ In a certain code ‘829’ means ‘how
(c) 75 percent (c) PRQS, RSQP
(d) 15 percent art thou’ ‘958’ means ‘thou art good’ and
(d) PRSQ, RSPQ ‘15873’ means ‘thy good and thou bad’.
84. If Giridhar's salary is 5/3 times Direction (90): A series is given with
of Hariraj's and Shaunak's is 95. Which of the following may
one term missing. Select the correct
2/3 times of Hariraj's, what is alternative from the given ones that
possibly be the code for ‘thou no
the ratio of Giridhar's salary to will complete the series.
good’?
Shaunak's? 90. BDG, KMP, TVY, ? (a) 508 (b) 780
(a) 9 : 10 (b) 10 : 9 (a) CDH (b) CEH (c) 507 (d) 780
(c) 5 : 2 (d) 2 : 5 (c) HEC (d) C E I Directions (96–100): Study the
85. Two cars travel from city A to Direction (91): Study the Following following information carefully and
city B at a speed of 30 and 36 km/ information carefully and answer the answer the questions given below:
hr respectively. If one car takes 3 questions given below:
hours lesser time than the other P, Q, R, S, T, V and W are travelling
A, B, C, D, E, F, G and H are sitting in three different vehicles, There are
car for the journey, then the around a circle facing the centre. B is
distance between City A and City at least two passengers in each vehicle
second to the right of H and third to I, II & III and only one of them is a
B is : the left of A. D is not an immediate
Practice Set-18 227

male. There are two engineers, two 96. What is V's profession? 99. Which of the following is not
doctors and three teachers among (a) Engineer correct?.
them. (b) Teacher (a) T - Male - Teacher
(i) R is a lady doctor and she does (c) Doctor (b) Q - Male - Engineer
not travel with the pair of sisters, (d) Data inadequate
(c) P - Female - Teacher
P and V. 97. In which vehicle does R travel ?
(ii) Q, a male engineer, travels with only (a) I (b) II (d) V - Female - Teacher
W, a teacher in vehicle I. (c) III (d) II or III 100. How many lady members are
(iii) S is a male doctor. 98. Which of the following represents there among them?
(iv) Two persons belonging to the same the three teachers ? (a) Three
profession do not travel in the (a) WTV (b) Four
same vehicle. (b) WTP (c) Three or Four
(v) P is not an engineer and travels in (c) WTV or WTP (d) Data inadequate
vehicle II. (d) Data inadequate

Answers with Explanation


14. (c) Our neighbour was bitten by
ENGLISH 2π × 3 × 108
their dog yesterday. =
1. (d) The king and the queen wished 6 × 10 –3
15. (a) We were told about the new
for a child. project by the MD. = π × 1011 rad/s
2. (a) Thirteen The equation for the electric field,
16. (b) Idiom come clean means; to
3. (b) he had no more golden plates admit and explain something that you along y-axis in the electromagnetic
left kept as a secret. wave is:
 x
4. (b) the king had not invited her 17. (d) Idiom in the dark means Ey = E 0 sin ω  t – 
knowing nothing about something; do  c 
5. (b) The correct spellings of other
words are: misdemeanour, beginning; not inform at all. = 33 sin π × 1011 (t – x/c)
chameleon. 18. (c) Here, the word angry will take 23. (c) Velocity of particle at time t is v
preposition 'with'.
6. (b) The correct spelling is: = at where a =
irreparable. 19. (b) The word Different takes
preposition 'from'.
7. (b) The word Vague (Adjective) ⇒ v=
means: not clear; indistinct; not giving 20. (b) Here, there were many old
enough information. timers (plural) – should be used.
The word Precise (Adjective) means: So, K.E. =
clear and accurate; exact; meticulous. PHYSICS
24. (d) Empty space (average) available
The word Sharp (Adjective) means: for each molecule
clear and definite; having a fine edge.
21. (b) = =
Hence, the words vague and sharp are =
antonymous.
= 0.037 × 10–24 m3
8. (a) 9. (b) ∴ Average distance between the
10. (b) The word Paucity (Noun) =
molecules
means: a small amount of something: = (0.037 × 10–24)1/3
less than enough of something; deficit; But, Epme < Eemp
= 3.4 × 10–9 m
shortfall. ∴ v1 < v2
11. (d) or, v2 > v1 =
12. (b) She exclaimed that he was a i.e., electron moves faster than proton
= 17 diameter
very stupid fellow. 2π c
13. (a) The police ordered the thief not
22. (b) ω = 2πv =
λ
25. (d) ξ = (in magnitude)
to move.
228 Practice Set-18

magnetic field) and capacitor (in the


⇒ ξ = electric field). 35. (a) Since, =
UEmax = max. energy stored in capacitor
⇒ = 200 V
= ⇒ [ ]=
26. (a) = ⇒ [ ] = [ML3T–1Q–2]
UBmax = max. energy stored in inductor

=
= 36. (c) i = =
⇒ =
Where, io is the current at this time. = 2 × 10–2 A

= For the given instant, = 20 mA
UE = UB 37. (a) Electrons, Protons and Helium
⇒ =
ions are deflected in magnetic field
i.e., = so, the compound can emit electrons,
proton and He2+.
⇒ = From energy conservation 38. (b) Current flowing through the
UE + UB = UEmax + UBmax circuit
(E – Ir) + (E – Ir) + (E – Ir) + (E – Ir) = 0
= ⇒ =
I=

=
27. (a) Moment of force ⇒ q=
Vab = E – Ir =
=
32. (a) This combination forms a
Similarly, Vbc = Vcd = Vda = 0
⇒ = balanced Wheatstone Bridge.
39. (b) When bar is lagged on the
So, RAB = = side, there is no loss of heat due to
radiation from the side and hence equal
amount of heat passes through every
33. (b) λ = = 72 cm cross-section of the bar and temperature
will fall linearly [Q is constant ∆θ/∆x is
= 2π
Phase difference = × path difference also constant].

= λ
40. (d) I =
28. (a) The force per unit length 60 = path difference
between the two wires is ⇒ = 1A
At A a current of 1A divides into 0.5 A
F µ 2i 2 μ 0i 2
= 0. = ∴ Path difference = = 12 cm and 0.5 A.
l 4π d 2π d At B the current of 0.5 A divides into 0.25
The force will be attractive as current A and 0.25 A.
34. (d) = eff.
direction in both are same. 41. (b) According to Pauli’s exclusion
29. (c) The thermal conductivity of ice principle, the electronic configuration of
is less than that of water, therefore, the ∴ Output = =9J number of subshells existing in a shell
rate of loosing energy decreases as ice is and number of electrons entering each
formed. It will therefore, take more time subshell is found.
for thickness to get doubled. ∴ h = Hence, on the basis of Pauli’s exclusion
 t d – t principle, the manifestation of band
30. (b) V = 4πσ  – structure in solids can be explained.
 k1 k2  = = 0.9 m
42. (b) Musical interval = Frequency
4π Q  t d – t  ∴ Velocity = ratio
⇒ V= +
A  k1 k2 
= =
31. (c) In an L-C circuit the energy
osillates between inductor (in the = m/s = 1.33
Practice Set-18 229

(ii) R is symmetric because


43. (b) mgh = (a, b) R (c, d) ⇒ (c, d) R (a, b) =
i.e., a + d =b+c
v=
or, ⇒ c+d=d+a
= =
(iii) R is transitive because
= 1.4 m/s if (a, b) R (c, d) and (c, d) R (e, f)
44. (a) Thickness = 0.32 cm
⇒(a, b) R (e, f)
∆b = 0.01 cm = =
i.e., a + d = b + c and c + f = d + e
% error = = 3% ⇒(a + d) + (c + f)= (b + c) + (d + e) So, sum of ∞ terms of geometric series
⇒ a+f=b+e =
Thus, R is reflexive, symmetric 51. (b)  Any girl may be stand on
45. (a) =
and transitive i.e., R is an equivalence any place so there is so difference in
relation. clockwise and anticlockwise direction
⇒ = 49. (a) Let first term of series is a and here.
common difference is d, then
⇒ x = 5R ∴ Required permutations =
S1= n [2a + (n – 1) d]....(i)
MATHEMATICS =
46. (d)  (x – 3) (x – 4) > 0, then S2= [2a + (2n – 1) d]....(ii)
= = 360
S3 = [2a + (3n – 1) d]....(iii)
52. (b) For identical roots D=0
From this we see that set { x | (x – 3)
(x – 4) > 0} is equal to {x | x < 3} ∪ {x ∴ 3(S2 – S1) = 3 ⇒ b 4 (k + 1)2 – =0
| x > 4}.
47. (d) x = ⇒ k2 + 2k + 1 – =0

= = (3n) [4a + 2(2n – 1)d – 2a – (n – 1)d]


⇒ k2 + +1=0
= (3x)[2a + (4n – 2 – n + 1)d]
=
⇒ k2 + +1=0
= (3n) [2a + (3n – 1)d] = S3
=
⇒ 2k2 – 5k + 2 = 0
50. (b) a = ,r= k =

and y = =

=
=
=
= 53. (a) If A = , then | A | =

∴ The value of x and y do not satisfy any 12 – (– 2) = 14 and A11 = 4, A12 = –1, A21
relation among the given alternatives. ∴ S∞ = = – (–2) = 2 and A22 = 3
48. (d) Let (a, b), (c, d) and (e, f) ∈ A
and given that (a, b) ∴ Adj. A=
R (c, d), iff a + d = b + c
(i) R is reflexive because  S∞ =
∴ A–1 =
(a, b) R (a, b) i.e., a + b = b + a
230 Practice Set-18

58. (b)  AB = 6, therefore AC = 3 and perpendicular PM from P on the


and EC = OD = 4 directrix x + 5 = 0 is x + 5
=
∴ =x+5
⇒ x + 6x + 9 + y = x2 + 10x + 25
2 2

54. (c) (i) Since, no straight line is ⇒ y2 = 4x + 16 = 4 (x + 4)


perpendicular to itself. Hence, R is not 61. (c) From the equation lx + my +
reflexive ny = 0
(ii) If a straight line a is ⊥ to −lx n
y = −
straight line b, then b is also ⊥ to a. m m
Hence, R is symmetric Putting the value of y in the equation of
(iii) If a is ⊥ to b and b is ⊥ c, then hyperbola, we get
a will not be ⊥ to c, (a will be parallel to 2
Radius = EA = x 2 1  lx n 
c). Hence, R is not transitive. − − −  = 1
Thus, R is only symmetric. a 2 b2  m m 
55. (b) On expansion, 59. (a) (r2 + r3) x2 l 2 x2 n2 2lnx
(1 + x)3 = 1 + 3x + 3x2 + x3 and ⇒ 2
− 2 2 − 2 2 − 2 2 =1
a m b m b m b
(1 – x)6 = 1 – 6C1x + 6C2x2 – 6C3x3 + 6C4x4
– 6C5x5 + x6
terms of x5 in product of all the two may ⇒
=
be obtained by multiplying those terms in
expansion of (1 – x)6 which contain x5, x4,
x3, x2 respectively to corresponding terms + =0
in the expansion of (1 + x)3.
∴ coefficient of x5 = If the line touches the hyperbola, then
= 1 + (– 6C5) + 3 × 6C4 + 3× (– 6C3) +  1 l2 
1× 6C2 = 4  2 − 2 2 
= . a m b 
= – 6 + 45 – 60 + 15 = – 6
56. (d) cos2 15° + cos2 45° + cos2 75° ⇒ a2l2 – b2m2 = n2
[ a + b + c = 2s] 62. (b) On squaring the given equation
=
= = , we get

= = =a
=
= 60. (a) Let a point P(x, y) on the
parabola.
The highest derivative in it is whose
= degree is 2.
63. (a) Let the coordinates of D be
57. (c) x = p sec θ ⇒ sec θ = (x, y)

and y = q tan θ ⇒ tan θ =


⇒ sec2 θ – tan2 θ =


⇒1=
∴ Distance of P from the focus (–3, 0)

⇒ x2q2 – p2y2 = p2q2 SP =
Practice Set-18 231

If E is the mid-point of the line AC, then

E = = (1, 1) = limx→0 =

and the coordinates of the mid-point of

BD = (a – b + c) + (a – c) x
= +
Since, the diagonals of a parallelogram  a b c
bisects one another. +  + +  x 2 + ....
= limx→0  2! 2! 2!
∴ The mid-point of AC will be the same
as the mid-point of BD. x4
x2 − + ...
3!
∴ =1 Since, the limit is given as 2, a finite 1 π/ 2 x
2 ∫0
= x sec 2 dx +
quantity, we must have 2
a–b+c =0
⇒ =1 π/ 2
a–c =0 1 x 2 π/ 2 x
⇒ x =1 =  2 x tan  − ∫ tan dx
(a + b + c) = 2 2 2 0 2 0 2
⇒ y =2
64. (c)  cos–1 + cos–1 =0 +
Solving these, we get
x y a = 1, b = 2, c = 1 π/ 2
x2 y2   x
=  x tan  =-
∴cos –1  . − 1− 2 1− 2  = 0 66. (b) Probability that A speaks
 a b a b   2 0
75
truth = = and Probability that π π π
100 = tan =
⇒ cos θ = – 2 2 2
80
B speaks truth = =
100 69. (c) In =1
⇒ = – cos θ ∴ Probability that they contradict each
a =4
other on the same incident
and b =3
3 4 4 3
⇒ = 1 −  + 1 −  Since, the sum of the focal distances
4 5 5 4 of any point on the ellipse = major axis

= 2 × 4 = 8
= + cos2 θ – = = = 70. (b) On writting the data in
ascending order
⇒ a2b2 + x2y2 – a2y2 – b2x2 ∴ In 35% cases they contradict on the 8, 10, 11, 13, 16, 18, 19, 22, 25, 31
= x2y2 + a2b2 cos2 θ – 2xy cos θ ab same incident.
Here, n = 10
⇒ a2b2 (1 – cos2 θ) 67. (d) Let I =
= b2x2 + a2y2 – 2xy cos θ ab ∴Median =
⇒ a2b2 sin2 θ = b2x2 + a2y2 – 2xy cos θ.ab Put x = a sec θ ⇒ dx = a sec θ tan θ dθ

∴ – = sin2 θ ∴ I=

x −x = ∫ sec θ d θ
65. (d) lim x→0 ae − b cos x + ce
x sin x I= loge (sec θ + tan θ)
= [size of 5th term + size
 x + x2 − a2 
= log e  
 a  of 6th term]
 
π / 2  x + sin x 
68. (d) ∫0  1 + cos x  dx = [16 + 18] = 17
 
232 Practice Set-18

76. (b) Seismography is the 82. (b) Let maximum marks be x


REASONING AND GENERAL scientific measuring and recording of
AWARENESS (RAGA) 25
the shock and vibrations of earthquakes. x = 56 + 3 4 = 90
100
71. (d) Martin Cooper, an American The study of these records is known
engineer, conceived the first handheld as seismology. The instrument for 90 × 100
x= = 360 marks
mobile phone while at Motorola in automatically detecting and recording 25
1973. He led the team that developed it the intensity, direction, and duration of 83. (a) Profit = (8 – 3) = 5
and brought it to market in 1983. He is a movement of the ground, especially of 3 × 100
considered the “father of the cell phone” an earthquake, is known as seismograph. ∴ Profit percentage =
5
and is also cited as the first person in 77. (a) 35th Goods and Service Tax
= 60%
history to make a handheld cellular (GST) Council Meeting was chaired by
phone call in public. the Union Finance & Corporate Affairs 84. (c) According to question
Minister Smt. Nirmala Sitharaman. Let Hariraj's salary be x
72. (d) The Rock Garden of
Girdhari’s salary
Chandigarh is a sculpture garden that 78. (b) x = 0·363636 .......
was created by Nek Chand, a government 5 5
x = 0·36 ...(i) = of Hariraj = x...(i)
official who started it secretly in his 3 3
spare time in 1957. It is also known as 100x = 36·36 ...(ii) Shaunak's salary
Nek Chand’s Rock Garden. Today it is Subtracting equation (i) from (ii) 2
2
spread over an area of 40 acres. It is 99x = 36 = Hariraj = x ...(ii)
3 3
completely built of industrial and home
36 4 Ratio of Girdhari's salary to
waste items. x = = .
73. (a) NABARD stands for 99 11 Shaunak's salary
National Bank for Agriculture and Rural π  2π  5 2
79. (a) x = tan 5 = tan  π +  = x: x = 5 : 2
Development. It was established on 12 3  3  3 3
July 1982 by a special Act of parliament
2π  π 85. (d) Let first car takes t hours
to focus on upliftment of rural India by = tan = tan  π – 
3  3 Then time taken by second car
increasing the credit flow for elevation
= (t – 3)
of agriculture & rural non-farm π
= – tan Distance between A and B be x
sector. It is headquartered in Mumbai 3
(Maharashtra). Both distances would be equal
= – tan 60º = – 3 30 t = 36 (t – 3)
74. (d) As per Article 213 of Indian
Constitution, an Ordinance promulgated 80. (d) Length of each piece 30 t = 36 t – 108
by the Governor of a state has to be (36 – 30) t = 108
1
laid before the Legislative Assembly =   m = 0.0625 m 6 t = 108
 16 
or where there is a Legislative Council 108
\ Required number of pieces t= = 18 hours
in the State, before both the Houses. It 6
ceases to operate at the expiration of  31.25   3125 × 100 
=  =  = 500 Distance between A and B = 30 ×
six weeks from the reassembly of the  0.0625   625  18 = 540 km.
Legislature, or if before the expiration
of this period a resolution disapproving 81. (d) Total weight of 39 girls 86. (c) Only daughter of woman's
it is passed by the Legislative Assembly = 60 × 39 kg
father  she herself.
and agreed to by the Legislative Council. = 2340 kg
Total weight of full class So, person  woman's son.
75. (b) The Indian National Army
was first formed in 1942 under Mohan = 64 × 65 Therefore, the ‘woman’ is the
Singh, by Indian prisoners of war of = 4160 kg mother of the person.
the British- Indian Army captured by Average weight of boys 87. (b) As,
Japan in the Malayan campaign and 4160 − 2340
at Singapore. However, it soon fell =
into decline. It was revived under the 26
leadership of Subhash Chandra Bose 1820
after his arrival in Southeast Asia in = = 70 kg.
26
1943.
Practice Set-18 233

Similarly, 91. (d) By analysing the above H K N M N O


8 11 14 13 14 15
circular arrangement, it is clear that
C or G  to the immediate right of B. +3 +3 +1 +1

E  to the immediate right of H. Finally the odd word pair is MNO.


A  to the immediate right of F. 94. (c) According to dictionary,
D  to the immediate right of G or C. the arrangement of the words is given
below:
Finally, ‘PUBLIC’ is written as Therefore, none of the options is
(iv) Shadiness,
‘DWRAGJ’. correct.
92. (b) According to the given (v) Shallow,
88. (d) According to the question,
information (iii) Shamelessly,
the direction diagram will be as follows
6x a 5y x = 5y ...(i) (ii) Shore,
and 2y b 3z  2y > 3z ...(ii) (i) Shortage
From (i), we get 95. (a) thou ⇒ 8, good ⇒ 5
6x The code for no may be 0.
=y ...(iii)
5 96. (a) By analysing the above
table, it is clear that Profession of 'V →
From (ii), we get Engineer.
A = Initial point 3  97. (c) By analysing the above table,
y> z ...(iv)
E = Finishing point 2 it is clear that R Travels in Vehicle III.
AB = 5 ft From (iii) and (iv), we get 98. (b) By analysing the above
B C = DE = 6 ft table, it is clear that Three Teachers
6x 3
B E = CD = 4 ft > z → W, P and T.
∴ Required distance, BE = 4 ft 5 2
99. (d) By analysing the above
89. (b) Decreasing order (agewise) 
12x > 15z table, it is clear that V → Female →
P > R > Q > S 
4x > 5z Engineer → Teacher.
Decreasing order (moneywise)

4x > 3z Therefore, option (d) is not correct.
R > S > P > Q

4x b 3z. 100. (b) By analysing the above
90. (b)
table, it is clear that total Females → W,
B D G K M P T V Y C E H 93. (d) B E H D G J
2 4 7 +4 11 13 16 +4 20 22 25 +4 3 5 8 2 5 8 4 7 10 P, V and R i.e. Four.
+2 +3 +2 +3 +2 +3 +2 +3 +3
+3 +3 +3 qqq
Finally, the missing term is CEH.
234 Practice Set-19

Air Force Airmen Group X & Y Exam


Practice Set-19

announcement, the president of Earth Direction (7–8): In the following


ENGLISH
leaned back in his presidential chair and questions, choose the word opposite in
Directions (1-4): Read the given gave his most reassuring smile. Well meaning to the given word.
passage and answer the questions that's it folks. Take care, while we get to 7. Escalate
that follows by selecting the most the bottom of this. You can be sure we (a) decrease (b) descend
appropriate option. are working on it hard. So, bye for now, (c) deliver (d) derive
The President of Earth glowered at the from your president, Goodbye! 8. Anarchy
autocue and the television camera in 1. The contradiction is conveyed in (a) curfew (b) permanence
the small presidential studio which was the phrase ridiculous public service (c) wholesome(d) order
where he should have been right now emergency announcements by the 9. Fill in the blank with the correct
word– phrasal verb.
instead of reading ridiculous public
It isn't easy to .......... children
service emergency announcements. (a) emergency (b) service nowadays.
It was his day off and he reckoned (c) public (d) ridiculous
(a) train up (b) bring up
that the world and its wretched public 2. The incongruity lies in–
(c) put up (d) leave out
service announcements should wait until (a) unfamiliar objects that cannot be Directions (10): In the following
tomorrow. Unfortunately, the world didn't touched question out of the four alternatives,
want to wait and the World Parliament (b) f a m i l i a r o b j e c t s s e e n i n choose one which can be substituted
had prepared this announcement for unexpected places for the given words/sentence.
him to read, so he was glowering at (c) strange objects seen everywhere 10. An official call to appear in a court
of law
everyone. The autocue started rolling and frequently
(a) summon
and the President relaxed has scowling (d) new objects appearing where (b) notice
face and beamed into the television none existed before (c) memorandum
camera, exuding bonhomie to the billions 3. The expression exuding bonhomie (d) petition
who were watching as he read from the means– Directions (11–12): In the following
autocue. (a) appearing tired questions, a sentence has been given
Scientists are today investigating in Active Voice/Passive Voice. Out of
(b) lacking patience
the four alternatives suggested, select
objects that are appearing across the (c) appearing friendly the one which best expresses the same
world and which seem to present a degree (d) showing pride sentence in Passive/Active Voice.
of danger to anybody touching them. 4. A word that can replace the phrase 11. I have bought a new car.
World scientists are advising that, under 'get to the bottom of' is– (a) A new car was bought by me.
no circumstances should these objects (a) estimate (b) explore (b) A new car is brought by me.
be touched. A heavy metal object that (c) clarify (d) misunderstand (c) A new car has been bought by me.
is floating on a pond would be such an (d) A new car had been bought by
Direction (5–6): In the following
example. A tree growing out of concrete me.
questions, groups of four words are 12. Teachers might have given their
would be another. However, there is given. In each group, one word is students some concessions.
no guarantee that objects will always correctly spelt. Find the correctly (a) Their students might have
have this incongruity and scientists are spelt word. given some concessions to their
advising the general public to proceed 5. (a) height (b) hieght teachers.
with great caution, especially when at (c) highte (d) heite (b) Their students might be given
home where unfamiliar objects may be 6. (a) label (b) lebal some concession by their
more easily identified. After reading the (c) labal (d) labbel teachers.
Practice Set-19 235

(c) Their students might be giving (c) She expressed how ugly she I2 ω
some concessions to their looked in that dress. (a) (b) ω
teachers. (d) She exclaimed that she looked I1 + I 2
(d) Students might have been given very ugly in that dress. I1ω (I1 + I 2 )ω
some concessions by their Direction (18–20): In the following (c) (d)
I1 + I 2 I1
teachers. questions, some of the sentences have
Directions (13): In the following errors and some have not. Find out 24. In which one of the following cases
question, four alternatives are given which part of the sentence has an will the liquid flow in a pipe be
for the idiom/phrase printed in bold error. The number of that part is most streamlined?
in the sentence. Choose the alternative your answer. If there is no error, your (a) Liquid of high viscosity and
which best expresses the meaning of answer is (d). high density flowing through a
the idiom/phrase. 18. All the members in the council (a)/ pipe of small radius.
13. The car in the backyard is proving began shouting (b)/at each other. (c)/ (b) Liquid of high viscosity and low
to be a white elephant.
No error (d). density flowing through a pipe
(a) very huge item of small radius.
19. No sooner did the fisherman Abdul
(b) costly and useless possession (c) Liquid of low viscosity and low
Sattar (a)/ see the bus plunge into the
(c) very expensive investment density flowing through a pipe
river (b)/that he immediately rowed
(d) useful material of large radius.
his boat to the site and jumped in.
Direction (14–15): In the following (c)/No error (d). (d) Liquid of low viscosity and high
questions, out of the four alternatives, density flowing through a pipe
choose the one which best expresses the 20. A pair of shoes (a)/were standing (b)/
of large radius.
meaning of the given word. in the corner. (c)/No error (d)
25. In the given p-V diagram,
14. Candid
PHYSICS the path (z) from A to B is
(a) overconfident
zig-zag path, but (1) is simple
(b) frank 21. Two bodies having masses m1 = 40
(c) arrogant path. Then,
g and m2 = 60 g are attached to the
(d) careless
end of a string of negligible mass
15. Zealous
(a) ardent (b) jealous
and suspended from massless p A
1
pulley. The acceleration of the
(c) furious (d) impatient B
Direction (16–17): In the following bodies is:
questions, a sentance has been given (a) 1 ms–2 (b) 2 ms–2
in Direct Speech. Out of the four (c) 0.4 ms –2
(d) 4 ms–2 2
alternatives suggested, select the one 22. A body of mass 4 kg moving V
which best expresses the same sentence with velocity 12 m/s collides with
in Indirect Speech. (a) W1 = W2
another body of mass 6 kg at rest. (b) ∆U1 = ∆U2
16. "Do you want balloons?" he said
to the child. If two bodies stick together after (c) W1 > W2
(a) He asked the child if it wanted collision, then the loss of kinetic (d) (b) and (c) are true
balloons. energy of system is: 26. A particle executes simple
(b) He asked the child whether it had (a) zero (b) 288 J harmonic oscillation with an
wanted balloons. (c) 172.8 J (d) 144 J amplitude a. The period of
(c) He asked the child did it want 23. A round disc of moment of inertia oscillation is T. The minimum
balloons. I2 about its axis perpendicular to time taken by the particle to
(d) He asked the child if it would its plane and passing through its travel half of the amplitude from
want balloons. centre is placed over another disc the equilibrium position is:
17. She said, "How ugly I look in of moment of inertia I1 rotating
this drees!" with an angular velocity ω about T T
(a) (b)
(a) She said that how ugly she was the same axis. The final angular 4 8
looking in that dress.
velocity of the combination of T T
(b) She exclaimed how ugly she (c) (d)
looked in that dress. discs is: 12 2
236 Practice Set-19

27. Two condensers, one of capacity C (c) Average energy density of (a) p2 = p1
C electric field equals the average (b) p2 > p1
and the other of capacity , are energy density of magnetic
2 (c) p2 < p1
connected to a V volt battery, as field
(d) None of the above (d) Cannot be predicted
shown. The work done is charging 34. The Earth is assumed to be a
fully both the condensers is: 30. Above curie temperature:
(a) a ferromagnetic substance sphere of radius R. A platform
becomes paramagnetic is arranged at a height R from
C (b) a paramagnetic substance the surface of the Earth. The
V C becomes diamagnetic escape velocity of a body from
2
(c) a d i a m a g n e t i c s u b s t a n c e
this platform is fve, where Ve is its
becomes paramagnetic
(d) a paramagnetic substance escape velocity from the surface of
1 the Earth. The value of f is:
(a) 2CV2 (b) CV 2 becomes ferromagnetic
4
31. When air is replaced by a 1
3 1 (a) 2 (b)
(c) CV 2 (d) CV 2 dielectric medium of constant K. 2
4 2 The maximum force of attraction
between two charges separated by 1 1
28. There are two coils A and B as (c) (d)
a distance. 3 2
shown in figure, A current starts
flowing in B as shown, when A is (a) increases K–1 times 35. A conducting circular loop is
moved towards B and stops when (b) increased K times placed in a uniform magnetic
(c) decreases K–1 times field, B = 0.025 T with its plane
A stops moving. The current in
(d) remains constant perpendicular to loop. The radius
A is counter-clockwise. B is kept
32. Two cells of emfs approximately of the loop is made to shrink at
stationary when A moves. We can 5 V and 10 V are to be accurately
infer that: a constant rate of 1 mms–1. The
compared using a potentiometer
A B induced emf when the radius is
of length 400 cm.
2 cm, is:
v (a) The battery that runs the
potentiometer should have (a) 2πµV (b) πµV
voltage of 8 V π
(c) μV (d) 2µV
(a) There is a constant current in (b) The battery of potentiometer 2
can have a voltage of 15 V and 36. In Young’s double slit experiment,
the clockwise direction in A
R adjusted so that the potential one of the slits is painted so that
(b) There is a varying current in A drop across the wire slightly
(c) There is no current in A intensity of light emitted from it is
exceeds 10 V
(d) There is a constant current in (c) Potentiometer is usually used half of that of the light-other slit.
the counter-clockwise direction for comparing resistances and Then,
in A not voltages (a) fringe system will disappear
29. In a plane electromagnetic (d) None of the above (b) bright fringes will becomes
33. In the given V-T diagram, what is brighter and dark fringes will
wave the electric field oscillates
the relation between pressures p1 be darker
sinusoidally at a frequency of
and p2? (c) Both bright and dark fringes will
2 × 1 0 10 H z a n d a m p l i t u d e
V become darker
48 Vm–1. Then, which one of the
following statement is true? p2 (d) dark fringes will become
(a) Wavelength the wave is 2 × 105 brighter and bright fringes
m p1 will become less bright
(b) Amplitude of oscillating 37. To explain his theory Bohr used:
θ2 (a) c o n s e r v a t i o n o f l i n e a r
magnetic field is 48 T
θ1 momentum
T
Practice Set-19 237

(b) c o n s e r v a t i o n o f a n g u l a r (c) drop to zero when α = β (a) 0, 5 (b) 6, 3


momentum (d) increase with time (c) 6, 6 (d) 5, 4
(c) conservation of quantum 42. Dimensions of the resistance in 47. If z + z–1 = 1, then z100 + z–100 is
an electrical circuit in terms of
frequency equal to:
dimension of mass M, length L,
(d) conservation of energy (a) L (b) –i
time T and current I, are:
38. A proper combination of 3 NOT (c) 1 (d) –1
(a) [ML2T–3A–1] (b) [ML2T–2]
and 1 AND gates is shown. If A = 48. If a and b are the roots of the
(c) [ML2T–1A–1] (d) [ML2T–3A–2]
0, B = 1, C =1, then the output of equations x2 + x + 2, then what is
43. For a cell of emf 2 V, a balance
this combination is: is obtained for 50 cm of the α10 + β10
equal to?
A potentiometer wire. If the cell is α –10 + β –10
shunted by a 2 Ω resistor and
B Y the balance is obtained across 40 (a) 4096 (b) 2048
cm of the wire, then the internal (c) 1024 (d) 612
C 49. If the non-zero numbers a, b, c
resistance of the cell is:
(a) 1 (a) 0.025 Ω (b) 0.50 Ω are in A ? and tan–1 a, tan–1 b,
(b) zero (c) 0.80 Ω (d) 1.00 Ω tan–1 c are also in AP, then
(c) not predictable 44. To obtain a magnified image at (a) a = b = c (b) b = 2ac
distance of distinct vision with a 2
(c) a = bc (d) c2 = ab
(d) None of these
simple microscope where should 50. What is the middle term in the
39. Consider telecommunication
the object be placed? 8
through optical fibres. Which of expansion of  1 – x  ?
(a) Away from focus  2
the following statements is not
(b) At focus
true? (c) Between focus and optical 35 x 4
(a)
(a) Optical fibres can be of graded centre 8
refractive index (d) Behind the object 17 x 5
(b)
(b) Optical fibres are subjected to 45. The instantaneous values of 8
electromagnetic interference alternating current and voltages
35 x 5
from outside in a circuit are given as: (c)
8
(c) Optical fibres have extermely 1
i=
sin (100 πt) ampere (d) None of these
low transmission loss 2
(d) Optical fibres may have   π 
1 51. lim  x tan x –   sec x  is
homogeneous core with a e=
sin (100 πt + π/3) volt x→ π / 2   2 
2
suitable cladding equal to:
The average power (in W)
40. Threshold wavelength depends (a) 1 (b) –1
consumed in the circuit is:
upon: (c) 0 (d) 2
1 3
(a) frequency of radiation (a) (b) d2 x
4 4 52. If y = x – ex, what is equal
(b) velocity of electrons dy2
to?
(c) work function 1 1
(c) (d) ex
(d) None of the above 2 8 (a) ex (b) −
(1 + e x )3
41. The displacement x of a particle
varies with time t as x = ae–αt + beβt,
MATHEMATICS ex ex
(c) – (d) –
where a, b, α and β are positive 46. Two finite sets having m and n (1 + e x ) (1 + e x ) 2
constants. The velocity of the elements. The total number of
x
particle will:
subsets of the first set is 56 more 53. Evaluate ∫e ( x + 1) dx.
than the total number of subsets
(a) decrease with time of the second set. Find the values (a) ex + C (b) xex + C
(b) be independent of α and β of m and n. (c) ex + x + C (d) ex + 1 + C
238 Practice Set-19

54. What is the foot of the 61. Let f(x) = (1 + b)2 x2 + 2bx + 1 (c) whose centre is (–1, 2)
perpendicular from the point (2, and m(b) be the minimum value
(d) whose eccentricity is 19
3) on the line x + y – 11 = 0? of f(x). As b varies, the range of
3
m(b) is:
(a) (1, 10) (b) (5, 6) 68. What is the value of sin 15º?
 1
(c) (6, 5) (d) (7, 4) (a) [0, 2] (b) 0, 
 2 (a) 3 –1 (b) 3 +1
55. The sum of 2 + 6 + 12 + 20 + .....∞ 1  2 2 2 2
1! 2! 3 4! (c)  , 1 (d) [0, 1]
is 2  3 –1 3 +1
3e (c) (d)
(a) (b) e 62. The probability that a leap year
2 3 +1 3 –1
selected at rander contains 53
(c) 2e (d) 3e Sundays is: 69. If the sides of a DABC satisfy 3a
56. The equation of the circle passing (a) 7/366 (b) 26/183  B  C
= b + c, then cot   cot   is
 2  2
through (4, 5) having the centre (c) 1/7 (d) 2/7 equal to:
at (2, 2) is: 63. In how many ways can one select
(a) 0 (b) 1
a cricket team of eleven from 17
(a) x2 + y2 + 4x + 4y – 5 = 0 players in which only 5 players (c) 2 (d) 3
(b) x2 + y2 – 4x – 4y – 5 = 0 can bowl, if each cricket team 1 0 1 x
70. Let A =  0 –1 and B =  0 1 
of 11 must include exactly 4    
(c) x2 y2
+ – 4x = 13
bowlers ? If AB = BA, then what is the
(d) x2 + y2 – 4x – 4y + 5 = 0
(a) 3960 value of x?
1
57. Find the value of log 27  . (b) 3961 (a) –1
 81 
(c) 3962 (b) 0
3 4 (d) None of these (c) 1
(a) (b)
4 3 2 (d) Any real number
–3 –4
64. What is ∫1 log x dx equal to?
(c) (d) REASONING AND GENERAL
4 3 (a) loge 2 (b) 1
AWARENESS (RAGA)
58. For all n ∈N ', n(n + 1) (n + 5) is 4 e
a multiple of: (c) log e   (d) log e   71. In which language did Babur
e 4 write his memoirs called ‘Tuzuk-
(a) 4 (b) 3 i-Baburi’?
(c) 5 (d) 7 65. If c2 = a2 + b2, then 4s (s – a) (s – b)
(a) Persian (b) Arabic
59. The solution of the equation (s – c) is equal to:
(c) Mongol (d) Turkish
 dy 
(a) s4 (b) b2 c2
log   = ax + by is
72. The point on the Earth surface
(c) c a 2 2 (d) a2b2
 dx  directly above the origin of an
66. What is the value of earthquake is known as
ex ex (a) Prime Centre
(a) = +C  4 12 
b a cos  cos –1 + cos –1
?
 5 13  (b) Hypo Centre
e – bx e a x (a) 63/65 (b) 33/65 (c) Epicentre
(b) = +C
–b a (c) 22/65 (d) 11/55 (d) Medi Centre
67. The equation 15x2 – 3y2 – 32x 73. Ornithology is the study of
e – by e a x
(c) = +C (a) Insects (b) Crustaceans
a b + 12y – 44 = 0 represents a
(c) Birds (d) Bees
(d) None of these hyperbola:
74. Breaking logical memory into
60. What is the standard deviation (a) the length of whose transverse
blocks of the same size is called
of numbers 7, 9, 11, 13 and 15? axis is 4 3 as
(a) 2.2 (b) 2.4 (b) the length of whose conjugate (a) Frames (b) Packets
(c) 2.6 (d) 2.8 axis is 4 (c) Segments (d) Pages
Practice Set-19 239

75. CENVAT is related to (c) 13.27 percent (a) 110 m (b) 70 m


(a) Sales Tax (d) 18.06 percent (c) 50 m (d) 40 m
(b) Excise Duty 83. A shopkeeper by selling 23 Lotto 89. The age of Ram is twice the
(c) Custom Duty shoes, earns a profit equal to the age of Shyam and half the age
(d) Service Tax selling price of 7 Lotto shoes. His of Sohan, Shyam is elder than
76. The Vice President of India may profit percentage is : Mohan. Who is the eldest?
be impeached by (a) 43.75 percent (a) Mohan (b) Ram
(a) Lok Sabha
(b) 30.4 percent (c) Sohan (d) Shyam
(b) Rajya Sabha
(c) Vidhan Sabha (c) 60.8 percent Direction (90): A series is given with
(d) Vidhan Parishad (d) 17.05 percent one term missing. Select the correct
77. How much year of extension was 84. In an army selection process, the alternative from the given ones that
given to the tenure of National ratio of selected to unselected was will complete the series.
Anti-profiteering Authority 3:1. If 60 less had applied and 30 90. 11, 13, 17, 19, ?
(NAAs) in 35th Goods and less selected, the ratio of selected (a) 22 (b) 23
Service Tax (GST) Council to unselected would have been (c) 29 (d) 31
Meeting? 5:1. How many candidates had Direction (91): Study the Following
(a) Five years applied for the process? information carefully and answer the
(b) Four years (a) 240 (b) 480 questions given below:
(c) Three years (c) 120 (d) 720 A, B, C, D, E, F, G and H are sitting
(d) Two years 85. A bank offers 5% compound around a circle facing the centre. B is
78. 2 × [0.3(1.3 + 3.7)] of 0.8 = ? interest per half year. A customer second to the right of H and third to
(a) 1.9 (b) 0.72 deposits ` 3200 each on 1st the left of A. D is not an immediate
(c) 2.16 (d) 2.4 January and 1st July of a year. At neighbour of either B or H and is
79. If sin (A/2) = x, then the value of the end of the year, the amount second to the right of F. C is fourth to
x is he would have gained by way of the right of G.
(a) √[(1 + cos A)/2] interest is: 91. Who is fourth to the right of H?
(b) √[(1–cosA)/2] (a) ` 976 (b) ` 244 (a) B
(c) √[(1 – sinA)/2] (c) ` 488 (d) ` 122 (b) A
(d) √[(1 + sinA/2] 86. Pointing to a boy, Deepa said, (c) F
"He is the son of my father's only (d) Data insufficient
169 16.9 brother". How is Deepa related
80. If = , then the value Direction (92): In following questions
0.169 x to that boy?
a stands for equal to’; b for ‘greater
of x is: (a) Sister than’; γ for ‘less than’ and d for ‘not
(a) 0.0169 (b) 1.69 (b) Cousin (sister)
equal to.
(c) 16.9 (d) 169 (c) Daughter 92. If ax γ by, bx a cz and b2 a ac,
81. The mean of marks secured by (d) Data Inadequate then
55 students in division A of class 87. In a certain code language, (a) ax b cy (b) ay a cz
X is 58, 45 students of division B “MASTER” is written (c) y γ z (d) y b z
is 54 and that of 75 students of as “RETSAM”. How is
Direction (93): Select the odd word/
division C is 52. Find the mean “LITERACY” written in that
letters/number/number pair from the
of marks of the students of three code language?
divisions of Class X. given alternatives.
(a) ETICRACY 93. (a) 5210 (b) 8432
(a) 53.7 (b) 54.4 (b) YCARETIL (c) 8648 (d) 7240
(c) 53 (d) 55.8 (c) YARCETIC Direction (44): Arrange the given
82. A student multiplied a number (d) ETICYACR words in the sequence in which they
by 6/7 instead of 7/6. What is 88. In an open ground, Rakesh occur in the dictionary.
the percentage error in the walks 20 m towards North, turns 94. (i) Application (ii) Approve
calculation? left and goes 40. He turns to his (iii) Appeal (iv) Astonishing
(a) 36.11 percent left again to walks 50 km. How (a) ii, iii, iv, i (b) iii, ii, i, iv
(b) 26.53 percent far is he from starting point? (c) iii, i, ii, iv (d) i, ii, iii, iv
240 Practice Set-19

95. In the following figure, rectangle answer the questions given below: 97. Who left for Bangalore?
represents Physicians, circle P, Q, R, S, T, V and W are seven friends (a) T
represents Racers, triangle who left for seven different destinations– (b) P
represents writers and square Delhi, Chennai, Hyderabad, Bangalore, (c) V
represents Mothers. Which set of Kolkata, Chandigarh and Jaipur-each (d) Data inadequate
letters represents Mothers who one on a different day of the week. R 98. On which day of the week did T
are not Racers? left for Jaipur on Monday. On the last leave ?
day of the week the person left for (a) Tuesday (b) Thursday
A
Bangalore. T left the next day of P, who (c) Sunday (d) Wednesday
C left for Chandigarh and on the previous 99. Which of the following
B I day of W. S left for Kolkata on Friday. combinations of person-place is
D H Q did not leave for either Hyderabad or not correct ?
E F Bangalore and W left for Delhi. (a) R – Jaipur
G 96. On which day of the week did Q (b) P – Chandigarh
leave ? (c) T – Hyderbad
(a) FGH (b) ECA (a) Sunday (d) All are correct
(c) DFG (d) DGFI (b) Saturday 100. Who left on Tuesday ?
Directions (96–100): Study the (c) Wednesday (a) P (b) W
following information carefully and (d) Data inadequate (c) Q (d) V

Answers with Explanation


12. (d) Students might have been 19. (c) ‘No sooner ............. than’ is
ENGLISH
given some concessions by their correct form of correlative. Hence, than
1. (d) 2. (b) teachers. he immediately rowed his ....... will be
3. (c) Bonhomie' means appearing 13. (b) correct usage.
friendly. 14. (b) The word Candid (Adjective) 20. (b) In separables, like scissors,
4. (c) Get to the bottom of can be means: saying what you think openly shears, trousers, shoes take plural verb
replaced with to 'clarify'. and honestly; frank. when used as subject. If 'a pair of' is
5. (a) 6. (a) Look at the sentence: used before them, then singular verb is
7. (a) The word Escalate (Verb) 'I don't trust him' he said in a candid used.
means: increase; intensify; to become statement. Hence, is/was standing ............
or make something greater, worse, more
serious, etc. 15 (a) The word Zealous (Adjective) should be used.
Look at the sentence: means: showing great energy and
enthusiasm for something. PHYSICS
We do not want to escalate the war.
Hence, its antonym should be decrease. The word Ardent (Adjective) 21. (b)
8. (d) The word Anarchy (Noun) means: very enthusiastic and showing (m2 – m1 ) (60 – 40) × 10
   
a= g=
means: a situation in a country, an strong feelings about something/ (m1 + m2 ) 60 + 40
organisation in which there is no somebody; passionate.
    = 2 ms–2
government, order or control. 16. (a) He asked the child if it wanted
balloons. 22. (c) Given, m1 = 4 kg, u1 = 12
Look at the sentence:
17. (d) She exclaimed that she looked m/s, m2 = 6 kg, u2 = 0
There was complete anarchy in the
classroom when their teacher was away. very ugly in that dress. 1 4× 6
18. (c) 'Each other' is used when two ∴ ∆KE = (12 – 0) 2
Hence, its antonym should be order. 2 (4 + 6)
persons or things are concerned; 'one
9. (b)
another' is used when more than two 1 24
10. (a) summon = × × (12) 2
persons or things are concerned. 2 10
11. (c) A new car has been bought Hence, at one another should be used
by me. here. = 172.8 J
Practice Set-19 241

23. (c) The angular momentum of a π  2π  2


⇒ t = 1 2 1  B0 
disc of moment of inertia I1 and rotating  as ω =  UB = B =  
 2π   T  2μ 0 2μ 0  2 
about its axis with angular velocity ω 6 
 T 
is 1 B2  E0 
T = .B20 = 0 ∴ B0 = c 
L1 = I1ω ⇒ t = 4μ 0 4μ 0  
When a round disc of moment of 12
2
inertia I2 is placed on first disc, then Hence, the particle travels half of  E0 
 c  2
angular momentum of the combination the implitude from the equilibrium in   = 1 E0
=
is T 4μ 0 4μ 20 c 2
s.
L2 = (I1 + I2)ω' 12  
E 02 1
In the absence of any external 27. (c) The two condensers in the = ∴ c = 
2  μ 
 1   0ε0 
torque, angular momentum remains circuit are in parallel order. 4μ 0  
conserved i.e., L1 = L2  μ 0 ε 0 
hence 
Iω C 3C μ0ε0 2 1
I1ω = (I1 + I2)ω' ⇒ ω' = 1 C' = C + = E 0 = ε 0 E 02
I1 + I 2 2 2 =
4μ 0 4
24. (b) F o r s t r e a m l i n e f l o w, The work done in the charging the
Hence option (c) is true.
rρ equivalent capacitor is stored in the
Reynold’s’s number N R ∝ should form of potential energy. 30. (a) Ferromagnetism decreases
η with rise in termperature. If we heat
Hence.
be less. For less value of NR. radius and a ferromagnetic substance, then at a
density should be small and viscosity W=U definite temperature the ferromagnetic
should be high. 1 1  3C  2 3 property of the substance suddenly
= C ' V2 =   V = CV
2

2 2 2  4 disappears and the substance becomes


25. (d) Work done depends upon the paramagnetic. The temperature above
28. (d) Coil A must be carrying a which a ferromagnetic substance
path followed by process.
constant current in counter-clockwise becomes paramagnetic is called the
So, W ≠W1 2 direction. That is why when A moves Curie temperature of the substance.
Since, change in internal energy towards B, current induced in B is
depends upon the initial and final state, it 1 q1q2
in counter-clockwise direction, as per 31. (a) F = .
does not depend upon the path followed 4π ∈ r 2
Lenz’s law. The current in B would
by the process ∆U1 = ∆U2 q1q2 F
stop when A stops moving. = = 0
The area under p-V diagram gives 4π ∈0 kr 2 k
29. (c) Wavelength,
the work done by the gas. 32. (b) In a potentiometer experiment,
Here. path (1) is greater than path c 3 × 108
λ=
= the emf of a cell can be measured if the
(2), so W1 > W2. Hence option (d) is f 2 × 1010 potential drop along the potentiometer
true. = 1.5 × 10–2m wire is more than the emf of the cell
26. (c) Let displacement equation of Hence, option (a) is false. to be determined. As values of emfs of
particle executing SHM is y = a sin ωt. two cells are approximately 5 V and 10
E0 48
As particle travels half of the B0 = = = 16 × 10–8 T V, therefore, the potential drop along the
c 3 × 108
amplitude from the equilibrium position, potentiometer wire must be more than
= 16 × 10–7 T
a 10 V. Hence, option (b) is correct.
so y = Hence, option (b) is false.
2 1
Energy density of electric field is 33. (c) The slope of the graph ∝
a ρ
Therefore, = a sin ωt ⇒ sin ωt
2 1 1 E 
2

UC = ε 0 E 2 = ε 0  0  34. (b) Escape energy = Binding


1 π
= = sin 2 2  2 energy of sphere
2 6
π π 1 1 GMm
⇒ ωt = ⇒ t= ε0 E02 ⇒ mv 'e2 =
= 2 R+h
6 6ω 4
242 Practice Set-19

(i) This works on the principle of 45. (d) Given equation are
2GM 2GM
⇒ ve' = = total internal reflection. 1
R+h 2R I= sin (100 πt )
(ii) It consists of core made up 2
(∵ h = R) of glass/slica/plastic with refractive
index n1. which is surrounded by a 1
2GM and e= sin (100 πt + π / 3)
But at surface of Earth, ve = glass or plastic cladding with refractive 2
R index n2 (n2 > n1). The refractive index
As given, ve' = fve 1
of cladding can be either changing ∴ i0 =
abruptly or gradually changing (graded 2
2GM 2GM 1
Hence, = f index fibre). and e0 =
2R R 2
(iii) There is a very little We know that average power
1 transmission loss through optical fibres.
∴f= Pav = Vrms × irms cos φ
2 (iv) There is no interference from
stray electric and magnetic fields to the 1 1
35. (b) Magnetic flux, φ = B. A = B. = × × cos 60º
signals through optical fibres. 2 2
πr2
40. (c) T h r e s h o l d w a v e l e n g t h i0
Induced emf, | e | V
depends upon work function. [∴ irms = and Vrms = 0 ]
2 2
dφ dr 41. (d) Given, x = ae–αt + beβt
= = Bπ2r 1 1 1 1
dt dt Velocity = × × = W
2 2 2 8
= 0.025 × π × 2 × 2 × 10–2 × 1 × 10–3 dx
v= = – aα e –α t + bβeβt = A + B
= πµV dt MATHEMATICS
where, A = –aαe . B = bβe
–αt βt
46. (b) Let A and B be two sets
I ( I1 + I 2 ) 2
36. (d) max = The value of term A = –aαe–αt having m and n elements respectively.
I min ( I1 – I 2 ) 2 decreases and of term B = bβeβt increases Then.
with time. As a result, velocity goes on Number of subsets of A = 2m
For I1 = I2; Imin = 0 and Imax = 4I1
increasing with time.
Number of subsets of B = 2n
l1 42. (d) Resistance,
and if I2 = , then Imin = 0.091 and Imax It is given that 2m – 2n = 56
2 Potential difference
R= ⇒ 2n (2m – n – 1) = 23 (23 – 1)
= 2.89 I1. As such bright fringes will Current
become less brighter while dark fringes ⇒ n = 3 and m – n = 3
V W  W ⇒ n = 3 and m = 6
will become brighter. = =   V = 
I QI  Q 47. (d) z + z–1 = 1 ⇒ z2 – z + 1 = 0
37. (b) Bohr postulated that the
angular momentum of the electron = So, dimensions of R z = – ω or – ω2
nh/2π. [Dimension work] For z = –ω, we have,
38. (b) The simplified circuit is = z100 + z–100 = (–ω)100 + (–ω)–100
[Dimonsion of charged]
shown in figure below. [Dimention of current] 1
= ω+ = ω + ω2 = –1
A ω
A [ML2 T –2 ]
= = [ML2T–3A–2] For z = –ω2, we have
B [AT] [A]
B
Y = A·B·C z100 + z–100 = (–ω2)100 + (–ω2)–100
C
C 1 1
 I1 – I 2  = ω200 + 200 = ω +
2
43. (b) r =  R = 0.5 Ω ω ω2
So, output Y = A . B . C  I  2
= ω2 + ω = –1
If A = 0, B = 1, C = 1 44. (c) Magnifying the image of 48. (d) Given that, (α, β) are the
∴ Y = 0 . 1 . 1 = 1. 0 . 0 = 0 an object by a simple microscope the roots of the equation x2 + x + 2 = 0.
39. (b) Some of the characteristics object should be placed between focus
then α + β = –1 ...(i)
of an optical fibre are as follows: and optical centre.
and α. β = 2 ...(ii)
Practice Set-19 243
10 10
α +β
Now we have = (αβ)10 = 0 ⇒ (x – 2)2 + (y – 2)2 = 13
α –10 +β –10 53. (b) We know that [ x] = 1
2x ⇒ x2 + y2 – 3x – 4y – 5 = 0
(2)10 = 1024 [from Eq. (ii)] Using the formula, 57. (d)
49. (a) Since, 2b = a + c...(i)
and 2 tan–1 b = tan–1 a + tan–1 c ∫ e '( f ( x) + f '( x))dx = e
x
f(x) + C 1
log 27   = log 3 2(3) –4 =
–4
log 3 3 =
–4
 81  3 3
Assuming f(x) = x f’(x) = 1 we get
2b a+c 58. (b) Let the given statement be
⇒ ⇒ ⇒ b2 = ac
∫e
x
1–b 2
1 – ac ( x + 1)dx = ex – x + C
P(n),
 [by Eq. (i)] 54. (d) T h e e q u a t i o n o f l i n e i.e., P(n) = n(n + 1) (n + 5) is a
i.e., a b and c are in GP perpendicular to given line multiple of 3.
Also 4b2 = 4ac For n = 1
x + y – 11 = 0
1 (1 + 1) (1 + 5) = 1 × 2 × 6 = 12 =
⇒ (a2 + c)2 – 4ac = 0 [by Eq. (i)] is –x + y + λ = 0
3 × 4.
⇒ (a – c)2 = 0 ⇒ a = c = b This equation passes through (2, 3) which is a multiple of 3, that is true.
 [by Eq. (i)] ∴ –2 + 3 + λ = 0 Let it is true for n = k,
8 ⇒ λ = –1 i.e., k (k + 1) (k + 5) = 3λ
x
50. (a) Given expresson = 1 –  ∴ From Eq. (ii), ⇒ k(k2 + 5k + k + 5) = 3λ
 2
–x + y – 1 = 0 ⇒ k3 + 6k2 + 5k = 3λ ...(i)
Here, n = 8, which is an even
⇒ y=x+1 For n = k + 1
number
∴ From Eq. (i), (k + 1) (k + 1 + 1) (k + 1 + 5)
8 
So  + 1 th term is the modle x + x + 1 – 11 = 0 =  (k + 1) (k + 2) (k + 6) = (k2 + 2k
term 2  + k + 2) (k + 6)
⇒ 2x = 10 ⇒ x = 5
 x
4 = (k2 + 3k + 2) (k + 6)
Now, T1 = 8C4 (1)8 – 4  –  Hence co-ordinates of foot of
= k2 + 6k2 + 3k2 + 18k + 2k + 12
 2 perpendicular from (2, 3) given line is
(5, 6) = (k3) + 9k2 + 20k + 12
8 7 6 5  x 4  35 x 4 = (3λ – 6k2 – 5k) + 9k2 + 20k + 12
= . . .  – 2 6 12 20  [using Eq. (i)]
4 3 2 1  16  8 55. (d) Let S = + + + + ......
1! 2! 3! 4! =  3λ + 3k2 + 15k + 12 = 3 (λ + k2 +
51. (d) lim [ x tan x – (π / 2) sec x] and S = 2 + 6 + 12 + 20 + .... + Tr (let) 5k + 4).
x →π / 2

 [∞ – ∞ form] S = 2 + 6 + 12 + ..... + Tn–1 = Tn which is a multiple of 3.


0 = 2 + 4 + 6 + 8 + ..... + n terms – Tn Therefore, P(k + 1) is true when
2 x sin x – π  0  P(k) is true. Hence, from the principle of
= lim  form  Tr = 2 + 4 + 6 + 8 + .... + n terms
x→π / 2 2 cos x 0  n mathematical induction, the statement is
⇒ Tr = [2 × 2 + (n – 1/2)] = n (2 true for all natural numbers n.
(2sin x + 2 x cos x) 2
= lim 
x→π / 2 –2sin x + n – 1) + n (n + 1) dy
59. (b) = e a +by = e a eby ⇒ e–by dy = eax
 [by L1 Hospital’s rule] ∴ nth term of the given series dx dx

2.1 + 2. π / 2.0 1 1
= Tr = ∴ – e – by = e a x + C
= = –1 b a
–2.1
52. (b) y = x + en 60. (d) ∵ x
⇒ Tr =
dy dx 1 7 + 9 + 11 + 13 + 15 55
∴ = 1 + ex = = = = 11
dx dy 1 + e x Now sum 5 5
1 ∞
1 Now SD
= n∑=1– + 2∑ = e + 2e = 3e
d2x ex dx (n – 2)! n =1 ( n –1)!
⇒ 2
= – x 2
. (7 – 11) 2 + (9 – 11) 2 + (11 –1) 2 +
dy (1 + e ) dy 56. (b) Here, r = Distance between
(13 –11) 2 + (15 –11) 2
ex 1 ex (4, 5) and (2, 2) =
= – . = – 5
(1 + e x ) 2 (1 + e x ) (1 + e x )3 ∴ r2 = 4 + 9 = 12
244 Practice Set-19

16 + 4 – 0 + 4 + 16 = 5C4 × 12C7 = 5C1 × 12C5


  = 16 19
(∵ nCr = nCn – r ....) e= 1+ =
5 3 3
  = 8 = 2.8 approx. 5 × 12 × 11× 10 × 9 × 8 68. (a) sin 15º = sin (45º – 30º)
=
61. (d) f’(x) = 2x (1 + b2) + 2b = 0 120
= sin 45º cos 30º – cos 45º sin 30º
b  n n(n –1)(n – 2)(n – 3)(n – 4) 
∴x= –   C5 = 120  1 3 1 1 3 –1
(1 + b 2 )   = × – × =
2 2 2 2 2 2
= 5 × 11 × 9 × 8 = 55 × 72
f’’(x) = 2 (1 + b2) is always positive.
so that f(x) is minimum when = 3960 ways 8 C
69. (c) cot cot
2 2 2
x= –
b 64. (c) Let t = ∫1
log x dx
1 + b2
2  1 
2
=
∴ min f(x) t= ∫
1
log x .1 dx =  log x . x – ∫ . x dx 
 x 1
2 b2 2b 2
= (1 + b ) – +1 = (x log x – x)21
(1 + b ) (1 + b 2 )
2 2
= 2 log 2 – 2 – 1 . log 1 + 1 = =
(1 + b 2 ) – b 2 1 = 2 log 2 – 1 – 0 = 2 log 2 – 1
= =
1 + b2 (1 + b 2 ) 4
= loge 4 – loge e = logr
1 e = = =
∴ m(b) = = + ve
1 + b2 65. (d) ∵ c2 = a2 + b2 ⇒ ∠C = 90º
Clearly, m(b) is always greater than ∴ 4s(s – a) (s – b) (s – c) 1 0  1 x 
70. (b) AB =   
zero and less than or equal to 1. 1
2
0 –1 0 1 
= 4∆2 = 4  ab  = a 2 b 2
∴ So, the range of m(b) is [0, 1]. 2 
1 x 
62. (d) In a leap year there are 366 =  
 –1 4 12  0 1 
days, it means 52 full weeks and two 66. (c) cos cos + cos –1 
 5 13 
days. These two days can be (Mon, 1 x  1 0 
Tues) (Tues. Wed.) (Wed, Thus) (Thus,  4 12 4
2
 12  
2
and BA =   
0 1  0 –1
–1
= cos.cos  . – 1 –   1 –   
Fri) (Fri Sat) (Sat. Sun) (Sun, Mon).  5 13 5  13  
2 [ cos–1 x + cos–1 y = cos–1 1 – x 
So, required probability = =  
7 (xy – 1 – x 2 . 1 – y 2 )] 0 –1 
63. (a) Cricket team ∴ AB = BA
48 16 144
= – 1– 1–
65 25 169 1 x  1 – x 
Total 17 players ∴   = 0 –1  ⇒ x = –x
48 9 25 48 3 5  0 –1  
= – = –
65 25 169 65 5 13
5 Bowlers 12 Batsmen ⇒ 2x = 0 ⇒ x = 0
48 3 48 –15 33
We are to select 11 players including = – = = REASONING AND GENERAL
exactly 4 bowlers. 65 13 65 65
67. (d) We have 16(x2 – 2x) – 3(y2
AWARENESS (RAGA)
Hence 4 bowlers will be selected
from 5 bowlers and remaining 7 players – 4y) = 44 71. (d) Tuzuk-i-Baburi (Letters of
will be selected from 12 batsman. = 16 (x – 1)2 – 3(y – 3)2 = 48 Babur) is an autobiographical work of
Now 4 bowlers out of 5 bowlers Babur written in the Chagatai or Turkish
( x –1) 2 ( y – 2) 2
can be selected in 5C4 ways 7 players = – =1 language which was his native tongue.
3 16
out of 12 players can be selected in 12C7 Turkish was the spoken language of the
ways. Hence by FPC. total number of This equation represents type hyper- Andijan- Timurids. The book is also
ways selecting the eleven players bola with eccentricity. known as Baburnama.
Practice Set-19 245

72. (c) The epicentre is the point 78. (d) 2 × [0·3 (1·3 + 3·7)] of 5x – 150 = 4x – 60
on the Earth’s surface that is directly 0·8 = ? x = 150 – 60 = 90
above the focus, the point where an ? = 2 × [0·3 (5)] × 0·8 Hence, number of candidates applied
earthquake originates. In seismology, = 2 × 1·5 × 0·8 = 3x – 30
the epicentre is the point on the Earth’s = 3 × 90 – 30
= 2·4
surface directly above the point where = 270 – 30 = 240
the fault begins to rupture, and is the 79. (b) x = sin ( A / 2 )
85. (c)
area of greatest damage.
73. (c) Ornithology is a branch of (1 − cos A)  R   P × 10 × 1
= A = P 1 +  +
zoology that concerns the study of birds. 2 100   2 × 100 
The origins of the word ornithology 80. (a) 2
 5  3200 × 10 × 1
come from the Greek ornithologos and 169 16.9 169 × 100 16.9 A = 3200 1 + +
= ⇔ =  100  100 × 2
late 17th-century Latin ornithologia 0.169 x 169 x
meaning “bird science”. Ornithologists 16.9 21 21
⇔ x=
= 0.0169 = 3200 × × + 160
may study the behavior, physiology, and 20 20
1000
conservation of birds and bird habitats. = 8 × 441 + 160
74. (d) Paging is a memory 81. (b) Mean of Marks
= 3528 + 160 = 3688
management scheme that allows 55 × 58 + 45 × 54 + 75 × 52 Interest = 3688 – 3200 = ` 488
=
processes physical memory to be 55 + 45 + 75 86. (b) The boy  son of Deepa's
discontinuous, and which eliminates 3190 + 2430 + 3900
= father's only brother  son of Deepa's
problems with fragmentation by
175 uncle  Deepa's cousin.
allocating memory in equal sized blocks
known as pages (size is power of 2, 9520 Therefore, ‘Deepa’ is the Cousin
= = 54.4
between 512 bytes and 8192 bytes). 175 sister of the ‘boy’.
External fragmentation is avoided by 82. (b) Let the number be x 87. (b) As,
using paging technique. R E T S A M
7 6
75. (b) Central Value Added x– x
Tax (CENVAT) is a tax levied on Percentage error = 6 7 × 100%
the manufacture or production of 7
x
movable and marketable goods In 6
opposite
India. Popularly known as Central 49 x – 36 x
= × 100% MASTER → RETSAM
Excise Duty, CENVAT is a duty on 7
the manufacture/ production of goods x × 42 Similarly,
6
in India. An excise duty is charged on L IT E RA C Y Y CAR E T I L
13 × 6 x
goods produced within the country. = × 100%
76. (b) The Vice President cannot 7 x × 42
be formally impeached, unlike the 1300
= %
President. The Constitution states that 49
the Vice President can be removed by opposite
= 26.53%
a resolution of the Rajya Sabha passed 83. (a) Shopkeeper’s profit LITERACY → YCARETIL
by an effective majority (more than     = (23 – 7) = 16 Finally, ‘LITERACY’ is written as
50% of effective membership (total Profit Percentage ‘YCARETIL’.
membership – Vacant seats)) and agreed
7 × 100 88. (c) According to the question,
to by a simple majority (50% of present
    = = 43·75% the direction diagram is as follows
and voting members) of the Lok Sabha 16
(Article 67(b)).
84. (a) Let the number of candidates
77. (d) Tenure of National Anti- selected and unselected be 3x and x
profiteering Authority (NAA) was due No. of candidates applied
to end by 30 November 2019. GST = 3x + x = 4x
Council extended this tenure by two 4 x − 60 5
years, in order to enable it to take up all =
x − 30 1
the pending cases.
246 Practice Set-19

Here, PQ = RT = 20 m, QR = TP by g cz  bx = cz...(ii) Racers = {H, I}


= 40 m and b a ac  b = ac...(iii)
2 2
Writers = {B, C, D}
and RS = 50 m
From (ii), we get Mothers = {D, G, H, F}
Now, TS = RS – RT = 50 – 20 = 30 m
∴ Required distance, cz So, the required set of letters
x= ...(iv)
b = {D, F, G}
PS =
cz Hence, the set of letters D, F, G are
Putting x = in (i), we get
= b mothers which are not racers.
= acz 46. (a) By analysing the above table, it
< by
b is clear that Person Q Day
 Sunday.
= = 50 m  acz < b2y
47. (c) By analysing the above table,
89. (c) Let the age of Shyam = x yr  acz < acy
it is clear that Bangalore Destination

Then, age of Ram = 2x yr [∴ b2 = ac from (iii)] Person V.
and age of Sohan = 4x yr 
z<y
∴ 4x > 2x > x 48. (d) By analysing the above table, it

y>z is clear that Person T Day
On arranging the above data, we get  Wednesday.
∴ Sohan > Ram > Shyam > Mohan 
y b z.
49. (d) By analysing the above table, it
So, Sohan is the eldest. 93. (d) 5210 = 5 × 2 = 10
is clear that
90. (b) 11 8432 = 8 × 4 = 32
11 + 2 = 13 8648 = 8 × 6 = 48 Person R Destination
 Jaipur;
13 + 4 = 17 7240 = 7 × 2 = 14 Person P Destination
 Chandigarh.
17 + 2 = 19 Finally the odd number is 7240.
Person T Destination
 Hyderabad;
19 + 4 = 94. (c) According to dictionary, the
Finally, the missing term is 23. sequence of words is: Therefore, all are correct.
91. (c) By analysing the above (iii) Appeal, 50. (a) By analysing the above table, it
circular arrangement, it is clear that (i) Application, is clear that Person P Day
 Tuesday.
F  Fourth to the right of H. (ii) Approve,
92. (d) According to the given (iv) Astonishing. qqq
information 95. (c) Physicians = {A, C, D,
ax g by  ax < by...(i) E, F}
Practice Set-20 247

Air Force Airmen Group X & Y Exam


Practice Set-20

In America, two brothers, Wilbur (c) scholership


ENGLISH and Orville Wright, were carrying out (d) partnarship
Directions (1-4): Read the following experiments at the same time. They
Directions (6): In the following
passage and answer the questions were known as the "Wright Brothers".
They worked hard and tried several question, choose the word opposite
that follow by selecting the most
experiments. In 1903, they were able in meaning to the given word as your
appropriate option.
to make a machine which could fly in answer.
You have all seen birds flying. For
thousands of years men watched the the air. 6. Sacred
birds and wondered how they had this Although the first really successful (a) profuse (b) prolific
marvellous gift of flying. Looking at flight took place in 1903, an Italian (c) profound (d) profane
these birds, man first began dreaming named Leonardo da Vinci had thought of Directions (7-8): ln the following
about going up in the sky. Men tried to a rough form of an aeroplane more than
questions out of the four alternative,
copy these birds. They tried to make big five hundred years ago.
choose the one which best expresses the
kites which could carry them up. They The first machine was not very
meaning of the given as your answer.
tried various methods but could not fly. good. Before showing it to the people
many improvements had to be made. The 7. Concurrence
One winter night a man was sitting
near the fire. While looking at the fire, prople were delighted to see the machine (a) occurrence (b) conquest
he saw smoke rising from it. The man fly. The public gradually began to take (c) currency (d) agreement
watching the smoke wondered if the interest in these machines. 8. Espionage
smoke could also take other things up 1. What made man dream of flying (a) planning (b) pioneering
with it. He made a bag of silk and burnt in the sky? (c) lineage (d) spying
some paper below it. As soon as the (a) Flight of an aeroplane Directions (9-10): In the following
smoke collected in the bag, it started (b) Flight of birds questions, four alternatives are given
rising up. (c) Rise of smoke for the idiom/phrase printed in
Several other people also tried this (d) Flying kites
experiment and were successful. They Underlined in the sentence. Choose
2. Wright Brothers invented the– the alternative which best expresses
made bigger and bigger bags and filled
them with hot air. In big balloons they (a) glider (b) balloon the meaning of the idiom/phrase as
put animals in a small cage and sent them (c) engine (d) flying machine your answer.
up in the air. When the animals returned 3. Leonardo da Vinci belonged to– 9. He will certainly come to grief if he
safely, they sent up a man in a balloon. (a) Italy (b) Germany does not leave his present friends.
Balloons could fly only in the direction (c) America (d) England (a) addicted
in which the wind blow. Improvements 4. A balloon rises up by the power (b) go upto the extreme
began to take place and the balloons of– (c) suffer
could stay in the air much longer.
(a) engine (b) diesel (d) enjoy
Some people were also trying to
(c) hot air (d) petrol 10. If you develop friendship with an
experiment with other methods of flying
in the air. One new method was of flying Directions (5): In the following individual you must stand by him
in gliders. A glider is like a present day question, there are four different through thick and thin.
aeroplane, but has no machine or engine. words, out of which only one word (a) think about his/her welfare
Unlike the balloons, the pilot of a glider is correctly spelt. Find the correctly (b) under all circumstances
can control the direction of travel but spelt word. (c) to accompany through a thick
the glider can stay up only as long as the 5. (a) entrepreneurship forest
currents of air can support it. (b) leadeship (d) in day and night
248 Practice Set-20

Directions (11-12): In the following 16. "Have you completed your (a) 5 Ω (b) 376.6 Ω
questions, out of the four alternatives assignment, Minu ?" said her (c) 1883 Ω (d) 3776 Ω
choose the one which can be substituted brother.
23. At what temperature, the kinetic
for the given words/ sentence. (a) Minu's brother asked Minu if you energy of a gas molecule will be
11. Words or lines written on the tomb have finished your assignment.
of a person half of its value at 27ºC:
(b) Minu's brother said to Minu if she
(a) Epitaph (b) Anecdote had finished her assignment. (a) 13.5 ºC (b) 150 ºC
(c) Calligraphy (d) Cenotaph (c) Minu's brother asked her if she (c) 150 K (d) 123 K
12. Study of the origin and history of had finished her assignment. 24. Four point charges q1, q2, q3 and
words (d) Minu's brother asked Minu if she q4 are placed at the corners of
has finished her as signment the square of side a, as shown in
(a) Etymology
17. Fill in the blank with the Past figure.
(b) Epistemology
Perfect Continuous Tense. The The potential at the centre of the
(c) Morphology
(d) Dictionary doctor was very tired as he ______ square is:
alone.
Directions: (13-14): In the following q1 a q2
questions a sentence has been given in (a) has worked
Active/Passive Voice. Out of the four (b) had worked
alternatives suggested, select the one (c) has been working P
a a
which best expresses the same sentence (d) had been working
in Passive/Active Voice. 18. Find out the correct world of the
13. Rosemary was moved to tears at asked grammatical category of the
the sight of the miserable beggar. given word. q4 q3
(a) The sight of the miserable beggar (a) Accentuate (b) Accentual (Given : q1 = 1 × 10–8 C, q2 = –2 ×
moved Rosemary to tears. (c) Accentuated (d) Accentuation 10–8 C, q3 = 3 × 10–8C, q4 = 2 × 10–8
(b) The sight of the miserable beggar Directions (19-20): In the following C, a = 1 m)
has moved Rosemary to tears. questions, some parts of the sentences
(c) The sight of the miserable beggar' have errors and some are correct. Find (a) 507 V (b) 607 V
moves Rosemary to tears. out which part of a sentence has an (c) 550 V (d) 650 V
(d) The sight of the miserable beggar error. The number of that part is the 25. A uniformly wound solenoidal
had moved Rosemary to tears. answer. If there is no error, then your coil of self inductance 2.4 × 10–4
answer is (d) i.e. No error. H and resistance 8 Ω is broken
14. Could you pass the salt?
19. Due to heavy work (a)/ in the office up into two identical coils. These
(a) Could the salt been passed? identical coils are then connected
(b) Could the salt be passed by (b)/I was unable to catch the five
thirty train. (c)/ No error (d) in parallel across a 12 V negligible
anyone?
20. Hari was appointed leader (a)/ of of resistance. The time constant
(c) Could the salt be past?
(d) Could the salt be passed? a group of young social workers for the current in the circuit is:
(b)/that have accepted to help and (a) 3 × 10–5 s (b) 3 × 10–4 s
Directions (15-16): In the following
develop the village in three months. (c) 3 × 10 s–3
(d) 3 × 104 s
questions, a sentence has been given in
Direct/Indirect Speech, Out of the four (c)/ No error (d) 26. A body is projected at an angle θ
alternatives suggested, select the one to the horizontal with momentum
which best expresses the same sentence PHYSICS of magnitude p. The magnitude of
in Indirect/Direct Speech. 21. Find the de-Brogile wavelength of momentum at the topmost point
15. ''Tinu, where have you been all an electron moving with a velocity will be:
these days ?" asked the Principal. 1.5 × 108 ms–1. Given the rest mass (a) p (b) p cos θ
(a) The Principal asked Tinu, “where of electron = 9.1 × 10–31 kg:
he had been all those days”. p
(a) 1.0 Å (b) 1.4 Å (c) (d) p sin θ
(b) The Principal asked Tinu, “where (c) 2.0 Å (d) 2.5 Å cos θ
he has been all those days”. 22. A wave is propagating in a 27. A circular loop of mass m and
(c) The Principal asked Tinu, “where medium of electric dielectric radius r lies on a horizontal table
had he been all those days”.
constant 2 and relative magnetic (xy-plane). A uniform magnetic
(d) The Principal asked to Tinu, permeability 50. The wave field is applied parallel to the
“where was he all those days”.
impedance of such a medium is: x-axis. The current I that should
Practice Set-20 249

flow in the loop so that it just tilts 50ºC on this gas thermometer (a) 62 km (b) 15.5 km
about one point on the table is: scale? (c) 31 km (d) None of these
y (a) 25.5 ºC (b) 49 ºC 36. Two electrons are situated 10 cm
(c) 49.04 ºC (d) 50 ºC apart. The gravitational force of
31. Figure (A) shows two capacitors attraction between them is Fg and
I connected in series and connected electrostatic attraction between
r by a battery. The graph (B) shows them is Fe. The ratio Fg/Fe is of the
the variation of potential as one order of:
x
r moves from left to right on the
(a) 1043 (b) 1036
branch containing the capacitors.
(c) 10 –43
(d) 10–36
Then: 37. A light and heavy body have same
C1 C2 Y kinetic energy, which of these will
mg mg
(a) (b)
πr 2 B 2πrB have higher momentum?
mg πrB (a) Lighter body
(c) (d) E
πrB mg (b) Heavier body
28. The effective resistance between (A) (B)
(c) Both have equal momentum
points P and Q of the electrical (d) Question appears incomplete
(a) C1 = C2 38. An insulating rod of length l
circuit shown in figure is: (b) C1 < C2 carries a charge q distributed
2R 2R (c) C1 > C2 uniformly on it. The rod is
(d) C1 and C2 cannot be compared pivoted at an end and is rotated
2R 32. In a series L-C-R circuit R = 300 at a frequency f about a fixed
Ω, L = 0.9 H, C = 2.0 µF, ω = 1000
r r perpendicular axis. The magnetic
P Q rad/s, the impedance of the circuit
moment of the system is:
is:
2R (a) Zero (b) πqfl2
(a) 1300 Ω (b) 900 Ω
(c) 500 Ω (d) 400 Ω 1 1
2R 2R 33. A projectile is thrown with an (c) πqfl 2 (d) πqfl 2
2 3
2Rr 8R (R + r ) initial velocity of (aiˆ + bjˆ ) ms–1. If 39. If 3 A of current is flowing between
(a) (b) the range of the projectile is twice
R+r 3R + r P and Q in the circuit, then the
the maximum height reached by
5R potential difference between
(c) 2r + 4R (d) + 2r it, then: points P and Q is:
2
(a) a = 2b (b) b = a
29. In Melde’s experiment, string 2 V, 2 Ω
vibrates in two loops when 18 gm (c) b = 2a (d) b = 4a
34. One gram of water, when changes P 3A 4Ω Q
wt tension is applied on it. It will
vibrate in three loops if tension on to steam requires 540 Cal. of heat. (a) 30 V (b) 22 V
it becomes: The increase in internal energy
(c) 20 V (d) 15 V
(a) 8 g wt (b) .24 g wt will be:
40. In Melde’s experiment 6 loops
(c) 36 g wt (d) .72 g wt (a) equal to zero are observed with a tension T. If
30. The value of the property X of a (b) equal to 540 Cal. the tension is made 9 times, the
certain substance is given by: (c) more than 540 Cal. number of loops observed will be:
Xt = X0 + 0.50t + 2.0 × 10–4 t2
(d) less than 540 Cal. (a) 1 (b) 3
Where, t is the temperature
35. A TV tower has a height of 75 m.
in ºC (Celsius) measured on a (c) 2 (d) 4
gas thermometer scale. What What is the maximum distance
upto which this TV transmission 41. The voltage gain of an amplifier
would be the celsius temperature
can be received? (Radius of earth without feedback is 100. If a
defined by the property X which
negative feedback is introduced,
corresponds to a temperature of = 6.4 × 106 m)
250 Practice Set-20

with a feedback fraction β = 0.1, and b both are neither even nor
1 0 1
then the gain of the feedback odd. Then R is:  
amplifier is: (a) Reflexive and symmetric (b) A–1 = 0 1 0
(b) Symmetric and transitive  
(a) 9.09 (b) 10 0 0 1 
(c) Symmetric but neither reflexive
(c) 100.1 (d) 90.0 nor transitive
0 –1 0
42. What will be the unit of time in (d) An equivalence relation  
that system in which the unit of (c) A–1 = 0 0 1
length is ‘metre’ unit of mass is a n + 1 + bn + 1  
49. If arithmetic mean 1
a n + bn 0 0 
kg and unit of force is ‘kg wt’?
between a and b then value of n
1 0 –1 0
(a) s (b) (9.8)2 s will be:  
9.8 (a) 0 (b) 1 (d) A–1 = 1 1 1
(c) – 1 (d) – 2  
(c) 9.8 s (d) 9.8 s 0 1 0 
43. A concave mirror of focal length f 50. If (p + q)th term of any geometric
produces a real image n times the progression is m and (p – q) th 54. The coefficient of x4 in expansion
size of the object. The distance of term is n, then pth term will be: 10
of  x – 3  is:
the object from the mirror is: (a) mn (b) m 2 2
x
n
(a) (n – 1)f (b) (n + 1)f n 405
(c) (d) (mn)3/2 (a)
n +1  n –1  m 256
(c)  f (d)  f
 n   n  51. m men and n women are to be
seated in a row such that two (b) 504
44. If φ is phase difference between 259
women are not sit together, If
current and voltage, the wattles 450
m > n, then number of ways of (c)
component of current is: seating will be: 263
(a) I cos φ (b) I sin φ
m! × ( m + 1)! (d) None of these
(c) I tan φ (d) I cos2 φ (a)
( m – n + 1)!
45. If C is the restoring couple per unit
55. Value of 1 – cos 2 60º is equal
radian twist and I is the moment n! × ( n + 1)! to: tan 45º
(b)
of inertia, the dimensional ( m – n + 1)! (a) cos 30º (b) cos2 30º
I
representation of 2π will be: m ! × ( m + 1)! (c) 2 sin2 30º (d) 2 cos 30º
C (c)
(a) [M0L0T–1] (b) [M0L0T1] ( m + n + 1)! π
56. tan θ – sin  π – θ  cos  – θ  is
2   2 
(c) [M0L1T–1] (d) [M1L2T–2] (d) m! n! equal to:
52. T h e c o m m o n r o o t o f t h e (a) tan θ sin2 θ (b) tan2 θ
MATHEMATICS equations
(c) cos2 θ (d) cot2 θ
46. If A = {a}, which of the following x2 – 5x + 6 = 0 and x2 – 6x + 8 = 0
57. The equation of the circle having
is true? is:
(a) 3 (b) 2 centre (1, – 2) and 1 passing
(a) A = a (b) I ⊂ A
(c) 4 (d) 5 through the point of intersection
(c) {a} ∈ A (d) a ∈ A
0 0 1 of lines 3x + y = 14, and 2x + 5y
47. If l, ω and ω2 are the cube roots
53. If A =  1 0 0  , then: = 18 is:
of unity, then (1 + ω – ω2) (l – ω  0 1 0 
+ ω2) is equal to: (a) x2 + y2 – 2x – 4y – 20 = 0
(a) 2 (b) – 2 0 1 1 (b) x2 + y2 – 2x + 4y – 20 = 0
 
(c) 4 (d) – 4 (a) A–1 = 0 0 1 (c) x2 + y2 + 2x – 4y – 20 = 0
48. Let R be a relation in the set of  
integers I, defined by aRb, iff a 1 0 0  (d) x2 + y2 + 2x + 4y – 20 = 0
Practice Set-20 251

8 1
58. If sin θ = and 90º < θ < 180º, (a)
3
(b) REASONING AND GENERAL
17
2 2 AWARENESS (RAGA)
then the value of the expression
1 π 71. Who built the Buddhist
2 sin θ + cos θ (c) (d)
is: 2 4 monuments at Sanchi?
3 cos θ + 5 sin θ
66. Discuss the continuity of the (a) Mughal dynasty
(a) (b) function (b) Maurya dynasty
 – x2 for x ≤ 0 (c) Gupta dynasty
(c) (d) 
5 x – 4 for 0 < x ≤ 1 (d) Chola dynasty
f ( x) = 
for 1 < x < 2 72. Which of the following
59. If in any triangle r10 = r2 + r3 + r,  4 x2 – 3
 for x > 2 is a primary function of
then triangle will be: 3 x + 4 haemoglobin?
(a) Isosceles (b) Equilateral (a) Continuous at x = 1, 2 and (a) Utilisation of energy
(c) Right angled (d) Acute angled discontinuous at x = 0 (b) Prevention of anaemia
60. A boy spends the following
(b) C ontinuous at x = 0, 1 and (c) Destruction of bacteria
amounts from his pocket money
discontinuous at x = 2 (d) To transport oxygen
during five consective months:
(c) Continuous at x = 0, 1, 2 73. Which of these is not an
Month 1 2 3 4 5 (d) None of these Operating System?
Amount ( in Rs. ) 10 20 12 8 15 67. The probability that an event (a) Android (b) iOS
The arithmetic mean of his happens in one trial is 0·6 If 3 (c) HTML (d) Symbian
expenditure in a month is: independent trials are made then 74. Which of the following is a Direct
(a) ` 12·00 (b) ` 13·00 the probability that the envent tax ?
(c) ` 14·00 (d) ` 15 may happen once, is: (a) Excise duty
61. If (2, 0) is the vertex and y-axis is (a) 0·936 (b) 0·064 (b) Customs duty
the directrix of a parabola, then (c) 0·288 (d) 0·784 (c) Service tax
its focus is: dy (d) Wealth tax
68. If y = xlog x, then equals:
(a) (1, 0) (b) (3, 0) dx 75. The boundary line between India
(c) (4, 0) (d) (– 2, 0) (a) log x . x log x – 1 and Pakistan is called
62. If the normal to the rectangular (b) x (log x – 1) .2 log x (a) McMahon line
hyperbola xy = c2 at the point (c) x log (log x) (b) Maginot line
t meets the curve again at the 1 (c) Radcliffe line
(d) log x . xlog x – 1
point t1, then: x (d) Sir Creek
(a) t3t1 = 1 (b) t3t1 = –1 π /2 1 76. Who among the following is
(c) tt13 = 1 (d) tt13 = – 1 69. The value of ∫0 1 + tan x
dx
the most important political
63. W h a t i s t h e d e g re e o f t h e is:
institution in a democratic
differential equation whose π π country?
(a) (b)
solution is (ex – 1)3 = c than y? 2 4 (a) President
(a) 2 (b) 1 π (b) Parliament
(c) 3 (d) None of these (c) (d) 1 (c) Prime Minister
6
64. If the distance between the (d) Cabinet Ministers
70. The condition that straight line
points (α, 2) and (3, 4) be 8, the 77. From which year, the 35th Goods
x cos α + y sin α = p touches the
value of a will be: and Service Tax (GST) Council
2 2
(a) 4 (b) 3 ± 15 ellipse x + y = 1, will be: decided to introduce electronic
2 2
a b invoicing system in a phase-wise
(c) 3 ± 60 (d) 6 (a) p2 = a2 cos2 α + b2 sin2 α manner for B2B (Business to
(b) p2 = a2 sin2 α + b2 cos2 α Business) transactions?
65. cos  tan –1 1 + tan –1 1  is equal (c) p2 = a sin α + b cos α (a) 2025 (b) 2020
3 2
to:  (d) p = a cos α + b sin α (c) 2030 (d) 2035
252 Practice Set-20

78. If 7/8th of 5/4th of a number is the son of 'X'. How is 'Z' related (a) F
315, then 5/9th of that number is to 'X'? (b) C
________ (a) Niece (c) D
(a) 123 (b) 81 (b) Nephew (d) None of the above
(c) 140 (d) 160 (c) Data inadequate Direction (92): In following questions
79. If cosAcotA/(1 – sinA) = x, then (d) Daughter a stands for equal to’; b for ‘greater
the value of x is 87. In a certain code language, than’; γ for ‘less than’ and d for ‘not
(a) 1 – cosecA (b) 1 + cosecA “NATURE” is written as equal to.
(c) 1 + secA (d) 1– secA “TANURE”. How is “LISTEN”
92. If abxy a c2z, bx b ay and a2 g bc,
written in that code language?
then
( 1) + ( 2 ) + ( 3 ) + ( 4 )
2 2 2 2
80. =? (a) NETSIL (b) TENSIL
(a) ax2 b cz (b) a2x2 b cz
(c) SILTEN (d) LITSEN
(a) 6 (b) 8 (c) b2 x b c2z (d) bx2 b c2z
88. Kamal of facing South. He
(c) 10 (d) 9 turns 135° in the anti-clockwise Direction (93): Select the odd word/
81. Of the 3 numbers whose average direction and then 18° in the letters/number/number pair from the
is 64, the first number is 1/3 clockwise direction. What given alternatives.
times the sum of other 2. The direction is he facing now? 93. (a) 81 (b) 125
first number is— (a) North (b) South-West
(c) 64 (d) 198
(a) 72 (b) 32 (c) East (d) South-West
(c) 96 (d) 48 94. Arrange the given words in the
89. The age of Amit is same as Sumit sequence in which they occur in
82. When a number is increased by because they are twins, Richa the dictionary.
34, it becomes 117% of itself. is younger than Sumit, Richa is
What is the number ? younger than Jyotsana but elder i. Monopoly
(a) 408 (b) 200 than Saurabh, Sumit is younger ii. Manifesto
than Jyotsana. Who is elder than iii. Manifold
(c) 510 (d) 306 Mohan. Who is the eldest?
83. 30% discount is offered on iv. Mankind.
(a) Amit (b) Jyotsana
an item. By applying a promo (c) Richa (d) Saurabh (a) ii, iii, iv, i
code the customer wins 10% (b) iii, ii, iv, i
90. In the following questions
cash back. What is the effective
which one set of letters when (c) i, iii, iv, ii
discount ?
sequentially placed at the gaps
(a) 40.7 percent (b) 40 percent (d) i, iv, iii, ii
in the given letter series shall
complete it? 95. In the following figure, rectangle
(c) 37 percent (d) 13 percent
represents fashion designers,
84. The third proportional of two _d_be_c_ed_b
circle represents equestrians,
numbers 9 and 15 is : (a) edbcb (b) cdebc triangle represents Campers
(c) ecdbc (d) eedcb and square represents Golfers.
(a) 21 (b) 30
Direction (91): Study the Following Which set of letter represents
(c) 25 (d) 45 Equestrians who are not fashion
information carefully and answer the
85. If the amount received at questions given below: designers?
the end of 2nd and 3rd year
at Compound Interest on a A, B, C, D, E, F, G and H are sitting
around a circle facing the centre. B is I
certain Principal is ` 25088, and H
` 28098.56 respectively, what is second to the right of H and third to
the left of A. D is not an immediate A B E
the rate of interest? G
neighbour of either B or H and is C
D
(a) 6 percent (b) 24 percent second to the right of F. C is fourth to
(c) 12 percent (d) 19 percent the right of G. F
86. 'X' is the father of 'Y' and 'Z'. 91. If E and F interchange their
(a) EG (b) DEG
'Y' is the son of 'X' but 'Z' is not places, who will be second to the
right of B ? (c) I (d) E
Practice Set-20 253

Directions (96-100): Read the following (a) Mala (b) Kapil


(a) Mala (b) Gopal
information and answer the questions (c) Alka (d) Gopal (c) Ram (d) Alka
given below it: 97. Whose age is between Mala and 99. Who is eldest?
Alka is older than Mala. Gopal is oder Kapil? (a) Alka (b) Mala
than Mala but younger than Alka.
(a) Gopal (b) Ram (c) Kapil (d) Gopal
Kapil is younger than Ram and Mala.
(b) Alka (d) Mala 100. Who is the youngest?
Mala is older than Ram.
98. Whose age is exactly in the (a) Mala (b) Ram
96. Whose age is between Gopal and
middle of all the five?
Ram? (c) Akla (d) Kapil

Answers with Explanation


ENGLISH 11. (a) Epitaph 1
12. (a) Etymology ∴ T2 = × 300 = 150 K
13. (a) The sight of the miserable 2
1. (b) 2. (d) 3. (a) 4. (c)
beggar moved Rosemary to tears. 24. (a) As the distance of each charge
5. (a) The correct spellings are: 14. (d) Could the salt be passed? from P is the same and let it to be r.
leadership; scholarship; partnership. 15. (a) The Principal asked Tinu
6. (d) The word Sacred (Adjective), “where he had been all those days”. 1  q1 q2 q3 q4 
V = ∑ Va = + + + 
means: holy; venerable; dedicated; 16. (c) Minu's brother asked her if she 4πε 0  r r r r 
sacrosanct; very important and treated had finished her assignment.
with great respect. 17. (d) Use had been working for the 9 × 109
The word Profane (Adjective) blank.    = [(1 – 2 + 3 + 2) × 10 –8 ]
means: unhallowed; impious; having 0.71
18. (b) It is 'Accentual.'
or showing a lack of respect for god 19. (a) Here, Due to a heavy workload V = 507 V
or holy things. .... should be used. L 2.4 × 10 –4
Look at the sentences: Look at the sentence: 25. (a) τ0 = =
R 8
Cows are sacred to Hindus. We have taken on extra staff to cope
We should discourage profane attitudes. with the increased workload. ⇒ τ0 = 3 × 10–5 s
7. (d) The word Concurrence 20. (c) According to 'a group of' that When broken upto into two identical
(Noun) means: agreement; an example has/had accepted ..... should be used. pieces and connected in parallel, then
of two or more things happening at the PHYSICS Lp L/4 L
same time. = =
h m0 Rp R/4 R
Look at the sentence: 21. (b) λ= and m =
mv v2 ⇒ τ = τ0 = 3 × 10–5 s
T h e d o c t o r m a y s e e k t h e 1–
c2 26. (b) p = mv
concurrence of a relative before carrying
Substituting values of various At topmost point v' = v cos θ.
out the procedure.
quantities and solving So, p' = mv'
8. (d) The word Espionage (Noun)
means: spying; the activity of secretly λ = 1.4 Å. = mv cos θ
getting important political or military μr μ0 = p cos θ
22. (c) Z= ×
information about other country, etc. εr ε0 27. (c) Torque experienced by the
Look at the sentence: coil is
Some of the commercial activities 50 τ = BI (πr2)
⇒ × 376.6 Ω
Z=
were a cover for espionage. 2 This torque will be balanced by
9. (c) Idiom come to grief means: to = 1883 Ω the torque produced by weight of coil
end in total failure; to be harmed in an E2 (acting from the centre)
accident; suffer. 23. (c) =
E1 ⇒ BI (πr2) = (mg)r
10. (b) Idiom through thick and thin
I = mg
means: even when there are problems or 1 T2
or, = ⇒
difficulties; under all circumstances. 2 273 + 27 πrB
254 Practice Set-20

28. (a) 32. (c) Impedance of circuit is 38. (d) Consider an element dx on the
2R 2R rod at a distance x from the axis.
Z = R 2 + (X L – X C ) 2
2
 106 
r r = (300) 2 + 1000 × 0.9 –  dx
P Q  1000 × 2 
  = 500 Ω.
33. (c) = aiˆ + bjˆ x
2R 2R and   R = 2H

= 2Rr Since u = (u cos θ)iˆ + (u sin θ) ˆj
R+r ⇒ u cos θ = ux = a The value of charge
29. (a) In Melde’s experiment and u sin θ = uy = b q
dq = dx
p2 × Tp = (p)2 × Tp' Also, we have, l
22 × 18 = 32 Tp' 2  u 2y  q
(u x )(u y ) = 2   ∴ dM = IA = × f × πx 2 dx
g  2g  l
T p' = ⇒ 2ab = b2 M qfπ l 2
∫ l ∫0
∴ dM = x dx
= 8 g wt ⇒ b = 2a 0

30. (c) The temperature on the scale 34. (d) Here the heat supplied (540 1
qfπ  x 3 
cal) is used in increasing the internal        = × 
using thermal property X, is related to l  3 0
gas centigrade scale (measured by gas energy and also in doing external work
thermometer) through the relation. because water changes to steam which qfπ l 3
= ×
occupies much larger volume. Hence, l 3
Xt – X0
t= × 100 increase in internal energy will be less qfπl 2
X100 – X 0 =
than 540 cal. Alternately ∆U = Q – W = 3
From given relation 540 – W. Here W is positive since work Hence, magnetic moment of the
X50 – X0 = 0.5 × 50 + 2 × 10–4 is done by the system, therefore, ∆U < π 2
× 50 × 50 540 cal. system = qfl
3
= 25 + 0.5 = 25.5 35. (c) d= 2rh 39. (c) V = 2 + Ir + IR = 2 + 3 × 2 +
and X100 – X0 = 0.5 × 100 + 2 3×4
  = 2 × 6.4 × 106 × 75
× 10–4 × 100 × 100 V = 20 V
   = 3.1 × 104
= 50 + 2 = 52 p 9T = 6 T
   = 31 km. 40. (c)
25.5 Gmm ∴    p = 2
∴ t= × 100 36. (c) Fg =
52 r2
q.q A
2550 and Fe = 9 × 109 . 2 41. (a) At =
= r 1 + βA
52
= 49.04 ºC Fg Gm 2 100
∴ = =
31. (b) When we move from left Fe 9 × 109 q 2 1 + 0.1 × 100
to right the voltage increases and this 100
6.67 × 10 –11 (9.1× 10 –31 ) 2
increase is just within the capacitors and  =    = = 9.09
9 × 109 (1.6 × 10 –19 ) 2 11
remains constant in the conducting wires.
Also for series combination  = 10–43  L 2   T2 
a b

1 37. (b) Since, p = 2mE, 42. (a) n1 = n2    


V∝ and according to graph  L1   T1 
C now kinetic energy is same.
Therefore, particle of heavier mass will Force = [MLT–2] and n2 = kg. wt
V 2 > V1
⇒ C1 < C2 have higher momentum. In SI system, Force = newton
Practice Set-20 255

1 newton 1 kg ×1 m × (1s) –2 a 0 + 1 + b0 + 1 0 0 1
= =  
1 kg wt 1 kg × 1 m × (T1 ) –2 a 0 + b0 ∴ |A| = 1 0 0
1 Comparing both the sides, n = 0.  
or, = T12 0 1 0 
9.8 50. (a) According to question,
1 m = ar p + q – 1      A11 = 0, A12 = 0, A13 = 1, A21

∴ T1 = s   = – (0 – 1) = 1.
9.8 n = ar p – q – 1
On multiplying, mn = a2r2 p – 2 A22 = 0, A23 = 0, A31
f
43. (c) mreal = –n = = – (0 – 1) = 1 and A33 = 0
f –u 1
⇒ –nf + nu = f ( mn ) 2 = ar p–1
0 1 0
 n +1  
⇒ u=  f ( mn ) ∴ Adj. A = 0 0 1
 n  ∴ a= p –1  
r
44. (b) Iv sin φ component of current 1 0 0 
gives on power consumption and hence p term = ar p – 1
th

is called wattless current. 0 1 0


( mn ) p –1 1  
     = .r = mn 0 0 1
I ML2 r p –1 ∴ A–1 =
45. (b) 2π = 2π A 
C MLT –2 × L 51. (a) The place of one man keep 1 0 0 
= [M0L0T1] fixed out of m men, rest (m – 1) men can
be sit down in (m – 1) ! ways with respect 0 −1 0
to him. Although, on fixed piace any man  
MATHEMATICS out of m men can be sit down by m ways. = 0 0 1
 
46. (d) If A = {a}, then a ∈ A. On sitting m men, (m – 1) place in 1 0 0 
47. (c) (1 – ω – ω2) (l – ω + ω2) between 2 men and 2 place at beginning
= (– ω2 – ω2) (– ω – ω) and in last, so total (m – 1 + 2) = m + 1 54. (a) (r + 1)th term of expension
[Because 1 + ω + ω2 = 0, so 1 + ω place are left on which n women can be x
10 – r
 3 
r
10
= – ω2 and 1 + ω2 = – ω] sit down by m + 1Pn ways.   = Cr ×   ×– 2 
2  x 
= – 2ω2 × – 2ω = 4ω3 Therefore, required numbers of
3r
= ( – 1) r × C r × × x10 − 3r
seating arrangement for m men and n 10
=4
women will be. 2 (10 – r )
48. (c) aRb ⇔ Both a and b are
neither even nor add. Let a be odd. then = m × (m – 1)! × m + 1Pn If x4 comes in this term. 10 – 3r =
a/o(/.R) a and let a be even. then a Ra. 4 or r = 2
Hence, R is not reflexive.
( m + 1)!
= m! × ∴ Required coefficient of x4
Let a and b be odd and even, then
( m + 1 – n )!
32
= ( – 1) × 10 C r ×
2
aRb ⇒ bRa m! × ( m + 1)!
Hence, R is symmetric. = 28
( m – n + 1)!
Let a, b be odd and even 10 × 9 9 405
respectively, then if R is transitive, then 52. (b) Roots of x2 – 5x + 6 = 0 are 2 = × =
1 × 2 256 256
aRb, bRa ⇒ aRa and 3
but, aRa is not true as a is odd, and roots of x2 – 6x + 8 = 0 are 2
1 – cos 2 30º
hence R is not transitive. and 4 55. (c)
∴ Common root is '2'. tan 45º
a n + 1 + bn + 1 a + b
49. (a) = 0 0 1
a n + bn 2  3
2
  1 – 
53. (c) A = 1 0 0 
a1 + b1    2 
= =
1+1 0 1 0  1
256 Practice Set-20

3 16 15 1 65
1– 1 – = = ` 13·00
= 4 = 2 17 17 17 5
    = = 5
1 45 40
– + – 61. (c) ∵ (2, 0) is the vertex and
2 17 17 17 y-axis is the directrix of the parabola.
1
and 2 sin2 30º = 2 ×   ∴ Axis of the parabola will be
2 1
    = – x-axis.
5
= Let the coordinates of its focus be
sec 45º + 2 tan 135º
2
(x, 0)
1 – cos 2 30º
( 2) ∴ The length of perpendicular
2
∴ = 2 sin2 30º = – 2 ( –1)
tan 45º from (2, 0) on the directrix = 2 and the
59. (c) Given r1 = r2 + r3 + r or r1 – r distance from (2, 0) to (x, 0) = (x – 2).
π
56. (a) tan θ – sin  – θ  cos = r2 + r3 ∴ x – 2 = 2 or x = 4
2 
π  62. (b) The normal at the point
 – θ or, 1 1 1
2  4R sin A cos B cos C  c
2 2 2  ct ,  to the rectangular hyperbola xy
= tan θ – cos θ sin θ 1 1 1  t
– 4R sin A sin B sin C = c2 is–
sin θ cos θ sin θ 2 2 2
= – t3x – yt – cr4 + c = 0
cos θ 1 1 1 1
= 4R sin A cos B cos C If it passes through the point t1
2
sin θ − cos θ sin θ 2 2 2
=  
cos θ 1 1 1 i.e.,  ct1 , c  then,
+ 4R sin C cos A cos B  t1 
sin θ (1 – cos 2 θ) 2 2 2
= ct
cos θ 1 1 1 1 1 t 3ct1 – – ct 4 + c = 0
or, sin A  cos B cos C – sin B sin C  t1
sin θ sin 2 θ 2  2 2 2 2 
= or, t 3t12 – t – t 4t1 + t1 = 0
cos θ 1  1 1 1 1 
= cos 2 A  sin 2 B cos 2 C + sin 2 C cos 2 B or, t 3t (t – t ) + (t – t ) = 0
= tan θ sin2 θ 1 1 1
57. (b) On solving 3x + y = 14 and
2x + 5y = 18, we get x = 4 and y = 2 or, sin 1 A cos 1 ( B + C ) or, (t1 – t ) + (t 3t1 + 1) =0
2 2
∴ Radius of the circle Since, t1 ≠ t, therefore = – 1
1 1 63. (b) (ex – 1)3 = c tan y
= (1 – 4 )2 + ( – 2 – 2 )2 =5 = cos A sin ( B + C )
2 2 dy
3 (ex – 1)2 ex = c sec2 y
∴ Equation of the circle is: 1  1  dx
or, sin A cos  90º – A 
(x – 1)2 + (y + 2)2 = 25 2 2 
(e )
3
 x
–1
∴ x2 + y2 – 2x + 4y – 20 = 0 From eqn. (1) c =
1  1  tan y
8 cos A sin  90º – A 
58. (d) sin θ = and 90º < θ < 180º 2  2 
17 Putting the value of c in eqn. (2),
1 we get
15 or sin A = cos2 1 A
2

(e )
3
∴ cos θ = – 2 2 x
–1
17 dy
3 (ex – 1)2 e =x sec 2 y
1 tan y dx
2 sin θ + cos θ ⇒ A = 45º
∴ 2
dy
3 cos θ + 5 sin θ
A = 90º x
( 2
or, 3ex tan y = e – 1 sec y ) dx
60. (b) A.M. of his expenditure for a
8 15
2× – month dy
17 17 or, 3ex tan y + (1 – ex) sec2 y =0
= 15 8 10 + 20 + 12 + 8 + 15 dx
3× – + 5× =
17 17 5 and the degree of this equation is l.
Practice Set-20 257

L. lim x → 2 f (x) 70. (a) Equation of line is x cos α +


64. (c) 8= ( α – 3)2 + ( 2 – 4 )2 ≠ R. lim x → 2 f (x) y sin α = p
or, 64 = α2 – 6α + 9 + 4 ∴ At x = 2, f (x) is discontinuous. or, y sin α = p – x cos α
or, α2 – 6α – 51 = 0 67. (c) The probability that event or, y = (– cot α)x + p cosec α
happens in one trial Comparing it form y = mx + c, m
6 ± 36 + 4 × 1 × 51 = 0.6 = – cot α and c = p cosec α
or, α =
2 ∴ The probability that event will If line y = mx + c touches the ellipse
not happen in one trial
6 ± 36 + 204
= = 1 – 0.6 = 0.4 = 1,
2
Since, the event may happen in
c2 = a2m2 + b2
= 3 ± 60 first, second or third trial.
∴ (p cosec α)2 = α2 (– cos α)2 + b2
∴ Required probability
1 1 or, p2 cosec2 α = α2 cot2 α + b2
65. (b) tan –1 + tan –1 = 3C1 × 0.6 × (0.4)2 = 0.288
3 2
68. (b) y = xlogx
 1 1  Taking logarithms of both sides, we or, p2 =
+
–1  3 2  get
= tan  1 1
1 – ×  log y = log x. log x = a2 cos2 α + b2 sin2 α
 3 2  Diff. w.r.t.x, we get
REASONING AND GENERAL
π 1 dy 1 1
= tan–1 (1) = = log x + log x AWARENESS (RAGA)
4 y dx x x
71. (b) The 'Great Stupa' at Sanchi
dy 2 y log x
 1 1 π 1 = is the oldest structure and was originally
cos  tan –1 + tan –1  = cos = dx x commissioned by the emperor Ashoka
 3 2 4 2
2 x log x . log x the Great in the 3rd century BCE. Ashoka
66. (d) L. lim x → 0 f (x) = belonged to the Mauryan Empire and
x
= lim h → 0 – (0 – h)2 most of the Buddhist monuments were
(log x −1)
= lim h → 0 – h2 = 0 =2 x . 2 log x build during this period only.
R. lim x → 0 f (x) = lim h → 0 5(0 + h) – 4 π/ 2 1 72. (d) The major function of
=–4
69. (b) I= ∫0 1 + tan x
dx haemoglobin is to transport oxygen
from the lungs to the body's tissues and
L. lim x → 0 f (x) ≠ R. lim x → 0 f (x) then transport carbon dioxide out of the
∴ At x = 0, f (x) is not continuous. π/ 2 cos x tissue back to the lungs.
= ∫0 cos x + sin x
dx
73. (c) Operating System is the low-
L. lim x → 1 f (x)
= lim h → 0 5 (1 – h) – 4 = 1 level software that supports a computer's
π  basic functions, such as scheduling
cos  – x 
R. lim x → 1 f (x) π/ 2 2  tasks and controlling peripherals.
and I = ∫0 dx
= lim h → 0 4 (1 + h)2 – 3 = 1 π  π 
cos  – x  + sin  – x  Hypertext Markup Language (HTML),
2  2  a standardised system for tagging text
and f (1) = 5 (1) – 4 = 1
L. lim x → 1 f (x) = R. lim x → 1 f (x) files to achieve font, colour, graphic,
π/ 2 sin x and hyperlink effects on World Wide
= f (1) = 1 = ∫0 sin x + cos x
dx
Web pages. All others are operating
∴ At x = 1, f (x) is continuous. systems.
L. lim x → 2 f (x) π/ 2 cos x + sin x 74. (d) A tax, such as income tax,
= lim h → 0 4(2 – h)2 – 3
∴ 2I = ∫0 cos x + sin x
dx
which is levied on the income or profits
of the person who pays it, rather than on
= 13 π/ 2
dx = ( x )0
π/ 2 goods or services is called as direct tax.
R. lim x → 2 f (x)
= ∫0 = π/2
Key examples of direct taxes are Income
= lim h → 0 3(2 + h) + 4 π tax, Wealth tax, Corporation tax.
= 10 ∴ I= 75. (c) The Radcliffe Line was
4
published on August 17, 1947 as a
258 Practice Set-20

boundary demarcation line between 5 5 ∴ Applying promo the price of item


India and Pakistan upon the Partition of Now, of 288 = × 288 = 160.
9 9 70 × 10
India. It was named after its architect, = =7
Sir Cyril Radcliffe, who as chairman cosA cotA 100
79. (b) x =
of the Border Commissions. McMahon (1 – sinA) Finally the effective discount
line is between India and China. Sir = 30 + 7 = 37%.
cosA
Creek is a 96 km tidal estuary on the cosA. 84. (c) Let third proportional be x
  = sinA
border of India and Pakistan. The Creek, 9 : 15 : : 15 : x
(1 − sin A)
which opens up into the Arabian Sea, 9 15
divides the Gujarat State of India from cos 2 A =
  = 15 x
the Sindh province of Pakistan. The sinA (1 − sin A)
9x = 15 × 15
Maginot Line named after the French 15 × 15
1 − sin 2 A
Minister of War Andre Maginot, was a   = x = = 5 × 5 = 25
sinA (1 – sinA) 9
line of concrete fortifications, obstacles,
(1 − sin A)(1 + sin A) Hence, third proportional is 25.
and weapon installations built by
  = 85. (c) Let the rate be r, then
France in the 1930s to deter invasion sin A. (1 − sin A)
by Germany. Constructed on the Fench  r 
n
1 + sin A 1 A = P 1 +
+1
side of its borders with Switzerland,   =
sin A
=
sin A  100 
Germany, and Luxembourg.
  = 1 + cosecA = R.H.S. 2
76. (c) The Prime Minister is the r 
80. (c) ? = ( 1 ) + ( 2 ) + ( 3 ) + ( 4 ) 25088 = P  1 + ...(i)
 100 
2 2 2 2
most important political institution in
any democratic country. The Prime = (1 + 2 + 3 + 4)
3
Minister is the leader of the country who = 10  r 
28098.56 = P  1 + ...(ii)
represents the entire population of the 81. (d) Let three numbers  100 
country. Controlling the other economic be x1, x2 and x3
variables, the main findings indicate that Total sum of three numbers Dividing eq. (ii) by (i)
the political institutions fundamentally x1 + x2 + x3 = 64 × 3 = 192 ...(i) 28098.56  r 
matter only for incipient democracies, = 1 +
1 25088  100 
and not for consolidated democracies. x1 = ( x 2 + x 3 )
3
Political institutions demonstrate 3x1 = x2 + x3 ...(ii) 28098.56 – 25088 r
that consolidated democracies and =
On putting, x2 + x3 = 3x1 in (i) 25088 100
political institutions are substituted for
x1 + 3x1 = 192 r 3010.56
determining economic growth. =
77. (b) The Council decided to 4x1 = 192 100 25088
introduce electronic invoicing system in 192 201056
a phase-wise manner for B2B (Business x1 = = 48 r= = 12 percent
4 25088
to Business) transactions from 1 January 82. (b) Let the number be x 86. (d) The Blood relationship tree/
2020. Using this system, no separate
117 chart can be constructed as given below:
e-way bill will be required in case of x + 34 = of x
e-invoice. It will help to combat the 100
menace of tax evasion. 117
⇒ x – x = 34
78. (d) Let the number be x 100
17 x
7 5 ⇒ = 34
× of x = 315 100
8 4
3400
35 ⇒        
x= = 200 X ⇒ Y's & Z's father.
x = 315 17
32 Y ⇒ X's son
83. (c) On 30% discount the price
315 × 32 [ Z' is not the son of 'X'.]
x= = 288 of item,
35 = 100 – 30 = 70 Therefore, 'Z' is the daughter of 'X'.
Practice Set-20 259

87. (c) As, N T


Finally, the set of letters 'ecdbc' will 64 = (4)3
complete the series. 198 = (14)2 + 2
A A
91. (d) None of these. According Finally the odd number is 198.
T N
to the question, E & F interchange their 94. (a) According to dictionary, the
U U places.
R
sequence of words is,
R
E E So, the new circular arrangement is (ii) Manifesto (iii) Manifold
Similarly, as follows. (iv) Mankind (i) Monopoly
L S 95. (c) Fashion designers
I I = {H, D, E, G, F}
L Equestrians = {E, G, I}
S
Campers = {B,C,D,E}
T T
E Golfers = {A,B}
E Therefore, second to the right of
N Hence ‘I’ letter represents equestrians
N B  E.
who are not fashion designers.
Finally, “LISTEN” written as 92. (a) According to the given
“SILTEN” in code language. information 96. (a) From (vi), we get
88. (b) According to the question, Gopal Ram
abxy a c2z  abxy = c2z ...(i) Between and ⇒
the direction diagram will be as follows Mala 2 2
bx b ay  bx > ay ...(ii)
3
and b a ac  b = ac...(iii)
2 2

From (ii), we get 97. (b) From (vi), we get


Kapil Ram
b2x > bay Between Mala and ⇒
 acx > bay [∴ b2 = ac from (iii)] 3 5 4
 cx > by ...(iv)
98. (a) From (vi), we get
Clearly, Kamal is facing South-West. From (i), we get Mala
89. (b) Amit = Sumit, Sumit > Richa Exactly in the middle ⇒
c 2z 3
Jyotsana > Richa > Saurabh by =
ax
Jyotsana > Sumit 99. (a) From (vi), we get
Putting (v) in (iv), we get
On arranging the above data, we get Alka
∴ Jyotsana > Sumit = Amit > Richa c 2z Eldest ⇒
cx = 1
> Saurabh. ax
So, Jyotsana is eldest among them.  ax2 > cz 100. (d) From (vi), we get
90. (c) The series is _d_be_c_ed_b  ax2 b cz. Kapil
youngest ⇒
The series becomes 93. (d) 81 = (9)2 5
edcbedcbedcb 125 = (5)3 qqq
260 Practice Set-21

Air Force Airmen Group X & Y Exam


Practice Set-21

new foreign language and seem relatively (a) for, to (b) in, for
ENGLISH simple once one has a hang of them. (c) to, for (d) on, to
Directions (1-4): Read the passage 1. Identify the word opposite in meaning 9. Choose the appropriate synonym
carefully and answer the questions that to the word "Cumbersome". of the word given below.
follow by choosing the best alternative. (a) awkward (b) bulky Opulent
What's the computer's mother tongue? (c) manageable(d) unwiedly (a) Greedy (b) Hungry
Which language does the computer 2. Identify the word closest in (c) Heavy (d) Wealthy
speak? What is a conglomeration of meaning to the word 'Precise'. 10. Change the following into indirect
computer languages called? Witticisms (a) defined (b) permissive speech by choosing correct
apart, we do need to communicate with (c) mild (d) diffused alternative.
computers. When the pentagon wanted 3. Why have European militaries Our teacher said to us, "sugar
a new computer language that could adopted ADA? dissolves in water."
do most of the routine jobs with ease, (a) Is easy and simple to use. (a) Our teacher told us that sugar
it made its scientists look at more than (b) They are under agreement with dissolved in water.
150 different computer languages that the US. (b) Our teacher told us that sugar has
existed in the United States. After a few (c) Can work on many parameters at dissolved in water.
months of intensive testing of most the a time. (c) Our teacher said to us that sugar
available languages, American scientists (d) All of the above. dissolves in water.
decided that none of them was suitable 4. Why did the DoD find it difficult (d) Our teacher told us that sugar
for the job. So, the US Department of to choose a name for their new dissolves in water.
Defense invented yet another language, computer language? 11. Choose the word similar in
making it about the 151. It then went (a) No name seemed precise. meaning.
about searching for a name the DoD (b) Every name had been already Industrious
didn't find anything "precise enough". used. (a) Successful (b) Sensible
So, a new female name was picked and (c) They did not have any repository (c) Punctual (d) Diligent
the newest computer language was called Directions (12-13): In the following
of the available names.
ADA. Lady Augusta Ada Lovelace could questions, out of four alternatives,
(d) They were unable to get approval
be considered the chief inspiration for choose the one which can be substituted
of the president for the chosen
the world's first computer engineer, the for the given words/sentence.
names.
father of the computer, Charles Stanley 12. A government by officials
Directions (5-6): Choose the correctly
Babbage. ADA, the language is quite (a) Oligarchy (b) Aristocracy
spelt word:
tough, complex and highly detailed (c) Plutocracy (d) Bureaucracy
5. (a) Souvenir (b) Suvenire 13. One who walks in sleep
piece of software and is being adopted (c) Suvenir (d) Souvenire
to much of the Western World's military (a) Somniloquist
6. (a) Quintessence (b) Egoist
and space applications. ADA can work (b) Qintessence (c) Somnambullist
with multiple parameters at one time, is (c) Quentesence (d) Altruist
quick efficient and precise. (d) Quintesence Directions (14-15): In the following
But the non-military world does 7. Choose the opposite of the word questions, four alternatives are
not exactly need ADA to carry on its given below. given for the idiom/phrase that is
business. There are about a dozen other Masticate underlined. Choose the alternatives
more popular languages that fit the (a) Conceal (b) Chew which best expresses the meaning of
bill very well. Though, almost all of (c) Review (d) Gobble the underlined idiom/phrase.
them seem quite tough, complex and 8. Fill in the blanks with the correct 14. The passing of anti-defection law
cumbersome to work on and write at first preposition. She has been living struck a chill to the heart of every
sight, they are not tougher than learning a ........this city ......... ten years. opportunistic legislator.
Practice Set-21 261

(a) caused anger PHYSICS T


(b) caused relief 2
21. If we throw a body upwards with
(c) aroused relief velocity of 4 ms–1, at what height (ii)
(d) awakened bitterness does its kinetic energy reduce to
1
15. Our house is within a stone's throw half of the initial value? (Take, g p
from the Red Building. = 10 ms–2)
T
(a) far off (b) far away (a) 4 m (b) 2 m
(c) very near to (d) beside (c) 1 m (d) 0.4 m
22. T h e m o m e n t u m o f a b o d y (iii)
16. In the following question, a 2 1
increased by 20%. The
sentence has been given in Active/ percentage increase in kinetic p
Passive Voice. Out of the four energy is:
alternatives suggested, select the T
(a) 54 (b) 44
one which best expresses the same (c) 100 (d) 50
sentence in Passive /Active Voice. 23. A solid sphere is rotating in free (iv)
1 2
Rain disrupted the last day's play space. If the radius of the sphere
between India and Sri Lanka. is increased keeping mass same, p
(a) The last day's play of India and which one of the following will (a) (iv) (b) (ii)
not be affected?
Sri Lanka were disrupted by rain. (c) (iii) (d) (i)
(a) Moment of inertia
(b) India's and Sri Lanka's play of the 26. The displacement of a particle
(b) Angular momentum
last day was disrupted by rain. varies with time according to the
(c) Angular velocity
(c) The last day's play between India relation y = a sin ωt + b cos ωt
(d) Rotational kinetic energy
and Sri Lanka was disrupted by (a) The motion is oscillatory but
24. A force F is applied on the wire
rain. not SHM
of radus r and length L and
(b) The motion is SHM with
(d) The last day's play between India change in the length of wire is l.
amplitude a + b
and Sri Lanka were disrupted by If the same force F is applied on
the wire of the: (c) The motion is SHM with
rain.
amplitude a2 + b2
17. Find out the correct proverb. 2l
l (d) The motion of SHM with
(a) diamond cuts iron (a) (b)
2 2 amplitude a 2 + b 2
(b) diamond cuts gold
3l 27. A resistance coil and a battery are
(c) diamond cuts silver (c) (d)  N o n e o f
2 given. In which of the following
(d) diamond cuts diamond these cases the heat generated is
18. Choose the most appropriate 25. Consider p-V diagram for an maximum ?
preposition: ideal gas shown in figure. (a) When the coil is directly
The Indian magpie indulges ........... p connected to the battery as
1
a long flight. such
(a) with (b) in (b) When the coil is divided into
Constant
(c) at (d) on p= V two equal parts and both parts
Directions (19-20): In the following are connected to the battery in
questions some of the sentences have parallel
errors and some have none. Find out 2
(c) When the coil is divided into
which part of a sentence has an error. V four equal parts which are
The number of that part is your answer. Out of the following diagram,
connected to the battery in
If there is no error, the answer is (d), i.e., w h i c h r e p r e s e n t s t h e T- p parallel
No error. diagram ? (d) When only half the coil is
19. Being (a)/a rainy day (b)/I could not connected to the battery
T
go out (c)/No error (d) 28. A cylindrical bar magnet is
2
20. He is (a) /capable to do this work (b) (i) rotated about its axis shown in
/within the stipulated period (c) /No figure. A wire is connected from
errror (d) 1 the axis and is made to touch
p
262 Practice Set-21

the cylindrical surface through (c) Wavelength of sound waves (a) 0.4 × 10–4 Nm
a contact. Then, (d) Loudness of sound waves (b) 0.4 Nm
32. Two batteries of emf E1 and E2 (c) 0.2 × 10–4 Nm
(E2 > E1) and internal resistances (d) None of these
Axis 36. In Young’s double slit experiment
r 1 a n d r 2 re s p e c t i v e l y a re
N
A connected in parallel as shown when wavelength used is 6000
in figure. Å and the screen is 40 cm from
ω the slits, the fringes are 0.012
Bar cm wide. What is the distance
Magnet between the slits?
(a) 0.024 cm (b) 2.4 cm
(c) 0.24 cm (d) 0.2 cm
37. In figure, the energy levels of
S the hydrogen atom have been
shown along with some transitions
(a) A direct current flows in the
marked A, B, C. The transitions A,
ammeter A
(a) The equivalent emf Eeq of the B and C respectively represents.
(b) No current flows through the two cell is between E1 and E2
ammeter A i.e., E < Eeq < E2.
(c) An alternating sinusoidal (b) The equivalent emf E eq is
current flows through the smaller that E1
ammeter A with a time period (c) The Eeq is given by Eeq = E1 +
T = 2π/ω E2 always
(d) A time varying non-sinusodial (d) Eeq is independent of internal (a) The first member of the Lyman
current flows through the resistance r1 and r2 series, third member of Balmer
ammeter A 33. Critical temperature of CO2 is series and second member of
29. An electromagnetic wave, going 31.2°C. In summer, the room Paschen series continuum
through vacuum is described by temperature is 40°C: (b) The ionisation potential of
E = E0 sin (kx – ωt). (a) CO2 cannot be liquified H, second member of Balmer
W h i c h o f t h e f o l l o w i n g i s (b) can be liquified with increase series and third member of
independent of wavelength ? Paschen series
of pressure
(a) k (b) ω (c) can be liquified with decrease (c) The series limit of Lyman series,
k of pressure second member of Balmer
(c) (d) kω
ω (d) can be liquified if temperature series and third member of
30. A current carrying circular loop of CO 2 is decreased below Paschen series
of radius R is placed in the xz 31.2°C (d) The series limit of Lyman
plane with centre at the origin. 34. The radius of a planet is R. A series, third member of Balmer
Half of the loop with x > 0 is satellite revolves in a circle it in series and second member of
now bent so that it now lies in a circle of radius r with angular Paschen series
the yz plane. speed ω. The acceleration due to 38. Digital circuit can be made by
(a) The magnitude of magnetic gravity on planet’s surface would be: repetitive use of this gate:
moment now diminishes (a) AND (b) OR
(b) The magnetic moment does not r 3ω r 2 ω3 (c) NOT (d) NAND
(a) (b)
change R 2R 39. The working of optical fibres is
(c) The magnitude of B at (0, 0, z), based on:
z >> R increases r 3ω 2 r 2ω2 (a) Dispersion of light
(c) (d)
(d) The magnitude of B at (0, 0, z), R2 R (b) Total internal reflection
z >> R unchanged 35. A magnet of dipole moment 2 (c) Polarisation of light
31. Change in temperature of the Am 2 is deflected through 30° (d) Interference of light
medium changes: from magnetic meridian. The 40. The de-Broglie wavelength of a
(a) Frequency of sound waves required defelecting torque is proton (charge = 1.6 × 10–19 C,
(b) Amplitude of sound waves (BH = 0.4 × 10–4) m = 1.6 × 10–27 kg) accelerated
Practice Set-21 263

through a potential difference of 51. If f be a function from the set


1 kV is: of natural numbers to the set of
(a) 600 Å (b) 0.9 × 10–12 m even natural numbers given by f
(c) 7 Å (d) 0.9 nm (x) = 2x. Then, f is:
41. A car moves from X to Y with a (a) one-to-one but not onto
uniform speed vu and returns to (b) onto but not one-one
Y with a uniform speed vd. The (c) Both one-one and onto
average speed for this round trip (d) Neither one-one nor onto
is: (a) Current leads source emf by dy
π 52. If xpyq = (x + y)p + q, then is
dx
2vd vu 3 equal to:
(a) (b) vu vd
vd + vu (b) Current leads source emf by y py
(a) (b)
vd vu π x qx
(c) (d) vu + vd
vd + vu 2 4 x qy
(c) (d) px
42. If h is Planck’s constant and λ is π y
(c) Source emf lead current by
h 4 53. The triangle formed by the
wavelength, has dimensions (d) None of the above
λ points A (2a, 4a), B (2a, 6a) and
of:
(a) Momentum MATHEMATICS C(2 a + 3 a, 5 a) is:
(b) Energy 46. I f A = { 1 , 2 , 5 , 6 } a n d B (a) right angled
(c) Mass = {1, 2, 3}, then what is (b) isosceles
(d) Velocity (A × B) ∩ (B × A) equal to: (c) equilateral
43. A galvanometer of resistance 100 (a) {1, 1}, (2, 1) (6, 1) (3, 2)} (d) None of these
Ω gives full sclae deflection for (b) {(1, 1) (1, 2), (2, 1) (2,2)} 54. The coefficient of x r in the
10 mA current. What should be (c) {(1, 1) (2, 2)}
e
expansion of e e :
the value of shunt so that it can
(d) {(1, 1) (1, 2) (2, 5) (2, 6)} 1  1 2r 3r
r

measure a current of 100 mA? (a)
47. The value of ii is:  + + + .....∞ 
(a) 11.11 Ω (b) 9.9 Ω r ! 1! 2! 3! 
(a) ω (b) – ω2
(c) 1.1 Ω (d) 4.4 Ω
(c) π/2 (d) None of these 1 1r 2 3r 
44. T h e r a d i u s o f c u r v a t u r e (b)  − + + .....∞ 
of the curved surface of a 48. What is the value of sec2 tan–1 r !  1! 2! 3! 
plano-convex lens 20 cm. If the  5
 11  ? 1 1r 2r 3r 
refractive index of the material (c)  + − + .....∞ 
of the lens be 1.5, it will: r !  1! 2! 3! 
(a) 12/96 (b) 217/921
(a) Act as a convex lens only (c) 146/121 (d) 267/121 (d) None of these
for the objects that lie on its 49. If in the expansion of (1 + x)m 55. The circle x2 + y2 + 4x – 4y + 4 =
curved side 0 touches
(1 – x)n the coefficient of x and x2
(b) Act as a concave lens for the (a) Only the x-axis
objects that lie on its curved are 3 and –6 respectively, then m
(b) Only the y-axis
side is:
(c) Both the axis
(c) Act as a convex lens irrespective (a) 6 (b) 9
(d) Neither of the axes
of the side on which the object (c) 12 (d) 24
56. What is the value of
lies 50. W h a t i s t h e s u m o f f i r s t
 9  27   3
(d) Act as a concave lens irrespective eight terms of the series log10   − log10   + log10  
 8  32   4
of side on which the object lies 1 1 1
1 − + − + ......?
45. In the given AC circuit, when 2 4 8 (a) 3 (b) 2
switch S is at position 1, the (c) 1 (d) 0
89 57
source emf leads current by π . (a) (b)
384
57. F o r e a c h n ∈ N , 3 2n – 1 i s
6 128 divisible by:
Now, if the switch is at position (a) 8 (b) 16
85
2, then (c) (d) None of these
128 (c) 32 (d) None of these
264 Practice Set-21

58. What is the equation of the curve (a) 1 km (b) 2 km 72. Which of the following is formed
passing through the origin and out of volcanic eruptions in
(c) 4 km (d) 7 km
satisfying the differential equation India?
dy = (y tan x + sec x) dx? 5π 2 (a) Deccan Plateau
66. If (tan–1 x)2 + (cot–1 x)2 = , then
(a) y = x cos x (b) y cos x = x 8 (b) Lakshadweep Islands
(c) xy = cos x (d) y sin x = x x is equal to:
(c) Western Ghats
59. The position of a point in time ‘t’ (a) – 1 (b) 1
(c) 0 (d) 2 (d) Himalayas
is given by x = a + bt – ct2, y = at
67. The equation of the hyperbola 73. Article 368 of the Indian
+ bt2. Its acceleration at time t' is:
whose eccentricity is 2 and the Constitution deals with :
(a) b – c (b) b + c
distance between the foci is 16, (a) Emergency Provisions
2 2
(c) 2b – 2c (d) 2 b + c taking transverse and conjugate (b) Right to Primary Education
axes of the hyperbola as x and y (c) Right to Information
60. If f (x) is defined as:
axes respectively is: (d) Amending Procedure
 2 kx + 3, if x < 1
f (x) =  (a) x2 – y2 = 0 (b) x2 – y2 = 32
74. Heavy water is :
2
1 − kx , if x > 1 (c) x2 – y2 = 2 (d) None of these
(a) Deuterium
then for what values of k does
A B C (b) Rain water
lim exist? 68. 2abc cos cos cos is equal
x→ 1 2 2 2 (c) Tritium oxide
to:
(a) 0 (b) 1 (d) Deuterium oxide
(a) (b + c – a)∆
(b) (c + a – b)∆ 75. Which of the following computer
(c) (d)
(c) (a + b – c)∆ memories is non-volatile ?
61. The median of distribution 83, (d) (a + b + c)∆
(a) DRAM (b) SRAM
54, 78, 64, 90, 59, 67, 72, 70, 73 is: 69. What is the value of
(a) 71 (b) 70 cos15° + cos 45° (c) ROM (d) RAM
?
(c) 72 (d) None of these 3 3
cos 15° + cos 45° 76. In present day computing, which
62. Three mangoes and three apples code is used and is accepted
are in box. If two fruits are 1 1
(a) (b) worldwide ?
chosen at random, the probability 4 2
(a) ASCII
that one is a mango and the other 1
(c) (d) None of these (b) Hollerith Code
is an apple is:
3 (c) EBCDIC
(a) 2/3 (b) 3/5
(c) 1/3 (d) None of these  cos θ sin θ 0  (d) ISCII
70. If the matrix  sin θ cos θ 0  is
63. In a polygon, the number of  0 0 1  77. Where was the world’s largest
diagonals is 54. The number of multi-stage, multi-purpose
sides of the polygon is: singular, then what is one of the
scheme Kaleshwaram Lift
(a) 10 (b) 12 value of θ?
Irrigation Project (KLIP)
(c) 9 (d) None of these π π inaugurated?
(a) (b)
2 dx 4 2
64. What is ∫ 2 equal to? (a) Wayanad, Kerala
0 x +4
(c) π (d) 0 (b) Cuddapah, Andhra Pradesh
π π (c) Medigadda,Telangana
(a) (b) REASONING AND GENERAL
2 4 (d) Bellary, Karnataka
π π AWARENESS (RAGA) 78. Which of the following numbers
(c) (d) 3
8 71. Which Governor General was is not a prime number?
65. The straight roads intersect at an called as the 'Father of Local (a) 731 (b) 227
angle of 60°. A bus one road is 2 Self-Government' in India ? (c) 347 (d) 461
km away from the intersection
and a car on the other road is 3 (a) Lord Wellesley 79. If tan (–7p/4) = x, then the value
km away from the intersection. (b) Lord Canning of x is
Then the direct distance between (c) Lord William Bentinck (a) – 1/2 (b) – 3/ 2
the two vehicles is: (d) Lord Ripon (c) 3/ 2 (d) 1
Practice Set-21 265

80. 0.5 ÷ 0.005 = ? (a) Grandson Direction (92): In following questions


(a) 10 (b) Granddaughter a stands for equal to’; b for ‘greater
(b) 100 (c) Son than’; γ for ‘less than’ and d for ‘not
(c) 1000 equal to.
(d) Cannot be determined
(d) None of these 92. If bcy g ax cy a bz and a2 g bc,
87. In a certain code language
then
81. The radius of a hemisphere is “TITANIC” is written as
(a) cx a abz (b) cx γ abz
14 cm, find its volume. “7371835” and “SEMATIC” is
written as “6241735”. How will (c) cx d abz (d) c2x γ a2z
(a) 11498.66 cubic cm
'MAINTAIN' be written in that 93. Identify the diagram that best
(b) 1916.44 cubic cm represents the relationship
(c) 2874.67 cubic cm code language?
among the given classes.
(d) 5749.33 cubic cm (a) 41387138 (b) 43187138
Kitchen, Utensils, Glass
(c) 41837138 (d) 41783138
82. Two students appeared for
88. A person walks 1 mile to West,
an examination. One of them (a)
turns left and walks 1 mile and
secured 10 marks more than the turns left and walks 1 mile and
other and his marks were 75% again turns left and walks 1 mile.
of the sum of their marks. The
What is the direction he is facing
marks obtained by them are : (b)
now?
(a) 52 and 42 (b) 68 and 58
(a) North (b) South
(c) 63 and 53 (d) 15 and 5
(c) East (d) West (c)
83. Ticket for an adult is ` 1200 and
a child is ` 800 respectively. One 89. Shailendra is shorter than keshav
child goes free with two adults. but taller than Rakesh, Madhav (d)
If a group has 25 adults and is the tallest. Aashish is a little
5 children what is the discount shorter than Keshav and little
taller than Shailendra. If they Direction (94): Select the related
the group gets ? word/letters/number from the given
(a) 17.24 percent stand in the order of increasing
alternatives.
(b) 24 percent heights, who will be the second?
94. TZ : GA : : QR : ?
(c) 24.41 percent (a) Aashish (b) Shailendra (a) KN (b) RS
(d) 11.76 percent (c) Rakesh (d) Madhav (c) NQ (d) JI
84. Praful has done 1/2 of a job in Direction (90): A series is given with 95. In the following figure, rectangle
30 days, Sarabjit completes the one term missing. Choose the correct represents Brand managers,
rest of the job in 10 days. In how alternative from the given ones that circle represents Cartoonists,
many days can they together do will complete the series. triangle represents Sculptors
the job ? and square represents Fathers.
90. ijk, mno, lmn, pqr, opq,?
(a) 30 days (b) 45 days Which set of letters represents
(a) stv (b) suv Fathers who are either Brand
(c) 15 days (d) 60 days
(c) stu (d) stt managers or Cartoonists?
85. A bank offers 20% compound
Direction (91): Study the following
interest per half year. A customer
information carefully and answer the
deposits ` 6,000 each on 1st A
questions given below:
January and 1st July of a year. At B
P, Q, R, S, T, V and W are sitting
the end of the year, the amount
around a circle facing at the centre. V
he would have gained by way of G C
is second to the left of P and second to
interest is:
the right of W. T is third to the right of F D
(a) ` 7680 (b) ` 3840 Q and is not an immediate neighbour
(c) ` 1920 (d) ` 960 of V, S is third to the right of R. I E
86. P is the only son of Q who is H
91. Who is second to the right of Q?
the only daughter of S. D is the
mother of B and daughter-in-law (a) R (b) W
of Q. How is B related to Q? (c) T (d) S
266 Practice Set-21

(a) GDF (b) FDB 96. Which of the following is when the eight students are
(c) BCG (d) FDH definitely false? arranged in descending order of
Directions (96–100): Study the (a) G is shorter than F. height?
(b) C is shorter than F. (a) B (b) F
following information carefully and
(c) G (d) B or G
answer the questions given below: (c) F is taller than C.
99. How many of them are definitely
(i) A, B, C, D, E, F, G and H are eight (d) All are true. shorter than F?
students, each having a different 97. If another student J, who is taller (a) Three
height. than E but shorter than G, is (b) Four
(ii) D is shorter than A but taller than added to the group, which of the (c) Five
G. following will be definitely true ? (d) Data inadequate
(iii) E is taller than H but shorter than (a) C and J are of the same height 100. Which of the following is redundant
(b) J is shorter than D. to answer all the above questions?
C.
(c) J is shorter than H. (a) (ii) only
(iv) B is shorter than D but taller (b) (ii) and (iii) only
(d) J is taller than A.
than F. (c) (iii), (iv) and (v) only
98. Which of the following will
(v) C is shorter than G. (d) All are necessary to answer
definitely be the third from top the above questions
(vi) G is not as tall as F.

Answers with Explanation


ENGLISH 17. (d) Correct proverb is diamond 4 m. It is also equal to potential
cuts diamond which means to describe energy. Hence,
1. (c) The antonym of the word
a situation in which a sharp witted person mgh= 4m
'Cumbersome' used in the passage is
meets its match. 4 4
'manageable'. ∴ h= = = 0.4 m
2. (a) The given word 'Precise' in 18. (b) The most appropriate preposi- g 10
tion is 'in' for the following sentence: The p2
the passage meant for 'defined'. 22. (b) Increase in KE =
Indian magpie indulges in a long flight. 2m
3. (c) The European militaries
19. (a) The sentence lacks in subject. 'It' p 6p
adopted ADA because of its tough and
should be used as subject. Hence, replace New momentum = p + 5 = 5
complex language as well as its space 2
'being' by 'It being'.  6 p
applications and its quick and precise  
20. (b) The word 'capable' is followed 36 p 2
functions. KEj = 5 =
by preposition 'of' and then a gerund will 2m 25 2m
4. (a) Actually, the Department of follow.
Defense was looking for the name of the Look at the sentences: 36 p 2 p 2 11. p 2
language which is precise. ∆KE = − =
He is quite capable of lying to get out 25 2m 2m 25.2m
5. (a) The correct spelling is souvenir. of trouble. Percentage Increase = 44%.
Souvenir means a memory, memento. I am perfectly capable of doing it my
6. (a) The correct spelling is Quin- 23. (b) Taking the same mass of
self. sphere if radius is increased, then
tessence which means purest essence of Hence, 'capable to do this work' moment of inertia, rotational kinetic
something. Hence option (a) is correct. should be replaced by 'capable of energy and angular velocity will change
7. (d) 8.(b) 9. (d)   10. (d) doing this work'. but according to law of conservation of
11. (d) 12.(d)  13. (c)   14. (c) momentum, angular momentum will not
15. (c) PHYSICS change.
16. (c) The sentence, the last day's play 21. (d) Initial kinetic energy of the 24. (a) Elongation,
between India and Sri Lanka, will be used body FL L
1 1 l= or l ∝
as a subject and 'by' will be used with 'The AY
= mv 2 = m(4) 2 = 8 m r2
rain'. As 'The last day’s play' is singular, 2 2 2 2
so ‘was’ will be used. Hence option (c) Let at height h, the kinetic energy l L r   1 1
∴ 2 = 2 ×  1  = 2 ×   =
will be the answer. r e d u c e s t o h a l f ,i . e ., i t b e c o m e s l1 L1  r2  2 2
Practice Set-21 267

l is bent in yz plane, then magnetic moment E1r2 + E2 r1


⇒ l2 = 1 due to half current loop in xy plane, ∴ Eeq = × req
2 r1r2
The change in the length of other wire  πR 2 
M1 = I   acting along z-direction.
l 2  ( E1r2 + E2 r1 ) rr
is · × 12
2 Magnetic moment due to half current = r1r2 (r1 + r2 )
25. (c) From the figure,  πR 2 
loop in yz plane. M2 = I   along E1r2 + E2 r1
pV = Constant 2  =
x-direction. (r1 + r2 )
Means T is constant.
Effective magnetic moment due to This shows that whaterver may be the
Option (c) is correct. entire bent current loop, values of r1 and r2, the value of Eeq is in
26. (d) y = a sin ωt + b cos ωt between E1 and E2. As E2 > E1.
Let a= A cos θ and b = A sin θ M′ = M12 + M 22
So E1 < Eeq < E2.
Then, a2 + b2= A2 (cos2 θ + sin2 θ) 33. (a) In summer, at the room
2 2
= A2  I πR 2   I πR 2  temperature gases cannot be liquified
=  2  +  
⇒ A= a 2 + b 2 2  above critical temperature.
From Eq. (i), y= A cos θ sin ωt + A sin I πR 2 v
= 2<M 34. (c) ω =
θ cos ωt 2 r
= A sin (ωt + θ) i.e., magnetic moment diminishes. GM
It is an equation of SHM with amplitude
So, v= ,
31. (c) Due to change in temperature, r
A= a 2 + b2 . velocity changes because V ∝ T
1 GM
Hence wavelength changes we get ⇒ ω =
27. (c) Since, battery supplies constant r r
V2 1 v GM
emf. So, power P = ⇒ Power ∝ As we know, v = (Here, v → ω2=
R R fixed). λ r3
So, R should be minimum to generate 32. (a) Refer figure the equivalent r ω = g × R2
3 2

maximum heat. In option (c) resistance internal resistance of two cells between
would be minimum. So, heat generated A and B is r 3ω 2
⇒ g =
would be maximum. R2
28. (d) As there is no change in 35. (a) π = MB sin θ
magnetic flux associated with the circuit, = 2 × 0.4 × 10–4 × sin 30°
no current is induced in the circuit. The
= 0.4 × 10–4 Nm
ammeter A shows no deflection.
θ λD λD
29. (c) Here, kx = θ0 or k = 0 36. (d) β = ⇒d =
x d β
θ0 6000 × 10 −10 × (40 × 10 −2 )
and ωt = θ0 or ω = 1 1 1 = = 0.2 cm.
t = + 0.012 × 10 –2
req r1 r2
k t 1 1 37. (a) A represents series limit of

= = = Lyman series, B represents third member
ω x  x v r1 + r2
  of Balmer series and C represents second
t = r1r2 member of Paschen series.
Where, v is the velocity of r1r2 38. (d) We know that the repetitive use
electromagnetic wave, which is ⇒ req = ...(i) of NAND and NOR gate gives digital
r1 + r2 circuits.
independent of wavelength of wave.
If Eeq is the equivalent emf of the two 39. (b) Working of optical fibres is
30. (a) For a circular loop of radius
cells in parallel between A and B, then based on total internal reflection.
R, carrying current l in xy plane, the
magnetic moment M = I × πR 2 . It Eeq E1 E2 E1r2 + E2 r1 n h h
= + = 40. (b) λ = = =
acts perpendicular to the loop along r r1 r2 r1r2 p 2mE 2mqV
eq
z-direction. When half of the current loop ⇒λ=
268 Practice Set-21

1  1 1 ∴ (A × B) ∩ (B × A) = {(1,1)
6.6 × 10−34
= (µ − 1)  −  (1, 2) (2, 1) (2, 2)}
f  R R2
2 × 1.6 × 10−27 × 1.6 × 10−19 × 1000
1
47. (d) Let A = ii
1 1 1 ⇒ log A = i log i
6.6 × 10 −34 = (15 − 1)  −  =
⇒ λ= = 0.9 × 10 −12 m  ∞ 20  40 ⇒ log A = i log (0 + i)
7.16 × 10 −22 = i [log 1 + i tan–1 ∞]
41. (a) Average speed f = + 40 cm. The lens behaves as ⇒ log A = i [0 + i π/2]
convex. = – π/2
Distance travelled If object lies on its curved side.
= ∴ A = e–π/2
Time taken R1 = 20 cm, R2 = ∞
5
Let t1 and t2 be times taken by the car 48. (c) sec2 tan–1  
to go from X to Y and then from Y to X, 1  1 1  11 
= (µ − 1)  − 
respectively. f′  R1 R2  ( 1 + tan2 θ = sec2 θ)
XY XY 5
Then, t1 + t2= +  1 1
vu vd = (15 − 1)  20 − ∞  = 1 + tan2 tan–1  11 
 
2
 v + vd  1   5 
= XY  u = = 1 +  tan tan −1   
 vu vd  40   11  
Total distance travelled f′ = 40 cm. The lens behaves as convex. 2
5 25 146
= XY + XY = 2XY 45. (b) In position 1, = 1 +  11  = 1 + =
  121 121
2 XY 2vd vu π ωL (1000)( 3 × 10 −3 )
∴ vav = = tan = = R 49. (c) (1 + x)m (1 – x)n
 v + vd  vd + vu 6 R
XY  u  m(m − 1) 2 
 vu vd  1 3 = 1 + mx + x + .... 
⇒ =  2 
h 3 R
42. (a) We know that, λ = n(n − 1) 2
mv  
∴ R= 3Ω × 1 − nx + x + .... 
h In position 2,  2 
∴ = mv.
λ  1  Coefficient of x = (m – n)
h X C  ωC  = 3 (given) ....(i)
Hence, has dimensions of tan φ = = Coefficient of
λ R R
momentum. m(m − 1) n(n − 1)
1 x2 = – mn + =–6
GI g 100 2 2
43. (a) S = = = 11.11 Ω 1000 (given)
I − Ig g 1000 × × 10 −6
tan φ = 3 =1 ⇒ m2 – m – 2mn + n2 – n = – 12
44. (c) Here, µ = 15 3 ⇒ m2 + n2 – 2mn – (m + n) = – 12
π ⇒ (m – n)2 – (m + n) = – 12
φ= ⇒ (3)2 – (m + n) = –12
O 4
⇒ m + n = 21 ....(ii)
MATHEMATICS If solving equation (i) and (ii), we get
(a) m = 12.
46. (b)
Given, A = {1, 2, 5, 6} and
B = {1,2, 3} 50. (c) Explanation (c),
A × B = {(1, 1) (1, 2) (1, 3) 1 1 1
O Given, series is 1 − + − + .....
(2, 1) (2, 2), (2, 3) (5, 1) 2 4 8
(5, 2) (5, 3) (6, 1) (6, 2) (6, 3)} Since, it is a geometric progression.
(b) B × A = {(1, 1) (1, 2) (1, 5) Here, first term, a = 1 and common
(1, 6) (2, 1), (2, 2) (2, 5) 1
If object lies on plane side, ratio, r = – < 1
(2, 6) (3, 1) (3, 2) 2
R1= ∞, R2 = – 20 cm
(3, 5) (3, 6)} ∴ The sum of first eight terms of the
Practice Set-21 269

series. dy y ∴ P (n) is divisible by 8, ∀


8
a (1 − r ) ∴ = n∈N
dx x
i.e.,S8 = 58. (b) Given differential equation is
(1 − r ) 53. (c) dy= (y tan x + sec x)dx
[By formula, dy
⇒ – y tan x = sec x
n AB = (2a − 2a ) 2 + (6a − 4a ) 2 = 2a dx
Sn = a (1 − r ) when r < 1]
(1 − r ) This is a linear equation of the form
BC = (2a + 3a − 2a ) 2 + (5a − 6a ) 2 dy
  1 8  = 2a + Py = Q
1 1 −  −   1 dx
1− CA = (2a + 3a − 2a ) 2 + (5a − 4a ) 2
  2   256
Therefore, P = – tan x, Q = sec x
= = = 2a and If = e ∫ Pdx = e ∫ − tan xdx = cos x
 1 1
1−  −  1+ ⇒ ∆ABC is an equilateral. ∴ The solution is:
 2  255 2 2 85
255 / 256 n
= = 256 × 3 = 128 54. (a) Expansion of ee
3/ 2
e x e 2 x e3 x e 4 x e rx
y cos x =∫ cos x.sec x xdx + C
51. (c) Given, f : N →N even and f +
1 + 1! 2! + + + .... + .. ⇒ y cos x = ∫ 1dx + C
3! 4! r!
(x) = 2x
⇒ y cos x = x + C
For one-one Let x1, x2 ∈ R 1  x x2 xr 
=1+ 1 + + + ...... + + .... Since, this curve passes through (0, 0)
Considering f (x1) and f (x2) 1!  1! 2! r!  ∴ 0= 0 + C ⇒ C = 0
Such that f (x1) = f (x2) Hence, the required equation of curve
1  2 x 2 x2 (2 x) r 
⇒ 2x1 = 2x2 ⇒ x1 = x2 + 1 + + + ...... + + .... is
2!  1! 2! r! 
So, f is one-one, y cos x = x
Hence coefficient of xr. 59. (d) Given point is x = a + bt – ct2
For onto
and y = at + bt2
Let y = f (x) 1 1r 2r 3r 4r 
=  + + + + .... d2x
y r !  1! 2! 3! 4! Acceleration in x-direction =
Then, y = 2x ⇒ x = (expressing x 
2 = – 2c dt 2
in terms of y) 55. (c) x2 + y2 + 4x – 4y + 4 = 0
∴ x ∈ N y ∈ N (even) (x + 2)2 + (y – 2)2 = 4 d2y
(x + 2)2 + (y – 2)2 = 22 and acceleration in y-direction = 2
(even element of codomain has pre- dt
[x – (–2)]2 + (y – 2)2 = 22 = 2b
image in domain) ∴ Resultant acceleration
∴ Centre = (–2, 2) and radius = 2
So, f is onto. 2 2
So, circle touch has both axes.  d2x   d2 y 
Hence, f is both one-one and onto.
9  27   2  +  2 
52. (a) On taking log both sides, we 56. (d) log10   – log10   + =  dt   dt 
8
   32 
get 3
p log x + q log y= (p + q) log (x + y) log10   = (−2c) 2 + (2b) 2
4

1 1 dy 9 3  27  = 2 b2 + c2
⇒ p + q = log10  ×  – log10  
x y dx 8 4  32 
60. (d) We have:
1  dy  = log10  27 × 32  lim f (x) = lim f (1 – h)
= (p + q) x + y 1 +   
 dx   32 27  x →1− h→ 0

= log10 (1) = 0 lim [2k (1 – h) + 3]


= h→
p + q  p + q − q  dy
0
p 57. (a) Let P (n) = 32n – 1
– = At n = 1, P (1) = 8, which is
x x + y  x + y y  dx = [2k (1 – 0) + 3] = 2k + 3
divisible by 8.
py − qx py − qx dy ∴ P (1) is true. and lim lim f (1 + h)
f (x)= h→
⇒ = y( x + y) x→1 0
dx Let P(k) is true, then P(k)
x( x + y )
= 32k – 1 = 8I lim
∴ P (k + 1) = 32k + 2 – 1 = (8I = h→ 0 [1 – k(1 + h)2]
+ 1) 9 – 1 = 72I + 8 = 8I1
270 Practice Set-21

= 1 – k (1 + 0)2 = 1 – k 1
= [tan–1 (1) – tan–1 (0)] = s ( s − a ) s ( s − b) s ( s − c )
Now, lim f (x) exists, if 2 = 2abc
x→1 bc ac ab
1π  π
lim f (x) = lim+ f (x) =  − 0 2abc
x→1 x →1 2 4  8 = s s ( s − a )( s − b)( s − c)
⇒ 2k + 3 = 1 – k ⇒ 3k = – 2 abc
65. (d) a = 2, b = 3, ∠c = 60°
a+b+c
∴ k= = 2 s ( s − a )( s − b)( s − c)
2
61. (a) On arranging in ascending = (a + b + c)∆
order, we get 54, 59, 64, 67, 70, 72, 73, 2 km
78, 83, 90 cos15° + cos 45°
n = 10 (even number) 69. (d)
60º cos3 15° + cos 2 45°
∴ Median 3km
= =
∴ c = a + b – 2 ab cos60°
2 2 2
cos15° + cos 45°
10  10 
Value of th term + Value of  + 1 th term 1 (cos15° + cos 45°)(cos 2 45° + cos 2 15° − cos 45° cos15°)
2  2  = 13 – 12 = 7
2 2 1
= 2 2
(cos 45° + cos 15° − cos 45° cos15°)
= Value of 5th term + Value of 6th term ⇒ c = 7 km
2 66. (b) Let tan–1 x = y 1
=
70 + 72 1
+ (cos 45° + cos 30° + sin 45° sin 30°) 2 −
cos15°
= = 71 π 2 2
2 Then cot–1 x = −y
2 = 1
62. (b) Total ways = 6C2 = 15 2
Favourable ways = 3C1 × 3C1 = 9 π 
2 5π 2
1  3 1  1  3 + 1
∴ y2 +  − y  = 8 + +  −  
9 3 2  2  2 2 2 2 2 2 2 
Required probability = =
15 5 ⇒ 16y2 – 8πy – 3π2 = 0 1
63. (b) Let number of sides of ⇒ (4y – 3π) (4y + π) = 0 =
polygon is n. Since, number of sides of 1 3 +1+ 2 3 3 +1
⇒ y = tan–1 x = –π/4, (3π/4) + −
polygon is equal to number of vertices of 2 8 4
(rejected)
polygon. 1
π 8 4
 Number of diagonals of polygon (Since it is greater than ) = = =
2 4+4+2 3−2 3−2 6 3
= 54 ∴ x = tan (–π/4) = 1 8
n(n − 1) 67. (b) Let the equation of the
∴ nC2 – n = 54 ⇒ – n = 54 70. (a) Since matrix
2 x2 y 2
⇒ n2 – 3n = 108 hyperbola be 2 − 2 = 1 cos θ sin θ 0
a b sin θ cos θ 0 is singular. Then
⇒ n – 3n – 108 = 0
2

The coordinates of the foci are (ae, 0) 0 0 1


⇒ (n + 9) (n – 12) = 0
and (–ae, 0) cos θ sin θ 0
n = 12
( n cannot be negative) ∴ 2ae = 16 ⇒ 2a 2 = 16 ⇒ a = sin θ cos θ 0 = 0
2 dx 4 2 0 0 1
64. (c) Let I = ∫ 2 Also, b2 = a2 (e2 – 1) = 32 (2 – 1) = 32
0 x +4
 dx 1 x Thus, a2 = 32 and b2 = 32 cos θ sin θ
∵ ∫ 2 = tan −1 
⇒ =0
x + a
2
a a Hence, the required equation is: sin θ cos θ
2
1 −1 x  x2 y 2 ⇒ cos2 θ – sin2 θ = 0
=  tan − = 1 ⇒ x2 – y2 = 32
2 2  0 32 32 π
⇒ cos 2θ = cos
2
1  −1  2   A B C
tan   − tan −1 (0)  68. (d) 2abc cos cos cos π
= 2  2 2 2 ⇒θ=
2  4
Practice Set-21 271

REASONING AND GENERAL of present-day computers. ASCII codes 83. (d) Price of tickets with discount
represent text in computers, telecommuni- 25 adults + 5 children
AWARENESS (RAGA)
cations equipment, and other devices. M (10 adults + 5 children) + 15 adults
71. (d) Lord Ripon is known as = 10 × 1200 + 15 × 1200
ost modern character-encoding schemes
the ‘Father of Local Self Government' = 12000 + 18000
are based on ASCII, although they support
in India. In his famous resolution = ` 30000.
many additional characters.
on local self-government on May Without Discount
18, 1882, Ripon recognised the twin 77. (c) On June 21, 2019, The first
= 25 × 1200 + 5 × 800
considerations of local government: phase of the world’s largest multi-stage, = 30000 + 4000
(i) administrative efficiency and (ii) multi-purpose scheme Kaleshwaram = ` 34000.
political education. Lift Irrigation Project (KLIP) worth Discount %
72. (a) The Deccan Plateau is made Rs 80,000 crore was inaugurated
 34000 − 30000 
up of lava flows or igneous rocks known by the Chief Minister of Telangana = 
  × 100%
34000
as the Deccan Traps. The rocks are Chandrashekar Rao at Medigadda in
spread over the whole of Maharashtra Jayashankar-Bhupalpally district near 4000 × 100 200
= %=
and parts of Gujarat and Madhya the borders with Maharashtra and 34000 17
Pradesh, thereby making it one of the Chhattisgarh. = 11·76%.
largest volcanic provinces in the world. 78. (a) 731 is not a prime number. 84. (c) Full job done by Praful in
The Deccan traps formed between 60 60 days
 −7π  1 1
and 68 million years ago at the end of 79. (d) x = tan  Left job = 1 – =
 4  2
the cretaceous period. 2
73. (d) Article 368 of the  −7π  1
= tan  4  tan (– q) = – tanq job is completed by Sarabjit
Constitution of India deals with the 2
amendment process. It empowers = 10 days
 3π   3π  Full job is completed by Sarabjit
Parliament to amend the Constitution = − tan  π +  = − tan  4 
 4    = 10 × 2 = 20 days
by way of addition, variation or repeal
1 day’s job of both
of any provision according to the = – tan 135° = – tan (90º + 45º)
procedure laid down therein, which = + cot 45º = 1 1 1 1+ 3
= + =
is different from the procedure for 60 20 6
80. (b) ? = 0.5 ÷ 0.005
ordinary legislation.
0.5 4 1
74. (d) Heavy water is deuterium = = 100 = =
oxide (D2O). It is a form of water that 0.005 60 15
contains a larger than normal amount 81. (d) It is given that r = 14 cm Time taken by both
of the hydrogen isotope deuterium (2H 2
Volume of hemisphere = πr 3 1
or D, also known as heavy hydrogen), 3 = days = 15 days
1
rather than the common hydrogen-1
2 22 15
isotope (1H or H, also called protium) = × × 14 × 14 × 14
3 7 85. (b) Amount
that makes up most of the hydrogen in
normal water. 44 2
= × 2 × 196  20  P × 40 ×1
75. (c) Read-only memory (ROM) 3 = P 1 +  +
 100  100 × 2
is a form of data storage in computers = 5749.33 cubic cm
and other electronic devices that is 82. (d) Let first student get x marks 36 6000 × 40
= 6000 × +
non-volatile. Its contents are retained and second (x + 10) marks 25 100 × 2
even when the device is powered off. 75
Other examples of non-volatile memory x= ( x + x + 10) 6000 × 36 × 4 240000
100 = +
include flash memory, ferroelectric 100 200
3
RAM (F-RAM), hard disk drives, 10 + x = (2 x + 10) = 60 × 144 + 1200
floppy disks, etc. 4 Amount = 8640 + 1200
40 + 4x = 6x + 30 Amount = 9840
76. (a) American Standard Code
10 = 2x ⇒ x = 5 Compound Interest = 9840 – 6000
for Information Interchange (ASCII) is
Their marks were 5 and 15. = ` 3840
the predominant character set encoding
272 Practice Set-21

86. (d) Only son of Q ⇒ R ↓ ↓ ↓ ↓ ↓ Kitchen


Only daughter of S ⇒ Q. 1 2 3 4 5 Utensils
D ⇒ B's mother Hence, Shailendra will be the second.
Glass
D ⇒ Q's daughter-in-law 90. (c)
So, Q ⇒ B's grandmother.
So, B ⇒ Q's grandson or grand- 94. (d) As, Similarly,
daughter
The Blood relationship tree/chart
can be constructed as given below:
Finally, the missing term is stu.
[Hints 41]: According to the given The missing term is JI.
information, we get 95. (b) Brand managers = {F, D, E,
V  Second to the left of P and H}
Second to the right of W. Cartoonists = {A, B}
T  Third to the right of Q. Sculptors = {G, F, H, I}
Therefore, the relationship cannot S  Third to the right of R Fathers = {B, C, D, G, F}
So, the required set of letters
be determined. So, the circular arrangement is as = {F, D, B}
87. (a) As, follows, [Hints 96–100]: By analysing the given
T I TANI C information, the descending order of






7 3 7 1 8 3 5
height of the eight students is as follows:
From (i) A > D > G
S E MA T I C
From (iii) C > E > H






6 24 1 7 35 From (vi) D > B > F


Similarly From (v) G > C
91. (a) By analysing the above
From (vi) F > G.
MA I N TA I N circular arrangement, it is clear that
1 2 3 4 5 6 7 8 ...(vii)



R  Second to the right of Q.




4 1 3 8 71 38 A> D > B > F> G > C > E > H


92. (c) According to the given
(Tallest)       (Shortest))
Finally, ‘MAINTAIN’ written as information
bcy g ax  bcy < ax ...(i) 96. (d) From (vii), it is clear that
41387138 in code language.
cy a bz  cy = bz  ...(ii) B>F>G>C>E
88. (c) According to the question,
and a2 g bc  a2 < bc ...(iii) Therefore, all are true.
the direction diagram will be as follows
From (ii), we get 97. (b) From (vii), it is clear that
c2y = bcz > a2z A>D>B>F>G>C
[∴ bc > a2 from (iii)] According to the question,
 c2y > a2y A>D>B>F>G>C>J>E>H Therefore,
From (i), we get 'J' is shorter than 'D' is definitely true.
ax > bcy I > a2y 98. (a) From (vii)
[∴ bc > a2 from (iii)] 1 2 3 4 5 6 7 8
Clearly, person is facing East.
 ax > a2y  x > ay A > D > B > F > G > C >E > H
89. (b) According to the question,
K > S > R  cx > acy Therefore, third from the top = B
Madhav is the tallest.  cx > abz[∴ cy = bz from (ii)] 99. (b) From (vii)
K > A > S  cx ≠ abz  cx d abz. F > G > C > E > H
 K=Keshav 
On arranging the   Therefore, shorter than F
93. (b) Kitchen, utensils, glass are
above data, we get S=Shailendra  complements of each other because, ⇒ G, C, E, H i.e. Four.
∴ M > K > A > S > R  R=Rakesh  100. (d) All are necessary to
  utensils and glass are in kitchen. So,
or  M=Madhav  answer the above question.
the best representeation among them is
  R < S < A < K < M  A=Aashish  given below—
  qqq
Practice Set-22 273

Air Force Airmen Group X & Y Exam


Practice Set-22

law, not only regards life and protects (c) entrenched class privileges
ENGLISH every man in enjoyment of it but also (d) justice and fair play
Directions (1–4): Read the passage furnishes him with everything necessary 5. Which of the following words is
given below and answer the questions for its support. For there is no man, so misspelt?
by selecting the most appropriate indecent or wretched but that he may
option. (a) Gregarious (b) Fatalist
demand a supply sufficient for all the
If the reason in the region of thought is necessities of life from the more opulent (c) Philistine (d) Celebasy
the aim of science, the rule of equality part of the community.” 6. Pick out the correct spelling of the
in the region of behaviour is the aim of 1. What is the fundamental tenet word among the following words.
democracy. Democracy is not a political of democracy according to the (a) Platetude (b) Platitud
arrangement or a form of government. It author? (c) Pllattitude (d) Platitude
is a pattern of life, an active conviction (a) Right to choose representatives 7. The Indirect transformation of –
which informs and inspires every thought, (b) Right to stand for public offices She said to the boy, "How much to
word and deed. Our present constitution (c) The presence of effective you expect to earn?" is:
of society induces in its more fortunate opposition (a) She asked the boy how much did
members far too great readiness to accept (d) The operation of the principle of he expected to earn
privilege as though it were inherent in the equality (b) She asked the boy how much did
social order, as though it were normal and 2. ".............. an active conviction
even proper and just. he expect to earn
which informs and inspires every (c) She asked the boy how much he
If we are sincere in our professions thought, word and deed". What
of democracy, we should not shut our expect earning
does it mean?
eyes to the most obvious defects of the (d) She asked the boy how much does
(a) A democrat admires the virtues
present social order. A system which does the expect to earning.
of democracy in whatever he
not offer security and decent employment 8. In which sentence 'As' is used as
thinks, says and does.
to multitudes of trained youngmen (b) A democrat has sound self conjunction?
suffers from fundamental vice. Society confidence in regard to the (a) We walked as fast as we could.
is in danger of splitting to pieces if the correctness of what he thinks, (b) She likes the same colour as I do.
few who have the benefits of civilisation says and does. (c) As he was poor I helped him.
are not willing to share them with the (c) A democrat always thinks right, (d) None of these
rest. No state is stable unless it procures speak aright and acts aright. 9. What is the meaning of 'Indigent'?
for all its members the essentials of a (d) A democrat believes in the
good life. We acknowledge that health is (a) Rich (b) Brave
concept of equality and this
better than disease, sufficiency is better (c) Intellectual (d) Poor
belief is reflected in his thinking,
than poverty, shelter is better than cold speech and action. 10. Punctuate the following sentence:
and exposure, ease of mind is better than 3. What is democracy essentially The creator answered yes but the
racking anxiety. It is our duty to obtain according to the author? accepting the burden you will
these essentials of civilised life for the (a) A form of government show your greatness of heart. Find
mass of the population, to work for basic (b) A political arrangement out the correct one.
economic justice for all, if necessary (c) A pattern of life itself (a) The creator answered "Yes but
by the imposition of higher taxes on (d) Both (a) and (b) by accepting the burden you will
incomes, land, value and inheritance. 4. The present constitution of our show your greatness of heart."
Riches were created by the maker for society is based on: (b) The creator answered "Yes". But
being spent on social purpose. It was by accepting the burden you will
(a) the principle of equality
Blackstone, not Lenin, who wrote “The show your greatness of heart.
(b) the principle of human dignity
274 Practice Set-22

(c) The Creator answered, "Yes but Directions (17): Choose the word 24. If earth is made of iron (density
by accepting the burden you will opposite in meaning to the given word. 7.5 g/cc.), then acceleration due
show your greatness of heart." 17.SEGREGATE to gravity will be:
(d) None of these (a) More
(a) join (b) unite
11. Antonym of 'Zenith' is – (b) Less
(c) separate (d) aggregate
(a) Nadir (b) Important (c) Zero
Directions (18): Four alternatives are
(c) Summit (d) Heaven (d) S a m e a s r a d i u s h a s n o t
given for the idiom/phrase italicised
12. Choose the correct antonym of the in the sentence. Choose the alternative changed
word, 'inferior'.
which best expresses the meaning of 25. Electric field at point P is given
(a) low (b) superb the idiom/phrase. → →
(c) good (d) superior 18.He was progressing by leaps and b y E = r E0 . T h e t o t a l f l u x
13. Fill in the blank with the sutiable bounds because of his hard work. through the given cylinder of
preposition. radius R and height h is:
(a) rapidly (b) slowly
Some trains are run ______
(c) peacefully (d) strongly
electricity.
Directions (19): A word has been
(a) from (b) on
written in four different ways out of
(c) with (d) by
which only one is correctly spelt. Find
14. Directions (14-15): In the following
questions the sentences have been the correctly spelt word.
given in Active/Passive voice. From 19.(a) Successfully
the given alternatives, choose the (b) Sucessfully (a) E0πR2h (b) 2E0πR2h
one which best expresses the given (c) Succesfully
(c) 3E0πR2h (d) 4E0πR2h
sentence in Passive/Active Voice. (d) Successfuly
26. In an A.C. generator, a coil with
"God helps those who help Directions (20): Identify the misspelt
themselves." N turns, all of the same area A
word .
20.(a) Elementary (b) Secondary and total resistance R, rotates
(a) Those who help themselves will
be helped by God with frequency ω in a magnetic
(c) Lanudary (d) Momentary
(b) Those who help themselves are field B. The maximum value of
helped by God. PHYSICS e.m.f. generated in the coil is:
(c) They are helped by God who have 21. C a l c u l a t e t h e d e - B r o g l i e (a) NABRω (b) NAB
helped themselves wavelength of an electron in a (c) NABR (d) NABω
(d) Who helped themselves they are metal at 27°C. Given Boltzmann 27. A boy aims at a bird from a point
helped by God. c o n s t a n t k = 1 . 3 8 × 1 0 –23 at a horizontal distance of 100 m.
15. They will ask you a lot of questions J mol–1 K–1:
at the interview. The gun can impart a velocity of
(a) 6.2 Å (b) 3.1 nm 500 m/s to the bullet. At what
(a) You will be asked to lot of (c) 3.1 Å (d) 6.2 nm
questions at the interview height above the bird must he
22. The rms value of the electric
(b) You are asked a lot of questions aim his gun in order to hit the
field of the light coming from
at the interview bird?
the sun is 720 N/C. The average
(c) You are being asked a lot of (a) 20 cm (b) 40 cm
total energy density of the
questions at the interview. (c) 50 cm (d) 100 cm
electromagnetic wave is:
(d) You were being asked a lot of
questions at the interview. (a) 4.58 × 10–6 J/m3 28. There are two values of time for
Directions (16): Sentences are given (c) 6.37 × 10–9 J/m3 which a projectile is at the same
with blank to be filled in with an (c) 81.35 × 10–12 J/m3 height. The sum of these two times
appropriate word (s). Four alternatives (d) 3.3 × 10–3 J/m3 is equal to:
are suggested for each question. 23. A given mass of a gas has 100 3T 4T
Choose the correct alternative out of cm3 volume at 373.15 K and 1 (a) (b)
the four: 2 3
atmospheric pressure. Its volume
16. The furniture .......... to be delivered at the same temperature and 4 3T
(c) (d) T
today. atmospheric pressure will be: 4
(a) is (b) are (a) 200 cm3 (b) 400 cm3 (T is the time of flight of the
(c) have (d) were (c) 25 cm3 (d) 100 cm3 projectile)
Practice Set-22 275

29. In the given loop, the magnetic µ0 I  1


field at the centre O is: (c) 1 −  into the page
2r π
µ0 I  1 
(d) 1 +  out of the page
2r  π 
(a) Zero (b) RvB 37. A current of 2A flow in a system
(c) vBL/R (d) vBL of conductors as shown in the
33. T h e p re s s u re re c o r d e d b y figure. The potential difference
constant volume gas thermometer VA – VB is:
µ0 I  r1 + r2  at 0°C, 100°C and at room
(a)  r r  out of the page
4 1 2 temperature are 8.00 × 106 N/
µ 0 I  r1 + r2  m2, 1.10 × 107 N/m2 and 8.51 ×
(b) into the page 106 N/m2:
4  r1r2 
(a) 16°C (b) 16.6°C
µ 0 I  r1 − r2  (c) 17.0°C (d) 18.0°C
(c) out of the page
4  r1r2  34. In an explosion, a rock breaks
into three parts. The two pieces
(d) µ 0 I  r1 − r2  into the page move in mutually perpendicular
4  r1r2  directions. The piece of 1 kg (a) + 2 V (b) + 1 V
moves with speed 12 m/s and (c) – 1 V (d) – 2 V
30. A man has five resistors each
1 other piece of mass 2 kg moves 38. Two waves of same frequency
of value Ω . The minimum with 8 m/s speed. If the third and of intensity I0 and 9I0 produce
5
resistance he can obtain by piece moves with speed of 40 interference. If at a certain point the
combining them is: m/s, then its mass will be: resultant intensity is 7I0, then the
1 minimum phase difference between
1 (a) 5 kg (b) 0.5 kg
(a) Ω (b) 25 Ω the two sound waves will be:
50 (c) 0.25 kg (d) 1 kg (a) 90° (b) 100°
1 1 e2 (c) 120° (d) 110°
(c) Ω (d) None of 35. The dimensions of × are:
ε 0 hc 39. A thermometer has its fixed
10 these points marked as 5 and 95.
31. A non-conducting solid sphere of (a) [A2L–3T4M–1] What is the correct temperature
radius R is uniformly charged. (b) [A–2L–4T3M] in °C when the thermometer
The magnitude of electric field (c) [A0M0L0T0] reads 59°?
due to the sphere at a distance r (a) 54°C (b) 60°C
from its centre: (d) [AT2L–3M–1]
(c) 64°C (d) 34°C
(a) Increases as r increases for r 36. A long straight conductor,
40. Which of the following has
<R carrying a current I, is bent into
minimum proportion of available
(b) Decreases as r increases for 0 the shape shown in the figure. volume filled by hard spheres
<r<∞ There is no cross arranged in various structures?
(c) Decreases as r increases for R
<r <∞ (a) Simple cubic
(d) Both (a) and (c) are correct (b) bcc
32. A conducting square loop of side (c) fcc
L and resistance R moves in its (d) Diamond
plane with a uniform velocity v 41. The ratio of nuclear magneton
perpendicular to one of its sides. contact at P. The radius of the and Bohr magneton is:
A magnetic induction B constant circular loop is r. The magnetic
in time and space, pointing me mp
field at the centre of the loop is:
perpendicular and into the plane (a) (b)
at the loop exists everywhere π0 I  1  mp me
(a) 1 +  into the page
with half the loop outside the 2r  π  2mp
field, as shown in figure. The µ I  1 (c) memp (d) me
induced e.m.f. is: (b) 0 1 −  out of the page
2r π
276 Practice Set-22

42. If there are n capacitors in parallel


connected to V volt source, then
the energy stored is equal to:


( π
4
)
sin  ω 13 + ω 20 π +  is equal to:
 π
(b) f   = 1
 4
1 (c) f(x) = tan x + cot x
1
nCV 2 (a) − 3 (b) −
(a) CV (b) 2 2  π
2 (d) f   = 2
1 3  4
1 (c) (d)
(c) CV2 (d) CV 2 54. What is the slope of the line
2n 2 2
48. If α and β are the roots of the perpendicular to the line
43. Two tuning forks A and B have
frequency 256 Hz and 262 Hz equation ax2 + bx + c = 0, then x y
what is the value of: + =1
respectively. When a third tuning 34
fork is sounded with A, few α β b 3 3
+ + =? (a) (b) −
beats are heard. When the same β α a 4 4
tuning fork is sounded with B, the
(a) –1 (b) 0 4 4
number of beats heard is doubled. (c) − (d)
The frequency of tuning fork C is: (c) 1 (d) 2 3 3
(a) 257 (b) 258 49. If the sum of the series 2, 5, 8, 55. The value of
(c) 259 (d) 260 11, ..... is 60100, then n is equal
1 1 1 1
44. A lens forms a sharp image on to: − + − + ...∞ is:
(a) 100 (b) 200 1.2 2.3 3.4 4.5
a screen. On inserting a parallel
sided glass slab between the (c) 150 (d) 250 4 e
lens and the screen, it is found 50. What is the term independent of (a) loge (b) loge
e 4
necessary to move the screen a x in the expansion of
distance d away from the lens in 2
(c) loge 4 (d) loge 2
order for the image to be sharp  3 x −2 1 56. What is the radius of the circle
again. If the refractive index of (1 + x + 2x ) 
3 −  ?
 2 3x touching X-axis at (3, 0) and
the material of the slab is n, the
(a) 1/3 Y-axis at (0, 3)?
thickness of the slab is:
(b) 19/54 (a) 3 units (b) 4 units
d (c) 1/4 (c) 5 units (d) 6 units
(a) nd (b)
n (d) No such term exists in the 57. If log10 2 = 0.3010, then what is
(n − 1)d nd expansion the number of digits in 2064?
(c) (d) 2x − π
n n −1 51. lim is equal to: (a) 81 (b) 82
45. The measurement 8.987652 km is x→ π / 2 cos x (c) 83 (d) 84
rounded to 4 significant figures. 58. For all n∈N, 41 n – 14 n is a
(a) 1 (b) 2
The value of measurement will be
written as: (c) –2 (d) 0 multiple of:
(a) 8.9876 (b) 8.9877 52. If y = sin –1 ( 1 − x ) + cos −1 x, (a) 26 (b) 27
(c) 8.987 (d) 8.988 then dy/dx is equal to: (c) 25 (d) None of these
1 −1 59. The solution of the equation
MATHEMATICS (a) (b)
dy y2 − y − 2
46. Let N be the set of integers. A x (1 − x )  x (1 − x ) = is:
dx x 2 + 2 x − 3
relation R on N is defined as R = {(x,
y) |xy > 0, x, y ∈ N|}. Then, which 1 1 y−2 1 x −1
(c) (d) None of these (a) log = log +C
one of the following is correct? x (1 + x ) 3 y +1 4 x+3
(a) R is symmetric but not reflexive
(b) R is reflexive but not symmetric dx 1 1 y +1 1 x+3
53. If ∫ sin 4 x + cos4 x = tan −1 f ( x ) , (b) log = log +C
(c) R is symmetric and reflexive 2 3 y−2 4 x −1
but not transitive
then 1 y +1 1 x+3
(d) R is an equivalence relation (c) log = log +C
tan x − cot x 4 y−2 3 x −1
47. If ω is imaginary cube root of (a) f(x) =
unity, then 2
(d) None of these above
Practice Set-22 277

x lying in the interval: 72. Which of the following items is


60. Following table gives the mean
(a) [–1, 1] under the State List ?
and variance of monthly demand
for four Products A, B, C and D (a) Agriculture
(b) [0, 1]
in a super market (b) Criminal Law
(c) [–1, 0] (c) Education
Prodcut A B C D
Mean demand 60 90 80 120 (d) [–1/ 2 , 1/ 2 ] (d) Defence
variance 12 25 36 16 73. Who among the following was
67. A variable chord PQ of the
the first Viceroy of India ?
For which product the demand is parabola y 2 = 4ax, subtends
(a) Lord Cornwallis
consistent? a right angle at the vertex, (b) Pitt
(a) Product A (b) Product B then the locus of the points of (c) Lord Canning
(c) Product C (d) Product D intersection of the normal at P (d) Robert Clive
61. The speed v of a particle moving and Q is: 74. The only bird that flies backward
along a straight line is given (a) a parabola is :
by a + bv = x 2 (where x is its (b) a hyperbola (a) Sparrow
distance from the origin). The (c) a circle (b) Koel
acceleration of the particle is: (d) None of these (c) Siberian Crane
x 68. What is (1 – sin2θ) (1 + tan2θ) (d) Humming bird
(a) bx (b) 75. Which of the following is an iron
a equal to?
ore ?
x x (a) sin2θ (b) cos2θ
(c) (d) (a) Bauxite (b) Magnetite
b ab (c) tan2θ (d) 1
(c) Lignite (d) Nitrite
62. A person draws two cards with  π
replacement from a pack of 52 69. If cotθ = 2cosθ, where   < θ 76. The letter ‘G’ used in ‘2G
2 Spectrum’ stands for :
cards. What is the chance that he < π, then what is the value of θ?
gets both the cards of same suit? (a) Governance (b) Global
(a) 1/4 (b) 3/13 5π (c) Generation (d) Google

(c) 1/16 (d) None of these (a) (b) 77. Which country is set to ban sale
63. In how many different ways can 6 3 of petrol and diesel cars from
five boys and five girls form a 3π 11π 2030?
circle such that the boys and (c) (d) (a) Scotland
4 12
girls set alternate? (b) Ireland
(a) 4! (b) 2880 1 p q (c) United Kingdom
(c) 5! (d) None of these 70. If the inverse of  0 x 0  is
(d) Iceland
64. what is the area bounded by the  0 0 1 
 1 − p − q 78. The largest 5 digit number
lines x = 0, y = 0 and x + y + 2 = 0? 0 1 0  then what is the
 0 0 1  exactly divisible by 95 is :
1 (a) 99936 (b) 99935
(a) sq unit (b) 1 sq unit
2 value of x? (c) 99940 (d) 99933
(c) 2 sq units (d) 4 sq units (a) 1 (b) 0 79. If (cotAcotB + 1) / (cotB – cotA)
65. Which of the following is not an = x, then the value of x is
1 1
identify? (c) –1 (d) + (a) cot (A + B) (b) cot (A – B)
p q
(a) sin4A – cos4A = sin2A – cos2A (c) tan (A – B) (d) tan (A + B)
tan 2 A − 1 80. 0.6 × 0.06 = ?
(b) = 2 sin2 A – 1 REASONING AND GENERAL
2 (a) 0.36 (b) 0.0036
tan A + 1 AWARENESS (RAGA) (c) 0.036 (d) 3.6
(c) sin6A + cos6A 71. The latest volume of foodgrains 81. The curved surface area of a
= 1 + 3 sin2A cos2A to be given per family as hemisphere is 27.72 sq cm and
(d) tan2A – sin2A = tan2A sin2A determined under ‘Annapurna volume is 19.404 cubic cm, find
Scheme’ is : its radius.


(
66. The formula sin −1  2 x 1 − x 2  )  (a) 35 kg (b) 20 kg (a) 4.2 cm (b) 2.1 cm
(c) 10 kg (d) 40 kg (c) 0.7 cm (d) 1.05 cm
= 2 sin–1x is true for all values of
278 Practice Set-22

82. When a number is increased by (a) CARPECIT 93. Three positions of a cube are
42, it becomes 121% of itself. (b) CAREPCIT shown below. What will come
What is the number ? (c) CARTICE opposite to face containing '1'?
(a) 504 (b) 200 (d) CARECEIT
(c) 630 (d) 378 88. I was facing East from where I
83. Rajan sells a machine for ` 52 turned to my left and walked 12
lakhs at a loss. Had he sold it ft, then I turned towards right (a) 2 (b) 3
for ` 62 lakh, his gain would and walked 6 ft. After that I (c) 4 (d) 6
have been 9 times the former walked 6 ft in South direction Direction (94): Select the related
loss. What is the cost price of the and at last I walked 6 ft in the word/letters/number from the given
machine ? West. Then, in which direction alternatives.
(a) ` 61 lakhs am I standing from the original 94. 15 : 62 : : 16 : ?
(b) ` 53 lakhs point?
(a) 66 (b) 64
(c) ` 67.77 lakhs (a) West (b) East
(c) 32 (d) 56
(d) ` 45 lakhs (c) South (d) North
95. In the following figure, rectangle
84. A mason can build a wall in 89. Anil is taller than Sunny and
represents Actors, circle
18 hours. After 9 hours he takes Sunny is shorter than Baby
represents Curators, triangle
a break. What fraction of the Anil is taller than Bose whose
represents Poets and square
wall is yet to be built ? height is less than Sunny Baby
represents Fathers. Which set of
is shorter than Anil. Who is the
(a) 0.9 (b) 0.25 letters represents Poets who are
shortest among them?
(c) 0.5 (d) 0.2 not Fathers?
(a) Anil (b) Baby
85. Simple interest on a certain sum (c) Sunny (d) Bose
of money for 3 years at 16% per
annum is half the compound Direction (90): A series is given with
F
interest on ` 12000 for 2 years at one term missing. Choose the correct
alternative from the given ones that E
10% per annum. The sum placed G
on simple interest is : will complete the series. D
90. AZ, BY, CX,?
(a) ` 5250 (b) ` 1325
(a) CW (b) DW C
(c) ` 2100 (d) ` 2625 A
(c) DX (d) DY
Direction (86): Real the information B I
Direction (91): Study the following
carefully and answer the following information carefully and answer the H
questions: questions given below:
If A + B means A is the father of B P, Q, R, S, T, V and W are sitting
J
If A × B means A is the sister of B around a circle facing at the centre. V
If A $ B means A is the wife of B is second to the left of P and second to (a) BI (b) FE
If A % B means A is the mother of B the right of W. T is third to the right of (c) HB (d) DC
If A ÷ B means A is the son of B Q and is not an immediate neighbour
Directions (96-100): Study the
86. Which among the following of V, S is third to the right of R.
following information carefully and
expressions is true if Y is the son 91. Who is to the immediate left of
answer the questions given below it.
S?
of X is definitely false? Six lectures—A, B, C, D, E and F —
(a) V (b) T
(a) W % L × T × Y ÷ X are to be delivered from Monday to
(c) Q (d) W
(b) W + L × T × Y ÷ X Sunday, one lecture every day.
Direction (92): In following questions
(c) X + L × T × Y ÷ W (i) Lecture C cannot be delivered on
a stands for equal to’; b for ‘greater
(d) W $ X + L + Y + T than’; γ for ‘less than’ and d for ‘not Friday.
87. In a certain code language, equal to. (ii) Lecture A is delivered immediately
“DISORDER” is written 92. If a2x a byz, czx a b2y and c2z a after lecture D.
as “OSIDREDR”. How is axy, then (iii) There should be a gap of two days
“PRACTICE” written in that (a) abc a xyz (b) abc b xyz between the lectures B and F.
code language? (c) abc d xyz (d) abc g xyz (iv) There is one holiday except
Practice Set-22 279

Saturday. Lecture F is delivered 97. On which day was holiday? 99. Which of the following was the
on the next day of holiday. (a) Sunday last tecture?
(v) Lecture E is delivered on (b) Friday (a) A (b) C
Wednesday and it is not (c) Monday (c) B
immediately followed by lecture F. (d) Cannot be determined
(d) None of the above
96. On which day was lecture D 100. Which of the following
98. How may lectures were delivered
delivered? statements is not necessary to
between F and D?
(a) Friday (b) Saturday determine the order of lectures?
(a) None (b) One
(c) Sunday (d) Thursday (a) (i) (b) (ii)
(c) Two (d) Three
(c) (v) (d) (i) and (ii)

Answers with Explanation


ENGLISH rank, degree or grade. = 8.85 × 10–12 × (720)2
13. (d) Suitable preposition here = 4.58 × 10–6 J/m3
1. (d) According to author, the
will be 'by' i.e. some trains are run by
underlying theme of democracy is PV
electricity. 1 1 P2V2
equality. 23. (c) =
14. (b) Since the tense in active T1 T2
2. (d) Conviction marks actions and
voice is simple present (helps) therefore
deeds. 1×100 4 × V2
in passive voice 'are helped' would be ⇒
=
3. (c) Democracy is a way of life. 373.15 373.15
used.
4. (a) Equality runs parallel with
Those who help themselves are 100
democracy. ∴ V2 =
helped by God.
5. (d) The correct word for Celebasy 4
15. (a) You will be asked a lot of
is - Celibacy, which means the state
questions at the interview.
of being unmarried voluntarily. Hence = 25 cm3
16. (a)
option (d) is correct. 24. (a) Since g ∝ ρ since density of
17. (b) The meaning of the word
6. (d) Correct spelling is platitude iron (7.5 g/cc) is more than of earth crust
Segregate are Cut-off, Discriminate,
which means a falt, dull or trite remark. (5.5 gm/cm), the value of g will increase.
Against, Dissociate, Isolate, Quarantine,
7. (b) The verb 'do' will change to → →
Separation, etc. Therefore, its Antonym 25. (c) Here, E = E0 r
'did' and 1st person 'you' to 3rd person
would be Unite.
'he' and the verb 'expect' will remain the
same. Thus She asked the boy how much
18. (a) rapidly r = xiˆ + yjˆ + zkˆ
did he expect to earn. Hence option (b) 19. (a) 20. (c) →
E = E0 xiˆ + E0 yjˆ + E0 zkˆ

is correct.
8. (c) In sentence, As he was poor PHYSICS φ0 = the electric flux per unit volume
I help him. 'As' is used as conjunction. h 3
9. (d) The meaning of Indigent is – 21. (d) λ = ,E = kT ∂E x ∂E y ∂E z
2mE 2 = + +
poor or lacking food, clothing and other ∂x ∂y ∂z
necessities of life because of poverty, ⇒ T = 273 + 27
needy, impoverished. = 300 K φ = E0 + E0 + E0 = 3E0

10. (c) The correct sentence is – Combining, ∴ Net flux= φ0 × volume

The Creator answered "Yes but by h 2 2
accepting the burden you will show your λ = = φ0 × πR h = 3E0 πR h
greatness of heart." 3mkT 26. (d) The e.m.f. generated would be
11. (a) The antonym of Zenith is = 6·6 × 10 −34 m maximum when flux (cutting) would be
– Nadir, Zenith means the point on the 3 × 9·1 × 10 −31 × 1·38 × 10 −23 × 300 maximum i.e., angle between area vector
celestial sphere vertically above a given of coil and magnetic field is 0°. The e.m.f.
position or observer and Nadir is the ⇒ λ = 6.2 × 10–9 m
generated is given by (as function of
lowest point vertically below. = 6.2 nm. time).
12. (d) The Antonym of inferior is 22. (a) Tatal average energy
e = NBAω cos ωt
superior. Inferior means lower in status, 2
= ε 0 Erms ⇒ emax= NABω
280 Practice Set-22

27. (a) R = 100 m, u = 500 m/s × ×A × × B MLT −2 L2 A2T 2


= ×
R × × × × 2
AT 2
ML T −1 + LT −1
2
t = Time taken =
u × ×L × × v
× × × × = [A0M0L0T0]
= 100 = 1 sec × ×D × × C
500 5 The side AD is cutting the flux and e.m.f. 36. (d) B =
µ0 I µ0 I µ0 I
+ +
1
Now, h = gt 2 induced across this side is BvL with 4π r 2r 4π r
2 corner A at higher potential.
µ0 I  1 
33. (c) For gas thermometer, the B=
1 + 
1 1 2r  π 
= × 10 × unknown temperature is given by:
2 25 p − p0 37. (a) Current branches equally 1 A
t(°C) = 1 × 100
each, along ABC and ADC, as they have
p100 − p0
= 1 m = 20 cm equal resistance. Potential difference
5 8·51 × 10 6 − 8·00 × 10 6
= × 100 (VA – VB) = current × resistance = 2 × 1
28. (d) tOA = tBC 1.1 × 10 7 − 8.00 × 10 6
V. Since current flows from A to B, A is
Y at a higher potential. Hence, VAB = + 2 V.
0.51 51
= × 100 = = 17°C 38. (c) Intensity ∝ Amplitude2
3.0 3
A B C ∴
a1 = I0
34. (b) P1 = 1 × 12
X = 12 kg m/s
O and a2 = 3 I 0
⇒ P2 = 2 × 8
∴ tOA + tOB = tBC + tOB = 16 kg m/s The resulting amplitude is
= T
2 2 given by
29. (b) Bstraight wire = 0 ⇒
P3 = 12 + 16
2 2
–→
A2 = a1 + a2 + 2a1a2 cos δ
µ0 I = 20 kg m/s (along OD )

⇒ Bsmall arc = ⊗ 7I0
4r2
=I 0 + 9 I 0 + 2 I 0 × 3 I 0 cos δ
µ I P2 (16) ⇒ – 3 = 6 cos δ

⇒ Bbig arc = 0 ⊗
4r1 R 1
O ⇒
cos δ = −
µ 0 I  r1 + r2  P1 (12) 2
⇒ Btotal =
⊗ ∴ δ = 120°
4  r1r2 
P3 (20) 39. (b) The correct interval of 100°C
30. (b) Minimum resistance is obtained (between fixed points) has been marked
when all are in parallel. P3 20
Mass = = wrongly as 90° on faulty thermometer.
ρr V 40 A temperature of 59° on this scale has
31. (d) E = r<R
3ε 0 = 0.5 kg interval (59 – 5) = 54° which is equivalent

E ∝r r<R 35. (c) h = [ML2T–1] 100
to 54 × = 60° correct interval on
c = [LT–1] 90
Q Celsius Scale. Thus, correct temprature

E = r>R 2
4πε 0 r 2  AT  1 is 60°C.
ε0 =
1 MLT  L 
−2  40. (d) It is because the diamond is the

E ∝ hardest substance.
r2  1 q1q2 
Hence, both (a) and (c) are correct.  From F = 4πε d 2  eh
0 41. (a) µN =
32. (d) As the side BC is outside the 2m p
and e2 = (AT)2 e → charge
field no e.m.f. is induced across BC.
Since, AB and CD are not cutting any 2 and µB = eh
flux the e.m.f. induced across these two ∴ 1 ×e
2me
ε 0 hc
sides will also be zero.
Practice Set-22 281

µN me Hence, R is an equivalence
9−r r

= relation.  3 x −2   1
µB m p = Cr  9
  − 
47. (b)  ω13 + ω20= ω + ω2 = –1  2  3x
42. (b) Energy stored by any system
of capacitors is


( ) π
sin  ω13 + ω 20 π + 
4 = (–1)r 9Cr  3 

9 −r

1  2
= CnetV 2
2
 π 1
Where V is source voltage. ∴ = sin  −π + 
= x −18 + 2r − r
 4
1 3r
2
Enet = 2 nCV 3π 1 For coefficient of x0, x–1 and
= − sin =−
4 2 x–2
43. (b) Let frequency of C be x
48. (b)
Given, quadratic equation is we get
∴ (262 – x) = 2(x – 256)
ax2 + bx + c = 0 –18 + r= 0, 18 + r = –1
∴ x = 258
Let (α, β) be the roots of given and –18 + r = –2
 1 equation ⇒ r = 18, 17, 16
44. (d) Shift = d = t 1 − 
n b which is not possible
b
∴ α + β= – and αβ =
nd a a Thus, no such term exists

t=
n −1 in the expansion of given
Now, we have
expression
45. (d) The significant figures are 4,
therefore only four digits will be in the α β b b
+ + = α+β + 51. (c) Given integral is of the form
measurement. The next fifth number is 6 β α a a
αβ 0
which is greater than 5, hence the number
7 to be rounded off must be increased by −b a b 0
one, hence the measurement has value = × +
a b a Using L Hospital rule
8.988.
2x − π 2
b b lim = lim = −2
MATHEMATICS = − + =0 x →π /2 cos x x →π /2 − sin x
a a
46. (d)  R= {(x, y) |xy > 0, x, y∈N}
n 52. (b) Put x = cos2 ⇒ θ =
Reflexive Let x, x ∈ N, then 49. (b) Here, [2 × 2 + (n – 1) × 3]
2 cos–1 x
xRx is x2 > 0, which is true,
= 60100 ∴ y = sin– 1 − x + cos−1 x

So , R is reflexive.
Symmetric
 n 
=sin–1 sin 2 θ + cos−1 cos2 θ
∵ Sn = 2 {2 a + (n − 1) d}
Let x, y ∈N, then xRy is xy > 
 ⇒

y = sin–1sinθ
0 ⇒ yx > 0 + cos–1cosθ
⇒ 3n2 + n – 120200 = 0

∴ xRy ⇒ yRx ⇒ y = θ + θ = 2θ
⇒ (n – 200) (3n + 601) = 0

So, R is also symmetric, ⇒ y = 2cos–1 x

n = 200
Transitive
dy
(Leaving negative value as 2 1
Let x, y, z ∈ N ∴
= − ×
number of sequence cannot dx 1− x 2 x
Now, xRy is xy > 0, because be negative) dy −1
x, y ∈ N =
53. (a) We have,x (1 − x )
50. (d) Let Tr+1 be the term independent dx
and yRz is yz > 0, because y, 9
−2
z∈N of x in  3 x − 1  dx
∫ sin 4 x + cos4 x =
1
tan −1 f ( x )
 2 3 x  2
∴ xRz is xz > 0, because x, z

∈N Dividing numerator and
∴ Tr + 1

denominator by cos 4x, we
So, R is transitive.
have
282 Practice Set-22

(1 + tan x ) sec
2 2
x
[When, put x = 1 in loge (1 + x)]
and loge 2
= 27 = 1 × 27,
which is a multiple of 27 that
I= ∫ 1 + tan 4 x
dx
is true
 1   1   1 
Put tan x = t, ⇒ sec2x dx = dt = 1 –   −   −   − .... ...(ii Let it is true for n = k,
2.3 4.5 6.7
1 + t2 x i.e., 41n – 14k= 27λ

∴I= ∫ 1+ t4 dt [When, put x
....(i)
= –1 in loge (1 – x)] For n = k + 1, 41k + 1 –14k + 1
 1 On adding Eqs (i) and (ii), we get
 1 + 2  = 41k41–14k 14

t 2loge 2
= ∫ 2
dt = (27λ + 14k)41 –14k14
 1  1 1   1 1  [using Eq. (i)]
 t −  + 2 = 1 + − + − + ...
t  1.2 2.3   3.4 4.5 
 = 27λ × 41 + 14k × 41
1 – 14 k × 14
⇒ 2 loge 2 – 1
Put t − = z = 27λ × 41 + 14 × 41k(41 –
k
t
= ... 14)
 
⇒ 1 + 1 dt = dz
= 27λ × 41 + 14k × 27
 2
 t  = 27(41λ+14k),
⇒ 1 − 1 + 1 − 1 + .. which is multiple of 27.
dz 1.2 2.3 3.4 4.5
∴f= ∫−
2
Therefore, P(k + 1) is true
t +2 4
= loge 4 – loge e = loge   when P(k) is true. Hence, from
 e the principle of mathematical
= 1 tan –1 z induction, the statement is
2 2 56. (a) Radius of the circle, AC =
true for all natural number n.
BC
 1 Y dy y2 − y − 2
t −  59. (a) = 2
=1 tan –1  t  dx x + 2x − 3
2 2
dy dx
⇒ =
I =
1
tan –1
( tan x − cot x ) 2
y − y−2 x2 + 2 x − 3
(0, 3) C (3, 3)
2 2 B
1 1 1 
3 ⇒  −  dy
∴ f(x)=

tan x − cot x
and 3  ( y − 2 ) ( y + 1) 
X
2 O 3 A (3, 0)
 
AC = OB = 3 and BC = OA = 3 = 1  1 − 1  dx
 π 1−1 ∴ Radius = 3 units 4  ( x − 1) ( x + 3) 
f  =
=0
 4 2 57. (d) Let x = 2064 1 y−2

On taking log on both sides, ∴ log

54. (d) Since, the slope of the line 3 y +1
we get
x y 3
logx = 64 log 20 = 64 ×
+ = 1 is − = 1 log x − 1 + C
4 3 4 1.3010 = 83.264 4 x+3
∴ Number of digits in 2064 =
∴ Slope of line perpendicular 60. (d) Since, coefficient of variance
83 + 1 = 84
this line SD
58. (b) Let the given statement be = × 100
 −1  4 P(n) Mean
= −  =
 3 / 4  3
i.e., P(n)
Coefficient of variance A
55. (a) We know that loge 2 = 41n –14n is a multiple of 27
12
For n = 1, = × 100 = 57%
= 1 + 1 + 1 + ...  ...(i) 60
1.2 3.4 5.6 i.e., P(1) = 41' – 14'
Practice Set-22 283

Coefficient of variance B There will be 5 places, π π π θ π


between two boys, these five − ≤θ ≤ , − ≤ ≤
= 25 × 100 = 5.5% 2 2 4 2 4
places can be filled by 5 girls
90
in 5! ways. 1 θ 1 1 1
Coefficient of variance C − ≤ sin ≤ , − ≤x≤
The required number of ways 2
2 2 2 2
36 = 4!×5! = 2880
= × 100 = 7.5%
80 64. (c) Given equation of lines So, x lines between
Coefficient of variance D x = 0, y = 0 and x + y + 2 = 0  1 1 
Y − , 
16 
2 2
= × 100 = 3.3%
120 67. (d)
Let P (at12, 2at1) be and Q be
x=0
Hence, we see that minimum coefficient (at22, 2at2) since PQ subtends
of variance is D, hence D product is (–2, 0)
(-2, 0) A a right angle at the vertex (0,
X' X 0)
consistent. 0

61. (c) Given equation is a + by2 = x2 B


(0, –2)
(0,–2) Hence t2 = –4 ...(i)
On differentiating, we get It (h, k) be the point of
y=0 intersection of normals at P
 dv  dx Y'
and Q, than
0 + b  2 v  = 2 x
 dt  dt Required area= Area of ∆ h = 2a + a(t12 + t22 + + t1t2)
OAB ....(ii)
dv dx
⇒ vb
= x 1
dt dt = 1 OA × OB = × 2 × 2 and k = – at1t2(t1 + t2) ....(iii)
2 2 In order to find the locus of
dv x dx
⇒ = . = 2 sq units (h, k) we have to eliminate
dt vb dt t1 and t2 between Eqs. (i), (ii)
65. (c) sin6 A + cos6 A
dv x  dx  = (sin2 A + cos2 A) (cos4A – and (iii)

= ∵ = v
dt b dt  cos2 A sin2A + sin4 A) k = 4a(t1 + t2) ...(iv)
= cos4 A + sin4 A – cos2A sin2 [from Eqs. (i) and (iii)]
62. (a) Total ways = 52 × 52,
A and h –2a
because cards are drawn with
replacement. [Because sin2A + cos2 A = 1] = a[(t1 + t2)2 – t1t2)
One suit out of four in 4C1 ways and two = (cos2 A + sin2 A)2 – 3 cos2
cards in 13 × 13 ways. A sin2 A ⇒ h – 2a= a  k 2

= 1 – 3 cos2 A sin2 A  2
+ 4 
Favourable ways = 4C1 × 13 × 13 16 a 
∴ sin6 A + cos6 A ≠ 1 + 3 cos2
   So, required probability
A sin2 A
[from Eq. (iv)]
4
C1 × 13 × 13 1
=
52 × 52
=
4 66. (d)(1 − x ) = 2sin x
sin–1 2 –1
⇒ h – 6a =

k2

Let sin {2 x (1 − x ) } = θ,
63. (b) After fixing up one boy on 16 a
2
–1
Hence, the required locus is
the table the remaining can be
arranged in 4! ways but boys 2sin–1x = θ, sin–1 x y2 = 16a(x – 6a)
and girls are to sit alternate. 68. (d) (1 – sin2θ) (1 + tan2θ)
θ θ
B1 = ,sin = x = cos2θ – sec2θ
2 2
B2 B5 1
As, we know = cos2θ × =1
cos2
–1 ≤ sin θ ≤ 1,
69. (a) Given, cot θ = 2cosθ
B4 – 1 ≤ 2x 1 − x2 ≤ 1 ⇒ cotθ(1 – 2sinθ) = 0
B3
284 Practice Set-22

π < θ < π, cot θ ≠ 0 78. (c) The largest 5 digit number 83. (b) Let the loss be ` x.
For = 9999 Then gain = 9x
2
1 95 99999 1052 S.P. = C.P. – Loss
∴ 1–2sinθ= sinθ =
95 C.P. = S.P. + Loss
2 499
475 = 5200000 + x ...(i)

⇒θ=
249 Again S.P. = C.P. + gain
6 190 5200000 = C.P. + 9x
59 C.P. = 6200000 – 9x ...(ii)
1 p q Hence, divisible by 95 = 99999 – 59 From equation (i) and (ii)
70. (a) Let, A = 0 x 0  and
0 0 1  = 99940 5200000 + x = 6200000 – 9x
79. (b)It is given that 10x = 6200000 – 5200000
1 − p −q  = 10,00000
cot A cot B + 1
B = 0 1 0 x = 1000000
cot B – cot A = 100000
0 0 1  x=
10
1 p q  cos A cos B The cost price of machine
. +1
Thus B = 0 1 0 
–1
sin A sin B = C.P. + x
=
0 0 1  cos B cos A = 5200000 + 100000

But B is inverse of A, sin B sin A = 5300000 = ` 53 Lakhs.
therefore A = B–1 84. (c) A mason can build a wall in
cosA. cosB + sinA sinB 18 hours
1 p q  and after a hour he takes a break.
⇒ 0 x 0 
sinA. sinB
= \ The fraction of the wall to be
0 0 1  sinA. cosB – cosA. sinB
sinA. sinB 9
built = = 0.5
1 p q 18
= 0 1 0 cos (A – B)
0 0 1  = = cot (A – B)  R 
2

sin (A – B) 85. (d) C.I. = P  1+  –1
⇒x=1
80. (c) ? = 0.6 × 0.06 = 0.036   100  
81. (b) Curved surface area = 2pr2  2

REASONING AND GENERAL 10 
22 = 12000  1 +  –1
AWARENESS (RAGA) 27.72 = 2 × × r2   100  
7
71. (a) 72. (c)
27.72 × 7   11 11  2 
73. (c) Lord Canning (1858-1862) r2
= = 12000 ×   ×  –1
74. (d) 75. (b) 76. (c) 44   10 10  
77. (b) In order to combat 2.52 × 7
= = 0.63 × 7 121 –100 
greenhouse gas emissions, Ireland 4 = 12000 
 100 
Government decided to ban the sale r2 = 0.09×7× 7
of new petrol and diesel vehicles by r = 0.3 × 7 = 2.1 cm 21
= 12000 ×
2030 as part of a major strategy to 100
82. (b) Let the number be x.
protect the environment. This is one ∴ According to question, = 120 × 21 = ` 2520
of 180 measures the Irish government 1 P×16×3
published in the ‘Climate Action Plan’. 121 But × 2520 =
x + 42 = ×x 2 100
The “Climate Action Plan” also includes 100
the elimination of non-recyclable plastic 121x 1260 × 100
– x = 42 P=
and higher fees on the production of 100 16 × 3
materials that are difficult to recycle. 420 × 100
The Irish Government’s objective is to 21x = = 105 × 25 = ` 2625
= 42 16
transition to a low-carbon and climate- 100
42 × 100 86. (d) Expression: W $ X + L + Y
resilient society and put the country to x= = 200
achieve net zero carbon emissions by 21 +T
2050. Finally, the number is 200. W–  X+
Practice Set-22 285

| Finally, the missing term is DW. [Hints 96-100] According to the given
L + [Hints 91]: According to the given information:
information, we get From (v), we get
|
V  Second to the left of P and Day
Y+ Second to the right of W. Lecture E Wednesday
| T  Third to the right of Q. From (iii) & (iv), we get
T + S  Third to the right of R
Day
So, the circular arrangement is as Holiday Thursday
So, Y is grandson of X.
follows, Day
Therefore, ‘Y is the son of X’ is So, Lecture B Tuesday
definitely false. Day
Lecture F Friday
87. (a) As,
From (ii), we get
DI S O R DER O S I D R E D R
Day
Lecture A Sunday
Opposite Day
Lecture D Saturday
Opposite
91. (d) By analysing the above By elimination, we get
Similarly, circular arrangement, it is clear that
Day
PR AC T I CE C A R P E C I T W  to the immediate right of S. Lecture C Monday
92. (a) According to the given Day Lectures
Opposite information Monday C
Opposite a2x a byz  a2 x = byz
Tuesday B
Finally, ‘PRACTICE’ is written as czx a b2y  czx = ba2y
Wednesday E
‘CARPECIT’ in code language. and c2z a axy  c2z = axy
Thursday Holiday
88. (d) According to the question, From (ii), we get
c2zx = cb2y Friday F
the direction diagram will be as follows
 axyz = cb2y [∴ c2z = from (iii)] Saturday D
 ax2 = cb2 Sunday A
 a2x2 = acb2 96. (b) By analysing the above
From (i), we get table, it is clear that
a2x2 = bxyz Delivered on
Lecture D  Saturday.
 acb2 = bxyz
97. (d) By analysing the above
[∴ a2x2 = acb2 from (iv)]
table, it is clear that
 abc = xyz on
 abc a xyz Holiday Thursday
Clearly E is to the North of A. 93. (c) The numbers 2, 3, 5 and 6 98. (a) By analysing the above
89. (d) According to the question, lie on the faces adjacent to number 1. table, we get
Anil > Sunny Therefore, 4 lies opposite 1.
Baby > Sunny Number of lectures between F and
94. (a) As, 15 × 4 + 2 = 62 D → 0 (none).
Anil > Sunny > Bose
Similarly, 16 × 4 + 2 = 66
Anil > Baby 99. (a) By analysing the above
On arranging the above data, we get Finally, the missing number is 66.
table, it is clear that
Anil > Baby > Sunny > Bose 95. (b) Actors = [G, E, D, C, B, H]
So, shortest person in Bose. Curators = [H, B, I, J] Last lecture (on Sunday) → A.
90. (b) Poets = [D, E, F] 100. (a) By analysing the given
Fathers = [A, B, C, D, I] information, it is clear that, statement (i)
The sent of letters = [E, F] is not necessary to determine the order
Hence E F will be required set of
of lectures.
letters who represents poets who are not
qqq
fathers.
286 Practice Set-23

Air Force Airmen Group X & Y Exam


Practice Set-23

(a) deities the appropriate number (a, b, c). If


ENGLISH
(b) animals and habitats there is no error, select the number 'd'.
Directions (1-3): Read the given (c) natural scenery 6. After you wll return (a)/ from
passage and answer the questions
(d) female figures Chennai (b)/ I will come and see you.
that follow by selecting the most
2. Find a word in the passage which (c)/ No error (d)
appropriate option.
is opposite of ‘minimised’— Directions (7-8): In the following
Raja Ravi Verma was the Indian King
(a) influenced (b) criticised questions a sentences has been given
and painter whose paintings brought
(c) exposed (d) enhanced in Active/Passive Voice. Out of the four
a momentous turn in “Indian art.”
alternatives suggested below, select
His works on great Indian epics 3. He was especially able to access
the one which best expresses the same
Ramayana and Mahabharata brought the historical documents in the
sentence in Passive/Active Voice.
omnipresent deities to the surroundings possession of—
7. The cat is running after the rat.
of earthly world. This showed excellent (a) national museums that curate
fusion of Indian traditional art with (a) The rat was being run after
them
European realism. These paintings (b) The rat is being run after by the
(b) certain individuals cat
influenced future generation artists and (c) families who inherited them
also influenced the literature and films. (c) The rat is run after by the rat
(d) his family members in Kilimanoor (d) The cat is is being run after by the
His representation of mythological
Directions (4): In the following rat
characters has become a part of the
question four alternatives are given 8. English is spoken all over the
Indian imagination of the classics. His
for the underlined or given idiom/ world.
style is criticised for being too gaudy
phrase. Choose the alternative which (a) All over the world English speaks
and sentimental.
best expresses the meaning of the (b) English speaks all over the world
Ravi Verma was born on April 29th, 1848
underlined part or given idiom/phrase. (c) The whole world speaks Engish
in Kilimanoor Palace in Kerala. Ravi
4. The bus had a close shave as its (d) People speak English all over the
Verma was brought up in an environment
driver swerved to the right a split wrold
of art and culture. At the age of seven, he
second before the on coming truck 9. Choose the correct meaning of
started painting the figures of animals,
run into it. the proverb/idiom: Blows his own
acts and scenes from daily life on the
wall with charcoal. As he grew up, he was (a) serious accident trumpet
exposed to the famous paintings of Italian (b) close collision (a) flatters
painters. Here, he was using indigenous (c) narrow escape (b) praises others
paints made from leaves, flowers. He (d) deep starts (c) admonishes others
enhanced his creativity by listening to the Directions (5): In the following (d) praises himself
music of veterans, watching Kathakali, questions out of the four alternatives, Directions (10-12): Tick the verb form
a folk dance form, going through the choose the one which can be substituted of the following:
manuscripts preserved in ancient families for the given words/sentence. 10. FAILURE
and listening to the artistic interpretation 5. Government by a ruler who was (a) Failed (b) Fallacy
of the epics. unlimited power. (c) Fiasco (d) None of the above
Raja Ravi Verma is most remembered (a) Despotism (b) Autocracy 11. If you _______ with him he will
for his paintings of beautifully sari-clad (c) Monarchy (d) Anarchy give you a pen.
women, who were depicted as graceful Directions (6): In the following quesion (a) will have gone
and shapely. some of the sentence have errors and (b) go
1. The themes of Ravi Verma’s some have none. Find out which part (c) would go
famous paintings were— of a sentence has an error and select (d) will go
Practice Set-23 287

12. You would be ill if you _____ so (a) Career (b) Creator 25. A r i g i d b a r o f m a s s M i s
much. (c) Center (d) Carrier supported symmetrically by three
(a) have had eaten 20. Point out the synonym of the word wires each of length l. Those at
(b) ate PENITENCE each end are of copper and the
(c) will have eaten middle one is of iron. The ratio of
(a) submission
(d) would have eaten (b) retribution their diameters, if each is to have
13. Which of the following is the (c) confinement the same tension, is equal to:
past participle form of the verb (d) repentance Yiron
'beseech'? (a) Ycopper/Yiron (b)
Ycopper
(a) Beseeches (b) Beseeching PHYSICS
(c) Besought (d) Beseeched 21. A ball moving with velocity 2 ms–1 2
Yiron Yiron
(c) (d)
14. Identify the correct tense in the collides head-on with another 2 Ycopper
Ycopper
following sentence: stationary ball of double the mass.
"He said that he was going to eat If the coefficient of restitution is 26. We consider a thermodynamic
it." 0.5, then their velocities (in ms–1) system. If ∆U represents the
after collision will be: increase in its internal energy and
(a) Present continuous
(a) 0, 1 (b) 1, 1 W the work done by the system,
(b) Past continuous
(c) 1, 0.5 (d) 0, 2 which of the following statements
(c) Present perfect
22. A spring of spring constant 5 is true?
(d) Past perfect × 103 Nm–1 is stretched initially
15. "Have you submitted your report (a) ∆U = – W in an adiabatic
by 5 cm from the unstretched process
yet" she said. (Change into position. Then, the work
Indirect speech) (b) ∆U = W in an isothermal
required to stretch it further by
(a) She asked him if he had submitted another 5 cm is: process
his report till then (a) 12.50 N-m (b) 18.75 N-m (c) ∆U = – W in an isothermal
(b) she asked him whether he has (c) 25.00 N-m (d) 6.25 N-m process
submitted the report yet 23. A thin circular ring of mass M and (d) ∆U = W in an adiabatic process
(c) She asked him if he was submitted radius r is rotating about its axis 27. Three charges q1, – q2 and q2 are
report till now with a constant angular velocity ω. placed as shown, S is a Gaussian
(d) She asked him to submit his Four objects each of mass m are surface. Electric field at any point
report immediately kept gently to the opposite ends on S is:
16. My mother said, "Don't go very of two perpendicular diameters of
the ring. The angular velocity of
far into the sea." (change into
the ring will be:
indirect speech)
(a) My mother told me about going Mω
(a)
far into th sea M + um (a) Due to q2 only
(b) My mother advised to not go out (b) Uniform at all points
( M + um)ω
far into the sea (b) (c) Zero at all the points
(c) My mother told me not togo very M
(d) Due to all the charges
far into the sea ( M + um)ω
(c) 28. The electric field associated with an
(d) My mother suggested that I M − um electromagnetic wave in vacuum is
should not swim far M ω
(d) given by E = iˆ 40 cos( kz − 6 × 108 t ),
Directions (17-18): Choose the corre- um
ctly spelt word: where E, z and t are in V/m,
24. What is the nature of graph
17. (a) Entrepreneur (b) Enterprenur between displacement (y) and metre and second, respectively.
(c) Enterprenuer (d) Entreprinuer acceleration (a) of SHM? The value of wave vector k is:
18. (a) Etiquete (b) Etiquette (a) Straight line (a) 2 m–1 (b) 0.5 m–1
(c) Ettiquete (d) Ettiquette (b) Parabola (c) 6 m–1 (d) 3 m–1
19. Which of the following words is 29. Two batteries, one of emf 18 V
(c) Hyperbola
mis-spelt? and internal resistance 2 Ω and
(d) Ellipse
the other of emf 12 V and internal
288 Practice Set-23

resistance 1 Ω, are connected 4t + 9 Wb. What is the induced (a) 6.28 × 10–5 V
as shown. The voltmeter V will emf at t = 2? (b) 4.8 × 10–5 V
record a reading of: (a) 3 V (b) 4 V (c) 6 × 10–5 V
(c) 9 V (d) 16 V (d) 1.6 × 10–5 V
33. Two charged particles traverse 37. An electron is projected with
identical helical paths in a uniform velocity along the
completely opposite sense in a axis of a current carrying long
→ solenoid. Which of the following
uniform magnetic field B = B0 kˆ . is true?
(a) They have equal z-components (a) The electron will be accelerated
(a) 15 V (b) 30V of momenta along the axis
(c) 14 V (d) 18V (b) They must have equal charges (b) The electron path will be
30. The figure below shows, the plot (c) They necessarily represent a circular about the axis
pV particle anti-particle pair (c) The electron will experience
of versus p for oxygen gas a force at 45° to the axis and
nT (d) The charge to mass ratio hence execute a helical path
at two different temperatures.  e  e 
satisfy (d) The electron will continue to
 m  +  m  = 0
1 2 move with uniform velocity
34. An object of mass m is raised along the axis of the solenoid
from the surface of the earth to 38. A p-n-p transistor is used in
a height equal to the radius of common emitter mode in an
the Earth, that is, taken from a amplifier circuit. A charge of 40 µA
distance R to 2R from the centre in the base current brings a charge
of the Earth. What is the gain in of 2 mA in collector current and of
its potential energy? 0.04 V in base emitter voltage. The
Read the following statements base current amplification factor is:
concerning the above curves. 1 1 (a) 5 (b) 50
(a) mgR (b) mgR
(i) The dotted line corresponds to 2 4 (c) 500 (d) 0.5
the ideal gas behaviour. 1 39. Which of the following is the
(c) 1 mgR 2 (d) mgR main source of magnetism in a
(ii) T1 > T2 2 6
pV bar magnet?
(iii) T he value of at the 35. The total energy of the electron (a) Spinning of electrons
nT orbiting around the nucleus in (b) Spinning of atoms
point where the curves meet the ground state of the atom is: (c) Orbital motion of electrons
on the y-axis is the same for (a) Less than zero (d) Polarisation of atoms
all gases. (b) Zero 40. Young’s double slit experimental
Which of the above statements is (c) More than zero arragement is shown in figure. If
true? (d) Sometimes less and sometime λ is the wavelength of light used
(a) Only (i) more than zero and ∠S1CS2 = θ, then the fringe
(b) (i) and (ii) 36. A bicycle wheel of radius 0.5 m width will be:
(c) All of these has 32 spokes. It is rotating at
the rate of 120 revolutions/min, S1
(d) None of these
31. Which of the following is not perpendicular to the horizontal
the property of equipotential component of Earth’s magnetic
θ
surfaces? field B H = 4 × 10 –5T. The emf C
(a) They do not cross each other induced between the rim and the
centre of the wheel will be:
(b) They are concentric spheres for
uniform electric field S2
(c) Rate of change of potential λ λ
with distance on them is zero (a) (b)
θ 2θ
(d) They can be imaginary spheres
32. The magnetic flux linked with a 2λ
(c) λθ (d)
coil varies with time as φ = 3t2 + θ
Practice Set-23 289

41. The total energy of electron in the Which of the above are correct? 54. The equation of the parallel
ground state of hydrogen atom is (a) I and II (b) I and III to the line 2x + 3y + 5 = 0 and
–13.6 eV. The kinetic energy of an (c) II and III (d) I, II and III passing through (1, 1) is:
electron in the first excited state is:
47. If z − 2 (a) 2x + 3y = 0
(a) 3.4 eV (b) 6.8 eV = 2 represents a circle,
z− 3 (b) 2x + 3y + 2 = 0
(c) 13.6 eV (d) 1.7 eV
42. A 1000 kHz carrier wave is then its radius is: (c) 2x + 3y + 8 = 0
1
modulated by an audio signal of (a) 1 (b) (d) 2x + 3y – 5 = 0
frequency range 100–5000 Hz. 3
3 55. The value of
Then, the width of the channel (c) (d) 1 1 1
(in kHz) is: 4 + + + .....∞ is:
(a) 10 (b) 20 48. How many real roots does the 2.3 4.5 6.7
(c) 30 (d) 40 quadratic equatics f(x) = x 2 + (a) log (2/e) (b) log (e/2)
43. A charged particle (charge q) 3| x | + 2 = 0 have? (c) 2/e (d) e/2
is moving in a circle of radius (a) one (b) two 56. What is the equation of circle which
R with uniform speed v. The (c) four (d) no real root touches the lines x = 0, y = 0 and x
associated magnetic moment µ
3 + 5 + 7 + ........ + n = 2?
is given by: 49. I f =7
5 + 8 + 11 + ....... + 10 terms (a) x2 + y2 + 2x + 2y + 1 = 0
qvR (b) x2 + y2 – 4x – 4y + 1 = 0
(a) (b) qvR2 the value of n is:
2 (c) x2 + y2 – 2x – 2y + 1 = 0
(a) 35 (b) 36
2
(c) qvR (d) qvR (c) 37 (d) 40 (d) None of the above
2 50. When is the coefficient of x in 4 57. If (log 3 x) 2 + log 3 x < 2, then
44. W h e n a p i e c e o f m e t a l i s
the expanstion of (1 + 2x + 3x2 + which one of the following is
illuminated by a monochromatic
light of wavelength λ, then 4x + .....) =?
2 ½ correct?
stopping potential is 3Vs. When (a) 1/4 (b) 1/16 1 1
(a) 0 < x < (b) < x<3
same surface is illuminated by (c) 1 (d) 1/128 9 9
light of wavelength 2λ, then 51. lim 1 + 2 + 3 + ..... + n is equal to: 1
stopping potential becomes Vs. x→∞ n2 + 1 (c) 3 < x < ∞ (d) ≤ x≤3
The value of threshold wavelength 9
(a) 1 (b) 2
for photoelectric emission will be: 58. For n ∈ N,  1  n5 +  1  n3 +  7  n is
 5  3  15 
(a) 4 λ (b) 8 λ (c) 1 (d) None of
2 these (a) an integer
4
(c) λ (d) 6 λ (b) a natural number
3 52. If y = {x + (1 + x 2 ) }m–1 then (1 +
45. If the rms current in a 50 Hz AC (c) a positive fraction
x2)y2 + xy1 – m2y is equal to:
circuit is 5 A, the value of the (d) None of these
(a) 2 (b) 1
current 1/300 s after its value dy
becomes zero is: (c) –1 (d) 0 59. If x = y (log y – log x + 1),
dx dx
(a) 5 2A (b) 5 3 / 2A 53. ∫ equals to: then the solution of the equation
(1 + x ) x − x 2 is:
(c) 5/6 A (d) 5 / 2A
1− x x
MATHEMATICS (a) 2
+C (a) log y = Cy
(1 – x )
46. Consider the following with
2( x − 1) y
regard to a relation R on a set of (b) +C = Cy
(b) log
(1 − x ) x
real numbers defined by xRy if
x
and only if 3x + 4y = 5 1+ x (c) log = Cx
(c) +C y
I. 0 R 1 II. 1 R 1 (1 + x) 2
2 y
III. 2 R 3 1+ x log = Cx
(d) +C (d) x
3 4 (1 − x )2
290 Practice Set-23

60. Consider the two series of 67. The equation of parabola whose (a) SYPS (b) MIPS
observations A and B as follows: focus and vertex are the points (c) BAUD (d) Byte
(0, 2) and (0, 4) respectively is: 74. The original name of
Series A 1019 1008 1015 1006 1002 ‘Mahabharata’ is :
(a) x2 + y = 32
Series B 1.9 0.8 1.5 0.6 0.2 (b) x2 + 8y = 2 (a) Kathasaritsagara

If the standard deviation of the (c) x2 + 8y = 32 (b) Jaya Samhita
(d) None of these (c) Rajatarangini
series A is 38 , then what is the 68. What is the value of (d) Bharat Katha
standard deviation of the series B? tan 9º – tan 27º – tan 63º + tan
75. An indirect instrument of
(a) 3.8 (b) 0.38 81º?
monetary policy is :
(a) 1 (b) 2
(c) 0.38 (d) 38 (a) Bank rate
(c) 3 (d) 4
61. The slope of the tangent to the (b) Cash reserve ratio
69. What is the value of 3 cosec
curve x = 3t2 + 1 and y = t3 – 1 at (c) Open market operations
20º – sec 20º? (d) Statutory liquidity ratio
x = 1 is:
(a) 1 (b) 4 76. Bodo and Dogri were added in
(a) 0 (b) 1 4
the 8th Schedule by the following
2 (c) 2 (d) 1
(c) (d) –2 amendment :
62. The probability of getting heads 70. If A =  x 0  and B =  0 0  (a) 81st Amendment
 a b
in both trials, when a balanced  y 0  (b) 85th Amendment
are the two matrices, then find (c) 91st Amendment
coin is tossed twice, will be:
(a) 1/4 (b) 1/2 AB. (d) 92nd Amendment
(c) 1 (d) 3/4 0 0  77. Where was the 12th session of
63. The number of ways of dividing (a)   Conference of States Parties
0 0 
15 men and 15 women into 15 to the Convention to Rights of
couples, each consisting of a man 0 0 Persons with Disabilities (CRPD)
(b)  
and a woman, is:  ax by  2019 held recently?
(a) 1240 (b) 1840  0 0 (a) London
(c) 1820 (d) 2005 (c)  ax + by 0  (b) New Delhi
 
64. What is the value of (c) Washington, D.C.
0 0 
1
(d) 0 ax + by  (d) New York
∫ 0 x(1 − x)
9
dx ?
  78. When 0.090909.....is converted
1 1 into a fraction, then the result is:
(a) (b)
110 111 REASONING AND GENERAL (a) 1/3 (b) 1/11
1 1 AWARENESS (RAGA) (c) 2/33 (d) 6/11
(c) (d)
112 119 71. Kundankulam Project is located 2
79. If 2sec A = x, then the value of x
in which state ? is
65. If sin A = 2 and cos B = 1 ,
(a) Karnataka (a) 1/(1 + cos A) + 1/(1 – cos A)
5 10
where A and B are acute angles, (b) Tamil Nadu
(b) 1/(1 + tan A) + 1(1 + sin A)
(c) Telangana
then what is the value of A + B? (c) 1(1 + sin A) + 1/(1 – sin A)
(a) 135º (b) 90º (d) Kerala
72. Presence of excess fluorine in (d) 1/(1 + cot A) + 1/(1 – cos A)
(c) 75º (d) 60º
water causes : 1
π –1  1   (a) Fluorosis 80. 2 + =?
66. sin  − sin  −   is equal to: 1
 3 2  (b) Dental Cavity 2+
2
(c) Tooth Decay
1 1 1
(a)
2
(b) (d) Respiratory disease (a) 7 (b) 6 1
3 73. Which of the following units is 2 2
(c) 1 (d) 1 used to measure the speed of a 2 1
4 computer ? (c) 2 (d) 1
5 3
Practice Set-23 291

81. The circumference of a circle (a) L is the daughter of T (a) W


is equal to the perimeter of (b) K is the son-in-law of I (b) T
an equilateral triangle. If the (c) I is the grandmother of L (c) P
radius of the circle is 14cm what (d) T is the father of J
is the length of the side of the (d) Data inadequate
87. In a certain code language,
equilateral triangle? Direction (92): Study the following
“DURING” is written as
(a) 88 cm (b) 88/3 cm information answer the questions
“GRUFQD”. How is “PICKED”
(c) 88/√3 cm (d) 88√3 cm given below it. In a certain code, the
written in that code language?
82. If 25% of an electricity bill is symbol for 0 is © and for 1 is ⊕. There
deducted, ` 1788 is still to be (a) SAFFIIA (b) TGFHHA
are no other symbols for all other
paid. How much was the bill ? (c) SFFHHA (d) SFHFHA
numbers greater than 1. The number
(a) ` 1430 (b) ` 2420 88. A policeman left his police post greater than 1 are to be written only
(c) ` 1466 (d) ` 2384 and proceeded South 4 km by using the two symbols given above.
83. A shopkeeper, sold cocoa seeds at on hearing a loud sound from The value of symbol for 1 doubles
the rate ` 1,230 per kg and bears point A. On reaching the place, itself everytime it shifts one place to
a loss of 5%. Now if he decides to he heard another sound and the left. Study the following example:
sell it at ` 1,353 per kg, what will proceeded 4 km to his left to the ‘0’ is written as ©.
be the result ? point B. From B he proceeded
(a) 4.76 percent gain ‘1’ is written as ⊕.
left to reach another place C, 4
(b) 9 percent gain ‘2’ is written as ⊕ ©.
km away. In which direction, he
(c) 4.5 percent loss ‘3’ is written as ⊕ ⊕.
has to go to reach his police post?
(d) 9 percent loss ‘4’ is written as ⊕ © © and so on.
(a) North (b) South
84. Mother can bake 20 cakes in 92. Which of the following will
4 hours, Mother and Dadima (c) East (d) West
represent ‘5’ ?
together can bake 60 cakes in 10 89. Seema's younger brother Sohan
(a) © ⊕ ⊕
hours. How many cakes Dadima in older than Seeta. Sweta is
(b) ⊕ ⊕ ⊕
can bake in 30 hours ? younger than Deepti but elder
(a) 30 (b) 75 than Seema. Who is the eldest? (c) ⊕ ⊕ © ©
(c) 60 (d) 25 (a) Seema (b) Sweta (d) ⊕ © ⊕
85. If the amount received at the (c) Seeta (d) Deepti Direction (93): Select the related
end of 2nd and 3rd year at Direction (90): A series is given with word/letters/number from the given
compound interest on a certain one term missing. Choose the correct alternatives.
principal is ` 28090, and ` alternative from the given ones that 93. Sheep : Flock : : Grapes : ?
29775.4 respectively, what is the will complete the series. (a) Bunch (b) Stock
rate of interest? (c) Hectare (d) Land
90. 23, 32, 42, 53, 65, ?
(a) 6 percent (b) 3 percent 94. Identify the diagram that best
(a) 78 (b) 88
(c) 12 percent (d) 10 percent represents the relationship
(c) 58 (d) 98
Direction (86): Real the information among the given classes.
Direction (91): Study the following
carefully and answer the following Food Supplement, Bournvita,
information carefully and answer the
questions: Horlicks.
questions given below:
If A + B means A is the father of B
P, Q, R, S, T, V and W are sitting
If A × B means A is the sister of B (a) (b)
around a circle facing at the centre. V
If A $ B means A is the wife of B
If A % B means A is the mother of B is second to the left of P and second to
If A ÷ B means A is the son of B the right of W. T is third to the right of (c) (d)
Q and is not an immediate neighbour
86. Which among the following
of V, S is third to the right of R.
options is true if the expression 95. Identify the diagram that best
91. Who is to the immediate right
‘I + T % J × L ÷ K’ is definitely represents the relationship
of R. among the given classes.
true?
292 Practice Set-23

Engine, Car, Battery ‘well being is essential’ is written 98. Which of the following may,
as ‘bm hr la xf’. represent ‘always well respect’?
(a) (b) ‘earned money being best’ is (a) la fo zt (b) qs dc mi

written as ‘zt qs mi xf’ (c) hr gu xf (d) gu la dc
96. Which of the following is the 99. What does the code ‘bm’ stand
(c) (d) for?
code for ‘money’?
(a) is (b) very
Directions (96–100): Study the (a) zt (b) qs
(c) money (d) respect
following information carefully and (c) mi (d) zt or qs 100. Which of the following may
answer the questions given below: represent ‘ la dc mi’?
97. In the given code language ‘very
‘very well earned respect’ is essential’ can be written as: (a) well earned respect
written as ‘fo mi la gu’. (b) best earned money
(a) fo la (b) hr fo
‘respect is always earned’ is (c) earned always well
written as ‘dc gu ml bm’. (c) hr bm (d) gu la
(d) essential very earned

Answers with Explanation


12. (b) You would be ill if you ate so Confinement means ' ' P u n i s h m e n t
ENGLISH much. for doing something or animal is kept
1. (d) The main theme of Ravi somewhere usually by force''.
Verma’s famous paintings were female 13. (c) The past participle of the verb
figures. – "beseech" is 'besought'. Hence option Repentance means "regret for any
2. (d) Opposite of minimised is (c) is correct. past action".
‘enhanced’. 14. (b) "He said that he was going to
3. (c) He was able to access historical eat it" the correct tense of this sentence PHYSICS
documents in the possession of families is – Past Continuous Tense. Structure of 21. (a) When two bodies collide
who inherited them. past continuous tense: head-on with coefficient of
4. (c) Idiom 'close shave' means: a Sub + was/were + v + ing + object
situation in which you only just manage Hence option (b) is correct. restitution e = v2 − v1 ...(i)
to avoid an accident. Idiom 'Narrow u2 − u1
15. (a) The correct option is 'she
escape' bears the same meaning. asked him if he had sumbitted his report From law of conservation of
Look at the sentence: till then'. momentum
He had a narrow escape when his car 16. (c) The correct option is 'My m1u1 + m2u2= m1v1 + m2v2
skidded on the ice. mother told me not to go very far into ⇒ v2 =

5. (b) Autocracy the sea'.  m1 − em2   (1 + e)m2 
6. (a) When two events are possible 17. (a) Entrepreneur means "someone  m + m  u1 +  m + m  u2
 1 2   1 2 
in future,. then the first part of the who organises a business venture and
sentence (that is likely earlier) should be assumes the risk for it" (A person who Put, u1 = 2 ms–1, u2 = 0, m1 = m
expressed in Present Indefinite. Hence , set up a business or businesses, taking and m2 = 2 m, e = 0.5
After you return ...... will be a correct on financial risks in the hope of profit) :
sentence. So option (a) is correct.  m−m 
v1 = 
× 2 ⇒ v1 = 0
7. (b) The rat is being run after by the 18. (b) The correct option is (b)  m + 2m 
cat. Etiquette.
Similarly,
8. (d) People speak English all over 19. (c) The correct spelling is 'centre'.
the world. 20. (d) Pentience means "a person  (1 + e)m1   m − em1 
u + 2 u
9. (d) Blows his own trumpet means who repents of sin". v2 =  m1 + m2  1  m1 + m2  2
to talk about himself/self achievements. Submission means "an act of
So option (d) praises himself is correct. giving a document proposal or piece of 1.5 × m 
=  × 2 = 1 ms–1
10. (a) Verb form of Failure is (a)
writing, etc. to someone so that it can be  3m 
considered or approved". 22. (b)
failed.
Retribution means "Punishment for 1 2 1
11. (b) If you go with him he will doing something wrong W1 = kx1 = × 5 × 103 × (5 × 10 −2 ) 2
give you a pen. 2 2
Practice Set-23 293

= 6.25 J Speed of electromagnetic (ii) At high temperature,


wave, the deviation of the gas is
W2 = 1 k ( x1 + x2 ) 2 ω less and at low temperature
2 v=
k the deviation of gas
= 1 × 5 × 103 (5 × 10−2 + 5 × 10−2 )2 Given, equation is more. In the graph,
2 → deviation for T 2 is greater
= 25 J E = iˆ 40 cos (kz − 6 × 108 t ) than for T1.
Network done = (W2 – W1) ...(ii) (iii) The two curves intersect
= (25 – 6.25) = 18.75 J Comparing Eqs. (i) and (ii), we at dotted line, so the
= 18.75 N-m get pV
value of at that point
23. (a) Initial angular momentum of ω= 6 × 108 and E0 = 40iˆ nT
ring L = Iω = Mr2ω on the y-axis is same
ω
Final angular momentum of Here, wave factor k = for all gases.
ring and four particles system v 31. (b) They are parallel surfaces in
L = (Mr2 + umr2)ω′ 6 × 108 uniform field.
= = 2 m–1
There is no torque on the 3 × 108 dφ
29. (c) It is clear that the two cells 32. (d) E = = 6t + 4, At, t = 2, we
system, therefore angular dt
moentum remains constant. oppose each other, hence the
effective emf in closed circuit find e = 16 V.
Mr2ω= (Mr2 + umr2)ω′ 33. (d) In a uniform magnetic field,
is 18 – 12 = 6 V and net
Mω resistance is 1 + 2 = 3 Ω the two charged particles will
ω′ = traverse identical helical paths
M + um (because in the closed circuit
24. (a) The relation between y and a in a completely opposite sense
the internal resistances of two
is: if the charge/mass ratio of
cells are in series).
a = – ω2y these two particles is same
The current in circuit will be and charges on them are of
It is the equation of a straight in direction of eroor shown in
line. opposite character. In this
figure.  e  e 
 D
2
Effective emf 6 situation  m  +  m  = 0
f /π  I = = =2
 2 4 Fl
1 2
25. (b) Y = 3
= A Total resistance holds good.
∆l / L πD 2 ∆l 34. (a) Potential energy of the object
The potential difference across
4 Fl 1 V will be same as the terminal at the surface of the Earth
∴D=
⇒∆∝
π.∆lY Y voltage of either cell. = − GM e m
V R
Dcopper Yiron
Hence, = 18 V PE of the object at a height

Diron Ycopper equal to the radius of the Earth
2Ω
26. (a) From first law of GMm
= −
thermodynamics 12 V 1 Ω 2R
∆Q = ∆U + ∆W ∴ Gain in PE of the object
For adiabatic process ∆Q = 0 Since, current is drawn from GMm  GMm 
∆U = – ∆W the-cell of 18 V, hence = − − 
V1 = E1 – Ir1 2R  R 
27. (c) As net charge enclosed by the
= [18 – (2 × 2)] GMm gR 2 × m
surface is zero = + =
= (18 – 4) = 14 V 2R 2R
→ → qenc 30. (c) (i) The dotted line in the
as ∫ E ·
dS = =0
∈0 diagram shows that there is ( GM = gR 2 )
(By Gauss’s law) no deviation in the
pV = 1 mgR
∴ E=0 value of for different 2
28. (a) E l e c t r o m a g n e t i c w a v e nT 35. (a) The total energy of the electron
equation temperatures T 1 and T 2 for orbiting around the nucleus in
E= E0 cos (kz – ωt)   ...(i) increasing pressure, so this gas the ground state of the atom is
behaves ideally. less than zero.
294 Practice Set-23

39. (a) The main source of magnetism qv 1


Ba 2 ω ∴ µ= × π R 2 = qvR .
36. (a) Induced emf e0 = in a bar magnet is spinning of 2π R 2
2 electrons.
44. (a) According to Einstein’s
a is length of each spoke.

40. (a) Fringe width, β= photoelectric equation
d
120 1 1 
n=
=2 d eV = hc  − 

60 But here, θ = ⇒ d = Dθ  λ λ0 
rad D
⇒ ω= 2πn = 4π
1st case 3eVs
s Dλ λ
a = 0.5 m
∴ β = = 1 1 
Dθ θ = hc  −  ...(i)
 λ λ0 
Ba 2 ω 41. (a) The energy of hydrogen atom
⇒e = 2nd case eVs =
2 when the electron revolves in
= 6.28 × 10–5 V −13.6  1 1 
nth orbit is E = eV hc  −  ...(ii)
37. (d) There is a uniform magnetic n2  2λ λ 0 

field B inside the current In the ground state, n = 1 From Eqs. (i) and (ii), we get
carrying long solenoid acting −13.6 λ0 = 4λ
along the axis of solenoid. E=
12 45. (b) Here V = 50 Hz, Iv = 5 A, I
The magnitude of force on the 1
electron of charge (–e) moving = – 13.6 eV = ?, t = s
→ [For n = 2] 300
with velocity v in a magnetic −13.6 I0 = 2 I v = 2 × 5 A

→ → E=
field B is | F | 22 From I= I0 sin ωt
→ →
= – 3.4 eV
= −e | v × B |= − evB sin θ So, kinetic energy of electron 1
= 5 2 sin100π ×
in the first excited state (i.e., 300

Here, angle θ between v and for n = 2) is KE = – E
3
→ = – (– 3.4) = 3.4 eV = 5 2 × = 5 3 / 2A
B is zero i.e., θ = 0° and sin 2
θ = 0. 42. (a) Width of the channel
Therefore, F = 0 = 2 f m (max) = 2 × 5000 MATHEMATICS
It means the electron will = 1000 Hz 46. (c) The relation is defind as xRy,
continue to move with a = 10 kHz if 3x + 4y = 5
uniform velocity along the 43. (a) A s r e v o l v i n g c h a rg e i s
axis of the solenoid. equivalent to a current, so
38. (b) For a transistor in emitter  1
ω If we take, (x, y) = 1,  and (x,
mode, ⇒
qf = q ×  2
∆VBE 0.04 2π 2 3
= y) =  ,  then these pairs are
Rin = ∆I 40µA 3 4
B
But ω= v satisfied by the given relation.
R
0.04
= = 1000 Ω Where, R is radius of circle and 1 1
40 × 10 −6 1R ⇔ 3.1 + 4. = 5 and
V is uniform speed of charged 2 2

Base current amplification particle.
factor, qv 2 3 2 3
Therefore, i= R ⇔ .3 + 4. = 5 .
∆I C 2mA 2π R 3 4 3 4
β=
=
∆I B 40 µA Now, magnetic moment 47. (d) We have, | z – 2 | = 2 | z – 3|
associated with charged particle
2 × 10 −3 is given by |z– 2|
= = 50 =2
40 × 10 −6
µ = iA = i × πR2 | z – 3|
Practice Set-23 295

⇒ |(x – 2) + iy|2 = 4 |(x – 3) + iy|2 n(n + 2) 1


⇒ =7 Put 1 + t =
⇒ (x – 2)2 + y2= 4 [(x –3)2 + y2] 5 × 37 z
⇒ n2 + 2n – 1295 = 0 1– z 1
⇒ x2 – 4x + 4 + y2 ∴ I= and dt = – 2 dz
⇒ n + 37n – 35n – 1295 = 0
2
z z
= 4x2 + 36 – 24x + 4y2 ⇒ n(n + 37) – 35(n – 37) = 0 dz
⇒ (n + 37) (n – 35) = 0 ∴ I = –2∫
⇒ 3x2 + 3y2 – 20x + 32= 0 (2 z – 1)
∴ n = 35
20 32 [Leaving the negative value] 2
⇒ x2 + y2 – x+ =0 = –2 (2 z – 1) = –2 –1
3 3 50. (c) (1 + 2x + 3x2 + 4x3 + ......)½ 1+ x

= [(1 – x)–2]1/2
 10 
2
32 = (1 – x)–1 = –2 1 – x
∴ Radius =  −  − = 1 + x + x2 + x3 + x4 + .... 1+ x
3 3
So, the required coefficient of x4 (1 – x ) 2( x – 1)
is 1 = –2 = +C
100 – 96 4 2 (1 – x ) (1 – x )
= = =
9 9 3 1 + 2 + 3 + .... + n 54. (d) Equation of line parallel to
51. (c) nlim
48. (d) Given quadratic equation →∞ n2 + 1 2x + 3y + 5 = 0 is
f(x) = x2 + 3| x | + 2 = 0 n(n + 1) 2x + 3y + λ = 0
= nlim But, it passes through (1, 1)
→∞ 2(n 2 + 1)
Case I. f(x) = x2 + 3x + 2 = 0 ∴ 2 + 3 + λ = 0 ⇒ λ = –5
(when, x > 0)  1 ∴ Required equation is
n 2 1 + 
⇒ x2 + 2x + x + 2 = 0  n 2x + 3y – 5 = 0
= nlim
→∞  1 
⇒ x (x + 2) + 1 (x + 2) = 0
2n 2  1 + 2  55. (b) 1 + 1 + 1 + .....∞
 n  2.3 4.5 6.7
⇒ (x + 2) (x + 1) = 0 1  1 1   1 1  1 1
= 1.(1 − 0) = =  2 − 3  +  4 − 5  +  6 − 7  + ......∞
∴ x = –2, –1 (but x > 0) 2(1 − 0) 2
52. (d) = 1 – loge 2
So, here no real roots exist.

 e
m −1  1.2 x  = loge – loge 2 = loge  
 2
Case II. f(x) = x2 – 3x + 2 = 0 2
y1 = m{x + (1 + x )} . 1 + 
(when, x < 0)  2 (1 + x2 )  56. (c) From the figure it is clear that
my coordinates of centre of circle are (1, 1)
3 ± 9 – 8 3 ±1 = and radius of circle is 1.
⇒ x = = 1 + x2
2 2 ∴ Equation of circle is
2
⇒ x = 2, 1 (but x < 0) ∴ y1 (1 + x2) = m2y2 Y

So, here also no real roots exist.


Again differentiating w.r.t x, we get
Hence, given quadratic equation
= 2y1y2 (1 + x2) + y12 2x = m2 . 2yy1
has no real roots. O
49. (a) We have, ⇒ y2 (1 + x2) + xy1 = m2y
⇒ y2 (1 + x2) + xy1 – m2y = 0
3 + 5 + 7 + ...... + n terms
=7 X
5 + 8 + 11 + ...... + 10 terms dx
53. (b) I = ∫ y=0
n (1 + x ) x ( 1 − x )
[2 × 3 + [n − 1]2]
∴ 2 =7 1
Put x =t ⇒ dx = dt x=0 x=2
10 2 x
[2 × 5 + (10 − 1) × 3]
2 (x – 1)2 + (y – 1)2 = 1
dt
I = 2∫ ⇒ x2 – 2x + 1 + y2 – 2y + 1 = 1
[ S = (2 a + (n − 1)d ) ] (1 + t ) 1 – t 2 ⇒ x2 + y2 – 2x – 2y + 1 = 0
296 Practice Set-23

57. (b)  (log3 x)2 + log2 x < 2  dy  sin B = 1 – cos2 B


⇒ (log3 x)2 + (log3 x) – 2 < 0 dy  dt 
3t 2 t
Now, =  dx  = = 1 3
⇒ (log3 x + 2) (log3 x – 1) < 0 dx   6t 2 = 1– =
⇒ – 2 < log3 x < 1  dt  10 10
1  dy  0 sin (A + B) = sin A cos B + cos
⇒ 3–2 < x < 31 ⇒ < x < 3 ∴   = 2 =0 A sin B
58. (b) Let 9  dx t =0
 1  1  7
62. (a) Probability of getting head in = 2 × 1 + 1 × 3
P(n) =   n5 +   n3 +   n 1 5 10 5 10
 5  3  15  one trial =
2
1 1 7 2+3 5
∴ P(1) =   (1)5 +   (1)3 +  1 ∴ Probability of getting heads in = =
5 3  15  both the trials 5 10 5 10
1 1 7 3 + 5 + 7 15 1 ∈ N 1 1 1 5 = 1 = sin 135º
= + + = = = = × = =
5 3 15 15 15 2 2 4 2
10
 1  1  7 63. (a) The number of ways of ∴ A + B = 135º
P(2) =   (2)5 +   (2)3 +   2
 2  3  15  choosing first couple
π –1  1  
= (15) × (15) = 152 66. (d) sin  – sin   
32 8 14 96 + 40 + 14 150 The number of ways of choosing  3  –2  
= + + = =
5 3 15 15 15 second couple π 1 
= sin  + sin –1   
= 10 ∈ N = 14 × 14 = (14)2 3  2 

∴ By mathematical induction P(n) is a

and so on
π π π
natural number. Thus, required number of ways = 152 = sin  +  = sin = 1

+ 142 + ..... + 12 3 6 2
dy y  log y + 1
59. (d) =   Put y = vx 15 × (15 + 1) × (30 + 1) 67. (c) S i n c e , t h e f o c u s a n d
dx x  x 
= = 1240 vertex of the parabola are on
dv 6
x-axis, therefore its directrix is parallel
∴ v + x = v log v + v  n(n + 1)(2n + 1) 
dx 2 to x-axis and axis of parabola y-axis.
 ∑ n = 6  Let the equation of the directrix be
dv = dx  
⇒ y = k. The directrix meets the axis of
v log v x 1

⇒ log (log v) = log x + log C


64. (a)  Let I = ∫
0
x(1 – x)9 dx the parabola at (0, k). But vertex is the
mid-point of the line segment joining the
= log Cx Put 1 – x = t ⇒ dx = –dt
0
focus to the point where directrix meets
y ∴ I = ∫1 (1 – t )t
9
∴ log = Cx (– dt ) axis of the parabola.
x
 – t11 t10 
1 k +2
60. (b) Standard deviation of the series B 1 ∴ =4⇒k=6
= ∫0 (– t 10
+ t )dt =  11 + 10 
9
4
1 2  0
= (19 + 0.82 + 1.52 + 0.62 + 0.22 ) Thus, the equation of directrix is
5 –1 1 –10 + 11 1 y = 6.
2 = + = = Let (x, y) be a point on the parabola,
 19 + 0.8 + 1.5 + 0.6 + 0.2  11 10 110 110
–  then
 5 65. (a) Give that A and B are acute 2
 | y – 6 |2 
69 angles i.e., A < 90º (x – 0)2 + (y – 2)2 =  
= – 1 = 138 – 1
5 and B < 90º and sin A  12 
2 1 ⇒ x2 + 8y = 32
= 0.38 = , cos B =
5 10 68. (d) tan 9º – tan 27º – tan 63º + tan 81º
61. (a) Given curve is We know that, = tan (90º – 81º) – tan (90º – 63º)
x = 3t2 + 1, y = t3 – 1 sin2 θ + cos2 θ = 1 – tan 63º + tan 81º
For x = 1, 3t2 + 1 = 1 ⇒ t = 0 = cot 81º – cot 63º – tan 63º + tan 81º
∴ cos A = 1 – sin 2 A
 dx  dy = (cot 81º + tan 81º) – (cot 63º + tan 63º)
∴   = 6t, = 3t 2
 dt  dt 4 1 2  cos 2 81º + sin 2 81º   cos 2 63º + sin 2 63º  2
= 1– = =2 – ×
5 5  sin 81º.cos81º   sin 63º.cos 63º  2
Practice Set-23 297

2 2 formation. A major cause of fluorosis 2


= – is the inappropriate use of fluoride 79. (c) x = 2 sec2 A =
sin162º sin126º cos 2 A
containing dental products such as
2
= 2 − 2 toothpaste and mouth rinses. =
sin18º cos36º 73. (b) MIPS (Million Instructions 1 – sin 2 A

= 2 2 Per Second) is a unit for measuring 1 – sinA + 1 + sinA


−        =
 5 – 1  5 + 1 computer speed. The number of MIPS (1 – sinA) (1 + sinA)
 4   4  (million instructions per second)
1 1
is a general measure of computing   = +
 5 −1 5 + 1 performance and by implication, the 1 + sinA 1 – sinA
∵ sin18º = , cos36º =  amount of work a larger computer can 1 2
 4 4  80. (c) ? = 2 + = 2+
do. 1 5
 1 1  2+
= 8 –  74. (b) The original name of 2
 5 – 1 5 + 1 Mahabarata was “Jaya Samhita” as 12 2
= =2
 5 + 1 − 5 + 1 2 coined by Vyasa. When first narrated, it 5 5
= 8  = 8. = 4 had only 8,800 shlokas and its original
 5 −1  4 81. (b) According to question
name was “Jaya” as written by Ganesha.
2pr = 3a
69. (b) 3cosec 20º − sec 20º The full 100,000 verses of the book 22
were completed several centuries later 2 × × 14 = 3a
3 1 3 cos 20º – sin 20º 7
= − = by addition of many stories and was
sin 20º cos 20º sin 20º cos 20º 88
finally named as ‘Mahabharata’. =a
 3  3
4 1 75. (c) The indirect or market 88
=  cos 20º − sin 20º 
2 sin 20º cos 20º  2 2  based instruments of monetary policy Side, (a) = cm
comprise open market operations and 3
= 4 82. (d) Let the amount of bill be ` x.
(sin 60º cos 20º – cos 60º sin 20º ) the use of ‘Repo’ rate. An open market
sin 40º
operation involves buying or selling 25
4 x − x = 1788
= . sin (60º − 20º ) = 4 of government securities from or to 100
sin 40º the public and banks. The RBI sells 3x
 x 0  0 0 government securities to control the = 1788
70. (a) AB =    4
 y 0 a b  flow of credit and buys government
securities to increase credit flow. 1788 × 4
x×0 + 0× a x × 0 + 0× b x= = 2384
=  76. (d) The 92nd Constitutional 3
 y ×0 + 0× a y × 0 + 0 × b  Amendment Act, 2003, amended the The bill was ` 2384.
0 0  Eight Schedule to the Constitution so 83. (a) Loss on selling of cocoa
=   as to nclude Bodo, Dogri, Santhali and seeds
0 0  Maithali Languages, thereby raising the 5
total number of languages listed in the = ` 1230 ×
100
REASONING AND GENERAL schedule to 22. = ` 61·50.
AWARENESS (RAGA) 77. (d) On June 12,2019, The 12th C.P. = S.P. + Loss
71. (b) Kudankulam Nuclear Power session of Conference of States Parties
= 1230 + 61·50
Plant is a nuclear power station in to the Convention to Rights of Persons
with Disabilities (CRPD) 2019 held at = ` 1291·5.
Kudankulam in the Tirunelveli district
the UN (United Nations) headquarters, If he sells for ` 1353
of Tamil Nadu. It is a joint Russia-India
project. Construction on the plant began New York. S.P. > C.P. he will get profit
on 31 March 2002, but faced several 78. (b) x = 0.090909..............= 0.09 Profit = S.P. – C.P.
delays due to the fishermen’s objection. 100x = 9.09 = 1353 – 1291·50
72. (a) Fluorosis is a disease caused Subtracting (i) from (ii), we get
by water that contains high amount = 61·5
99x = 9
of fluoride and particularly in ground 61·5 × 100 61500
water. It leads to hypomineralisation 9 1 Profit % = =
x = = 1291·5 12915
of tooth enamel caused by ingestion 99 11
of excessive fluoride during enamel = 4·76% gain.
298 Practice Set-23

84. (a) In 1 hr. number of cakes Similarly, Food supplement


20 Horlicks Bournvita
baked by mother = = 5 cakes
4
In 1 hr., number of cakes baked by
95. (c) Engine and Battery are the
60
mother and Dadima = = 5 cakes parts of a car. So, the best representation
10 is:
Finally, ‘PICKED’ will be written
In 1 hr., no. of cakes baked by
as 'SFFHHA'.
Dadima = 1 cake
In 30 hrs, no. of cakes baked by 88. (d) According to the question, Battery Engine
Dadima = 30 cakes the direction diagram will be as follows
85. (a) By using formula Car
 R 
n
Solution for (96-100):
A = P 1 + 
 100  very well earned respect fo mi la gu ...(i)

respect is always earned dc gu mi bm ...(ii)


2
 R  well being is essential
bm hr la xf ...(iii)
28090 = P 1 +  ...(i)
 100 
zt qs mi xf ...(iv)
earned money being best

3
Clearly, from C the policeman will From equation (i) and (ii), respect ⇒ gu
 R  have to go towards West to reach police
29775.4 = P 1 +   ...(ii) From equation (i), (ii) and (iv), earned
 100  post at D.
⇒ mi
Dividing equation (ii) by (i), we get 89. (d) According to the question,
sequence is as follows From equation (ii) and (iii), is ⇒ bm
3
 R  Deepti > Sweta > Seema > Sohan From equation (i) and (iii), well ⇒ la
29775.4 1 + 
 100  > Seeta From equation (iii) and (iv), being ⇒ xf
= 2
28090  R  So, Deepti is the eldest.
1 +  From equation (i), very ⇒ fo
 100  90. (a) The sequence of the series From equation (ii), always ⇒ de
 R  is: From equation (iii), essential ⇒ hr
= 1 + 
 100  From equation (iv), money ⇒ ztor qs
23 + 9 = 32
R 29775.4 – 28090 best ⇒ zt or qs
= 32 + 10 = 42
100 28090 96. (d) The code for 'money' is 'zt'
4 2 + 11 = 53 or 'qs'.
1685.4
= 53 + 12 = 65 97. (b) The code of
28090
168540 65 + 13 = Very ⇒ fo
R= = 6% Finally, the missing term is 78.
28090 The code of essential ⇒ hr
91. (b) By analysing the above
86. (b) Expression : I + T % J × L 98. (d) The Code of
circular arrangement, it is clear that
÷K always ⇒ dc
I+ T  to the immediate right of R.
The Code of well ⇒ la
| 92. (d) According to the given
T–  K+ information The Code of respect ⇒ gu
| | 0© 99. (a) The code of
J– L+ 1⊕ bm ⇒ is
So, ‘K’ is son-in-law of ‘I’.
using the given pattern in (d), we get 100. (c) The code of
87. (c) As, ⊕ © ⊕  4 + 0 + 1 = ‘5’. la ⇒ well
93. (a) As, sheep are in flock. The Code of dc ⇒ always
Similarly, grapes will be in bunch.
The Code of mi ⇒ earned
94. (d) Bournvita and Horlicks,
both are food supplement then qqq
Practice Set-24 299

Air Force Airmen Group X & Y Exam


Practice Set-24

3. ‘Each hive is a commonwealth’ Directions (10): In the following


ENGLISH means the hive is– question, out of the four Alternatives,
Directions (1-4): Read the given (a) over-crowded choose the one which best expresses
passage and answer the questions (b) has a queen the meaning of the given word that is
that follow by selecting the most (c) self-governing your answer.
appropriate option. (d) without a leader 10. Corroborate
The scene presented by a community of 4. ‘To exert themselves’ means the
bees is the more astonishing, the more we (a) collaborate (b) substantiate
become acquainted with its details. Each bees are– (c) co-operate (d) correlate
hive is a commonwealth of which the (a) having influence Directions (11-12): In the following
queen is nominally the head, receiving (b) quite aggressive questions, out of the four alternatives,
the greatest honour and care from her (c) busy toiling choose the one which can be substituted
industrious subjects. With a greater (d) very exhausted for the given words/ sentence that is
wisdom than can be claimed by men, Directions (5-6): In the following your answer.
these creatures allow no disputes about questions, groups of four words are 11. A roundabout way for speaking
the succession to the throne to induce given. In each group, one word is
them to injure each other, but they require (a) Loose-tongued
correctly spelt. Find the correctly spelt (b) Loquacious
the parties themselves individually to
settle the quarrel between each other, word that is your answer. (c) Circumlocution
without prolonged interference with 5. (a) Onomatopoeia (d) Talkative.
the duties of the hive, indeed, they (b) Onomotopoei 12. An order requiring a person to
may be said with truth to have adopted (c) Onomatopoia attend a court
the advice: “Let those who make the (d) Onamotipoei
quarrels, be the only ones to fight.” 6. (a) bureacracy (b) bereaucracy (a) Courtship (b) Agreement
Only one queen is permitted to (c) buereacracy(d) bureaucracy (c) Subpoena (d) Command
hold office in the community at a time, Directions (7-8): In the following Directions (13-14): In the following
but while her claims are undisputed, questions, choose the word opposite in questions, a sentence has been given
she is treated with singular respect and meaning to the given underlined word in Active Voice/Passive Voice. Out of
affection. Indeed, her presence and the the four alternatives suggested, select
prospect of a future generation, appear that is your answer.
7. As a writer, he is very conceited the one which best expresses the same
the chief motives of the insects to exert sentence in Passive/ Active Voice that
themselves. about his work.
is your answer.
1. The writer’s observation that the (a) proud (b) honest
13. Open the door.
bees “settle the quarrel between (c) modest (d) modem
each other, without prolonged 8. His punishment will be a deterrent (a) The door must be opened.
interference with the duties of the to others. (b) The door will be opened.
hive” suggests that he/she is– (a) determinant (c) The door is opened.
(a) pointing a bee character (b) detriment (d) Let the door be opened.
(b) observing the traits of worker (c) encouragement 14. I did not trust anybody.
bees (d) enrichment (a) Nobody was trusted by me.
(c) appreciating the queen bee Directions (9): In the following (b) Anybody had been trusted by me.
(d) condemning the behaviour of ill- questions, four alternatives are given (c) Nobody would be trusted by me.
tempered human beings. for the idiom/phrase printed in bold (d) Nobody has been trusted by me.
2. ‘ T h e p ro s p e c t o f a f u t u re in the sentence. Choose the alternative
generation’, suggests that the Directions (15-16): In the following
which best expresses the meaning of questions, a sentence has been given in
writer is .......... in his/her outlook. he idiom/phrase that is your answer.
(a) ambivalent Direct/Indirect form. Out of the four
9. On the cards alternatives suggested, select the one
(b) nationalistic
(c) sympathetic (a) impossible (b) shocking which best expresses the same sentence
(d) appreciative (c) evident (d) anticipated in Indirect/Direct form.
300 Practice Set-24

15. He said, "It used to be a lovely, (a) 2 : 1 (b) 1 : 2 R


quiet street." (c) 1 : 1 (d) None of these (a) (2π − θ)θ
4π 2
(a) He said that it used to be a , lovely, 22. A radiation of energy E falls
 θ
quiet street. normally on a perfectly reflecting (b) R 1 − 
 2π 
(b) He pointed out that it had used to surface. The momentum
be a lovely, quiet street. transferred to the surface is:  θ
(c) R  
(c) He said that there used to be a (a) E/c (c) 2E/c 2π
lovely, quiet street. (c) Ec (d) E/c2  2π − θ 
(d) R 
(d) He inquired whether there was a 23. At room temperature, the rms  4π 
lovely, quiet street. speed of the molecules of a 29. A one metre long string of
16. The Prime Minister said that no certain diatomic gas is found to weight 5 × 10 –4 kg having 20
one would be allowed to disturb be 1930 m/s. The gas is: N tension is tied at both the
the peace. (a) H2 (b) F2 ends. The string is plucked at 25
(a) The Prime Minister said, "We (c) O2 (d) Cl2 cm distance from one end. The
shall not allow any one to disturb 24. The surface density of electric frequency of vibration of the
the peace." charge at a place on the earth’s string will be:
(b) The Prime Minister said, "We surface where the rate of fall of
would not allow no one to disturb (a) 400 Hz (b) 100 Hz
the peace potential is 250 V, is: (c) 200 Hz (d) 256 Hz
(c) The Prime Minister said, "No one (a) 2.0 × 10–9 C/m2 30. W h i c h o f t h e f o l l o w i n g
will disturb the peace." (b) 2.21 × 10–9 C/m2 cylindrical rods will conduct
(d) The Prime Minister said, "No one (c) 3.36 × 10–9 C/m2 more heat when their ends are
can disturb the peace," maintained at the same steady
(d) 3.5 × 10–9 C/m2
17. Choose the opposite of the word temperature?
MASTICATE. 25. A circular loop of radius R
carrying current I lies in x-y (a) Length 1 m, radius 1 cm
(a) Conceal (b) Chew
plane with its centre at origin. (b) Length 2 m, radius 1 cm
(c) Review (d) Gobble
The total magnetic flux through (c) Length 2 m, radius 2 cm
Directions (18-20): In the following
x-y plane is: (d) Length 1 m, radius 2 cm
questions, some of the sentence have
errors and some have none. Find out (a) Directly proportional to I 31. A particle of mass m and charge
which part of a sentence has an error. (b) Directly proportional to R + q is placed at rest in a uniform
The number of the part is your answer. (c) Directly proportional to R2 electric field as shown and released.
If there is no error, your answer is (d) (d) Zero The kinetic energy it attains after
i.e. No error. 26. A particle moves along the moving a distance y is:
18. Americans are accustomed to (a)/ parabolic path y = ax2 in such a
drinking coffee (b)/ with their meals.
way that the x-component of the
(c)/ No error (d)
velocity remains constant, say c.
19. He asked (a)/ whether either of the
The acceleration of the particle is:
brothers (b)/ were at home. (c)/ No
(a) ackˆ (b) 2ac ˆj
2
error (d)
20. After the teacher had told the boys (c) ac 2 kˆ (d) a 2 cjˆ
(a)/ how to pronounce the word (b)/
all of them in one voice repeated the 27. If a current is passed through a (a) 1 qEy (b) qEy
word again. (c)/ No error (d) spring then the spring will: 2
(a) Expand (c) qE2y (d) 1
m(qEy )
PHYSICS (b) Compress 2
21. A nucleus of mass M initially at (c) Remain same 32. For two coils with number of
rest splits into two fragments (d) None of these turns 500 and 200 each of length
2M′ 28. A and B are two points on a 1 m and cross-sectional area 4 ×
of masses M ′ and (M >
3 3 uniform ring of resistance R. 10–4 m2, the mutual inductance
M′). Find the ratio of de-Broglie The ∠ACB = θ, where C is the is:
wavelengths λ1 and λ2 of the two centre of the ring. The equivalent (a) 0.5 H (b) 0.05 mH
fragments respectively: resistance between A and B is: (c) 0.5 µH (d) 5 µH
Practice Set-24 301

33. A body is projected horizontally µ 0 I In 2 ˆ (a) [A–1] (b) [A]


with a speed of 20 ms–1. Its speed (a) k
8π 3a (c) [A–2] (d) [A2]
nearly after 5 s is:
µ I In 4 ˆ 44. In the circuit given below, the
(a) 54 ms–1 (b) 20 ms–1 (b) 0 k
(c) 50 ms–1 (d) 70 ms–1 4π 3a charge in µC, on the capacitor
34. The moment of inertia of a disc µ 0 I In 4 ˆ having capacity 5µF is:
(c) (−k )
of radius R and mass M about a 4π 3a
diameter as axis is:
(d) Zero
(a) MR2 (b) MR 2 39. A 2.0 V potentiometer is used to
2 determine the internal resistance
2 of a 1.5 V cell. The balance point
(c) MR (d) 5 MR 2 of the cell in open circuit is 75
4 4 cm. When a resistor of 10 Ω is
35. Two wires A and B are streteched connected across the cell, the
between two fixed points. The balance point shifts to 60 cm. The
density, diameter and tension of internal resistance of the cell is:
B are double of A. The ratio of (a) 4.5 (b) 9
(a) 1.5 Ω (b) 2.5 Ω
(c) 7 (d) 15
frequency to that of A is: (c) 3.5 Ω (d) 4.5 Ω 45. The energy band gap is maximum
(a) 4 : 1 (b) 1 : 4 40. The resistance of a germanium in:
(c) 2 : 1 (d) 1 : 2 junction diode whose V – I is (a) metals
36. A slab consists of two parallel shown in fugure is (Vk = 0.3 V): (b) superconductors
layers of two different materials (c) insulators
of same thickness and having (d) semiconductors
thermal conductivities in the
ratio 3 : 2. If one end is at steam MATHEMATICS
point temperature and other
at 25°C, the temperature of 2 3
46. If f(x) = x + , x ∈ R, then what
interfaces is: 3 2
(a) 50°C (b) 62.5°C is f–1(x) equal to?
(c) 70°C (d) 60°C
37. A bullet of mass a is fixed on a (a) 5 kΩ (b) 0.2 kΩ 3 2 3 9
(a) x+ (b) x−
big wooden block of mass c with  10  2 3 2 4
velocity b. The final velocity of (c) 2.3 kΩ (d)   kΩ
2.3 
block, when bullet gets imbedded 2 4 2 2
41. Value of acceleration due to (c) x– (d) x–
in the block is: gravity is 9.8 m/s2. Its value in km/ 3 9 3 3
(a) b (b) b + a hr2 is:
·a 1+ ω ω2 −ω
a+b c (a) 110583 km/hr2
47. The value of 1 + ω 2
ω −ω 2
a a+c (b) 123147 km/hr2
(c) ·b (d) ·b is equal to 2
a+c a (c) 135908 km/hr2 ω + ω ω −ω 2
38. Infinite number of straight (d) 127008 km/hr2 (ω is an imaginary cube root of
wires each carrying current I 42. The magnification of a compound
microscope is 30 and the focal unity.)
are equally placed as shown in
the figure. Adjacent wires have length of its eye piece is 5 cm. (a) ω (b) 2ω
The magnification produced by (c) 2ω2 (d) –3ω2
current in opposite directions.
the objective, when the image is
Net magnetic field at point P is: to be formed at least distance of log 2 17
distinct vision (25 cm) is: 48. log 8 17 − 2 is equal to:
log 9 23 log 3 23
(a) 5 (b) 6
(c) 8 (d) 10 (a) 0 (b) 1
43. The dimensions of L are:
23
RCV (c) 17 (d)
L = self induction, R = resistance 8 17
V = voltage, C = capacitance
302 Practice Set-24

2 3 points ( ± b2 − a2 , 0 ) to the line Wage (in `) Number of


y  y  y
49. If x = 1 + + +   + ..... ,
2  2   2 ax cos φ + by sin φ = ab? workers
where | y | < 2, what is the value (a) a2 (b) b2 800 7
of y? (c) ab (d) a/b 820 14
55. The coefficient in the expansion
(a) x – 1 (b) x − 1 860 19
x 2x e7 x + e3 x
of is: 900 25
e5 x
920 20
(c) 2 x − 2
2x + 1
(d) (a) 0 (b) 1
x 2x 980 10
(c) 2 (d)  N o n e o f
50. In the expansion of (1 + x) n , these 1000 5
what is the sum of even binomial 56. Find the equations of the circle (a) ` 890 (b) ` 890.5
coefficients? whose centre is at (–1, 2) and (c) ` 891.2 (d) ` 8918
(a) 2n (b) 2n –1 which passes through the point 61. For all real x, the minimum
(c) 2n + 1 (d) None of (3, 5). 1 − x + x2
these value of is
(a) x2 + y2 + 2x + 4y + 20 = 0 1 + x + x2
51. The function f(x) (b) x2 + y2 + 2x – 4y + 20 = 0 1
(a) 0 (b)
 x2 / a ,0 ≤ x < 1  (c) x2 + y2 – 2x + 4y + 20 = 0 3
 a ,1 ≤ x < 2  is
=  (d) x2 + y2 + 2x – 4y – 20 = 0 (c) 1 (d) 3
 (2 b2 − 4 b) / x 2 
 , 2 ≤ x < ∞ 57. W h a t i s t h e v a l u e o f l o g 62. From a committes of 8 persons,
 1  in how many ways can we choose
( a + a2 + 1) + log  ?
continuous for 0 ≤ x < ∞ , then
 a + a2 + 1  a chairman and a vice-chairman
the most suitable values of a
(a) 1 (b) 0 assuming one person cannot hold
and b are
1 more than one position?
(a) a = 1, b = –1 (c) 2 (d) (a) 54 (b) 55
(b) a = –1, b = 1 + 2 2
58. Let P(n): n2 + n + 1 is an even (c) 52 (d) 56
(c) a = –1, b = 1 63. A card is drawn from a well
integer, If P(k) is assumed true
(d) None of these shuffled pack of cards. The
⇒ P(k + 1) is true. Therefore,
52. If x = cos t and y = sin t, then probability of getting a queen of
P(n) is true:
d2 y club or king of heart is:
what is equal to? (a) for n > 1
dx 2 (a) 1/52 (b) 1/26
(a) y–3 (b) y3 (b) for at n ∈ N (c) 1/13 (d) None of
(c) –y–3 (d) –y3 (c) for n > 2 these
1 (d) None of these π dx
53. ∫ dx is equal to: 59. The solution of the equation (x2 64. What is ∫ equal to?
3 0 1 + 2 sin 2 x
(sin x cos x)
+ xy)dy = (x2 + y2) dx is:
π
−2 (a) π (b)
(a) +C y 3
(a) log x = log (x – y) + + C
(tan x) x π π
(c) (d)
(b) 2 tan x + C y 3 3
(b) log x = 2 log (x – y) + +C
x 65. If a = 4, b = 3, ∠A = 60º then c
(c) 2
+C is the root of the equations.
(tan x) y
(c) log x = log (x – y) + +C (a) c2 – 3c – 7 = 0
x
(d) −2 tan x + C (b) c2 + 3c + 7 = 0
(d) None of the above
54. What is the product of the (c) c2 – 3c + 7 = 0
60. Find the mean wage from the
perpendicular from the two (d) c2 + 3c – 7 = 0
data given below:
Practice Set-24 303

66. If 4 sin–1 x + cos–1 x = π then x is (a) The Speaker of Lok Sabha cm and its radius is 35cm, find
equal to: (b) Chief Justice of India its volume.
(c) The Prime Minister of India (a) 6930 cubic cm
1 3
(a) (b) (d) None of these (b) 13860 cubic cm
2 2
73. Buddhist Scriptures are (c) 3465 cubic cm
1 1 (d) 107800 cubic cm
(c) – (d) contained in Tri-Pitakas. What is
2 2 the meaning of Pitakas ? 82. Daily local train ticket costs ` 30
67. The foci of the ellipse 25 (x + 1)2 (a) Baskets (b) Bank and Monthly Pass costs ` 670.8.
+ 9 (y + 2)2 = 225 are: (c) Box (d) Book If I buy the Monthly Pass and
(a) (–1, 2) and (–1, –6) 74. Digboi Oil Refinery is located in travel for 26 days in a month
(b) (1, 2) and (1, 6) the state of : than I save ?
(c) (1, 2) and (–1, –6) (a) Punjab (b) Assam (a) 12 percent (b) 14 percent
(d) (–1, 2) and (1, 6) (c) Gujarat (d) Bihar (c) 25 percent (d) 10 percent
68. What is the maximum value of 75. Which part of potato is edible ? 83. If a merchant offers a discount
(a) Fruit (b) Flower of 20% on the list price, then he
sin2 x?
(c) Leaf (d) Stem makes a loss of 10%. What %
(a) –1 profit or % loss will she make if
(b) 0 76. Expand the term CD-ROM :
he sells at a discount of 5% of the
(c) 1 (a) Compactable Disk Read only list price ?
Memory
(d) infinity (a) 6.875 percent profit
(b) Compact Disk Read only
1° (b) 20 percent profit
69. What is the value of sin 292 ? Memory
2 (c) Compactable Data Read only (c) 14 percent profit
1 1 Memory (d) 30 percent loss
(a) 2+ 3 (b) − 2− 3 84. Meghnad can do a piece of
3 3 (d) Compact Data Read only
Memory work in 45 hours. If he is joined
1 1 77. Name the Indian delegate, who by Jayshri, who is 50% more
(c) 2 + 2 (d) − 2+ 2
2 2 participated in the 12th session efficient, in what time will they
of Conference of States Parties together finish the work ?
70. Find x, y, z and w, if
to the Convention to Rights of (a) 9 hours (b) 6 hours
3x y Persons with Disabilities (CRPD)
= x+ 4 x− y  (c) 12 hours (d) 18 hours
 2 z 3 w  z + w z − w + 3 x  2019 held in New York.
85. The effective annual rate of
(a) 2, 1, 2, 2 (b) 2, 2, 2, 2 (a) Shakuntala Doley Gamlin interest corresponding to a
(c) 2, 1, 1, 2 (d) 1, 2, 1, 2 (b) Pramod Kumar Misra nominal rate of 7% per annum
(c) Raghvendra Singh payable half yearly is :
REASONING AND GENERAL (d) Rashmi Verma (a) 14 percent
AWARENESS (RAGA) 78. If 6/7th of 8/5th of a number is (b) 14.25 percent
192, then 3/4th of that number is
71. Unemployment which occurs ........ (c) 7 percent
when workers move from one (d) 1.72 percent
(a) 105 (b) 77
job to another job is known as :
(c) 36 (d) 80 86. K is the brother of N and X. Y
(a) Cyclical unemployment
79. What is the value of cos 4p/3? is the mother of N and Z is the
(b) Technological unemployment (a) + 1/2 (b) √3/2 father of K. Which following
(c) Seasonal unemployment (c) – 1/2 (d) –√3/2 statements is not definitely true?
(d) Frictional unemployment 80. ? = 1001.101 + 101.01 + 11.11 (a) K is the son of Z.
72. If the President wishes to resign, (a) 1122.221 (b) 1113.221 (b) Y is the wife of Z.
he should submit his / her (c) 1113.23 (d) 1022.312
(c) K is the son of Y.
resignation in writing addressed 81. If the curved surface area of a
to : right circular cone is 10010 sq (d) N is the brother of X.
304 Practice Set-24

87. P is the brother of Q and R. S (a) RPQ Answer figures:


is R's mother. T is P's father. (b) TWS
Which of the follow statements
(c) QPR (a) (b)
cannot be definitely true?
(d) None of the above
(a) T is Q's father.
(b) S is P's mother. Direction (92): Study the following
information answer the questions (c) (d)
(c) P is S's son.
given below it. In a certain code, the
(d) Q is T's son. symbol for 0 is © and for 1 is ⊕. There
88. B is to the South-West of A, C is are no other symbols for all other Directions (96–100): In each of the
to the East of B and South-East numbers greater than 1. The number following questions, a group of letters
of A and D is to the North of C greater than 1 are to be written only is to be coded by number/symbol
in line with B and A. In which by using the two symbols given above. codes as per the codes given below
The value of symbol for 1 doubles and the conditions that follow. In
direction of A is D located?
each question, a group of letters is
(a) North (b) East itself everytime it shifts one place to
given followed by groups of number/
(c) South-East (d) North-East the left. Study the following example:
symbol code numbered (a), (b), (c) and
89. Mohan correctly remember 92. Which of the following will (d). The correct combination of codes
that his father's birthday is represent ‘10’ ? is your answer. If none of the four
before 20th January but after (a) © © ⊕ ⊕ © combinations is correct, your answer
16th January whereas his (b) ⊕ © ⊕ © is (e), i.e. ‘None of these’.
sister correctly remembers (c) ⊕ ⊕ ⊕ © Letter P D A F L H M R K T B E U
that their father's birthday is Code 3 6 # 8 $ 4 2 7 @ 9 5 1 %
(d) ⊕ © ⊕ ⊕
after 18th January but before Conditions
23rd January. On which date Direction (93) : Select the related
word/letters/number from the given (i) If the first letter is a vowel and
in January is definitely their the last letter is a consonant,
father's birthday? alternatives.
both are to be coded as ‘0’.
(a) 18th 93. HEK : MJP : : NKQ : ? (ii) If the first letter is a consonant
(b) 19th (a) SRX (b) SQY and the last letter is a vowel,
(c) SPV (d) SUW both are to be coded as'©'.
(c) 20th
94. Identify the diagram that best (iii) If the first as well as the last
(d) Data inadequate let­ters are vowels, both are to
represents the relationship
Direction (90): A series is given with among the given classes. be coded as the code for first
one term missing. Choose the correct let­ter.
Football, Volleyball, Outdoor
alternative from the given ones that 96. KEDURT
Games, Carrom.
will complete the series. (a) @ 16%7 @
90. Seed, Stem, leaf, ? (a) (b) (b) 916%7 @
(c) 916%79
(a) Plant (b) Tree
(d) None of the above
(c) Fruit (d) Flower
(c) (d) 97. EDRTMP
Direction (91): Study the following
(a) 167923 (b) 167921
information carefully and answer the
questions given below: 95. Which answer figure will (c) 067920 (d) 067923
P, Q, R, S, T, V and W are sitting complete the pattern in the 98. FHRAMU
around a circle facing at the centre. V question figure? (a) 847 # 28 (b) %47 # 28
is second to the left of P and second to Question figure : (c) 847 # 2% (d) © 47 # 2 ©
the right of W. T is third to the right of 99. ULTPHE
Q and is not an immediate neighbour (a) % $ 934% (b) 1 $ 9341
of V, S is third to the right of R. (c) % $ 9341 (d) 1 $ 934%
91. In which of the following groups 100. HKEDLB
?
is the first person sitting between (a) 5 @ 16$4 (b) 4 @ 16$5
the second and the third persons? (c) ©@ 16$4 (d) 0 @ 16$5
Practice Set-24 305

Answers with Explanation


19. (c) Either of is followed by Plural = 2 × 10–3 kg
ENGLISH Noun/Pronoun but a Singular Verb. = 2 gram
1. (d) It suggests that he/she is Hence, was at home should be used. Since the gram-molecular
condemning the behaviour of ill-tempered 20. (c) The word repeat means: to say weight of gas is 2 g, therefore,
human beings. or write something again or more than this gas is hydrogen.
2. (b) The writer is nationalistic in his once.
outlook. Hence, repeated the word again dV
24. (b) E = −
3. (c) It means that each hive is a should be replaced by said the word dr
self-governing. again. Also,
4. c) It means that the bees are busy
toiling. PHYSICS σ σ σ σ
E= =
= =
5. (a) The correct spelling is: ε ε0 ε r ε0 × 1 ε0
onomatopoeia which means the fact of h
21. (c) λ = · (From law of conser- σ = Eε0 = 250 × 8.86 × 10–12
words containing sounds similar to the p
noises they describe. For example, hiss. = 2.21 × 10–9 C/m2
vation of momentum) y
6. (d) The correct spelling is: → →
bureaucracy. | p1 | = | p2 |
I
7. (c) The word Conceited (Adjective) ∴ λ1 = λ2
means: having too much pride in yourself ∴ Ratio = 1 : 1. 25. (d) x
and what you do. 22. (b) Initial momentum of surface,
Look at the sentence: E
pi = , where c is velocity of
It is very conceited of you to assume c
that your work is always the best. light. Since, here total flux entering
equals the total flux leaving
The word Modest (Adjective)  Since, the surface is perfectly
and hence net flux equals zero.
means: not talking much about your own reflecting, so the same
abilities or possessions. momentum will be reflected 26. (b) y = ax2
Hence, the words conceited and completely. Final momentum dy
modest are antonymous. ⇒
= y = (2ax)x
pf = E dt
8. (c) The word Deterrent (Noun)
{ }
c
means: a thing that makes somebody less ∴ Change is momentum dx
= 2acx  =x=c
likely to do something. −E E dt
∆p = pf – pi= − Further a
Hence, the words deterrent and c c
encouragement are antonymous. −2E d2y
= = = y = 2acx = 2ac 2
9. (d) Idiom on the cards means: c dt 2
anticipated; likely to happen. 27. (b) Due to flow of current in same
Thus, momentum transferred
10. (b) The word Corroborate (Verb) direction in two adjacent loops,
2E
means: to provide evidence or information to the surface is · an attractive magnetic force
that supports a statement, theory, etc; c
will be produced due to which
confirm. 2RT spring will get compressed.
23. (a) Vrms =
Look at the sentence: M
The evidence was corroborated by two 3RT i
independent witnesses. ⇒ M =
2
(Vrms )
Hence, the words corroborate and
substantiate are synonymous. ⇒ R = 3.31 Jmol–1 K–1
T = (273 + 27 K) = 300 K
11. (c) 12. (c)
and vrms = 1930 m/s
13. (d) Let the door be opened.
14. (a) Nobody was trusted by me. 3 × 8.31 × 300
∴ M =

15. (a) 16. (a) 17. (d) 18. (d)\ (1930) 2
306 Practice Set-24

28. (a) Since, R∝I 1 2  qE  2 K ·A(θ − 25)


⇒ K.E. =
m  y =
2  m ∆x
R

⇒ R1 = (aθ) ⇒ K.E.= qEy.
2πa ⇒ 300 – 3θ = 2θ – 50
32. (b) M = µ0n2n1Al
Rθ 5θ = 350

R1 = 4
2π = 4π × 10−7 × 500 × 200 × ×1 ⇒ θ = 70°C
R1 104 37. (c) F r o m c o n s e r v a t i o n o f
A B = 160π × 10–7 momentum
= 0.05 × 10–3 H. a.b + c.0 = (a + c)v
θ
a a R2 33. (a) v= u 2 + g 2t 2 ab
c ∴
v=
v= 400 + (100)(25)
a+c
= 54 ms–1 38. (a) Magnetic field due to straight
34. (c) By theorem of perpendicular wire is:

Similarly, axes µ I
B = 0 [sin θ1 + sin θ 2 ]
R Iz = Ix + Iy 4πd
R2=
(2πa − aθ)
2πa → → →
MR 2 ∴ B = B1 + B2 + .....

R ⇒
= Ix + Ix = 2Ix

R2 = (2π − θ) 2
2π →
( I y = I x ) B = B1kˆ + B2 ( − kˆ) + B3 kˆ + ....

R1 R2 B = B1 – B2 + B3 – B4 + ....

Since, RAB = MR 2
R1 + R2 ∴
Ix =
4 µ0 I  1 1 1 
⇒B=
1 − + − + ....
Rθ(2π − θ) 16πd  2 3 4 

RAB = nA TA rB dB
4π 2 35. (c) = × ×
nB TB rA dA
29. (c) On plucking at 25 cm distance
the wire vibrates in two T 2r 2d
= × ×
segments. Therefore, 2T r d θ1 θ1 = 30º
λ =1m 2 d θ2
= θ2 = 30º
1 20 1

n =
1 5 × 10 −4 ∴ nA : nB= 2 : 1
36. (c) For slabs in series, same
= 4 × .104 amount of heat passes through
= 200 Hz each slab. Here, cos 30° = d

a
30. (d) Rate of heat conduction ∆x ∆x
3
⇒ d = a cos 30° =
a
KA·∆θ 2
Q/t =

l µ0 I × 2
2 2
∴B=
ln 2
K .πr .∆θ r 16π 3a
= ∝ 100ºC θ1 25ºC
l l µ I ln 2
B= 0

r2/l is maximum for condition. 8π 3a
1 3K 2K l 
31. (b) K.E. = mv 2 39. (b) Since, r =  − 1 × R
2  l′ 
qE Q 3K ·A(100 − θ)  75 
v2 = 2ay where a =
= ⇒ r =  − 1 × 10

m t ∆x  60 
Practice Set-24 307

15 L 0 ω2 1

r= × 10 = ⇒
60 R × Current × Time 0 ω 1
= 2.5 Ω Putting dimensions 2ω + ω ω ω 2

∆V L [T ]
40. (b) R= = = [ A−1 ] Expand according to c1
∆i RCV [ A][T ]
⇒ 0 – 0 + [2ω + ω2] {ω2 – ω}
2.3 − 0.3 44. (b) Potential difference across the ⇒ 2ω3 – 2ω2 + ω4 – ω3 {ω3 – 1}
=
10 × 10 −3 branch de is 6 V.
⇒ ω3 – 2ω2 + ω2 . ω
R = 0.2 × 103 Ω Net capacitance of de branch
is 2.1 µF. ⇒ 1 – 2 ω2 + 1 × ω
= 0.2 kΩ
−2 So, q = CV ⇒ 1 + ω2 – 2 ω2 {1 + ω + ω2}
 L2   T2 
41. (d) n1 = n2     , ⇒ q = 2.1 × 6 µC ⇒ – ω2 – 2 ω2 {1 + ω = ω2}
 L1   T1  ⇒ q = 12.6 µC ⇒ –3 ω2
[Here n2 = 9.8] P o t e n t i a l a c r o s s 3 µ F
capacitance is 1 2
48. (a) log 2 17 log 2 17
 1m   1s 
−2 3 −3
∴ n1 = 9.8 
  12.6 1 log 2 23
1km  1hr  V= log 2 23
3 2
 1m   1s 
−2 = 4.2 volt 2 2
= 9.8  Potential across 2 and 5 log 2 17 log 2 17
  3
1000 m   60×60s  combination in parallel is = −3 =0
log 2 23 log 2 23
 1  6 – 4.2 = 1.8 V
= 9.8  × 3600 × 3600  So, q′ = (1.8) (5) y  y  y
2 3
1000  49. (c) x = 1 + +   +   + ......
= 9 µC 2 2 2
= 98 × 36 × 36 45. (c) Metals have minimum energy
g = 127008 km/hr2 gap and insulators have 1
⇒ x = ⇒x= 2
42. (a) When final image is formed at maximum energy gap and y 2− y
1−
least distance of distinct vision semiconductors have energy 2
then the magnifying power gap lying between insulators ⇒ 2x – xy = 2 ⇒ y = 2 x − 2
D and metals. x
= M 0 × 1 +  50. (c) We know that,
 fe  MATHEMATICS
(1 + x)n = nC0 + nC1x + nC2x2
 25  2 3
30 = M 0 1 +  46. (b)  f(x) = x+ = y (say)
+ nC3x3 + nC4x4 + ... + nCnxn
 5 3 2
and (1 – x)n = nC0 – nC1x + nC2x2
M0 = 5
6y − 9
⇒ 4x + 9 = 6y ⇒ x = – nC3x3 + nC4x4 + ..+ (–1)n Cnxn
4
∴ (1 + x)n + (1 – x)n = 2nC0 + 2 nC2x2
43. (a) L 6 x − 9 3x 9
⇒ f–1(x) = = − + 2 nC4x4 + ..
RCV 4 2 4 Put x = 1, (1 + 1)n + (1 – 1)n
L 1+ ω ω2 −ω
= 2 {nC0 + nC2 + nCn + .....}
Here, = time constant
R 47. (d) 1 + ω 2 ω −ω 2
1 n
2 ⇒ nC0 + nC2 + nC4 + .. =
(2 + 0)
= [T] ω + ω ω −ω 2 2
Q ⇒ c1 → c1 + c2 c → c3 + c1 ⇒ nC0 + nCy + nCx + .... = 2n + 1
and C= 51. (c) For continuty at x = 1
V 1 + ω + ω2 ω2 1
L L L ⇒ 1 + ω + ω 2
ω 1 1

= = RHL = LHL= V ⇒ a
RCV Q RQ 2ω + ω 2 ω ω a
R ·V
V
∴ a = 1, – 1
{1 + ω + ω2 = 0}
308 Practice Set-24

[a 2 (b 2 − a 2 ) cos 2 φ − a 2 b 2 ] 59. (b) Putting y = vx


For continuty at x = 2 d1d2 = −

a 2 cos 2 φ + b 2 sin 2 φ
RHL = RHL = V = a + b2 – 2b 1+ v dx
dv =
When a = 1, b2 – 2b – 1 = 0
a 2 (−b 2 sin 2 φ − a 2 cos 2 φ) 1− v x
= – − 2 2 = a2
a cos φ + b 2 sin 2 φ
∴b
= 1± 2  2  dx
⇒  −1 +  dv =
7x 3x
55. (c) e + e = e2x + e–2x We know that  1– v x
When a = –1, b2 – 2b + 1 = 0
e5 x ⇒ – v – 2 log (1 – v) = log x + C
∴b = 1 2 x (2 x) 2 (2 x)3
e2x = 1 + + + + ...∞ ...(i) (integrate)
52. (c)  x = cos t, y = sin t 1! 2! 3!
2 x (2 x) 2 (2 x)3 (2 x) 4
e =1 − +
–2x
− + − ...∞\\ y  x − y
dx dy 1! 2! 3! 4

∴ = –sin t and = cos t ⇒−
− 2 log  = log x + C
dt dt x  x 
...(ii)
On adding Eqs. (i) and (ii), we get y
dy dy / dt cos t

= =– = – cot t ⇒−
− 2 log ( x − y ) + 2 log x
dt dx / dt sin t  (2 x) (2 x) 2
 4 x
e2x + e–2x = 2 1 + + + ....∞ 
d2y d  dy  dt  2! 4!  = log x + C
=  × y
dx 2 dx  dx  dx Hence, 2 is a coefficient term. ∴ log x = 2 log (x – y) +
+C
56. (d) T h e c e n t r e o f t h e c i r c l e x
d2y dt 60. (c) Let the assumed mean be A
2 = cosec2 t . is at (–1, 2). Since, the point
dx dx = ` 900. The given data can
(3, 5) lies on the circle, the distance
of the centre from this be written as under
= – cosec2 t 1 = − 1
sin t sin 3 t point is tha radius r of the circle.
2
d y Therefore, we obtain Wage Number dxi = xi fidi
∴ 2 = –y–3 (in `) xi of –A
dx r= 2 2
(−1 − 3) + (2 − 5) = 16 + 9 = 5
workers = xi –
1 The equation of the circle with fi 900
53. (a) I = ∫ 3
tan x cos x 4
dx centre at (–1, 2) and radius 5 is
given by 800 7 –100 –700
sec 2 x (x – 1)2 + (y – 2)2 = 52 820 14 –80 –1120
= ∫ (tan x) 3/ 2
dx
⇒ x + y + 2x – 4y – 20 = 0
2 2 860 19 –40 –760
Put tan x = t
57. (b) 900 25 0 0
⇒ sec2 x dx = dt  1 920 20 20 400
log (a + a 2 + 1) + log   2
−2  a + a +1  980 10 80 800
∴ I = ∫ t 3/ 2 dt =
+C 1000 5 100 500
tan x ⇒ log (a + a 2 + 1) + log (a + a 2 + 1) −1
54. (a) Given, ax cos φ + by sin φ – ab Σfi = 100 Σfidi = –880
⇒ log (a + a 2 + 1) – log (a + 2
a + 1)
=0
[log m = n log m] = 0
n Here, A = 900
At point ( + b 2 − a 2 , 0 )
58. (d) Given, P(n) = n2 + n + 1 ∴ Mean = X
2 2
a b – a cos φ − ab At n = 1, P(1) = 3, which is not an

d1 =
= A + 1/N (Σfidi)
a 2 cos 2 φ + b 2 sin 2 φ even integer.
∴ P(1) is not true (principle of
880 
At point ( − b 2 − a 2 , 0 ) = 900 +  −
induction is not  100 
− a b 2 − a 2 cos φ − ab applicable).
d2 =
= 891.2
a 2 cos 2 φ + b 2 sin 2 φ Also, n(n + 1) + 1 is always an odd
Hence, mean wage = ` 891.2
integer.
Practice Set-24 309

61. (b) Let π dx 67. (a) The given equation of an ellipse


= ∫ 0  1 − tan x  2 can be rewritten as
1 − x + x2 = 2x 2−
y = 1−
1+ x + x 2
1 + x + x2  1 + tan 2 x  ( x + 1) 2 ( y + 2) 2
+ =1
9 25
2 2 π sec 2 x
⇒ y = 1 −
1
= 1−
t
= ∫ 0 1 − 3 tan 2 x
dx whose centre is (–1, –2) and semi-
+ x +1 major and semi-minor axes are 5
x and 3 respectively i.e.,
sec 2 x
π/ 2
2 = 2∫ dx b = 5 and a = 3
Since, y is minimum when is 0 1 + 3 tan 2 x
maximum t (By definite integral property) ∴ The eccentricity of the ellipse is
1 given by
⇒ t = + 1 + x is minimum 2 π /2 sec 2 x a2 = b2 (1 – a2)
3 ∫0  1  2
x = dx
2 4
dt 1   + (tan x) ⇒ 9 = 25 (1 – e2) ⇒ e =
= 1− 2 = 0 3 5
dx x
Put t = tan x ⇒ dt = sec2 x dx On shifting the origin at (–1, –2)
2 ∞ dt the given equation reduces to
d 2t 2 ∴ I = ∫
∴ x = 1 – 1, 2
= 3 3  1  2
0 x2 y 2
dx x + =1 ...(i)
 +t 9 25
 3
d 2t 2 where, x = X – 1 and y = Y – 2
At x = 1, = > 0 , (minimum) 2
dx 2 t −1 ∞
= × 3[tan ( 3 − t )]0 ...(ii)
3 ∴ Coordinates of the foci are (X
2 1
∴ y(min) = 1 − = 2 = 0, Y = + be) i.e., (–1, 2) and
3 3 = [tan −1 (∞) − tan −1 (0)] (–1, – 6)
3
62. (d) Out of B persons, one person 68. (c) Let f(x) = sin2 x
can be chosen for chairman in 8 2 π  π 1
=  − 0 = f(x) = (2sin 2 x)
ways. Now, a person cannot hold 3 2  3 2
more than one position i.e., out
65. (a) By cosine rule cos A 1
of 7 persons, one person can be = (1 − cos 2 x)
2 2 2
chosen for vice-chairman in 7 =
b +c −a 2
ways. 2bc 1 1
f(x) = − cos 2 x
Hence by FPC total number of 2 2
(3) 2 + c 2 − (4) 2
ways = 8 × 7 = 56 cos 60º =  −1 ≤ cos 2 x ≤ 1
2×3× c
63. (b) n(S) = 52 1 1 1
n(E1) = 1, n(E2) ⇒ − ≤ cos 2 x ≤
1 9 + c 2 − 16 2 2 2
⇒ =
2 2×3× c
= 1, n(E ∩ E 2 ) = φ ⇒ 1 ≥ − 1 cos 2 x ≥ −1
⇒ 3c = c2 – 7 = c2 – 3c – 7 = 0 2 2 2
∴ P(E1 ∪ E 2 ) π −1 
66. (a) 4 sin–1 x +  − sin x = π ⇒ 1 + 1 ≥ 1
= P(E1) + P (E2) – P(E1 ∩ E 2 ) 2 2 2 2

1 1 1 π π 1 1 1
= ⇒ 3 sin–1 x = π − = – cos 2 x ≥ − ⇒ 0 ≤ f ( x) ≤ 1
+ −0 = 2 2 2 2 2
52 52 26
Hence maximum value of sin2 x is 1.
dx
⇒ sin–1 x = π
π
64. (c) I = ∫ 0 1 + 2sin 2 x 6
 1
69. (d) sin  292  º
 2
π dx π 1
= ∫ 0 2 − cos 2 x ⇒ x = sin = sin 30º =
= sin
585º
=−
1 − cos 585º
6 2 2 2
310 Practice Set-24

canons of scriptures. The 3 pitakas = 126 × 56


1 − cos 225º 1 + cos 45º
= − = are: Sutta Pitaka, Vinay Pitaka and
2 2 Abhidhamma Pitaka.         = 4× 4× 3× 3× 7 × 7
74. (b) Digboi Oil Refinery is
1 + 1/ 2 1 situated in the Tinsukia district of h = 4 × 3 × 7 = 84
=− = − 2 +1
2 2 Assam. Commissioned by Assam oil 1 2
V = πr h
Company Ltd in 1901, it is India’s 3
2 +1 2 1 oldest refinery. 1 22
= − × = − 2+ 2 = × × 35 × 35 × 84
2 2 2 2 75. (d) The edible part of potato 3 7
is a tuber (i.e., the swollen end of an   = 107800 cubic cm
Where, negative sign shows that underground stem). Potato is one of 82. (b) Cost of ticket for 26 days
º some 150 tuber-bearing species of the = ` 30 × 26
 1
sin  292  : lies in the third quadrant. genus Solanum (family Solanaceae). = ` 780
 2 76. (b) A CD-ROM is a pre-pressed Cost of monthly pass = 780 – 670·8
70. (a) The matrix on each side of order optical compact disc which contains = 109·2
data. The name stands for ‘‘Compact
2 × 2. Equating the corresponding Disc Read-Only Memory’’. Computers 109·2 × 100
Saving% = = 14%
elements, we get can read CD-ROMs, but cannot write on 780
the CD-ROMs which are not writable or
3x = x + 4 ⇒ 2x = 4 ⇒ x = 2 83. (a) Let the M.P. of article be x
erasable.
y = x – y ⇒ 2y = x = 2 ⇒ y = 1 77. (a) Mrs Shakuntala Doley x × 80 4
S.P. = = x
2z = z + w ⇒ z = w Gamlin, Secretary, Department 100 5
of Empowerment of Persons with
3w = z – w + 3x ⇒ z = 4w – 6 Disabilities (DEPwD) under Ministry C.P. × 90 9
S.P. = = C.P.
of Social Justice and Empowerment 100 10
On substituting w in terms of z,
participated in the conference. 4 9
we get x = C.P.
78. (a) Let the number be x 5 10
z = 4z – 6 ⇒ 3z = 6 ⇒ z = 2 x×6 8
   × = 192 40 8
Hence, we obtain x = 2, y =1, z = 7 5 C.P. =x= x
45 9
2 and w = 2. 192 × 35 24 S.P. of 5% discount
  x = = × 35 = 140
6×8 6 95 19
REASONING AND GENERAL = x= x
AWARENESS (RAGA) 3 3 100 20
   of x = × 140 = 3 × 35 = 105 S.P. > C.P.
71. (d) Frictional unemployment 4 4
is unemployment that occurs because it  19 8 
4x  π
takes workers some time to move from 79. (c) cos= cos  π +   x − x 
3  3 20 9
one job to another. Unemployment due Profit% = × 100%
= – cos p/3 = – cos 60º = – 1/2 8
to frictional unemployment is usually x
thought to last only short period of time. 9
80. (a) ? = 1001.101 + 101.01 +
72. (d) According to Article 56 of 11.11  171x − 160 x 
the Indian Constitution, the President ? = 1122.221   × 100%
180
may, by writing under his hand addressed =
81. (d) prl = C.S.A. of a cone 8
to the vice-president, resign his office. x
22 9
Any resignation addressed to the vice-
× 35 × l = 10010
President under the (above mentioned) 7 11x × 9 × 100%
clause should be communicated by him 10010 = = 6·875%
180 × 8 x
to the speaker of the Lok Sabha. l= = 91cm
110
73. (a) Tripitaka is a Pali word 84. (d) Work done in 1 hour by
meaning Three Baskets. It is the h= 2
l −r 2
= 2
91 − 35 2 Meghnad
traditional term used by Buddhist 1
traditions to describe their various = (91 + 35) (91 − 35) = parts
45
Practice Set-24 311

Work done in 1 hour by Jaishree 87. (d) P, Q, R are children of same not

Q  to the left of P and R
3 parents. So, S. who is R's mother and T,
= parts between P and R.
45 × 2 is P's father will be mother and father of not

Q  to the right of V and S
1 3 all three. However, it is not mentioned
between V and S.
Work done by both = + whether Q is male or female. So, (d)
45 90 cannot be definitely true. Therefore, none of the options is
3+ 2 5 88. (d) According to the question, correct.
= =
90 90 the direction diagram will be as follows 92. (b) Using the given pattern is
Time taken by both option (b), we get
⊕ © ⊕ © = 8 + 0 + 2 + 0 = ‘10’.
90
= = 18 hours. 93. (c)
5 H E K M J P N K Q S P V
85. (d) Difference of S.I. and C.I. 8 5 11 13 10 16 14 11 17 19 16 22
n −3 +6 −3 +6 −3 +6 −3 +6
 R   P × 7 ×1
= P 1 +  – 1 – Clearly, D is the North-East of A. Finally, the related word is SPV.
 100   100
89. (b) Days by Mohan, [17th, 18th 94. (b)
 2  7P
= P 1 + 3.5  – 1 – or 19th] January. Outdoor Games
  100   100 Days by his sister, [19th, 20th, 21st Carrom
or 22nd] January. Foot- Volley-
 2  7P ball ball
= P  103.5  – 1 – Required day is 19th January as it is
 100   100 common in both the groups.
 (103.5) 2 – (100) 2 7  90. (d) The series is: Football and Volleyball are outdoor
= P  2
–  Seed, Stem, Leaf, Flower games, but Carrom is indoor game.
 (100) 100 
Finally, the missing term is flower 95. (a) Answer figure (a) will
 203.5 × 3.5 7  which complete the series. complete the pattern of the question
= P  2
– 
 (100) 100  [Hints 91]: According to the given figure.
information, we get 96. (d) K E D U R T
 712.25 – 700  12.25
= P  2  = P V  Second to the left of P and ↓ ↓ ↓ ↓ ↓ ↓
 (100)  (100) 2
Second to the right of W. @ 1 6 % 7 9
Effective% of S.I. and C.I. T  Third to the right of Q. Condition not applicable
P ×12.25 S  Third to the right of R 97. (c) E D R T M P
(100) 2 So, the circular arrangement is as ↓ ↓ ↓ ↓ ↓ ↓
= × 100%
712.25 P ×1 follows, 0 6 7 9 2 0
(100) 2 Condition (i) is applicable.
98. (d) F H R A M U
12.25
× 100% ↓ ↓ ↓ ↓ ↓ ↓
= 712.25
 4 7 # 2 
1225
= × 100% = 1.72% Condition (ii) is applicable.
71225
99. (a) U L T P H E
86. (d) K is the brother N and X, So,
91. (d) By analysing the above ↓ ↓ ↓ ↓ ↓ ↓
Y, who is the mother of N and Z, who is
the father of K, will be mother and father circular arrangement, we get % $ 9 3 4 %
not
of N, K and X of whom K is a male. R  to the right of P and Q Condition (iii) is applicable.
Thus, K is the son of Y and Z, and Y is between P and Q. 100. (b) H K E D L B
not
the wife of Z. But, N is either brother or T  to the left of W and S ↓ ↓ ↓ ↓ ↓ ↓
sister of X. between W and S. 4 @ 1 6 $ 5
qqq
312 Practice Set-25

Air Force Airmen Group X & Y Exam


Practice Set-25

(c) To study correctly (c) in explaining


ENGLISH (d) None of these (d) to explaining
Directions (1-4): Read the passage 2. If one acquires knowledge and 9. Choose the appropriate synonym
given below and answer the questions develops mental self-culture, one of word OPULENT.
that follow. will not be the victim of (a) Greedy (b) Hungry
It is your duty to train and develop your (a) pugnacity (c) Heavy (d) Wealthy
mind and acquire knowledge, as much (b) superstition and demagogy Directions (10-11): Fill in the blanks
knowledge as you possibly, can obtain. (c) breed with appropriate word out of the four
Knowledge is like a deep well, fed by (d) None of these alternatives.
perennial springs and your Mind is the 3. Mind has been compared to 10. One who hates mankind is
little bucket that you drop into it: you (a) a deep sea
will get as much as you can assimilate. called_______
(b) a perennial spring (a) egoist (b) egotist
The Brain, which is the physical organ
(c) a bucket (c) introvert (d) misanthrope
of the mind, is one of the two precious
products of the aeons of Evolution: (d) a deep well 11. He was murdered ______.
the other is the imponderable ‘Social 4. The author implies that most men (a) in cold blood
instinct’. This wonderful Brain, whose and women (b) out of blood
every convolution represents millions of (a) do not possess capacity for free (c) about blood
years of Time, really distinguishes you judgement (d) from blood
from the animals. Many animals have (b) are manipulated by priests and 12. Which sentence is correct?
very powerful sense organs; the eagle, politicians (a) The news of the train accident
the ant and the dog have keener sense of (c) are arrogant in their behaviour are shocking.
sight than Man. But no animal has a more (d) refer to the sentence “Mind is the (b) The news of the trian accident
evolved Brain and higher Intelligence. If little bucket”. is shocking.
you do not develop and use this Brain to 5. Find out the correctly spelt (c) The new of the train accidents
the utmost of your power, you are more word: is shocking.
akin to the beasts than to Homo-sapiens. (a) aroplane (b) aroplain (d) None of the these
Knowledge amd mental self-culture (c) aeroplain (d) aeroplane 13. Choose the correct sentence–
will confer untold blessings upon you. 6. Fill in the blank with appropriate (a) Two hundred rupees is not a
You will not be the victim of superstition preposition : big sum
and demagogy in religion and politics. "There was dust ______the (b) Two hundred rupee are not a
You will know your duty and do it. To be big sum
carpet".
wise and independent in your religion and (c) Two hundred ruppe are not a
(a) underneath (b) towards
your politics, not to be doped and duped big sum
(c) inside (d) near (d) Two hundred ruppes is not a
by the selfish priests and the scheming
7. F i l l i n t h e b l a n k w i t h t h e big sum
politicians of Capitalism and so called
appropriate gerund : Direction (14-15): In the following
Socialism: is this not a noble aim worth
striving for? Most men and women "He avoided ____ her about his questions, some of the sentences have
today are not free and wise: they are like plans". error and some have none. Find out
kites flown by the priests and politicians (a) providing (b) giving which part of a sentence has an error.
whow hold the string. They are fleeced (c) selling (d) telling If there is No error then, the answer
and fooled on account of their ignorance. 8. Fill in the blank in the following is (d).
Half the ills of man are due to ignorance. s e n t e n c e w i t h a p p ro p r i a t e 14. (a) The passer-by told us / (b)
1. What according to the passage, is infinitive: where was the marriage hall / (c)
the duty of human beings? "I am always rewarded _____ the and even led us to it / (d) No Error
(a) To make ourselves cultured difficult concept". 15. (a) The increase in consumption
(b) To acquire knowledge and (a) to explain is directly / (b) proportional to the
develop the mind (b) explaining increase / (c) in income/ (d) No Error
Practice Set-25 313

Directions (16) : In the following 22. A p a r a l l e l p l a t e c a p a c i t o r r


r
question out of the four alternatives, consists of two circular plates (a) (b)
choose the one which best expresses 2 4
each of radius 2 cm, separated
the meaning of the given word. by a distance of 0.1 mm. (c) 2r (d) 4r
16. Successive If voltage across the plates 28. If we add impurity to a metal,
(a) rapid is varying at the rate of 5 those atoms also deflect
(b) victorious × 10 13 Vs –1, then the value of electrons. Therefore,
(c) beneficent displacement current is: (a) The electrical and thermal
(d) consecutive conductivity both increase
Direction (17) : In the following (a) 5.50 A
(c) 5.56 × 102 A (b) The electrical and thermal
question choose the word opposite in conductivity both decrease
meaning to the given word. (c) 5.56 × 103 A
(c) The electrical conductivity
17. Candid (d) 2.28 × 104 A increases but thermal
(a) outspoken (b) frank 23. The ratio of vapour density of conductivity decrease
(c) devious (d) disguised two gases at same temperature is
Directions (18-19): In the following (d) The electrical conductivity
9 : 1. The ratio of the rms speed decreases but thermal
questions groups of four words are of the molecules of the gases will
given. In each group one word is conductivity increase
be: 29. A closed end organ pipe gives
correctly spelt. Find the correctly spelt (a) 1 : 1 (b) 1 : 3
word. resonance at 17 cm and 50 cm
(c) 3 : 1 (d) 9 : 1
18. (a) mischevious lengths of air column at 30°C with
24. A magnet is moving towards a
(b) miscarriage a tuning fork of frequency 512 Hz.
coil along its axis and the e.m.f.
(c) misdemeanour The speed of sound at 0°C is:
(d) misnomar induced in the coil is E. If the
coil also starts moving towards (a) 338 m/s (b) 332 m/s
19. (a) harassment (c) 320 m/s (d) 350 m/s
(b) committment the magnet with the same speed,
(c) breevement the induced e.m.f. will be: 30. A monatomic gas at 17°C is
(d) temparament suddenly compressed to 1/8th
ε of its original volume. It
Directions (20) : In the following (a) (b) ε
questions the sentence have been given 2 temperature after compression
in Active/Passive voice. From the given (c) 2ε (d) 4ε will be:
alternatives, choose the one which best 25. P and S are two points in a (a) 17°C (b) 136°C
expresss the given sentence in Passive/ (c) 887°C (d) N o n e o f
uniform electric field E shown in
Active voice. these
20. This shirt cannot be worn by me the figure. If the potentials at P and
S are denoted by Vp and Vs, then: 31. A 1 m long rod has mass of 0.12
any longer.
(a) I cannot wear this shirt any kg. Its moment of inertia about an
longer axis passing through one end and
(b) Wearing of this shirt any perpendicular to rod is:
longer is not possible (a) 0.04 kg m2 (b) 0.06 kg m2
(c) This shirt is too worn out to be (c) 0.08 kg m2 (d) 0.10 kg m2
worm any longer 32. In the electric field of a point
(d) This worn out shirt cannot be (a) VP > VS (b) VP < VS
charge q shown, a charge is
worn any longer (c) VP = VS (d) VP > VS > 0 carried from A to B and from A
26. The greatest range of a rifle
to C. Compare the work done:
PHYSICS bullet at ground level is 6000
m. Its maximum range up an
21. Find the ratio of de-Broglie inclined plane of angle 30° is:
wavelengths λ1 and λ2 associated (a) 1000 m (b) 2000 m
respectively with a deutron (c) 6000 m (d) 4000 m
moving with a velocity 2v and 27. An electron is moving in a
an alpha particle moving with circle of radius r in a uniform
velocity v: magnetic field B. Suddenly the (a) Work done is greater along the
(a) 1 : 2 (b) 2 : 1 B path AC than along AB
field is reduced to · The radius
(c) 1 : 1 (d) N o n e o f 2 (b) Work done is the same in both
these of the circle now becomes: the cases
314 Practice Set-25

(c) Work done is greater along the given temperature. The ratio of
path AB than along AC two velocities (VH/V0) will be:
(d) Work done is zero in both the (a) 1 : 4 (b) 4 : 1
cases (c) 1 : 1 (d) 32 : 1
33. A coil of wire of radius R has 43. A square metal loop of side 10
200 turns and self-inductance of cm and resistance 1 Ω is moved
108 mH. The self-inductance of with a constant velocity partly
a similar coil of 500 turns will inside a uniform magnetic field
be: of 2 Wbm –2, directed into the
(a) 0.784 A, 0.392 A, 0.392 A paper, as shown in the figure.
(a) 375 mH (b) 0.468 A, 0.529 A, 0.240 A The loop is connected to a
(b) 527 mH (c) 0.396 A, 0.729 A, 0.126 A network of five resistors each of
(c) 675 mH (d) None of these value 3 Ω. If a steady current of
(d) None of these 37. A tube open at both the ends 1 mA flows in the loop, then the
34. A cricket ball is hit at 45° to the vibrates with fundamental speed of the loop is:
horizontal with a kinetic energy frequency of n. The tube is
K. The kinetic energy at the immersed vertically in water till
highest point is: it is half filled. The fundamental
K frequency of the tube will now be:
(a) Zero (b) (a) n/2 (b) 3n/4
2
(c) n (d) 2n
(c) 0.707 K (d) K 38. In an adibatic change, the
35. A wire ABCDEF is bent in the p re s s u re a n d d e n s i t y o f a (a) 0.5 cms–1 (b) 1 cms–1
form as shown in figure. The diatomic gas (γ = 1.4) changes (c) 2 cms–1 (d) 4 cms–1
wire carries a current I and is from (p, d) to (p′, d′). If d′/d = 32
44. If E.M.J. are G denote energy,
placed in a uniform magnetic then p′/p will be:
mass, angular momentum
field of induction B parallel to 1 (b) 32 and gravitational constant
(a)
positive Y-axis. If each side is of 128 EJ 2
length L, the force experienced respectively, then has
by the wire will be: (c) 128 (d) None of M 5 G2
these the dimensions of:
39. A particle under the influence of (a) length (b) angle
force F = Cx moves from x = 0 (c) mass (d) time
to x = x1. The work done in this 45. Imagine a light planet revolving
process is equal to: round a very massive star in a
1 2 circular orbit of radius R and
(a) Cx12 (b) Cx1
2 period of revolution T. If the
3
gravitational force of attraction
(c) Cx1 (d) Zero between the planet and the star
40. What is the current gain for is proportional to R–5/2 then:
a transistor used as common
emitter amplifier, if the current (a) T2 ∝ R3 (b) T2 ∝ R7/2
gain of the common base n-p-n (c) T2 ∝ R3/2 (d) T2 ∝ R3.76
transistor is 0.96?
(a) I B L a l o n g t h e p o s i t i v e (a) 16 (b) 24 MATHEMATICS
Z-direction (c) 20 (d) 32
41. The volume of a cube is 1.76 cm3, 46. If f(x) = cos (logx x) then
(b) I B L a l o n g t h e n e g a t i v e 1
then the volume of twenty five
Z-direction f(x) f(y) – [ f ( x / y) + f ( xy)] is
such cubes is: 2
(c) 2IBL along the positive (a) 4.4 × 101 cm3 equal to
Z-direction (b) 44 cm3
(a) zero (b) 1
(d) 2IBL along the negative (c) 44.0 cm3 f ( x) f ( y )
Z-direction 2
(d) 44.00 cm3
36. In circuit shown in figure, the 42. Velocity of sound is measured in (c) f (x + y) (d)  N o n e of
values of I1, I2 and I3 are: hydrogen and oxygen gases at a these
Practice Set-25 315

 x 1 (a) 4 (b) 5
3+i (b)  − 1 (1 − x 2 ) − sin −1 x + C
47. If z = then (z101 + i103)105 2 2 (c) 6 (d) 8
2 61. The equation of tangent to the
equals to 1
(c) (1 − x 2 ) + sin −1 x + C
curve y = b e – x/a at the point
2
(a) z (b) z2
(c) z 3
(d) None of (d) None of the above where it crosses y = axis is:
these 54. The distance between the lines (a) ax + by = 1 (b) ax – by = 1
48. The quadratic equation x2 + bx + 4x + 3y = 11 and 8x + 6y = 15 is: x y
x y
(a) 7/2 (b) 7/3 (c) − = 1 (d) + =1
4 = 0 will have real roots, it (c) 7/5 (d) 7/10 a b a b
(a) only b≥4 55. The value of
62. In shuffling a pack of cards 3
are accidently dropped then the
(b) only b ≥ 4 2 4 6 8
+ + + + .....∞ is chance that missing card should
(c) –4 < b < 4 1! 3! 5! 7!
be of different suits is:
(a) 1/e (b) e (a) 169/425 (b) 261/425
(d) b ≤ −4, b ≤ 4
(c) 2e (d) 3e (c) 104/425 (d) None of
49. What is the value of 76/2. 76/2, 71/2 56. Find the length of the intercepts these
.... upto ∞ made on the y-axis by the circle 1 1 x
63. If + = , then the value of
6 x2 + y2 + 6x – 8y – 16 = 0 6! 7! 8!
(a) log   (b) ∞
7 (a) 2 (b) 8 2 x is:
6 (a) 63 (b) 64
(c) (d) 7 (c) 4 2 (d) 2 3
7 (c) 66 (d) 65
50. T h e v a l u e o f t h e t e r m 57. Find the value of loga b × logb c ×
64. What is the value of
independent of x in the logc d × logd a π /2
9 (a) 0 (b) 1 ∫0 sin 2 x log (cot x) dx ?
 1
expansion of  x 2 −  is: (a) 0 (b) π log 2
 x ab
(c) –1 (d)
(a) 9 (b) 18 cd π log 2
(c) 48 (d) 84 58. What is the value of (c) –π log 2 (d) 2
 π  π  5π   7π 
3
1 − cot x
cos   + cos   + cos 
 9  3  9 
+ cos   ?
 9 
51. lim is equal to 65. With usual notations, if in a
x→ π / 4 2 − cot x − cot 3 x
(a) 1 (b) –1 b+ c c+ a a+ b
∆ABC = = then
3 11 1 1 11 12 13
(a) (b) (c) − (d)
2 4 2 2 cos A : cos B : cos C is equal to:
1 2 59. The solution of the equation (a) 7 : 19 : 25
(c) (d) (b) 19 : 7 : 25
2 5 dy
x2 = x 2 + zy + y2 is (c) 12 : 14 : 20
52. W h a t i s t h e d e r i v a t i v e o f dx (d) 19 : 25 : 20
 1  −1  y  66. If nCr – 1 = 36, nCr = 84 and nCr + 1 =
sin −1   with respect to (a) tan   = log x + C
 1 + t2  z 126, then value of r will be:
(a) 1 (b) 2
 1  −1  x 
cos −1   (b) tan   = log x + C (c) 3 (d) None of
 y
 1 + t2  ? these
(a) 1 (b) –1 −1  x 
(c) tan   = log y + C  1 
(c) 2 (d) –2  y 67. If sin  sin −1 + cos −1 x = 1, then
 5 
−1  y 
 1 − x (d) tan   = log y + C what is x equal to:
53. ∫x  1 + x  dx is equal to:  x
(a) 0 (b) 1
x  1 –1 60. What is the value of n for
(a)
2
 − 1 (1 − x ) + sin x + C which the numbers 1, 2, 3, 4 1
2 2 (c) (d)
... n have variance? 5 5
316 Practice Set-25

68. The equation of an ellipse whose 73. Which part of the Constitution 5 1
of India has been described as (c) 35 (d) 45
focus is (–1, 1), directrix is x – y 6 3
the soul of the Constitution ?
1 81. If curved surface area of a
+ 3 = 0 and eccentricity is , (a) Fundamental Rights cylinder is 1320 sq cm and height
2 (b) Directive Princples of State is 10 cm, find its radius.
will be: Policy (a) 21 cm (b) 42 cm
(a) 7x2 + 2xy + 7y2 + 10x – 10y + 7 (c) Preamble (c) 63 cm (d) 10.5 cm
(d) Panchayats 82. When a number is increased by
=0
74. Which water body separates 18, it becomes 106% of itself.
(b) 7x2 – 2xy + 7y2 – 10x + 10y + 7 Andaman and Nicobar Islands? What is the number ?
=0 (a) 300 (b) 216
(a) Andaman Sea
(c) 270 (d) 162
(c) 7x2 – 2xy + 7y2 – 10x – 10y – 7 = (b) Bay of Bengal 83. A trader had 16 quintals of
0 (c) Ten Degree Channel wheat. He sold a part of it at
(d) 7x2 – 2xy + 7y2 + 10x + 10y – 7 (d) Eleventh Degree Channel 17% profit and the rest at 27%
=0 75. Which of the following vitamins profit, so that he made a total
is necessary for clotting of blood? profit of 21%. How much wheat
69. What is the value of tan 105°?
(a) K (b) C did he sell at 27% profit?
3 +1 3 +1 (a) 960 kg (b) 640 kg
(c) A (d) B
(a) (b) (c) 320 kg (d) 1280 kg
3 −1 1− 3 76. The monitor of a computer is : 84. A can do a work in 15 days and
3 −1 3+2 (a) an input device B in 20 days. If they work on it
(c) (d)
3 +1 3 −1 (b) an output device together for 6 days, then what
(c) a storage device fraction of work is left ?
70. Find the value or values of x
(d) a processing device (a) 1/4 (b) 2/9
such that (c) 1/5 (d) 3/10
77. Which Indian State will
 2  − 3   − 1 85. A sum fetched a total simple
x2   + x  =   be benefitted by the India-
interest of ` 7056 at the rate of
1  1 2 Bangladesh Friendship Bridge
8 percent per year in 7 years.
(a) 0 1 (Feni Bridge/ Maitree Setu)
What is the sum ?
constructed over Feni River?
(b) 1 (a) ` 12600 (b) ` 15120
(a) Meghalaya (b) Manipur (c) ` 10080 (d) ` 7560
(c) (c) Tripura (d) Mizoram 86. Examine the following
78. If the number 583_437 is relationships among members of
(d) Both (b) and (c) completely divisible by 9, then a family of six persons - A, B, C,
the smallest whole number in the D, E and F.
REASONING AND GENERAL place of the blank digit will be : 1. The number of males equals that
AWARENESS (RAGA) (a) 4 (b) 5 of females.
71. The present monetary system in 2. A and E are sons of F.
(c) 3 (d) 6
India is managed by : 3. D is the mother of two, one boy
3
79. tan A is equal to
(a) Nationalized Banks and one girl.
(a) 3tanA – tan3A(1 – 3tan2A) 4. B is the son of A.
(b) The State Bank of India
(c) The Central Finance Ministry (b) 3tanA + tan3A(1 – 3tan2A) 5. There is one married couple in the
(d) The Reserve Bank of India (c) 3tanA – tan3A(1 + 3tan2A) family at present.
72. The Khilafat Movement was (d) 3tanA + tan3A(1 + 3tan2A) Which one of the following
organized to protest against : inferences can be drawn from
1 1 1 the above ?
(a) religious interference by the 80. 20 + 30 – 15 = ?
British 2 3 6 (a) A, B and C are all females.
(b) Russian Revolution (b) A is the husband of D.
1 2
(c) dismemberment of Turkey (a) 34 (b) 35 (c) D is the grand daughter of F.
6 3
(d) suppression of Pathans (d) E and F are children of D.
Practice Set-25 317

87. In a certain code language, Direction (92): Study the following


B
“KINGFISHER” is written information answer the questions
as “+$@*!$><^?”. How is given below it. In a certain code, the I J
“NEIGH” written in that code symbol for 0 is © and for 1 is ⊕. There C
E G H
language? are no other symbols for all other
(a) ^@$*< (b) @^$<* numbers greater than 1. The number F A
(c) @$^*< (d) @^$*< greater than 1 are to be written only D
by using the two symbols given above.
88. A boy first goes in South direction,
The value of symbol for 1 doubles (a) HEG (b) AHG
then he turns towards left and
itself everytime it shifts one place to (c) FEGH (d) FE
travels for some distance. After
the left. Study the following example: Directions (96-100): Study the
that he turns right and moves
certain distance. At last he turns ‘0’ is written as ©. following information carefully and
left and travels again for some ‘1’ is written as ⊕. answer the questions given below:
distance. Now, in which direction Seven people–A, B, C, D, E, F and
‘2’ is written as ⊕ ©.
is he moving? G–are having different hobbies, viz.,
(a) South (b) West ‘3’ is written as ⊕ ⊕. Travelling, Reading, Dancing, Painting,
(c) East (d) North ‘4’ is written as ⊕ © © and so on. Sculpting, Singing and Pottery making,
92. Which of the following number but not necessarily in the same order.
89. Meena correctly remembers
will be represented by ⊕ ⊕ ⊕ © Each of them belong to different
that her father's birthday is
after 18th May but before 22nd ⊕? State, viz., Punjab, Odisha, Kerala,
May. Her brother correctly (a) 25 (b) 21 Rajasthan, Maharashtra, Gujarat and
remembers that their father's (c) 35 (d) 29 Karnataka, but not necessarily in the
birthday is before 24th May but same order. A belongs to Maharashtra.
Direction (93) : Select the odd word/
after 20th May. On which date in D likes pottery making. The person
letters/number/number pair from the
May was definitely their father's who likes sculpting is from the State
given alternatives.
birthday? of Odisha. The person who likes
93. (a) Herd (b) Stock
(a) 20th (b) 19th dancing is from the State of Gujarat.
(c) 18th (d) 21st (c) Gang (d) Cub F does not belong to Gujarat, Odisha,
90. Arrange the given words in the 94. Arrange the given words in the Punjab or Rajasthan. F does not like
sequence in which they occur in sequence in which they occur in singing, reading or painting. B does
the dictionary. the dictionary. not belong to Kerala, Odisha, Punjab
i. Stable ii. Stranger (i) Drama or Rajasthan. B does not like painting,
(ii) Dramatical travelling reading or singing. C does
iii. Stability iv. Struggle
not like sculpting and he is not from
(a) iii, ii, i, iv (b) iii, i, ii, iv (iii) Dramaticism
Rajasthan or Punjab. Neither D nor
(c) iii, iv, i, ii (d) iv, i, ii, iii (iv) Dramality G belongs to Punjab. A does not like
Direction (91): Study the following (v) Dramatise reading. The person from Kerala likes
information carefully and answer the (a) i, iv, ii, iii, v singing.
questions given below: (b) i, iv, ii, v, iii 96. Who among the following likes
P, Q, R, S, T, V and W are sitting singing?
(c) i, iv, v, iii, ii
around a circle facing at the centre. V (a) A (b) C
is second to the left of P and second to (d) iv, i, v, iii, ii (c) E (d) G
the right of W. T is third to the right of 95. In the following figure, rectangle 97. Which one of the following
Q and is not an immediate neighbour represents Plumbers, circle combinations is true according
of V, S is third to the right of R. represents Bakers, triangle to the given information?
represents Jugglers and square (a) A–Travelling–Maharashtra
91. Who is third to the left of V?
represents Joggers. Which set of (b) C–Dancing–Gujarat
(a) T (b) S letters represents Plumbers who (c) E–Reading–Karnataka
(c) W (d) R are either Bakers or Jugglers? (d) D–Pottery Making–Rajasthan
318 Practice Set-25

98. Who among the following belongs (a) Sculpting–Odisha (a) Travelling
to the State of Karnataka? (b) Pottery making–Karnataka (b) Sculpting
(a) B (b) D (c) Dancing–Gujarat (c) Painting
(c) F (d) E (d) Singing–Kerala (d) Reading
99. Which of the following 100. The person who belongs to
combinations is true about G? Punjab, likes:

Answers with Explanation


the sentence is as follows: Subject + ask
ENGLISH + object + questions - word + assertive dE
22. (c) Id= ε 0 A
1. (b) The first sentence of the passage clause. dt

{ }
gives the answer. For example, A dV V
2. (b) The second sentence of the I told my firend, " Why are you sad?" = ε 0 E=
d dt d
second paragraph gives the answer. (Direct)
3. (c) Refer to the sentence “Mind is I asked my friend why he was sad. 8.86 × 10−12 × 3.14 × (2 × 10−2 ) 2
⇒ Id = × 5 × 1013
the little bucket”. (Indirect). 0.1 × 10−3
4. (b) Answer can be inferred from the
Hence, " Where was the marriage hall" = Id= 5.56 × 103 A
third last sentence of the passage.
should be replaced with where the
5. (d) Aeroplane is correctly spelt (Vrms )1 M2
word. mariage hall was". 23. (b) =
(Vrms ) 2 M1
6. (d) Underneath is correct option. 15. (d)
Preposition 'underneath' is used directly 16. (d) The word successive means
2d 2 1 1
under another object or covered by it. coming one after the other without a = = =
Hence option (a) is correct. Correct break. For example, 2d1 9 3
sentence - "There was dust underneath This was their fifth successive win.
the carpet". 24. (c) The relative velocity of
Out of the answer choices, the
7. (d) 'Telling' is appropriate gerund approach equals 2v and hence
word consecutive means following
to fill in the blank. Hence option (d) is the induced e.m.f. becomes 2ε.
correct. Correct sentence- "He avoided continuously.
Hence, successive and consecutive are 25. (a) In the direction of electric field,
telling her about his plans".
8. (a) 'To explain' is appropriate synonymous. potential decreases.
infinitive to fill in the blank. In traditional 17. (c) The word Candid (Adjective) Vp > Vs
description of English, the infinitive is the means not hiding one's thoughts; frank
basic dictionary form of a verb used non- Rmax u2
and honest. 26. (d) =
finitely, with or without 'To', So correct The word Devious (Adjective) means not
Ground g
sentence is - " I am always rewarded to
honest, direct or straight forward. = 6000 m
explain that difficult concept". Hence
option (a) is correct. Hence, candid and devious are
antonymous. Rmax u2
9. (d) Incline
=
18. (c) misdemeanour g (1 + sin β)
10. (d) A misanthrope is a person who
dislikes humankind and avoids human 19. (a) harassment
= 6000
society. Hence option (d) is correct. 20. (a) I cannot wear this shirt any
11. (a) Murder is always committed in 1 + sin 30
longer.
cold blood. Hence option (a) is correct. ⇒
Rmax = 4000 m
12. (b) News is a singular noun so with PHYSICS Incline
singular subject singular verb is used.
Hence option (b) will be correct.
21. (c) For deuteron, p1 = 2m × 2v 27. (c) Since r = mv
= 4mv qB
13. (a) Two hundred rupees is a singular For α-particle, p2 = 4m × v
unit hence, singular verb is useful. Hence = 4mv 1
option (a) will be correct. ⇒
r∝
∴ p1 = p2 B
14. (b) I n I n d i r e c t s p e e c h o f so that λ1 = λ2 B
S o , a s B b e c o m e s ,r
interrogative sentence, the structure of ∴ Ratio 1 : 1. becomes 2r. 2
Practice Set-25 319

28. (b) The electrons increase their = 108 × 6.25 39. (b) From the figure,
number of collisions increasing = 675 mH
the thermal and electrical A
34. (b) At the highest point vy = 0 and
resistance. Hence, both the vx = v cos 45

Force (F) →
conductivities decrease.
29. (c) vt = n × 2(l2 – l1) 1 2
Since, K = mv is initial
= 512 × 2 2 Cx1
energy of the ball. Also 1
× (50 – 17) cm/s Cx 2
2 1
= 337.92 m/s 1
K′ = m(vx2 + v 2y )
v0 = vt – 0.61t 2 B
= 337.92 – 0.61 × 30 O Distance (x) →
= 319.62 m/s 1 1
=  mv 2 
= 320 m/s 2 2 Work = Area of triangle OAB
30. (c) A sudden change implies K 1
adiabatic process for which ⇒ K′ =
= x1Cx1
2 2
T2V2γ −1 = T2V2γ −1
35. (a) Force on BC and DE are equal 1 2
and opposite and hence cancel = Cx1
(273 +
γ −1
17) V1 = T2 (V / 8) γ −1 2
out. So, the force on CD in
BIL. x
⇒ T2 = 290(8)5/3 – 1

36. (a) We know, I1 = I2 + I3
Alternately work = ∫0 Fdx
5

(Since γ = for Applying loop rule in BCADB, x
3 – (0.1)I2 + 2 – (0.1)I2 + 2 – 5I1 = 0 = ∫0 Cxdx
monoatomic gas) ⇒ 5I1 + 0.2I2 = 4
= 290 × (23)2/3 Applying loop rule in BDACB, 1 2
= Cx1

= 1160 K 2
– (0.1)I3 + 2 – (0.1)I3 + 2 – 5I1 = 0
⇒ T2 = 887°C
31. (a) M.I. about an axis passing one ⇒5I1 + 0.2I3 = 4 α 0.96
∴ I2 = I3 and I1 = 2I2 40. (b) β = = = 24
end and perpendicular to rod 1 − α 1 − 0.96
I1= 0.784 A
Ml 2 41. (c) Volume of 25 cubes
= I2= 0.392 A
3 I3= 0.392 A = 1.76 × 25
Here, M = 0.12 kg
v = 44.00
and l=1 37. (c) For open tube n =
2l Since the product cannot be more
0·12 × (1) 2 accurate than the least accurate

M.I. = λ
3 l measurement.
and for closed tube =
= 0.04 kg-m2 2 4 ∴ Volume = 44.0 cm3
32. (b) WA → B= Q(VB – VA) v v
WA → C= Q(VC – VA) n′ = = =n yRT
4×l/2 2l 42. (a) V=
Since, VC – VB as both B and C M
∴ n′ = n
lie on an equipotential surface. 38. (c) For an adiabatic change PVγ = Molecular weight of hydrogen
µ N 2 πr 2
constant. MH = 2
33. (c) L1= 0 1 , L2 = µ0 N 2 π r
2 2 γ
Molecular weight of oxygen
2 ⇒ P  mass  = constant MO = 32
L2  N2   density 
= 
L1  N1  P
VH
=
HO
⇒ γ = constant V0 MH
2 d
N 
⇒ L2 = L1  2 
P
γ 32
 N1  P′ P′  d ′  =
⇒ = ⇒ =   2
d (d ′) γ P d
2
500 
∴ L2= 108 × 
= (32)1.4 = (32)7/5 = 4
 200  1
= (2)7 = 128
320 Practice Set-25

43. (c) It is a balanced Wheatstone 1 ∴ T(6 + 1) = 9C6 x(18 – 18) . (–1)6


Bridge. – [cos (log x – log y) + cos (log x
2 + log y)] 9.8.7
Hence, Rnet = 3 + 1 = 4 Ω Tr = 9C6 . 1 = = 84
Since, I = 1 mA = cos (log x) cos (log y) 3.2.1

BIv 1 51. (a) Factorse both numerator and



10–3 = – [2 cos (log x) cos (log y)]
2 denomimtor
R
=0 (1 − cot x)i1 + cot x + cot α
lim
2(0.1)v −1 + i 3 x + π / 4 (1 – cot x ) (2 + cot x + cot 2 x )

10–3 = 47. (c) iz = =ω
4 2

v = 2 cms–1 1+1+1 3
= =
 1 2 +1+1 4
44. (b) ET
2
⇒ z =   ω = − iω
 i
M 5G 2 −1
 1 
Also, ω3n = 1, i4n = 1, i3 = –1 ...(i) 52. (a) Let v = sin  
[ ML2T −2 ][ ML2T −1 ]2  1 + t2 
= ∴ z101 + i103= (–i)101 ω101 + i103
[ M ]5 [ M −1 L3T −2 ]2 = –iω2 – i [By Eq. (i)]  1 
−1
= –i(1 + ω2) = –i(–ω) = iω and x = cos  
ML2T −2 × M 2 L4T −2  1 + t2 
= ∴ Now, (z101 + i103)106= i105 ω105 = i . ω2
M 5 M −2 L0T −2 Again, let t = tan θ
= i(iz)3 = i4z3 = z3
48. (d) Given that, the equation x2 + ∴ u=
= M0L0T0
bx + 4 = 0 have real roots, if discriminate  tan θ 
sin −1  − sin −1 (sin θ) = tan −1 t
= Dimensions of angle
(d) = B2 – 4AC ≥ 0  sec θ 
45. (b) For satellite motion, ⇒ b2 – 4(1) (4) ≥ 0 ⇒ b2 – 16 ≥ 0
du 1
Gravitational attraction ⇒ (b – 4) (b + 4) ≥ 0 ⇒ =
dt 1+ t2
∴ b ≤ –4, b ≥ 4
= Centripetal force and y =
49. (d) 76/7, 76/7 , 76/73 ....... ∞
2

GMm mv 2  1   1 
= 61 1 1  cos −1  = cos −1 
R 5/ 2
R = 7  + 2 + 3 + ......∞  2  
 7 7 7   1+ t   1 + tan θ 
2

GM  a  – cos–1 (cos θ) = tan–1 t



v2 =  1/ 7   1/ 7 

R3/ 2 =7
6  1−1/ 7 
=7
6
 6/ 7   S∞ = 1 − r  du 1
⇒ =
dt 1 + t 2
 2πR 
2
4π 2 R 2 = 71 = 7
T = 
2
 = 9
du du dt
 v  GM/R3/ 2  2 1 ∴ = = =1
50. (d) Given  x −  dt dv
 x dt
4π 2 7/ 2 General term x(1 – x)
= R ∝ R 7/ 2
GM Tr + 1 = Cr (1) x , in (1 + x)
n n–r 2 n
53. (b) I = ∫ (1 – x 2 )
dx

Similarly
MATHEMATICS x – x2
Tr + 1 = 9Cr (x2)9 – r . = ∫– dx + ∫ dx
46. (a) We have f(x) = cos (loge x), r 9
(1 – x 2 ) (1 – x 2 )
therefore  1  2 1
 −  , in  x −  1 2x 1 – x 2 –1
x
2 ∫ (1 – x 2 ) ∫ (1 – x 2 ) dx
1 x = dx +
f(x)f(y) – [ f ( x / y ) + f ( xy )]
2 = 9Cr x18 – 2i (–1)r x–r
= cos (log x) cos (log y) = 9Cr x(18 – 3r) (–1)r ...(i) 1 1
= – .2 (1 – x 2 ) ∫ (1 – x 2 ) dx – ∫ dx
For independent term, 2 (1 – x 2 )
1
– [cos log (x/y) + cos (log (xy))] Put 18 – 3r = 0
2 x  2 1 –1
⇒ 3r = 18 =  – 1 (1 – x ) – sin x + C
= cos (log x) cos (log y) 2  2
⇒ r =6
Practice Set-25 321

54. (d) Given lines are 4x + 3y = 11 1 Since, the curve crosses y-axis (i.e.,
15 = cos 20º + − 2sin 30º cos 20º x = 0)
and 4x + 3y = 2
2 ⇒ y = be–0 ⇒ y = b
15 1 1
11 – = cos 20º + – cos 20º = On differentating Eq. (i), we get
Distance between them = 2 2 2
16 + 9 59. (a) Putting, y = vx we get dy
= −b e − x / a
7 7 dx a
= = dx
2 × 5 10 v+x = 1 + v + v2
dy  dy 
At point (0, b)  dx 
2 4 6 8 (0 − b )
55. (b) + + + + ....∞ dx dx
1! 3! 5! 7! ∴ =
1+ v 2 x
= −b e −0/ a = −b
(1 + 1) (1 + 3) (5 + 1) (7 + 1) ⇒ tan–1 v = log x + C a a
= + + + + .....∞
1! 3! 5! 7! So, the required equation of tangent
−1  y 
1 1 1 1  ⇒ tan   = log x + C is
x
 + + + + .....∞ −b
1! 3! 5! 7! y–b = ( x − 0)
= 60. (b) Mean of the numbers
1 1 1 a
 
+ 1 + + + + ....∞ n(n + 1)
 2! 4! 6!  y x x y
2 n +1 ⇒ −1 = − ⇒ + = 1
= = b a a b
e − e −1 e + e −1 n 2
= + =e 62. (a) Total ways = 52C3 = 22100
2 2 ∴ Variance
56. (b) Comparing the given equation  n + 1 
2
n + 1
2
 n + 1
2 There are 4 suits in a pack of cards, so
1 –  +2 –  + ..... +  n – 
with the general form of = 2   2  2  three suits can be selected in 4C2, ways
n
the equation of the circle. and one card each from different can be
x2 + y2 – 2gx + 2fy + c = 0 we get  n + 1
(12 + 22 + 32 + ..... + n 2 ) + − 2  selected in 13C1 × 13C1 × 13C1 ways. So.
 2 
g = 3, f = – 4 and c = –16 [1 + 2 + 3 + ......] favourable ways
⇒2=
n
We have intercept on the X-axis = 4C3 × 13C1 × 13C1 × 13C1
2
= 2 f – c = 2 9 + 16 = 2 25 = 10 ⇒ 2n = 1 n(n + 1)(2n + 1) + n(n + 1) = 8788
2

Intercept on the Y-axis 6 4 ∴ Required probability =


8788
22100
 n + 1  n(n + 1) 
= 2 t 2 – c = 2 16 + 16 –2   
 2   2  =
169
425
= 2 32 = 8 2 ⇒ 2n
1 1 x
57. (b) Given expression can be 63. (b) Given, + =
 2n + 1 n + 1 n + 1  6! 7! 8!
= n(n + 1)  + –
written as  6 4 2 
log m b log m c log m d log m a 1 1 x
. . . =1 ⇒ + =
log m a log m b log m c log m d  4n + 2 − 3n − 3  6! 7 × 6! 8 × 7 × 6!
⇒ 2 = (n + 1)  
 12
1  1 x
58. (d) 1+ =
⇒ 24 = (n + 1) (n – 1) 6!  7  8 × 7 × 6!
 π  π  5π   7π 
cos   + cos   + cos   + cos   ⇒ n2 – 1 = 24
 9  3  9  9 1 1 x
⇒ n2 = 25 ⇒ + =
= cos (20º) + cos (60º) + cos (100º) 1 7 8× 7
⇒ n = +5
+ cos (140º) ⇒n=5 (∴ n cannot be negative) 7 +1 x
⇒ =
1 61. (d) Given equation of curve is 7 8× 7
= cos 20º + + 2 cos120º cos 20º
2 y = be–x/a...(i) ⇒ x = 8 × 8 = 64
322 Practice Set-25

π /2 n
Cr = x2 + y2 + 9 – 2xy + 6x – 6y
64. (a) Let I = ∫0 sin 2 x log (cot x )dx ⇒
n ⇒ 7x2 + 2xy + 7y2 + 10x –10y + 7 = 0
...(i) C r +1
a a 69. (b) tan 105º = tan (60º + 45º)
 ∫ f ( x)dx =
∫ f (a − x)dx = n!
×
(r + 1)!× (n − r − 1)!
= tan 60º + tan 45º
0 0
r ! × (n − r )! n!
π /2 π  π  1 − tan 60º tan 40º
= ∫0 sin 2  − x  log cot  − x  dx
2  2 
= 84  tan A + tan B 
π /2 126  tan (A + B) = 1 − tan A tan B 
I= ∫ 0
sin 2 x log(tan x) dx  ...(ii)
r +1 2
⇒ = 3 +1 3 +1
On adding Eqs. (i) and (ii) we get n–r 3 = =
1 − 3.1 1− 3
2I = ∫0
π /2
sin 2 x [log cot x + log (tan x )] dx ⇒ 5r + 3 = 2n...(ii)
70. (b) From the left hand side, we
On solving (1) and (2), we have r = 3
= ∫0
π /2
sin 2 x [log cot x + log (tan x )] dx have
 1 
67. (d) Given, sin  sin −1 − cos −1 x 2  −3  2 x 2   −3 x 
π /2  5  x2   + x   =  2 + 
= ∫0 sin 2 x. log1. dx = 0 1  1 x   x 
=1
∴ I=0  2 x 2 − 3x 
 1 =  2 
65. (a) If each ratio be k, then we have ⇒ sin   + cos −1 x = sin–1 (1)
−1
 5  x +x 
b + c = 11k . c + a = 12k
 1 −1  π −1
 2 x 2 − 3x   −1
⇒ sin   = sin  sin  − cos x
−1
a + b = 13k ∴  2 = 2
 5  2  x +x   
So that, 2(a + b + c) = 36k ⇒ a + b
+ c = 18k −1  1  π ⇒ 2x2 – 3x = –1 and x2 + x = 2
⇒ sin   = − cos –1 x On solving the first equation 2x2 – 3x
∴ a = 7k, b = 6k.c = 5k  5 2
+1=0
Nowcos  π
−1 −1
  sin x + cos x =  we get x = 1, 1/2
2 The second equation x2 + x = 2 is
b 2 + c 2 − a 2 36 + 25 − 49
A= = −1  1 
satisfied only when x = 1.
2bc 2.6.5 ⇒ sin–1 (x) = sin  
 5 Hence, the solution is x = 1.
12 1
= =
60 5 ⇒x= 1 REASONING AND GENERAL
5 AWARENESS (RAGA)
19 5
Similarly. cos B = and cos C = 68. (a) Let a point P(x, y) be on the
35 7 71. (d) The Reserve Bank of India
ellipse, announces the Monetary Policy for
1 10 5 ∴ Distance of P from the focus (–1, 1) every financial year and presents
∴ cos A : cos B : cos C = : :
5 35 7 periodic review of the same from time
= 7 : 19 : 25 2
= ( x + 1) + ( y − 1)
2
to time.
n
C r −1 and the length the perpendicular PM 72. (c) In support of the Khilafat
66. (c) n on the directrix movement Gandiji inaugurated the
Cr
x− y+3 Non-Cooperation campaign on August
=
n! r ! × (n − r )! 2 1, 1920.
= × 73. (c) Preamble to the Constitution
(r –1)! × (n − r + 1)! n!
( x + 1) 2 + ( y − 1) 2 is soul of the Constitution.
∴ 74. (c) Andaman and Nicobar
36
= Islands are separated by Ten Degree
84 ( x − y + 3) 1 ( x − y + 3)
=e = Channel.
2 2 2
r 3 75. (a) Vitamin K is necessary for
⇒ =
n – r +1 7 ⇒ (x + 1)2 + (y – 1)2 = 1 ( x – y + 3) 2 clotting of blood. It was discovered
8 by Dam and Doisy of United States in
⇒ 10r = 3n + 3 ...(i)
⇒ 8x + 16x + 8 + 8y – 16y + 8
2 2 1935.
Practice Set-25 323

76. (b) A monitor is first and 1320 × 7 85. (a) Let the sum be X
foremost an output device because it r= \ According to question,
44 × 10
displays information already held in the X×8×7
computer. r = 21 cm 7056 =
82. (a) Let a number be x 100
77. (c) On June 16, 2019, Riva
Ganguly Das, High Commissioner 106 7056 × 100
x + 18 = x
\ X=
of India to Bangladesh visited the 100 8×7
construction of the India-Bangladesh
106 = ` 12600
Friendship Bridge 1 (Feni Bridge/ x − x = 18
Maitree Setu) and land port in Ramgarh 100 86. (b) Clearly, there are three
Upazila of Khagrachhari, Bangladesh. 106 x − 100 x males and three females. Now, A, E and
= 18 B are sons and hence males. So, C, D
The construction of the bridge over 100
Feni River had begun in 2017 and is 6x = 18 × 100 and F are females.
expected to be completed by April 2020. x = 300. D has one male and one female
It will connect Tripura with Chittagong 83. (b) Let he sells x quintals at child. Clearly, A and D form a couple
port of Bangladesh.Its foundation for ` 17%. having two children, namely B and C.
construction was laid down jointly by According to question, Hence, A is the husband of D.
Indian Prime Minister Narendra Modi 87. (d) As,
x × 117 (16 − x ) × 127
and Bangladesh Prime Minister Sheikh +
100 100 K I N G F I S H E R
Hasina in 2015. ↓ ↓ ↓ ↓ ↓ ↓ ↓ ↓ ↓ ↓
78. (d) 583 – 437 is completely 16 × 121 + $ @ * ! $ > < ^ ?
=
divisible by 9. 100 Similarly,
   583 m 437
117 x − 127 x 16 × 121 16 × 127 N E I G H
Sum of digits of this number must = −
100 100 100 ↓ ↓ ↓ ↓ ↓
be divided by 9 @ ^ $ * <

5 + 8 + 3 + m + 4 + 3 + 7 = 36 − 10 x 16 × 6
= − Finally, NEIGH will be written as
                m + 30 = 36 100 100 @ ^ $ * <.
             m = 36 – 30 = 6 960
x= = 9·6 quintals 88. (c) According to the question,
Hence, the digit is 6 to be divisible 10
by 9. the direction diagram will be as follows
= 9·60 × 100
3tan A – tan 3 A = 960 kg
79. (a) tan 3A =
1–3tan 2 A Quantity sold at 27%
3tan A – tan3 A = tan3 A (1 – 3 tan2 A) = 1600 – 960 = 640 kg
or tan 3A (1 – 3 tan2 A) = 3 tan
A – tan3 A 1
84. (d) A’s one day work =
tan3 A = 3tan A – tan 3A (1 – 3tan2 A) 15 Clearly, boy is moving towards East.
1 1 1 1 89. (d) Days by Meena, 19th 20th
80. (b) ? = 20 + 30 –15 B’s one day work = or 21st May.
2 3 6 20
Days by her brother, 21st, 22nd or
1 1 1 1 1 23rd May.
= 20 + + 30 + –15 − (A + B)’s one day work = +
2 3 6 15 20 Clearly, 21st May is common in both
1 1 1 the groups and hence it is the required
= (20 + 30 –15) + + – 4+3 7 day.
2 3 6 = =
60 60 90. (b) According to dictionary,
4 2 (A + B)’s 6 days work
=  35 +  = 35 the arrangement of the words is given
 6 3 7 7 below–
= 6× =
81. (a) C.S.A. of a cylinder = 2prh 60 10 (iii) Stability (i) Stable
(ii) Stranger (iv) Struggle
22 7 3
1320 = 2 × × r ×10 Left work = 1 − = . Finally, (iii), (i), (ii), (iv) is the
7 10 10 correct sequence.
324 Practice Set-25

[Hints 91]: According to the given ⊕ ⊕ ⊕ © ⊕  16 + 8 + 4 + 0 + 1 [Hints 96–100)]:


information, we get = ‘29’. Person Hobby State
V  Second to the left of P and 93. (d) The odd word is cub. A Painting Maharashtra
Second to the right of W.
T  Third to the right of Q. 94. (a) According to dictionary, the B Dancing Gujarat
arrangement of given words is, C Singing Kerala
S  Third to the right of R
So, the circular arrangement is as (i) Drama (iv) Dramality D Pottery making Rajasthan
follows, (ii) Dramatical (iii) Dramaticism E Reading Punjab
(v) Dramatise. F Travelling Karnataka
95. (c) Plumbers = {A, E, F, G, H} G Sculpting Odisha
Bakers = {C, D, E, F} 96. (b) C likes singing.
Jugglers = {G, H, I, J} 97. (d) The combination D–Pottery
Joggers = {B, C, E, G, I} Making–Rajasthan is correct.
91. (a) By analysing the above the required set of letters 98. (c) F belongs to Karnataka.
circular arrangement, it is clear that 99. (a) G–Sculpting–Odisha is correct.
= {F, E, G, H} 100. (d) E belongs to Punjab and he likes
T  third to the left of V.
Hence, the set of letters FEGH are reading.
92. (d) According to the given
plumbers who are either bakers or Jugglers. qqq
pattern

You might also like